Você está na página 1de 838

MA 12 - Matemtica Discreta Edio 2012

Atualizado at Junho de 2014


1
Nmeros Naturais

Sumrio
1.1 Introduo . . . . . . . . . . . . . . . . . . . . . . . 2

1.2 O Conjunto dos Nmeros Naturais . . . . . . . . . . 3

1.3 O Axioma da Induo . . . . . . . . . . . . . . . . . 4

1.4 As Duas Operaes: Adio e Multiplicao . . . . 5

1.5 A Ordenao nos Nmeros Naturais . . . . . . . . . 6

1.6 Exerccios Recomendados . . . . . . . . . . . . . . . 8

1.7 Textos Complementares . . . . . . . . . . . . . . . . 10

1
1.1 Introduo

Deus criou os nmeros naturais. O resto obra dos homens.

Leopold Kronecker

Enquanto os conjuntos constituem um meio auxiliar, os nmeros so um dos dois

objetos principais de que se ocupa a Matemtica. O outro objeto o espao,

juntamente com as guras geomtricas nele contidas. Os nmeros so objetos

abstratos que foram desenvolvidos pelo homem para servir como modelos que

permitem contar e medir e, portanto avaliar as diferentes quantidades de uma

grandeza.

Os compndios tradicionais dizem o seguinte:

Nmero o resultado da comparao entre uma grandeza e a unidade. Se

a grandeza discreta, essa comparao chama-se uma contagem e o resultado

um nmero inteiro; se a grandeza contnua, a comparao chama-se uma

medio e o resultado um nmero real.

Nos padres atuais de rigor matemtico, o trecho acima no pode ser consi-

derado como uma denio matemtica, pois faz uso de idias (como grandeza,

unidade, discreta, contnua) e processos (como comparao) de signicado no

estabelecido. Entretanto, todas as palavras que nela aparecem possuem um

sentido bastante claro na linguagem do dia-a-dia. Por isso, embora no sirva

para demonstrar teoremas a partir dela, a denio tradicional tem o grande

mrito de nos revelar para que servem e por qual motivo foram inventados os

nmeros. Isto muito mais do que se pode dizer sobre a denio que encon-

tramos no nosso dicionrio mais conhecido e festejado, conforme reproduzimos

a seguir.

Nmero. [Do lat. numeru.] S.m. 1. Mat. O conjunto de todos os conjuntos

equivalentes a um conjunto dado.

Discutiremos este ponto logo mais, quando tratarmos de nmeros cardinais.

+ Para Saber Mais - Comentrios sobre Denies e axiomas - Cli-

que para ler


Nmeros Naturais Unidade 1

+ Na Sala de Aula - Reexes sobre a sala de aula - Clique para ler

1.2 O Conjunto dos Nmeros Naturais

Lentamente, medida em que se civilizava, a humanidade apoderou-se desse

modelo abstrato de contagem (um, dois, trs, quatro, ...) que so os nmeros

naturais. Foi uma evoluo demorada. As tribos mais rudimentares contam

apenas um, dois, muitos . A lngua inglesa ainda guarda um resqucio desse

estgio na palavra thrice, que tanto pode signicar trs vezes como muito

ou extremamente.

As necessidades provocadas por um sistema social cada vez mais complexo

e as longas reexs, possveis graas disponibilidade de tempo trazida pelo

progresso econmico, conduziram, atravs dos sculos, ao aperfeioamento do

extraordinrio instrumento de avaliao que o conjunto dos nmeros naturais.

Decorridos muitos milnios, podemos hoje descrever concisa e precisamente

o conjunto N dos nmeros naturais, valendo-nos da notvel sntese feita pelo

matemtico italiano Giuseppe Peano no limiar do sculo 20.

N um conjunto, cujos elementos so chamados nmeros naturais . A

essncia da caracterizao de N reside na palavra sucessor. Intuitivamente,


0 0
quando n, n N, dizer que n o sucessor de n signica que n0 vem logo depois
0
de n, no havendo outros nmeros naturais entre n e n . Evidentemente, esta

explicao apenas substitui sucessor por logo depois, portanto no uma

denio. O termo primitivo sucessor no denido explicitamente. Seu uso

e suas propriedades so regidos por algumas regras, abaixo enumeradas:

a) Todo nmero natural tem um nico sucessor;

b) Nmeros naturais diferentes tm sucessores diferentes;

c) Existe um nico nmero natural, chamado um e representado pelo smbolo

1, que no sucessor de nenhum outro;

d) Seja X um conjunto de nmeros naturais (isto , X N). Se1 X e se,


alm disso, o sucessor de todo elemento de X ainda pertence a X , ento
X = N.

3
Unidade 1 O Axioma da Induo

As armaes (a), (b), (c) e (d) acima so conhecidas como os axiomas de

Peano. Tudo o que se sabe sobre os nmeros naturais pode ser demonstrado

como consequncia desses axiomas.

+ Para Saber Mais - Sobre o sistema de numerao - Clique para ler

+ Para Saber Mais - Um comentrio gramatical - Clique para ler

+ Na Sala de Aula - Uma recomendao - Clique para ler

1.3 O Axioma da Induo

O ltimo dos axiomas de Peano conhecido como o axioma da induo . Ele

a base de um eciente mtodo de demonstrao de proposies referentes a

nmeros naturais (demonstraes por induo, ou por recorrncia). Enunciado

sob a forma de propriedades em vez de conjuntos, ele se formula assim:

Seja P (n) uma propriedade relativa ao nmero natural n. Suponhamos que

i) P (1) vlida;

ii) Para todo n N, a validez de P (n) implica a validez de P (n0 ), onde n0 o

sucessor de n.
Ento P (n) vlida qualquer que seja o nmero natural n.

Com efeito, se chamarmos de X o conjunto dos nmeros naturais n para

os quais P (n) vlida, veremos que:


1 X em virtude de (i); e que
n X n0 X em virtude de (ii).

Logo, pelo axioma da induo, conclumos que X = N.

Definio 1 Esta formulao do Axioma da Induo chamada de Princpio de Induo

Matemtica

4
Nmeros Naturais Unidade 1

+ Para Saber Mais - Cuidado! - Clique para ler

1.4 As Duas Operaes: Adio e Multipli-


cao

Entre os nmeros naturais esto denidas duas operaes fundamentais:

a adio, que aos nmeros n, p N faz corresponder a soma n+p e a

multiplicao , que lhes associa o produto np.


A soma n+p o nmero natural que se obtm a partir de n aplicando-se

p vezes seguidas a operao de tomar o sucessor. Em particular, n+1 o

sucessor de n, n + 2 o sucessor do sucessor de n, etc. Por exemplo, tem-se

2+2=4 simplesmente porque 4 o sucessor do sucessor de 2.


De agora em diante, o sucessor do nmero natural n ser designado por

n + 1.
Quanto ao produto, pe-se n1 = n por denio e, quando p 6= 1, np a

soma de p parcelas iguais a n.


Em ltima anlise, a soma n + p e o produto np tm mesmo os signicados
que lhes so atribudos pelas explicaes dadas acima. Entretanto, at que

saibamos utilizar os nmeros naturais para efetuar contagens, no tem sentido

falar em  p vezes e  p parcelas. Por isso, as operaes fundamentais devem

ser denidas por induo, como se segue.

Adio: n+1 = sucessor de n


n + (p + 1) = (n + p) + 1 . Esta ltima
e

igualdade diz que se sabemos somar p a todos os nmeros naturais n, sabemos

tambm somar p + 1: a soma n + (p + 1) simplesmente o sucessor (n + p) + 1

de n + p . O axioma da induo garante que a soma n + p est denida para

quaisquer n, p N.

Multiplicao: n1 = n e n(p+1) = np+n. Ou seja: multiplicar um nmero

n por 1 no o altera. E se sabemos multiplicar todos os nmeros naturais n por


p, sabemos tambm multiplic-los por p+1: basta tomar n(p+1) = np+n. Por
induo, sabemos multiplicar todo n por qualquer p. Estas operaes gozam das

conhecidas propriedades de associatividade, comutatividade e distributividade.

As demonstraes so feitas por induo. (Voltaremos ao assunto na Unidade

5 de MA12, onde mais detalhes sero apresentados.)

5
Unidade 1 A Ordenao nos Nmeros Naturais

1.5 A Ordenao nos Nmeros Naturais

Nossa breve descrio do conjunto N dos nmeros naturais termina com a

relao de ordem m < n.


Dados m, n N, diz-se quem menor do que n, e escreve-se m < n,
para signicar que existe algum p N tal que n = m + p. (Isto quer dizer

que n o sucessor do sucessor... do sucessor de m, o ato de tomar o sucessor

sendo iterado p vezes.)


A relao m < n tem as seguintes propriedades:

Transitividade: Se m<n e n<p ento m < p.

Tricotomia: m, n N,
Dados vale uma, e somente uma, das alternativas:

m = n, m < n ou n < m.

Monotonicidade: Se m < n ento, para qualquer p N, tem-se m+p < n+p


e mp < np.

Boa-ordenao: Todo subconjunto no-vazioX N possui um menor ele-


mento. Isto signica que existe um elemento m0 X que menor do que

todos os demais elementos de X . A boa-ordenao pode muitas vezes

substituir com vantagem a induo como mtodo de prova de resultados

referentes a nmeros naturais.

So muito raros e pouco interessantes os exemplos de demonstrao por

induo que podem ser dados sem usar as operaes fundamentais e as desi-

gualdades. Por isso, somente agora apresentamos um deles, seguido de uma

demonstrao por boa-ordenao.

Exemplo 1. Queremos provar a validez, para todo nmero natural n, da

igualdade

P (n) : 1 + 3 + 5 + . . . + (2n 1) = n2
Usaremos induo. Para n = 1, P (1) se resume a armar que 1 = 1. Supondo

P (n) verdadeira para um certo valor de n, somamos 2n+1 a ambos os membros


da igualdade acima, obtendo

1 + 3 + 5 + . . . + (2n 1) + (2n + 1) = n2 + 2n + 1,

6
Nmeros Naturais Unidade 1

ou seja:

1 + 3 + 5 + . . . + [2(n + 1) 1] = (n + 1)2 .
Mas esta ltima igualdade P (n + 1). Logo P (n) P (n + 1). Assim, P (n)
vale para todo n N. Podemos ento armar que a soma dos n primeiros

nmeros mpares igual ao quadrado de n.

Exemplo 2. (Usando boa-ordenao.) Lembremos que um nmero natural

p chama-se primo quando no pode ser expresso como produto p = mn de dois


nmeros naturais, a menos que um deles seja igual a 1 (e o outro igual a p);

isto equivale a dizer que os fatores m, n no podem ser ambos menores do que
p. Um resultado fundamental em Aritmtica diz que todo nmero natural

primo ou um produto de fatores primos. Provaremos isto por boa ordenao.

Usaremos a linguagem de conjuntos. Seja X o conjunto dos nmeros naturais

que so primos ou produtos de fatores primos. Observemos que se m e n


pertencem a X ento o produto mn pertence a X . Seja Y o complementar

de X . Assim, Y o conjunto dos nmeros naturais que no so primos nem

so produtos de fatores primos. Queremos provar que Y vazio. Isto ser

feito por reduo ao absurdo (como sempre se d nas demonstraes por boa-

ordenao). Com efeito, se Y no fosse vazio, haveria um menor elemento

aY. a pertenceriam a X . Como a


Ento todos os nmeros menores do que

no primo, ter-se-ia a = m n, com m < a e n < a, logo m X e n X .

Sendo assim, mn X . Mas mn = a, o que daria a X , uma contradio.

Segue-se que Y = , concluindo a demonstrao.

7
Unidade 1 Exerccios Recomendados

1.6 Exerccios Recomendados

1. Dado o nmero natural a, seja Y N um conjunto com as seguintes

propriedades:

(1) aY;
(2) n Y n + 1 Y .

Prove que Y contm todos os nmeros naturais maiores do que ou iguais

a a.

(Sugesto: considere o conjunto X = Ia Y , onde Ia o conjunto dos nmeros

naturais 6 a, e prove, por induo, que X = N.)

2. Use o exerccio anterior para provar que 2n + 1 6 2n para todo n>2 e,

em seguida, que n2 < 2n para todo n > 5.


3. Complete os detalhes da seguinte demonstrao do Princpio de Boa

Ordenao: Seja A N um conjunto que no possui um menor ele-

mento. Considere o conjunto X formado pelos nmeros naturais n tais


que 1, 2, ..., n no pertencem a A. Observe que 1 X e, alm disso, se

n X ento todos os elementos de A so > n + 1. Como n + 1 no


pode ser o menor elemento de A, conclua que n + 1 X . Logo, por

induo, segue-se que X = N. Portanto A vazio.

4. Prove, por induo, que


 n + 1 n
6n
n
para todo n>3 e conclua da que a sequncia

3

4
1, 2, 3, 4 . . .

decrescente a partir do terceiro termo.

5. Prove, por induo, que

n(n + 1)(2n + 1)
1 + 2 2 + 3 2 + + n2 = .
6
6. Critique a seguinte argumentao: Quer-se provar que todo nmero natu-

ral pequeno. Evidentemente, 1 um nmero pequeno. Alm disso, se n


for pequeno, n+1 tambm o ser, pois no se torna grande um nmero

pequeno simplesmente somando-lhe uma unidade. Logo, por induo,

todo nmero natural pequeno.

8
Nmeros Naturais Unidade 1

7. Use a distributividade para calcular (m + n)(1 + 1) de duas maneiras

diferentes e em seguida use a lei do corte para concluir que m+n = n+m.
8. Seja X N um conjunto no-vazio, com a seguinte propriedade: para
qualquer n N, se todos os nmeros naturais menores do que n perten-

cem a X ento n X . Prove que X = N. (Sugesto : boa ordenao.)

9. Seja P (n) uma propriedade relativa ao nmero natural n. Suponha que


P (1), P (2) so verdadeiras e que, para qualquer n N, a verdade de
P (n) e P (n + 1) implica a verdade de P (n + 2). Prove que P (n)
verdadeira para todo n N.

10. Use induo para provar que

1
13 + 23 + 33 + + n3 = n2 (n + 1)2 .
4

9
Unidade 1 Textos Complementares

1.7 Textos Complementares

Na Sala de Aula Reexes sobre a sala de aula


Do ponto de vista do ensino em nvel do ensino mdio, no tem cabimento

expor a Matemtica sob forma axiomtica. Mas necessrio que o professor

saiba que ela pode ser organizada sob a forma acima delineada. Uma linha de

equilbrio a ser seguida na sala de aula deve basear-se nos seguintes preceitos:

1. Nunca dar explicaes falsas sob o pretexto de que os alunos ainda no tm

maturidade para entender a verdade. (Isto seria como dizer a uma criana que

os bebs so trazidos pela cegonha.) Exemplo: innito um nmero muito

grande. Para outro exemplo, vide RPM 29, pgs. 13-19.

2. No insistir em detalhes formais para justicar armaes que, alm de

verdadeiras, so intuitivamente bvias e aceitas por todos sem discusso nem

dvidas. Exemplo: o segmento de reta que une um ponto interior a um ponto

exterior de uma circunferncia tem exatamente um ponto em comum com essa

circunferncia.

Em contraposio, fatos importantes cuja veracidade no evidente, como

o Teorema de Pitgoras ou a Frmula de Euler para poliedros convexos, devem

ser demonstrados (at mesmo de vrias formas diferentes).

Excetuam-se, naturalmente, demonstraes longas, elaboradas ou que fa-

am uso de noes e resultados acima do alcance dos estudantes desse nvel

(como o Teorema Fundamental da Algebra, por exemplo).

Provar o bvio transmite a falsa impresso de que a Matemtica intil. Por

outro lado, usar argumentos elegantes e convincentes para demonstrar resulta-

dos inesperados uma maneira de exibir sua fora e sua beleza. As demons-

traes, quando objetivas e bem apresentadas, contribuem para desenvolver o

raciocnio, o esprito crtico, a maturidade e ajudam a entender o encadeamento

lgico das proposies matemticas.

3. Ter sempre em mente que, embora a Matemtica possa ser cultivada por

si mesma, como um todo coerente, de elevado padro intelectual, formado por

conceitos e proposies de natureza abstrata, sua presena no currculo escolar

no se deve apenas ao valor dos seus mtodos para a formao mental dos

jovens.

A importncia social da Matemtica provm de que ela fornece modelos

10
Nmeros Naturais Unidade 1

para analisar situaes da vida real. Assim, por exemplo, conjuntos so o

modelo para disciplinar o raciocnio lgico, nmeros naturais so o modelo para

contagem e nmeros reais so o modelo para medida; funes ans servem de

modelo para situaes, como o movimento uniforme, em que os acrscimos da

funo so proporcionais aos acrscimos da varivel independente. E assim por

diante.

11
Unidade 1 Textos Complementares

Na Sala de Aula Uma recomendao


No se deve dar muita importncia eterna questo de saber se 0 (zero) deve

ou no ser includo entre os nmeros naturais. (Vide Meu Professor de Ma-

temtica, pg. 150.) Praticamente todos os livros de Matemtica usados nas

escolas brasileiras consideram 0 como o primeiro nmero natural (consequente-


mente 1 o segundo, 2 o terceiro, etc). Como se viu acima, no adotamos

esse ponto-de-vista. Trata-se, evidentemente, de uma questo de preferncia.

Deve-se lembrar que o smbolo 0 (sob diferentes formas grcas) foi empregado
inicialmente pelos maias, posteriormente pelos hindus, difundido pelos rabes e

adotado no ocidente, no como um nmero e sim como um algarismo, com o

utilssimo objetivo de preencher uma casa decimal vazia. (No caso dos maias, a

base do sistema de numerao era 20, e no 10.) De resto, a opo do nmero

natural para iniciar a sequncia no se limita a escolher entre 0 e 1. Frequente-

mente esquecemos que, do mesmo modo que conhecemos e usamos o zero mas

comeamos os nmeros naturais com 1, a Matemtica grega, segundo apre-


sentada por Euclides, no considerava 1 como um nmero. Nos Elementos,
encontramos as seguintes denies:

Unidade aquilo pelo qual cada objeto um. Nmero uma multitude de

unidades.

12
Nmeros Naturais Unidade 1

Comentrios sobre Denies e axiomas Para Saber Mais


Uma denio matemtica uma conveno que consiste usar um nome, ou

uma breve sentena, para designar um objeto ou uma propriedade, cuja descrio

normalmente exigiria o emprego de uma sentena mais longa. Vejamos algumas

denies, como exemplo.

ngulo a gura formada por duas semirretas que tm a mesma origem.

Primos entre si so dois ou mais nmeros naturais cujo nico divisor

comum a unidade.

Mas nem sempre foi assim. Euclides, por exemplo, comea os Elementos

com uma srie de denies, das quais selecionamos as seguintes:

Linha um comprimento sem largura.

Superfcie o que possui comprimento e largura somente.

Quando uma reta corta outra formando ngulos adjacentes iguais, cada

um desses ngulos chama-se reto e as retas se dizem perpendiculares .

As denies de ngulo e de nmeros primos entre si, dadas acima, bem

como as denies de ngulo reto e retas perpendiculares dadas por Euclides,

so corretas. Elas atendem aos padres atuais de preciso e objetividade. Por

outro lado, nas denies de linha e supercie, Euclides visa apenas oferecer ao

seu leitor uma imagem intuitiva desses conceitos. Elas podem servir para ilustrar

o pensamento geomtrico mas no so utilizveis nos raciocnios matemticos

porque so formuladas em termos vagos e imprecisos.

Na apresentao de uma teoria matemtica, toda denio faz uso de termos

especcos, os quais foram denidos usando outros termos, e assim sucessiva-

mente. Este processo iterativo leva a trs possibilidades:

a) Continua indenidamente, cada denio dependendo de outras anteriores,

sem nunca chegar ao m.

b) Conduz a uma circularidade, como nos dicionrios. (Onde se v, por exemplo:

compreender perceber, perceber entender e entender compreender.)

13
Unidade 1 Textos Complementares

c) Termina numa palavra, ou num conjunto de palavras (de preferncia dotadas

de conotaes intuitivas simples) que no so denidas, isto , que so tomadas

como representativas de conceitos primitivos. Exemplos: ponto, reta, conjunto.

Evidentemente, as alternativas (a) e (b) acima citadas no convm Ma-

temtica. A alternativa (c) a adotada. Se prestarmos ateno, veremos que

foi assim que aprendemos a falar. Numerosas palavras nos foram apresentadas

sem denio e permanecem at hoje em nosso vocabulrio como conceitos

primitivos, que aprendemos a usar por imitao e experincia.

Para poder empregar os conceitos primitivos adequadamente, necessrio

dispor de um conjunto de princpios ou regras que disciplinem sua utilizao

e estabeleam suas propriedades. Tais princpios so chamados axiomas ou

postulados . Assim como os conceitos primitivos so objetos que no se denem,

os axiomas so proposies que no se demonstram.

Uma vez feita a lista dos conceitos primitivos e enunciados os axiomas

de uma teoria matemtica, todas as demais noes devem ser denidas e as

armaes seguintes devem ser demonstradas.

Nisto consiste o chamado mtodo axiomtico . As proposies a serem

demonstradas chamam-se teoremas e suas consequncias imediatas so

denominadas corolrios. Uma proposio auxiliar, usada na demonstrao de

um teorema, chamada um lema.

Ser um axioma ou ser um teorema no uma caracterstica intrnseca de

uma proposio. Dependendo da preferncia de quem organiza a apresentao

da teoria, uma determinada proposio pode ser adotada como axioma ou ento

provada como teorema, a partir de outra proposio que a substituiu na lista

dos axiomas.

A seguir veremos um resumo da teoria matemtica dos nmeros naturais,

onde os conceitos primitivos so nmero natural e sucessor e os axiomas so

os de Peano.

14
Nmeros Naturais Unidade 1

Sobre o sistema de numerao Para Saber Mais


Um engenhoso processo, chamado sistema de numerao decimal , permite

representar todos os nmeros naturais com o auxlio dos smbolos 0, 1, 2, 3, 4,

5, 6, 7, 8 e 9. Alm disso, os primeiros nmeros naturais tm nomes: o sucessor

do nmero um chama se dois, o sucessor de dois chama-se trs, etc. A partir

de um certo ponto, esses nomes tornam-se muito complicados, sendo prefervel

abrir mo deles e designar os grandes nmeros por sua representao decimal.

(Na realidade, os nmeros muito grandes no possuem nomes. Por exemplo,

como se chamaria o nmero 101000 ?).


Deve car claro que o conjunto N = {1, 2, 3, . . .} dos nmeros naturais

uma sequncia de objetos abstratos que, em princpio, so vazios de signicado.

Cada um desses objetos (um nmero natural) possui apenas um lugar determi-

nado nesta sequncia. Nenhuma outra propriedade lhe serve de denio. Todo

nmero tem um sucessor (nico) e, com exceo de 1, tem tambm um nico

antecessor (nmero do qual sucessor).

Vistos desta maneira, podemos dizer que os nmeros naturais so nmeros

ordinais : 1 o primeiro, 2 o segundo, etc.

15
Unidade 1 Textos Complementares

Para Saber Mais Um comentrio gramatical


Quando dizemos o nmero um, o nmero dois ou o nmero trs, as

palavras um, dois e trs so substantivos, pois so nomes de objetos. Isto

contrasta com o uso destas palavras em frases como um ano, dois meses e

trs dias, onde elas aparecem para dar a ideia de nmero cardinal, isto , como

resultados de contagens. Nesta frase, um, dois e trs no so substanti-

vos. Pertencem a uma categoria gramatical que, noutras lnguas (como francs,

ingls e alemo, por exemplo) chamada adjetivo numeral e que os gramticos

brasileiros e portugueses, h um par de dcadas, resolveram chamar de numeral

apenas. Este comentrio visa salientar a diferena entre os nmeros naturais,

olhados como elementos do conjunto N, e o seu emprego como nmeros cardi-

nais. Este segundo aspecto ser abordado no captulo seguinte.

16
Nmeros Naturais Unidade 1

Cuidado! Para Saber Mais


O axioma da induo uma forma sagaz e operacional de dizer que qualquer

nmero natural n pode ser alcanado se partirmos de 1 e repetirmos suciente-


mente a operao de tomar o sucessor de um nmero. Ele est presente (pelo

menos de forma implcita) sempre que, ao armarmos a veracidade de uma pro-

posio referente aos nmeros naturais, vericamos que ela verdadeira para

n = 1, n = 2, n = 3 e dizemos e assim por diante.... Mas preciso ter

cuidado com esta ltima frase. Ela pressupe que P (n) P (n0 ) para todo

n N. No nal deste captulo, apresentamos como exerccios algumas propo-

sies demonstrveis por recorrncia, bem como alguns curiosos paradoxos que

resultam do uso inadequado do axioma da induo.

17
MA12 - Unidade 1
Numeros Naturais

Paulo Cezar Pinto Carvalho

PROFMAT - SBM

January 27, 2014


Os Numeros Naturais

Numeros Naturais: modelo abstrato para contagem.


N = {1, 2, 3, ...}
Uma descricao precisa e concisa de N e dada pelos Axiomas
de Peano.
Nocao fundamental: a de sucessor de um numero natural (ou
seja, o numero que, intuitivamente, vem logo depois dele).

PROFMAT - SBM MA12 - Unidade 1 , Numeros Naturais slide 2/8


Os Axiomas de Peano

a) Todo numero natural tem um unico sucessor;


b) Numeros naturais diferentes tem sucessores diferentes;
c) Existe um unico numero natural, chamado um e representado
pelo smbolo 1, que nao e sucessor de nenhum outro;
d) Seja X um conjunto de numeros naturais (isto e, X N). Se
1 X e se, alem disso, o sucessor de todo elemento de X
ainda pertence a X , entao X = N.

PROFMAT - SBM MA12 - Unidade 1 , Numeros Naturais slide 3/8


O Axioma da Inducao

O ultimo dos axiomas de Peano e conhecido como Axioma da


Inducao e e a base para um metodo de demonstracao para
propriedades relativas aos numeros naturais (demonstracoes
por inducao).
Seja P(n) uma propriedade relativa ao numero natural n.
Suponhamos que:
i) P(1) e valida;
ii) Para todo n N, a validez de P(n) implica a validez de P(n0 ),
onde n0 (ou n + 1) e o sucessor de n.
Entao P(n) e valida qualquer que seja o numero natural n.

PROFMAT - SBM MA12 - Unidade 1 , Numeros Naturais slide 4/8


Exemplo: uma demonstracao por inducao

Provar a validez, para todo numero natural n, da igualdade


P(n) : 1 + 3 + 5 + . . . + (2n 1) = n2
Para n = 1, P(1) se resume a afirmar que 1 = 1. Supondo
P(n) verdadeira para um certo valor de n, somamos 2n + 1 a
ambos os membros da igualdade acima, obtendo

1 + 3 + 5 + . . . + (2n 1) + (2n + 1) = n2 + 2n + 1,

ou seja:

1 + 3 + 5 + . . . + [2(n + 1) 1] = (n + 1)2 .

Mas esta ultima igualdade e P(n + 1). Logo


P(n) P(n + 1).
Assim, P(n) vale para todo n N.

PROFMAT - SBM MA12 - Unidade 1 , Numeros Naturais slide 5/8


As Duas Operacoes: Adicao e Multiplicacao

A soma n + p e o numero natural que se obtem a partir de n


aplicando-se p vezes seguidas a operacao de tomar o sucessor.
Em particular, n + 1 e o sucessor de n, n + 2 e o sucessor do
sucessor de n, etc.
Quanto ao produto, poe-se n 1 = n por definicao e, quando
p 6= 1, np e a soma de p parcelas iguais a n.
Estas operacoes podem ser formalizadas usando inducao.

PROFMAT - SBM MA12 - Unidade 1 , Numeros Naturais slide 6/8


Usando inducao para definir as operacoes

Adicao:
n + 1 = sucessor de n
n + (p + 1) = (n + p) + 1 .
Multiplicacao:
n1=n
n(p + 1) = np + n.
As propriedades destas operacoes (comutativa, associativa,
etc) podem ser demonstradas por inducao.

PROFMAT - SBM MA12 - Unidade 1 , Numeros Naturais slide 7/8


A Ordenacao nos Numeros Naturais

Dados m, n N, diz-se que m e menor do que n, e escreve-se


m < n, para significar que existe algum p N tal que
n = m + p.
Propriedades:
a) Se m < n e n < p entao m < p.
b) Dados m, n N, vale uma, e somente uma, das alternativas:
m = n, m < n ou n < m.
c) Se m < n entao, para qualquer p N, tem-se m + p < n + p e
mp < np.
d) (Propriedade da Boa Ordenacao) Todo subconjunto nao-vazio
X N possui um menor elemento. Isto significa que existe um
elemento n0 X que e menor do que todos os demais
elementos de X .

PROFMAT - SBM MA12 - Unidade 1 , Numeros Naturais slide 8/8


Lista de Exerccios
Unidade 1

1. O diagrama abaixo, em que a seta indica o sucessor de cada elemento,


representa a estrutura dos numeros naturais imposta pelos axiomas de
Peano.

1 2 3 4 5 6 7 ...

Em cada um dos diagramas a seguir, exatamente um dos axiomas de


Peano e violado. Diga qual e ele.

1 2 3 4 5 6 7 ...
a)

1 2 3 4 5 6 7 ...
b)

1 2 3 4 5 6 7 ...
c)

1 2 3 4 5 6 7 ...
d)

2. Prove, por inducao, que


n(n + 1)(2n + 1)
1 + 22 + 32 + + n2 = .
6

3. Diga onde esta o erro da seguinte demonstracao da afirmativa


1 + 2 + 4 + 8 + . . . + 2n = 2n+1 .
A propriedade e trivialmente valida para n = 1. Suponhamos que seja
valida para n, ou seja 1+2+4+8+. . .+2n = 2n+1 . Entao 1+2+4+8+

1
. . . + 2n + 2n+1 = 2n+1 + 2n+1 = 2.2n+1 = 2n+2 . Portanto, a propriedade
tambem e valida para n + 1. Logo, pelo Princpio da Inducao Finita,
1 + 2 + 4 + 8 + . . . + 2n = 2n+1 para todo n .

4. Usando inducao e a propriedade associativa da adicao, demonstre a lei


do corte: Se m, n e p sao numeros naturais tais que m + p = n + p,
entao m = n. [Sugestao: use inducao em p, notando que o caso base
da inducao e o segundo axioma de Peano. ]

5. Demonstre a propriedade transitiva da ordem: Se m, n e p sao numeros


naturais tais que m < n e n < p, entao m < p.

2
Solucoes da Lista de Exerccios
Unidade 1

1. a) O quarto axioma e violado. O subconjunto {1, 2, 5, 6, . . .} contem 1


e o sucessor de cada elemento, mas nao e igual a N.
b) O terceiro axioma e violado. O elemento 2 tambem nao e sucessor
de um natural.
c) O segundo e terceiro axiomas sao violados. O numero 3 e sucessor
de dois numeros diferentes e, alem disso, 2 tambem nao e sucessor de
nenhum natural.
d) O segundo axioma e violado. O numero 2 e sucessor de dois numeros
diferentes (1 e 7).
1.2.3
2. A formula vale para n = 1, ja que 12 = 6
.
Suponhamos que ela valha para um certo n, ou seja, 12 + 22 + 32 + +
n2 = n(n+1)(2n+1)
6
. Somando (n + 1)2 a ambos os lados da igualdade,
obtemos:
n(n+1)(2n+1)
12 + 22 + 32 + + n2 + (n + 1)2 = 6
+ (n + 1)2 =
(n+1)(2n2 +n+6n+6) (n+1)(2n2 +7n+6) (n+1)(n+2)(2n+3)
6
= 6
= 6
Logo, a formula tambem e valida para n + 1. Portanto, pelo Princpio
da Inducao, a formula e valida para todo n natural.

3. Na verdade, a propriedade nao vale para n = 1, ja que 1 + 21 6= 21+1 .

4. Para p = 1, a afirmativa vale pelo segundo axioma de Peano: se os


sucessores m + 1 e n + 1 de m e n sao iguais, entao m = n.
Suponhamos agora que a propriedade valha para algum natural p. Isto
e, m + p = n + p implique m = n. Suponhamos que m + (p + 1) =
n + (p + 1). Pela propriedade associativa da adicao, a igualdade e
equivalente a (m + 1) + p = (n + 1) + p. Mas pela hipotese de inducao,
isto implica m + 1 = n + 1, que por sua vez implica m = n (pelo caso
em que p = 1). Logo, se a propriedade vale para p entao vale tambem
para p + 1.

1
Portanto, pelo Princpio da Inducao, a lei do corte vale para todo p
natural.

5. Suponhamos que m < n e n < p. Entao, pela definicao da ordem,


existem naturais r e s tais que m + r = n e n + s = p. Substituindo
a expressao de n fornecida pela primeira igualdade na segunda, temos
(m + r) + s = p, que e equivalente a m + (r + s) = p. Logo, m < p.

2
2
Nmeros Cardinais

Sumrio
2.1 Introduo . . . . . . . . . . . . . . . . . . . . . . . 2

2.2 Funes . . . . . . . . . . . . . . . . . . . . . . . . . 2

2.3 A Noo de Nmero Cardinal . . . . . . . . . . . . 4

2.4 Conjuntos Finitos . . . . . . . . . . . . . . . . . . . 7

2.5 Exerccios Recomendados . . . . . . . . . . . . . . . 10

2.6 Exerccios Suplementares . . . . . . . . . . . . . . . 10

1
Unidade 2 Introduo

2.1 Introduo

A importncia dos nmeros naturais provm do fato de que eles constituem

o modelo matemtico que torna possvel o processo de contagem. Noutras

palavras, eles respondem a perguntas do tipo: Quantos elementos tem este

conjunto?

Para contar os elementos de um conjunto necessrio usar a noo de cor-

respondncia biunvoca, ou bijeo. Trata-se de um caso particular do conceito

de funo, abordado aqui de forma breve, que ser desenvolvido com maiores

detalhes na Unidade 3 de MA11.

2.2 Funes

Definio 1 Dados os conjuntos X, Y , uma funo f : X Y (l-se uma funo de

X em Y ) uma regra (ou conjunto de instrues) que diz como associar a

cada elemento xX um elemento y = f (x) Y .

O conjunto X chama-se o domnio e Y o contra-domnio da funo f .


Para cada x X , o elemento f (x) Y chama-se a imagem de x pela funo

f , ou o valor assumido pela funo f no ponto x X . Escreve-se x 7 f (x)


para indicar que f transforma (ou leva) x em f (x).

Exemplos particularmente simples de funes so a funo identidade f :


X X , denida por f (x) = x para todo x X e as funes constantes

f : X Y , onde se toma um elemento c Y e se pe f (x) = c para todo

x X.

+ Para Saber Mais - Recomendaes - Clique para ler

Exemplo 1 Sejam X e R o conjunto dos nmeros


o conjunto dos tringulos do plano

reais (que abordaremos logo mais). Se, a cada t X , zermos corresponder o

nmero real f (t) = rea do tringulo t, obteremos uma funo f : X R.

2
Nmeros Cardinais Unidade 2

Sejam S o conjunto dos segmentos de reta do plano e o conjunto das Exemplo 2


retas desse mesmo plano. A regra que associa a cada segmento AB S sua

mediatriz g(AB) dene uma funo g : S .

A correspondncia que associa a cada nmero natural n seu sucessor n + 1 Exemplo 3


dene uma funo s : N N, com s(n) = n + 1.

Uma funo f :X Y chama-se injetiva quando elementos diferentes Definio 2


em X so transformados por f em elementos diferentes em Y. Ou seja, f
0 0
injetiva quando x 6= x em X f (x) 6= f (x ).

Esta condio pode tambm ser expressa em sua forma contrapositiva:

f (x) = f (x0 ) x = x0 .

Nos trs exemplos dados acima, apenas o terceiro de uma funo injetiva.

(Dois tringulos diferentes podem ter a mesma rea e dois segmentos distintos

podem ter a mesma mediatriz mas nmeros naturais diferentes tm sucessores

diferentes.)

Diz-se que uma funo f :X Y sobrejetiva quando, para qualquer Definio 3


elemento y Y, pode-se encontrar (pelo menos) um elemento xX tal que

f (x) = y .

Nos trs exemplos dados acima, apenas o segundo apresenta uma funo

sobrejetiva. (Toda reta do plano mediatriz de algum segmento mas apenas

os nmeros reais positivos podem ser reas de tringulos e o nmero 1 no

sucessor de nmero natural algum.)

Chama-se imagem do subconjunto A X pela funo f : X Y ao Definio 4


subconjunto f (A) Y formado pelos elementos f (x), com x A.

3
Unidade 2 A Noo de Nmero Cardinal

Portanto, uma funo f : X Y sobrejetiva quando f (X) = Y . O


conjuntof (X), imagem do domnio X pela funo f chama-se tambm a

imagem da funo f .
Nos Exemplos 1, 2 e 3, a imagem da funo f o conjunto dos nmeros

reais positivos, a imagem de g todo o conjunto e a imagem de s o


conjunto dos nmeros naturais 2.

Dada a funo f : X Y , para saber se um certo elemento b Y pertence

ou no imagem f (X), escrevemos a equao f (x) = b e procuramos


achar algum x X que a satisfaa. Consequentemente, para mostrar que f

sobrejetiva deve-se provar que a equao f (x) = y possui uma soluo x X ,

seja qual for o y Y dado.

+ Para Saber Mais - Recomendao - Clique para ler

Exemplo 4 Considere a tentativa de denir uma funo f : N N, estipulando que,

para todon N, o nmero natural p = f (n) deve ser tal que p2 + 3 = n. O


nmero p = f (n) s pode ser encontrado se n for igual a 4, 7, 12, 19, ... pois
2
nem todos os nmeros naturais so da forma p + 3. Assim, esta regra no

dene uma funo com domnio N, porque tem excees.

Exemplo 5 Indiquemos com X o conjunto dos nmeros reais positivos e com Y o

conjunto dos tringulos do plano. Para cada x X , ponhamos f (x) = t caso t


seja um tringulo cuja rea x. Esta regra no dene uma funo f : X Y

porque ambgua: dado o nmero x > 0, existe uma innidade de tringulos

diferentes com rea x.

2.3 A Noo de Nmero Cardinal

A conceito de nmero cadinal se estabelece por meio da noo de bijeo.

Definio 5 Uma funo f : X Y chama-se uma bijeo, ou uma correspondncia


biunvoca entre X e Y quando ao mesmo tempo injetiva e sobrejetiva.

4
Nmeros Cardinais Unidade 2

SejamX = {1, 2, 3, 4, 5} e Y = {2, 4, 6, 8, 10}. Denindo f : X Y Exemplo 6


pela regra f (n) = 2n, temos uma correspondncia biunvoca, onde f (1) = 2,

f (2) = 4, f (3) = 6, f (4) = 8 e f (5) = 10.

Um exemplo particularmente curioso de correspondncia biunvoca, que Exemplo 7


estende o exemplo anterior, foi descoberto pelo fsico Galileu Galilei, que viveu

h quatrocentos anos. Seja P o conjunto dos nmeros naturais pares:

P = {2, 4, 6, . . . , 2n, . . .}.

Obtm-se uma correspondncia biunvoca f : N P pondo-se f (n) = 2n para


todo n N. O interessante deste exemplo que P um subconjunto prprio

de N.

Sejam Y a base de um tringulo e X um segmento paralelo a Y, unindo Exemplo 8


os outros dois lados desse tringulo. P o vrtice oposto
Seja ainda base

Y . Obtm-se uma correspondncia biunvoca f : X Y associando a cada

x X o ponto f (x) onde a semirreta P x intersecta a base Y .

5
Unidade 2 A Noo de Nmero Cardinal

Figura 2.1: Correspondncia biunvoca entre dois segmentos

Exemplo 9 Neste exemplo, X = C \ {P } o conjunto obtido retirando da circun-

ferncia o ponto P e Y uma reta perpendicular ao dimetro que no passa

por P . Dena a correspondncia biunvoca f : X Y pondo, para cada

x X, f (x) = interseco da semirreta P x com a reta Y .

Figura 2.2: O crculo sem um ponto e a reta

Definio 6 Diz-se que dois conjuntos X e Y tem omesmo nmero cardinal quando

se pode denir uma correspondncia biunvoca f : X Y .

Cada um dos quatro exemplos acima exibe um par de conjuntos X, Y com

o mesmo cardinal.

6
Nmeros Cardinais Unidade 2

Sejam X = {1} e Y = {1, 2}. Evidentemente no pode existir uma Exemplo 10


correspondncia biunvoca f : X Y , portanto X e Y no tm o mesmo
nmero cardinal.

+ Para Saber Mais - A palavra nmero no dicionrio - Clique para ler

2.4 Conjuntos Finitos

Dado n N, indiquemos com a notao In o conjunto dos nmeros naturais


de 1 at n. Assim, I1 = {1}, I2 = {1, 2}, I3 = {1, 2, 3} e, mais geralmente,

um nmero natural k pertence a In se, e somente se, 1 k n.

Seja X um conjunto. Diz-se que X nito, X tem n elementos


e que Definio 7
quando se pode estabelecer uma correspondncia biunvoca f : In X .

O nmero natural n chama-se ento o nmero cardinal do conjunto X ou,


simplesmente, o nmero de elementos de X . A correspondncia f : In X

chama-se uma contagem dos elementos de X . Pondo f (1) = x1 , f (2) =

x2 , . . . , f (n) = xn , podemos escrever X = {x1 , x2 , . . . , xn }. Para todo n, o


conjunto In nito e seu nmero cardinal n. Assim, todo nmero natural n

o nmero cardinal de algum conjunto nito.

A m de evitar excees, admite-se ainda incluir o conjunto vazio entre

os conjuntos nitos e diz-se que tem zero elementos. Assim, por denio,

zero o nmero cardinal do conjunto vazio.

Diz-se que um conjunto X innito quando ele no nito. Isto quer

dizer que X no vazio e que, no importa qual seja n N , no existe

correspondncia biunvoca f : In X .
No Exemplo 6 acima, temos X = I5 e f : X Y uma contagem

dos elementos de Y . Assim, Y um conjunto nito, com 5 elementos. O

conjunto N dos nmeros naturais innito. Com efeito, dada qualquer funo

f : In N , no importa qual n se xou, pomos k = f (1) + f (2) + + f (n)


e vemos que, para todo x In , tem-se f (x) < k , logo no existe x In tal

7
Unidade 2 Conjuntos Finitos

que f (x) = k . Assim, impossvel cumprir a condio de sobrejetividade na

denio de correspondncia biunvoca.

O nmero cardinal de um conjunto nito X , que indicaremos com a notao


n(X), goza de algumas propriedades bsicas, entre as quais destacaremos as

seguintes:

1. O nmero de elementos de um conjunto nito o mesmo, seja qual for


a contagem que se adote. Isto signica que se f : Im X e g : In X
so correspondncias biunvocas ento m = n.

2. Todo subconjunto Y de um conjunto nito X nito e n(Y ) n(X).


Tem-se n(Y ) = n(X) somente quando Y = X .

3. Se X e Y so nitos ento X Y nito e tem-se n(X Y ) = n(X) +


n(Y ) n(X Y ) .

4. Sejam X , Y conjuntos nitos. Se n(X) > n(Y ), nenhuma funo f :


X Y injetiva e nenhuma funo g : Y X sobrejetiva.

As demonstraes destes fatos se fazem por induoo ou por boa-ordenao.

(Veja, por exemplo, [Lima]: Curso de Anlise, vol. 1, pgs. 33-38.) A primeira
parte do item 4. acima conhecida como o princpio das casas de pombos : se

h mais pombos do que casas num pombal, qualquer modo de alojar os pombos

dever colocar pelo menos dois deles na mesma casa. As vezes, o mesmo fato

chamado o princpio das gavetas : se m > n, qualquer maneira de distribuir


m objetos em n gavetas dever por ao menos dois desses objetos na mesma

gaveta. (Na referncia [Lima] citada, este o Corolrio 1 na pgina 35.)

O princpio das casas de pombos, com toda sua simplicidade, possui inte-

ressantes aplicaes. Vejamos duas delas.

Exemplo 11 Tomemos um nmero natural de 1 a 9. Para xar as ideias, seja 3 esse

nmero. Vamos provar que todo nmero natural m possui um mltiplo cuja
representao decimal contm apenas os algarismos 3 ou 0. Para isso, conside-

remos o conjunto X = {3, 33, . . . , 33 . . . 3}, cujos elementos so os m primeiros

nmeros naturais representados somente por algarismos iguais a 3. Se algum

dos elementos de X for mltiplo de m, nosso trabalho acabou. Caso contrrio,

8
Nmeros Cardinais Unidade 2

formamos o conjunto Y = {1, 2, . . . , m 1} e denimos a funo f :XY


pondo, para cada x X,

f (x) = resto da diviso de x por m.

Como X tem mais elementos do que Y, o princpio das casas de pombos

assegura que existem elementos x1 < x2 no conjunto X tais que f (x1 ) = f (x2 ).
Isto signica que x1 e x2 , quando divididos por m, deixam o mesmo resto. Logo

x2 x1 mltiplo de m. Mas claro que se x1 tem p algarismos e x2 tem p + q


algarismos ento a representao decimal de x2 x1 consiste em q algarismos

iguais a 3 seguidos de p algarismos iguais a 0.

Vamos usar o princpio das gavetas para provar que, numa reunio com n Exemplo 12
pessoas (n 2), h sempre duas pessoas (pelo menos) que tm o mesmo n-

mero de amigos naquele grupo. Para ver isto, imaginemos n caixas, numeradas
com 0, 1, . . . , n 1. A cada uma das n pessoas entregamos um carto que

pedimos para depositar na caixa correspondente ao nmero de amigos que ela

tem naquele grupo. As caixas de nmeros 0 e n1 no podem ambas receber

cartes pois se houver algum que no tem amigos ali, nenhum dos presentes

pode ser amigo de todos, e vice-versa. Portanto temos, na realidade, n cartes

para serem depositados em n 1 caixas. Pelo princpio das gavetas, pelo menos

uma das caixas vai receber dois ou mais cartes. Isto signica que duas ou mais

pessoas ali tm o mesmo nmero de amigos entre os presentes.

+ Para Saber Mais - Sobre Conjuntos Innitos - Clique para ler

+ Para Saber Mais - Fantasia Matemtica - Clique para ler

+ Para Saber Mais - Cuidado! - Clique para ler

9
Unidade 2 Exerccios Recomendados

2.5 Exerccios Recomendados

1. Prove, por induo, que se X um conjunto nito com n elementos

ento existem n! bijees f : X X.


2. Prove, por induo, que um conjunto com n elementos possui 2n subcon-

juntos.

3. Sejam X e Y dois conjuntos nitos, com m e n elementos, respectiva-


mente. Mostre que existem nm funes f : X Y . Voc seria capaz
de resolver diretamente o Exerccio 2, utilizando este resultado?

2.6 Exerccios Suplementares

1. Dena uma funo sobrejetiva f :NN tal que, para todo n N, a

equao f (x) = n possui uma innidade de razes x N.


Sugesto: Todo nmero natural se escreve, de modo nico sob a forma
2a b, onde a, b N e b mpar.

2. Dados n (n > 2) objetos de pesos distintos, prove que possvel deter-

minar qual o mais leve e qual o mais pesado fazendo 2n 3 pesagens

em uma balana de pratos. esse o nmero mnimo de pesagens que

permitem determinar o mais leve e o mais pesado?

3. Prove que, dado um conjunto com n elementos, possvel fazer uma la

com seus subconjuntos de tal modo que cada subconjunto da la pode

ser obtido a partir do anterior pelo acrscimo ou pela supresso de um

nico elemento.

4. Todos os quartos do Hotel Georg Cantor esto ocupados, quando chegam


os trens T1 , T2 , . . . , (em quantidade innita), cada um deles com innitos
passageiros. Que deve fazer o gerente para hospedar todos?

10
Unidade 2 Textos Complementares

2.7 Textos Complementares

Para Saber Mais Recomendaes


1. importante ressaltar que f (x) a imagem do elemento x X pela

funo f, ou o valor da funo f no ponto x X . Os livros antigos, bem

como alguns atuais, principalmente os de Clculo, costumam dizer a funo

f (x) quando deveriam dizer a funo f . Algumas vezes essa linguagem

inexata torna a comunicao mais rpida e ca difcil resistir tentao de

us-la. Mas indispensvel a cada momento ter a noo precisa do que se est

fazendo.

Na prtica, h algumas funes com as quais simples e natural lidar usando

a terminologia correta. Por exemplo, fcil acostumar-se a escrever as funes

sen : R R e log : R+ R, guardando as notaes sen x log x para os


e

nmeros reais que so os valores destas funes num dado ponto x. Por outro

lado, quando se trata de uma funo polinomial, o bom-senso nos leva a dizer

a funo x2 5x + 6

em vez da forma mais correta e mais pedante a funo p:RR tal que

p(x) = x2 5x + 6

para todo x R . Caso anlogo se d com a funo exponencial ex , embora


recentemente se tenha tornado cada vez mais frequente escrever exp(x) = ex
e assim poder falar da funo exp : R R.
2. Deve-se ainda recordar que uma funo consta de trs ingredientes: domnio,
contra-domnio e a lei de correspondncia x 7 f (x). Mesmo quando dizemos

simplesmente a funo f , cam subentendidos seu domnio X e seu contra-

domnio Y. Sem que eles sejam especicados, no existe a funo. Assim

sendo, uma pergunta do tipo Qual o domnio da funo f (x) = 1/x ?,

estritamente falando, no faz sentido. A pergunta correta seria: Qual o

maior subconjunto X R tal que a frmula f (x) = 1/x dene uma funo

f :XR ? Novamente, a pergunta incorreta mais simples de formular.

Se for feita assim, preciso saber seu signicado.

Segue-se do que foi dito acima que as funes f : X Y e g : X0 Y 0


so iguais se, e somente se, X = X 0, Y = Y 0 e f (x) = g(x) para todo x X .

12
REFERNCIAS BIBLIOGRFICAS Unidade 2

Recomendao Para Saber Mais

3. Em muitos exemplos de funes f : X Y , principalmente na Matemtica


Elementar, X e Y so conjuntos numricos e a regra x 7 f (x) exprime o valor

f (x) por meio de uma frmula que envolve x. Mas em geral no precisa ser
assim. A natureza da regra que ensina como obter f (x) quando dado x

inteiramente arbitrria, sendo sujeita apenas a duas condies:

a) No deve haver excees: a m de que a funo f tenha o conjunto X


como domnio, a regra deve fornecer f (x), seja qual for x X dado.

b) No pode haver ambiguidades: a cada x X , a regra deve fazer corres-

ponder um nico f (x) em Y . Os exemplos a seguir ilustram essas exigncias.

13
Unidade 2 Textos Complementares

Para Saber Mais A palavra nmero no dicionrio

As vezes se diz que os conjuntos X e Y so (numericamente) equivalentes


quando possvel estabelecer uma correspondncia biunvoca f : X Y , ou
seja, quando X e Y tm o mesmo nmero cardinal.

Isto explica (embora no justique) a denio dada no dicionrio mais

vendido do pas. Em algumas situaes, ocorrem em Matemtica denies

do tipo seguinte: um vetor o conjunto de todos os segmentos de reta do

plano que so equipolentes a um segmento dado. (Denio por abstrao.)

Nessa mesma veia, poder-se-ia tentar dizer: nmero cardinal de um conjunto

o conjunto de todos os conjuntos equivalentes a esse conjunto. No caso

do dicionrio, h um conjunto de defeitos naquela denio, com um nmero

cardinal razoavelmente elevado. Os trs mais graves so:

1. Um dicionrio no um compndio de Matemtica, e muito menos de L-

gica. Deve conter explicaes acessveis ao leigo (de preferncia, corretas). As

primeiras acepes da palavra nmero num dicionrio deveriam ser quanti-

dade e resultado de uma contagem ou de uma medida.

2. A denio em causa s se aplica a nmeros cardinais, mas a ideia de nmero

deveria abranger os racionais e, pelo menos, os reais.

3. O conjunto de todos os conjuntos equivalentes a um conjunto dado um

conceito matematicamente incorreto. A noo de conjunto no pode ser usada

indiscriminadamente, sem submeter-se a regras determinadas, sob pena de con-

duzir a paradoxos, ou contradies. Uma dessas regras probe que se forme

conjuntos a no ser que seus elementos pertenam a, ou sejam subconjuntos

de, um determinado conjunto-universo. Um exemplo de paradoxo que resulta

da desateno a essa regra o conjunto X de todos os conjuntos que no

so elementos de si mesmos. Pergunta-se: X ou no um elemento de si

mesmo? Qualquer que seja a resposta, chega-se a uma contradio.

14
REFERNCIAS BIBLIOGRFICAS Unidade 2

Sobre Conjuntos Innitos Para Saber Mais

Para encerrar estas consideraes a respeito de nmeros cardinais, faremos

alguns comentrios sobre conjuntos innitos.

Em primeiro lugar, convm esclarecer que a maior contribuio de Cantor

no foi a adoo da linguagem e da notao dos conjuntos e sim suas desco-

bertas sobre os nmeros cardinais de conjuntos innitos. Ele foi o primeiro a

descobrir que existem conjuntos innitos com diferentes cardinalidades ao pro-

var que no pode haver uma correspondncia biunvoca entre N e o conjunto

R dos nmeros reais e que nenhum conjunto X pode estar em correspondncia


biunvoca com o conjunto P(X) cujos elementos so os subconjuntos de X .

Alm disso, ele mostrou que a reta, o plano e o espao tri-dimensional (ou

mesmo espaos com dimenso superior a trs) tm o mesmo nmero cardinal.

Estes fatos, que atualmente so considerados corriqueiros entre os matemticos,

causaram forte impacto na poca (meados do sculo dezenove).

A segunda observao diz respeito a funes f :XX de um conjunto em

si mesmo. Quando X nito, f injetiva se, e somente se, sobrejetiva (veja

a referncia [Lima].) Mas isto no verdadeiro paraX innito. Por exemplo,


se denirmos a funo f : N N pondo, para cada n N, f (n) = nmero

de fatores primos distintos que ocorrem na decomposio de n, veremos que f

sobrejetiva mas no injetiva. (Para cada b N existe uma innidade de

nmeros n tais que f (n) = b.) Alm disso, as funes f : N N, g : N N,

h : N N e : N N, denidas por

f (n) = n + 1,
g(n) = n + 30,
(2.1)
h(n) = 2n e
(n) = 3n

so injetivas mas no so sobrejetivas. Estas quatro funes so protagonistas

da historinha seguinte que fecha a unidade.

15
Unidade 2 Textos Complementares

Para Saber Mais Fantasia Matemtica

O Grande Hotel Georg Cantor tinha uma innidade de quartos, numera-

dos consecutivamente, um para cada nmero natural. Todos eram igualmente

confortveis. Num m-de-semana prolongado, o hotel estava com seus quartos

todos ocupados, quando chega um viajante. A recepcionista vai logo dizendo:

 Sinto muito, mas no h vagas.

Ouvindo isto, o gerente interveio:

 Podemos abrigar o cavalheiro, sim senhora.

E ordena:

 Transra o hspede do quarto 1 para o quarto 2, passe o do quarto 2

para o quarto 3 e assim em diante. Quem estiver no quarto n, mude para o

quarto n + 1. Isto manter todos alojados e deixar disponvel o quarto 1 para

o recm-chegado.

Logo depois chegou um nibus com 30 passageiros, todos querendo hospe-

dagem. A recepcionista, tendo aprendido a lio, removeu o hspede de cada

quarto n para o quarto n + 30 e acolheu assim todos os passageiros do nibus.

Mas cou sem saber o que fazer quando, horas depois, chegou um trem com

uma innidade de passageiros. Desesperada, apelou para o gerente que pron-

tamente resolveu o problema dizendo:

 Passe cada hspede do quarto n para o quarto 2n. Isto deixar vagos todos

os apartamentos de nmero mpar, nos quais poremos os novos hspedes.

 Pensando melhor: mude quem est no quarto n para o quarto 3n. Os novos
hspedes, ponha-os nos quartos de nmero 3n+2. Deixaremos vagos os quartos
de nmero 3n + 1. Assim, sobraro ainda innitos quartos vazios e eu poderei

ter sossego por algum tempo.

16
REFERNCIAS BIBLIOGRFICAS Unidade 2

Cuidado! Para Saber Mais

No confunda conjunto innito com aquele que tem um nmero muito grande

(porm nito) de elementos. Quando, na linguagem comum, se diz algo como -

J ouvi isto uma innidade de vezes, trata-se de uma mera fora de expresso.

No h distncias innitas (mesmo entre duas galxias bem afastadas) e at

o nmero de tomos do universo nito. (O fsico Arthur Eddington estimou

o nmero de prtons do universo em 136 2256 . O nmero de tomos

certamente menor pois todo tomo contm ao menos um prton.) E importante

ter sempre em mente que nenhum nmero natural n maior do que todos os

demais: tem-se sempre n < n + 1.

17
MA12 - Unidade 2
Numeros Cardinais

Paulo Cezar Pinto Carvalho

PROFMAT - SBM

February 17, 2014


Numeros cardinais

A importancia dos numeros naturais provem do fato de que


eles constituem o modelo matematico que torna possvel o
processo de contagem.
Contar um conjunto X significa estabelecer uma
correspondencia biunvoca entre os elementos de X e os de
um subconjunto de N da forma
In = {x N|x n} = {1, 2, . . . , n}.
Quando e possvel estabelecer tal correspondencia biunvoca,
dizemos que X e um conjunto finito e que n e o numero
cardinal ou numero de elementos de X .

PROFMAT - SBM MA12 - Unidade 2 ,Numeros Cardinais slide 2/7


Propriedades

a) O resultado da contagem (ou seja, o numero cardinal de X ) e


sempre o mesmo, nao importando a contagem que seja feita.
b) Todo subconjunto Y de um conjunto finito X e finito e
n(Y ) n(X ). Tem-se n(Y ) = n(X ) somente quando Y = X .
Observacao: A fim de evitar excecoes, o conjunto vazio e
includo entre os conjuntos finitos e diz-se que tem zero
elementos.

PROFMAT - SBM MA12 - Unidade 2 ,Numeros Cardinais slide 3/7


Conjuntos Infinitos

Diz-se que um conjunto X e infinito quando ele nao e finito.


Isto quer dizer que X nao e vazio e que, nao importa qual seja
n N , nao existe correspondencia biunvoca f : In X .
Exemplo: o conjunto N dos numeros naturais e infinito.
Dada qualquer funcao f : In N , nao importa qual seja n ,
tomamos k = f (1) + f (2) + + f (n).
Para todo x In , tem-se f (x) < k; logo nao existe x In tal
que f (x) = k.
Assim, f nao pode ser uma correspondencia biunvoca.

PROFMAT - SBM MA12 - Unidade 2 ,Numeros Cardinais slide 4/7


Comparando conjuntos infinitos
Dois conjuntos X e Y tem a mesma cardinalidade quando e
possvel estabelecer uma correspondencia biunvoca entre X e
Y (isto e, existe uma funcao bijetiva f : X Y ).
Exemplo: os conjuntos N dos numeros naturais e
P = {2n|n N} dos numeros pares tem a mesma
cardinalidade.
A bijecao ja esta dada na definicao de P: a funcao f : N P
definida por f (n) = 2n e uma bijecao de N em P.
Os conjuntos N e N N dos pares de numeros naturais
tambem tem a mesma cardinalidade.

PROFMAT - SBM MA12 - Unidade 2 ,Numeros Cardinais slide 5/7


Conjuntos enumeraveis

Um conjunto e enumeravel quando e finito ou tem a mesma


cardinalidade de N.
Por exemplo, os conjuntos {2, 5}, N e N N sao enumeraveis.

PROFMAT - SBM MA12 - Unidade 2 ,Numeros Cardinais slide 6/7


Um exemplo de conjunto nao enumeravel

Um conjunto infinito e necessariamente enumeravel? NAO!


O conjunto de todas as sequencias em que os termos sao 0 ou
1 nao e enumeravel.
Prova: o metodo da diagonal de Cantor.
Trocando o n-esimo termo da n-esima sequencia produz-se
uma nova sequencia que nao esta na enumeracao proposta!

PROFMAT - SBM MA12 - Unidade 2 ,Numeros Cardinais slide 7/7


Lista de Exerccios
Unidade 2

1. Prove, por inducao, que se X e um conjunto finito com n elementos,


esses elementos podem ser ordenados de n! modos.
2. Prove, por inducao, que um conjunto com n elementos possui 2n sub-
conjuntos.
3. Dados 3n objetos de pesos iguais, exceto um, mais pesado que os de-
mais, mostre que e possvel determinar este objeto com n pesagens em
uma balanca de pratos. Mostre tambem que este e o numero mnimo de
pesagens que permitem, com certeza, determinar o objeto mais pesado.
4. Prove que, dado um conjunto com n elementos, e possvel fazer uma
fila com seus subconjuntos de tal modo que cada subconjunto da fila
pode ser obtido a partir do anterior pelo acrescimo ou pela supressao
de um unico elemento. [Sugestao: para passar de n para n + 1, liste
primeiro os subconjuntos que nao tem um dado elemento.]
5. Diga onde esta o erro na seguinte demonstracao da afirmativa Todas
as bolas de bilhar tem a mesma cor.
Seja P (n) a propriedade todas as bolas em um conjunto com n bolas
tem a mesma cor. A propriedade e trivialmente verdadeira para n = 1.
Suponhamos agora que ela seja verdadeira para n e consideremos um
conjunto com n + 1 bolas B = {b1 , b2 , . . . , bn , bn+1 }. Os subconjuntos
{b1 , b2 , . . . , bn , } e {b2 , . . . , bn , bn+1 } de B tem n elementos cada; logo,
pela hipotese de inducao, todas as bolas em cada um deles tem a mesma
cor. Mas os elementos b2 , . . . , bk sao comuns a esses dois subconjuntos.
Da, conclumos que todos os n + 1 elementos de B tem a mesma cor,
o que mostra que a propriedade vale para n + 1. Logo, pelo Princpio
da Inducao, em uma colecao com n bolas todas tem a mesma cor, para
todo n N.
6. Diga se cada conjunto abaixo e finito ou infinito, justificando:
o conjunto de todas as pessoas que ja viveram na Terra.

1
o conjunto de todos os multiplos de 7 cuja representacao decimal
termina com 3578.
o conjunto de todos os numeros naturais cuja representacao de-
cimal tenha apenas algarismos diferentes de zero, cuja soma seja
menor que 1000.
o conjunto de todos os numeros racionais que podem ser escritos
como fracao com denominador menor que 1000.
o conjunto de todos os numeros primos.

7. Sejam X e Y dois conjuntos infinitos enumeraveis. Isto significa que


existem sequencias (x1 , x2 , x3 , . . .) e (y1 , y2 , y3 , . . .) incluindo todos os
elementos de X e Y , respectivamente. Explique como construir uma
sequencia incluindo todos os elementos de X Y , mostrando assim que
X Y tambem e enumeravel.

8. Considere o conjunto N2 de todos os pares ordenados de numeros na-


turais. Encontre uma sequencia que inclua todos os elementos de N2 ,
mostrando assim que N2 e enumeravel. Isto mostra que o conjunto dos
numeros racionais tambem e enumeravel. Por que?

9. Mostre que o conjunto de todos os subconjuntos de N e nao enumeravel.


[Sugestao: associe cada subconjunto de N a uma sequencia em que os
termos sao iguais a 0 ou 1.]

2
Solucoes da Lista de Exerccios
Unidade 2

1. Um conjunto com 1 elemento so pode ser ordenado de 1 = 1! modo,


o que mostra que a propriedade vale para n = 1. Suponhamos que ela
valha para n e consideremos um conjunto {a1 , a2 , . . . , an , an+1 } com n+
1 elementos. As possveis ordenacoes desse conjunto podem ser obtidas
tomando cada uma das n! ordenacoes de {a1 , a2 , . . . , an } e inserindo
an+1 em uma das n + 1 posicoes possveis, gerando assim n!(n + 1) =
(n + 1)! possveis ordenacoes. Logo, a propriedade tambem vale para
n + 1. Portanto, pelo Princpio da Inducao, vale para todo n natural.

2. Neste caso, convem comecar de n = 0, para o qual a propriedade


vale, ja que o conjunto vazio possui 1 = 20 subconjunto. Suponha-
mos que a propriedade valha para n e consideremos um conjunto A =
{a1 , a2 , . . . , an , an+1 } com n + 1 elementos. Cada subconjunto de A ou
e um subconjunto {a1 , a2 , . . . , an } ou e a uniao de um tal subconjunto
com an+1 . Ou seja, cada subconjunto de {a1 , a2 , . . . , an } da origem a 2
subconjuntos de A, que tem, assim, 2.2n = 2n+1 subconjuntos. Logo,
a propriedade tambem vale para n + 1. Portanto, pelo Princpio da
Inducao, vale para todo n 0.

3. Para n = 1, basta de fato uma pesagem, feita com dois dos objetos: se
ela indicar um objeto mais pesado do que o outro, ele e o procurado; se
os objetos tiverem pesos iguais, o objeto que ficou de fora na pesagem
e o mais pesado. Suponhamos agora que seja possvel determinar qual
e mais pesado dentre 3n objetos com n pesagens e consideremos um
conjunto com 3n+1 objetos. Dividimos estes objetos em tres grupos
com 3n objetos cada e comparamos o peso de dois deles. Se um deles
for mais pesado, o objeto procurado esta nele; senao, esta no grupo
que ficou de fora da pesagem. De qualquer modo, pela hipotese de
inducao, ele pode ser encontrado em n pesagens adicionais, para um
total de n+1 pesagens. Logo, a propriedade vale para conjuntos de 3n+1
objetos e, pelo Princpio da Inducao, para conjuntos com 3n objetos,
qualquer que seja n.

1
4. Finito, infinito, finito, finito, infinito

5. A sequencia {x1 , y1 , x2 , y2 , x3 , y3 , . . .} inclui todos os elementos de X


Y , que e, portanto, enumeravel.

6. Um modo de construir tal sequencia e ordenar os pares ordenados de


numeros naturais de acordo com a soma das coordenadas: primeiro, os
que tem soma 2, depois 3, e assim por diante, dando origem a sequencia
{(1, 1), (1, 2), (2, 1), (1, 3), (2, 2), (3, 1), . . .}, o que mostra que N N e
enumeravel. Cada par (m, n) corresponde aos numeros racionais m n
e mn
. Logo, a partir dessa sequencia podemos construir uma outra
sequencia que inclui todos os numeros racionais, o que mostra que o
conjunto dos racionais tambem e enumeravel.

7. Cada subconjunto X de N corresponde a uma sequencia (xn ) em que


xn = 1 se n X e xn = 0 caso contrario. Como o conjunto de tais
sequencias e nao enumeravel, o conjunto de todos os subconjuntos de
N tambem e nao enumeravel.

2
3
O Princpio de Induo
Matemtica

Sumrio
3.1 Introduo . . . . . . . . . . . . . . . . . . . . . . . 2

3.2 O Poder do Mtodo de Induo . . . . . . . . . . . 2

3.3 Exerccios Recomendados . . . . . . . . . . . . . . . 12

3.4 Exerccios Suplementares . . . . . . . . . . . . . . . 13

3.5 Textos Complementares . . . . . . . . . . . . . . . . 14

1
Unidade 3 Introduo

3.1 Introduo

Nesta unidade e na prxima, mostraremos como utilizar o Axioma de Induo

para denir com rigor objetos matemticos e tambm como utiliz-lo como

poderoso instrumento para demonstrar os mais variados resultados envolvendo

nmeros naturais. Algumas das noes introduzidas nesta e na prxima unidade

sero retomadas de modo mais sistemtico nas Unidades 5 a 8.

3.2 O Poder do Mtodo de Induo

Comecemos com a pergunta:

O que signicam expresses do tipo 1 + 2 + + n e 1 2 n?


Note que as operaes de adio e de multiplicao nos nmeros naturais (ou em

qualquer sistema numrico) so binrias, isto , elas relacionam dois elementos

de cada vez. Apesar disso, temos uma ideia bastante intuitiva do signicado

das expresses acima, at mesmo no que diz respeito aos pontinhos que nelas

aparecem. Existe, contudo, um modo de tornar mais rigorosas denies desse

tipo por meio do Princpio de Induo Matemtica, como veremos mais adiante.

Antes, porm, recordemos este princpio que demonstramos na Unidade 1.

Princpio de Induo Matemtica Se P (n) uma propriedade relativa ao


nmero natural n, tal que

i) P (1) vlida;

ii) Para todo n N, a validez de P (n) implica a validez de P (n + 1).

Ento P (n) vlida qualquer que seja o nmero natural n.

+ Para Saber Mais - Comentrio - Clique para ler

+ Para Saber Mais - Induo Emprica vs Induo Matemtica - Cli-

que para ler

2
O Princpio de Induo Matemtica Unidade 3

Para denir uma expresso En , para todo nmero natural n, basta denirmos
E1 e mostrar, para todo n N, como obter sem ambiguidade En+1 a partir de
En .
Nesse caso, dizemos que En foi denido por recorrncia .

Vejamos como intervm o Princpio de Induo Matemtica para justicar

este tipo de denio. Seja X o subconjunto de N, determinado pela condio:

n X En est denido .

X , temos
Pela caracterizao do conjunto que 1 X e, para todo n N,
n X n + 1 X . Portanto, X = N.

Denies por recorrncia podem ser utilizadas para dar um signicado a

expresses como no incio da unidade.

Denimos S1 = 1. Em seguida, supondo Sn denido, pomos Exemplo 1

Sn+1 = Sn + (n + 1).

Damos assim, um sentido matemtico preciso expresso:

Sn = 1 + 2 + + n.

Por outro lado, denindo 1! = 1 e pondo (n + 1)! = n!(n + 1), supondo que
n! esteja denido, damos tambm, neste caso, um sentido matemtico para a

expresso:

n! = 1 2 n.

Para generalizar os exemplos acima, vamos introduzir a noo de sequncia.

Teremos oportunidade de comprovar, ao longo do curso, o quanto central este

conceito.

Uma sequncia de elementos de um conjunto A uma funo x : N A. Definio 1

3
Unidade 3 O Poder do Mtodo de Induo

Tendo em vista que uma funo conhecida quando se sabe qual a ima-

gem de todo elemento de seu domnio, uma sequncia x: N A pode ser

representada como

x(1), x(2), . . . , x(n), . . . ,


ou ainda, denotando x(n) por xn , podemos represent-la concisamente por

(xn ).
Por motivo de economia, quando dissermos que um conjunto A possui uma
adio ou uma multiplicao satisfazendo s leis bsicas da aritmtica, esta-

remos supondo que em A esto denidas duas operaes com propriedades

semelhantes s correspondentes operaes nos naturais.

Seja agora (xn ) uma sequncia de elementos de um conjunto A que possui

duas operaes, de adio e de multiplicao, satisfazendo s leis bsicas da

aritmtica.

Definio 2 Denem-se Sn e Pn em A, como se segue: S1 = P1 = x1 e

Sn+1 = Sn + xn+1 e Pn+1 = Pn xn+1 .

Isto d sentido s seguintes expresses:

S n = x 1 + x2 + + xn e P n = x1 x2 xn .

Somas e produtos, como acima, sero tambm escritos com as notaes de

somatrios e produtrios:
n
X n
Y
Sn = xi e Pn = xi ,
i=1 i=1

que se leem como somatrio quando i varia de 1 at n de xi  e produto

quando i varia de 1 at n de xi , respectivamente.

Note que a partir de uma sequncia dada (xn ), pudemos denir de modo
natural duas outras sequncias, a saber: (Sn ) e (Pn ).

Dada uma sequncia constante, x(n) = a, para todo n N, onde a A,

os termos da sequncia Pn a ela associada so por denio as potncias de a.

Pela sua importncia, destacamos essa denio a seguir.

4
O Princpio de Induo Matemtica Unidade 3

Seja a um elemento de um conjunto A munido de uma multiplicao sujeita Definio 3


n
s leis bsicas da aritmtica. As potncias a de a, com n N, so denidas
1 n+1
por recorrncia como segue: a = a e a = an a.

Quando a 6= 0, convenciona-se denir a0 = 1. Isto ser especialmente con-

veniente quando estendermos as potncias para expoentes no necessariamente

nmeros naturais. Isto se tornar bem mais claro na Unidade 13 de MA11.

Neste exemplo, queremos determinar uma frmula para a soma dos n pri- Exemplo 2
meiros nmeros naturais: Sn = 1 + 2 + + n.
Conta-se a seguinte histria sobre o matemtico alemo Carl Friedrich Gauss

(1777-1855), considerado um dos maiores gnios da matemtica de todos os

tempos, quando ainda garoto. Na escola, o professor, para aquietar a turma de

Gauss, mandou os alunos calcularem a soma de todos os nmeros naturais de

1 a 100. Qual no foi a sua surpresa quando, instantes depois, o menino deu

a resposta: 5050. Indagado sobre como tinha descoberto to rapidamente o

resultado, Gauss, ento, descreveu o mtodo a seguir.

Sendo

Sn = 1 + 2 + + n,

o objetivo encontrar uma frmula fechada1 para Sn .


Somando a igualdade acima, membro a membro, com ela mesma, porm

com as parcelas do segundo membro em ordem invertida, temos que

Sn = 1 + 2 + + n
Sn = n + (n 1) + + 1

2Sn = (n + 1) + (n + 1) + + (n + 1)

Da segue-se que 2Sn = n(n + 1) e, portanto,

n(n + 1)
Sn = .
2
1 Uma frmula fechada para Sn , a grosso modo, uma funo de n que permite calcular

diretamente os valores de Sn fazendo um pequeno nmero de clculos.

5
Unidade 3 O Poder do Mtodo de Induo

Vamos ser crticos com relao prova acima. Para a maioria das pessoas,

essa prova parece impecvel, mas se algum nos perguntasse o que est escon-

dido atrs dos pontinhos, talvez nos sentssemos embaraados. Tambm, como

ter absoluta certeza de que nada acontece fora do nosso controle, exatamente

na imensa regio coberta pelos pontinhos?

Para no pairar nenhuma dvida sobre o nosso resultado, vamos provar a

frmula utilizando o Princpio de Induo Matemtica.

Considere a sentena sobre os naturais:

n(n + 1)
P (n) : 1 + 2 + + n = . (3.1)
2
Note que
1(1 + 1)
P (1) : 1=
2
verdadeira.

Observe tambm que

(n + 1)(n + 2)
P (n + 1) : 1 + 2 + + n + (n + 1) = .
2
Agora, suponhamos que para algum n N, tenhamos P (n) verdadeira, isto
, a frmula (1.1) vlida para tal valor de n. Somando n + 1 a ambos os lados

dessa igualdade, temos que verdadeira a igualdade

n(n + 1)
1 + 2 + + n + (n + 1) = +n+1=
2
n(n + 1) + 2(n + 1) (n + 1)(n + 2)
= ,
2 2
o que estabelece a veracidade de P (n + 1).

Pelo Princpio de Induo, tem-se que a frmula P (n) verdadeira para

todo n N.

+ Na Sala de Aula - Consideraes sobre o Rigor - Clique para ler

Exemplo 3 Queremos validar a frmula

n(n + 1)(2n + 1)
P (n) : 12 + 22 + + n2 = . (3.2)
6

6
O Princpio de Induo Matemtica Unidade 3

Note que
1(1 + 1)(2 + 1)
P (1) : 12 =
6
verdadeira.

Suponha que, para algum n N, se tenha que P (n) verdadeira, isto ,


(1.2) vlida. Somando (n + 1)2 a ambos os lados da igualdade (1.2), temos
que

n(n + 1)(2n + 1)
12 + 22 + + n2 + (n + 1)2 = + (n + 1)2 =
6
n(n + 1)(2n + 1) + 6(n + 1)2 (n + 1)[n(2n + 1) + 6(n + 1)]
= =
6 6
(n + 1)[(n + 1) + 1][2(n + 1) + 1]
,
6
estabelecendo assim a veracidade de P (n + 1).

Portanto, a frmula vlida para todo n N.

Vamos provar que verdadeira, para todo n N, a frmula: Exemplo 4


1 1 1 n
P (n) : + + + = . (3.3)
1.2 2.3 n(n + 1) n+1

Observemos inicialmente que

1 1
P (1) : =
1.2 1+1
verdadeira.

Suponhamos que, para algum n, tem-se que P (n) verdadeira, ou seja,


que a frmula (1.3) seja verdadeira para esse valor de n. Somando a ambos os
1
lados dessa igualdade , temos que
(n + 1)(n + 2)

1 1 1 1
+ + + + =
1.2 2.3 n(n + 1) (n + 1)(n + 2)

n 1 n+1
+ = ,
n + 1 (n + 1)(n + 2) n+2
mostrando, assim, que P (n + 1) verdadeira.

7
Unidade 3 O Poder do Mtodo de Induo

Portanto, pelo Princpio de Induo Matemtica, temos que a frmula vale

para todo n N.

A seguir, vamos estabelecer, por meio de induo, as propriedades usuais

das potncias.

Proposio 4 Sejam a, b A e m, n N. Ento,

i) am an = an+m .

ii) (am )n = amn .

iii) (a b)n = an bn .

Demonstrao Provaremos (i), deixando o restante como exerccio.

Fixemos a A e m N, arbitrariamente. Demonstremos a propriedade

por induo sobre n.

Para n = 1, a propriedade vlida, pois, pelas denies,

am a1 = am a = am+1 .

Por outro lado, supondo que am an = am+n , temos que

am an+1 = am (an a) = (am an ) a = am+n a = am+n+1 .

Isso, pelo Princpio de Induo Matemtica, prova a nossa propriedade.

Exemplo 5 Utilizando a noo de potncia e de suas propriedades, mostraremos que 3


n n
divide 5 + 2 11 n N.
nos inteiros, para todo
1 1
De fato, para n = 1, temos que 3 divide 5 + 2 11 = 27.
n n
Suponha, agora, que, para algum n 1, saibamos que 3 divide 5 + 2 11 .

Logo, existe um nmero inteiro a tal que

5n + 2 11n = 3a.

Mutiplicando por 5 ambos os lados da igualdade acima, temos

5 3a = 5n+1 + 5 2 11n = 5n+1 + 2 11 11n 12 11n .

8
O Princpio de Induo Matemtica Unidade 3

Da segue-se a igualdade

5n+1 + 2 11n+1 = 5 3a + 12 11n ,

cujo segundo membro divisvel por 3, por ser igual a 3(5a + 4 11n ).
Assim, provamos que 3 divide5n+1 + 2 11n+1 , o que, pelo Princpio de
n n
Induo Matemtica, acarreta que 3 divide 5 + 2 11 , para todo nmero

natural n.

Pode ocorrer que uma determinada propriedade seja vlida para todos os

nmeros naturais a partir de um determinado valor a, mas no necessariamente


para valores menores. Como proceder nesses casos? Por exemplo, como provar

que a desigualdade 2n > n2 verdadeira para todo valor de n natural maior ou

igual do que 5? Fazemos isso baseados na seguinte pequena generalizao do

Princpio de Induo Matemtica:

Seja P (n) uma sentena sobre N, e seja a N. Suponha que: Teorema 1


(i) P (a) verdadeira, e

(ii) qualquer que seja n N, com n a, sempre que P (n) verdadeira,

segue-se que P (n + 1) verdadeira.

Ento, P (n) verdadeira para todo nmero natural n a.

Dena o conjunto
Demonstrao

S = {m N; P (m + a 1) }.

Por (i) temos que 1 S. m S , temos que P (m + a


Por outro lado, se

1) verdadeira. Logo, por (ii), P (m + 1 + a 1) verdadeira. Portanto,


m + 1 S . Em vista do Princpio de Induo Matemtica, temos que S = N.
Consequentemente, P (n) verdadeira para todo n a.

Vamos mostrar que a desigualdade na sentena P (n) : 2n > n2 verdadeira, Exemplo 6


para todo nmero natural n 5.

9
Unidade 3 O Poder do Mtodo de Induo

Note que P (1) : 21 > 12 verdadeira, P (2) : 22 > 22 falsa, P (3) : 23 > 32
4 2
falsa e P (4) : 2 > 4 falsa. Tudo isso no importa, pois queremos vericar

a veracidade dessa desigualdade para n 5.


5 2
De fato, temos que P (5) : 2 > 5 verdadeira. Seja n 5 tal que

2n > n2 . Multiplicando ambos os lados da desigualdade acima por 2, obtemos


2n+1 > 2n2 . Note que 2n2 > (n + 1)2 , se n 3, pois tal desigualdade
n+1
equivalente a n(n 2) > 1. Da, deduzimos que 2 > (n + 1)2 , o que
signica que P (n + 1) verdadeira, estabelecendo o resultado em vista do

Teorema 1.

Exemplo 7 Um banco tem um suprimento ilimitado de notas de 3 e de 5 (unidades de

moeda). Mostre que ele pode pagar qualquer quantia (de unidades de moeda)

maior do que 7.
Para isto, basta mostrar que a sentena:

P (n) : A equao 3x + 5y = n tem soluo em (N {0}) (N {0}),


verdadeira para todo n 8.

De fato, ela verdadeira para n = 8, pois a equao 3x + 5y = 8 admite a

soluo (x, y) = (1, 1).


Suponha agora que a equao 3x + 5y = n tenha uma soluo (a, b) para
algum n 8; isto , 3a + 5b = n. Note que, para qualquer soluo (a, b),

devemos ter a 1 ou b 1.

Se b 1, observando que 3 2 5 1 = 1, segue que

3(a + 2) + 5(b 1) = 3a + 5b + 3 2 5 1 = 3a + 5b + 1 = n + 1,

o que mostra que a equao 3x + 5y = n + 1 admite a soluo (a + 2, b 1)


em (N {0}) (N {0}).
Se, por acaso, b = 0, ento, a 3; usando a igualdade 3 3 + 5 2 = 1,
temos

3(a 3) + 5 2 = 3a 3 3 + 5 2 = 3a + 5b + 1 = n + 1,

o que mostra que a equao 3x + 5y = n + 1 admite a soluo (a 3, b + 2)


em (N {0}) (N {0}).

10
O Princpio de Induo Matemtica Unidade 3

Mostramos assim, que, em qualquer caso, a equao 3x+5y = n+1 admite


soluo, sempre que a equao 3x + 5y = n, para algum n 8, tenha soluo.

Como o resultado vale para n = 8, segue a concluso desejada pelo Teorema 1.

Note que n0 = 8 o menor valor de n para o qual a equao tem soluo

para todo n n0 .

11
Unidade 3 Exerccios Recomendados

3.3 Exerccios Recomendados

1. Mostre, por induo, a validez das seguintes frmulas:

(a) 1.20 + 2.21 + 3.22 + + n.2n1 = 1 + (n 1)2n ;


2 n1
nn1
  
1 1 1
(b) 1+ 1+ 1 + = ,
1 2 n1 (n 1)!
(c) 1.1! + 2.2! + 3.3! + + n.n! = (n + 1)! 1.

2. Sejam a e b nmeros reais distintos. Mostre que, para todo n N, vale

a igualdade:

bn+1 an+1
bn + abn1 + a2 bn2 + + an1 b + an = .
ba
3. Se sen 6= 0, mostre que, para todo n N, vale a igualdade:

sen 2n+1
cos cos 2 cos 22 cos 2n = .
2n+1 sen
Sugesto: Use a frmula sen 2 = 2 sen cos .

4. Para todo n N, mostre que, nos inteiros,

(a) 80 divide 34n 1;

(b) 9 divide 4n + 6n 1;

(c) 8 divide 32n + 7;

(d) 9 divide n4n+1 (n + 1)4n + 1.

5. Mostre que

(a) n! > 2n , se n 4;

(b) n! > 3n ,se n 7;

(c) n! > 4n , se n 9.

6. Prove que, para todo n natural, vale a desigualdade:

1 3 5 2n 1 1
.
2 4 6 2n 3n + 1

12
O Princpio de Induo Matemtica Unidade 3

7. Mostre que o nmero de diagonais de um polgono convexo de n lados

dado por
n(n 3)
dn = .
2

3.4 Exerccios Suplementares

1. Mostre que n0 = 32 o menor valor para o qual a equao 5x + 9y = n


2
possui soluo em (N {0}) para todo n n0 .

2. Prove que, para qualquer nmero natural n:

a) n3 + (n + 1)3 + (n + 2)3 divisvel por 9;


b) 32n+2 + 8n 9 divisvel por 16;
c) 4n + 15n 1 divisvel por 9;
d) 11n+2 + 122n+1 divisvel por 133;
n
e) 23 + 1 divisvel por 3n+1 .

3. Prove que:

a) 2n > n, onde n um nmero natural arbitrrio;


1 3 5 (2n 1) 1
b) , para qualquer n N;
2 4 6 2n 2n + 1
1 1 1 13
c) + + + > , se n N e n 2.;
n+1 n+2 2n 24

d) 2n > 1 + n 2n1 , se n N e n 2.

4. Suponha que x+ x1 seja um nmero natural. Prove que xn + x1n tambm

um nmero natural, qualquer que seja o nmero natural n.

5. Mostre que o nmero 111 . . . 1 (formado por 3n algarismos iguais a 1)


n
divisvel por 3 .

Sugesto: Para o passo indutivo, divida o nmero escrito com 3n+1


algarismos iguais a 1 pelo nmero formado por 3n algarismos iguais a 1

e verique que o resultado um nmero divisvel por 3.

13
Unidade 3 Textos Complementares

3.5 Textos Complementares

Para Saber Mais Comentrio

Note que, na argumentao acima, poderia parecer que estamos usando

o fato de P (n) ser verdadeira para deduzir que P (n + 1) verdadeira para

em seguida concluir que P (n) verdadeira. O que est ocorrendo? Estamos

usando a tese para provar o resultado?

A resposta no! Preste bem ateno, pois essa a parte mais delicada de

toda a trama.

Dado um nmero natural n, temos duas possibilidades:

(a) P (n) verdadeira, ou (b) P (n) falsa.

A hiptese (ii) do Princpio no exige em absoluto que assumamos P (n)


verdadeira para todo n N, podendo eventualmente ser falsa para algum valor

de n, ou mesmo para todos os valores de n. O que a hiptese (ii) exige

que sempre que algum n pertena categoria (a), acima, ento n + 1 tambm
pertena a essa mesma categoria; no exigindo nada quando n pertencer
categoria (b).

Por exemplo, a sentenaP (n) : n = n+1 satisfaz (por vacuidade) a hiptese


(ii) do Princpio, j que nenhum n N pertence categoria (a). O que falha

para que o Princpio de Induo nos garanta que P (n) verdadeira para todo

n que a hiptese (i) no vericada, pois P (1) : 1 = 2 falsa!

14
O Princpio de Induo Matemtica Unidade 3

Induo Emprica vs Induo Matemtica Para Saber Mais

preciso ter clareza que a Induo Matemtica diferente da induo

emprica das cincias naturais, em que comum, aps um certo nmero de

experimentos, necessariamente nito, enunciar leis gerais que governam o fen-

meno em estudo. Essas leis so tidas como verdades, at prova em contrrio.

Na matemtica, no h lugar para armaes verdadeiras at prova em con-

trrio. A Prova por Induo Matemtica trata de estabelecer que determinada

sentena sobre os naturais sempre verdadeira .


A induo emprica foi batizada, de modo irnico, pelo matemtico, lsofo

e grande humanista ingls do sculo passado, Bertrand Russel (1872-1970), de

induo galincea, com base no seguinte conto:

Havia uma galinha nova no quintal de uma velha senhora. Diariamente,


ao entardecer, a boa senhora levava milho s galinhas. No primeiro dia, a
galinha, desconada, esperou que a senhora se retirasse para se alimentar. No
segundo dia, a galinha, prudentemente, foi se alimentando enquanto a senhora
se retirava. No nonagsimo dia, a galinha, cheia de intimidade, j no fazia
caso da velha senhora. No centsimo dia, ao se aproximar a senhora, a galinha,
por induo, foi ao encontro dela para reclamar o seu milho. Qual no foi a
sua surpresa quando a senhora pegou-a pelo pescoo com a inteno de p-la
na panela.

15
Unidade 3 Textos Complementares

Na Sala de Aula Consideraes sobre o Rigor


Neste curso, o nosso objetivo mostrar como se pode estabelecer um maior

padro de rigor no tratamento de problemas matemticos que ocorrem no En-

sino Mdio, mas isso no deve ser tomado ao p da letra na sua prtica docente.

Certos argumentos informais, quando acompanhados de um raciocnio correto,

so corriqueiramente aceitos. Por exemplo, o argumento utilizado por Gauss

para somar os n primeiros nmeros naturais perfeitamente aceitvel. Por-

tanto, um conselho: use o formalismo para ajudar e no para atrapalhar e

nunca o deixe se sobrepor criatividade, pois, via de regra, primeiro vem a

descoberta para depois vir a formalizao. Procure estimular sempre os seus

alunos a serem criativos. Num primeiro momento, deixe as ideias uirem, s

depois preocupe-se com a sua organizao e formalizao.

16
MA12 - Unidade 3
O Metodo da Inducao

Paulo Cezar Pinto Carvalho

PROFMAT - SBM

31 de Janeiro de 2014
Definicoes por inducao ou recorrencia

Como definir, apropriadamente, n! = 1 2 . . . n?


i) Definimos 1! = 1
ii) A seguir, supondo n! definido, fazemos
(n + 1)! = n! (n + 1).
Note que
i) garante que 1! esta bem definido.
ii) garante que, se n! esta bem definido, (n + 1)! tambem esta.
Logo, pelo Princpio da Inducao Finita, n! esta bem definido
para todo n natural.

PROFMAT - SBM MA12 - Unidade 3, O Metodo da Inducao slide 2/14


Somatorios e Produtorios
Seja (xn ) uma sequencia de elementos de um conjunto A
dotado de operacoes de adicao e multiplicacao.
O somatorio
n
X
Sn = xi = x1 + x2 + + xn
i=1

e o produtorio
n
Y
Pn = xi = x1 x2 . . . xn
i=1

podem ser definidos como se segue:


S1 = P1 = x1
Sn+1 = Sn + xn+1 , para todo n N
Pn+1 = Pn xn+1 , para todo n N

PROFMAT - SBM MA12 - Unidade 3, O Metodo da Inducao slide 3/14


Demonstrando igualdades

Obter uma expressao para Sn = 1 + 3 + . . . + (2n 1).


S1 = 1
S2 = 4
S3 = 9
...
Conjectura: Sn = n2 , para todo n N.

PROFMAT - SBM MA12 - Unidade 3, O Metodo da Inducao slide 4/14


A prova por inducao

Seja P(n) : 1 + 3 + + (2n 1) = n2 .


i) P(1) : 1 = 12 e verdadeira.
ii) Suponhamos que para algum n N, tenhamos P(n)
verdadeira.
Somando 2(n + 1) 1 = 2n + 1 a ambos os lados dessa
igualdade, temos:

1 + 3 + + (2n 1) + (2n + 1) = n2 + 2n + 1 = (n + 1)2

o que mostra que P(n + 1) tambem e verdadeira.


Pelo Princpio de Inducao, tem-se que a formula P(n) e
verdadeira para todo n N.

PROFMAT - SBM MA12 - Unidade 3, O Metodo da Inducao slide 5/14


Demonstrando desigualdades

Demonstrar a desigualdade de Bernoulli: (1 + h)n 1 + nh,


para todo n natural e todo h > 1
i) Como (1 + h)1 e 1 + 1.h sao ambos iguais a 1 + h, P(1) e
verdadeira.
ii) Suponhamos que P(n), para algum n , seja verdadeira, ou
seja, (1 + h)n 1 + nh.
Multiplicando ambos os lados por (1+h):
(1 + h)n+1 (1 + nh)(1 + h) = 1 + (n + 1)h + nh2 .
Mas 1 + (n + 1)h + nh2 1 + (n + 1)h.
Logo, (1 + h)n+1 1 + (n + 1)h, o que mostra que P(n + 1) e
verdadeira.
Portanto, pelo Princpio da Inducao, P(n) vale para todo n
natural.

PROFMAT - SBM MA12 - Unidade 3, O Metodo da Inducao slide 6/14


Aplicacoes em aritmetica

Mostrar que, para todo n N, 4n + 6n 1 e divisvel por 9.


i) Como 41 + 6.1 1 = 9, a propriedade vale para n = 1.
ii) Suponha, agora, que, para algum n 1, saibamos que
4n + 6n 1 e divisvel por 9. Logo, 4n + 6n 1 = 9k, ou seja,
4n = 9k 6n + 1, para algum inteiro k.
Mutiplicando por 4 ambos os lados:
4n+1 = 9k 24n + 4.
Logo
4n+1 + 6(n + 1) 1 = 9k 24n + 4 + 6(n + 1) 1 =
9k 18n + 9 = 9(k 2n + 1).
Portanto, 4n+1 + 6(n + 1) 1 e divisvel por 9.
Logo, pelo Princpio da Inducao , 4n + 6n 1 e divisvel por 9
para todo numero natural n.

PROFMAT - SBM MA12 - Unidade 3, O Metodo da Inducao slide 7/14


A Torre de Hanoi


 
   


Transferir a pilha de discos para uma outra haste, deslocando


um disco de cada vez, de modo que, a cada passo, um disco
nunca esteja colocado sobre um disco menor.

1 O jogo tem solucao para cada n N?


2 Em caso afirmativo, qual e o numero mnimo jn de
movimentos para resolver o problema com n discos?
PROFMAT - SBM MA12 - Unidade 3, O Metodo da Inducao slide 8/14
Torre de Hanoi: o jogo sempre tem solucao!

Obviamente, o jogo tem solucao para n = 1.


Suponhamos que o jogo tenha solucao para n discos e vamos
mostrar que, da, decorre que o jogo tambem tem solucao
para n + 1 discos.
Primeiro, transferimos os n discos superiores para uma das
outras hastes (isto e possvel, pela hipotese de inducao).

 
 
    
   

PROFMAT - SBM MA12 - Unidade 3, O Metodo da Inducao slide 9/14


A seguir, transferimos o disco inferior para a outra haste.


 
     


Finalmente, transferimos os demais n discos para a haste em


que colocamos o disco maior (e possvel, pela hipotese de
inducao e pelo fato de o disco inferior ser maior que todos os
outros)


 
 
   
 

Pelo Princpio da Inducao, conclumos que o jogo tem solucao


para todo n N.
Torre de Hanoi: Qual e o numero mnimo de
movimentos?

Executar a tarefa para n + 1 discos necessariamente envolve


retirar os n discos superiores, colocando-os em outra haste e,
depois de mover o disco inferior, recoloca-los sobre ele.
O numero mnimo jn de movimentos e, portanto, tal que
j1 = 1
jn+1 = jn + 1 + jn = 2jn + 1, para todo n N.
E facil mostrar, por inducao, que jn = 2n 1.
(Na Unidade 8, aprenderemos a encontrar a expressao para o
termo geral de sequencias definidas por recorrencias como
esta.)

PROFMAT - SBM MA12 - Unidade 3, O Metodo da Inducao slide 11/14


A Pizza de Steiner
Qual e o maior numero de partes em que se pode dividir o
plano com n cortes retos?
Numero de cortes Numero maximo de partes
1 2
2 4
3 7
... ...
n1 pn1
n pn
n+1 pn+1
O padrao observado acima sugere que o numero maximo de
pedacos obtidos com n cortes e:
n(n + 1)
pn = 2 + 2 + 3 + . . . + n = +1
2
.
PROFMAT - SBM MA12 - Unidade 3, O Metodo da Inducao slide 12/14
A Pizza de Steiner: prova por inducao

Com apenas um corte obtemos dois pedacos. Portanto, a


formula esta correta para n = 1, pois p1 = 1(1+1)
2 + 1 = 2.
Admitamos que, para algum valor de n, a formula para pn
esteja correta. Vamos mostrar que a formula para pn+1
tambem esta correta.
O ponto crucial e mostrar que sao acrescentados n + 1
pedacos no (n + 1)-esimo corte.

PROFMAT - SBM MA12 - Unidade 3, O Metodo da Inducao slide 13/14


Para que o (n + 1)-esimo corte produza o numero maximo de
pedacos, ele deve encontrar cada um dos n cortes anteriores
em pontos que nao sao de intersecao de dois cortes. Neste
caso, como n pontos subdividem uma reta em n + 1 partes, ele
subdivide n + 1 regioes, criando assim, n + 1 novos pedacos.

Logo,

n(n + 1) (n + 1)(n + 2)
pn+1 = pn + n + 1 = +1+n+1 = +1
2 2
o que mostra que a formula esta correta para n + 1.
Pelo Princpio da Inducao, a formula esta correta para todo
n N.

PROFMAT - SBM MA12 - Unidade 3, O Metodo da Inducao slide 14/14


Lista de Exerccios
Unidade 3

1. Demonstre, por inducao, as seguintes identidades:

n(n + 1)(n + 2)
a) 1.2 + 2.3 + . . . + n.(n + 1) = ;
3
 2
3 3 3 n(n + 1)
b) 1 + 2 + . . . + n = ;
2
c) 1.20 + 2.21 + 3.22 + + n.2n1 = 1 + (n 1)2n ;
2 n1
nn1
  
1 1 1
d) 1 + 1+ 1 + = ;
1 2 n1 (n 1)!

e) 1.1! + 2.2! + 3.3! + + n.n! = (n + 1)! 1.

2. Demonstre, por inducao, as seguintes desigualdades:

a) 2n > n, onde n e um numero natural arbitrario;


1 3 5 (2n 1) 1
b) , para qualquer n N.
2 4 6 2n 3n + 1

3. Considere a sequencia (xn ) correspondente ao metodo de Newton para



calcular 2, ou seja, a sequencia definida por x1 = 1, xn+1 = 2 xn + x2n .
1

a) Mostre que 1 xn 32 , para todo n.


2
b) Mostre que xn+1 2 = 2x1n xn 2 , para todo n.

(Isto explica porque o erro no calculo de 2 cai tao rapidamente no
Metodo de Newton.)

4. Prove que, para qualquer numero natural n:

a) n3 + (n + 1)3 + (n + 2)3 e divisvel por 9;


b) 32n+2 + 8n 9 e divisvel por 16;

1
c) 4n + 15n 1 e divisvel por 9;
d) 11n+2 + 122n+1 e divisvel por 133;
n
e) 23 + 1 e divisvel por 3n+1 .

5. Um plano esta dividido em regioes por varias retas. Prove que e possvel
colorir essas regioes com duas cores de modo que quaisquer duas regioes
adjacentes tenham cores diferentes (dizemos que duas regioes sao ad-
jacentes se elas tiverem pelo menos um segmento de reta em comum).

6. (O queijo de Steiner) Seja qn o numero de regioes determinadas no


espaco tridimensional por n planos (equivalentemente, o maior numero
de partes em que um queijo pode ser dividido por n cortes planos).

a) Explique por que qn+1 = qn + pn , para todo n N, onde pn e o


numero maximo de regioes em que n retas dividem o plano.
n3 + 5n + 6
b) Mostre que qn = , para todo n N
6
7. Considere uma linha poligonal formada por 2 semiretas e por n seg-
mentos de reta. A figura ilustra a situacao para n = 2. Encontre uma
formula para o numero maximo de regioes determinadas pela linha po-
ligonal e demonstre que sua formula esta correta.

8. No problema da Torre de Hanoi, suponha que se deseja passar n discos


de uma haste extrema para outra, mas que nao seja permitido pas-
sar diretamente um disco de um extremo para o outro (isto e, todo
movimento deve ter origem ou destino na haste central). Assim, por
exemplo, para passar um unico disco sao necessarios dois movimentos
(o primeiro para leva-lo a haste central e o segundo para leva-lo da
haste central ao outro extremo).

a) Verifique que sao necessarios no mnimo 8 movimentos para trans-


ferir 2 discos.

2
b) Sendo hn o numero de movimentos necessarios para n discos, ex-
presse hn+1 em termos de hn .
c) Moste que o numero mnimo de movimentos para transferir n
discos e hn = 3n 1, para todo n N.

3
Solucoes da Lista de Exerccios

Observacao: as solucoes dos exerccios abaixo ficaram faltando na Unidade 2

3. Para n = 1, basta de fato uma pesagem, feita com dois dos objetos: se
ela indicar um objeto mais pesado do que o outro, ele e o procurado; se
os objetos tiverem pesos iguais, o objeto que ficou de fora na pesagem
e o mais pesado. Suponhamos agora que seja possvel determinar qual
e mais pesado dentre 3n objetos com n pesagens e consideremos um
conjunto com 3n+1 objetos. Dividimos estes objetos em tres grupos
com 3n objetos cada e comparamos o peso de dois deles. Se um deles
for mais pesado, o objeto procurado esta nele; senao, esta no grupo
que ficou de fora da pesagem. De qualquer modo, pela hipotese de
inducao, ele pode ser encontrado em n pesagens adicionais, para um
total de n+1 pesagens. Logo, a propriedade vale para conjuntos de 3n+1
objetos e, pelo Princpio da Inducao, para conjuntos com 3n objetos,
qualquer que seja n.

4. A propriedade vale para um conjunto com um unico elemento a1 : seus


dois unicos subconjuntos sao e {a1 } e e possvel passar do primeiro
ao segundo acrescentando-se a1 . Suponhamos que a propriedade seja
valida para conjuntos com n elementos e consideremos o conjunto com
n + 1 elementos X = {a1 , a2 , . . . , an , an+1 }. Consideremos a lista L
dos subconjuntos de {a1 , a2 , . . . , an , an+1 } satisfazendo as condicoes do
enunciado (ela existe, pela hipotese de inducao) e formemos uma lista
de subconjuntos de X do seguinte modo: comecamos com L e acrescen-
tamos a lista L0 que consiste dos subconjuntos de L em ordem reversa,
acrescentando-se an+1 a cada um deles. A nova lista e formada por to-
dos os subconjuntos de X (ela lista primeiro todos os subconjuntos que
nao contem an+1 e, a seguir, todos que o contem). Alem disso, sempre e
possvel passar de um subconjunto ao proximo da lista acrescentando-
se ou retirando-se um elemento. De fato, pela hipotese de inducao isto
ocorrem em L; a passagem do ultimo subonjunto de L para o primeiro
de L0 ocorre pela adicao de an+1 ; finalmente, a passagem de cada sub-
conjunto de L0 para o proximo se da de forma inversa a ocorrida em L.
Assim, a propriedade vale tambem para conjuntos com n+1 elementos.
Logo, por inducao, vale para qualquer conjunto finito.

1
Unidade 3

1.2.3
1. a) Como 1.2 = , a igualdade vale para n = 1. Suponhamos,
3
agora, que ela seja valida para algum n N, ou seja, 1.2 + 2.3 +
n(n + 1)(n + 2)
. . .+n.(n+1) = . Somando (n+1)(n+2) a ambos
3
os membros da igualdade, obtemos
n(n + 1)(n + 2)
1.2 + 2.3 + . . . + n.(n + 1) + (n + 1)(n + 2) = +
3
(n + 1)(n + 2)(n + 3)
(n + 1)(n + 2) = ,
3
o que mostra que a igualdade tambem vale para n + 1. Logo, por
inducao, a igualdade vale para todo n N.
 2
3 1.2
b) Como 1 = , a igualdade vale para n = 1. Suponhamos,
2
3 3
agora, que ela seja valida2 para algum n N, ou seja, 1 + 2 +
n(n + 1)
. . . + n3 = . Somando (n + 1)3 a ambos os membros
2
da igualdade, obtemos
 2
n(n + 1)
13 + 23 + . . . + n3 + (n + 1)3 = + (n + 1)3 =
2
 2   2 
2 n 2 n + 4n + 4
(n + 1) + (n + 1) = (n + 1) =
4 4
 2
(n + 1)(n + 2)
,
2
o que mostra que a igualdade tambem vale para n + 1. Logo, por
inducao, a igualdade vale para todo n N.
c) Como 1.20 = 1 = 1 + 0.21 , a igualdade vale para n = 1. Su-
ponhamos, agora, que ela seja valida para algum n N, ou seja,
1.20 +2.21 +3.22 + +n.2n1 = 1+(n1)2n . Somando (n+1).2n
a ambos os membros da igualdade, obtemos:
1.20 + 2.21 + 3.22 + + n.2n1 + (n + 1).2n = 1 + (n 1)2n +
(n + 1).2n = 1 + 2n.2n = 1 + n.2n+1 ,

2
o que mostra que a igualdade tambem vale para n + 1. Logo, por
inducao, a igualdade vale para todo n N.
  2  n
1 1 1
d) Correcao: a igualdade deveria ser 1 + 1+ 1 + =
1 2 n
(n + 1)n
.
n! 1 1
Como 1 + 11 = 2 = 21! , a igualdade vale para n = 1. Supo-
nhamos,  agora, que
2 ela seja valida
n para algum n N, ou seja,
n
 
1 1 1 (n + 1)
1+ 1+ 1 + = . Multiplicando am-
1 2 n n!
1
n+1
bos os membros da igualdade por 1 + n+1 , obtemos:
  2  n  n+1
1 1 1 1
1+ 1+ 1 + 1+ =
1 2 n n+1
(n+1)n 1
n+1 (n+1)n n+2 n+1
n!
1 + n+1
= n! n+1
=
(n+2)n+1 n+1

n!(n+1)
= (n+2)
(n+1)!
,
o que mostra que a igualdade tambem vale para n + 1. Logo, por
inducao, a igualdade vale para todo n N.
e) Como 1.11 = 2! 1, a igualdade vale para n = 1. Suponhamos,
agora, que ela seja valida para algum n N, ou seja, 1.1! + 2.2! +
3.3! + + n.n! = (n + 1)! 1. Somando (n + 1).(n + 1)! a ambos
os membros da igualdade, obtemos:
1.1! + 2.2! + 3.3! + + n.n! + (n + 1).(n + 1)! = (n + 1)! 1 +
(n + 1).(n + 1)! = (n + 1)!(n + 2) 1 = (n + 2)! 1,
o que mostra que a igualdade tambem vale para n + 1. Logo, por
inducao, a igualdade vale para todo n N.

2. a) Como 21 > 1, a desigualdade vale para n = 1. Suponhamos,


agora, que ela seja valida para algum n N, ou seja, 2n > n.
Multiplicando ambos os membros da igualdade por 2, obtemos
2n+1 > 2n. Mas, para todo n natural, 2n n + 1 (ja que esta
desigualdade e equivalente a n 1). Portanto, 2n+1 > n + 1, o
que mostra que a desigualdade tambem vale para n + 1. Logo, por
inducao, a desigualdade vale para todo n N.
b) Como 12 = 3+11
, a desigualdade vale para n = 1. Suponhamos,
agora, que ela seja valida para algum n N, ou seja,

3
1 3 5 (2n 1) 1
. Multiplicando ambos os mem-
2 4 6 2n 3n + 1
2n + 1
bros da desigualdade por , obtemos:
2(n + 1)
1 3 5 (2n 1) (2n + 1) 1 2n + 1
.
2 4 6 2n 2(n + 1) 3n + 1 2(n + 1)
Para mostrar que a desigualdade tambem vale para n + 1, precisa-
2n + 1 1
mos mostrar que p ou, equi-
3n + 1 2(n + 1) 3(n + 1) + 1
(2n + 1)2 1
valentemente, 2
. Mas
(3n + 1)(2(n + 1)) 3n + 4
(2n + 1)2 1 (2n + 1)2 (3n + 4) (3n + 1)(2n + 2)2
= =
(3n + 1)(2(n + 1))2 3n + 4 (3n + 1)(2n + 2)2 (3n + 4)
n
< 0.
(3n + 1)(2n + 2)2 (3n + 4)
1 2n + 1 1
Logo, de fato temos < p e, por-
3n + 1 2(n + 1) 3(n + 1) + 1
1 3 5 (2n 1) (2n + 1) 1
tanto, <p , o que mos-
2 4 6 2n 2(n + 1) 3(n + 1) + 1
tra que a desigualdade tambem vale para n+1. Logo, por inducao,
a desigualdade vale para todo n N, sendo estrita para todo
n > 1.

3. a) Como x1 = 1, temos 1 x1 32 e, assim, a desigualdade vale


para n = 1.
Suponhamos que ela seja valida para  um certon N. Como
xn 32 , temos x2n 34 . Logo, n+1 = 12 xn + x2n 12 1 + 34 =

2
7
6
> 1. Por outro lado, xn+1 32 = xn 3x
2xn
n +2
= (xn 1)(x
2xn
n 2)
. Como
3
1 xn 2 , (xn 1) 0 e (xn 2) < 0, o que mostra que
xn+1 32 0, ou seja xn+1 23 . Logo, a desigualdade tambem
vale para n + 1. Por inducao, ela vale para todo n natural.
 
b) xn+1 2 = 2 xn + xn 2 = 2x1n (x2n + 2 2 2xn ) = 2x1n (xn
1 2
2
2) .

4. a) Como 13 + 23 + 33 = 36, que e divisvel por 9, a propriedade


vale para n = 1. Suponhamos que ela seja valida para algum

4
n N, ou seja, n3 + (n + 1)3 + (n + 2)3 = 9k, para algum inteiro
k. Da, (n + 1)3 + (n + 2)3 + (n + 3)3 = 9k + (n + 3)3 n3 =
9k +n3 +9n2 +27n+27n3 = 9(k +n2 +3n+3), o que mostra que
(n + 1)3 + (n + 2)3 + (n + 3)3 tambem e divisvel por 9. Portanto,
por inducao, a propriedade vale para todo n natural.
b) Como 34 + 8 9 = 80, que e divisvel por 16, a propriedade
vale para n = 1. Suponhamos que ela seja valida para algum
n N, ou seja, 32n+2 + 8n 9 = 16k, onde ou, equivalentemente,
32n+2 = 16k 8n + 9 para algum k N. Multiplicando por 9 os
dois lados da igualdade, obtemos 32(n+1)+2 = 144k 72n+81. Da,
32(n+1)+2 + 8(n + 1) + 9 = 144k 72n + 81 + 8(n + 1) 9 = 144k
64n+80 = 16(9k4n+5), o que mostra que 32(n+1)+2 +8(n+1)+9
e divisvel por 16 e, assim, que a propriedade tambem vale para
n + 1. Portanto, por inducao, a propriedade vale para todo n
natural.
c) Como 41 + 15.1 1 = 18, que e divisvel por 9, a propriedade vale
para n = 1. Suponhamos que ela seja valida para algum n N, ou
seja, 4n + 15n 1 = 9k, ou, equivalentemente, 4n = 9k 15n + 1
para algum k N. Multiplicando por 4 ambos os membros da
igualdade, obtemos 4n+1 = 36k60n+4, ou seja, 4n+1 15(n+1)
1 = 36k 60n+4+15(n+1)1 = 36k 45n+18 = 9(4k 5n+2),
o que mostra que 4n+1 15(n + 1) 1 e divisvel por 9, ou seja que
a propriedade tambem vale para n + 1. Portanto, por inducao, a
propriedade vale para todo n natural.
d) Neste caso, e mais conveniente comecar com n = 0. Como 112 +
121 = 133, a propriedade vale para n = 0. Suponhamos que ela
seja valida para algum n N, ou seja, 11n+2 + 122n+1 = 133k, ou,
equivalentemente, 122n+1 = 13k 11n+2 para algum k N. Multi-
plicando por 122 = 144 ambos os membros da igualdade, obtemos
122(n+1)+1 = 133.144k 144.11n+2 . Logo, 122(n+1)+1 11(n+1)+2 =
133.144k 144.11n+2 + 11.11n+2 = 133(144k 11n+2 ), o que mos-
tra que 122(n+1)+1 11(n+1)+2 e divisvel por 133 e, assim, que a
propriedade vale para n + 1. Portanto, por inducao, a propriedade
vale para todo n natural.
1
e) Como 23 + 1 = 9, que e divisvel por 31+1 = 9, a propriedade e
valida para n = 1. Suponhamos que ela seja valida para algum n
n n
N, ou seja, 23 +1 = k.3n+1 ou, equivalentemente, 23 = k.3n+1 1,

5
para algum k N. Elevando ao cubo ambos os membros da
3n 3 n+1

igualadade, obtemos 2 = (k.3n+1 1)3 e, da, 23 +1 =
3 n+1 3 2 n+1 2 n+1 3 3n+3 2 2n+3
k (3 ) 3k (3 ) + 3k.3 1+1 = k 3 k 3 +
n+2 n+2 3 2n+1 2 n+1 3n+1
k3 = 3 (k 3 k 3 + k). Logo, 2 + 1 e divisvel
por 3n+2 , ou seja, a propriedade vale para n + 1. Portanto, por
inducao, ela vale para todo n N.

5. Uma reta divide o plano em duas regioes adjacentes, que certamente


podem ser coloridas com duas cores. Portanto, a propriedade vale para
n = 1 reta. Suponhamos que ela valha para toda subdivisao formada
por n retas e incluamos uma reta adicional. Uma coloracao satisfazendo
as condicoes do enunciado pode ser obtida trocando a cor de todas as
regioes que ficam em um dos semiplanos determinados pela nova reta, o
que mostra que a propriedade tambem vale para n + 1 retas. Portanto,
por inducao, vale para subdivisoes geradas por qualquer quantidade de
retas.

6. a) Ao acrescentar-se o plano n+1, os planos ja existentes determinam


neste plano n retas de intersecao que, por sua vez, determinam
pn regioes planas. Cada uma destas regioes planas, divide em
duas uma das regioes determinadas anteriormente, criando assim
pn novas regioes do espaco. Portanto, o numero qn+1 de regioes
determinadas por n + 1 planos e dado por qn+1 = qn + pn .
b) Com n = 1 plano sao determinadas duas regioes; isto e, q1 = 2.
3
Como 1 +5.1+6 6
= 2, a formula esta correta para n = 1. Supo-
nhamos que ela esteja correta para algum n N, ou seja, qn =
n3 + 5n + 6 n3 + 5n + 6 n2 + n + 2
. Entao, qn+1 = qn + pn = + =
6 6 2
n3 + 3n2 + 8n + 12 (n3 + 3n2 + 3n + 1) + (5n + 5) + 6
= =
6 6
(n+1)3 +5(n+1)+6
6
, o que mostra que a formula tambem esta correta
para n + 1. Logo, por inducao, ela esta corrreta para todo n
natural.

7. O numero maximo de regioes determinadas por um ziguezague formado


2
por 2 semirretas e n segmentos de reta e n +n+4
2
. Para n = 1, sao de-
12 +1+4
terminadas 3 regioes; como 2 = 3, a formula proposta esta correta
para n = 1. Suponhamos que a formula esteja correta para n segmentos

6
de reta. Um segmento de reta pode ser acrescentado transformando-se
uma das semi-retas em um segmento (o que faz com que duas regioes
se transformem em uma so) e acrescentando-se uma nova semirreta
intersectando os n segmentos ja existentes e a outra semirreta; isto de-
termina sobre esta semirreta uma total de n + 2 segmentos. Portanto,
no processo sao acrescentadas n + 2 1 = n + 1 regioes. Logo, o
numero de regioes determinadas por um ziguezague com n segmentos
2 n2 +3n+6 (n2 +2n+1)+(n+1)+4
e n +n+4
2
+ n + 1 = 2
= 2
, o que mostra que a
formula tambem esta correta para n + 1. Logo, por inducao, ela esta
correta para todo n natural.

8. a) Em 2 movimentos, passa-se o disco menor para a terceira haste;


a seguir, o disco maior deve ser passado para a central; em mais
2 movimentos, o disco menor deve voltar para a primeira haste; o
disco maior e passado para a terceira haste; finalmente, em mais
dois movimentos, o disco menor passa para a terceira haste, para
um total de 8 movimentos.
b) hn+1 = 3hn + 2
c) Para n = 1, sao necessarios dois movimentos. Como 31 1 = 2, a
formula esta correta para n = 1 disco. Suponhamos que ela esteja
correta para n discos, isto e, hn = 3n 1. Entao hn+1 = 3hn + 2 =
3(3n 1) + 2 = 3n+1 1, o que mostra que a formula tambem
esta correta para n + 1 discos. Logo, por inducao, a formula esta
correta para qualquer numero de discos.

7
4
Aplicaes do Princpio de
Induo Matemtica
Sumrio
4.1 Exerccios Recomendados . . . . . . . . . . . . . . . 9

4.2 Exerccios Suplementares . . . . . . . . . . . . . . . 9

4.3 Textos Complementares . . . . . . . . . . . . . . . . 11

1
Unidade 4

Apresentaremos nesta unidade algumas aplicaes ldicas do Princpio de

Induo Matemtica ao mundo material.

Exemplo 1 [A Torre de Hani]

Voc provavelmente j conhece esse jogo bastante popular e que pode ser

facilmente fabricado ou ainda encontrado em lojas de brinquedos de madeira.

O jogo formado por n discos de dimetros distintos com um furo no

seu centro e uma base onde esto ncadas trs hastes. Numa das hastes,

esto enados os discos, de modo que nenhum disco esteja sobre um outro de

dimetro menor (veja gura abaixo).


 

   

Figura 1

O jogo consiste em transferir a pilha de discos para uma outra haste, des-

locando um disco de cada vez, de modo que, a cada passo, a regra acima seja

observada.

As perguntas naturais que surgem so as seguintes:

1. O jogo tem soluo para cada n N?

2. Em caso armativo, qual o nmero mnimo jn de movimentos para

resolver o problema com n discos?

Usando Induo Matemtica, vamos ver que a resposta primeira pergunta

armativa, qualquer que seja o valor de n. Em seguida, deduziremos uma

frmula que nos fornecer o nmero jn .


Considere a sentena

P (n) : O jogo com n discos tem soluo .

2
Aplicaes do Princpio de Induo Matemtica Unidade 4

Obviamente, P (1) verdade. Suponha que P (n) seja verdadeiro, para

algum n; ou seja, que o jogo com n discos tem soluo. Vamos provar que o

jogo com n+1 discos tem soluo.

Para ver isso, resolva inicialmente o problema para os n discos superiores da


pilha, transferindo-os para uma das hastes livre (isso possvel, pois estamos

admitindo que o problema com n discos possua soluo):

 
  
    

Figura 2

Em seguida, transra o disco que restou na pilha original (o maior dos

discos) para a haste vazia:


 
     

Figura 3

Feito isto, resolva novamente o problema para os n discos que esto juntos,
transferindo-os para a haste que contm o maior dos discos:


 
 
   
 
Figura 4

Isso mostra que o problema com n + 1 discos tambm possui soluo, e,


portanto, por Induo Matemtica, que P (n) verdadeira para todo n N.

3
Unidade 4

Para determinar uma frmula para jn , veja que, para resolver o problema

para n+1 discos com o menor nmero de passos, temos, necessariamente,

que passar duas vezes pela soluo mnima do problema com n discos. Temos,

ento, que

jn+1 = 2jn + 1.

Obtemos, assim, uma sequncia (jn ) denida recorrentemente. Pode-se

mostrar, sem diculdade, por induo, que seu termo geral dado por

jn = 2n 1.

(Este tipo de sequncias, as recorrncias, ser estudado de modo sistemtico

nas Unidades U7 e U8.)

+ Para Saber Mais - Origem da Torre de Hani - Clique para ler

Exemplo 2 [Os Coelhos de Fibonacci]

Trata-se do seguinte problema proposto e resolvido pelo matemtico italiano

Leonardo de Pisa em seu livro Liber Abacci, de 1202:

Quot paria coniculorum in uno anno ex uno pario germinentur.

Como no se ensina mais latim nas escolas, a vai uma traduo:

Quantos casais de coelhos descendem de um casal em um ano.

Leonardo passa a explicar o seu problema e a sua soluo, como segue (com

adaptao nossa):

Um casal de coelhos recm-nascidos foi posto num lugar cercado. Deter-

minar quantos casais de coelhos ter-se-o aps um ano, supondo que, a cada

ms, um casal de coelhos produz outro casal e que um casal comea a procriar

dois meses aps o seu nascimento.

Vamos organizar a nossa contagem na tabela a seguir.

4
Aplicaes do Princpio de Induo Matemtica Unidade 4

nmero de casais nmero de casais


ms total
do ms anterior recm-nascidos
o
1 0 1 1
o
2 1 0 1
o
3 1 1 2
o
4 2 1 3
o
5 3 2 5
o
6 5 3 8
o
7 8 5 13
o
8 13 8 21
o
9 21 13 34
o
10 34 21 55
o
11 55 34 89
o
12 89 55 144

Portanto, o nmero de casais de coelhos em um determinado ms igual ao

nmero total de casais do ms anterior acrescido do nmero de casais nascidos

no ms em curso, que igual ao nmero total de casais do ms anterior ao

anterior.

Se denotarmos o nmero de coelhos existentes no n-simo ms por un ,


temos, ento, que

un = un1 + un2 , u1 = u2 = 1.

Essas relaes denem, por recorrncia, uma sequncia de nmeros naturais,

chamada de sequncia de Fibonacci , cujos elementos, chamados de nmeros de

Fibonacci, possuem propriedades aritmticas notveis, que ainda hoje so objeto

de investigao.

+ Para Saber Mais - O que uma Recorrncia? - Clique para ler

+ Para Saber Mais - Leonardo de Pisa - Fibonacci - Clique para ler

5
Unidade 4

Exemplo 3 [O Enigma do Cavalo de Alexandre]

Num mosaico romano, Bucfalo, o cavalo de Alexandre, o Grande, repre-

sentado como um fogoso corcel cor de bronze. Nesse exemplo, vamos provar

que isso uma falcia (uma grande mentira).

Inicialmente, provaremos que todos os cavalos tm mesma cor. De fato,

considere a sentena:

P (n) : Num conjunto com n cavalos, todos tm a mesma cor .

Note que P (1) obviamente verdadeira. Agora, suponha o resultado vlido


para conjuntos contendo n cavalos. Considere um conjunto

C = {C1 , C2 , . . . , Cn , Cn+1 }

com n+1 cavalos. Decompomos o conjunto C numa unio de dois conjuntos:

C = C 0 C 00 = {C1 , . . . , Cn } {C2 , . . . , Cn+1 },

cada um dos quais contm n cavalos.

Pela hiptese indutiva, segue-se que os cavalos em C0 tm mesma cor, ocor-


00
rendo o mesmo para os cavalos em C . Como

C2 C 0 C 00 ,

segue-se que os cavalos de C0 tm a mesma cor dos cavalos de C 00 , permitindo

assim concluir que todos os cavalos em C tm a mesma cor.

Assim, a nossa demonstrao por induo est terminada, provando que

P (n) verdadeira para todo n N.


Agora, todo mundo sabe (voc sabia?) que Marengo, o famoso cavalo de

Napoleo, era branco. Logo, Bucfalo deveria ser branco.

Onde est o erro nessa prova?

Sugesto: Para ach-lo, sugerimos que voc tente provar que, se P (1)

verdadeira, ento P (2) verdadeira.

Esse problema foi inventado pelo matemtico hngaro George Polya (1887-

1985).

6
Aplicaes do Princpio de Induo Matemtica Unidade 4

[Descobrindo a Moeda Falsa] Exemplo 4


n
Tm-se 3 moedas de ouro, sendo uma delas falsa, com peso menor do

que as demais. Dispe-se de uma balana de dois pratos, sem nenhum peso.

Vamos mostrar, por induo sobre n, que possvel achar a moeda falsa com

n pesagens.

Para n = 1, isso fcil de ver, pois, dadas as trs moedas, basta pr uma

moeda em cada prato da balana e descobre-se imediatamente qual a moeda

falsa.

Suponha, agora, que o resultado seja vlido para algum valor de n e que se
n+1
tenha que achar a moeda falsa dentre 3 moedas dadas. Separemos as 3n+1
moedas em 3 grupos de 3n moedas cada. Coloca-se um grupo de 3n moedas

em cada prato da balana. Assim, poderemos descobrir em que grupo de 3n


moedas encontra-se a moeda falsa. Agora, pela hiptese de induo, descobre-

se a moeda falsa com n pesagens, que, junto com a pesagem j efetuada,

perfazem o total de n + 1 pesagens.

[A Pizza de Steiner] Exemplo 5


O grande gemetra alemo Jacob Steiner (1796-1863) props e resolveu,

em 1826, o seguinte problema:

Qual o maior nmero de partes em que se pode dividir o plano com n


cortes retos?

Pensando o plano como se fosse uma grande pizza, temos uma explicao

para o nome do problema.

Denotando o nmero mximo de pedaos com n cortes por pn , vamos provar


por induo a frmula:
n(n + 1)
pn = + 1.
2
Para n = 1, ou seja, com apenas um corte, claro que s podemos obter

dois pedaos. Portanto, a frmula est correta, pois

1(1 + 1)
p1 = + 1 = 2.
2
Admitamos agora que, para algum valor de n, a frmula para pn esteja

correta. Vamos mostrar que a frmula para pn+1 tambm est correta.

7
Unidade 4

Suponhamos que, com n cortes, obtivemos o nmero mximo n(n+1)/2+1


de pedaos e queremos fazer mais um corte, de modo a obter o maior nmero

possvel de pedaos.

Vamos conseguir isso se o (n + 1)-simo corte encontrar cada um dos n


cortes anteriores em pontos que no so de interseo de dois cortes (faa um

desenho para se convencer disso).

Por outro lado, se o (n+1)-simo corte encontra todos os n cortes anteriores,


ele produz n+1 novos pedaos: o corte comea em um determinado pedao

e, ao encontrar o primeiro corte, ele separa em dois o pedao em que est,

entrando em outro pedao. Ao encontar o segundo corte, ele separa em dois o

pedao em que est, entrando em outro pedao, e assim sucessivamente, at

encontrar o n-simo corte separando o ltimo pedao em que entrar em dois.

Assim, so obtidos n + 1 pedaos a mais dos que j existiam; logo,

n(n + 1) (n + 1)(n + 2)
pn+1 = pn + n + 1 = +1+n+1= + 1,
2 2
mostrando que a frmula est correta para n+1 cortes. O resultado segue

ento do Princpio de Induo Matemtica.

8
Aplicaes do Princpio de Induo Matemtica Unidade 4

4.1 Exerccios Recomendados

1. Prove que, qualquer que seja o nmero natural n maior do que 3, existe
um polgono convexo com n lados e exatamente 3 ngulos agudos.

2. Um plano est dividido em regies por vrias retas. Prove que pos-
svel colorir essas regies com duas cores de modo que quaiquer duas

regies adjacentes tenham cores diferentes (dizemos que duas regies so

adjacentes se elas tiverem pelo menos um segmento de reta em comum).

3. A sequncia (an ) denida pelos dados: a1 = 1, a2 = 2, an+1 = an an1


se n > 2. Prove que an+6 = an para todos os nmeros naturais n.
Descreva todos os termos dessa sequncia.

4. A sequncia a1 , a2 , . . . , an , . . . de nmeros tal que a1 = 3, a2 = 5 e


n
an+1 = 3an 2an1 para n > 2. Prove que an = 2 + 1 para todos os
nmeros naturais n.

4.2 Exerccios Suplementares

1. Ache o erro na prova do seguinte

 Teorema Todos os nmeros naturais so iguais .

Vamos provar o resultado mostrando que, para todo n N, verdadeira

a sentena

P (n) : : dado n N, todos os nmero naturais menores ou iguais do

que n so iguais.

(i) P (1) claramente verdadeira.

(ii) Suponha que P (n) n 1 = n. Somando 1 a


seja verdadeira, logo

ambos os lados dessa igualdade, obtemos n = n + 1. Como n era igual

a todos os naturais anteriores, segue que P (n + 1) verdadeira.

Portanto, P (n) 'e vedadeira para todo n N.

2. (O queijo de Steiner) Para fazer a sua pizza, Steiner teve que cortar,
primeiro, o queijo. Imaginando que o espao um enorme queijo, voc

9
Unidade 4 Exerccios Suplementares

seria capaz de achar uma frmula para o nmero mximo de pedaos que

poderamos obter ao cort-lo por n planos?

3. Mostre que a sequncia de Fibonacci satisfaz s seguintes identidades:

(a) u1 + u2 + + un = un+2 1.

(b) u1 + u3 + + u2n1 = u2n .

(c) u2 + u4 + + u2n = u2n+1 1.

(d) u21 + u22 + + u2n = un un+1 .



1+ 5
4. Sabendo que q = raiz da equao x2 = x + 1, mostre que
2
q n = un q + un1 .

5. Prove que
u3n+2 1
u3 + u6 + u9 + + u3n = .
2
6. Dada a recorrncia an+2 = 2an+1 + an , com a1 = 1 e a2 = 3, ache uma

frmula para an .

10
Aplicaes do Princpio de Induo Matemtica Unidade 4

4.3 Textos Complementares

Origem da Torre de Hani Para Saber Mais

Esse jogo foi idealizado e publicado pelo matemtico francs Edouard Lucas,

em 1882, que, para dar mais sabor sua criao, inventou a seguinte lenda:

Na origem do tempo, num templo oriental, uma Divindade colocou 64 discos

perfurados de ouro puro ao redor de uma de trs colunas de diamante e ordenou

a um grupo de sacerdotes que movessem os discos de uma coluna para outra,

respeitando as regras acima explicadas. A Divindade sentenciou que, quando

todos os 64 discos fossem transferidos para uma outra coluna, o mundo acabaria.

Voc no deve se preocupar com a iminncia do m do mundo, pois, se,

a cada segundo, um sacerdote movesse um disco, o tempo mnimo para que

ocorresse a fatalidade seria de 264 1 segundos e isto daria, aproximadamente,

um bilho de sculos!

11
Unidade 4 Textos Complementares

Para Saber Mais O que uma Recorrncia?

Uma recorrncia uma frmula que dene um elemento de uma sequncia

a partir de termos anteriores.

Uma recorrncia do tipo:

xn = xn1 + xn2 , (4.1)

s permite determinar o elemento xn se conhecermos os elementos anteriores

xn1 e xn2 , que, para serem calculados, necessitam do conhecimento dos dois

elementos anteriores, e assim por diante. Fica, portanto, univocamente denida

a sequncia quando so dados x1 e x2 . A sequncia de Fibonacci corresponde

recorrncia (4.1), onde x1 = x2 = 1.


Quando dada uma recorrncia, um problema importante determinar uma

frmula fechada para o termo geral da sequncia, isto , uma frmula que no

recorre aos termos anteriores. No caso da sequncia de Fibonacci, existe uma

tal frmula, chamada frmula de Binet , que apresentamos a seguir e que ser

demonstrada em um contexto mais geral na Unidade 8.

Para todo n N, tem-se que

 n  n
1+ 5 1 5
2
2
un =
5

notvel que seja necessrio recorrer a frmulas envolvendo nmeros ir-

racionais para representar os elementos da sequncia de Fibonacci, que so



1+ 5
nmeros naturais. Mais notvel, ainda, que o nmero = seja a
2
1 5
proporo urea que aparece nas artes, e que = 1 seja o simtrico
2
de seu inverso. Intrigante essa inesperada relao entre criar coelhos e a divina

proporo, no?

12
Aplicaes do Princpio de Induo Matemtica Unidade 4

Leonardo de Pisa - Fibonacci Para Saber Mais

Leonardo de Pisa (1170-1250), lho de Bonacci, e por isso apelidado Fi-

bonacci, teve um papel fundamental no desenvolvimento da Matemtica no

Ocidente. Em 1202, publicou o livro Liber Abacci, que continha grande parte

do conhecimento sobre nmeros e lgebra da poca. Esta obra foi responsvel

pela introduo na Europa do sistema de numerao indo-arbico e pelo pos-

terior desenvolvimento da lgebra e da aritmtica no mundo ocidental.

13
MA12 - Unidade 4
Mais Sobre Inducao

Paulo Cezar Pinto Carvalho

PROFMAT - SBM

7 de Marco de 2014
Comecando de um certo natural n0

Seja P(n) uma propriedade relativa ao numero natural n e


seja n0 um numero natural. Suponhamos que:
i) P(n0 ) e valida.
ii) Para todo n n0 , a validez de P(n) implica na validez de
P(n + 1).
Entao, P(n) e verdadeira para todo numero natural n n0 .

Prova: Basta mostrar, por inducao, que Q(n) : P(n + n0 1)


e valida para todo n natural.

PROFMAT - SBM MA12 - Unidade 4, Mais Sobre Inducao slide 2/13


Exemplo

Mostrar que P(n) : 2n > n2 , para todo numero natural n 5.

i) Temos que P(5) : 25 > 52 e verdadeira.


ii) Seja n 5 tal que 2n > n2 .
Multiplicando ambos os lados da desigualdade acima por 2,
obtemos 2n+1 > 2n2 .
Mas 2n2 > (n + 1)2 ?
Sim, para n 3, pois e equivalente a n(n 2) > 1.
Da, 2n+1 > (n + 1)2 , o que significa que P(n + 1) e
verdadeira.
Logo, pela forma generalizada do Princpio de Inducao
Matematica, a desigualdade vale para todo numero natural
n 5.

PROFMAT - SBM MA12 - Unidade 4, Mais Sobre Inducao slide 3/13


Usando mais de um antecessor

Seja P(n) uma propriedade relativa ao natural n.


Suponhamos que:
i) P(1) e P(2) sao validas.
ii) Para todo n N, a validez de P(n) e P(n + 1) implicam a
validez de P(n + 2).
Entao, P(n) e verdadeira para todo numero natural n.

Prova: Basta mostrar, por inducao, que Q(n) : P(n) e


P(n + 1) sao validas e verdadeira para todo natural n.

PROFMAT - SBM MA12 - Unidade 4, Mais Sobre Inducao slide 4/13


Os coelhos de Fibonacci
Um casal de coelhos recem-nascidos foi posto num lugar
cercado. Determinar quantos casais de coelhos ter-se-ao apos
um ano, supondo que, a cada mes, um casal de coelhos
produz outro casal e que um casal comeca a procriar dois
meses apos o seu nascimento.
numero de casais numero de casais
mes total
do mes anterior recem-nascidos
1o 0 1 1
2o 1 0 1
3o 1 1 2
4o 2 1 3
5o 3 2 5
6o 5 3 8
7o 8 5 13
8o 13 8 21

PROFMAT - SBM MA12 - Unidade 4, Mais Sobre Inducao slide 5/13


O numero de casais de coelhos em um determinado mes (a
partir do terceiro) e igual ao numero total de casais do mes
anterior acrescido do numero de casais nascidos no mes em
curso, que e igual ao numero total de casais do mes anterior
ao anterior.
Se un e o numero de casais no n-esimo mes, temos
u1 = 1
u2 = 1
un+2 = un + un+1 , para todo n N
Estas relacoes definem a chamada sequencia de Fibonacci.
E facil mostrar, por inducao, que
 n  n
1+ 5
2 12 5
un =
5

PROFMAT - SBM MA12 - Unidade 4, Mais Sobre Inducao slide 6/13


Inducao Completa

Seja P(n) uma propriedade relativa ao natural n.


Suponhamos que:
i) P(1) e valida.
ii) Para todo n N, a validez de P(k), para todo k n, implica
na validez de P(n + 1).
Entao, P(n) e verdadeira para todo numero natural n.

Prova: Basta mostrar, por inducao, que


Q(n) : P(1), P(2), . . . , P(n) sao todas validas e verdadeira
para todo natural n.

PROFMAT - SBM MA12 - Unidade 4, Mais Sobre Inducao slide 7/13


Exemplo

Pn
k=0 ak
Seja an uma sequencia definida por a0 = 2 e an+1 = ,
n+2
para cada natural n. Qual e o termo geral de an ?
Os primeiros termos da sequencia sao
a1 = 22 = 1, a2 = 2+1
3 = 1, a3 =
2+1+1
4 = 1,
o que sugere que an = 1, para todo n 1, com a0 = 2.
i) P(1) : a1 = 1 e verdadeira.
ii) Suponhamos, agora, que P(k) seja valida (isto e, ak = k) para
todo k tal que 1P k n.
n
k=0 ak
Entao, an+1 = n+2 = 2+n.1
n+2 = 1,
o que mostra que a formula vale para n + 1.
Logo, por inducao completa, an = 1, para todo n 1.

PROFMAT - SBM MA12 - Unidade 4, Mais Sobre Inducao slide 8/13


Lista de Exerccios
Unidade 4

1. Demonstre, por inducao, as seguintes desigualdades:

a) n! > 2n , para n 4;
b) n! > 3n , para n 7;
1 1 1 13
c) + + + > , para n 2;
n+1 n+2 2n 24

n n1
d) 2 > 1 + n 2 , para n 2.

2. Dados n (n > 2) objetos de pesos distintos, prove que e possvel deter-


minar qual o mais leve e qual o mais pesado fazendo 2n3 pesagens em
uma balanca de pratos. Mostre, tambem, que este e o numero mnimo
de pesagens que permitem, com certeza, determinar o mais leve e o
mais pesado.

3. Mostre que existem inteiros nao negativos x e y tais que 7x + 8y = n


para todo n 42. Seria possvel tomar um numero menor que 42 na
afirmativa acima?

4. Prove que, qualquer que seja o numero natural n maior do que ou igual
a 3, existe um polgono convexo com n lados e exatamente 3 angulos
agudos. [Sugestao: Modifique um dos angulos agudos de um polgono
com n lados para produzir um angulo agudo e um obtuso.]

5. A sequencia a1 , a2 , . . . , an , . . . de numeros e tal que a1 = 3, a2 = 5 e


an+1 = 3an 2an1 para n > 2. Prove que an = 2n + 1 para todos os
numeros naturais n.

6. Mostre que o termo geral da sequencia de Fibonacci e


 n  n
1+ 5
2
12 5
Fn = .
5

1
7. Mostre que a sequencia de Fibonacci satisfaz as seguintes identidades:

a) F1 + F2 + + Fn = Fn+2 1.

b) F1 + F3 + + F2n1 = F2n .

c) F2 + F4 + + F2n = F2n+1 1.

d) F12 + F22 + + Fn2 = Fn Fn+1 .

3 n2

8. Seja (Fn ) a sequencia de Fibonacci. Mostre que Fn 2
, para
todo n N.

9. Use inducao completa para demonstrar o Teorema Fundamental da


Aritmetica: todo numero natural n 2 e primo ou e um produto de
numeros primos.

10. Em uma das versoes do jogo de Nim, dois jogadores se alternam, re-
tirando 1, 2 ou 3 palitos de um monte de n palitos. O jogador que
remove o ultimo palito perde. Mostre que o primeiro a jogar tem uma
estrategia vencedora se, e somente se, o resto da divisao de n por 4 e
diferente de 1.

11. Considere um conjunto de n pontos. Conecta-se pares desses pontos


por segmentos de reta, ate que nao seja possvel acrescentar um novo
segmento sem formar um ciclo, isto e, um caminho fechado percorrendo
os segmentos de reta tracados. A figura mostra duas possveis situacoes
ao final do processo, para n = 6.

a) Mostre que, ao terminar o processo, cada par de pontos esta ligado


por um unico caminho formado por segmentos.
b) Mostre, por inducao, que, ao terminar o processo, terao sempre
sido tracados n 1 segmentos.

2
12. Ache o erro na prova do seguinte teorema: Todos os numeros
naturais sao iguais.
Vamos provar o resultado usando inducao completa.
(i) P (1) e claramente verdadeira.
(ii) Suponha que P (k) seja verdadeira, para todo k tal que 1 n. Entao
n = n 1. Somando 1 a ambos os lados dessa igualdade, obtemos
n = n + 1. Como n era igual a todos os naturais anteriores, segue que
P (n + 1) e verdadeira.
Portanto, P (n) e verdadeira para todo n N .

3
Solucoes da Lista de Exerccios
Unidade 4

1. a) Como 4! = 24 e 24 = 16, de fato temos n! > 2n para n = 4.


Suponhamos que a desigualdade valha para algum n 4, ou seja
n! > 2n . Multiplicando os dois lados da desigualdade por n + 1,
obtemos (n+1)! > (n+1)2n > 2.2n = 2n+1 (a ultima desigualdade
vale porque n + 1 > 2 para todo n 4). Logo, a desigualdade
tambem vale para n + 1. Portanto, por inducao, ela e valida para
todo n 4.
b) Como 7! = 5040 e 37 = 2187, de fato temos n! > 3n para n = 7.
Suponhamos que a desigualdade valha para algum n 7, ou seja
n! > 3n . Multiplicando os dois lados da desigualdade por n + 1,
obtemos (n+1)! > (n+1)3n > 3.3n = 3n+1 (a ultima desigualdade
vale porque n + 1 > 3 para todo n 7). Logo, a desigualdade
tambem vale para n + 1. Portanto, por inducao, ela e valida para
todo n 7.
1 1 7 13
c) Como + = > , a desigualdade vale para n = 2.
3 4 12 24
Suponhamos que ela seja valida para algum n 2, ou seja,
1 1 1 13 1 1
+ + + > . Subtraindo e somando +
n+1 n+2 2n 24 n+1 2n + 1
1
a ambos os lados da desigualdade, obtemos
2n + 2
1 1 1 13 1 1 1
+ + + > + + =
n+2 n+2 2(n + 1) 24 2n + 1 2n + 2 n + 1
13 2(n + 1) + (2n + 1) 2(2n + 1) 13 1
+ = + >
24 2(n + 1)(2n + 1) 24 2(n + 1)(2n + 1)
13
. Logo, a desigualdade tambem e valida para n + 1. Portanto,
24
por inducao, ela e valida para todo n 2.

d) Como 1 + 2 2 < 4 = 22 , a desigualdade vale para n = 2. Ela
tambem vale para n = 3, ja que 1 + 3 22 < 23 . Suponhamos
que ela seja valida para algum n 3, ou seja, 2n > 1 + n 2n1 .
Multiplicando os dois lados da desigualdade por 2, obtemos 2n+1 >

1

2(1 + n 2n1 ) = 2 + n 2 2n . Mas, para todo n 3, temos

n 2 = n + ( 2 1)n > n + 0, 4n > n + 1. Logo, a desigualdade
tambem vale para n + 1. Portanto, por inducao, a propriedade
vale para todo n 2.

2. Certamente a propriedade vale para n = 2: basta 1 = 2.3 3 pesagem


para determinar o mais leve e o mais pesado. Suponhamos que 2n 3
pesagens sejam suficientes para determinar o mais leve e o mais pesado
dentre objetos a1 , a2 , . . . , an (onde n 2 ) e suponhamos que um objeto
adicional an+1 seja acrescentado. Com 2n 3 pesagens, determinamos
o mais leve e o mais pesado dentre a1 , a2 , . . . , an . Com duas pesagens
adicionais, comparamos estes dois objetos com o adicional, determi-
nando o mais leve e o mais pesado dentre os n + 1 objetos, utilizando
no total 2n 3 + 2 = 2(n + 1) 3 pesagens. Logo, a propriedade
tambem vale para conjuntos com n + 1 objetos. Portanto, por inducao,
vale para conjuntos com n objetos para todo n 2.

3. Mostre que existem inteiros nao negativos x e y tais que 7x + 8y = n


para todo n 42. Seria possvel tomar um numero menor que 42 na
afirmativa acima?

4. Certamente a propriedade vale para n = 3, ja que existem triangulos


com 3 angulos agudos. Suponhamos que a propriedade vale para algum
n = 3, isto e, existe um polgono convexo com n lados e exatamente
e angulos agudos. Tomemos um destes angulos agudos, de medida
e tracemos uma reta que intersecta apenas os lados que formam este
angulo, de modo a determinar um triangulo com um outro angulo agudo
e um obtuso (por exemplo, um triangulo com angulos iguais a 45 2 ,
135 2 e . Esta reta produz um polgono convexo com n + 1 lados,
ainda com 3 angulos agudos. Logo, a propriedade vale para n + 1.
Portanto, por inducao, vale para todo n natural.

5. A expressao do termo geral esta correta para n = 1 e n = 2, ja que


a1 = 3 = 21 + 1 e a2 = 5 = 22 + 1. Suponhamos que ela esteja correta
para n e n + 1. Entao an+2 = 3an+1 2an = 3(2n+1 + 1) 2(2n + 1) =
3.2n+1 2n+1 + 1 = 2n+2 + 1. Logo, a expressao tambem esta correta
para n + 2. Portanto, por inducao, ela e valida para todo n natural.

6. A expressao esta correta para n = 0 e n = 1, ja que F0 = 0 =

2
 0  0    
1+ 5
2
12 5 1+ 5
2
12 5

5
e F1 = 1 =
5
. Suponhamos que a ex-
pressao esteja
   
correta para
 
n e n
 
+ 1. Entao Fn+2 = Fn + Fn+1 =
n n n+1 n+1
1+ 5
2
12 5 1+ 5
2
1 5
2

5
+
5
=
 n    n  
1+ 5
2
1+ 1+2 5 1 5
2
1+ 12 5

5

5
=
 n  2  n  2
1+ 5 1+ 5 1 5 1 5
2 2 2 2

5

5
=
 n+2  n+2
1+ 5
2
12 5

5
.
Logo, a expressao tambem esta correta para n + 2. Portanto, por
inducao, ela esta correta para todo n natural

7. a) A propriedade vale para n = 1, ja que F1 = 1 e F3 1 = 2 1 =


1. Suponhamos que ela seja valida para um natural n, ou seja,
F1 + F2 + + Fn = Fn+2 1. Somando Fn+1 aos dois lados da
igualdade, obtemos F1 + F2 + + Fn + Fn+1 = Fn+1 + Fn+2 1 =
Fn+3 1, o que mostra que a igualdade tambem vale para n + 1.
Portanto, por inducao, ela vale para todo n natural.
b) A propriedade vale para n = 1, ja que F1 = F2 = 1. Suponhamos
que ela seja valida para um natural n, ou seja, F1 + F3 + +
F2n1 = F2n . Somando F2n+1 aos dois lados da igualdade, obtemos
F1 + F3 + + F2n1 + F2n+1 = F2n + F2n+1 = F2n+2 , o que mostra
que a igualdade tambem vale para n + 1. Portanto, por inducao,
ela vale para todo n natural.
c) A propriedade vale para n = 1, ja que F2 = 1 e F3 1 = 1.
Suponhamos que ela seja valida para um natural n, ou seja, F2 +
F4 + + F2n = F2n+1 1. Somando F2n+2 aos dois lados da
igualdade, obtemos F2 +F4 + +F2n +F2n+2 = F2n+1 +F2n+2 1 =
F2n+3 1, o que mostra que a igualdade tambem vale para n + 1.
Portanto, por inducao, ela vale para todo n natural.
d) A propriedade vale para n = 1, ja que F12 = F1 F2 = 1. Su-
ponhamos que ela seja valida para um natural n, ou seja, F12 +
F22 + + Fn2 = Fn Fn+1 . Somando Fn+1
2
aos dois lados da igual-
2 2 2 2 2
dade, obtemos F1 + F2 + + Fn + Fn+1 = Fn Fn+1 + Fn+1 =

3
F n + 1(Fn + Fn+1 ) = F n + 1F n + 2, o que mostra que a igual-
dade tambem vale para n+1. Portanto, por inducao, ela vale para
todo n natural.
1
8. A propriedade vale para n = 1 e n = 2, ja que F1 = 1 > 32 e
3 0

F2 = 1 = 2 . Suponhamos que a desigualdade seja valida para n e
n2 n1 n2
n + 1. Entao Fn+2 = Fn + Fn+1 = 23 + 32 = 32 1 + 32 =


3 n2 5 3 n2 9 3 n
    
2 2
> 2 4
= 2
. Logo, a desigualdade vale para n + 2.
Portanto, por inducao, vale para todo n natural.

9. Como 2 e primo, a propriedade vale para n = 2. Suponhamos que ela


seja valida para todo natural k tal que 2 k n. Se n + 1 nao for
primo, entao pode ser expresso na forma a.b, onde a e b sao numeros
naturais maiores que 1 e menores que n + 1. Portanto, pela hipotese de
inducao, cada um dos numeros a e b e primo ou um produto de primos,
o que mostra que n + 1 e um produto de primos. Logo, a propriedade
tambem vale para n + 1. Logo, por inducao (completa), a propriedade
vale para todo n natural.

10. A afirmativa e verdadeira se o numero de palitos e 1, 2, 3 ou 4. No


primeiro caso, o primeiro jogador nao tem uma estrategia vencedora,
ja que e obrigado a tirar o unico palito e perde o jogo. Nos demais,
ele pode, tirando 1, 2 ou 3 palitos, respectivamente, deixar o segundo
jogador com apenas um palito e, assim, garantir a vitoria. Suponhamos
agora, que a propriedade seja verdadeira para todo natural k menor ou
igual a n e consideremos um jogo com n + 1 palitos. Se n + 1 4,
a afirmativa e verdadeira, como mostrado acima. Caso contrario, se o
resto da divisao de n + 1 por 4 nao e 1, o primeiro jogador pode sempre
retirar 1, 2 ou 3 palitos de modo a deixar o segundo jogador com um
numero de palitos menor ou igual a n tal que o resto da divisao por 4
e 1. Pela hipotese de inducao, esta nao e uma posicao vencedora para
o segundo jogador e, portanto, o primeiro ganha o jogo. Por outro
lado, se o resto da divisao de n + 1 por 4 for 1, o primeiro jogador
nao tem uma estrategia vencedora, ja que qualquer jogada faz com que
o segundo tenha uma quantidade de palitos menor ou igual a n, com
resto da divisao por 4 diferente de 1, podendo assim ganhar o jogo.
Logo, a propriedade vale para n + 1 palitos. Portanto, por inducao vale
para qualquer quantidade de palitos.

4
11. a) Se dois pontos nao estao conectados por um caminho, pode-se liga-
los por um segmento sem que um ciclo seja criado. Por outro lado,
se dois pontos estao conectados por dois caminhos diferentes, eles
formam um ciclo. Logo, ao final do processo cada par de pontos
esta ligado por um unico caminho.
b) A propriedade vale para n = 1, ja que, neste caso, o numero de
segmentos e 0 = 1 1. Suponhamos que a propriedade valha
para todos os conjuntos nos quais o numero de pontos seja menor
ou igual a n e suponhamos que o processo foi encerrado para
um conjunto com n + 1 pontos. A retirada de qualquer segmento
desta configuracao decompoe o conjunto de pontos em dois outros,
respectivamente com n1 e n2 pontos, tais que n1 + n2 = (n +
1). Em cada um destes conjuntos nao ha ciclos e acrescentando-
se qualquer segmento forma-se um ciclo. Assim, como n1 n
e n2 n, ha neles, pela hipotese de inducao, n1 1 e n2 1
segmentos. Logo, o numero total de segmentos com n + 1 pontos
e (n1 1) + (n2 1) + 1 = n1 + n2 1 = n. Logo, a propriedade
vale para conjuntos com n + 1 pontos. Portanto, por inducao, vale
para conjuntos com quaisquer quantidade de pontos.

12. O argumento nao funciona na passagem de n = 1 para n = 2.

5
5

Progresses Aritmticas

Sumrio
5.1 Introduo . . . . . . . . . . . . . . . . . . . . . . . 2

5.2 Primeiros Exemplos . . . . . . . . . . . . . . . . . . 2

5.3 Soma dos Termos de uma PA . . . . . . . . . . . . 6

5.4 Somas Polinomiais . . . . . . . . . . . . . . . . . . . 9

5.5 Exerccios Recomendados . . . . . . . . . . . . . . . 14

5.6 Exerccios Suplementares . . . . . . . . . . . . . . . 16

5.7 Textos Complementares . . . . . . . . . . . . . . . . 20

1
Unidade 5 Introduo

5.1 Introduo

As Progresses Aritmticas (PA) constituem-se na famlia mais simples de


sequncias denidas recorrentemente. Elas so comuns na vida real e sempre
aparecem quando se apresentam grandezas que sofrem variaes iguais em in-
tervalos de tempos iguais como, por exemplo, no clculo de juros simples, ou
desvalorizao de um bem ao longo do tempo.
Nessa unidade, voc encontrar tambm a frmula que fornece a soma dos
n primeiros termos de uma PA, frmula que generaliza a que foi descoberta por
Gauss, quando menino, conforme vimos na Unidade 3.
Em seguida, so denidas generalizaes do conceito de PA, introduzindo
as PAs de segunda ordem, terceira ordem, etc. Esse tpico, em geral, no
explorado no Ensino Mdio, mas coloca disposio do professor mtodos
poderosos para calcular somas.

5.2 Primeiros Exemplos

So comuns na vida real, grandezas que sofrem variaes iguais em inter-


valos de tempos iguais. Vejamos algumas situaes concretas.

Exemplo 1 Uma fbrica de automveis produziu 400 veculos em janeiro e aumen-


tou mensalmente sua produo de 30 veculos. Quantos veculos produziu em
junho?
Soluo Os valores da produo mensal, a partir de janeiro, so 400, 430, 490,

520, 550, . . . . Em junho, a fbrica produziu 550 veculos.


Poderamos ter evitado escrever a produo ms a ms, racionando do modo
a seguir. Se a produo aumenta de 30 veculos por ms, em 5 meses ela
aumenta 5 30 = 150 veculos. Em junho, a fbrica produziu 400 + 150 = 550
veculos.

Progresses aritmticas so sequncias nas quais o aumento de cada termo


para o seguinte sempre o mesmo.
A sequncia (400, 430, 460, 490, 520, 550, . . .) um exemplo de uma pro-
gresso aritmtica.

2
Progresses Aritmticas Unidade 5

O aumento constante de cada termo para o seguinte chamado de razo


de progresso. A razo da progresso acima igual a 30.
Vamos denio formal.

Uma progresso aritmtica uma sequncia na qual a diferena entre cada Definio 1
termo e o termo anterior constante. Essa diferena constante chamada de
razo da progresso e representada pela letra r .

As sequncias (5, 8, 11, 14, . . .) e (7, 5, 3, 1, . . .) so progresses aritmticas Exemplo 2


cujas razes valem respectivamente 3 e 2.

Em uma progresso aritmtica ( a1 , a2 , a3 , . . .), para avanar um termo,


basta somar a razo; para avanar dois termos, basta somar duas vezes a razo,
e assim por diante. Assim, por exemplo, a13 = a5 + 8r, pois, ao passar de a5
para a13 , avanamos 8 termos; a12 = a7 + 5r, pois avanamos 5 termos ao
passar de a7 para a12 ; a4 = a17 13r, pois retrocedemos 13 termos ao passar
de a17 para a4 e, de modo geral,

an = a1 + (n 1)r,

pois, ao passar de a1 para an , avanamos n 1 termos.

Em uma progresso aritmtica, o quinto termo vale 30 e o vigsimo termo Exemplo 3


vale 50. Quanto vale o oitavo termo dessa progresso?
Soluo a20 = a5 + 15r , pois ao passar do quinto termo para o vigsimo,
4
avanamos 15 termos. Logo, 50 = 30 + 15r e r = . Analogamente, a8 =
3
4
a5 + 3r = 30 + 3. = 34. O oitavo termo vale 34.
3

Qual a razo da progresso aritmtica que se obtm inserindo 10 termos Exemplo 4


entre os nmeros 3 e 25?
Soluo. Temos a1 = 3 e a12 = 25. Como a12 = a1 +11r , temos 25 = 3+11r .

Da, r = 2.

3
Unidade 5 Primeiros Exemplos

Exemplo 5 O cometa Halley visita a Terra a cada 76 anos. Sua ltima passagem por
aqui foi em 1986. Quantas vezes ele visitou a Terra desde o nascimento de
Cristo? Em que ano foi sua primeira passagem na era crist?
Soluo Os anos de passagem do cometa foram 1986, 1910, 1834,... e formam

uma progresso aritmtica de razo 76. O termo de ordem n dessa progresso


an = a1 + (n 1)r, isto , an = 1986 76(n 1) = 2062 76n. Temos
2062
an > 0 quando n < = 27, 13 . . . . Portanto, os termos positivos dessa
76
progresso so os 27 primeiros, a1 , a2 , a3 , . . . , a27 . Logo, ele nos visitou 27
vezes na era crist e sua primeira passagem na era cristo foi no ano a27 =
2062 76 27 = 10.
Poderamos tambm ter resolvido o problema aproveitando o fato dos termos
dessa progresso serem inteiros.
Em uma progresso aritmtica de termos inteiros e razo no-nula, todos os
termos do o mesmo resto quando divididos pelo mdulo da razo. Como 1986
dividido por 76 d resto 10, todos os anos em que o cometa por aqui passou
do resto 10 quando divididos por 76. A primeira visita ocorreu entre os anos
1 e 76, inclusive. Entre esses anos, o nico que dividido por 76 d resto 10 o
ano 10. Para descobrir a ordem desse termo, usamos an = a1 + (n 1)r, isto
, 10 = 1986 76(n 1). Da,
2062
n= = 27.
76

Muitas vezes conveniente enumerar os termos de uma progresso aritm-


tica a partir de zero, conforme mostra o exemplo a seguir.

Exemplo 6
O preo de um carro novo de R$ 15 000,00 e diminui de R$1 000,00 a
cada ano de uso. Qual ser o preo com 4 anos de uso?
Soluo Chamando o preo com n anos de uso de an , temos a0 = 15000 e

queremos calcular a4 . Como a desvalorizao anual constante, (an ) uma


progresso aritmtica. Logo, a4 = a0 + 4r = 15000 + 4 (1000) = 11000.
O preo ser de R$11 000,00.

4
Progresses Aritmticas Unidade 5

Os lados de um tringulo retngulo formam uma progresso aritmtica Exemplo 7


crescente. Mostre que a razo dessa progresso igual ao raio do crculo
inscrito.
Soluo. Chamemos os lados do tringulo de x r, x, x + r . Esse um bom

truque para facilitar as contas; ao representar uma progresso aritmtica com


um nmero mpar de termos, comear pelo termo central.
Como a progresso crescente, a hipotenusa o ltimo termo. Pelo Teo-
rema de Pitgoras, (x + r)2 = (x r)2 + x2 . Da, x2 = 4rx e, j que x 6= 0
pois x um dos catetos, x = 4r. Os lados so ento 3r, 4r e 5r. O permetro
S 6r2
2p = 3r + 4r + 5r = 12r e a rea = = r.
p 6r

Determine 4 nmeros em progresso aritmtica crescente, conhecendo sua Exemplo 8


soma 8 e a soma de seus quadrados 36.
Soluo Um bom truque, para representar progresses aritmticas com um

nmero par de termos, chamar os dois termos centrais de x y e x + y . Isso


faz com que a razo seja (x + y) (x y) = 2y .
A progresso ento x 3y , x y , x + y , x + 3y .
Temos
(
(x 3y) + (x y) + (x + y) + (x + 3y) = 8
(x 3y)2 + (x y)2 + (x + y)2 + (x + 3y)2 = 36
(
4x = 8
4x2 + 20y 2 = 36
(
x=2
y = 1

Como a progresso crescente, y > 0. Logo, x = 2 e y = 1. Os nmeros so


1, 1, 3, 5.

Em uma progresso aritmtica, o termo geral dado por um polinmio em


n, an = a1 + (n 1)r = r . n + (a1 r). Se r 6= 0, ou seja, se a progresso
no for estacionria (constante), esse polinmio de grau 1. Se r = 0, isto ,
se a progresso for estacionria, esse polinmio de grau menor que 1.

5
Unidade 5 Soma dos Termos de uma PA

Por esse motivo, as progresses aritmticas de razo r 6= 0 so chamadas


de progresses aritmticas de primeira ordem.
Reciprocamente, se em uma sequncia o termo de ordem n for dado por
um polinmio em n, de grau menor que ou igual a 1, ela ser uma progresso
aritmtica. Com efeito, se xn = an + b, (xn ) uma progresso aritmtica na
qual a = r e b = a1 r, ou seja, r = a e a1 = a + b.

Como em uma progresso aritmtica an = a0 + nr, a funo que associa a


cada natural n o valor de an simplesmente a restrio aos naturais da funo
am a(x) = a(0) + rx.
Portanto, pensando em uma progresso aritmtica como uma funo que
associa a cada nmero natural n o valor an , o grco dessa funo formado
por uma sequncia de pontos colineares no plano.
Em outras palavras, (an ) uma progresso aritmtica se e somente se os
pontos do plano que tm coordenadas (1, a1 ), (2, a2 ), (3, a3 ), etc. esto em
linha reta.

Figura 5.1: Grco de uma PA

5.3 Soma dos Termos de uma PA

Baseados na ideia de Gauss, usada para calcular a soma 1 + 2 + + 100,


podemos calcular a soma dos n primeiros termos de uma progresso aritmtica
qualquer.

6
Progresses Aritmticas Unidade 5

A soma dos n primeiros termos da progresso aritmtica (a1 , a2 , a3 , ...) Teorema 2


(a1 + an )n
Sn = .
2

Temos Sn = a1 + a2 + a3 + + an1 + an e, escrevendo a soma de trs Demonstrao


para frente, Sn = an + an1 + an2 + + a2 + a1 . Da,

2Sn = (a1 + an ) + (a2 + an1 ) + (a3 + an2 ) + + (an1 + a2 ) + (an + a1 ).

Observe que, ao passar de um parntese para o seguinte, a primeira parcela


aumenta de r e a segunda parcela diminui de r, o que no altera a soma.
Portanto, todos os parnteses so iguais ao primeiro, (a1 + an ). Como so n
parnteses, temos
(a1 + an )n
2Sn = (a1 + an ) . n e Sn = .
2

Qual o valor da soma dos 20 primeiros termos da progresso aritmtica Exemplo 9


2, 6, 10, . . . ?
Soluo a20 = a1 + 19r = 2 + 19 4 = 78.

(2 + 78)20
S20 = = 800.
2

A soma dos n primeiros nmeros inteiros e positivos Exemplo 10


n
X n(n + 1)
k = 1 + 2 + 3 + + n = .
k=1
2

Observe que Sn , no exemplo anterior, um polinmio do segundo grau em


n, sem termo independente.

7
Unidade 5 Soma dos Termos de uma PA

Exemplo 11 A soma dos n primeiros nmeros mpares


(1 + 2n 1)n
1 + 3 + 5 + + (2n 1) = = n2 .
2

Observe que Sn , no exemplo anterior, tambm um polinmio do segundo


grau em n, sem termo independente. Isto se generaliza como segue.
A soma dos n primeiros termos de uma progresso aritmtica
 
(a1 + an )n [a1 + a1 + (n 1)r]n r 2 r
Sn = = = n + a1 n.
2 2 2 2
Observe que, se r 6= 0, ento Sn um polinmio do segundo grau em n,
desprovido de termo independente. Se r = 0, Sn um polinmio de grau
menor que 2, sem termo independente.
Reciprocamente, todo polinmio do segundo grau em n, desprovido de termo
independente, o valor da soma dos n primeiros termos de alguma progresso
aritmtica. Com efeito P (n) = an2 + bn a soma dos n primeiros termos da
r r
progresso aritmtica na qual = a e a1 = b, ou seja, r = 2a e a1 = a + b.
2 2
Definio 3 Dene-se para sequncias o operador , chamado de operador diferena,
por an = an+1 an .

Portanto, da denio segue imediatamente que uma sequncia (an ) uma


progresso aritmtica se e somente se (an ) = (an+1 an ) constante.

Definio 4 Uma progresso aritmtica de segunda ordem uma sequncia (an ) na qual
as diferenas an = an+1 an , entre cada termo e o termo anterior, formam
uma progresso aritmtica no-estacionria.

Exemplo 12 A sequncia (an ) = (1, 3, 6, 10, 15, 21, . . .) uma progresso aritmtica de
segunda ordem porque a sequncia das diferenas entre cada termo e o anterior,

(bn ) = (an ) = (an+1 an ) = (2, 3, 4, 5, 6, . . .)

uma progresso aritmtica no-estacionria.

8
Progresses Aritmticas Unidade 5

+ Para Saber Mais - PAs de Ordem Superior - Clique para ler

5.4 Somas Polinomiais

A pergunta que nos colocamos como calcular somas do tipo nk=1 P (k),
P

onde P (k) um polinmio em k .


Se o polinmio P (k) = a0 + a1 k + a2 k 2 + + am k m , temos que
Pn Pn Pn Pn 2
Pn m
k=1 P (k) = k=1 a0 + k=1 a1 k + k=1 a2 k + + k=1 am k
= a0 k=1 1 + a1 k=1 k + a2 k=1 k 2 + + am nk=1 k m ,
Pn Pn Pn P

que pode ser calculado desde que saibamos calcular, para p N, somas do tipo:
n
X
k p = 1p + 2p + + np .
k=1

A soma dos quadrados dos n primeiros nmeros inteiros e positivos Exemplo 13


n
X
12 + 22 + + n2 = k2
k=1

e pode ser calculada do modo a seguir:


n
X n
X n
X n
X n
X
3 3 2
(k + 1) = k +3 k +3 k+ 1.
k=1 k=1 k=1 k=1 k=1

Os dois primeiros somatrios tm vrias parcelas comuns, pois


n
X
(k + 1)3 = 23 + 33 + + n3 + (n + 1)3
k=1

e n
X
k 3 = 13 + 23 + 33 + + n3 .
k=1

Simplicando as parcelas comuns aos dois membros, obtemos


n
X n
X n
X
3 3 2
(n + 1) = 1 + 3 k +3 k+ 1.
k=1 k=1 k=1

9
Unidade 5 Somas Polinomiais

Como n
X n(n + 1)
k = 1 + 2 + + n =
k=1
2
e n
X
1 = 1 + 1 + + 1 = n,
k=1
temos n
X n(n + 1)
(n + 1)3 = 13 + 3 k2 + 3 + n.
k=1
2
Da,
n
X 2n3 + 3n2 + n
2 n(n + 1)(2n + 1)
k = = .
k=1
6 6
n
Observe que 1 + 2 + + n =
X
2 2 2
k 2 um polinmio do terceiro grau
k=1
em n.

Exemplo 14 Sabendo que


n
X
12 + 22 + + n2 = k2
k=1

um polinmio do terceiro grau em n, poderamos ter determinado o valor de

p(n) = 12 + 22 + 32 + + n2

pondo p(n) = an3 + bn2 + cn + d. Assim, temos

p(1) = 12 , p(2) = 12 + 22 , p(3) = 12 + 22 + 32 e p(4) = 12 + 22 + 32 + 42 .

Obtemos o sistema de equaes




a+b+c+d=1

8a + 4b + 2c + d = 5


27a + 9b + 3c + d = 14

64a + 16b + 4c + d = 30
1 1 1
Resolvendo, encontramos a = , b = , c = , d = 0. Ento
3 2 6
1 1 1 n(n + 1)(2n + 1)
12 + 22 + 32 + + n2 = n3 + n2 + n = .
3 2 6 6

10
Progresses Aritmticas Unidade 5

Os teoremas a seguir generalizam os ltimos exemplos.

n
Teorema 5
X
p p p
1 + 2 + 3 + + n = p
k p um polinmio de grau p + 1 em n.
k=1

Vamos proceder por induo sobre p. Para p = 1, o teorema j foi provado Demonstrao
anteriormente. n
Suponhamos agora que
X
k p seja um polinmio de grau p + 1 em n,
k=1
para todo p {1, 2, . . . , s}, Mostraremos que essa armao verdadeira para
n
p = s + 1, isto , mostraremos que
X
k s+1 um polinmio de grau s + 2 em
k=1
n. Observe que

(k + 1)s+2 = k s+2 + (s + 2)k s+1 + ,

onde os termos que no foram escritos explicitamente formam um polinmio de


grau s em k . Temos ento,
n
X n
X n
X
s+2 s+2
(k + 1) = k + (s + 2) k s+1 + F (n),
k=1 k=1 k=1

onde F (n) um polinmio de grau s + 1 em n, pela hiptese da induo.


Simplicando os termos comuns aos dois primeiros somatrios, obtemos
n
X
(n + 1)s+2 = 1 + (s + 2) k s+1 + F (n).
k=1

Da,
n
X (n + 1)s+2 1 F (n)
k s+1 =
k=1
s+2
que um polinmio de grau s + 2 em n.

11
Unidade 5 Somas Polinomiais

n
Corolrio 6 Se F um polinmio de grau p ento
X
F (k) um polinmio de grau
k=1
p + 1 em n.

Exemplo 15 n
Vamos calcular Sn =
X
k(k+2). Pelo corolrio, sabemos que o valor dessa
k=1
soma um polinmio do terceiro grau em n. Ento Sn = an3 + bn2 + cn + d.
Atribuindo a n os valores 1, 2, 3 e 4 obtemos as equaes


a+b+c+d=3

8a + 4b + 2c + d = 11


27a + 9b + 3c + d = 26

64a + 16b + 4c + d = 50
1 3 7
Resolvendo, encontramos a = , b = , c = , d = 0. Ento,
3 2 6
3 2
1 3 7 2n + 9n + 7n n(n + 1)(2n + 7)
S n = n3 + n2 + n = = .
3 2 6 6 6

+ Para Saber Mais - PA com Termo Geral Polinomial - Clique para ler
O exemplo a seguir conhecido como Teorema Fundamental da Somao
e fornece uma tcnica bastante eciente para o clculo de somas.

Exemplo 16 n
Mostre que
X
ak = an+1 a1 .
k=1
Soluo

n
X
ak = a1 + a2 + a3 + + an1 + an =
k=1
(a2 a1 ) + (a3 a2 ) + (a4 a3 ) + + (an an1 ) + (an+1 an ) =
an+1 a1 .

12
Progresses Aritmticas Unidade 5

n Exemplo 17
Calcule
X
k(k + 1)
k=1
Soluo Determinaremos ak tal que ak = k(k + 1) = k 2 + k . Como(ak )
uma progresso aritmtica de segunda ordem, (ak ) uma progresso aritmtica
de terceira ordem. Logo, ak um polinmio de terceiro grau. Se

ak = ak 3 + bk 2 + ck + d,
ak = ak+1 ak
= a(k + 1)3 + b(k + 1)2 + c(k + 1) + d [ak 3 + bk 2 + ck + d]
= 3ak 2 + (3a + 2b)k + (a + b + c) = k 2 + k.
1 1
Devemos ter 3a = 1, 3a + 2b = 1, a + b + c = 0. Da, a = , b = 0, c =
3 3
1 3 1
e d arbitrrio. Logo, ak = k k + d.
3 3
n
X n
X
k(k + 1) = ak = an+1 a1
k=1 k=1
(n + 1)3 (n + 1) n(n + 1)(n + 2)
= +dd= .
3 3

13
Unidade 5 Exerccios Recomendados

5.5 Exerccios Recomendados

1. Formam-se n tringulos com palitos, conforme a gura. Qual o nmero


de palitos usados para construir n tringulos?

Figura 5.2:

2. Calcule a soma de todos os inteiros que divididos por 11 do resto 7 e


esto compreendidos entre 200 e 400.

3. Quanto vale o produto (a)(aq)(aq 2 )(aq 3 ) . . . (aq n1 )?

4. Um quadrado mgico de ordem n uma matriz n n, cujos elementos


so os inteiros 1, 2, . . . , n2 , sem repetir nenhum, tal que todas as linhas
e todas as colunas tm a mesma soma. O valor dessa soma chamado
de constante mgica. Por exemplo, os quadrados

17 24 1 8 15

1 5 9 8 1 6 23 5 7 14 16

3 5 7 e

8 3 4 4 6 13 20 22


6 7 2 4 9 2 10 12 19 21 3

11 18 25 2 9

so mgicos, com constantes mgicas respectivamente iguais a 15, 15


e 65. Alis, os dois ltimos so hipermgicos, pois as linhas, colunas e
tambm as diagonais tm a mesma soma. Calcule a constante mgica de
um quadrado mgico de ordem n.

5. Suprimindo um dos elementos do conjunto {1, 2, . . . , n}, a mdia aritm-


tica dos elementos restantes 16,1. Determine o valor de n e qual foi o
elemento suprimido.

14
Progresses Aritmticas Unidade 5

6. Um bem, cujo valor hoje de R$ 8000,00, desvaloriza-se de tal forma


que seu valor daqui a 4 anos ser de R$ 2000,00. Supondo que o valor
do bem cai segundo uma linha reta, determine o valor do bem daqui a 3
anos.

7. Prove que a soma de todos os inteiros positivos de n dgitos, n > 2,


igual ao nmero 49499...95500...0, no qual h n 3 dgitos sublinhados
que so iguais a 9 e n 2 dgitos sublinhados que so iguais a 0.

8. Considere um jogo entre duas pessoas com as seguintes regras:


i) Na primeira jogada, o primeiro jogador escolhe um nmero no conjunto
A = {1, 2, 3, 4, 5, 6, 7} e diz esse nmero.
ii) As pessoas jogam alternadamente.
iii) Cada pessoa ao jogar escolhe um elemento de A, soma-o ao nmero
dito pela pessoa anterior e diz a soma.
iv) Ganha quem disser 63.
Qual dos jogadores tem uma estratgia vencedora e qual essa estratgia?

9. Na primeira fase do campeonato brasileiro de futebol, que disputado por


24 clubes, quaisquer dois times jogam entre si uma nica vez. Quantos
jogos h?

10. Qual o nmero mximo de regies em que n retas podem dividir o plano?

11. H dois tipos de anos bissextos: os que so mltiplos de 4 mas no de


100 e os que so mltiplos de 400.

(a) Quantos so os anos bissextos entre 1997 e 2401?

(b) Se 1o de janeiro de 1997 foi quarta-feira, que dia ser 1o de janeiro


de 2500?

(c) Escolhido um ano ao acaso, qual a probabilidade dele ser bissexto?

12. O nmero triangular Tn denido como a soma dos n primeiros ter-


mos da progresso aritmtica 1, 2, 3, 4, . . .. O nmero quadrangular Qn
denido como a soma dos n primeiros termos da progresso aritmtica

15
Unidade 5 Exerccios Suplementares

1, 3, 5, 7, . . .. Analogamente so denidos nmeros pentagonais, hexago-


nais, etc. A gura abaixo justica essa denominao.
Determine o nmero j -gonal de ordem n.

Figura 5.3:

13. Mostre que ak = bk ento ak bk constante.

14. Use o teorema fundamental da somao para calcular:


n
(a)
X
3k .
k=1

n
(b)
X
k . k!.
k=1

n
1
(c)
X
.
k=1
k(k + 1)

5.6 Exerccios Suplementares

1. Os ngulos internos de um pentgono convexo esto em progresso arit-


mtica. Determine o ngulo mediano.

2. Se 3 x, x, 9 x, . . . uma progresso aritmtica, determine x e
calcule o quinto termo.
3. Calcule a soma dos termos da progresso aritmtica 2, 5, 8, 11,... desde
o 25o at o 41o termo, inclusive.

16
Progresses Aritmticas Unidade 5

4. Quantos so os inteiros, compreendidos entre 100 e 500, que no so


divisveis nem por 2, nem por 3 e nem por 5? Quanto vale a soma desses
inteiros?
5. Determine o maior valor que pode ter a razo de uma progresso aritm-
tica que admita os nmeros 32, 227 e 942 como termos da progresso.
6. De quantos modos o nmero 100 pode ser representado como uma soma
de dois ou mais inteiros consecutivos? E como soma de dois ou mais
naturais consecutivos?
7. Os inteiros de 1 a 1000 so escritos ordenadamente em torno de um
crculo. Partindo de 1, riscamos os nmeros de 15 em 15, isto , riscamos
1, 16, 31,... O processo continua at se atingir um nmero j previamente
riscado. Quantos nmeros sobram sem riscos?

8. Podem os nmeros 2, 3, 5 pertencer a uma mesma progresso
aritmtica?
9. Um bem, cujo valor hoje de R$ 8000,00, desvaloriza-se de tal forma
que seu valor daqui a 4 anos ser de R$ 2000,00. Supondo constante a
desvalorizao anual, qual ser o valor do bem daqui a 3 anos?
p
10. Calcule a soma de todas as fraes irredutveis, da forma , que perten-
72
am ao intervalo [4,7].
11. Qual a maior potncia de 7 que divide 1000!?
12. Calcule o valor das somas dos n primeiros termos das sequncias?

(a) 13 , 23 , 33 , . . .

(b) 1 . 4, 3 . 7, 5 . 10, 7 . 13, . . .

13. Representando por bxc a parte inteira do real x, isto , o maior nmero
inteiro que menor que ou igual a x e por {x} o inteiro mais prximo do
real x, determine:

(a) b 1c + b 2c + b 3c + + b n2 1c.

(b) b 3 1c + b 3 2c + b 3 3c + + b 3 n3 1c.

17
Unidade 5 Exerccios Suplementares

1 1 1 1
(c) + + + + .
{ 1} { 2} { 3} { 1000}

(d) { 1} + { 2} + { 3} + + { 1000}.

14. Determine o primeiro termo e a razo da progresso aritmtica na qual a


soma dos n primeiros termos , para todo n:

(a) Sn = 2n2 + n

(b) Sn = n2 + n + 1

15. Determine no quadro abaixo:

1
3 5
7 9 11
13 15 17 19
21 23 25 27 29
... ... ... ... ... ...

(a) o primeiro elemento da 31a linha.

(b) a soma dos elementos da 31a linha.

16. Refaa o Exerccio Recomendado 8 para o caso do vencedor ser quem


disser 64.
17. Refaa o exerccio anterior para o conjunto {3, 4, 5, 6}.
18. Mostre que no Exerccio Recomendado 8, se o conjunto fosse A =
{3, 5, 6, 7}, o segundo jogador tem a estratgia que impede o primeiro
jogador de ganhar.
19. Uma bobina de papel tem raio interno 5cm, raio externo 10cm e a espes-
sura do papel 0,01cm. Qual o comprimento da bobina desenrolada?
20. Dividem-se os nmeros naturais em blocos do modo seguinte:
(1), (2, 3) (4, 5, 6) (7, 8, 9, 10) (11.12.13.14) . . . . Em seguida suprimem-
se os blocos que contm um nmero par de elementos, formando-se o
quadro:

18
Progresses Aritmticas Unidade 5

1
4 5 6
11 12 13 14 15
... ... ... ... ... ... ...

Determine:

(a) o primeiro elemento da linha k .

(b) o elemento central da linha k .

(c) a soma dos elementos da linha k .

(d) a soma dos elementos das k primeiras linhas.

21. Prove: se an um polinmio de grau p ento an um polinmio de


grau p 1.
22. Prove o Corolrio 6.
23. Quantos so os termos comuns s progresses aritmticas

(2, 5, 8, 11, . . . , 332) e (7, 12, 17, 22, . . . , 157)?

24. Benjamin comeou a colecionar calendrios em 1979. Hoje, sua coleo


j tem algumas duplicatas - por exemplo, o calendrio de 1985 igual ao
de 1991 - mas ainda no est completa.

(a) Em que ano Benjamim completar sua coleo?

(b) Quando a coleo estiver completa, quantos calendrios diferentes


nela haver?

25. A razo entre as somas dos n primeiros termos de duas progresses arit-
2n + 3
mticas , para todo valor de n. Quanto vale a razo entre seus
4n 1
termos de ordem n?

19
Unidade 5 Textos Complementares

5.7 Textos Complementares

Para Saber Mais PAs de Ordem Superior

De modo geral, uma progresso aritmtica de ordem k (k > 2) uma


sequncia na qual as diferenas entre cada termo e o termo anterior formam
uma progresso aritmtica de ordem k 1.
Exemplo A tabela abaixo mostra uma sequncia (an ) = (n3 n) e suas
diferenas
(an ), (2 an ) = (an ), (3 an ) = (2 an ) etc...

n an an 2 an 3 an
0 0 0 6 6
1 0 6 12 6
2 6 18 18 6
3 24 36 24 6
4 60 60 30 2
5 120 90 2
6 210 2
7 2
Se (an ), como parece, for constante, (2 an ) ser uma progresso aritm-
tica, (an ) ser uma progresso aritmtica de segunda ordem e (an ) ser uma
progresso aritmtica de terceira ordem. Isso verdade, pois

an = n3 n
an = an+1 an = (n + 1)3 (n + 1) [n3 n] = 3n2 + 3n,
2 an = 3(n + 1)2 + 3(n + 1) [3n2 + 3n] = 6n + 6,
3 an = 6(n + 1) + 6 [6n + 6] = 6

e 3 an realmente constante.
Observe que, nesse quadro, a soma de dois elementos lado a lado igual
ao elemento que est embaixo do primeiro desses elementos. Isso nos permite
calcular os elementos que esto assinalados por 2 na tabela acima. Da direita
para a esquerda, eles so iguais a 6, 30 + 6 = 36, 90 + 36 = 126 e 210 + 126 =
336. Portanto, a7 = 336 e este foi o processo mais extico que voc j viu para
calcular a7 = 73 7.

20
Progresses Aritmticas Unidade 5

Proposio Toda sequncia na qual o termo de ordem n um polinmio


em n, do segundo grau, uma progresso aritmtica de segunda ordem e,
reciprocamente, se (an ) uma pregresso aritmtica de segunda ordem ento
(an ) um polinmio de segundo grau em n.
Demonstrao Com efeito, se an = an2 + bn + c, com a 6= 0, temos

an = an+1 an = a(n + 1)2 + b(n + 1) + c (an2 + bn + c) = 2an + (a + b),

que do primeiro grau em n. Pelo que comentamos acima, (an ) uma


progresso aritmtica no-estacionria.
Por outro lado, se (an ) uma progresso aritmtica de segunda ordem,
bn = an = an+1 an uma progresso aritmtica com razo diferente de
zero e

b1 + b2 + b3 + + bn2 + bn1 =
(a2 a1 ) + (a3 a2 ) + (a4 a3 ) + + (an an1 ) + (an+1 an ) =
an+1 a1

um polinmio do segundo grau em n. Em consequncia, an tambm um


polinmio do segundo grau em n.
Observao O resultado anterior ser generalizado mais adiante.

21
Unidade 5 Textos Complementares

Para Saber Mais PA com Termo Geral Polinomial

Com o corolrio acima, podemos generalizar o teorema em [Para Saber


Mais: PAs de Ordem Superior], conforme foi prometido l.
Teorema (an ) uma progresso aritmtica de ordem p, (p > 2), se, e somente
se an um polinmio de grau p em n.

Demonstrao Vamos proceder por induo sobre p.


Para p = 2, o teorema foi provado em [Para Saber Mais: PAs de Ordem
Superior, Proposio].
Suponhamos que o teorema seja verdadeiro para todo p {2, 3, . . . , s}.
Mostraremos que essa armao verdadeira para p = s + 1.
Se (an ) uma progresso aritmtica de ordem s + 1, bn = an = an+1 an
uma progresso aritmtica de ordem s e, pela hiptese da induo, bn um
n
polinmio de grau s em n. Ento
X
bk = an+1 a1 , pelo corolrio do
k=1
Teorema 5, um polinmio de grau s + 1 em n. Se an um polinmio de grau
s + 1 em n, an um polinmio de grau s em n, conforme voc facilmente
vericar. Pela hiptese da induo, (an ) uma progresso aritmtica de
ordem s, ou seja, (an ) uma progresso aritmtica de ordem s + 1.

22
MA12 - Unidade 5
Progressoes Aritmeticas

Paulo Cezar Pinto Carvalho

PROFMAT - SBM

9 de Marco de 2013
Sequencias Numericas

Progressoes Aritmeticas sao casos particulares de sequencias


numericas.
Uma sequencia numerica e uma funcao x : N R.
E comum denotar x(n) por xn e usar (xn ) para representar a
sequencia.

PROFMAT - SBM MA12 - Unidade 5, Progressoes Aritmeticas slide 2/16


Progressoes Aritmeticas

Uma progressao aritmetica e uma sequencia na qual a


diferenca entre cada termo e o termo anterior e constante.
Essa diferenca constante e chamada de razao da progressao e
representada pela letra r .
Mais formalmente: (an ) e uma progressao aritmetica quando
existe um numero real r tal que an+1 = an + r , para todo
n N.

PROFMAT - SBM MA12 - Unidade 5, Progressoes Aritmeticas slide 3/16


Termo Geral

Em uma progressao aritmetica (an )


para avancar um termo, basta somar a razao;
para avancar dois termos, basta somar duas vezes a razao;
e assim por diante;
Por exemplo, a13 = a5 + 8r
De modo geral,
an = a1 + (n 1)r ,
pois, ao passar de a1 para an , avancamos n 1 termos.

PROFMAT - SBM MA12 - Unidade 5, Progressoes Aritmeticas slide 4/16


Exemplo

O cometa Halley visita a Terra a cada 76 anos. Sua ultima


passagem por aqui foi em 1986. Quantas vezes ele visitou a
Terra desde o nascimento de Cristo? Em que ano foi sua
primeira passagem na era crista?
Os anos de passagem do cometa foram 1986, 1910, 1834,... e
formam uma progressao aritmetica de razao 76.
O termo de ordem n dessa progressao e

an = a1 + (n 1)r = 1986 76(n 1) = 2062 76n.

2062
Temos an > 0 quando n < = 27, 13 . . . . Portanto, os
76
termos positivos dessa progressao sao os 27 primeiros.
Logo, ele nos visitou 27 vezes na era crista e sua primeira
passagem na era crista foi no ano a27 = 2062 76 27 = 10.

PROFMAT - SBM MA12 - Unidade 5, Progressoes Aritmeticas slide 5/16


Comecando de a0 ou a1 ?

O preco de um carro novo e de R$ 15 000,00 e diminui de


R$1 000,00 a cada ano de uso. Qual sera o preco com 4 anos
de uso?
Chamando o preco com n anos de uso de an , temos
a0 = 15000 e queremos calcular a4 .
Como a desvalorizacao anual e constante, (an ) e uma
progressao aritmetica.
Logo, a4 = a0 + 4r = 15000 + 4 (1000) = 11000, ou seja,
o preco sera de R$11 000,00.

PROFMAT - SBM MA12 - Unidade 5, Progressoes Aritmeticas slide 6/16


Progressoes aritmeticas e funcoes afins

Em uma progressao aritmetica de razao nao nula, o termo


geral e dado por um polinomio do primeiro grau em n:

an = a1 + (n 1)r = r . n + (a1 r ).

Reciprocamente, se em uma sequencia o termo de ordem n for


dado por um polinomio do primeiro grau em n ela sera uma
progressao aritmetica de razao nao nula.
Com efeito, se xn = an + b, (xn ) e uma progressao aritmetica
na qual r = a e a1 = a + b.

PROFMAT - SBM MA12 - Unidade 5, Progressoes Aritmeticas slide 7/16


O grafico de uma progressao aritmetica

Portanto, uma progressao aritmetica pode ser vista como uma


funcao afim, restrita ao domnio dos numeros naturais.
Em consequencia, o grafico de uma progressao aritmetica e
um conjunto de pontos igualmente espacados sobre uma reta.

PROFMAT - SBM MA12 - Unidade 5, Progressoes Aritmeticas slide 8/16


Soma dos termos de uma progressao aritmetica

Sn = a1 + a2 + . . . + an1 + an
Sn = an + an1 + . . . + a2 + a1

2Sn = (a1 + an ) + (a1 + an ) + . . . + (a1 + an ) + (a1 + an )

(a1 + an )n
Sn = .
2

PROFMAT - SBM MA12 - Unidade 5, Progressoes Aritmeticas slide 9/16


Exemplos

A soma dos n primeiros numeros inteiros e positivos e

n(n + 1) n2 n
1 + 2 + 3 + + n = = + .
2 2 2
A soma dos n primeiros numeros mpares e

(1 + 2n 1)n
1 + 3 + 5 + + (2n 1) = = n2 .
2
Em ambos os casos, a expressao da soma e dada por um
polinomio do segundo grau em n, sem termo independente,

PROFMAT - SBM MA12 - Unidade 5, Progressoes Aritmeticas slide 10/16


Soma dos termos de PAs e funcoes quadraticas

 
(a1 + an )n [a1 + a1 + (n 1)r ]n r r
Sn = = = n2 + a1 n.
2 2 2 2

O termo geral da sequencia (Sn ) das somas dos n primeiros


termos de uma progressao artimetica com r 6= 0 e dado por
um polinomio do segundo grau sem termo independente.
Reciprocamente, todo polinomio do segundo grau em n sem
termo independente e o valor da soma dos n primeiros termos
de alguma progressao aritmetica com r 6= 0.

PROFMAT - SBM MA12 - Unidade 5, Progressoes Aritmeticas slide 11/16


Progressoes Aritmeticas de Segunda Ordem

O Operador (ou de diferenca) para uma sequencia e


definido por:
an = an+1 an

Uma sequencia (an ) e uma progressao aritmetica se e somente


se (an ) e uma sequencia constante.
Dizemos que uma sequencia (an ) e uma progressao aritmetica
de segunda ordem quando (an ) e uma progressao aritmetica
nao constante.
A sequencia (Sn ) da soma dos n primeiros termos de uma
progressao aritmetica e uma progressao aritmetica de segunda
ordem.

PROFMAT - SBM MA12 - Unidade 5, Progressoes Aritmeticas slide 12/16


Teorema

Uma sequencia e uma progressao aritmetica de segunda


ordem se e somente se seu termo geral e dado por um
polinomio do segundo grau.
Se an = an2 + bn + c, entao
an = a(n + 1)2 + b(n + 1) + c an2 bn c = 2an + (2a + b),
que e um polinomio do primeiro grau. Logo, (an ) e uma
P.A. nao constante e (an ) e P.A. de segunda ordem.
Se (an ) e P.A. de segunda ordem, entao (bn ) = (an ) e uma
P.A. nao constante. Mas an = a1 + (b1 + ... + bn1 ). A soma
em parenteses e a soma dos n 1 primeiros termos da P.A.
(bn ); logo tanto esta soma quanto an sao expressos por
polinomios do segundo grau em n.

PROFMAT - SBM MA12 - Unidade 5, Progressoes Aritmeticas slide 13/16


Revisitando a Pizza de Steiner
Qual e o maior numero de partes em que se pode dividir o
plano com n cortes retos?
Numero de cortes (n) Numero maximo de partes (pn )
1 2
2 4
3 7
4 11
No n-esimo corte, n regioes sao acrescentadas. Portanto,
(pn ) e uma P.A. e pn e uma P.A. de segunda ordem.
A expressao de pn e da forma pn = an2 + bn + c. Mas
p(1) = a + b + c = 2
p(2) = 4a + 2b + c = 4
p(3) = 9a + 3b + c = 7
Resolvendo o sistema, encontramos a = 12 , b = 21 e c = 1 e
n2 + n + 2
pn =
2
PROFMAT - SBM MA12 - Unidade 5, Progressoes Aritmeticas slide 14/16
Progressoes Aritmeticas de Ordem Superior

Definicao Uma sequencia (an ) e progressao aritmetica de


ordem k quando an e progressao aritmetica de ordem k 1.
Observacao: definicao por recorrencia

Teorema: Uma sequencia e uma progressao aritmetica de


ordem k se e somente se seu termo geral e um polinomio de
grau k.
Ver demonstracao no livro-texto.

PROFMAT - SBM MA12 - Unidade 5, Progressoes Aritmeticas slide 15/16


Uma aplicacao
Obter uma expressao para a soma Sn = 12 + 22 + . . . + n2
Como Sn = (n + 1)2 e um polinomio de segundo grau, Sn
e uma P.A. de segunda ordem. Portanto Sn e uma P.A. de
terceira ordem e seu termo geral e da forma
Sn = an3 + bn2 + cn + d.
Para calcular os valores de a, b, c e d, basta usar os valores
de Sn para n = 1, 2, 3 e 4.


a+b+c +d =1
8a + 4b + 2c + d = 5


27a + 9b + 3c + d = 14
64a + 16b + 4c + d = 30

1 1 1
Resolvendo: a = , b = , c = , d = 0. Entao
3 2 6
1 1 1 n(n + 1)(2n + 1)
12 + 22 + 32 + + n2 = n3 + n2 + n = .
3 2 6 6
PROFMAT - SBM MA12 - Unidade 5, Progressoes Aritmeticas slide 16/16
Lista de Exerccios
Unidade 5

1. Formam-se n triangulos com palitos, conforme a figura. Qual o numero


de palitos usados para construir n triangulos?

Figura 1:

2. Calcule a soma de todos os inteiros que divididos por 11 dao resto 7 e


estao compreendidos entre 200 e 400.

3. Quanto vale o produto (a)(aq)(aq 2 )(aq 3 ) . . . (aq n1 )?

4. Um quadrado magico de ordem n e uma matriz n n, cujos elementos


sao os inteiros 1, 2, . . . , n2 , sem repetir nenhum, tal que todas as linhas
e todas as colunas tem a mesma soma. O valor dessa soma e chamado
de constante magica. Por exemplo, os quadrados

17 24 1 8 15
1 5 9 8 1 6 23 5 7 14 16

8 3 4 3 5 7 e 4 6 13 20 22

6 7 2 4 9 2 10 12 19 21 3
11 18 25 2 9

sao magicos, com constantes magicas respectivamente iguais a 15, 15


e 65. Alias, os dois ultimos sao hipermagicos, pois as linhas, colunas e
tambem as diagonais tem a mesma soma. Calcule a constante magica
de um quadrado magico de ordem n.

1
5. Suprimindo um dos elementos do conjunto {1, 2, . . . , n}, a media aritmetica
dos elementos restantes e 16,1. Determine o valor de n e qual foi o ele-
mento suprimido.

6. Um bem, cujo valor hoje e de R$ 8000,00, desvaloriza-se de tal forma


que seu valor daqui a 4 anos sera de R$ 2000,00. Supondo que o valor
do bem cai segundo uma linha reta, determine o valor do bem daqui a
3 anos.

7. Prove que a soma de todos os inteiros positivos de n dgitos, n > 2, e


igual ao numero 49499...95500...0, no qual ha n 3 dgitos sublinhados
que sao iguais a 9 e n 2 dgitos sublinhados que sao iguais a 0.

8. Considere um jogo entre duas pessoas com as seguintes regras:


i) Na primeira jogada, o primeiro jogador escolhe um numero no con-
junto A = {1, 2, 3, 4, 5, 6, 7} e diz esse numero.
ii) As pessoas jogam alternadamente.
iii) Cada pessoa ao jogar escolhe um elemento de A, soma-o ao numero
dito pela pessoa anterior e diz a soma.
iv) Ganha quem disser 63.
Qual dos jogadores tem uma estrategia vencedora e qual e essa es-
trategia?

9. Na primeira fase do campeonato brasileiro de futebol, que e dispu-


tado por 24 clubes, quaisquer dois times jogam entre si uma unica vez.
Quantos jogos ha?

10. Qual o numero maximo de regioes em que n retas podem dividir o


plano?

11. Ha dois tipos de anos bissextos: os que sao multiplos de 4 mas nao de
100 e os que sao multiplos de 400.

(a) Quantos sao os anos bissextos entre 1997 e 2401?

(b) Se 1o de janeiro de 1997 foi quarta-feira, que dia sera 1o de janeiro


de 2500?

(c) Escolhido um ano ao acaso, qual a probabilidade dele ser bissexto?

2
12. O numero triangular Tn e definido como a soma dos n primeiros termos
da progressao aritmetica 1, 2, 3, 4, . . .. O numero quadrangular Qn e de-
finido como a soma dos n primeiros termos da progressao aritmetica
1, 3, 5, 7, . . .. Analogamente sao definidos numeros pentagonais, hexa-
gonais, etc. A figura abaixo justifica essa denominacao.
Determine o numero j-gonal de ordem n.

Figura 2:

13. Mostre que ak = bk entao ak bk e constante.


n
X
14. Mostre que ak = an+1 a1 . Use este fato (conhecido como o
k=1
Teorema Fundamental da Somacao) para calcular:
n
X
(a) 3k .
k=1

n
X
(b) k . k!.
k=1

n
X 1
(c) .
k=1
k(k + 1)

3
Solucao da Lista de Exerccios
Unidade 5

1. O aumento de um triangulo causa o aumento de dois palitos. logo,


o numero de palitos constitui uma progressao aritmetica de razao 2.
an = a1 + (n 1)r = 3 + (n 1)2 = 2n + 1.

2. 200 = 11.18 + 2; logo, 205 = 11.18 + 7.


400 = 11.36 + 4 = 11.35 + 15; logo, 392 = 11.35 + 7
As parcelas a somar sao 11.18+7, 11.19+7, 11.20+7, . . . , 11.35+7, que
formam uma progressao aritmetica de razao 11, cujo primeiro termo e
205, cujo ultimo termo e 392 e cujo numero de termos e 35 17 = 18.
(205 + 392)18
A soma vale S = = 5373
2
1+2+3++(n1) n(n1)
3. a.(aq).(aq 2 ).(aq 3 ). . . . .(aq n1 ) = an .q 2 = an q 2

2 2
4. A soma de todos os elementos da matriz e 1 + 2 + + n2 = (n +1)n
2
.
Como a soma de todos os elementos e igual a n vezes a constante
2 2 2
magica, a constante magica vale C = n1 (n +1)n
2
= n(n2+1) .

5. Considerando a menor e a maior das medias que podem ser obtidas,

1 + 2 + + (n 1) 2 + 3 + + n
16.1
n1 (n 1)

n n+2
2
16, 1 2
30, 2 n 32, 2
n so pode valer 31 ou 32.
Chamemos de k o numero suprimido.
Se n = 31,
1 + 2 + + 31 k = 483
496 k 483

1
k = 13
1 + 2 + + 32 k
Se n = 32, = 16, 1
31
1 + + 32 k = 499, 1, o que e absurdo, pois k nao seria inteiro.
Logo, n = 31; o numero suprimido e igual a 13.

6. A desvalorizacao total e de R$ 6.000,00 e a desvalorizacao anual e de


R$ 6.000,00/4 = R$ 1.500,00. Portanto, em tres anos a desvalorizacao
foi de R$ 4.500,00 e o valor do bem sera R$ 8.000,00 - R$ 4.500,00 =
R$ 3.500,00.

7. A soma pedida e a soma de uma progressao aritmetica de razao 1, com


primeiro termo igual a 10n1 e ultimo termo igual a 10n 1.

(10n1 + 10n 1)(10n 10n1 ) 102n 102n2 10n + 10n+1


S= =
2 2
1 2n n1 1 2n2 n
= [10 + 10 ] [10 + 10 ]
2 2
n1 n n4 n+1
1 z }| { z }| { 1 z }| { z }| {
= . 1000 . . . 00 1000 . . . 00 . 1000 . . . 001 1000 . . . 00
2 2
n3 n n3 n1
1 z }| { z }| { z }| { z }| {
= .98 999 . . . 99 1000 . . . 00 = 494 999 . . . 99 55 000 . . . 00
2

8. Quem disser 55 ganha o jogo, pois nao permite ao adversario alcancar


63 e, escolhendo o complemento para 8 do numero escolhido pelo ad-
versario, alcancara o 63.
Analogamente, as posicoes ganhadoras sao 63, 55, 47, 39, 31, 23, 15, 7.
O primeiro jogador tem a estrategia ganhadora: comecar dizendo 7 e, a
partir da, escolher sempre o complemento para 8 do numero escolhido
pelo adversario.

9. O botafogo joga 23 vezes, o Santos joga (sem contar a partida contra


o Botafogo, ja contada) 22 vezes etc. A resposta e 23 + 22 + 21 + +
1 + 0 = (23+0).24
12
= 276.

10. Se ha n retas, a colocacao de mais uma reta cria n + 1 novas regioes.


Portanto, se an e o numero de regioes para n retas, an+1 = an + (n + 1).
Trata-se, portanto, de uma progressao aritmetica de segunda ordem.

2
an = An2 + Bn + C
Como a1 = 2, a2 = 4 e a3 = 7, temos
A+B+C =2
4A + 2B + C = 4
9A + 3B + C = 7
Resolvendo, A = 12 , B = 21 , C = 1
n2 + n + 2
an = .
2
11. (a) Sao multiplos de 4 os anos 2000, 2004, 2008,. . . , 2400.
an = a1 + (n 1)r
2400 = 2000 + (n 1)4
n = 101
Mas 2100, 2200, 2300 nao sao bissextos por serem multiplos de
100, mas nao de 400.
A resposta e 98.
(b) Um ano nao-bissexto e formado por 52 semanas e 1 dia e um ano
bissexto e formado por 52 semanas e 2 dias. Se um ano nao-
bissexto comeca numa segunda-feira, por exemplo, o ano seguinte
comecara numa terca; se for bissexto, o ano seguinte comecara
numa quarta.
De 1997 a 2500 sao multiplos de 4 os anos 2000, 2004, 2008,. . . ,
2496, num total de 125 anos. Mas 2100, 2200 e 2300 nao sao bis-
sextos por serem multiplos de 100, mas nao de 400. Ha, portanto,
122 anos bissextos.
Se 1997 comecou numa quarta-feira, 2500 comecara (25001997)+
122 = 625 dias de semana depois. Como 625 = 7x89 + 2 o ano
2500 comecara numa sexta-feira.
(c) Em cada bloco de 400 anos ha 100 anos que sao multiplos de 4 e,
destes, 3 nao sao bissextos por serem multiplos de 100, mas nao
97
de 400. A resposta e = 0, 2425.
400
12. 1 + (j 1) + (2j 3) + + [1 + (n 1).(j 2)] =
1 + 1 + (n 1).(j 2) n[(j 2)n j + 4]
= .n = .
2 2
3
13. ak = bk ak+1 ak = b + k + 1 kk ak+1 bk+1 = ak bk
para todo k e ak bk e constante.
n
X
14. ak = (a2 a1 ) + (a3 a2 ) + . . . + (an+1 a1 ) = an+1 a1 (os
k=1
demais termos se cancelam).

1
Pn 1
Pn 3n+1 3
3k = k
P
(a) 2 k=1 3 (3 1) = 2 k=1 3k = .
4
n
X n
X n
X n
X
(b) k.k! = [(k + 1).k! k!] = [(k + 1) k] = k! =
k=1 k=1 k=1 k=1
(n + 1)! 1.
n n   n
X 1 1 X 1 X 1
(c) = = = =
k(k + 1) k=1 k
k=1 
k+1 k=1
k

1 n
= 1 =
n+1 n+1

4
6

Progresses Geomtricas

Sumrio
6.1 Introduo . . . . . . . . . . . . . . . . . . . . . . . 2

6.2 Primeiros Exemplos . . . . . . . . . . . . . . . . . . 3

6.3 A Frmula das Taxas Equivalentes . . . . . . . . . . 8

6.4 A Soma dos Termos de uma Progresso Geomtrica 9

6.5 Exerccios Recomendados . . . . . . . . . . . . . . . 11

6.6 Exerccios Suplementares . . . . . . . . . . . . . . . 12

6.7 Textos Complementares . . . . . . . . . . . . . . . . 18

1
Unidade 6 Introduo

6.1 Introduo

Ao utilizarmos progresses aritmticas para modelar problemas de juros simples,

obtemos uma situao como segue. Considere um capital de R$ 10.000, 00


aplicado a uma taxa de juros mensal de 2%:

Ms Valor Inicial Juros Valor Final

1 10.000 10.000 2% = 200 10.200


2 10.200 10.000 2% = 200 10.400
3 10.400 10.000 2% = 200 10.600
4 10.600 10.000 2% = 200 10.800
5 10.800 10.000 2% = 200 11.000
6 11.000 10.000 2% = 200 11.200

Voc j viu algum aplicar dinheiro dessa forma? Pense na sua caderneta de

poupana. O mais verossmil que o juro incida sobre juros, pois j no segundo

mes o nosso capital no mais R$ 10.000,00, mas R$ 10.200,00; logo esse

capital que deve ser remunerado no segundo ms. Obtemos assim uma nova

tabela (com arredondamento na segunda casa decimal):

Ms Valor Inicial Juros Valor Final

1 10.000 10.000 2% = 200 10.200


2 10.200 10.200 2% = 204 10.404
3 10.404 10.404 2% = 208, 08 10.612, 08
4 10.612, 08 10.612, 08 2% = 212, 24 10.824, 32
5 10.824, 32 10.824, 32 2% = 216, 49 11.040, 81
6 11.040, 81 11.040, 81 2% = 220, 82 11.260, 92

O que se nota nessa tabela que, a menos das aproximaes feitas, o quociente

entre o nosso capital em um ms e o do ms anterior constante igual a 1, 02.


Isto motiva a seguinte denio:

Definio 1 Uma Progresso Geomtrica (PG) uma sequncia numrica na qual a taxa
de crescimento (ou decrescimento) de cada termo para o seguinte sempre a

mesma.

2
Progresses Geomtricas Unidade 6

Portanto, de acordo com o problema acima, as PGs modelam fenmenos

como o aumento de um capital aplicado a uma taxa anual prexada. Da mesma

forma, as PGs modelam o crescimento de uma populao a uma taxa anual xa

ou, ainda, o decaimento da radiao emitida por um material radioativo.

Assim, as PGs aparecem muito frequentemente no s nas aplicaes, mas

tambm, em vrios contextos matemticos e por isso, certamente, so muito

mais interessantes do que as progresses aritmticas.

Nessa unidade, voc encontrar um grande leque de problemas em cuja soluo

intervm as PGs. Alguns desses problemas so resolvidos usando a muito til

frmula das taxas equivalentes, como no Exemplo 10, onde se pede para deter-

minar quanto a populao de um pas crescer em n anos se ela cresce a uma

taxa de 2% ao ano.

Em seguida, ser deduzida a frmula da soma dos n primeiros termos de uma

PG e o clculo do limite da soma dos termos de uma PG decrescente.

A unidade se encerra com mais uma tcnica de somao, a frmula da so-

mao por partes, que aumentar o nosso arsenal de tcnicas de somao de

sequncias, permitindo, em particular, somar sequncias cujos termos so ob-

tidos multiplicando termos de uma PA com termos de uma PG. A semelhana

dessa frmula com a frmula de integrao por partes do Clculo Integral no

mera coincidncia, pois a somao corresponde discretizao da integrao.

6.2 Primeiros Exemplos

Um problema interessante, que costuma deixar os alunos intrigados e os

professores desconados, o problema a seguir, adaptado de um problema do

exame nacional da MAA (Mathematical Association of America).

Uma pessoa, comeando com R$ 64,00, faz seis apostas consecutivas, em Exemplo 1
cada uma das quais arrisca perder ou ganhar a metade do que possui na ocasio.

Se ela ganha trs e perde trs dessas apostas, pode-se armar que ela:

A) Ganha dinheiro.

B) No ganha dinheiro nem perde dinheiro.

C) Perde R$ 27,00.

3
Unidade 6 Primeiros Exemplos

D) Perde R$ 37,00.

E) Ganha ou perde dinheiro, dependendo da ordem em que ocorreram suas

vitrias e derrotas.

Comentrio. Em geral os alunos escolhem uma ordem para ver o que acon-

teceu; alis, essa at uma boa estratgia. Por exemplo, se ela vence as trs

primeiras apostas e perde as ltimas trs, o seu capital evoluiu de acordo com

o esquema: 64 96 144 216 108 54 27.


Se ela comeou com R$ 64,00 e terminou com R$ 27,00, ela perdeu R$

37,00. J houve um progresso. Sabemos agora que a resposta s poder ser

(D) ou (E).

Em seguida os alunos costumam experimentar uma outra ordem; por exem-

plo, ganhando e perdendo alternadamente. Obtm-se: 64 96 48 72


36 54 27. Nessa ordem a pessoa tambm perdeu R$ 37,00.

Em seguida, experimentam outra ordem, torcendo para que a pessoa no ter-

mine com R$ 27,00, o que permitiria concluir que a resposta (E). Infelizmente

encontram que a pessoa novamente termina com R$ 27,00 e permanecem na

dvida. Alguns se dispem a tentar todas as ordens possveis, mas logo desistem

ao perceber que h 20 ordens possveis.

Soluo A melhor maneira de abordar problemas nos quais h uma grandeza

varivel, da qual conhecida a taxa (porcentagem) de variao, concentrar

a ateno, no na taxa de variao da grandeza, e sim no valor da grandeza

depois da variao.

Neste problema, devemos pensar assim: Cada vez que ganha, o capital
1 1 3
aumenta (ou seja, 50%) e passa a valer 1+ = do que valia; cada vez
2 2 2
1 1 1
que perde, o capital diminui de (ou seja, 50%) e passa a valer 1 = do
2 2 2
que valia.

Pensando assim, ca claro que se a pessoa vence as trs primeiras apostas e

perde as trs ltimas, a evoluo de seu capital se d de acordo com o esquema:

3 3 3 3 3 3 3 3 3 1
64 64 64 64 64
2 2 2 2 2 2 2 2 2 2
3 3 3 1 1 3 3 3 1 1 1
64 64
2 2 2 2 2 2 2 2 2 2 2

4
Progresses Geomtricas Unidade 6

3 3 3 1 1 1
Ela termina com 64 = 27 Reais. Alm disso, ca claro
2 2 2 2 2 2
tambm que se as vitrias e derrotas tivessem ocorrido em outra ordem, isso

apenas mudaria a ordem dos fatores, sem alterar o produto, e a pessoa tambm

terminaria com R$ 27,00.

Se ela comeou com R$ 64,00 e terminou com R$ 27,00 ela perdeu R$

37,00. A resposta (D).

Aumentando de 20% o raio da base de um cilindro e diminuindo de 30% Exemplo 2


sua altura, de quanto variar seu volume?

O volume diretamente proporcional ao quadrado do raio e altura. Por-

tanto, V = kr2 h, onde k a constante de proporcionalidade. Sabemos que

k = , mas isso irrelevante para o problema.

Depois da variao, os novos valores de r e de h sero r0 = 1, 2r e h0 = 0, 7h,


pois o aumento de 20% passa a valer 120% = 1, 2 do que valia e o que diminui
de 30% passa a valer 70% = 0, 7 do que valia.

O novo volume ser

V 0 = k(1, 2r)2 0, 7h = 1, 008 k r2 h = 100, 8%V.

O volume aumenta de 0, 8%.

O que deve ter cado claro nesses exemplos que se uma grandeza tem

taxa de crescimento igual a i, cada valor da grandeza igual a (1 + i) vezes

o valor anterior. Progresses geomtricas so sequncias nas quais a taxa de

crescimento i de cada termo para o seguinte sempre a mesma.

A populao de um pas hoje igual a P0 e cresce 2% ao ano. Qual ser Exemplo 3


a populao desse pas daqui a n anos?

Se a populao cresce 2% ao ano, em cada ano a populao de 102%


da populao do ano anterior. Portanto, a cada ano que passa, a populao

sofre uma multiplicao de 102% = 1, 02. Depois de n anos, a populao ser


n
P0 1, 02 .

5
Unidade 6 Primeiros Exemplos

Exemplo 4 A torcida de certo clube hoje igual a P0 e decresce 5% ao ano. Qual ser

a torcida desse clube daqui a n anos?

Se a torcida decresce 5% ao ano, em cada ano a torcida de 95% da torcida


anterior. Portanto, a cada ano que passa, a torcida sofre uma multiplicao por

95% = 0, 95. Depois de n anos, a torcida ser P0 0, 95n .

Exemplo 5 A sequncia (1, 2, 4, 8, 16, 32, . . .) um exemplo de uma progresso geo-

mtrica. Aqui a taxa de crescimento de cada termo para o seguinte de 100%,


o que faz com que cada termo seja igual a 200% do termo anterior.

Exemplo 6 A sequncia (1000, 800, 640, 512, . . .) um exemplo de uma progresso


geomtrica. Aqui, cada termo 80% do termo anterior. A taxa de crescimento

de cada termo para o seguinte de 20%.

claro ento que numa progresso geomtrica cada termo igual ao anterior

multiplicado por 1 + i, onde i a taxa de crescimento dos termos. Chamamos

1+i de razo da progresso e representamos a razo por q.


Portanto, uma progresso geomtrica uma sequncia na qual constante
o quociente da diviso de cada termo pelo termo anterior. Esse quociente

chamado de razo da progresso e representado pela letra q. A razo q de

uma progresso geomtrica simplesmente o valor de 1 + i, onde i a taxa de

crescimento constante de cada termo para o seguinte.

Exemplo 7 As sequncias (2, 6, 18, 54, . . .)


(128, 32, 8, 2, . . .) so progresses geo-
e
1
mtricas cujas razes valem respectivamente q1 = 3 e q2 = . Suas taxas de
4
3
crescimento so respectivamente i1 = 2 = 200% e i2 = = 75%, pois
4
q = 1 + i.

Em uma progresso geomtrica (a1 , a2 , a3 , . . .), para avanar um termo

basta multiplicar pela razo; para avanar dois termos, basta multiplicar duas

vezes pela razo, e assim por diante.

Por exemplo, a13 = a5 q 8 , pois avanamos 8 termos ao passar de a5 para a13 ;


a17
a12 = a7 q 5 , pois avanamos 5 termos ao passar de a7 para a12 ; a4 = , pois
q 13

6
Progresses Geomtricas Unidade 6

ao passar de a17 para a4 , retrocedemos 13 termos; de modo geral, an = a1 q n1 ,


pois, ao passar de a1 para an , avanamos n 1 termos.
Em muitos casos mais natural numerar os termos a partir de zero, como

foi feito nos Exemplos 3 e 4; nesse caso, an = a0 q n , pois avanamos n termos

ao passar de a0 para an .

Em uma progresso geomtrica, o quinto termo vale 5 e o oitavo termo Exemplo 8


vale 135. Quanto vale o stimo termo dessa progresso?

Temos a8 = a5 q 3 , pois ao passar do quinto termo para o oitavo, avanamos


3 2 2
3 termos. Logo, 135 = 5q e q = 3. Analogamente, a7 = a5 q = 5 . 3 = 45.

O stimo termo vale 45.

Como em uma progresso geomtrica an = a0 q n , a funo que associa a

cada natural n o valor de an simplesmente a restrio aos naturais da funo


x
exponencial a(x) = a(0)q . Portanto, pensando em uma progresso geomtrica

como uma funo que associa a cada nmero natural n o valor an , o grco

dessa funo formado por uma sequncia de pontos pertencentes ao grco

de uma funo exponencial.

Figura 6.1: Grco de uma PG

Qual a razo da progresso geomtrica que se obtm inserindo 3 termos Exemplo 9


entre os nmeros 30 e 480?

Temos a1 = 30 e a5 = 480. Como a5 = a1 q 4 , 480 = 30q 4 , q 4 = 16 e

q = 3.

7
Unidade 6 A Frmula das Taxas Equivalentes

Um resultado importante a frmula que relaciona taxas de crescimento

referidas a perodos de tempo diversos. o que abordaremos na prxima seo.

6.3 A Frmula das Taxas Equivalentes

Lema 2 Se I a taxa de crescimento de uma grandeza relativamente ao perodo de


tempo T e i a taxa de crescimento relativamente ao perodo t, e se T = nt,
n
ento 1 + I = (1 + i) .

Demonstrao Seja G0 o valor inicial da grandeza. Aps um perodo de tempo T, o


1
valor da grandeza ser G0 (1 + I) . Como um perodo de tempo T equivale
a n perodos de tempo iguais a t, o valor da grandeza ser tambm igual a

G0 (1 + i)n . Logo, G0 (1 + i)1 = G0 (1 + i)n e 1 + I = (1 + i)n .

Exemplo 10 2% ao ano, quanto crescer em 25 anos?


Se a populao de um pas cresce

Temos i = 2% = 0, 02 e n = 25. Da, 1 + I = (1 + i) = (1 + 0, 02) 5


n 2
=

1, 6406 e I = 0, 6406 = 64, 06%.

Exemplo 11 Uma bomba de vcuo retira, em cada suco, 2% do gs existente em certo


recipiente. Depois de 50 suces, quanto restar do gs inicialmente existente?

Temos i = 2% = 0, 02 e n = 50. Da, 1+I = (1+i)n = (10, 02)5 0 =



0, 3642 e I = 0, 6358 = 63, 58%. A quantidade de gs diminuir de aproxi-
madamente 63, 58%. Restaro aproximadamente 36, 42% do gs inicialmente

existente.

Outro resultado importante a frmula que fornece a soma dos n primeiros


termos de uma progresso geomtrica. Isto fornecido na seo seguinte.

8
Progresses Geomtricas Unidade 6

6.4 A Soma dos Termos de uma Progresso

Geomtrica

A soma nos n primeiros termos de uma progresso geomtrica (an ) de Lema 3


n
1q
razo q 6= 1, Sn = a1 .
1q

Sn = a1 + a2 + a3 + + an1 + an . Multiplicando por q , obtemos Demonstrao


qSn = a2 + a3 + a4 + + an + an+1 .
n
Subtraindo, temos Sn qSn = a1 an+1 , isto , Sn (1 q) = a1 a1 q e,
n
1q
nalmente, Sn = a1 .
1q

Diz a lenda que o inventor do xadrez pediu como recompensa 1 gro de Exemplo 12
trigo pela primeira casa, 2 gros pela segunda, 4 pela terceira e assim por diante,

sempre dobrando a quantidade a cada casa nova. Como o tabuleiro de xadrez

tem 64 casas, o nmero de gros pedidos pelo inventor do jogo a soma dos

64 primeiros termos da progresso geomtrica 1, 2, 4, . . .. O valor dessa soma

1 qn 1 264
Sn = a1 =1 = 264 1.
1q 12

Calculando, obtemos um estupendo nmero de dgitos:

18 446 744 073 709 551 615.

Nas progresses geomtricas em que |q| < 1, a soma dos n primeiros termos
tem um limite nito quando n . Como nesse caso lim q n = 0 temos
n

10
lim Sn = a1 ,
n 1q
isto ,
a1
lim Sn = .
n 1q

9
Unidade 6 A Soma dos Termos de uma Progresso Geomtrica

Exemplo 13 O limite da soma 0, 3 + 0, 03 + 0, 003 + . . . quando o nmero de parcelas


0, 3 1
tende a innito igual a = . O resultado intuitivo pois somando um
1 0, 1 3
nmero muito grande de termos da progresso encontraremos aproximadamente
1
a dzima peridica 0, 33333 = .
3

Exemplo 14 Calcule o limite da soma da progresso geomtrica

1 1 1 1
+ + + + ....
2 4 8 16
1
q1
Temos que lim Sn = = 2 1 = 1.
n 1q 1 2
O resultado admite uma interessante parfrase. Suponha que Salvador deva
1
correr 1 km. Inicialmente ele corre metade dessa distncia, isto , km; em
2
1
seguida ele corre metade da distncia que falta, isto , km; depois, metade
4
1
da distncia restante, isto , km, e assim por diante.
8
Depois de n etapas, Salvador ter corrido

1 1 1 1
+ + + + n km.
2 4 8 2
Se n for grande, essa soma ser aproximadamente igual a 1 km.

n
X
O teorema da somao, ak = an+1 a1 , tambm nos permitiria deter-
k=1
minar o valor da soma dos n primeiros termos de uma progresso geomtrica.

Supondo q 6= 1 e observando que q k1 = q k q k1 = q k1 (q 1), temos

n n n
X X
k1 a1 X k1
a1 + a2 + a3 + + an = ak = a1 q = q
k=1 k=1
q 1 k=1
a1 1 qn
= (q n+11 q 0 ) = a1 .
q1 1q

+ Para Saber Mais - A Frmula de Somao por Partes - Clique para ler

+ Na Sala de Aula - Sobre o Ensino de Progresses - Clique para ler

10
Progresses Geomtricas Unidade 6

6.5 Exerccios Recomendados

1. Aumentos sucessivos de 10% e 20% equivalem a um aumento nico de

quanto?

2. Aumentando sua velocidade em 60%, de quanto voc diminui o tempo de

viagem?

3. Um decrescimento mensal de 5% gera um decrescimento anual de quanto?

4. Mantida constante a temperatura, a presso de um gs perfeito inver-

samente proporcional a seu volume. De quanto aumenta a presso quando

reduzimos de 20% o volume?

5. Um carro novo custa R$ 18 000,00 e, com 4 anos de uso, vale R$ 12 000,00.

Supondo que o valor decresa a uma taxa anual constante, determine o valor

do carro com 1 ano de uso.

6. Determine trs nmeros em progresso geomtrica, conhecendo sua soma

19 e a soma de seus quadrados 133.

7. Nmero perfeito aquele que igual a metade da soma dos seus divisores

positivos. Por exemplo, 6 perfeito pois a soma dos seus divisores 1+2+
p p1 p
3 + 6 = 12. Prove que, se 2 1 um nmero primo, ento 2 . (2 1)
um nmero perfeito.

8. Calcule o valor da soma de n parcelas 1 + 11 + + 111 . . . 1.


9. Um garrafo contm p litros de vinho. Retira-se um litro de vinho do

garrafo e acrescenta-se um litro de gua, obtendo-se uma mistura homognea;

retira-se, a seguir um litro da mistura e acrescenta-se um litro de gua e assim

por diante. Qual a quantidade de vinho que restar no garrafo aps n dessas

operaes?

10. Larga-se uma bola de uma altura de 5cm. Aps cada choque com o solo,

ela recupera apenas 4/9 da altura anterior. Determine:

a) A distncia total percorrida pela bola.

b) O tempo gasto pela bola at parar.

11. Uma faculdade recebe todos os anos 300 alunos novos no primeiro semestre

e 200 alunos novos no segundo semestre. 30% dos alunos so reprovados no

11
Unidade 6 Exerccios Suplementares

primeiro perodo e repetem o perodo no semestre seguinte. Sendo an e bn ,


respectivamente, o nmero de alunos do primeiro perodo no primeiro e no

segundo semestres do ano n, calcule lim an e lim bn .


12. Se (an ) uma progresso geomtrica de termos positivos, prove que (bn )
denida por bn = log an uma progresso aritmtica.

13. Se (an ) uma progresso aritmtica, prove que (bn ) denida por bn = ean
uma progresso geomtrica.
" #
1 2
14. Seja A= . Determine An .
2 4

6.6 Exerccios Suplementares

1. Descontos sucessivos de 10% e 20% equivalem a um desconto nico de

quanto?

2. Um aumento de 10% seguido de um desconto de 20% equivale a um desconto


nico de quanto?

3. O perodo de um pndulo simples diretamente proporcional raiz qua-

drada do seu comprimento. De quanto devemos aumentar o comprimento para

aumentar de 20% o perodo?

4. Se a base de um retngulo aumenta de 10% e a altura diminui de 10%, de

quanto aumenta a rea?

5. Os lados de um tringulo retngulo formam uma progresso geomtrica

crescente. Determine a razo dessa progresso.

6. Os lados de um tringulo esto em progresso geomtrica. Entre que valores

pode variar a razo?



3

6
7. Qual o quarto termo da progresso geomtrica 2, 2, 2,...?
8. A soma de trs nmeros em progresso geomtrica 19. Subtraindo-se 1

ao primeiro, eles passam a formar um progresso aritmtica. Calcule-os.

9. Quatro nmeros so tais que os trs primeiros formam uma progresso

aritmtica de razo 6, os trs ltimos uma progresso geomtrica e o primeiro

nmero igual ao quarto. Determine-os.

12
Progresses Geomtricas Unidade 6

10. Mostre que o nmero 444 . . . 488 . . . 89, formado por n dgitos iguais a 4,

n1 dgitos iguais a 8 e um dgito igual a 9, um quadrado perfeito. Determine

sua raiz quadrada.

11. A espessura de uma folha de estanho 0,1mm. Forma-se uma pilha

de folhas colocando-se uma folha na primeira vez e, em cada uma das vezes

seguintes, tantas quantas j houveram sido colocadas anteriormente. Depois

de 33 dessas operaes, a altura da pilha ser, aproximadamente:

a) a altura de um poste de luz;

b) a altura de um prdio de 40 andares;

c) o comprimento da praia de Copacabana;

d) a distncia Rio-So Paulo;

e) o comprimento do equador terrestre.

12. Calcule a soma dos divisores de 12.600 que sejam:

a) positivos;

b) mpares e positivos.

13. Determine as geratrizes das dzimas peridicas:

a) 0, 141414141 . . . b) 0, 345454545 . . .
c) 0, 999999999 . . . d) 1, 711111111 . . .
14. Determine os limites das somas abaixo:
2 2
a) 2+ + +
3 9
1 2 1 2 1 2
b) + 2 + 3 + 4 + 5 + 6 +
7 7 7 7 7 7
1 3 5 7 9
c) + + + + +
2 4 8 16 32
2 3
d) 1 + 2x + 3x + 4x + . . ., 1 < x < 1;
1 1 1 1
e) 1 +
2 4 8 16
15. Na gura abaixo temos uma linha poligonal, de lados ora perpendiculares a

AB , ora perpendiculares a AC . Sendo a e b, respectivamente, os dois primeiros


lados da poligonal, pede-se determinar:

a) o comprimento da mesma;

b) o comprimento do n-simo lado da poligonal.

13
Unidade 6 Exerccios Suplementares

Figura 6.2: Poligonal

16. Na gura abaixo temos uma espiral formada por semicrculos cujos centros

pertencem ao eixo das abscissas. Se o raio do primeiro semicrculo igual a 1

e o raio de cada semicrculo igual metade do raio do semicrculo anterior,

determine:

a) o comprimento da espiral;

b) a abscissa do ponto P, ponto assinttico da espiral.

Figura 6.3: Espiral

17. Na gura abaixo temos uma sequncia de crculos tangentes a duas retas.

O raio do primeiro crculo 1 e o raio do segundo r < 1. cada crculo

tangencia externamente o crculo anterior. Determine a soma dos raios dos n


primeiros crculos.

18. Seja Sn a soma das reas dos n primeiros quadrados obtidos a partir de

um quadrado Q1 de lado 1 pelo seguinte processo: os vrtices do quadrado

Qn+1 so os pontos mdios dos lados de Qn . Determine quais das armaes

abaixo so verdadeiras:

14
Progresses Geomtricas Unidade 6

Figura 6.4: Sequncia de crculos

1) possvel escolher Sn de modo que Sn > 1, 9;


2) possvel escolher Sn de modo que Sn > 2;
3) possvel escolher Sn de modo que Sn > 2, 1;
4) possvel escolher Sn de modo que Sn = 2;
5) possvel escolher Sn de modo que Sn = 1, 75.
19. Sendo x e y positivos, calcule:

s r s r

q q
a) x x x x... b) x y x y...

20. Comeando com um segmento de tamanho 1, dividimo-lo em trs partes

iguais e retiramos o interior da parte central, obtendo dois segmentos de com-

primento 1/3. Repetimos agora essa operao com cada um desses segmentos

e assim por diante. Sendo Sn a soma dos comprimentos dos intervalos que

restaram depois de n dessas operaes, determine:

a) O valor de Sn ;
b) O valor de lim Sn ;
c) Certo livro, muito citado em aulas de anlise de erros de livros didticos,

arma que, ao nal, o conjunto dos pontos no retirados vazio. Isso verdade?

21. O rdio-226 tem meia-vida (perodo de tempo em que metade da massa

inicialmente presente se desintegra) de 1600 anos. A taxa de variao da massa

constante. Em quanto tempo a tera parte da massa inicialmente presente se

desintegrar?

15
Unidade 6 Exerccios Suplementares

22. Sejam a = 111 . . . 1 (n dgitos iguais a 1) e b = 100 . . . 05 (n 1 dgitos


iguais a 0). Prove que ab + 1 um quadrado perfeito e determine sua raiz

quadrada.

23. A curva de Koch obtida em estgios pelo processo seguinte:

i) No estgio 0, ela um tringulo equiltero de lado 1.

ii) O estgio n+1 obtido a partir do estgio n, dividindo cada lado em

trs partes iguais, construindo externamente sobre a parte central um tringulo

equiltero e suprimindo ento a parte central (ver gura abaixo). Sendo Pn e

An respectivamente o permetro e a rea do n-simo estgio da curva de Koch,


determine:

a) Pn b) An c) lim Pn d) lim An .

Figura 6.5: A curva de Koch

1
24. Pitgoras , que estudou a gerao dos sons, observou que duas cordas

vibrantes, cujos comprimentos estivessem na razo de 1 para 2, soariam em

unssono. Hoje sabemos que a razo das frequncias dos sons emitidos por

essas cordas seria a razo inversa dos seus comprimentos, isto , de 2 para 1 e

que duas cordas vibram em unssono se e s se a razo de seus comprimentos

uma potncia inteira de 2.

A frequncia da nota l-padro (o l central do piano) 440Hz e a frequncia

do l seguinte, mais agudo, 880Hz (Hz a abreviatura de hertz, unidade de

frequncia que signica ciclo por segundo).

1 Pitgoras, matemtico de Samos, cerca de cinco sculos e meio antes de Cristo.

16
Progresses Geomtricas Unidade 6

A escala musical ocidental (de J.S.Bach pra c), dita cromtica, divide esse

intervalo em doze semitons iguais, isto , tais que a razo das frequncias de

notas consecutivas constante.

Sabendo que essas notas so

L - L# - SI - D - D# - R - R# - MI - F - F# - SOL - SOL# - L

determine:

a) a frequncia desse d, primeiro d seguinte ao l padro;

b) a frequncia do sinal de discar de um telefone, que o primeiro sol anterior

ao l padro;

c) a nota cuja frequncia 186Hz.

25. A lei de Weber (Ernest Heinrich Weber; 1795-1878; siologista alemo),

para resposta de seres humanos a estmulos fsicos, declara que diferenas mar-

cantes na resposta a um estmulo ocorrem para variaes de intensidade do

estmulo proporcionais ao prprio estmulo. Por exemplo, um homem, que sai

de um ambiente iluminado para outro, s percebe uma variao da luminosidade

se esta for superior a 2%; s distingue entre solues salinas se a variao da

salinidade for superior a 25%, etc...

Fechner (Gustav Theodor Fechner; 1801-1887; fsico e lsofo alemo) pro-

pe um mtodo de construo de escalas baseado na lei de Weber: props que

enquanto os estmulos variassem em progresso geomtrica, as medidas das

respostas variassem em progresso aritmtica.

a) Mostre que numa escala de Fechner, as medidas da resposta y e do estmulo

x se relacionam por y = a + b log x.


b) Uma das mais conhecidas escalas de Fechner a que mede a sensao de

rudo. Ela denida por L = 120 + 10 log10 l, onde L a medida da sensao


2
d e rudo em decibis (dB) e l a intensidade sonora, medida em W/m . Duas

bandas de heavy metal provocam um rudo quantos decibis acima do rudo

provocado por uma banda?

26. Determine o valor de:


n
X k2 X
a) b) k . 2k
k=1
2k k=1

17
Unidade 6 Textos Complementares

6.7 Textos Complementares

Na Sala de Aula Sobre o Ensino de Progresses

1. No encha a cabea de seus alunos com casos particulares desnecessrios.

Isso s serve para obscurecer as ideias gerais e acaba dicultando as coisas.

Saber que, numa progresso aritmtica, cada termo a mdia aritmtica entre

seu antecedente e seu consequente no s no substitui, ou pelo menos no

substitui de modo eciente, o conhecimento de que uma progresso aritmtica

uma sequncia na qual a diferena entre cada termo e o termo anterior

constante, como uma consequncia imediata disso. Realmente, se x, y , z ,


esto em progresso aritmtica, y x = z y. Da, quem se interessar em
x+z
calcular y obter y= .
2
Do mesmo modo so conhecimentos desnecessrios:

Em uma progresso aritmtica com um nmero mpar de termos, o termo

central a mdia aritmtica dos extremos.

Em uma progresso aritmtica, a soma de dois termos equidistantes dos

extremos igual a soma dos extremos.

Em uma progresso geomtrica cada termo a mdia geomtrica entre seu an-
tecedente e seu consequente. (Seria isso verdadeiro para a progresso 1, 2, 4?)

Em uma progresso geomtrica com um nmero mpar de termos, o termo


central a mdia geomtrica dos extremos (Seria isso verdadeiro para a pro-

gresso 1, 2, 4?)
Em uma progresso geomtrica, o produto de dois termos equidistantes dos

extremos igual ao produto dos extremos.

2. Na maioria dos livros se encontram as frmulas an = a1 + (n 1)r, para


n1
progresses aritmticas e an = a1 q , para progresses geomtricas. Nada

contra essas frmulas, j que usualmente o que se conhece de uma progresso

so o primeiro termo e a razo.

Entretanto bom lembrar que o conhecimento apenas dessas frmulas cos-

tuma atrapalhar muitos alunos quando a progresso comea em a0 . cer-

tamente mais eciente saber que para avanar um termo basta somar r ou

multiplicar por q, para avanar dois termos basta somar 2r ou multiplicar q 2 ,


etc... Assim, facilmente se conclui que an = a0 + nr e an = a1 + (n 1)r,

18
Progresses Geomtricas Unidade 6

nas progresses aritmticas, e que an = a0 q n e an = a1 q n1 , nas progresses

geomtricas.

3. Em alguns livros se encontram, alm da frmulaan = a1 +(n1)r, frmulas


an a1 an a1
como a1 = an (n 1)r, r = , n = 1+ , supostamente para
n1 r
facilitar o clculo. Depois nos queixamos que os alunos no sabem resolver

equaes do primeiro grau!

Mais cedo ou mais tarde, aparecer um livro com uma frmula para o clculo
an a1
do 1: 1=n .
r
4. Alguns livros chegam ao cmulo de trazerem duas verses da (desnecessria)
an a1 an+2 a1
frmula para o clculo de r: r = e r = , a segunda para ser
n1 n+1
usada quando a progresso tiver n + 2 termos, isto , dois termos extremos e
mais n termos entre eles, como no Exemplo 4 da Unidade 5.

5. Alguns livros trazem uma frmula para o clculo do produto dos n primeiros

termos de uma progresso geomtrica, Pn = ( a1 an )n .
Em primeiro lugar, essa frmula est errada. Por ela, o produto dos trs
3 3
primeiros termos da progresso 1, 2, 4, . . . seria ( 1 . 4) = 2 = 8.
2 n
Em segundo lugar, se corrigirmos essa frmula obteremos Pn = (a1 an ) e,

nas progresses cujos termos no so todos positivos, teremos algum trabalho

em descobrir se Pn = (a1 an )n ou se Pn = (a1 an )n .


Em terceiro lugar, no h o menor interesse, prtico ou terico, em deter-

minar o produto dos termos de uma progresso geomtrica.

Em quarto lugar, muito simples determinar o produto dos termos de uma

progresso geomtrica. Com efeito, isso j foi feito no Exerccio 3 da Unidade

5.

6. Moderao nos problemas. Problemas em que so dados a soma do 24O


termo com o 47O e pedida a diferena entre o 36O e o 11O no aparecem na

vida real, no so interessantes e no desenvolvem o raciocnio. Uma pergunta

que devemos sempre nos fazer a seguinte: Se meu professor de Matemtica

tivesse feito estes problemas, eu teria gostado de Matemtica?

7. Tenha sempre em mente que uma progresso geomtrica uma sequn-

cia na qual a taxa de crescimento de cada termo para o seguinte sempre

a mesma e esse instrumento matemtico foi criado para descrever grandezas

19
Unidade 6 Textos Complementares

que variam com taxa de crescimento constante. absurdo, mas infelizmente

comum, ensinar progresses geomtricas e no relacion-las idia de taxa de

crescimento.

8. A melhor maneira de resolver problemas com progresses com um nmero

pequeno de termos escrev-las e esquecer completamente as frmulas para

calcular termos e somas de termos conforme zemos nos Exemplos 7 e 8 da

Unidade 5.

Entretanto, ao contrrio do que ocorria com as progresses aritmticas, no

h nenhuma vantagem, ao escrever progresses geomtricas, em comear pelo


x
termo central. Chamar trs nmeros em progresso geomtrica de , x, xq
q
em vez de cham-los de x, xq , xq 2 , s serve para criar desnecessariamente

denominadores e complicar as contas.

9. Calculadoras so indispensveis para a resoluo de quase todos os problemas

de progresses geomtrica da vida real.

10. Se voc ensina exponenciais e logaritmos antes de progresses, no h

grandes diculdades em falar intuitivamente de limite da soma dos termos de

uma progresso geomtrica pois, ao fazer os grcos das funes exponenciais e

logartmicas, voc j deve ter comentado quais os limites de ax quando x tende


para + ou para . Se a primeira noo de limite aparece no limite da soma

da progresso geomtrica, os Exemplos 13 e 14 de progresses geomtricas so

muito bons.

20
Progresses Geomtricas Unidade 6

A Frmula de Somao por Partes Para Saber Mais

Encerramos esta seo com a chamada frmula de somao por partes.

Temos

(ak bk ) = ak+1 bk+1 ak bk = ak+1 (bk+1 bk )+bk (ak+1 ak ) = ak+1 bk +bk ak

Da resulta

ak+1 bk = (ak bk ) bk ak .
Somando, obtemos a frmula de somao por partes:

n
X n
X
ak+1 bk = an+1 bn+1 a1 b1 ak ak .
k=1 k=1

n
X
Exemplo Calcule k3k .
k=1
1
Temos 3k = 3k+1 3k = 3k (3 1) = 2 3k . Logo, 3k = 3k e
2
n n
X
k 1X
k3 = k3k .
k=1
2 k=1

Aplicando a frmula de somao por partes

n
X n
X
ak+1 bk = an+1 bn+1 a1 b1 bk ak .
k=1 k=1

com ak+1 = k (logo, ak = k 1 e ak = ak+1 ak = 1) e bk = 3k , temos

n n  n 
X
k1X k 1 n+1
X
k
k3 = k3 = n . 3 0 3 .1 .
k=1
2 k=1
2 k=1

Mas
n
X 1 3n 3n+1 3
3k = 3 =
k=1
13 2 2
Da resulta

n
X n n+1 3n+1 3 2n 1 n+1 3
k3k = 3 + = 3 + .
k=1
2 4 4 4 4

21
MA12 - Unidade 6
Progressoes Geometricas

Paulo Cezar Pinto Carvalho

PROFMAT - SBM

10 de Marco de 2013
Progressoes Geometricas

Uma progressao geometrica e uma sequencia na qual o


quociente entre cada termo e o termo anterior e constante.
Esse quociente constante e chamado de razao da progressao e
representado pela letra q.
Mais formalmente: (an ) e uma progressao geometrica quando
existe um numero real q tal que an+1 = an q, para todo
n N.

PROFMAT - SBM MA12 - Unidade 6, Progressoes Geometricas slide 2/11


Termo Geral

Em uma progressao geometrica (an )


para avancar um termo, basta multiplicar pela razao;
para avancar dois termos, basta multiplicar duas vezes pela
razao;
e assim por diante;
Por exemplo, a13 = a5 q 8
De modo geral,
an = a1 q n1 ,
pois, ao passar de a1 para an , avancamos n 1 termos.

PROFMAT - SBM MA12 - Unidade 6, Progressoes Geometricas slide 3/11


Comecando de a0 ou a1 ?

A populacao de um pas e hoje igual a 100.000 habitantes e


cresce 2% ao ano. Qual sera a populacao desse pas daqui a
10 anos?

Em 10 perodos, a populacao e multiplicada 10 vezes por 1,02


a10 = a0 q 10 = 100.000 1, 0210 = 121.899 habitantes.

PROFMAT - SBM MA12 - Unidade 6, Progressoes Geometricas slide 4/11


Exemplo

Uma pessoa, comecando com R$ 64,00, faz seis apostas


consecutivas, em cada uma das quais arrisca perder ou ganhar
a metade do que possui na ocasiao. Se ela ganha tres e perde
tres dessas apostas, pode-se afirmar que ela:
A) Ganha dinheiro.
B) Nao ganha dinheiro nem perde dinheiro.
C) Perde R$ 27,00.
D) Perde R$ 37,00.
E) Ganha ou perde dinheiro, dependendo da ordem em que
ocorreram suas vitorias e derrotas
V, V, D, V, D, D:
64 96 144 72 108 54 27
D, V, D, D, V, V:
64 32 48 24 12 18 27
O valor final e sempre R$ 27,00?

PROFMAT - SBM MA12 - Unidade 6, Progressoes Geometricas slide 5/11


A cada vitoria, a quantia e multiplicada por 32 ; a cada derrota,
e multiplicada por 21 .
Apos tres vitorias e tres derrotas, os R$ 64,00 sao
multiplicados tres vezes por 32 e tres vezes por 12 .
Logo, ao final, independentemente da ordem das vitorias e
3 1 3
derrotas, a pessoa tera 64 32 2 = 27 reais; ou seja, ela
perde 37 reais (alternativa D).

PROFMAT - SBM MA12 - Unidade 6, Progressoes Geometricas slide 6/11


Progressoes geometricas e funcoes exponenciais

Como em uma progressao geometrica an = a0 q n , a funcao


que associa a cada natural n o valor de an e simplesmente a
restricao aos naturais da funcao exponencial a(x) = a0 q x .
O grafico de uma progressao geometrica:

PROFMAT - SBM MA12 - Unidade 6, Progressoes Geometricas slide 7/11


Soma dos termos de uma progressao geometrica

Sn = a1 + a2 + a3 + . . . + an1 + an
qSn = a2 + a3 + a4 + + an + an+1

Sn qSn = a1 an+1
(1 q)Sn = a1 (1 q n )

1 qn
Sn = a1 + a2 + . . . + an = a1
1q

PROFMAT - SBM MA12 - Unidade 6, Progressoes Geometricas slide 8/11


Mostre que o numero 444 . . . 488 . . . 89, formado por n dgitos
iguais a 4, n 1 dgitos iguais a 8 e um dgito igual a 9, e um
quadrado perfeito e determine sua raiz quadrada.
444 . . . 488 . . . 89 = 
4 102n1 + . . . + 10n + 8 10n1 + . . . + 10 + 9


10n 1 10n1 1
= 4 10n + 80 +9
10 1 10 1
4 102n + 4 10n + 1
=
9 
2 10n + 1 2

=
3
Logo 444 . . . 488 . . . 89 e um quadrado perfeito e sua raiz
2 10n + 1 20 . . . 01
quadrada e = (n 1 zeros)
3 3
= 6 . . . 67 (n 1 seis).

PROFMAT - SBM MA12 - Unidade 6, Progressoes Geometricas slide 9/11


Limite da soma dos termos de uma progressao
geometrica

1 qn
Sn = a1 + a2 + . . . + an = a1
1q
Quando |q| < 1, temos lim q n = 0.
n
Em consequencia,
1 qn 10
lim Sn = lim a1 = a1
n n 1q 1q
Logo:
a1
lim Sn = a1 + a2 + . . . = .
n 1q

PROFMAT - SBM MA12 - Unidade 6, Progressoes Geometricas slide 10/11


Exemplo

Qual e o limite da soma 0, 3 + 0, 03 + 0, 003 + . . . quando o


numero de parcelas tende a infinito?

0, 3 1
0, 3 + 0, 03 + 0, 003 + . . . = = .
1 0, 1 3

Em outras palavras, a geratriz da dzima periodica


1
0, 33333 e .
3

PROFMAT - SBM MA12 - Unidade 6, Progressoes Geometricas slide 11/11


Lista de Exerccios
Unidade 6

1. Aumentos sucessivos de 10% e 20% equivalem a um aumento unico de


quanto?
2. Aumentando sua velocidade em 60%, de quanto voce diminui o tempo de
viagem?
3. Um decrescimento mensal de 5% gera um decrescimento anual de quanto?
4. Mantida constante a temperatura, a pressao de um gas perfeito e inver-
samente proporcional a seu volume. De quanto aumenta a pressao quando
reduzimos de 20% o volume?
5. Um carro novo custa R$ 18 000,00 e, com 4 anos de uso, vale R$ 12 000,00.
Supondo que o valor decresca a uma taxa anual constante, determine o valor
do carro com 1 ano de uso.
6. Determine tres numeros em progressao geometrica, conhecendo sua soma
19 e a soma de seus quadrados 133.
7. Numero perfeito e aquele que e igual a metade da soma dos seus divisores
positivos. Por exemplo, 6 e perfeito pois a soma dos seus divisores e 1 + 2 +
3 + 6 = 12. Prove que, se 2p 1 e um numero primo, entao 2p1 . (2p 1) e
um numero perfeito.
8. Calcule o valor da soma de n parcelas 1 + 11 + + 111 . . . 1.
9. Um garrafao contem p litros de vinho. Retira-se um litro de vinho do gar-
rafao e acrescenta-se um litro de agua, obtendo-se uma mistura homogenea;
retira-se, a seguir um litro da mistura e acrescenta-se um litro de agua e
assim por diante. Qual a quantidade de vinho que restara no garrafao apos
n dessas operacoes?
10. Larga-se uma bola de uma altura de 5cm. Apos cada choque com o solo,
ela recupera apenas 4/9 da altura anterior. Determine:
a) A distancia total percorrida pela bola.
b) O tempo gasto pela bola ate parar.

1
11. Uma faculdade recebe todos os anos 300 alunos novos no primeiro semes-
tre e 200 alunos novos no segundo semestre. 30% dos alunos sao reprovados
no primeiro perodo e repetem o perodo no semestre seguinte. Sendo an e
bn , respectivamente, o numero de alunos do primeiro perodo no primeiro e
no segundo semestres do ano n, calcule lim an e lim bn .
12. Se (an ) e uma progressao geometrica de termos positivos, prove que (bn )
definida por bn = log an e uma progressao aritmetica.
13. Se (an ) e uma progressao aritmetica, prove que (bn ) definida por bn = ean
e uma progressao geometrica.
 
1 2
14. Seja A = . Determine An .
2 4

2
Solucoes da Lista de Exerccios
Unidade 6

1. 100 1, 1.100 = 110 1, 2.110 = 132


A resposta e 32%.

2. Sejam v e t, respectivamente,a velocidade antiga e o tempo gasto e


sejam v 0 e t0 a velocidade e o tempo depois do aumento.
vt = v 0 t0
vt = 1, 6vt0
t0 = 1
1,6
t = 0.6251 = 62.5%t
O tempo se reduz em 37,5%.

3. 1 + 1 = (1 + i)n
1 + I = (1 0, 05)12
I = 0, 9512 1 0, 46
Aproximadamente 46%.

4. Sejam P a pressao e V o volume e sejam P 0 a pressao e V 0 o volume


depois da variacao.
P V = P 0V 0
P V = P 0 .0, 8V
P0 = 1
0,8
P = 1, 25P = 125%P
A pressao aumenta de 25%.

5. Os valores formam uma progressao geometrica.


a4 = a0 .q 4
12000 = 18000q 4
q
q = 4 23
q
a1 = a0 q = 18000 4 23 , ou seja, R$ 16264,84.

1
6. Sejam a, aq, aq 2 os numeros.
a + aq + aq 2 = 19
a2 + (aq)2 + (aq 2 )2 = 133
Da, a(1 + q + q 2 ) = 19
a2 (1 + q 2 + q 4 ) = 133
133
Dividindo, a(1 q + q 2 ) = 19
=7
1+q+q 2 19
Da, 1q+q 2
= 7
3
q= 2
ou q = 23 .
Se q = 23 , substituindo vem a = 4; se q = 23 , substituindo vem a = 9.
Os numeros sao 4, 6, 9 ou 9, 6, 4.

7. Se 2p 1 e primo, os divisores de 2p1 (2p 1) sao 1, 2, 22 , . . . , 2p1 , (2p


1), 2(2p 1), 22 (2p 1), . . . 2p1 (2p 1).
A soma desses divisores e 2p1 (2p 1) sao 1 + 2 + 22 + + 2p1 + (2p
1).(1 + 2 + 22 + + 2p1 ) = 2p.(1 + 2 + 22 + + 2p1 ) = 2p.(2p 1) =
2.2p1 (2p 1).
10k 1
8. A k-esima parcela da soma vale 1 + 10 + 10k1 = . A soma
9
e igual a

m n
X 10k 1 1X k n 10 10n 1 n 10n+1 10 9n
= 10 = . = .
k=1
9 9 k=1 9 9 9 9 81

9. Em cada operacao a quantidade de vinho reduz-se em p1 . Os valores


da quantidade de vinho formam uma progressao geometrica de razao
1 p1 .
 n
1
A resposta e 1 .
p
10. O tempo que r a bola gasta para, partindo de repouso, cair de uma
2h
altura h e . Como as alturas (em metros) das quedas sao 5,
g

2
4 4 2

9
.5 9
.5, . . . , supondo g = 10n/s2 , os tempos de queda (em segundos)
2
serao 1, 31 , 32 ,. . .
2 1
O tempo total de queda e 1 + 32 + 32 + = = 3 segundos.
1 23
A este tempo devemos adicional o tempo gasto pela bola nas subidas,
que e o mesmo, a excecao do 1s da queda inicial.
A resposta e 5s, aproximadamente.
300
11. lim an = 300 + 0, 3.200 + 0, 32 .300 + 0, 33 .200 + = +
1 0, 32
0, 3.200
= 396
1 0, 32
200
lim bn = 200 + 0, 3.300 + 0, 32 .200 + 0, 33 .300 + = +
1 0, 32
0, 3.300
= 319.
1 0, 32
12. bn+1 bn = log an+1 log an = log an+1
an
= log q = constante.

bn+1 ean+1
13. = an = ean+1 an er = constante.
bn e
14. A2 = 5A
 
n n1 5n1 2.5n1
A =5 A=
2.5n1 4.5n1

3
7
Recorrncias Lineares de
Primeira Ordem
Sumrio
7.1 Introduo . . . . . . . . . . . . . . . . . . . . . . . 2
7.2 Sequncias Denidas Recursivamente . . . . . . . . 3
7.3 Exerccios Recomendados . . . . . . . . . . . . . . . 4
7.4 Exerccios Suplementares . . . . . . . . . . . . . . . 4
7.5 Recorrncias Lineares de Primeira Ordem . . . . . . 5
7.6 Exerccios Recomendados . . . . . . . . . . . . . . . 9
7.7 Exerccios Suplementares . . . . . . . . . . . . . . . 9
7.8 Textos Complementares . . . . . . . . . . . . . . . . 10

1
Unidade 7 Introduo

7.1 Introduo
O assunto dessa unidade o estudo mais aprofundado das sequncias num-
ricas denidas recursivamente (ou por recorrncia) que abordamos nas Unidades
3 e 4.
Conforme vimos anteriormente, uma sequncia denida recursivamente
se ela for dada por uma regra (recorrncia) que permite calcular um termo
qualquer por meio de um ou mais termos anteriores. Por exemplo, PAs, PGs,
fatorial, potncias com expoentes nmeros naturais e a sequncia de Fibonacci
so denidas por recorrncia.
Nesta unidade, so estudadas as recorrncias lineares de primeira ordem, ou
seja, sequncias em que um termo qualquer denido por uma expresso que
envolve o termo anterior, sem elev-lo a um expoente maior do que 1, como,
por exemplo:

1) progresses aritmticas: an = an1 + r;


2) progresses geomtricas: an = an1 q ;
3) fatorial: an = nan1 ;
4) potncias com expoente natural: an = aan1 .
Note que, para denir uma sequncia desse modo, no basta dar a recorrn-
cia, mas preciso dizer qual o seu primeiro termo. Isto bvio nos casos de
PAs e PGs. No caso (3), obtemos o fatorial se tomarmos a1 = 1. Se tomarmos
a1 = 2, por exemplo, obtemos a sequncia:
a1 = 2, a2 = 4, a3 = 12, a4 = 48, . . . ,
que no representa o fatorial. Temos tambm que (4) somente dene as po-
tncias de a se tomarmos a1 = a.
Na presente unidade, vamos nos dedicar, essencialmente, a determinar fr-
mulas fechadas para algumas recorrncias lineares de primeira ordem, onde, por
frmula fechada, entendemos uma expresso an = (n) para an como funo
de n. Quando determinamos uma frmula fechada para uma recorrncia, di-
zemos que ela foi resolvida. Aprenderemos como resolver recorrncias do tipo
an+1 = can + f (n), onde f uma funo com domnio o conjunto dos naturais
e c uma constante.

2
Recorrncias Lineares de Primeira Ordem Unidade 7

7.2 Sequncias Denidas Recursivamente


Muitas sequncias so denidas recursivamente (isto , por recorrncia),
ou seja, por intermdio de uma regra que permite calcular qualquer termo em
funo do(s) antecessor(es) imediato(s).

A sequncia (xn ) dos nmeros naturais mpares 1, 3, 5, 7, . . . pode ser de- Exemplo 1
nida por xn+1 = xn + 2 (n > 1), com x1 = 1.

Qualquer progresso aritmtica (xn ) de razo r e primeiro termo a pode Exemplo 2


ser denida por xn+1 = xn + r (n > 1), com x1 = a.

Qualquer progresso geomtrica (xn ) de razo q e primeiro termo a pode Exemplo 3


ser denida por xn+1 = q xn (n > 1), com x1 = a.

A sequncia (Fn ), dita de Fibonacci, cujos termos so 1, 1, 2, 3, 5, . . . e na Exemplo 4


qual cada termo a soma dos dois imediatamente anteriores, denida por
Fn+2 = Fn+1 + Fn (n > 0), com F0 = F1 = 1.

fcil ver que uma recorrncia, por si s, no dene a sequncia. Por exemplo,
a recorrncia do Exemplo 1, xn+1 = xn + 2, satisfeita no apenas pela
sequncia dos nmeros mpares, mas por todas as progresses aritmticas de
razo 2. Para que a sequncia que perfeitamente determinada necessrio
tambm o conhecimento do(s) primeiro(s) termo(s).
Observe que, nos Exemplos 1, 2 e 3 temos recorrncias de primeira ordem,
isto , nas quais cada termo expresso em funo do antecessor imediato, e
que, no Exemplo 4, temos uma recorrncia de segunda ordem, ou seja, na qual
cada termo expresso em funo dos dois antecessores imediatos.

+ Para Saber Mais - Dois Exemplos Mais Sosticados - Clique para ler

3
Unidade 7 Exerccios Recomendados

7.3 Exerccios Recomendados


1. Determine x5 na sequncia denida por xn+2 = 2xn+1 + xn , x0 = x1 = 1.
2. Seja xn o nmero mximo de regies em que n retas podem dividir o plano.
Caracterize xn recursivamente.
Sugesto: Lembre-se da pizza de Steiner.
3. Prove que uma recorrncia de primeira ordem, xn+1 = f (xn ), com uma
condio inicial x1 = a, tem sempre uma e uma s soluo.
4. Prove que uma recorrncia de segunda ordem xn+2 = f (xn+1 , xn ), com
condies iniciais x1 = a e x2 = b, tem sempre soluo nica.
5. Determine xn , dada a sequncia:
a) xn+1 = 2xn e x1 = 3; b) xn+1 = xn + 3 e x1 = 2.

7.4 Exerccios Suplementares


1. Seja xn o nmero mximo de regies em que n crculos podem dividir o
plano. Caracterize xn recursivamente.
2. Determine o nmero de permutaes caticas de 5 elementos.
3. Prove que o nmero de permutaes caticas de n elementos
n
X (1)n
Dn = n! .
k=0
k!

4
Recorrncias Lineares de Primeira Ordem Unidade 7

7.5 Recorrncias Lineares de Primeira Ordem


Uma recorrncia de primeira ordem expressa xn+1 em funo de xn . Ela
dita linear se (e somente se) essa funo for do primeiro grau.

As recorrncias xn+1 = 2xn n2 e xn+1 = nxn so lineares e a recorrncia Exemplo 5


xn+1 = x2n no linear. As duas ltimas so ditas homogneas, por no
possuirem termo independente de xn .

No h grandes diculdades na resoluo de uma recorrncia linear homo-


gnea de primeira ordem, conforme mostram os exemplos a seguir.

Resolva a recorrncia xn+1 = nxn , x1 = 1. Exemplo 6


Soluo. Temos
x2 = 1x1
x3 = 2x2
x4 = 3x3
... ... ...
xn = (n 1)xn1
Da, multiplicando, obtemos xn = (n 1)!x1 . Como x1 = 1, temos xn =
(n 1)!.

Resolva a recorrncia xn+1 = 2xn . Exemplo 7


Soluo. Temos
x2 = 2x1
x3 = 2x2
x4 = 2x3
... ... ...
xn = 2xn1
Da, multiplicando, obtemos xn = 2n1 x1 . claro que como no foi prescrito
o valor de x1 , h uma innidade de solues para a recorrncia, xn = C 2n1 ,
onde C uma constante arbitrria.

5
Unidade 7 Recorrncias Lineares de Primeira Ordem

As recorrncias lineares no-homogneas de primeira ordem que mais facil-


mente se resolvem so as da forma xn+1 = xn + f (n).
Com efeito, temos

x2 = x1 + f (1)
x3 = x2 + f (2)
x4 = x3 + f (3)
... ... ...
xn = xn1 + f (n 1)

n1
Somando, obtemos xn = x1 +
X
f (k).
k=1

Exemplo 8 Resolva a recorrncia xn+1 = xn + 2n , x1 = 1.

Soluo. Temos

x2 = x1 + 2
x3 = x2 + 2 2
x4 = x3 + 2 3
... ... ...
xn = xn1 + 2n1

Somando, resulta

xn = x1 + (2 + 22 + 23 + + 2n1 )
= 1 + 2 + 22 + 23 + + 2n1
2n 1
= 1
21
= 2n 1.

6
Recorrncias Lineares de Primeira Ordem Unidade 7

Resolva xn+1 = xn + n, x1 = 0. Exemplo 9


Soluo. Temos

x2 = x1 + 1
x3 = x2 + 2
x4 = x3 + 3
... ... ...
xn = xn1 + (n 1).

Somando, resulta

xn = x1 + 1 + 2 + 3 + + (n 1)
= 1 + 2 + 3 + + (n 1)
n(n 1)
= .
2

O teorema a seguir mostra que qualquer recorrncia linear no-homognea


de primeira ordem pode ser transformada em uma da forma xn+1 = xn + f (n).

Se an uma soluo no-nula da recorrncia xn+1 = g(n)xn , ento a Teorema 1


Soluo de
substituio xn = an yn transforma a recorrncia xn+1 = g(n)xn + h(n) em
Recorrncias Lineares
yn+1 = yn + h(n)[g(n) an ]1 .
de Primeira Ordem

A substituio xn = an yn transforma Demonstrao

xn+1 = g(n)xn + h(n) em an+1 yn+1 = g(n)an yn + h(n).

Mas, an+1 = g(n)an , pois an soluo de xn+1 = g(n)xn . Portanto, a equao


se transforma em
g(n)an yn+1 = g(n)an yn + h(n),

ou seja, yn+1 = yn + h(n)[g(n) an ]1 .

7
Unidade 7 Recorrncias Lineares de Primeira Ordem

Exemplo 10 Resolva xn+1 = 2xn + 1, x1 = 2.


Soluo. Uma soluo no-nula de xn+1 = 2xn , por exemplo, xn = 2n1 ,
conforme vimos no Exemplo 7. Fazendo a substituio xn = 2n1 yn , obtemos
2n yn+1 = 2n yn + 1, ou seja, yn+1 = yn + 2n . Da se tem

y2 = y1 + 21
y3 = y2 + 22
y4 = y3 + 23
... ... ...
yn = yn1 + 2(n1) .

Somando, resulta

yn = y1 + 21 + 22 + 23 + + 2(n1)
(21 )n1 1
= y1 + 21
21 1
= y1 21n + 1.

Como xn = 2n1 yn e x1 = 2, temos y1 = 2 e yn = 3 21n . Da, xn =


3 2n1 1.

Exemplo 11 Resolva xn+1 = 3xn + 3n , x1 = 2.


Soluo. Uma soluo no-nula de xn+1 = 3xn , por exemplo, xn = 3n1
(ou qualquer outra progresso geomtrica de razo 3). Faamos a substituio
xn = 3n1 yn . Obtemos 3n yn+1 = 3n yn + 3n , ou seja, yn+1 = yn + 1. Da,
yn uma progresso aritmtica de razo 1. Logo, yn = y1 + (n 1)1. Como
xn = 3n1 yn e x1 = 2, temos y1 = 2 e yn = n + 1. Da, xn = (n + 1)3n1 .

8
Recorrncias Lineares de Primeira Ordem Unidade 7

7.6 Exerccios Recomendados


1. Resolva a recorrncia do Exerccio Recomendado 1, da Seo 3 (pizza de
Steiner).

2. Quantas so as sequncias de n termos, todos pertencentes a {0, 1}, que


possuem em nmero mpar de termos iguais a 0?

3. Quantas so as sequncias de n termos, todos pertencentes a {0, 1, 2},


que possuem em nmero mpar de termos iguais a 0?

4. Sheila e Helena disputam uma srie de partidas. Cada partida iniciada


por quem venceu a partida anterior. Em cada partida, quem iniciou tem
probabilidade 0,6 de ganh-la e probabilidade 0,4 de perd-la. Se Helena
iniciou a primeira partida, qual a probabilidade de Sheila ganhar a n-
sima partida?

5. Resolva as seguintes recorrncias:


a) xn+1 = (n + 1)xn + n, x1 = 1;
b) (n + 1)xn+1 + nxn = 2n 3, x1 = 1;
c) xn+1 nxn = (n + 1)!, x1 = 1.

7.7 Exerccios Suplementares


1. Um crculo foi dividido em n (n > 2) setores. De quantos modos podemos
color-los, cada setor com uma s cor, se dispomos de k (k > 2) cores
diferentes e setores adjacentes no devem ter a mesma cor?

2. A torcida do Fluminense tem hoje p0 membros. A taxa anual de natalidade


i, a mortalidade j e, alm disso, todo ano um nmero xo de R
torcedores desiste de vez. Se i > j , determine o nmero de torcedores
daqui a n anos. A torcida est condenada a extino?

3. Ache o nmero mximo de regies em que n crculos podem dividir o


plano, ou seja resolva a recorrncia do Exerccio Suplementar 1 da Seo
4.

9
Unidade 7 Textos Complementares

7.8 Textos Complementares

Para Saber Mais Dois Exemplos Mais Sosticados


Apresentaremos aqui dois exemplos mais sosticados.
Exemplo 1. Quantas so as sequncias de 10 termos, pertencentes a {0, 1, 2},
que no possuem dois termos consecutivos iguais a 0?
Soluo. Chamando xn o nmero de sequncias com n termos, o valor de
xn+2 ser a soma das seguintes quantidades:
i) O nmero de sequncias de n + 2 termos que comeam por 1 e no possuem
dois zeros consecutivos. Isso precisamente igual a xn+1 , pois se o primeiro
termo 1, para formar a sequncia basta determinar os termos a partir do
primeiro, o que pode ser feito de xn+1 modos.
ii) O nmero de sequncias de n + 2 termos que comeam por 2 e no possuem
dois zeros consecutivos. Analogamente, isso igual a xn+1 .
iii) O nmero de sequncias de n + 2 termos que comeam por 0 e no possuem
dois zeros consecutivos. Se o primeiro termo zero, temos dois modos de
escolher o segundo termo (1 ou 2) e, escolhido o segundo termo, temos xn
modos de escolher os demais. H, pois, 2xn sequncias comeadas em 0.
Logo, xn+2 = 2xn+1 + 2xn . fcil ver que x1 = 3 e que x2 = 8. Da
obtemos x3 = 2x2 + 2x1 = 22, x4 = 60, . . . , x10 = 24 960.
Exemplo 2. Seja Dn o nmero de permutaes caticas de 1, 2, . . . , n, isto ,
o nmero de permutaes simples de 1, 2, . . . , n, nas quais nenhum elemento
ocupa o seu lugar primitivo. Mostre que Dn+2 = (n+1)(Dn+1 +Dn ), se n > 1.
Soluo. Calculemos Dn+2 , o nmero de permutaes simples de 1, 2, . . . , n+
2 nas quais nenhum elemento ocupa o seu lugar primitivo. As permutaes
podem ser divididas em dois grupos: aquelas nas quais o 1 ocupa o lugar do
nmero que ocupa o primeiro lugar e aquelas nas quais isso no ocorre.
Para formar uma permutao do primeiro grupo, devemos escolher o nmero
que trocar de lugar com o 1, o que pode ser feito de n + 1 modos, e, em
seguida, devemos arrumar os demais n elementos nos restantes n lugares, sem
que nenhum desses elementos ocupe o seu lugar primitivo, o que pode ser feito
de Dn modos. H, portanto, (n + 1) Dn permutaes no primeiro grupo.

10
Recorrncias Lineares de Primeira Ordem Unidade 7

Para formar uma permutao do segundo grupo, temos de escolher o lugar


que ser ocupado pelo nmero 1 (chamemos esse lugar de k ), o que pode
ser feito de n + 1 modos, e, em seguida devemos arrumar os restantes n + 1
elementos dos demais n + 1 lugares, sem que o elemento k ocupe o primeiro
lugar e sem que nenhum dos demais elementos ocupe o seu lugar primitivo, o
que pode ser feito de Dn+1 modos. H, portanto, (n + 1) Dn+1 permutaes
no segundo grupo.
Consequentemente, Dn+2 = (n + 1)(Dn+1 + Dn ), como queramos demons-
trar.

11
MA12 - Unidade 7
Recorrencias Lineares de Primeira Ordem

Paulo Cezar Pinto Carvalho

PROFMAT - SBM

17 de Marco de 2013
Sequencias definidas por recorrencia

Sequencias definidas por regras que permitem calcular


qualquer termo em funcao dos anteriores (usualmente do
antecessor imediato ou de uma quantidade pequena de
antecessores imediatos).
Uma progressao aritmetica de primeiro termo a e razao r :
x1 = a, xn+1 = xn + r , para n 1
A sequencia de Fibonacci:
x1 = 1, x2 = 1, xn+2 = xn + xn+1 , para n 1.
Equivalentemente, x1 = 1, x2 = 1, xn = xn2 + xn1 , para
n 3.
Nao basta dar a lei de recorrencia; e preciso tambem fornecer
o(s) primeiro(s) termo(s)

PROFMAT - SBM MA12 - Unidade 7, Recorrencias Lineares de Primeira Ordem slide 2/10
Exemplo

Quantas sao as sequencias de 10 termos, pertencentes a


{0, 1, 2}, que nao tem dois termos consecutivos iguais a 0?
Seja xn o numero de sequencias com n termos respeitando as
condicoes do enunciado. Vamos contar separadamente as
sequencias, de acordo com seu termo inicial.
O numero dessas sequencias comecando por 1 e xn1 .
O numero dessas sequencias comecando por 2 tambem e xn1 .
O numero dessas sequencias comecando por 0 e 2xn2
Logo xn = 2xn1 + 2xn2 , para n 3.
Alem disso, x1 = 3
Da, obtemos
x3 = 2x2 + 2x1 = 22,
x6 = 448,

PROFMAT - SBM MA12 - Unidade 7, Recorrencias Lineares de Primeira Ordem slide 3/10
Recorrencias Lineares de Primeira Ordem

Uma recorrencia de primeira ordem expressa xn+1 em funcao


de xn .
Ela e dita linear quando essa funcao for do primeiro grau.

As recorrencias xn+1 = 2xn n2 e xn+1 = nxn sao lineares e a


recorrencia xn+1 = xn2 nao e linear.

As duas ultimas sao ditas homogeneas, por nao possuirem


termo independente de xn .

PROFMAT - SBM MA12 - Unidade 7, Recorrencias Lineares de Primeira Ordem slide 4/10
Recorrencias lineares homogeneas
Resolver a recorrencia xn+1 = nxn .

x2 = 1x1
x3 = 2x2
x4 = 3x3
... ... ...
xn = (n 1)xn1

Multiplicando:

xn = (n 1)!x1

Solucao geral: xn = C (n 1)!

PROFMAT - SBM MA12 - Unidade 7, Recorrencias Lineares de Primeira Ordem slide 5/10
Recorrencias lineares da forma xn+1 = xn + f (n)

x2 = x1 + f (1)
x3 = x2 + f (2)
x4 = x3 + f (3)
... ... ...
xn = xn1 + f (n 1)

Somando:
n1
X
xn = x1 + f (k).
k=1

PROFMAT - SBM MA12 - Unidade 7, Recorrencias Lineares de Primeira Ordem slide 6/10
Exemplo
Resolver a recorrencia xn+1 = xn + 2n , x1 = 1.

x2 = x1 + 2
x3 = x2 + 22
x4 = x3 + 23
... ... ...
xn = xn1 + 2n1

Somando:
xn = x1 + (2 + 22 + 23 + + 2n1 )
= 1 + 2 + 22 + 23 + + 2n1
2n 1
=1
21
= 2n 1.
PROFMAT - SBM MA12 - Unidade 7, Recorrencias Lineares de Primeira Ordem slide 7/10
Recorrencias lineares nao homogeneas em geral

Se an e uma solucao nao-nula da recorrencia xn+1 = g (n)xn ,


entao a substituicao xn = an yn transforma a recorrencia
h(n)
xn+1 = g (n)xn + h(n) em yn+1 = yn +
g (n) an
A substituicao xn = an yn transforma

xn+1 = g (n)xn + h(n) em an+1 yn+1 = g (n)an yn + h(n).

Mas, an+1 = g (n)an , pois an e solucao de xn+1 = g (n)xn .


Portanto, a equacao se transforma em

g (n)an yn+1 = g (n)an yn + h(n),

h(n)
Ou seja, em yn+1 = yn + .
g (n) an

PROFMAT - SBM MA12 - Unidade 7, Recorrencias Lineares de Primeira Ordem slide 8/10
Exemplo
Resolver xn+1 = 2xn + 1, x1 = 2
Uma solucao nao-nula de xn+1 = 2xn e, por exemplo,
xn = 2n1 .
Fazendo a substituicao xn = 2n1 yn , obtemos
2n yn+1 = 2n yn + 1, ou seja, yn+1 = yn + 2n .
Da:

y2 = y1 + 21
y3 = y2 + 22
... ... ...
yn = yn1 + 2(n1) .

Somando:

yn = y1 + 21 + 22 + + 2(n1)
= y1 + 1 2(n1) .
PROFMAT - SBM MA12 - Unidade 7, Recorrencias Lineares de Primeira Ordem slide 9/10
A recorrencia original e xn+1 = 2xn + 1, x1 = 2.

Fizemos a substituicao xn = 2n1 yn e obtivemos a solucao


yn = y1 + 1 2(n1) para a nova recorrencia.

Da,
xn = 2n1 yn = 2n1 y1 + 1 2(n1) = 2n1 y1 + 2n1 1


= (1 + y1 )2n1 1

Finalmente, de x1 = 2 e de xn = 2n1 yn , temos 2 = 20 y1 .


Logo, y1 = 2 e a solucao da recorrencia e

xn = 3 2n1 1, para n 1

PROFMAT - SBM MA12 - Unidade 7, Recorrencias Lineares de Primeira Ordem slide 10/10
Lista de Exerccios
Unidade 7

1. Determine x5 na sequencia definida por xn+2 = 2xn+1 + xn , x0 = x1 = 1.


2. Seja xn o numero maximo de regioes em que n retas podem dividir o
plano. Caracterize xn recursivamente.
Lembre-se da pizza de Steiner.
3. Prove que uma recorrencia de primeira ordem, xn+1 = f (xn ), com uma
condicao inicial x1 = a, tem sempre uma e uma so solucao.
4. Prove que uma recorrencia de segunda ordem xn+2 = f (xn+1 , xn ), com
condicoes iniciais x1 = a e x2 = b, tem sempre solucao unica.
5. Determine xn , dada a sequencia:
a) xn+1 = 2xn e x1 = 3; b) xn+1 = xn + 3 e x1 = 2.
6. Resolva a recorrencia do Exerccio 2.
7. Quantas sao as sequencias de n termos, todos pertencentes a {0, 1}, que
possuem em numero mpar de termos iguais a 0?
8. Quantas sao as sequencias de n termos, todos pertencentes a {0, 1, 2},
que possuem em numero mpar de termos iguais a 0?
9. Sheila e Helena disputam uma serie de partidas. Cada partida e iniciada
por quem venceu a partida anterior. Em cada partida, quem iniciou tem
probabilidade 0,6 de ganha-la e probabilidade 0,4 de perde-la. Se Helena
iniciou a primeira partida, qual e a probabilidade de Sheila ganhar a n-esima
partida?
10. Resolva as seguintes recorrencias:
a) xn+1 = (n + 1)xn + n, x1 = 1;
b) (n + 1)xn+1 + nxn = 2n 3, x1 = 1;
c) xn+1 nxn = (n + 1)!, x1 = 1.

1
Solucoes da Lista de Exerccios
Unidade 7

1.

x2 = 2x1 + x0 = 2.1 + 1 = 3
x3 = 2x2 + x1 = 2.3 + 1 = 7
x4 = 2x3 + x2 = 2.7 + 3 = 17
x5 = 2x4 + x3 = 2.17 + 7 = 41

2. O numero maximo de regioes e determinado quando, para cada n, a reta


n + 1 intercepta as n ja existentes. Neste caso, a nova reta subdivide
n+1 regioes, criando assim n+1 novas regioes. Logo, o numero maximo
de regioes xn determinado por n retas satisfaz xn+1 = xn +(n+1), para
n = 0, 1, 2, . . . , com x0 = 1.

3. O valor de x1 esta bem definido, ja que x1 = a. Suponhamos agora que


xn esteja bem definido. Entao, como xn+1 = f (xn ), o valor de xn+1
tambem esta bem definido para todo natural n.

4. Os valores de x1 e x2 estao bem definidos, ja que x1 = a e x2 = b.


Suponhamos agora que xn e xn+1 estejam bem definidos. Entao, como
xn+2 = f (xn , xn+1 ), o valor de xn+2 tambem esta bem definido. Logo,
pelo Principio da Inducao Finita, o valor de xn esta bem definido para
todos natural n.

5. a) A razao xn+1 /xn entre dois termos consecutivos e constante e igual


a 2. Logo, a sequencia e uma progressao geometrica de razao
2. Como o primeiro termo e x1 = 3, o termo geral e dado por
xn = 3.2n1 .
b) A diferenca xn+1 xn entre dois termos consecutivos e constante e
igual a 3. Logo, a sequencia e uma progressao aritmetica de razao
3. Como o primeiro termo e x1 = 2, o termo geral e dado por
xn = 2 + 3n 1 = 3n + 1.

1
6. Do exerccio 2, o numero maximo xn de regioes em que n retas podem
dividir o plano satisfaz a recorrencia xn+1 = xn + n + 1, com x0 = 1.
Da:

x0 = 1
x1 = x0 + 1
x2 = x1 + 2

xn = xn1 + n

Somando, resulta:

n(n + 1)
xn = 1 + 1 + + n = 1 +
2

7. Seja xn o numero de sequencias de n termos 0 ou 1 com quantidade


mpar de termos iguais a 0. O numero de sequencias de n+1 termos 0 ou
1 com numero mpar de termos iguais a 0 e igual ao numero de sequen-
cias comecadas com 1, seguindo de uma sequencia de n termos com
numero mpar de zeros somado ao numero de sequencias comecadas
com 0, seguido de uma sequencia de n termos com um numero par de
zeros. Portanto, xn+1 = xn + (2n xn ) = 2n (para a segunda parcela,
note que 2n e o numero total de sequencias formadas por 0 ou 1). Logo,
xn = 2n1 , para todo n.

8. Sequencias de n + 1 termos 0, 1 ou 2 com um numero mpar de termos


iguais a 0 podem ser de dois tipos: as que comecam com 1 ou 2, seguido
por uma sequencia de n termos com numero mpar de zeros e as que
comecam com 0, seguido por uma sequencia de n termos com numero
par de zeros. Da, temos a recorrencia xn+1 = 2xn + (3n xn ), ou seja,
xn+1 = xn + 3n , com x1 = 1. Termos:

x1 = 1
x2 = x1 + 3 1

xn = xn1 + 3n1

2
Somando, resulta
3n 1
xn = 1 + 3 + + 3n1 = .
2

9. Para Sheila ganhar a (n + 1)-esima partida, ou ela ganha a n-esima


partida (com probabilidade xn ) e ganha a seguinte (com probabilidade
condicional 0,6) ou perde a n-esima (com probabilidade 1xn ) e ganha
a seguinte (com probabilidade condicional 0,4). Logo, a probabilidade
xn+1 de vitoria na (n + 1)-esima partida e dada por xn+1 = 0, 6xn +
0, 4(1 xn ), ou seja, xn+1 = 0, 2xn + 0, 4, com x1 = 0, 4. Para resolver
a recorrencia comecamos com uma solucao nao nula de xn+1 = 0, 2xn ;
por exemplo, an = (0, 2)n1 . Fazendo a substituicao xn = (0, 2)n1 yn ,
0,4
temos (0, 2)n yn+1 = (0, 2)n yn + 0, 4, ou seja, yn+1 = yn + (0,2) n , com

y1 = x1 /a1 = 0, 4. Temos:

y1 = 0, 4
0, 4
y2 = y1 +
0, 2
0, 4
y3 = y2 +
(0, 2)2

0, 4
yn = yn1 +
(0, 2)n1

Somando, vem

0, 4 0, 4 1 (0, 2)n
yn = 0, 4 + + + = 0, 4
0, 2 (0, 2)n1 0, 8(0, 2)n1

Finalmente
1 (0, 2)n
xn = (0, 2)n1 yn = .
2
10. a) Uma solucao da equacao homogenea xn+1 = (n + 1)xn e an = n!.
Fazendo a substituicao xn = an yn , temos

(n + 1)!yn+1 = (n + 1)n!yn + n,

3
ou seja,
n
yn+1 = yn + ,
(n + 1)!
x1 !
com y1 = 1!
= 1.
Da:
y1 = 1
1
y2 = y1 +
2!

n1
yn = yn1 +
n!
Somando:
1 n1
yn = 1 + + + .
2! n!
Mas n1
n!
= n1
n!
1
= (n1)! n!1 . Logo
   
1 1 1 1 1 1
yn = 1 + ( ) + + +
1! 2! (n 2)! (n 1)! (n 1)! n!
1 1
=1+1 =2
n! n!
Finalmente
xn = n!yn = 2n! 1.
b) Uma solucao da equacao homogenea (n + 1)xn+1 = nxn e an =
(1)n
n
. Fazendo a substituicao xn = an yn , temos
(1)n+1 yn+1 = (1)n+1 yn + 2n 3,
ou seja,
yn+1 = yn + (1)n+1 (2n 3),
com y1 = xa11 = 1.
Assim, temos:
y1 = 1
y2 = y1 + (2 3)

yn = yn1 + (1)n (2(n 1) 3)

4
Somando:

yn = 1 1 1 + 3 5 + 7 9 + + (1)n (2(n 1) 3)

Quando n e par,

yn = 2 + (1 + 3) + (5 + 7) + ((2(n 2) 3)) + (2(n 1) 3)


= 2 + (n 2) = n 4

e xn = an yn = nn = 1 n4 .
Quando n e mpar, yn = (n 5) (2(n 1) 3) = n e xn =
an yn = 1.
As duas expressoes podem ser colocadas em uma unica, escre-
2+2(1)n
vendo, por exemplo: xn = 1 n
.
c) Uma solucao da equacao homogenea xn+1 = nxn e an = (n 1)!.
Fazendo a substituicao xn = an yn , temos n!yn+1 = n!yn + (n + 1)!,
ou seja, yn+1 = yn + (n + 1), com y1 = x1 /a1 = 1.
Assim

y1 = 1
y2 = y1 + 2

yn = yn1 + n

Somando,
n(n + 1)
yn = 1 + 2 + + n =
2
(n+1)!
e x n = an y n = 2
.

5
8
Recorrncias Lineares de
Segunda Ordem
Sumrio
8.1 Introduo . . . . . . . . . . . . . . . . . . . . . . . 2

8.2 A equao Caracterstica . . . . . . . . . . . . . . . 3

8.3 Recorrncias de Segunda Ordem . . . . . . . . . . . 4

8.4 Exerccios recomendados . . . . . . . . . . . . . . . 11

1
Unidade 8 Introduo

8.1 Introduo

Um exemplo de recorrncia linear de segunda ordem a recorrncia que

dene a sequncia de Fibonacci: xn = xn1 + xn2 .

Mais geralmente, uma recorrncia linear de segunda ordem uma recorrncia

do tipo

f (n)xn + g(n)xn1 + h(n)xn2 + k(n) = 0,

onde f, g, h e k so funes cujos domnios so o conjunto dos nmeros naturais


e f (n) nunca se anula. Quando k = 0, a recorrncia dita homognea. Para
que uma recorrncia do tipo acima nos dena uma sequncia, preciso estipular

os valores dos seus dois termos iniciais.

Nesta unidade, apresentaremos uma tcnica para resolver recorrncias line-

ares homogneas de segunda ordem, com coecientes constantes. Essa tcnica

consiste em encontrar progresses geomtricas da forma rn que resolvem a re-

corrncia e cujas razes r so razes de uma equao algbrica do segundo grau

chamada equao caracterstica da recorrncia. O termo geral da sequncia

ento obtido como uma combinao linear dessas progresses com coecientes

determinados graas aos valores dos termos iniciais x1 e x2 .


Essa tcnica pode ser plenamente justicada usando lgebra Linear (cf. [7])

e a mesma utilizada na resoluo de equaes diferenciais lineares homog-

neas com coecientes constantes, onde as PGs so substitudas por funes

exponenciais.

Tal como na teoria das equaes diferenciais, as solues de uma equa-

o com coecientes constante se obtm somando uma soluo particular da

equao dada s solues da equao homognea associada.

Para saber (bem) mais sobre sequncias recorrentes, recomendamos a lei-

tura do artigo Sequncias Recorrentes, de Carlos Gustavo Moreira, que o leitor

encontrar anexado ao material dessa semana e que no ser cobrado nas ava-

liaes, por ser de carter complementar.

2
Recorrncias Lineares de Segunda Ordem Unidade 8

8.2 A equao Caracterstica

Inicialmente, trataremos das recorrncias lineares de segunda ordem homo-

gneas com coecientes constantes, isto , recorrncias da forma

xn+2 + pxn+1 + qxn = 0.

Suporemos sempre q 6= 0, pois se q = 0, a recorrncia seria, na realidade, uma

recorrncia de primeira ordem.

A cada recorrncia linear de segunda ordem homognea, com coecientes

constantes, da forma acima, associaremos uma equao do segundo grau, r2 +


pr + q = 0, chamada equao caracterstica . A nossa suposio preliminar de

que q 6= 0 implica que 0 no raiz da equao caracterstica.

A recorrncia xn+2 = xn+1 + xn tem equao caracterstica r2 = r + 1. As Exemplo 1


razes da equao caracterstica so

1+ 5 1 5
r1 = e r2 = .
2 2

O teorema a seguir mostra que se as razes da equao caracterstica so

r1 e r2 , ento qualquer sequncia da forma an = C1 r1n + C2 r2n soluo da

recorrncia, quaisquer que sejam os valores das constantes C1 e C2 .

r2 + pr + q = 0 so r1 e r2 , ento an = C1 r1n + C2 r2n


Se as razes de Teorema 1
soluo da recorrncia xn+2 + pxn+1 + qxn = 0, quaisquer que sejam os valores

das constantes C1 e C2 .

Substituindo an = C1 r1n + C2 r2n na recorrncia xn+2 + pxn+1 + qxn = 0, Demonstrao


obtemos, agrupando convenientemente os termos,

C1 r1n (r12 + pr1 + q) + C2 r2n (r22 + pr2 + q)

= C1 r1n 0 + C2 r2n 0 = 0.

3
Unidade 8 Recorrncias de Segunda Ordem

Exemplo 2 A equao xn+2 + 3xn+1 4xn = 0 temr2 + 3r 4 = 0 como equao


caracterstica. As razes da equao caracterstica so 1 e 4. De acordo com
n n
o Teorema 1, todas as sequncias da forma an = C1 1 + C2 (4) so solues

da recorrncia.

8.3 Recorrncias de Segunda Ordem

O teorema a seguir mostra que, se r1 6= r2 , todas as solues da recorrncia


tm a forma apontada no Teorema 1.

Teorema 2 Se as razes der2 + pr + q = 0 so r1 e r2 , com r1 6= r2 , ento todas as


n n
solues da recorrncia xn+2 +pxn+1 +qxn = 0 so da forma an = C1 r1 +C2 r2 ,

C1 e C2 constantes.

Demonstrao Sejayn uma soluo qualquer de xn+2 + pxn+1 + qxn = 0. Determinemos


constantes C1 e C2 que sejam solues do sistemas de equaes

C1 r1 + C2 r2 = y1


C1 r12 + C2 r22 = y2

isto ,
r22 y1 r2 y2 r1 y2 r12 y1
C1 = e C2 = .
r1 r2 (r2 r1 ) r1 r2 (r2 r1 )
Isso possvel pois r1 6= r2 e r1 6= 0 e r2 6= 0.
n n
Armamos que yn = C1 r1 + C2 r2 para todo n natural, o que provar o
n n
teorema. Com efeito, seja zn = yn C1 r1 C2 r2 . Mostraremos que zn = 0

para todo n. Temos

zn+2 +pzn+1 +qzn = (yn+2 +pyn+1 +qyn )C1 r1n (r12 +pr1 +q)C2 r2n (r22 +pr2 +q).
O primeiro parntese igual a zero porqueyn soluo de xn+2 +pxn+1 +qxn =
0; os dois ltimos parnteses so iguais a zero porque r1 e r2 so razes de
r2 + pr + q = 0. Ento zn+2 + pzn+1 + qzn = 0.
2 2
Alm disso, como C1 r1 +C2 r2 = y1 e C1 r1 +C2 r2 = y2 , temos z1 = z2 = 0.

Mas, se zn+2 + pzn+1 + qzn = 0 e z1 = z2 = 0, ento zn = 0 para todo n.

4
Recorrncias Lineares de Segunda Ordem Unidade 8

Vamos determinar as solues da recorrncia Exemplo 3


xn+2 + 3xn+1 4xn = 0.

A equao caracterstica r2 + 3r 4 = 0, tem razes 1 e 4. De acordo

com os Teoremas 1 e 2, as solues da recorrncia so as sequncias da forma

an = C1 1n +C2 (4)n , isto , an = C1 +C2 (4)n , onde C1 e C2 so constantes


arbitrrias.

Fibonacci revisitado. Determinemos o nmero de Fibonacci Fn denido por Exemplo 4


Fn+2 = Fn+1 + Fn , com F0 = F1 = 1.

A equao caracterstica r2 = r + 1 e as suas razes so dadas por



1+ 5 1 5
r1 = e r2 = .
2 2
Ento, n n
 
1+ 5 1 5
Fn = C1 + C2 .
2 2
Para determinar C1 e C2 , basta usar F0 = F1 = 1.
Obtemos o sistema

C1 + C2 = 1


C1 1+2 5
+ C2 12 5
=1

Logo,
 
5+1 1+ 5 n 51 1 5 n
 
Fn = + ,
2 5 2 2 5 2
isto ,  
1 1 + 5 n+1 1 1 5 n+1
 
Fn = .
5 2 5 2

Se as razes da equao caracterstica forem complexas, a soluo an =


C1 r1n + C2 r2n , C1 e C2 constantes arbitrrias pode ser escrita de modo a evitar

clculos com complexos. Pondo as razes na forma trigonomtrica, teremos:

r1 = (cos + i sen ), r2 = (cos i sen )

5
Unidade 8 Recorrncias de Segunda Ordem

r1n = n (cos n + i sen n), r2n = n (cos n i sen n).


Logo,

C1 r1n + C2 r2n = n [(C1 + C2 ) cos n + i(C1 C2 ) sen n].

claro que C10 = C1 + C2 e C20 = i(C1 C2 ) so novas constantes e a soluo

pode ser escrita

an = n [C10 cos n + C20 sen n].

Exemplo 5 A recorrncia xn+2 +xn+1 +xn = 0 tem equao caracterstica r2 +r+1 = 0,


cujas razes so

1+i 3 1i 3
r1 = e r2 = ,
2 2

que so complexas de mdulo =1 e argumento principal = .
3
A soluo

n n
xn = n [C1 cos n + C2 sen n] = C1 cos + C2 sen .
3 3

O que aconteceria se as razes da equao caracterstica fossem iguais? Os

teoremas a seguir respondem essa pergunta.

Teorema 3 Se as razes de r2 + pr + q = 0 r1 = r2 = r, ento, an =


so iguais,

C1 rn + C2 nrn soluo da recorrncia xn+2 + pxn+1 + qxn = 0, quaisquer que


sejam os valores das constantes C1 e C2 .

p
Demonstrao Se as razes so iguais, ento r= . Substituindo an = C1 rn + C2 nrn
2
na recorrncia

xn+2 + pxn+1 + qxn = 0


obtemos, agrupando convenientemente os termos,

C1 rn (r2 + pr + q) + C2 nrn (r2 + pr + q) + C2 rn r(2r + p)

= C1 rn 0 + C2 nrn 0 + C2 rn r0 = 0.

6
Recorrncias Lineares de Segunda Ordem Unidade 8

Se as razes de r2 + pr + q = 0 so iguais, r1 = r2 = r, ento todas as Teorema 4


n n
solues da recorrncia xn+2 + pxn+1 + qxn = 0 so da forma C1 r + C2 nr ,

C1 e C2 constantes.

yn uma soluo qualquer de xn+2 + pxn+1 + qxn = 0.


Seja Determine
Demonstrao
constantes C1 e C2 que sejam solues do sistema de equaes.

C1 r + C2 r = y 1

,
2 2
C1 r + 2C2 r = y2

isto ,
y1 y2 y2 ry1
C1 = 2 2 e C2 = .
r r r2
Isso possvel pois r 6= 0.
n n
Armamos que yn = C1 r + C2 nr para todo n natural, o que provar o
n n
teorema. Com efeito, seja zn = yn C1 r C2 nr . Mostraremos que zn = 0

para todo n. Temos

zn+2 + pzn+1 + qzn = (yn+2 + pyn+1 + qyn )


C1 rn (r2 + pr + q) C2 nrn (r2 + pr + q) C2 rn r(2r + p).

O primeiro parntese igual a zero porque yn xn+2 + pxn+1 +


soluo de

qxn = 0; o segundo e o terceiro parnteses so iguais a zero porque r raiz


2
de r + pr + q = 0; o quarto igual a zero porque 2r + p = 0 j que, quando
p
r1 = r2 = r, tem-se r = . Ento zn+2 + pzn+1 + qzn = 0
2
2 2
Alm disso, como C1 r + C2 r = y1 e C1 r + 2C2 r = y2 , temos z1 = z2 = 0.

Mas, se zn+2 + pzn+1 + qzn = 0 e z1 = z2 = 0 ento zn = 0 para todo n.

A recorrncia xn+2 4xn+1 + 4xn = 0 tem equao caracterstica r2 Exemplo 6


4r + 4 = 0. As razes so r1 = r2 = 2 e a soluo da recorrncia xn =

C1 2n + C2 n2n .

O teorema a seguir mostra um processo para resolver algumas recorrncias

no-homogneas.

7
Unidade 8 Recorrncias de Segunda Ordem

Teorema 5 Se an uma soluo da equao

xn+2 + pxn+1 + qxn = f (n),

ento a substituio x n = an + y n transforma a equao em

yn+2 + pyn+1 + qyn = 0.

Demonstrao Substituindo xn por an + y n na equao, obtemos

(an+2 + pan+1 + qan ) + (yn+2 + pyn+1 + qyn ) = f (n).

Mas an+2 + pan+1 + qan = f (n) pois an a soluo da equao original. Logo,

a equao se transformou em

yn+2 + pyn+1 + qyn = 0.

De acordo com o Teorema 5, a soluo de uma recorrncia no-homognea

constituda de duas parcelas: uma soluo qualquer da no-homognea e a

soluo homognea. A soluo da homognea, sabemos achar. Uma soluo

da no-homognea, procuraremos por tentativas.

Exemplo 7 xn+2 6xn+1 + 8xn = n + 3n tem equao caracterstica


A recorrncia

r2 6r + 8 = 0, cujas razes so r1 = 2 e r2 = 4. Portanto, a soluo da


n
homognea, isto , de xn+2 6xn+1 + 8xn = 0 hn = C1 + C2 4 . Tentaremos

agora descobrir uma soluo particular, tn , da recorrncia

xn+2 6xn+1 + 8xn = n + 3n .

Ora, se substituirmos tn em xn+2 6xn+1 +8xn devemos encontrar n+3n . Que

tipo de funo deve ser tn ? bastante razovel imaginar que tn seja a soma de

um polinmio do primeiro grau com uma exponencial de base 3. Tentaremos

tn = An + B + C3n . Substituindo em

xn+2 6xn+1 + 8xn = n + 3n ,

8
Recorrncias Lineares de Segunda Ordem Unidade 8

obtemos 3An+3B4AC3n = n+3n . tn ter soluo se 3A = 1, 3B4A = 0


e C = 1. Logo,
1 4
A= , B= e C = 1.
3 9
Da,
1 4
tn = n + 3n .
3 9

xn+2 6xn+1 + 8xn = 1 + 2n tem equao caracterstica


A recorrncia Exemplo 8
r2 6r+8 = 0, cujas razes so r1 = 2 e r2 = 4. Portanto, a soluo da equao
n n
homognea, isto , de xn+2 6xn+1 +8xn = 0 hn = C1 2 +C2 4 . Tentaremos

agora descobrir uma soluo particular, tn da recorrncia xn+2 6xn+1 + 8xn =

1 + 2n . Ora, se substituirmos tn em xn+2 6xn+1 + 8xn devemos encontrar


1 + 2n . Que tipo de funo deve ser tn ? bastante razovel imaginar que
tn seja a soma de um polinmio constante com uma exponencial de base 2.
n
Tentaremos tn = A + B2 . Substituindo em

xn+2 6xn+1 + 8xn = 1 + 2n ,

obtemos 3A = 1 + 2n . Essa igualdade impossvel. A recorrncia no admite


n
soluo da forma tn = A + B2 .

Parando para pensar no que aconteceu, vericamos que era bvio que a

nossa tentativa no podia dar certo. O esprito da nossa tentativa era tentar

uma constante A para que obtivssemos uma constante que igualaramos a

1 e tentar B2n para gerar uma exponencial que pudssemos igualar a 2n .


n n
claro que o termo B2 no poderia cumprir o seu papel. B2 soluo da
homognea ( a soluo da homognea que obtida pondo C1 = B e C2 = 0)
e, substitudo da equao, daria zero e no uma exponencial que pudssemos

igualar a 2n .
Vamos corrigir a nossa tentativa para tn = A + Bn2n . Sempre que na

nossa tentativa em algum bloco no cumprir o seu papel, fazemos a correo

aumentando o grau, isto , multiplicando o bloco por n. Agora, substituindo


n n
obtemos 3A B4B2 = 1 + 2 .
Se 3A = 1 e 4B = 1, isto ,
1 1
A= e B= ,
3 4

9
Unidade 8 Recorrncias de Segunda Ordem

temos a soluo
1 n2n
tn = .
3 4
A soluo da recorrncia a soma de hn com tn . Portanto,

n 1 n2n
n
xn = C 1 2 + C 2 4 + .
3 4

10
Recorrncias Lineares de Segunda Ordem Unidade 8

8.4 Exerccios recomendados

1. Resolva as recorrncias a seguir:

a) xn+2 + 5xn+1 + 6xn = 0.

b) xn+2 + 6xn+1 + 9xn = 0.

c) xn+2 + 2xn+1 + 2xn = 0.

d) xn+2 5xn+1 + 6xn = n.

e) xn+2 5xn+1 + 6xn = 1 + 3 4n .

f) xn+2 5xn+1 + 6xn = 2n .

g) xn+2 5xn+1 + 6xn = n + 3n .

h) xn+2 6xn+1 + 9xn = n 3n .

i) xn+2 + xn = 1.

j) xn+2 6xn+1 + 9xn = 1 + n3n .

2. Resolva as recorrncias a seguir:

a) xn+2 + 5xn+1 + 6xn = 0; x0 = 3; x1 = 6.

b)xn+2 + xn+1 6xn = 6 8n; x0 = 1; x1 = 4. No gabarito,


considera x1 = 2
c)xn+2 4xn+1 + 4xn = 2n+3 ; x0 = 3; x1 = 6. Tirar dvida

3. Quantas so as sequncias de n termos, todos pertencentes a {0, 1, 2},


que no possuem dois termos consecutivos iguais a 0?
Gabarito errado

4. Determine o nmero de modos de cobrir um tabuleiro 2 n com domins


21 iguais.

11
Unidade 8 Exerccios recomendados

5. Uma planta tal que cada uma de suas sementes produz, um ano aps

ter sido plantada, 21 novas sementes e, a partir da, 44 novas sementes

a cada ano. Se plantarmos hoje uma semente e se, toda vez que uma

semente for produzida ela for imediatamente plantada, quantas sementes

sero produzidas daqui a n anos?

6. O salrio de Carmelino no ms n Sn = a + bn. Sua renda mensal for-

mada pelo salrio e pelos juros de suas aplicaes nanceiras. Ele poupa

anualmente 1/p de sua renda e investe sua poupana a juros mensais de

taxa i. Determine a renda de Carmelino no ms n.

7. Cinco times de igual fora disputaro todo ano um torneio. Uma taa

ser ganha pelo primeiro time que vencer trs vezes consecutivas. Qual

a probabilidade da taa no ser ganha nos n primeiros torneios?

8. Em um jogo, em cada etapa Olavo, pode fazer 1 ou 2 pontos. De quantos

modos ele pode totalizar n pontos?

9. Mostre que


2 5+1 n 2 51
(1 5) + (1 + 5)n
2 5 2 5
, para todo natural n, um nmero inteiro.


10. Mostre que a parte inteira de (1 + 3)2n+1 sempre par.

12
MA12 - Unidade 8
Recorrencias Lineares de Segunda Ordem

Paulo Cezar Pinto Carvalho

PROFMAT - SBM

17 de Marco de 2013
Recorrencias lineares de segunda ordem homogeneas
com coeficientes constantes

Sao recorrencias da forma

xn+2 + pxn+1 + qxn = 0,

com q 6= 0 (se q = 0, a recorrencia e, na verdade, de primeira


ordem)
A equacao caracterstica da recorrencia e:

r 2 + pr + q = 0
Veremos a seguir que as razes da equacao caracterstica
desempenham um papel fundamental na expressao da solucao
geral para a recorrencia.
Como q 6= 0, essas razes sao necessariamente nao nulas.

PROFMAT - SBM MA12 - Unidade 8, Recorrencias Lineares de Segunda Ordem slide 2/14
Razes da equacao caracterstica e solucoes da
recorrencia

Se r e tal que r 2 + pr + q = 0 , entao xn = r n e solucao da


recorrencia xn+2 + pxn+1 + qxn = 0.
Substituindo xn = r n na recorrencia:

xn+2 + pxn+1 + qxn = r n+2 + pr n+1 + qr n

= r n (r 2 + pr + q) = r n 0 = 0.

PROFMAT - SBM MA12 - Unidade 8, Recorrencias Lineares de Segunda Ordem slide 3/14
Consequencia: Se r1 e r2 sao razes distintas de
r 2 + pr + q = 0, entao xn = C1 r1n + C2 r2n e solucao da
recorrencia xn+2 + pxn+1 + qxn = 0, quaisquer que sejam os
valor das constantes C1 e C2 .
Sejam yn = r1n , zn = r2n e xn = C1 yn + C2 zn .
xn+2 + pxn+1 + qxn =
(C1 yn+2 + C2 zn+2 ) + p(C1 yn+1 + C2 zn+1 ) + q(C1 yn + C2 zn )
= C1 (yn+2 + pyn+1 + qyn ) + C2 (zn+2 + pzn+1 + qzn ) =
= C1 0 + C2 0 = 0,
ja que yn e zn sao solucoes da recorrencia.
(De modo geral, se yn e zn sao solucoes de uma recorrencia
linear homogenea, qualquer combinacao linear de yn e zn
tambem e solucao da recorrencia.)

PROFMAT - SBM MA12 - Unidade 8, Recorrencias Lineares de Segunda Ordem slide 4/14
Resolvendo a recorrencia: caso r1 6= r2

Se as razes de r 2 + pr + q = 0 (q 6= 0) sao r1 e r2 , com


r1 6= r2 , entao todas as solucoes da recorrencia
xn+2 + pxn+1 + qxn = 0 sao da forma an = C1 r1n + C2 r2n , C1 e
C2 constantes.
Seja (xn ) uma solucao qualquer de xn+2 + pxn+1 + qxn = 0. E
sempre possvel escolher constantes C1 e C2 tais que:

C1 r1 + C2 r2 = x1

C1 r12 + C2 r22 = x2

(o sistema sempre tem solucao unica).


Vamos provar que xn = C1 r1n + C2 r2n para todo n natural.

PROFMAT - SBM MA12 - Unidade 8, Recorrencias Lineares de Segunda Ordem slide 5/14
A afirmativa vale para n = 1 e n = 2, ja que C1 e C2 foram
escolhidos de modo que isto ocorra.
Suponhamos valida para naturais n e n + 1. Temos
xn+2 + pxn+1 + qxn = 0. Logo
xn+2 = p(C1 r1n+1 + C2 r2n+1 ) q(C1 r1n + C2 r2n )
= C1 r1n (pr1 + q) C2 r2n (pr2 + q)
Somando e subtraindo C1 r2n+2 + C2 r2n+2 :
xn+2 =
C1 r1n (r12 + pr1 + q) C2 r2n (r22 + pr2 + q) + C1 r2n+2 + C2 r2n+2
Mas as expressoes entre parenteses se anulam, levando a
xn+2 = C1 r2n+2 + C2 r2n+2 ,
o que completa a prova por inducao.

PROFMAT - SBM MA12 - Unidade 8, Recorrencias Lineares de Segunda Ordem slide 6/14
Exemplo

Determinar as solucoes da recorrencia


xn+2 + 3xn+1 4xn = 0.

A equacao caracterstica r 2 + 3r 4 = 0, tem razes 1 e 4.

As solucoes da recorrencia sao as sequencias da forma


xn = C1 1n + C2 (4)n , isto e,

xn = C1 + C2 (4)n ,

onde C1 e C2 sao constantes arbitrarias.

PROFMAT - SBM MA12 - Unidade 8, Recorrencias Lineares de Segunda Ordem slide 7/14
A sequencia de Fibonacci
F0 = F1 = 1, Fn+2 = Fn+1 + Fn , para n 0.
A equacao caracterstica e r 2 r 1 = 0, que tem razes

1+ 5 1 5
r1 = e r2 = .
2 2
Logo:
 
1+ 5 n 1 5 n
 
Fn = C1 + C2 .
2 2
Os valores de C1 e C2 sao obtidos usando F0 = F1 = 1:

C1 + C2 = 1

C1 1+2 5
+ C2 12 5

=1

PROFMAT - SBM MA12 - Unidade 8, Recorrencias Lineares de Segunda Ordem slide 8/14
Resolvendo o sistema, encontramos:

5+1 51
C1 = e C2 =
2 5 2 5
Logo:
 n  n
5+1 1+ 5 51 1 5
Fn = +
2 5 2 2 5 2

 
1 1 + 5 n+1 1 1 5 n+1
 
= .
5 2 5 2

PROFMAT - SBM MA12 - Unidade 8, Recorrencias Lineares de Segunda Ordem slide 9/14
O caso r1 = r2

Se as razes de r 2 + pr + q = 0 sao iguais (r1 = r2 = r ) entao


yn = nr n e solucao da recorrencia xn+2 + pxn+1 + qxn = 0.
Como zn = r n tambem e solucao, xn = C1 r n + C2 nr n e
solucao da recorrencia, quaisquer que sejam as constantes C1
e C2 .
Substituindo na recorrencia:
yn+2 + pyn+1 + qyn = (n + 2)r n+2 + p(n + 1)r n+1 + qnr n
= nr n (r 2 + pr1 + q) + r n+1 (2r + p).
O primeiro parenteses e igual a zero (r e raiz da equacao
caracterstica); como as razes da equacao caracterstica sao
iguais, temos r = p2 . Logo, 2r + p = 0 e o segundo
parenteses tambem e igual a zero.
Logo, yn = nr n e, de modo mais geral, xn = C1 r n + C2 nr n , e
solucao da recorrencia.

PROFMAT - SBM MA12 - Unidade 8, Recorrencias Lineares de Segunda Ordem slide 10/14
Resolvendo a recorrencia: caso r1 = r2

Se as razes de r 2 + pr + q = 0 sao iguais, r1 = r2 = r , entao


todas as solucoes da recorrencia xn+2 + pxn+1 + qxn = 0 sao
da forma C1 r n + C2 nr n , C1 e C2 constantes.
A prova e analoga ao caso em que r1 6= r2 .
primeiro, observamos que podemos escolher as constantes C1 e
C2 de modo que

C1 r + C2 r = x1
,
C1 r 2 + 2C2 r 2 = x2

(o sistema tem solucao unica para todo r 6= 0).


depois, verificamos por inducao que xn = C1 r n + C2 nr n , para
todo n 1.

PROFMAT - SBM MA12 - Unidade 8, Recorrencias Lineares de Segunda Ordem slide 11/14
Exemplo

Resolver a recorrencia xn+2 4xn+1 + 4xn = 0.

A equacao caracterstica e r 2 4r + 4 = 0. As razes sao


r1 = r2 = 2.

A solucao da recorrencia e xn = C1 2n + C2 n2n .

PROFMAT - SBM MA12 - Unidade 8, Recorrencias Lineares de Segunda Ordem slide 12/14
E se a recorrencia nao for homogenea?
Suponhamos que an seja uma solucao da recorrencia
xn+2 + pxn+1 + qxn = f (n). Toda solucao da recorrencia e da
forma an + yn , onde yn e uma solucao da recorrencia
homogenea xn+2 + pxn+1 + qxn = 0.
(Logo, para encontrar a solucao geral de uma recorrencia nao
homogenea, bastaencontrar uma solucao particular e
soma-la a solucao geral da recorrencia homogenea.)
Seja xn uma solucao da recorrencia. Temos:
xn+2 + pxn+1 + qxn = f (n)
an+2 + pan+1 + qan = f (n)
Subtraindo:
(xn+2 an+2 ) + p(xn+1 an+1 ) + q(xn an ) = 0
Logo, yn = xn an e uma solucao da recorrencia homogenea,
o que equivale a dizer que xn = an + yn , onde yn e solucao da
recorrencia homogenea.
PROFMAT - SBM MA12 - Unidade 8, Recorrencias Lineares de Segunda Ordem slide 13/14
Exemplo
Considere a recorrencia xn+2 5xn+1 + 6xn = 2.
Encontre uma solucao particular constante para a recorrencia.
Encontre a solucao geral da recorrencia.
Ache a solucao da recorrencia em que x1 = 0 e x2 = 2.
Para que an = C seja solucao, deve-se ter C 5C + 6C = 2,
ou seja, C = 1
A equacao caracterstica da recorrencia homogenea e
r 2 5r + 6 = 0, cujas razes sao r1 = 3 e r2 = 2. A solucao
geral da recorrencia homogenea e yn = C1 3n + C2 2n e a
solucao geral da recorrencia original e xn = yn + an , isto e:
xn = C1 3n + C2 2n + 1
Para que x1 = 0 e x2 = 2, deve-se ter

3C1 + 2C2 + 1 = 0
9C1 + 4C2 + 1 = 2
Resolvendo o sistema, encontramos C1 = 1 e C2 = 2, que
leva a solucao: xn = 3n 2.2n + 1
PROFMAT - SBM MA12 - Unidade 8, Recorrencias Lineares de Segunda Ordem slide 14/14
LISTA DE EXERCCIOS DO AMBIENTE VIRTUAL

Lista de Exerccios
Unidade 8

1. Resolva as recorrencias a seguir:

a) xn+2 + 5xn+1 + 6xn = 0.

b) xn+2 + 6xn+1 + 9xn = 0.

c) xn+2 5xn+1 + 6xn = n.

d) xn+2 5xn+1 + 6xn = 2n .

e) xn+2 + xn = 1.

2. Resolva as recorrencias a seguir:

a) xn+2 + 5xn+1 + 6xn = 0; x0 = 3; x1 = 6.

b)xn+2 + xn+1 6xn = 6 8n; x0 = 1; x1 = 4.

c)xn+2 4xn+1 + 4xn = 2n+3 ; x0 = 3; x1 = 6.

3. Quantas sao as sequencias de n termos, todos pertencentes a {0, 1, 2},


que nao possuem dois termos consecutivos iguais a 0?

4. Determine o numero de modos de cobrir um tabuleiro 2 n com do-


minos 2 1 iguais.

5. Uma planta e tal que cada uma de suas sementes produz, um ano
apos ter sido plantada, 21 novas sementes e, a partir da, 44 novas
sementes a cada ano. Se plantarmos hoje uma semente e se, toda
vez que uma semente for produzida ela for imediatamente plantada,
quantas sementes serao produzidas daqui a n anos?

6. Cinco times de igual forca disputarao todo ano um torneio. Uma taca
sera ganha pelo primeiro time que vencer tres vezes consecutivas. Qual
a probabilidade da taca nao ser ganha nos n primeiros torneios?

1
7. Em um jogo, em cada etapa Olavo, pode fazer 1 ou 2 pontos. De
quantos modos ele pode totalizar n pontos?

2
Solucoes da Lista de Exerccios
Unidade 8

1. a) As razes da equacao caracterstica r2 + 5r + 6 = 0 sao r1 = 2 e


r2 = 3. Logo, a solucao geral e xn = C1 (2)n + C2 (3)n , onde
C1 e C2 sao constantes arbitrarias.
b) A equacao caracterstica r2 + 6r + 9 = 0 tem duas razes iguais a
-3. Logo, a solucao geral e xn = C1 (3)n + C2 n(3)n , onde C1 e
C2 sao constantes arbitrarias.
c) Como as razes da equacao caracterstica r2 5r + 6 = 0 sao
r1 = 2 e r2 = 3, a solucao geral da equacao homogenea e xn =
C1 2n + C2 3n . Por outro lado, tentando uma solucao particular
da forma xn = An + B, obtemos 2An + 2B 3A = n, que se
verifica quando A = 21 e B = 43 . Portanto, xn = 12 n + 34 e uma
solucao particular e, em consequencia, a solucao geral da equacao
nao-homogenea e xn = 12 n + 34 + C1 2n + C2 3n .
d) A solucao geral da homogenea e a mesma do exerccio anterior.
Como xn = 2n e solucao da homogenea, na busca por uma solucao
particular temos xn = An2n , o que leva a A(n + 2)2n+2 5A(n +
1)2n+1 + 6An2n . Da, obtemos A = 21 , que fornece a solucao
particular xn = 12 n2n = n2n1 . A solucao geral da equacao
nao homogenea e xn = 12 n2n = n2n1 + C1 2n + C2 3n .
e) As solucoes da equacao caracterstica r2 + 1 = 0 sao r1 = i e
r2 = i. A solucao geral da equacao homogenea e xn = C1 in +
C2 (i)n = C1 cos n 2
+ C2 n
2
. Tentando uma solucao particular
da forma xn = A, obtemos 2A = 1, ou seja, xn = A = 1/2.
A solucao geral da equacao nao homogenea e, portanto, xn =
1
2
+ C1 cos n2
+ C2 n
2
.
2. a) No exerccio 1a), encontramos a solucao geral xn = C1 (2)n +
C2 (3)n . Substituindo n = 0 e n = 1 na solucao, encontramos:
C1 + C2 =3
2C1 3C2 = 6

1
Resolvendo, encontramos C1 = 0 e C2 = 3, levando a solucao
xn = 3(2)n .
b) As solucoes da equacao caracterstica r2 + r 6 = 0 sao r1 = 2
e r2 = 3, conduzindo a solucao geral xn = C1 2n + C2 (3)n
para a solucao homogenea. Tentando uma solucao particular da
forma xn = An + B, encontramos que a equacao e satisfeita para
A = 2 e B = 0. Assim, xn = 2n e uma solucao particular e
xn = 2n + C1 2n + C2 (3)n e a solucao geral da equacao nao
homogenea.
Substituindo n = 0 e n = 1 nesta solucao, obtemos:

C1 + C2 =1
2C1 3C2 =2

Resolvendo, encontramos C1 = 0 e C2 = 1. Portanto, a solucao e


xn = 2n + 2n
c) A equacao caracterstica r2 4r + 4 = 0 tem duas razes iguais a
2, conduzindo a solucao geral xn = C1 2n + C2 n2n para a parte ho-
mogenea. Tentando uma solucao particular da forma xn = An2 2n ,
verificamos que xn = n2 2n e uma solucao particular e que, assim,
xn = n2 2n + C1 2n + C2 n2n e a solucao geral da equacao nao ho-
mogenea.
Substituindo n = 0 e n = 1 encontramos:

C1 =3
2C1 + 2C2 =4

Resolvendo, encontramos C1 = 3 e C2 = 1. Logo, a solucao da


equacao e xn = 32n n2n + n2 2n

3. Seja xn o numero de sequencias formadas por n termos iguais a 0, 1


ou 2 sem dois zeros repetidos. As sequencias de n + 2 termos que nao
tem dois termos consecutivos podem comecar por 0, 1 ou 2 . As que
comecam por 0 tem o proximo elemento igual a 1 ou 2 e, a seguir,
uma sequencia de n termos sem zeros repetidos. Logo, ha 2xn tais
sequencias. As que comecam por 1 ou 2 tem, a seguir uma sequencia
de n + 1 termos sem zeros repetidos. Logo, ha 2xn+1 sequencias deste

2
tipo. Assim, xn satisfaz a recorrencia xn+2 = 2xn + 2xn+1 , ou seja,
xn+2 2xn+1 2xn = 0, com x1 = 3 e x2 = 8 (todas as 3 sequencias de
comprimento 1 cumprem o requisito e todas as 32 = 9 de comprimento
2, exceto a 00, tambem cumprem a condicao).

As razesda equacao caracterstica r2 2r 2 =
0 sao r1 = 1 + 3 e
r2 = 1 3, levando a solucao geral xn = C1 (1 + 3)n + C2 (1 + 3)n
para a recorrencia. Substituindo n = 1 e n = 2, obtemos

(1 + 3)C1 + (1 3)C2 = 3

(3 + 2 3)C1 + (3 2 3)C2 = 8

Resolvendo o sistema, encontramos C1 = 3+26 3 e C2 = 326 3 . Logo o
numero de sequencias
por n
termos iguais
a 0,
1 nou 2 sem dois zeros
32 3 n 3+2 3
repetidos e xn = 6 (1 + 3) + 6 (1 3) .
4. Considere um tabuleiro com 2 linhas e n + 2 colunas. Para preencher o
canto esquerdo do tabuleiro, ha duas alternativas: colocar um domino
em pe, restando tabuleiro com 2 linhas e n+1 colunas a preencher, ou
colocar dois dominos deitado restando um tabuleiro com 2 linhas e n
colunas. Logo, o numero xn de modos de preencher um tabuleiro 2 n
com dominos 2 1 satisfaz a recorrencia xn+2 = xn + xn+1 , com x1 = 1
e x 2 = 2. Esta e exatamente a sequencia de
 Fibonacci
 estudada no
 n+1  n+1
1 1+ 5 1 1 5
exemplo 4. Logo, temos xn = Fn = 5 2
5 2
.

5. No ano n + 2 sao geradas 21 sementes para cada semente gerada no


ano n + 1 e 44 sementes para cada semente gerada nos anos anteriores.
Logo, se xn denota o numero de sementes geradas no ano n, temos

xn+2 = 21xn+1 + 44(xn + xn1 + + x1 + x0 ),

com x1 = 1 e x2 = 44 + 21.21 = 485. Para transformar esta recorrencia


em uma recorrencia linear de segunda ordem, escrevemos a expressao
para xn+1 :

xn+1 = 21xn + 44(xn1 + xn2 + + x1 + x0 ),

Subtraindo as duas expressoes, obtemos:

xn+2 = 22xn+1 + 23xn ,

3
ou seja,
xn+2 22xn+1 23xn = 0
A equacao caracterstica r2 22r23 = 0 tem razes r1 = 23 e r2 = 1,
levando a solucao geral xn = C1 23n + C2 (1)n para a recorrencia.
Usando as condicoes iniciais, obtemos
23C1 C2 =21
529C1 + C2 =485
Resolvendo, encontramos C1 = 11/12 e C2 = 1/12. A solucao da
recorrencia e, assim, xn = 11
12
1
23n + 12 (1)n .
6. Seja pn a probabilidade de que a taca nao seja ganha nos primeiros
n torneios. Qualquer time pode ganhar o primeiro torneio. Vamos
exprimir pn+2 em funcao de pn e pn+1 usando probabilidade condicional.
Se o segundo torneio for ganho por um time diferente do que ganhou
o primeiro (o que ocorre com probabilidade 54 ), tudo se passa como se
a serie de torneios estivesse comecando no segundo torneio. Ou seja, a
probabilidade condicional de que a taca nao seja ganha ate o torneio
n + 2 e igual a pn+1 . Se o segundo torneio for ganho pelo mesmo time
do primeiro, mas no terceiro nao (ocorre com probabilidade 15 . 54 ), tudo
se passa como se a serie de torneios comecasse no terceiro jogo e a
probabilidade condicional de que a taca nao seja ganha ate o torneio
n + 2 torneio e igual a pn . Finalmente, se os tres primeiros torneios
forem ganhos pelo mesmo time, a taca e ganha na terceira realizacao e,
portanto, a probabilidade condicional de que ela nao tenha sido ganha
ate o torneio n + 2 e igual a zero, para todo n 1. Assim, temos
pn+2 = 45 pn+1 + 15 . 54 pn , com p1 = p2 = 1 (ja que a taca certamente
nao e ganha nas duas primeiras realizacoes).

A equacao

caracterstica
2 4 1 4 2+2 2 22 2
r 5 r 5 . 5 = 0 tem razes r1= 5 e r2 = 5 . Logo, a solucao
n n
2+2 2 22 2
geral da recorrencia e pn = C1 5
+ C2 5
.
Substituindo n = 1 e n = 2, temos:
! !
2+2 2 22 2
C1 + C2 = 1
5 5
! !
12 + 8 2 12 8 2
C1 + C2 = 1
25 25

4

10+5 2 105 2
Resolvendo, encontramos C1 =  eC =
16  2 16
, levando a solucao
n n
pn = 10+5
16
2 2+2 2
5
+ 105 2 22 2
16 5

7. Seja xn o numero de modos de obter 1 ou 2 pontos no primeiro jogo.


No primeiro caso, ele tem que obter n+1 pontos nos jogos seguintes; no
segundo caso, ele tem que obter n pontos a seguir. Logo, xn+2 = xn +
xn+1 , com x1 = 1 e x2 = 2. Esta e a recorrencia que define a sequencia
 n+1  n+1
de Fibonacci Fn . Logo, xn = Fn = 15 1+2 5 15 12 5 , para
todo n.

5
Referncias Bibliogrcas

[1] Carmo, Manfredo P.; Morgado, Augusto C., Wagner, Eduardo & Pitom-

beira, Joo Bosco. Trigonometria e Nmeros Complexos . Rio de Janeiro:

SBM, Coleo Professor de Matemtica.

[2] Eves, Howard. An Introduction to the History of Mathematics . New York:


Holt, Rinehart and Winston, 1964.

[3] Figueiredo, Djairo G. Anlise I Rio de Janeiro: LTC, 1996.

[4] Figueiredo, Djairo G. Nmeros Irracionais e Transcedentes Rio de Janeiro:

SBM, Coleo Iniciao Cientca.

[5] Halmos, Paul. Naive Set Theory. New York: Springer, 1974.

[6] Hefez, Abramo e Fernandez, Ceclia de Souza. Introduo lgebra Linear .


Rio de Janeiro: SBM, Coleo PROFMAT, 2012.

[7] Fernandes, C. S. Hefez, A. Introduo lgebra Linear . SBM, Coleo

PROFMAT. 2

[8] Lima, Elon Lages. Coordenadas no Espao. Rio de Janeiro: SBM, Coleo

Professor de Matemtica.

[9] Lima, Elon Lages. Curso de Anlise, Vol. 1. Rio de Janeiro: SBM, Projeto

Euclides, 1976.

[10] Lima, Elon Lages. Logaritmos. Rio de Janeiro: SBM, Coleo Professor de

Matemtica.

[11] Lima, Elon Lages. Meu Professor de Matemtica e Outras Histrias . Rio
de Janeiro: SBM, Coleo Professor de Matemtica.

bibitemelon-analisereal Lima, Elon Lages. Anlise Real, Vol. 1. Rio de Janeiro:


IMPA, Coleo Matemtica Universitria.

13
9
Matemtica Financeira

Sumrio
9.1 Introduo . . . . . . . . . . . . . . . . . . . . . . . 2

9.2 Juros Compostos . . . . . . . . . . . . . . . . . . . . 2

9.3 A Frmula das Taxas Equivalentes . . . . . . . . . . 7

9.4 Exerccios Recomendados . . . . . . . . . . . . . . . 9

1
Unidade 9 Introduo

9.1 Introduo

Nesta unidade e na prxima, sero apresentados rudimentos de Matemtica

Financeira, cuja inspirao vem da vida real. Esse conhecimento fundamental

em sociedades de consumo, como a nossa, e deve fazer parte da bagagem

cultural de todo cidado que nelas vive para que saiba defender minimamente

os seus interesses.

Quotidianamente, estamos frente a problemas prticos, tais como se deve-

mos ou no parcelar uma compra e, se for o caso, em quantas parcelas? Se

devemos ou no antecipar o pagamento de uma dvida, usando o dcimo ter-

ceiro salrio? Esses so desaos que, se resolvidos corretamente, nos auxiliam

a tomar decises que podem proporcionar uma boa economia.

A ferramenta matemtica bsica que utilizada nesse tipo de questes so

as progresses geomtricas, bastando, para resolv-las, modelar corretamente

cada problema.

O assunto principal de que tratamos o clculo de juros em diversas si-

tuaes decorrentes da operao de emprstimo, seja em aplicaes (quando

emprestamos), seja em compras a crdito (quando tomamos emprestado).

Esta unidade repousa sobre um resultado (teorema) fundamental que nos

diz como se transforma um capital inicial quando aplicado por um perodo de

tempo, sendo submetido a um regime de juros compostos.

9.2 Juros Compostos

Uma das importantes aplicaes de progresses geomtricas a Matem-

tica Financeira. A operao bsica da matemtica nanceira a operao de

emprstimo.

Algum que dispe de um capital C (chamado de principal ), empresta-o

a outrem por um certo perodo de tempo, e aps esse perodo, recebe o seu

capital C e volta, acrescido de uma remunerao J pelo emprstimo. Essa

remunerao chamada de juro. A soma C +J chamada de montante e ser


J
representada por M. A razo i= que a taxa de crescimento do capital,
C
ser sempre referida ao perodo da operao e chamada de taxa de juros.

2
Matemtica Financeira Unidade 9

Lcia tomou um emprstimo de R$ 100,00. Dois meses aps, pagou R$ Exemplo 1


140,00. Os juros pagos por Lcia so de R$ 40,00 e a taxa de juros de
40
= 0, 40 = 40% ao bimestre. O principal, que a dvida inicial de Lcia,
100
igual a R$ 100,00; o montante, que a dvida na poca do pagamento, de

R$ 140,00.

Manuel tomou um emprstimo de 100 reais, a juros de taxa 10% ao ms. Exemplo 2
Aps um ms, a dvida de Manuel ser acrescida de 0, 10 100 reais de juros

(pois J = iC ), passando a 110 reais. Se Manuel e seu credor concordarem em

adiar a liquidao da dvida por mais um ms, mantida a mesma taxa de juros,

o emprstimo ser quitado, dois meses depois de contrado, por 121 reais, pois

os juros relativos ao segundo ms sero de 0, 10 110 reais = 11 reais. Esses

juros assim calculados so chamados de juros compostos. Mais precisamente, no

regime de juros compostos , os juros em cada perodo so calculados, conforme

natural, sobre a dvida do incio desse perodo.

As pessoas menos educadas matematicamente tm tendncia a achar que

juros de 10% ao ms do em dois meses juros de 20%. Note que juros de 10%
ao ms do em dois meses de juros de 21%.

No regime de juros compostos de taxa i, um principalC0 transforma-se, Teorema 1


depois de n perodos de tempo, em um montante Cn = C0 (1 + i)n .

Basta observar que os valores do capital crescem a uma taxa constante i Demonstrao
e, portanto, formam uma progresso geomtrica de razo 1 + i.

Pedro investe 150 reais a juros de 12% ao ms. Qual ser o montante de Exemplo 3
Pedro trs meses depois?

Soluo. C3 = C0 (1 + i)3 = 150(1 + 0, 12)3 = 210, 74 reais.

importante perceber que o valor de uma quantia depende da poca qual

ela est referida. Se eu consigo fazer com que meu dinheiro renda 10% ao ms,

3
Unidade 9 Juros Compostos

-me indiferente pagar agora R$ 100,00 ou pagar R$ 110,00 daqui a um ms.

mais vantajoso pagar R$ 105,00 daqui a um ms do que pagar R$ 100,00

agora. mais vantajoso pagar R$ 100,00 agora do que pagar R$ 120,00 daqui

a um ms.

No fundo, s h um nico problema de Matemtica Financeira: deslocar

quantias no tempo.

Outro modo de ler o Teorema 1, Cn = C0 (1 + i)n , que uma quantia, hoje


igual a C0 , transformar-se-, depois de n perodos de tempo, em uma quantia
n
igual a C0 (1 + i) . Isto , uma quantia, cujo valor atual A, equivaler no
n
futuro, depois de n perodos de tempo, a F = A(1 + i) .

Essa a frmula fundamental da equivalncia de capitais: Para obter o valor

futuro, basta multiplicar o atual por (1 + i)n . Para obter o valor atual, basta

n
dividir o futuro por (1 + i) .
O exemplo a seguir , pode-se dizer, um resumo de todos os problemas de

Matemtica Financeira.

Exemplo 4 Pedro tomou um emprtimo de 300 reais, a juros de 15% ao ms. Dois

meses aps, Pedro pagou 150 reais e, um ms aps esse pagamento, Pedro

liquidou seu dbito. Qual o valor desse ltimo pagamento?

Soluo. Os esquemas de pagamento abaixo so equivalentes. Logo, 300 reais,

na data 0, tm o mesmo valor de 150 reais dois meses aps, mais um paga-

mento igual a P, na data 3.

Figura 9.1: Esquemas de pagamento

Igualando os valores, na mesma poca (0, por exemplo), dos pagamentos nos

dois esquemas, obtemos

150 p
300 = 2
= .
(1 + 0, 15) (1 + 0, 15)3

4
Matemtica Financeira Unidade 9

da, P = 283, 76. O ltimo pagamento foi de R$ 283,76.

Pedro tem duas opes de pagamento na compra de um televisor: Exemplo 5


i) trs prestaes mensais de R$ 160,00 cada;

ii) sete prestaes mensais de R$ 70,00 cada.

Em ambos os casos, a primeira prestao paga no ato da compra. Se o

dinheiro vale 2% ao ms para Pedro, qual a melhor opo que Pedro possui?

Soluo. Para comparar, determinaremos o valor dos dois conjuntos de paga-

mentos na mesma poca, por exemplo na poca 2. Os esquemas de pagamentos

so:

Figura 9.2: Esquemas de pagamento

Para comparar, determinaremos o valor dos dois conjuntos de pagamentos na

mesma poca. Por exemplo, na poca 2, temos,

a = 60(1 + 0, 02)2 + 160(1 + 0, 02) + 160 = 489, 66


70
b = 70(1 + 0, 02)2 + 70(1 + 0, 02) + 70 +
1 + 0, 02
70 70 70
+ + + = 480, 77.
(1 + 0, 02)2 (1 + 0, 02)3 (1 + 0, 02)4

Pedro deve preferir o pagamento em seis prestaes.

um absurdo que muitas pessoas razoavelmente instrudas achem que o

primeiro esquema melhor pois o total pago de R$ 480,00 ao passo que no

segundo esquema o total pago de R$ 490,00.

Para xar, faremos mais alguns exemplos.

5
Unidade 9 Juros Compostos

Exemplo 6 Pedro tem trs opes de pagamento na compra de vesturio.

i) vista, com 30% de desconto.

ii) em duas pretaes mensais iguais, sem desconto, vencendo a primeira um

ms aps a compra.

iii) em trs prestaes mensais iguais, sem desconto, vencendo a primeira no

ato da compra.

Qual a melhor opo para Pedro, se o dinheiro vale, para ele, 25% ao ms?

Soluo. Fixando o preo do bem em 30, temos os trs esquemas abaixo

Figura 9.3: Esquemas de pagamento

Comparando os valores, por exemplo, na poca 0, obtemos:

a = 21
15 15
b = + = 21.6
1 + 0, 25 (1 + 0, 25)2
10 10
c = 10 + + = 24, 4.
1 + 0, 25 (1 + 0, 25)2

A melhor alternativa a primeira e a pior a em trs prestaes.

Exemplo 7 Uma loja oferece duas opes de pagamento:

i) vista, com 30% de desconto.

ii) em duas prestaes mensais iguais, sem desconto, a primeira prestao sendo

paga no ato da compra.

Qual a taxa mensal dos juros embutidos nas vendas a prazo?

6
Matemtica Financeira Unidade 9

Soluo. Fixando o valor do bem em 100, temos os esquemas de pagamentos

abaixo:

Figura 9.4: Esquemas de pagamento

Igualando os valores, por exemplo, na poca 0 (a data usada nessas comparaes


50
chamada de data focal), obtemos 70 = 50 + . Da, i = 1, 5 = 150%. A
1+i
loja cobra 150% ao ms nas vendas a prazo.

Investindo seu capital a juros mensais de 8%, em quanto tempo voc Exemplo 8
dobrar o seu capital inicial?

Soluo. Temos C0 (1 + 0, 08)n = 2C0 . Da,

log 2
1, 08n = 2 e n= =9
log 1, 08
Em aproximadamente nove meses voc dobrar o seu capital inicial.

9.3 A Frmula das Taxas Equivalentes

Um importante resultado que j foi obtido na Unidade 6 e ser repetido a

Frmula das taxas equivalentes. Se a taxa de juros relativamente a um

determinado perodo de tempo igual a i, a taxa de juros relativamente a n


perodos de tempo I tal que 1 + I = (1 + i)n .

A taxa anual de juros equivalente a 12% ao ms I tal que 1+I = Exemplo 9


(1 + 0, 12)12
. Da, I
= 2, 90 = 290% ao ano.

7
Unidade 9 A Frmula das Taxas Equivalentes

Um erro muito comum achar que juros de 12% ao ms equivalem a juros

anuais de 12 12% = 144% ao ano. Taxas como 12% ao ms e 144% ao ano

so chamadas de taxas proporcionais, pois a razo entre elas igual razo

dos perodos aos quais elas se referem.

Taxas proporcionais no so equivalentes. Um (pssimo) hbito em Ma-

temtica Financeira o de anunciar taxas proporcionais como se fossem equi-

valentes. Uma frase como 144% ao ano, com capitalizao mensal signica

que a taxa usada na operao no a taxa de 144% anunciada e sim a taxa

mensal que lhe proporcional.

Portanto, a traduo da expresso 144% ao ano, com capitalizao mensal

12% ao ms. As pessoas menos educadas matematicamente podem pensar

que os juros sejam realmente de 144% ao ano, mas isso no verdade. Como

vimos no Exemplo 9, os juros so de 290% ao ano.

A taxa de 144% ao ano chamada de taxa nominal e a taxa de 290% ao

ano chamada de taxa efetiva.

Exemplo 10 24% ao ano com capitalizao semestral signica 12% ao semestre; 1%

ao ms com capitalizao trimestral signica 3% ao trimestre e 6% ao ano

com capitalizao mensal signica 0,5% ao ms.

Exemplo 11 Vernica investe seu dinheiro a juros de 6% ao ano com capitalizao

mensal. Qual a taxa anual de juros qual est investido o capital de Vernica?

Soluo. O dinheiro de Vernica est investido a juros de taxa i = 0, 5% ao ms.


A taxa anual equivalente a I tal que 1+I = (1+i)12 . Da, I = 0, 0617 = 6, 17%

ao ano. A taxa de 6% ao ano nominal e a taxa de 6,17% ao ano efetiva.

Exemplo 12 A taxa efetiva semestral correspondente a 24% ao semestre com capitali-

zao mensal I tal que 1 + I = (1 + 0, 04)6 . Da, I = 26, 53% ao semestre.

8
Matemtica Financeira Unidade 9

9.4 Exerccios Recomendados

1. Investindo R$ 450,00 voc retira, aps 3 meses, R$ 600,00. A que taxa

mensal de juros rendeu seu investimento?

2. Determine as taxas mensais equivalentes a 100% ao ano e a 39% ao trimes-

tre.

3. Determine as taxas anuais equivalentes a 6% ao ms e a 12% ao trimestre.

4. Determine as taxas efetivas anuais equivalente a:

a) 30% ao ano, com capitalizao mensal.

b) 30% ao ano, com capitalizao trimestral.

c) i ao ano, capitalizados k vezes ao ano.

5. Qual o limite, quando k tende para innito, da resposta ao item c) do

problema anterior? Neste caso diz-se que os juros esto sendo capitalizados

continuamente e i chamado de taxa instantnea de juros.

6. Use a resposta do problema anterior para dar uma denio nanceira do

nmero e.

7. Determine

a) a taxa efetiva trimestral equivalente a 12% ao trimestre com capitalizao

contnua.

b) a taxa instantnea anual equivalente taxa efetiva anual de 60%.

c) a taxa instantnea semestral equivalente taxa efetiva anual de 60%.

8. A Mesbla, em vrios natais, ofereceu a seus clientes duas alternativas de

pagamento:

a) pagamento de uma s vez, um ms aps a compra.

b) pagamento em trs prestaes mensais iguais, vencendo a primeira no ato

da compra.

Se voc fosse cliente da Mesbla, qual seria a sua opo?

9. O Foto Studio Sonora convidou, em dezembro de 1992, os seus clientes

a liquidarem suas prestaes mensais vincendas, oferecendo-lhes em troca um

desconto. O desconto seria dado aos que pagassem, de uma s vez, todas as

prestaes a vencer em mais de 30 dias, e seria de 30%, 40% ou 50%, conforme

9
Unidade 9 Exerccios Recomendados

fossem pagas uma, duas ou trs prestaes. Supondo que o dinheiro valia 27%

ao ms, a oferta era vantajosa?

10. Lcia comprou um exaustor, pagando R$ 180,00, um ms aps a compra

e R$ 200,00, dois meses aps a compra. Se os juros so de 25% sobre o saldo

devedor, qual o preo vista?

11. Uma geladeira custa R$ 1 000,00 vista e pode ser paga em trs prestaes

mensais iguais. Se so cobrados juros de 6% ao ms sobre o saldo devedor,

determine o valor da prestao, supondo que a primeira prestao paga:

a) no ato da compra;

b) um ms aps a compra;

c) dois meses aps a compra.

12. ngela tomou um emprstimo de R$ 400,00, por dez meses. Os juros

foram de 3% ao ms durante os quatro primeiros meses, de 5% ao ms durante

os cinco meses seguintes e de 9% ao ms no ltimo ms. Calcule:

a) a taxa mdia de juros.

b) o montante pago.

10
MA12 - Unidade 9
Matematica Financeira

Paulo Cezar Pinto Carvalho

PROFMAT - SBM

31 de Marco de 2013
Conceitos Basicos

Principal (P): capital investido


Juro (J): remuneracao do capital
Montante (M = P + J): principal acrescido do juro
J
Taxa de juros (i = ): razao entre juro e principal (referido a
P
duracao do investimento)

PROFMAT - SBM MA12 - Unidade 9, Matematica Financeira slide 2/14


Juros Compostos

Regime de investimento em que o capital e reinvestido por


sucessivos perodos, acrescido dos juros.
(que sempre incidem sobre o capital acumulado).
Exemplo: Principal de R$ 10000,00, investidos por 3 meses a
taxa de 2% ao mes.
Ao final do 1o mes: M = 10000 + 0, 02 10000 = 10200
Ao final do 2o mes: M = 10200 + 0, 02 10200 = 10404
Ao final do 3o mes: M = 10404 + 0, 02 10404 = 10612, 08

PROFMAT - SBM MA12 - Unidade 9, Matematica Financeira slide 3/14


Teorema

No regime de juros compostos de taxa i, um principal C0


transforma-se, depois de n perodos de tempo, em um
montante Cn = C0 (1 + i)n .
No exemplo anterior, o montante ao final de 3 meses e:
M = 10000 (1 + 0, 02)3 = 10612, 08

PROFMAT - SBM MA12 - Unidade 9, Matematica Financeira slide 4/14


Juros compostos juros simples

Juros simples: M = C (1 + n i)
Juros compostos: M = C (1 + i)n
Muito raramente, o modelo de juros simples e realista, ja que
os juros estao disponveis para reinvestimento.

PROFMAT - SBM MA12 - Unidade 9, Matematica Financeira slide 5/14


O dinheiro no tempo

Valor acumulado apos n perodos: Cn = C0 (1 + i)n .


C0 e o valor atual do capital.
Cn e o valor futuro do capital
Para deslocar uma quantia para o futuro: multiplicar por
1 + i em cada perodo.
Para deslocar uma quantia para o passado: dividir por 1 + i
em cada perodo.

PROFMAT - SBM MA12 - Unidade 9, Matematica Financeira slide 6/14


Comparando fluxos de caixa

Um fluxo de caixa e uma sequencia de pagamentos e


recebimentos ao longo do tempo.
Para comparar fluxos de caixa e essencial reduzi-los a uma
mesma data, movimentando, se necessario, as quantias ao
longo do tempo.

PROFMAT - SBM MA12 - Unidade 9, Matematica Financeira slide 7/14


Exemplo
Pedro tomou um emprestimo de 300 reais, a juros de 15% ao
mes. Dois meses apos, Pedro pagou 150 reais e, um mes apos
esse pagamento, Pedro liquidou seu debito. Qual o valor desse
ultimo pagamento?
1) Representar os pagamentos ao longo do tempo:

2) Escolher uma epoca para igualar os pagamentos: 0, por


exemplo.
3) Igualar os pagamentos nesta data:
150 P
300 = + .
(1 + 0, 15)2 (1 + 0, 15)3
O ultimo pagamento foi P = R$ 283,76.
PROFMAT - SBM MA12 - Unidade 9, Matematica Financeira slide 8/14
Exemplo

Pedro tem duas opcoes de pagamento na compra de um


televisor:
i) tres prestacoes mensais de R$ 160,00 cada;
ii) sete prestacoes mensais de R$ 70,00 cada.
Em ambos os casos, a primeira prestacao e paga no ato da
compra. Se o dinheiro vale 2% ao mes para Pedro, qual a
melhor opcao?

PROFMAT - SBM MA12 - Unidade 9, Matematica Financeira slide 9/14


Solucao

Os esquemas de pagamento:

Calculando os valores na epoca 2:

a = 160(1 + 0, 02)2 + 160(1 + 0, 02) + 160 = 489, 66


70
b = 70(1 + 0, 02)2 + 70(1 + 0, 02) + 70 +
1 + 0, 02
70 70 70
+ + + = 480, 77.
(1 + 0, 02)2 (1 + 0, 02)3 (1 + 0, 02)4

Pedro deve preferir o pagamento em sete prestacoes.

PROFMAT - SBM MA12 - Unidade 9, Matematica Financeira slide 10/14


Uma loja oferece duas opcoes de pagamento:
i) a vista, com 30% de desconto.
ii) em duas prestacoes mensais iguais, sem desconto, a primeira
prestacao sendo paga no ato da compra.
Qual a taxa mensal dos juros embutidos nas vendas a prazo?
Esquemas de pagamento:

Igualando os valores na epoca 0:


50
70 = 50 + .
1+i
Da, i = 1, 5 = 150%

PROFMAT - SBM MA12 - Unidade 9, Matematica Financeira slide 11/14


A Formula das Taxas Equivalentes

Se a taxa de juros relativamente a um determinado perodo de


tempo e igual a i, a taxa de juros relativamente a n perodos
de tempo e I tal que 1 + I = (1 + i)n .

Exemplo: A taxa anual de juros equivalente a 12% ao mes e


I tal que 1 + I = (1 + 0, 12)12 Da,I 2, 90 = 290% ao ano.

PROFMAT - SBM MA12 - Unidade 9, Matematica Financeira slide 12/14


Taxas equivalentes e taxas proporcionais

Uma taxa mensal de 2% ao mes nao resulta em uma taxa


anual de 24% ao ano, e sim de
1, 0212 1 1, 268 1 = 26, 8%.
Ha, no entanto, o (mau) habito de se referir a esta taxa como
sendo de 24% ao ano, capitalizados mensalmente.
A taxa de 24% ao ano e a taxa proporcional, enquanto a de
26, 8% ao ano e a taxa efetiva.

PROFMAT - SBM MA12 - Unidade 9, Matematica Financeira slide 13/14


Exemplo

Um capital e a juros de 6% ao ano com capitalizacao mensal.


Qual a taxa anual de juros a qual esta investido o capital?
A taxa de 6% ao ano e a taxa proporcional, que corresponde a
6
uma taxa mensal de i = 12 = 0, 5% ao mes.
A taxa efetiva anual e I tal que 1 + I = 1, 00512 , o que
fornece I = 0, 0617.
Logo, a taxa efetiva anual e de 6, 17%.

PROFMAT - SBM MA12 - Unidade 9, Matematica Financeira slide 14/14


Lista de Exerccios
Unidade 9

1. Investindo R$ 450,00 voce retira, apos 3 meses, R$ 600,00. A que taxa


mensal de juros rendeu seu investimento?
2. Determine as taxas mensais equivalentes a 100% ao ano e a 39% ao
trimestre.
3. Determine as taxas anuais equivalentes a 6% ao mes e a 12% ao trimestre.
4. Determine as taxas efetivas anuais equivalente a:
a) 30% ao ano, com capitalizacao mensal.
b) 30% ao ano, com capitalizacao trimestral.
c) i ao ano, capitalizados k vezes ao ano.
5. Qual o limite, quando k tende para infinito, da resposta ao item c) do
problema anterior? Neste caso diz-se que os juros estao sendo capitalizados
continuamente e i e chamado de taxa instantanea de juros.
6. Use a resposta do problema anterior para dar uma definicao financeira do
numero e.
7. Determine
a) a taxa efetiva trimestral equivalente a 12% ao trimestre com capitalizacao
contnua.
b) a taxa instantanea anual equivalente a taxa efetiva anual de 60%.
c) a taxa instantanea semestral equivalente a taxa efetiva anual de 60%.
8. A Mesbla, em varios natais, ofereceu a seus clientes duas alternativas de
pagamento:
a) pagamento de uma so vez, um mes apos a compra.
b) pagamento em tres prestacoes mensais iguais, vencendo a primeira no ato
da compra.
Se voce fosse cliente da Mesbla, qual seria a sua opcao?
9. O Foto Studio Sonora convidou, em dezembro de 1992, os seus clientes a
liquidarem suas prestacoes mensais vincendas, oferecendo-lhes em troca um
desconto. O desconto seria dado aos que pagassem, de uma so vez, todas as
prestacoes a vencer em mais de 30 dias, e seria de 30%, 40% ou 50%, conforme

1
fossem pagas uma, duas ou tres prestacoes. Supondo que o dinheiro valia
27% ao mes, a oferta era vantajosa?
10. Lucia comprou um exaustor, pagando R$ 180,00, um mes apos a compra
e R$ 200,00, dois meses apos a compra. Se os juros sao de 25% sobre o saldo
devedor, qual e o preco a vista?
11. Uma geladeira custa R$ 1 000,00 a vista e pode ser paga em tres
prestacoes mensais iguais. Se sao cobrados juros de 6% ao mes sobre o saldo
devedor, determine o valor da prestacao, supondo que a primeira prestacao
e paga:
a) no ato da compra;
b) um mes apos a compra;
c) dois meses apos a compra.
12. Angela tomou um emprestimo de R$ 400,00, por dez meses. Os juros
foram de 3% ao mes durante os quatro primeiros meses, de 5% ao mes durante
os cinco meses seguintes e de 9% ao mes no ultimo mes. Calcule:
a) a taxa media de juros.
b) o montante pago.
13. Numa operacao de desconto bancario, o cliente assina uma promissoria
de um certo valor, mas recebe um valor menor, obtido aplicando uma taxa
de desconto sobre o valor da promissoria. Por exemplo, se a promissoria e de
R$ 5000,00 e a taxa de desconto e de 10%, o cliente recebe R$ 4500,00 e se
compromete a pagar R$ 5000,00 ao banco. Nesta operacao qual e a taxa de
juro que esta sendo efetivamente cobrada pelo banco?

2
Solucoes da Lista de Exerccios
Unidade 9

1. 600 = 450(1 + i)3


600 1/3

i = 450 1 = 10, 06%

2. (a) 1 + I = (1 + i)12
1 + 1 = (1 + i)12
i = 21/12 1 = 5, 95%
(b) 1 + I = (1 + i)3
1 + 0, 39 = (1 + i)3
i = 1, 391/3 1 = 11, 60%

3. (a) 1 + I = (1 + i)12
I = 1, 0612 1 = 101, 22%
(b) 1 + I = (1 + i)4
I = 1, 124 1 = 57, 37%

4. (a) A taxa e de 30%/12 = 2, 5% ao mes.


1 + I = (1 + i)12
I = 1, 02512 1 = 34, 49%
(b) A taxa e de 30%/4 = 7, 5% ao trimestre.
1 + I = (1 + i)4
I = 1, 0754 1 = 33, 55%.
(c) A taxa relativa ao perodo de capitalizacao e i/k.
1 + I = (1 + ki )k
I = (1 + ki )k 1

5. lim(1 + ki )k 1 = ei 1

6. O numero e e o valor de montante gerado em um ano por um principal


igual a 1, a juros de 100% ao ano, capitalizados continuamente.

1
7. (a) e 1 = e0,12 1 = 12, 75%
(b) ln(1 + i) = ln 1, 6 = 47, 00%
(c) Aproveitando o tem anterior, 47, 00%/2 = 23, 50%.

8. Seja 0 a data de compra. Seja 3 o preco do artigo. Usemos a data 1


como data focal.
Na alternativa a), paga-se A = 3.
1
Na alternativa b), paga-se B = 1+i
+ 1 + (1 + i)
i2
BA= 1+i
>0
Logo, como B > A, a alternativa a) e prefervel.

9. (a) Supondo uma prestacao vincenda de 100 e tomando a data atual


como focal:
aceitando: pago, na data 0, 70.
nao aceitando: pago, na data 1, 100, o que equivale a pagar, na
data 0,
100
1+0,27
= 78, 74
A proposta e vantajosa.
(b) Supondo duas prestacoes vincendas de 100 cada uma e tomando
a data atual como focal:
aceitando: pago, na data 0, 120.
nao aceitando: pago, na data 1, 100, e na data 2, 100, o que
equivale a pagar na data 0,
100 100
1+0,27
+ (1+0,27)2 = 140, 74

A proposta e vantajosa.
(c) Supondo tres prestacoes vincendas de 100 cada uma e tomando a
data atual como focal:
aceitando: pago, na data 0, 150.
nao aceitando: pago, na data 1, 100, na data 2, 100, e, na data 3,
100, o que equivale a pagar, na data 0,
100 100 100
1+0.27
+ (1+0.27)2 + (1+0.27)3 = 189, 56

A proposta e vantajosa

2
10. Trazendo os valores a data da compra, obtemos, para o preco a vista,
180 200
+ = R$272, 00.
1, 25 1, 252
11. (a) Tomando a data focal um mes antes da compra,
3
P
= 1000 11,06
1,06 0,06
P = 352, 93
(b) Tomando a data focal no ato da compra,
3
P = 1000 11,06
0,06
P = 374, 11
(c) Tomando a data focal um mes depois da compra,
3
P.1, 06 = 1000 11,06
0,06
P = 396, 56

12. O montante pago foi 400.1, 034 .1, 055 .1, 09 = 626, 30.
A taxa media de juros e calculada por 626, 30 = 400.(1+i)10 , i = 4, 59%
ao mes.

13. A taxa de juros i e tal que 5000 = 4500(1+i), o que fornece i = 0, 111....
Logo, a taxa de juro que esta sendo efetivamente cobrada e de 11, 1%.

3
Referncias Bibliogrcas

[1] Carmo, Manfredo P.; Morgado, Augusto C., Wagner, Eduardo & Pitom-

beira, Joo Bosco. Trigonometria e Nmeros Complexos . Rio de Janeiro:

SBM, Coleo Professor de Matemtica.

[2] Eves, Howard. An Introduction to the History of Mathematics . New York:

Holt, Rinehart and Winston, 1964.

[3] Figueiredo, Djairo G. Anlise I Rio de Janeiro: LTC, 1996.

[4] Figueiredo, Djairo G. Nmeros Irracionais e Transcedentes Rio de Janeiro:

SBM, Coleo Iniciao Cientca.

[5] Halmos, Paul. Naive Set Theory . New York: Springer, 1974.

[6] Hefez, Abramo e Fernandez, Ceclia de Souza. Introduo lgebra Linear .

Rio de Janeiro: SBM, Coleo PROFMAT, 2012.

[7] Fernandes, C. S. Hefez, A. Introduo lgebra Linear . SBM, Coleo

PROFMAT. 2

[8] Lima, Elon Lages. Coordenadas no Espao . Rio de Janeiro: SBM, Coleo

Professor de Matemtica.

[9] Lima, Elon Lages. Curso de Anlise , Vol. 1. Rio de Janeiro: SBM, Projeto

Euclides, 1976.

[10] Lima, Elon Lages. Logaritmos. Rio de Janeiro: SBM, Coleo Professor de

Matemtica.

[11] Lima, Elon Lages. Meu Professor de Matemtica e Outras Histrias . Rio

de Janeiro: SBM, Coleo Professor de Matemtica.

bibitemelon-analisereal Lima, Elon Lages. Anlise Real, Vol. 1. Rio de Janeiro:

IMPA, Coleo Matemtica Universitria.

11
10
Matemtica Financeira
(continuao)

Sumrio
10.1 Introduo . . . . . . . . . . . . . . . . . . . . . . . 2

10.2 Renda Perptua . . . . . . . . . . . . . . . . . . . . 2

10.3 Sistemas de Amortizao . . . . . . . . . . . . . . . 6

10.4 Exerccios Recomendados . . . . . . . . . . . . . . . 12

10.5 Exerccios Suplementares . . . . . . . . . . . . . . . 13

1
Unidade 10 Introduo

10.1 Introduo

Nesta unidade, continuaremos o estudo de Matemtica Financeira iniciado

na Unidade 9 e que se encerrar na prxima unidade. Os principais resultados

dessa unidade analisam essencialmente trs tipos de emprstimos, geralmente

de longo prazo como, por exemplo, nanciamentos da casa prpria ou de bens

durveis.

O primeiro tipo de emprstimo, refere-se situao em que a taxa de juros

pr-xada e o valor da prestao tambm. O Teorema 2 fornece uma frmula

que permite saber quanto da dvida foi pago aps n pagamentos (amortizao

da dvida).

Cada parcela paga de um emprstimo consiste de duas partes: uma se refere

ao pagamento dos juros e a outra se refere ao abatimento do principal da dvida,

chamada de amortizao.

O segundo tipo de emprstimo estudado o Sistema de Amortizao Cons-

tante (SAC), em que a parte da prestao que visa amortizar a dvida cons-
tante. O Teorema 4 permite calcular a cada ms o valor da prestao especi-

cando o valor da amortizao (constante), o valor da parcela relativa aos juros

(varivel) e, nalmente, o estado atual da dvida (no caso da pessoa querer

quitar a dvida, por exemplo).

O terceiro tipo de emprstimo o Sistema Francs ou Tabela Price, em que

as prestaes e a taxa de juros so constantes. O Teorema 5 fornece frmulas

para calcular, ms a ms, o valor da prestao (constante), a parcela relativa

amortizao do principal, a parcela relativa aos juros pagos e o estado atual da

dvida.

Os clculos nanceiros podem se complicar bastante em presena de forte

inao, como foi o caso no Brasil alguns anos atrs.

10.2 Renda Perptua

Um conjunto de quantias (chamadas usualmente de pagamentos ou termos),

referidas a pocas diversas, chamada de srie, ou de anuidade (apesar no

nome, nada a ver com ano) ou, ainda, renda. Se esses pagamentos forem iguais

e igualmente espaados no tempo, a srie dita uniforme.

2
Matemtica Financeira (continuao) Unidade 10

O valor de uma srie uniforme de n pagamentos iguais a P , um tempo antes Teorema 2


1 (1 + i)n
do primeiro pagamento, , sendo i a taxa de juros, igual a A = P .
i

Demonstrao

Figura 10.1: Srie uniforme

O valor da srie na poca 0

P P P P
A= + 2
+ 3
+ + ,
1 + i (1 + i) (1 + i) (1 + i)n

que a soma de n termos de uma progresso geomtrica. temos

 n
1
1
P 1+i 1 (1 + i)n
A= 1 =P .
1+i 1 1+i
i

O corolrio seguinte trata do valor de uma renda perptua. Rendas perp-

tuas aparecem em locaes. Com efeito, quando se aluga um bem, cede-se a

posse do mesmo em troca de um aluguel, digamos, mensal. Ento, o conjunto

dos aluguis constitui uma renda perptua ou perpetuidade.

O valor de uma perpetuidade de termos iguais a P , um tempo antes do Corolrio 3


P
primeiro pagamento, , sendo i a taxa de juros, igual a .
i

Basta fazer n tender para innito no Teorema 2.


Demonstrao

3
Unidade 10 Renda Perptua

Exemplo 1 Um bem, cujo preo R$ 120,00, vendido em 8 prestaes mensais iguais,

a primeira sendo paga um ms aps a compra. Se os juros so de 8% ao ms,

determine o valor das prestaes.

Soluo. Um pequeno comentrio: essas prestaes so ditas postecipadas,

pois a primeira prestao s paga um tempo depois da compra.

Figura 10.2:

Igualando os valores na poca 0 (essa a escolha natural da data de comparao:

um tempo antes do primeiro termo da srie), obtemos:

1 (1 + 0, 08)8
120 = P
0, 08
0, 08
P = 120 = 20, 88.
1 0, 088

As prestaes so de R$ 20,88.

Exemplo 2 Um bem, cujo preo vista R$ 120,00, vendido em 6 prestaes mensais

iguais, antecipadas (isto , a primeira paga no ato da compra). Se os juros

so de 10% ao ms, determine o valor das prestaes.

4
Matemtica Financeira (continuao) Unidade 10

Figura 10.3: Comparando sries

Igualando os valores na poca 1 (essa escolha, que pode parecer extica,

muito conveniente pois dispomos de uma frmula que calcula diretamente o

valor da srie nessa poca), obtemos:

120 1 (1 + 0, 1)6
=P
1 + 0, 1 0, 1
P
= 25, 05.

Se o dinheiro vale 1% ao ms, por quanto deve ser alugado um imvel que Exemplo 3
vale 40 mil reais?

Soluo. Quando voc aluga um imvel, voc cede a posse do imvel em troca
de uma renda perptua cujos termos so iguais ao valor do aluguel. Ento,

o valor do imvel deve ser igual ao valor do conjunto de aluguis. Temos, de

acordo com o Corolrio 3,

P P
40 = = = 40 0, 01 = 0, 4 mil reais.
i 0, 01

Helena tem duas alternativas para obter uma copiadora: Exemplo 4


a) Alug-la por 35 ao ano. Nesse caso, o locador se responsabiliza pelas despesas

de manuteno.

b) Compr-la por 150. Nesse caso, j que a vida econmica da copiadora de

5 anos, Helena vender a copiadora aps 5 anos. O valor residual da copiadora

5
Unidade 10 Sistemas de Amortizao

aps 5 anos de 20. As despesas de manuteno so de responsabilidade de

Helena e so de 5 por ano, nos dois primeiros anos e de 8 por ano, nos anos

seguintes. Se o dinheiro vale 7% ao ano, qual a melhor opo?

Soluo. Vamos tomar receitas como positivas e despesas como negativas.


Na segunda alternativa, o uxo de caixa de Helena ser:

Figura 10.4: Alternativa (a)

Vamos determinar o uxo uniforme equivalente.

Figura 10.5: Alternativa (b)

Igualando os valores na poca 0, obtemos

5 5 8 8 12 1 1, 075
150 + = P .
1, 07 1, 072 1, 073 1, 074 1, 075 0, 07

Da, P = 39, 78. Comprar a copiadora equivalente a ter um custo anual

de 39,78. Como o aluguel corresponde a um custo anual de 35,00, a melhor

alternativa para Helena alugar.

10.3 Sistemas de Amortizao

Quando um banco empresta dinheiro (crdito pessoal ou desconto de dupli-

catas), o tomador do emprstimo emite uma nota promissria, que um papel

6
Matemtica Financeira (continuao) Unidade 10

no qual o tomador se compromete a pagar ao banco, em uma data xada, uma

certa quantia, que chamada de valor de face da promissria.

O banco ento desconta a promissria para o cliente, isto , recebe a pro-

missria de valor de face F e entrega ao cliente uma quantia A (menor que F,


naturalmente). A diferena F A chamada de desconto.

Os bancos efetuam o desconto de acordo com a frmula A = F (1 d . t),


onde d uma taxa xada pelo banco e chamada de taxa de desconto bancrio

(ou taxa de desconto simples por fora) e t o prazo da operao, medido na

unidade de tempo a que se refere a taxa.

Pedro desconta uma promissria de valor 100, com vencimento em 60 dias, Exemplo 5
em um banco cuja taxa de desconto de 12% ao ms.

a) Quanto Pedro receber?

b) Qual a taxa mensal de juros que Pedro est pagando?

Soluo. Ora, A = F (1 dt) = 100(1 0, 12 . 2) = 76.


Logo, Pedro receber agora 76, para pagar 100 em 60 dias.

Se i a taxa mensal de juros qual cresce a dvida de Pedro, temos 100 =


76(1 + i)2 . Da, i = 0, 1471 = 14, 71%.
Observe que anunciar a taxa de desconto e no a taxa de juros um modo sutil

de fazer crer aos mais ingnuos estarem eles pagando juros menores que os que

realmente lhes esto sendo cobrados.

Quando se paga parceladamente um dbito, cada pagamento efetuado tem

dupla nalidade. Uma parte do pagamento quita os juros e outra parte amortiza

(abate) a dvida.

Pedro tomou um emprstimo de 100, a juros mensais de taxa 10%. Quitou- Exemplo 6
-o em trs meses, pagando a cada ms os juros devidos e amortizando 30% da

dvida no primeiro ms e 30% e 40% nos dois meses seguintes.

Na planilha abaixo, Ak , Jk , Pk e Dk so, respectivamente, a parcela de

amortizao, a parcela de juros, a prestao e o estado da dvida (isto , o valor

da dvida aps o pagamento da prestao) na poca k.

7
Unidade 10 Sistemas de Amortizao

k Pk Ak Jk Dk
0 100
1 40 30 10 70
2 37 30 7 40
3 44 40 4
Para facilitar a compreenso, olhe cada linha na ordem Ak , Dk , Jk e Pk .

Os sistemas usuais de amortizao so o sistema de amortizao constante

(SAC) e o sistema francs de amortizao, tambm chamado de Tabela Price

(Richard Price foi um economista ingls). O sistema francs caracterizado

por prestaes constantes.

Exemplo 7 Uma dvida de 100 paga, com juros de 15% ao ms, em 5 meses, pelo

SAC. Faa a planilha de amortizao.


1
Soluo. Como as amortizaes so iguais, cada amortizao ser de da
5
dvida inicial.

A planilha , portanto:

k Pk Ak Jk Dk
0 100
1 35 20 15 80
2 32 20 12 60
3 29 20 9 40
4 26 20 6 20
5 23 20 3
Para facilitar a compreenso, olhe cada linha na ordem Ak , Dk , Jk e Pk .

Teorema 4 No SAC, sendo n o nmero de pagamentos e i a taxa de juros, temos

D0 nk
Ak = , Dk = D0 , Jk = iDk1 , Pk = Ak + Jk .
n n

Demonstrao Se a dvida D0 amortizada em n quotas iguais, cada quota igual a

D0
Ak = .
n

8
Matemtica Financeira (continuao) Unidade 10

O estado da dvida, aps k amortizaes,

D0 nk
Dk = D0 k = D0 .
n n

As duas ltimas frmulas so bvias.

Uma dvida de 150 paga, em 4 meses, pelo sistema francs, com juros de Exemplo 8
8% ao ms. Faa a planilha de amortizao.

No sistema francs, as prestaes so constantes. Pelo Teorema 4, cada

prestao vale

i 0, 08
P = D0 n
= 150 = 45, 29.
1 (1 + n) 1 1, 084

k Pk Ak Jk Dk
0 150, 00
1 45, 29 33, 29 12, 00 116, 71
2 45, 29 35, 95 9, 34 80, 76
3 45, 29 38, 83 6, 46 41, 93
4 45, 29 41, 93 3, 35

Para mais fcil compreenso, olhe cada linha na ordem Pk , Jk , Ak e Dk .

No sistema francs de amortizao, sendo n o nmero de pagamentos e i Teorema 5


a taxa de juros, temos

i
Pk = D0 ,
1 (1 + i)n
1 (1 + i)(nk)
Dk = D0 ,
1 (1 + i)n
Jk = iDk1 , A = Pk Jk .

9
Unidade 10 Sistemas de Amortizao

Demonstrao A primeira frmula simplesmente o Teorema 4 e as duas ltimas frmulas

so bvias. Quanto segunda frmula, observe que Dk a dvida que ser

liquidada, postecipadamente, por nk pagamentos sucessivos a Pk . Portanto,

novamente pelo Teorema 4, temos

1 (1 + i)(nk)
Dk = Pk .
i
Substituindo o valor de Pk , obteremos a segunda frmula.

Exemplo 9 Em um ms cuja inao foi de 25%, Paulo Jorge investiu seu capital a

juros de 30% ao ms. Evidentemente, isso no signica que Paulo Jorge tenha

aumentado seu poder de compra em 30%, pois, embora a quantidade de reais de

Paulo Jorge tenha crescido 30%, o valor do real sofreu uma reduo. Dizemos

nesse caso que 30% ao ms a taxa nominal de juros mensais de Paulo Jorge.

Suponhamos que, no incio do referido ms, o capital C de Paulo Jorge

pudesse comprar x artigos de preo unitrio igual a p. No m do ms, o capital


passou a ser 1, 3C e o preo unitrio passou a ser 1, 25p. Logo, Paulo Jorge
poder agora comprar
1, 3C
= 1, 04x artigos.
1, 25p
O poder de compra de Paulo Jorge aumentou de 4% nesse ms.

Essa taxa de 4% ao ms, qual cresceu o poder de compra de Paulo Jorge,

chamada de taxa real de juros .

Exemplo 10 Em algumas situaes (prazos pequenos, juros de mora) so usados juros

simples e no juros compostos. No regime de juros simples, os juros em cada

poca so calculados sobre o principal e no sobre o montante da poca anterior.

Por exemplo, um principal igual a 100, a juros simples de 10% ao ms evolui

de acordo com a tabela abaixo:

n 0 1 2 3 4 ...
Cn 100 110 120 130 140 . . .
No h diculdade em calcular juros simples pois a taxa incide sempre sobre o

capital inicial. No nosso exemplo, os juros so sempre de 10% de 100, ou seja,

10.

10
Matemtica Financeira (continuao) Unidade 10

claro ento que, Cn = C0 + niC0 , o que faz com que os valores de Cn


formem uma progresso aritmtica.

Olhando para os grcos de evoluo de um mesmo principal C0 a juros de

taxa i, a juros simples e a juros compostos, observamos que o montante a juros

compostos superior ao montante a juros simples, exceto se o prazo for menor

que 1. por isso que juros simples s so utilizados em cobranas de juros em

prazos inferiores ao prazo ao qual se refere a taxa de juros combinada.

Figura 10.6: Comparando juros

11
Unidade 10 Exerccios Recomendados

10.4 Exerccios Recomendados

1. Um televisor, cujo preo vista de R$ 400,00, vendido em dez presta-

es mensais iguais. Se so pagos juros de 6% ao ms sobre o saldo devedor,

determine o valor das prestaes, supondo a primeira prestao paga:

a) no ato da compra.

b) um ms aps a compra.

c) dois meses aps a compra.

2. Se a taxa corrente de juros de 0,6% ao ms, por quanto se aluga um

imvel cujo preo a vista R$ 50 000,00, supondo:

a) o aluguel mensal pago vencido?

b) o aluguel mensal pago adiantadamente?

3. Supondo juros de 0,5% ao ms, quanto voc deve investir mensalmente,

durante 30 anos, para obter ao m desse prazo, por 30 anos, uma renda mensal

de R$ 100,00?

4. Supondo juros de 0,5% ao ms, quanto voc deve investir mensalmente,

durante 35 anos, para obter, ao m desse prazo, uma renda perptua de R$

100,00.

5. Faa as planilhas de amortizao de uma dvida de R$ 3 000,00, em 8

pagamentos mensais, com juros de 10% ao ms:

a) pela tabela Price.

b) pelo SAC.

6. Leigh investiu 30% do seu capital a juros de 10% ao ms e os 70% restantes

a 18% ao ms. Qual a taxa mdia de juros obtidas?

7. Laura quer comprar um violo em uma loja que oferece um desconto de

30% nas compras vista ou pagamento em trs prestaes mensais, sem juros

e sem desconto. Determine a taxa mensal de juros embutida nas vendas a

prazo, supondo o primeiro pagamento:

a) no ato da compra.

b) um ms aps a compra.

c) dois meses aps a compra.

8. Regina tem duas opes de pagamento:

12
Matemtica Financeira (continuao) Unidade 10

a) vista, com x% de desconto.

b) em duas prestaes mensais iguais, sem juros, vencendo a primeira um ms

aps a compra.

Se o dinheiro vale 5% ao ms, para que valores de x ela preferir a segunda

alternativa?

9. Um banco efetua descontos taxa de 6% ao ms. Qual a taxa mensal de

juros cobrada pelo banco nas operaes:

a) de um ms?

b) de dois meses?

c) de trs meses?

10. Um banco efetua descontos taxa de 6% ao ms, mas exige que 20% do

valor efetivamente liberado sejam aplicados no prprio banco, a juros de 2% ao

ms. Essa a chamada reciprocidade. Qual a taxa mensal de juros paga pelos

tomadores de emprstimo por dois meses?

10.5 Exerccios Suplementares

1. No clculo de juros, considera-se sempre o ano comercial de 360 dias, ou

seja, 12 meses de 30 dias. Essa a chamada regra dos banqueiros. Os juros

assim calculados so chamados de ordinrios, ao passo que os juros calculados

com o ano de 365 (ou 366) dias so chamados de exatos e no so usados em

lugar nenhum.

a) Mostre que, dados o principal e a taxa anual, os juros ordinrios produzidos

em t dias so maiores que os exatos.

b) Para um principal de R$ 1 000,00 e juros de 12% ao ano, determine os juros

simples, ordinrios e exatos, produzidos em 16 dias.

c) Refaa o item (b) para juros compostos.

2. Uma conta de R$ 700,00 vencia no dia 25 de outubro de 1996 e foi paga

em 5 de novembro de 1996. Quais os juros pagos, se os juros de mora so de

12% ao ms?

3. Determine a melhor e a pior alternativa para tomar um emprstimo por trs

meses:

13
Unidade 10 Exerccios Suplementares

a) juros simples de 16% ao ms.

b) juros compostos de 15% ao ms.

c) desconto bancrio com taxa de desconto de 12% ao ms.

4. Henrique vai emprestar dinheiro a Mrio, por quatro meses e pretende receber

juros compostos de 12% ao ms. Como Mrio s pretende pagar juros simples,

qual a taxa mensal de juros simples que Henrique deve cobrar?

5. Quando uma operao pactuada por um nmero inteiro de perodos de

tempo, h trs modos de calcular os juros relativos a fraes de perodos:

a) S so pagos juros nos perodos inteiros de tempo.

b) So pagos juros compostos durante todo o perodo. Essa a chamada

conveno exponencial .

c) So pagos juros compostos nos perodos inteiros e juros simples nas fraes

de perodos de tempo. Essa a chamada conveno linear .

Evidentemente o processo (a) se aplica quando os bancos pagam e, o pro-

cesso (c), quando recebem.

Em 5 de janeiro de 1996 foi feito um investimento de 300 reais, a juros de

15% ao ms. Determine, pelos trs processos, o montante em 12 de abril de

1996.

6. Considere a amortizao de uma dvida de R$ 35 000,00, em 180 meses,

com juros de 1% ao ms, pelo sistema francs. Determine:

a) o valor da centsima prestao.

b) o estado da dvida nessa poca.

7. Refaa o problema anterior pelo SAC.

8. Considere a amortizao de uma dvida em 150 meses, com juros de 1% ao

ms, pelo sistema francs.

a) De quanto se reduzir a prestao, dobrando-se o prazo?


o
b) Que frao da dvida j ter sido amortizada na poca do 75 pagamento?

9. Considere a amortizao de uma dvida em 150 meses, com juros de 1% ao

ms, pelo SAC.

a) De quanto se reduzir a prestao inicial, dobrando-se o prazo?


o
b) Que frao da dvida j ter sido amortizada na poca do 75 pagamento?

14
Matemtica Financeira (continuao) Unidade 10

10. Uma lanterna de Gol, original, custa R$ 280,00 e tem vida til de 5 anos.

Uma lanterna alternativa custa R$ 70,00 e tem vida til de 1 ano. Gilmar

precisa trocar a lanterna de seu Gol. Considere que o dinheiro vale 12% ao ano,

que lanterna ele deve preferir?

11. Um equipamento pode ser alugado por R$ 75,00 mensais ou comprado

por R$ 2 000,00. A vida til do equipamento de 30 meses e o valor residual

ao m desse perodo de R$ 300,00. Se o equipamento for comprado, h um

custo mensal de R$ 5,00 de manuteno. Considere o valor do dinheiro de 1%

ao ms, qual deve ser a deciso: comprar ou alugar?

12. As cadernetas de poupana renderam 1 416% em um ano cuja inao foi

de 1 109%. Qual a rentabilidade real?

15
MA12 - Unidade 10
Matematica Financeira - continuacao

Paulo Cezar Pinto Carvalho

PROFMAT - SBM

31 de Marco de 2013
Series de pagamentos
Uma serie uniforme de pagamentos e uma sequencia de
pagamentos iguais e igualmente espacados ao longo do tempo.
Teorema: O valor de uma serie uniforme de n pagamentos
iguais a P, um tempo antes do primeiro pagamento, e, sendo
1 (1 + i)n
i a taxa de juros, igual a A = P .
i

O valor da serie na epoca 0 e


P P P P
A= + 2
+ 3
+ +
1+i (1 + i) (1 + i) (1 + i)n
 n
1
1 1+i
P 1 (1 + i)n
= 1
= P .
1 + i 1 1+i i
PROFMAT - SBM MA12 - Unidade 10, Matematica Financeira - continuacao slide 2/13
Exemplo
Um bem, cujo preco a vista e R$ 120,00, e vendido em 6
prestacoes mensais iguais, a primeira paga no ato da compra.
Se os juros sao de 10% ao mes, determine o valor das
prestacoes.

Igualando os valores na epoca 1:

120 1 (1 + 0, 1)6
=P
1 + 0, 1 0, 1
P 25, 05.
PROFMAT - SBM MA12 - Unidade 10, Matematica Financeira - continuacao slide 3/13
Exemplo

Helena tem duas alternativas para obter uma copiadora:


a) Compra-la por 150. Nesse caso, ja que a vida economica da
copiadora e de 5 anos, Helena vendera a copiadora apos 5
anos. O valor residual da copiadora apos 5 anos e de 20. As
despesas de manutencao sao de responsabilidade de Helena e
sao de 5 por ano, nos dois primeiros anos, e de 8 por ano, nos
anos seguintes.
b) Aluga-la por 5 anos, com o locador se responsabilizando pelas
despesas de manutencao.
Se o dinheiro vale 7% ao ano, qual e o valor justo do aluguel?

PROFMAT - SBM MA12 - Unidade 10, Matematica Financeira - continuacao slide 4/13
Solucao
O fluxo de caixa da primeira alternativa:

O da segunda alternativa:

Igualando os valores na epoca 0:


5 5 8 8 12 1 1, 075
150 + = P .
1, 07 1, 072 1, 073 1, 074 1, 075 0, 07
Da, P = 39, 78.
PROFMAT - SBM MA12 - Unidade 10, Matematica Financeira - continuacao slide 5/13
Perpetuidades

Teorema: O valor de uma perpetuidade de termos iguais a P,


um tempo antes do primeiro pagamento, e, sendo i a taxa de
P
juros, igual a .
i
O valor da serie perpetua e:

1 (1 + i)n P
A = lim P = .
n i i

PROFMAT - SBM MA12 - Unidade 10, Matematica Financeira - continuacao slide 6/13
Exemplo

Se a taxa de juros e de 1% ao mes, quanto deve ser


economizado mensalmente durante 20 anos para assegurar
uma perpetuidade de R$ 1000,00?
O esquema de pagamentos e recebimentos e:

O valor economizado, na epoca 240, deve ser igual a


1000
= 100.000.
0, 01

PROFMAT - SBM MA12 - Unidade 10, Matematica Financeira - continuacao slide 7/13
Na data 0, o valor economizado e

1 (1, 01)240
P
0, 01
Levando-se este valor para a epoca 240, obtem-se

1 (1, 01)240 (1, 01)240 1


P (1, 01)240 = P = 494, 63P
0, 01 0, 01
Logo, devemos ter 494, 63P = 100.000 e o valor mensal a ser
economizado deve ser igual a R$ 202,17.

PROFMAT - SBM MA12 - Unidade 10, Matematica Financeira - continuacao slide 8/13
Sistemas de amortizacao
Ao se pagar em parcelas um debito, cada pagamento Pk tem
dupla finalidade:
uma parte (Jk ) quita os juros devidos.
o restante (Ak ) amortiza parte da dvida;
Planilha de amortizacao
k Pk Ak Jk Dk
0 D0
... ... ... ... ...
k 1 Pk1 Ak1 Jk1 Dk1
k Pk Ak Jk Dk
... ... ... ... ...
Relacoes:
Pk = Jk + Ak
Jk = i Dk1
Dk = Dk1 Ak

PROFMAT - SBM MA12 - Unidade 10, Matematica Financeira - continuacao slide 9/13
Sistema de Amortizacao Constante (SAC)

No sistema de amortizacao constante (SAC), a parcela de


amortizacao Ak em cada pagamento e constante.
Em consequencia:
D0
Ak = ,
n
nk
Dk = D0 ,
n
Jk = iDk1 ,
Pk = Ak + Jk .

PROFMAT - SBM MA12 - Unidade 10, Matematica Financeira - continuacao slide 10/13
Exemplo

Uma dvida de 100 e paga, com juros de 15% ao mes, em 5


meses, pelo SAC. Faca a planilha de amortizacao.
1
Como as amortizacoes sao iguais, cada amortizacao sera de 5
da dvida inicial.
k Pk Ak Jk Dk
0 100
1 35 20 15 80
2 32 20 12 60
3 29 20 9 40
4 26 20 6 20
5 23 20 3

PROFMAT - SBM MA12 - Unidade 10, Matematica Financeira - continuacao slide 11/13
Sistema Frances de Amortizacao (Tabela Price)

O sistema frances de amortizacao e caracterizado por


pagamentos Pk iguais.
Em consequencia:

i
Pk = D0 ,
1 (1 + i)n

1 (1 + i)(nk)
Dk = D0 ,
1 (1 + i)n

Jk = iDk1

Ak = Pk Jk .

PROFMAT - SBM MA12 - Unidade 10, Matematica Financeira - continuacao slide 12/13
Exemplo
Uma dvida de 150 e paga, em 4 meses, pelo sistema frances,
com juros de 8% ao mes. Calcule o valor da prestacao e faca
a planilha de amortizacao.
O valor da prestacao e
i 0, 08
P = D0 n
= 150 = 45, 29.
1 (1 + n) 1 1, 084
.
A planilha de amortizacao e:
k Pk Ak Jk Dk
0 150, 00
1 45, 29 33, 29 12, 00 116, 71
2 45, 29 35, 95 9, 34 80, 76
3 45, 29 38, 83 6, 46 41, 93
4 45, 29 41, 93 3, 35 0

PROFMAT - SBM MA12 - Unidade 10, Matematica Financeira - continuacao slide 13/13
Lista de Exerccios
Unidade 10

1. Um televisor, cujo preco a vista e de R$ 400,00, e vendido em dez


prestacoes mensais iguais. Se sao pagos juros de 6% ao mes sobre o saldo
devedor, determine o valor das prestacoes, supondo a primeira prestacao
paga:
a) no ato da compra.
b) um mes apos a compra.
c) dois meses apos a compra.
2. Se a taxa corrente de juros e de 0,6% ao mes, por quanto se aluga um
imovel cujo preco a vista e R$ 50 000,00, supondo:
a) o aluguel mensal pago vencido?
b) o aluguel mensal pago adiantadamente?
3. Supondo juros de 0,5% ao mes, quanto voce deve investir mensalmente,
durante 30 anos, para obter ao fim desse prazo, por 30 anos, uma renda
mensal de R$ 100,00?
4. Supondo juros de 0,5% ao mes, quanto voce deve investir mensalmente,
durante 35 anos, para obter, ao fim desse prazo, uma renda perpetua de R$
100,00.
5. Faca as planilhas de amortizacao de uma dvida de R$ 3 000,00, em 8
pagamentos mensais, com juros de 10% ao mes:
a) pela tabela Price.
b) pelo SAC.
6. Leigh investiu 30% do seu capital a juros de 10% ao mes e os 70% restantes
a 18% ao mes. Qual a taxa media de juros obtidas?
7. Laura quer comprar um violao em uma loja que oferece um desconto de
30% nas compras a vista ou pagamento em tres prestacoes mensais, sem juros
e sem desconto. Determine a taxa mensal de juros embutida nas vendas a
prazo, supondo o primeiro pagamento:
a) no ato da compra.
b) um mes apos a compra.

1
c) dois meses apos a compra.
8. Regina tem duas opcoes de pagamento:
a) a vista, com x% de desconto.
b) em duas prestacoes mensais iguais, sem juros, vencendo a primeira um
mes apos a compra.
Se o dinheiro vale 5% ao mes, para que valores de x ela preferira a segunda
alternativa?
9. Considere a amortizacao de uma dvida de R$ 35 000,00, em 180 meses,
com juros de 1% ao mes, pelo sistema frances. Determine:
a) o valor da centesima prestacao.
b) o estado da dvida nessa epoca.
10. Refaca o problema anterior pelo SAC.
11. Considere a amortizacao de uma dvida em 150 meses, com juros de 1%
ao mes, pelo sistema frances.
a) De quanto se reduzira a prestacao, dobrando-se o prazo?
b) Que fracao da dvida ja tera sido amortizada na epoca do 75o pagamento?
12. Considere a amortizacao de uma dvida em 150 meses, com juros de 1%
ao mes, pelo SAC.
a) De quanto se reduzira a prestacao inicial, dobrando-se o prazo?
b) Que fracao da dvida ja tera sido amortizada na epoca do 75o pagamento?

2
Solucoes da Lista de Exerccios
Unidade 10

10
1. (a) 400
= P 11,06
1,06 0,06
P = 51, 27
10
(b) 400 = P 11,06
0,06
P = 54, 35
10
(c) 400.1, 06 = P 11,06
0,06
P = 57, 61

2. (a) P = Ai = 50000.0, 006 = 300, 00


A P
(b) 1+i
= c
50000
P = 1,006
.0, 006 = 298, 21
11,005360
3. O montante que voce deve acumular e 100 = 0,005
= 16.679, 16
360
Para isso, P 11,005
0,005
.1, 005360 = 16.779, 16 e P = 16, 60
100
4. O montante que voce deve acumular e 0,005
= 20.000
420
Para isso, P 11,005
0,005
.005420 = 20.000 e P = 14, 04
0,1
5. A prestacao pela tabela Price e P = 3.000 11,1 8 = 562, 33

A amortizacao pelo SAC e 3.000/8 = 375

1
TABELA PRICE
EPOCA PRESTACAO JUROS AMORTIZACAO ESTADO DA DIVIDA
0 - - - 3 000,00
1 562,33 300,00 262,33 2737,67
2 562,33 273,77 288,56 2.449,11
3 562,33 244,91 317,42 2.131,69
4 562,33 213,17 349,16 1.782,53
5 562,33 178,25 384,08 1.398,45
6 562,33 139,84 422,49 975,96
7 562,33 97,60 464,73 511,23
8 562,33 51,12 511,23 -
SAC
EPOCA PRESTACAO JUROS AMORTIZACAO ESTADO DA DIVIDA
0 - - - 3 000,00
1 675,00 300,00 375,00 2.625,00
2 637,50 262,50 375,00 2.250,00
3 600,00 225,00 375,00 1.875,00
4 562,50 187,50 375,00 1.500,00
5 525,00 150.00 375,00 1,125,00
6 487,50 112,50 375,00 750,00
7 450,00 75,00 375,00 375,00
8 412,00 37,50 375,00 -

6. O montante e 0, 3.1, 1t + 0, 7.1, 18t


A taxa media de juros e calculada por 0, 3.1, 1t + 0, 1.1, 18t = 1.(1 +
i)t i = (0, 3.1, 1t + 0, 1.1, 18t )1/t 1, onde t e o numero de meses do
investimento. Se t = 1, a taxa e 15, 60%; se t = 2, e 15,66%; se t ,
a taxa e 18%.

7. (a) Usando a data da compra como data focal e considerando um


preco igual a 30,
10 10
21 = 10 + 1+i + (1+i)2

Resolvendo, i = 51, 08%.


(b) Tomando para data focal a data da compra e considerado um
preco igual a 30,
10 10 10
21 = 1+i + (1+i)2 + (1+i)3

2
Resolvendo, i = 20, 20%
(c) Tomando para data focal a data da compra e considerado um
preco igual a 30,
10 10 10
21 = (1+i) 2 + (1+i)3 + (1+i)4

Resolvendo, i = 12, 81%.

8. Arbitrando o preco em 100 e usando a data da compra como data focal,


50 50
100 x > 1,05
+ 1,052

x < 7, 03%
0,01
9. (a) A prestacao e 35.000 11,01180 = 420, 06

180
(b) 420, 06 11,01
0,01
= 23.056, 28

10. A amortizacao e 35.000/180 = 194, 44


A dvida na epoca da 99a prestacao e 81.194, 44 = 15750.
Os juros da centesima prestacao sao 157,50 e a centesima prestacao e
igual a 194,44+157,50=351,94.

11. (a) Supondo a dvida igual a 100, a prestacao para 150 meses e
0,01
P150 = 100 11,01150 = 1, 29 e a prestacao para 300 meses e

0,01
P300 = 100 11,01300 = 1, 05

A reducao e de 0, 24/1, 29 = 18%, aproximadamente

12. (a) A dvida igual a 300, a prestacao para 150 meses e


300
P150 = 150 +3=5
A prestacao para 300 meses e P300 = 300
300
+3=4
A reducao e de 20%.
(b) 50%.

3
Referncias Bibliogrcas

[1] Carmo, Manfredo P.; Morgado, Augusto C., Wagner, Eduardo & Pitom-

beira, Joo Bosco. Trigonometria e Nmeros Complexos . Rio de Janeiro:

SBM, Coleo Professor de Matemtica.

[2] Eves, Howard. An Introduction to the History of Mathematics . New York:

Holt, Rinehart and Winston, 1964.

[3] Figueiredo, Djairo G. Anlise I Rio de Janeiro: LTC, 1996.

[4] Figueiredo, Djairo G. Nmeros Irracionais e Transcedentes Rio de Janeiro:

SBM, Coleo Iniciao Cientca.

[5] Halmos, Paul. Naive Set Theory . New York: Springer, 1974.

[6] Hefez, Abramo e Fernandez, Ceclia de Souza. Introduo lgebra Linear .

Rio de Janeiro: SBM, Coleo PROFMAT, 2012.

[7] Fernandes, C. S. Hefez, A. Introduo lgebra Linear . SBM, Coleo

PROFMAT. 2

[8] Lima, Elon Lages. Coordenadas no Espao . Rio de Janeiro: SBM, Coleo

Professor de Matemtica.

[9] Lima, Elon Lages. Curso de Anlise , Vol. 1. Rio de Janeiro: SBM, Projeto

Euclides, 1976.

[10] Lima, Elon Lages. Logaritmos. Rio de Janeiro: SBM, Coleo Professor de

Matemtica.

[11] Lima, Elon Lages. Meu Professor de Matemtica e Outras Histrias . Rio

de Janeiro: SBM, Coleo Professor de Matemtica.

bibitemelon-analisereal Lima, Elon Lages. Anlise Real, Vol. 1. Rio de Janeiro:

IMPA, Coleo Matemtica Universitria.

16
11
Combinatria I

Sumrio
11.1 Introduo . . . . . . . . . . . . . . . . . . . . . . . 2

11.2 Princpios Bsicos . . . . . . . . . . . . . . . . . . . 2

1
Unidade 11 Introduo

11.1 Introduo

Combinatria um vasto e importante campo da matemtica que engloba

temas como a Combinatria Enumerativa, Combinatria Algbrica, Combina-

tria Extrema, Teoria de Grafos e muito mais. As suas aplicaes so inmeras

e vo desde Probabilidade e Estatstica e Teoria dos Jogos at campos to

abstratos quanto a Computao Terica.

A combinatria foi responsvel pela introduo de novos mtodos em matem-

tica e requereu o desenvolvimento de um modo prprio de raciocnio. Para se

ter sucesso no seu estudo, preciso adquirir certas atitudes e formas de pensar.

No nosso curso, veremos apenas rudimentos de Combinatria Enumerativa, que

essencialmente a arte da contagem. Contar uma atividade bsica e saber

faz-lo corretamente importante e de grande utilidade prtica.

No Ensino Mdio, a parte da matemtica que se ocupa de contagem chama-se

Anlise Combinatria e geralmente ela considerada uma matria difcil. Ali

se aprendem frmulas para arranjos, combinaes, com repetio ou sem repe-

tio, permutaes, permutaes circulares, caticas, etc., mas no se aprende

o essencial, que raciocinar!

Ao invs de apresentar um formulrio e pedir para que seja decorado, o que

se prope aqui focar em alguns princpios e tcnicas bsicas e desenvolver

um raciocnio combinatrio prprio que permitir resolver uma grande gama de

problemas.

Esta unidade baseia-se no Princpio Fundamental da Contagem que diz simples-

mente que, se temos x modos de escolher um objeto e y modos de escolher

outro, temos xy modos de escolher os dois objetos. Esse princpio utilizado

nas mais variadas situaes.

11.2 Princpios Bsicos

O princpio fundamental da contagem diz que se h x modos de tomar uma


deciso D1 e, tomada a deciso D1 , h y modos de tomar a deciso D2 , ento

o nmero de modos de tomar sucessivamente as decises D1 e D2 xy .

2
Combinatria I Unidade 11

Com 5 homens e 5 mulheres, de quantos modos de pode formar um casal? Exemplo 1


Soluo. Formar um casal equivale a tomar as decises:

D1 : Escolha do homem (5 modos).


D2 : Escolha da mulher (5 modos).
H 5 5 = 25 modos de formar casal.

Uma bandeira formada por 7 listras que devem ser coloridas usando apenas Exemplo 2
as cores verde, azul e cinza. Se cada listra deve ter apenas uma cor e no se

pode usar cores iguais em listras adjacentes, de quantos modos se pode colorir

a bandeira?

Soluo. Colorir a bandeira equivale a escolher a cor de cada listra. H 3 modos

de escolher a cor da primeira listra e, a partir da, 2 modos de escolher a cor de

cada uma das outras 6 listras. A resposta 3 26 = 192.

Quantos so os nmeros de trs dgitos distintos? Exemplo 3


Soluo. O primeiro dgito pode ser escolhido de 9 modos, pois ele no pode

ser igual a 0. O segundo dgito pode ser escolhido de 9 modos, pois no pode

ser igual ao primeiro dgito. O terceiro dgito pode ser escolhido de 8 modos,

pois no pode ser igual nem ao primeiro nem ao segundo dgito.

A resposta 9 9 8 = 648.

Voc deve ter percebido nesses exemplos qual a estratgia para resolver

problemas de Combinatria:

1) Postura. Devemos sempre nos colocar no papel da pessoa que deve fazer a

ao solicitada pelo problema e ver que decises devemos tomar. No Exemplo

3, ns nos colocamos no papel da pessoa que deveria escrever o nmero de

trs dgitos; no Exemplo 2, ns nos colocamos no papel da pessoa que deveria

colorir a bandeira; no Exemplo 1, ns nos colocamos no papel da pessoa que

deveria formar o casal.

2) Diviso. Devemos, sempre que possvel, dividir as decises a serem tomadas

em decises mais simples. Formar um casal foi dividido em escolher o homem

e escolher a mulher; colorir a bandeira foi dividido em colorir cada listra; formar

um nmero de trs dgitos foi dividido em escolher cada um dos trs dgitos.

3
Unidade 11 Princpios Bsicos

Vamos voltar ao exemplo anterior Quantos so os nmeros de trs dgitos


distintos? para ver como algumas pessoas conseguem, por erros de estratgia,
tornar complicadas as coisas mais simples.

Comeando a escolha dos dgitos pelo ltimo dgito, h 10 modos de escolher

o ltimo dgito. Em seguida, h 9 modos de escolher o dgito central, pois no

podemos repetir o dgito j usado. Agora temos um impasse: de quantos modos

podemos escolher o primeiro dgito: A resposta depende. Se no tivermos

usado o 0, haver 7 modos de escolher o primeiro dgito, pois no poderemos

usar nem o 0 nem os dois dgitos j usados nas demais casas; se j tivermos

usado o 0, haver 8 modos de escolher o primeiro dgito.

Um passo importante na estratgia para resolver problemas de Combinatria :

3) No adiar diculdades. Pequenas diculdades adiadas costumam se trans-

formar em imensas diculdades. Se uma das decises a serem tomadas for mais

restrita que as demais, essa a deciso que deve ser tomada em primeiro lugar.

No Exemplo 3, a escolha do primeiro dgito era uma deciso mais restrita do

que as outras, pois o primeiro dgito no pode ser igual a 0. Essa portanto a

deciso que deve ser tomada em primeiro lugar e, conforme acabamos de ver,

posterg-la s serve para causar problemas.

Exemplo 4 O cdigo Morse usa duas letras, ponto e trao, e as palavras tm de 1 a 4

letras. Quantas so as palavras do cdigo Morse?

Soluo. H 2 palavras de uma letra. H 2 2 = 4 palavras de duas letras, pois


h dois modos de escolher a primeira letra e dois modos de escolher a segunda

letra; analogamente, h 222 = 8 palavras de trs letras e 2222 = 16


palavras de 4 letras. O nmero total de palavras 2 + 4 + 8 + 16 = 30.

Exemplo 5 Quantos divisores inteiros e positivos possui o nmero 360? Quantos divi-

sores so pares? Quantos so mpares? Quantos so quadrados perfeitos?

Soluo. a) 360 = 23 32 5. Os divisores inteiros e positivos de 360 so os



nmeros da forma 2 3 5 , com

{0, 1, 2, 3} , {0, 1, 2} e {0, 1}.

H 4 3 = 24 maneiras de escolher os expoentes , e . H 24 divisores.

4
Errado: 4 x 3 x 2 = 24
Combinatria I Unidade 11

b) Para o divisor ser par, no pode ser 0. H 3 3 2 = 18 divisores pares. Rever o


assunto.
c) Para o divisor ser mpar, dever ser 0. H 132=6 divisores mpares.

Claro que poderamos ter achado essa resposta subtraindo (a)-(b).

d) Para o divisor ser quadrado perfeito, os expoentes , e devem ser pares.

H 221=4 divisores que so quadrados perfeitos.

Quantos so os nmeros pares de trs dgitos distintos? Exemplo 6


Soluo. H 5 modos de escolher o ltimo dgito. Note que comeamos pelo

ltimo dgito, que o mais restrito; o ltimo dgito s pode ser 0, 2, 4, 6 ou 8.

Em seguida, vamos ao primeiro dgito. De quantos modos se pode escolher

o primeiro dgito? A resposta depende: se no tivermos usado o 0, haver 8

modos de escolher o primeiro dgito, pois no poderemos usar nem o 0 nem o

dgito usado na ltima casa; se tivermos usado o 0, haver 9 modos de escolher

o primeiro dgito, pois apenas o 0 no poder ser usado na primeira casa.

Esse tipo de impasse comum na resoluo de problemas e h dois mtodos

de venc-lo.

O primeiro mtodo consiste em voltar atrs e contar separadamente. Con-

taremos separadamente os nmeros que terminam em 0 e os que no terminam

em 0.

Para os que terminam em 0, h 9 modos de escolher o primeiro dgito e 8

modos de escolher o dgito central. H 1 9 8 = 72 nmeros que terminam

em 0.

Para os que no terminam em 0, h 4 modos de escolher o ltimo dgito,

8 modos de escolher o primeiro e 8 modos de escolher o dgito central. H

4 8 8 = 256 nmeros que no terminam em 0.

A resposta 72 + 256 = 328.


O segundo mtodo consiste em ignorar uma das repeties do problema,

o que nos far contar em demasia. Depois descontaremos o que houver sido

contado indevidamente.

Primeiramente fazemos de conta que o 0 pode ser usado na primeira casa

do nmero. Procedendo assim, h 5 modos de escolher o ltimo dgito (s

pode ser 0, 2, 4, 6 ou 8), 9 modos de escolher o primeiro dgito (no podemos

repetir o dgito usado na ltima casa; note que estamos permitindo o uso do 0

5
Unidade 11 Princpios Bsicos

na primeira casa) e 8 modos de escolher o dgito central. H 5 9 8 = 360


nmeros, a inclusos os que comeam por 0.

Agora vamos determinar quantos desses nmeros comeam por zero; so

esses os nmeros que foram contados indevidamente. H 1 modo de escolher o

primeiro dgito (tem que ser 0), 4 modos de escolher o ltimo dgito (s pode

ser 2, 4, 6 ou 8 lembre-se que os dgitos so distintos) e 8 modos de escolher


o dgito central (no podemos repetir os dgitos j usados). H 1 4 8 = 32

nmeros comeados por 0.

A resposta : 360 32 = 328.


claro que este problema poderia ter sido resolvido com um truque. Para

determinar quantos so os nmeros pares de trs dgitos distintos, poderamos

fazer os nmeros de trs dgitos distintos menos os nmeros mpares de trs

dgitos distintos.

Para os nmeros de trs dgitos distintos, h 9 modos de escolher o primeiro

dgito, 9 modos de escolher o segundo e 8 modos de escolher o ltimo. H

9 9 8 = 648 nmeros de trs dgitos distintos.

Para os nmeros mpares de trs dgitos distintos, h 5 modos de escolher

o ltimo dgito, 8 modos de escolher o primeiro e 8 modos de escolher o dgito

central. H 5 8 8 = 320 nmeros mpares de trs dgitos distintos.

A resposta : 648 320 = 328.

6
Combinatria I Unidade 11

Exerccios Recomendados

Nvel 01 1. Quantos so os gabaritos possveis de um teste de 10 questes de mltipla-

escolha, com 5 alternativas por questo?

Nvel 02 2. Quantos subconjuntos possui um conjunto que tem n elementos?

Nvel 02 3. De quantos modos 3 pessoas podem se sentar em 5 cadeiras em la?

Nvel 03 4. De quantos modos 5 homens e 5 mulheres podem se sentar em 5 bancos

de 2 lugares, se em cada banco deve haver um homem e uma mulher?

Nvel 02 5. De quantos modos podemos colocar 2 reis diferentes em casas no-

adjacentes de um tabuleiro 8 8? E se os reis fossem iguais?

Nvel 02 6. De quantos modos podemos colocar 8 torres iguais em um tabuleiro 88,


de modo que no haja duas torres na mesma linha ou na mesma coluna?

E se as torres fossem diferentes?

Nvel 03 7. De um baralho comum de 52 cartas, sacam-se sucessivamente e sem

reposio duas cartas. De quantos modos isso pode ser feito se a primeira

carta deve ser de copas e a segunda no deve ser um rei?

Nvel 04 8. O conjunto A possui 4 elementos e, o conjunto B , 7 elementos. Quantas

funes f : A B existem? Quantas delas so injetoras?

Nvel 05 9. a) De quantos modos o nmero 720 pode ser decomposto em um produto

de dois inteiros positivos? Aqui consideramos, naturalmente, 890 como


sendo o mesmo que 90 8.
H uma resposta diferente e mais
b) E o nmero 144? explicada.
Ver acervo.

7
MA12 - Unidade 11
Combinatoria

Paulo Cezar Pinto Carvalho

PROFMAT - SBM

29 de Abril de 2013
Princpio Fundamental da Contagem

Problemas de contagem sao resolvidos explorando o


significado das operacoes aritmeticas de adicao, subtracao,
multiplicacao e divisao, com destaque para o papel da
multiplicacao.

Princpio Multiplicativo (ou Princpio Fundamental da


Contagem):
Se ha x modos de tomar uma decisao D1 e, tomada a decisao
D1 , ha y modos de tomar a decisao D2 , entao o numero de
modos de tomar sucessivamente as decisoes D1 e D2 e xy .

PROFMAT - SBM MA12 - Unidade 11, Combinatoria slide 2/8


Exemplo

Uma bandeira e formada por 7 listras que devem ser coloridas


usando apenas as cores verde, azul e cinza. Se cada listra deve
ter apenas uma cor e nao se pode usar cores iguais em listras
adjacentes, de quantos modos se pode colorir a bandeira?
Colorir a bandeira equivale a escolher a cor de cada listra.
Ha 3 modos de escolher a cor da primeira listra;
A partir da, 2 modos de escolher a cor de cada uma das
outras 6 listras.
A resposta e 3 26 = 192.

PROFMAT - SBM MA12 - Unidade 11, Combinatoria slide 3/8


Exemplo

Quantos sao os numeros de tres dgitos distintos?


Formar o numero equivale a escolher cada um de seus dgitos.
O primeiro dgito pode ser escolhido de 9 modos, pois ele nao
pode ser igual a 0.
O segundo dgito pode ser escolhido de 9 modos, pois nao
pode ser igual ao primeiro dgito.
O terceiro dgito pode ser escolhido de 8 modos, pois nao pode
ser igual nem ao primeiro nem ao segundo dgito.
A resposta e 9 9 8 = 648.

PROFMAT - SBM MA12 - Unidade 11, Combinatoria slide 4/8


A estrategia

1) Postura. Colocar-se no papel da pessoa que deve fazer a acao


solicitada pelo problema e ver que decisoes devemos tomar.
2) Divisao. Sempre que possvel, dividir as decisoes a serem
tomadas em decisoes mais simples.
3) Nao adiar dificuldades. Se uma das decisoes a serem
tomadas for mais restrita que as demais, essa e a decisao que
deve ser tomada em primeiro lugar.

PROFMAT - SBM MA12 - Unidade 11, Combinatoria slide 5/8


Violando a terceira regra

Quantos sao os numeros de tres dgitos distintos?


A escolha mais restrita e a do dgito das centenas, porque ele
nao pode ser 0.
O que ocorre se ignoramos este fato e decidimos comecar pelo
dgito das unidades?
O dgito das unidades pode ser escolhido de 10 modos.
O das dezenas pode ser escolhido de 9 modos ( nao pode ser
igual ao das unidades).
O das centenas pode ser escolhido de ... depende! (podem ser
7 ou 8 possibilidades, dependendo do 0 ter ou nao ter sido
usado nas dezenas ou unidades).

PROFMAT - SBM MA12 - Unidade 11, Combinatoria slide 6/8


Nem sempre basta usar o Princpio Fundamental

Quantos sao os numeros pares de tres dgitos distintos?


Comecando escolhendo dgito das unidades, que e o mais
restrito: ha 5 possibilidades (0, 2, 4, 6 ou 8).
A seguir, escolhemos o dgito das centenas. O numero de
possibilidades para esta escolha ... depende.
Se 0 for o dgito das unidades, ha 9 possibilidades para o das
centenas (so nao pode ser 0).
Se 0 nao for o dgito das unidades, ha 8 possibilidades para o
das centenas (nao pode ser 0 nem o algarismo das unidades).
Solucao: Dividir em casos, contados separadamente, e somar
os resultados.

PROFMAT - SBM MA12 - Unidade 11, Combinatoria slide 7/8


Dividindo em casos e somando
Quantos sao os numeros pares de tres dgitos distintos?
Para os numeros que terminam em 0
Para o dgito das unidades, ha apenas 1 possibilidade.
Para o das centenas, ha 9 possibilidades (so nao pode ser 0).
Para o das dezenas, ha 8 possibilidades (nao pode ser os dois
ja usados).
Logo, ha 1 9 8 = 72 numeros terminados em 0.
Para os numeros que nao terminam em 0
Para o dgito das unidades ha 4 possibilidades.
Para o das centenas, ha 8 possibilidades (nao pode ser 0 nem
o das unidades).
Para o das dezenas, ha 8 possibilidades (nao pode ser os dois
ja usados).
Logo, ha 4 8 8 = 256 numeros terminados em 0.
Portanto, ha 72 + 256 = 328 numeros pares de tres algarismos
distintos.

PROFMAT - SBM MA12 - Unidade 11, Combinatoria slide 8/8


Lista de Exerccios
Unidade 11

1. Quantos sao os gabaritos possveis de um teste de 10 questoes de


multipla-escolha, com 5 alternativas por questao?

2. Quantos subconjuntos possui um conjunto que tem n elementos?

3. De quantos modos 3 pessoas podem se sentar em 5 cadeiras em fila?

4. De quantos modos 5 homens e 5 mulheres podem se sentar em 5 bancos


de 2 lugares, se em cada banco deve haver um homem e uma mulher?

5. De quantos modos podemos colocar 2 reis diferentes em casas nao-


adjacentes de um tabuleiro 8 8? E se os reis fossem iguais?

6. De quantos modos podemos colocar 8 torres iguais em um tabuleiro


8 8, de modo que nao haja duas torres na mesma linha ou na mesma
coluna? E se as torres fossem diferentes?

7. De um baralho comum de 52 cartas, sacam-se sucessivamente e sem re-


posicao duas cartas. De quantos modos isso pode ser feito se a primeira
carta deve ser de copas e a segunda nao deve ser um rei?

8. O conjunto A possui 4 elementos e, o conjunto B, 7 elementos. Quantas


funcoes f : A B existem? Quantas delas sao injetoras?

9. a) De quantos modos o numero 720 pode ser decomposto em um pro-


duto de dois inteiros positivos? Aqui consideramos, naturalmente,
8 90 como sendo o mesmo que 90 8.
b) E o numero 144?

1
Solucoes da Lista de Exerccios
Unidade 11

1. A primeira pergunta pode ser respondida de 5 modos; a segunda, de 5


modos, etc.
A resposta e 5 5 5 = 510 = 9.765.625.

2. Para formar um subconjunto, deve-se decidir, para cada elemento do


conjunto, se ele pertencera ou nao ao subconjunto. Ha 2 modos de
decidir o que fazer com o primeiro elemento do conjunto, 2 modos com
o segundo, etc.
A resposta e 2 2 2 = 2n

3. A primeira pessoa tem 5 escolhas; a segunda, 4; a terceira, 3. A resposta


e 5 4 3 = 60.

4. Os bancos em que os homens se sentam podem ser escolhidos de 54


321 = 120 modos, o mesmo ocorrendo com os bancos das mulheres.
Em cada banco, os casais podem se sentar de 2 modos diferentes. A
resposta e 1202 25 = 460.800.

5. As 64 casas do tabuleiro dividem-se, naturalmente, em tres grupos:

(a) as 4 casas dos vertices;


(b) as 24 casas da borda do tabuleiro, mas que nao sao vertices;
(c) as restantes 36 casas, que sao interiores ao tabuleiro.

Vamos separar a nossa contagem conforma o tipo de casa ocupada pelo


rei negro:

(a) ha 4 possveis para o rei negro e 60 para o rei branco;


(b) ha 24 possveis para o rei negro e 58 para o rei branco;
(c) ha 36 possveis para o rei negro e 55 para o rei branco.

1
A resposta e 4 60 24 58 + 36 55 = 3612.
Se os reis sao iguais, a resposta passa a ser a metade da resposta ante-
rior, pois, trocando a posicao dos reis, agora obtem-se a mesma confi-
guracao.

6. Havera uma torre em cada linha e em cada coluna. A posicao da torre


da primeira linha pode ser escolhida de 8 modos; a da segunda linha,
de 7, etc.
A resposta e 8 7 6 5 4 3 2 1 = 40.320.
Se as torres fossem diferentes, para cada uma das escolhas de posicao,
teramos que escolher uma das torres. A resposta seria, portanto, 8
877665544332211 = (8!)2 = 1.625.702.400.

7. Se a primeira carta e o rei de copas, a segunda pode ser escolhida de 48


modos (pode ser qualquer carta, exceto os 4 reis). Se a primeira carta e
de copas mas nao e o rei, ela pode ser escolhida de 12 modos. Neste caso,
a segunda carta pode ser escolhida de 47 modos (nao pode ser a primeira
escolhida, nem nenhum dos 4 reis). A resposta e 48 + 12 47 = 612.

8. Para a funcao ser injetora, elementos diferentes devem ter imagens


diferentes. Ha 7 modos de escolher a imagem do primeiro elemento de
A, 6 modos de escolher a imagem do segundo elemento, etc.
A resposta e 7 6 5 4 = 840.

9. (a) Como 720 = 24 32 51 , 720 possui 5 3 2 = 30 divisores. Aos


pares, estes divisores formam produtos iguais a 720. Logo, ha 15
modos de escrever 720 como produtos de divisores.
(b) Como 144 = 24 32 , 144 possui 3 3 = 9 divisores. Com eles,
podem ser formados 4 pares de divisores cujo produto e 144 e,
alem disso, pode ser formado 12 12. Assim, ha 5 modos de
escrever 144 como um produto de divisores.

2
12
Combinatria II
Continuao
Sumrio
12.1 Introduo . . . . . . . . . . . . . . . . . . . . . . . 2

12.2 Permutaes e Combinaes . . . . . . . . . . . . . 2

1
Unidade 12 Introduo

12.1 Introduo

Nesta unidade, so estudadas as permutaes e as combinaes, desenvolvendo

modos especcos de contagem. No h frmulas a decorar, mas procedimentos

de contagem a compreender.

A unidade termina com uma lista de 10 problemas; resolva quantos puder,

redigindo as suas solues.

12.2 Permutaes e Combinaes

H alguns (poucos) problemas de Combinatria que, embora sejam aplica-

es do princpio bsico, aparecem com muita frequncia. Para esses problemas,

vale a pena saber de cor as suas respostas. O primeiro desses problemas o:

Problema das permutaes simples

De quantos modos podemos ordenar em la n objetos distintos?

n modos;
A escolha do objeto que ocupar o primeiro lugar pode ser feita de

a escolha do objeto que ocupar o segundo lugar pode ser feita de n 1 modos;

a escolha do objeto que ocupar o terceiro lugar pode ser feita de n 2 modos,

etc...; a escolha do objeto que ocupar o ltimo lugar pode ser feita de 1 modo.

A resposta n(n 1)(n 2) 1 = n!.


Cada ordem que se d aos obejtos chamada de uma permutao simples

dos objetos. Assim, por exemplo, as permutaes simples das letras a, b e c


so (abc), (acb), (bac), (bca), (cab) e (cba).
Portanto, o nmero de permutaes simples de n objetos distintos Pn =
n!.

Exemplo 1 Quantos so os anagramas da palavra calor? Quantos comeam com

consoantes?

Soluo. cada anagrama corresponde a uma ordem de colocao dessas 5 letras.

O nmero de anagramas P5 = 5! = 120.


Para formar um anagrama comeado por consoante devemos primeiramente

escolher a consoante (3 modos) e, depois, arrumar as quatro letras restantes em

2
Combinatria II Continuao Unidade 12

seguida consoante ( 4! = 24 modos). H 3 24 = 72 anagramas comeados

por consoante.

De quantos modos podemos arrumar em la 5 livros diferentes de Mate- Exemplo 2


mtica, 3 livros diferentes de Estatstica e 2 livros diferentes de Fsica, de modo

que livros de uma mesma matria permaneam juntos?

Soluo. Podemos escolher a ordem das matrias de 3! modos. Feito isso,

h 5! modos de colocar os livros de Matemtica nos lugares que lhe foram

destinados, 3! modos para os de Estatstica e 2! modos para os de Fsica.

A resposta 3!5!3!2! = 6 120 6 2 = 8 640.

Quantos so os anagramas da palavra BOTAFOGO? Exemplo 3


Soluo. Se as letras fossem diferentes a resposta seria 8!. Como as trs letras

O so iguais, quando as trocamos entre si obtemos o mesmo anagrama e no

um anagrama distinto, o que aconteceria se fossem diferentes. Isso faz com que

na nossa contagem de 8! tenhamos contado o mesmo anagrama vrias vezes,

3! vezes precisamente, pois h 3! modos de trocar as letras O entre si.


8!
A resposta = 6 720.
3!

De modo geral, o nmero de permutaes de n objetos, dos quais so


n!
iguais a A, so iguais a B, so iguais a C, etc, Pn,,,... = .
!!! . . .

De quantos modos podemos dividir 8 objetos em um grupo de 5 objetos e Exemplo 4


um de 3 objetos?

Soluo. Um processo de fazer a diviso colocar os objetos em la; os 5

primeiros formam o grupo de 5 e os 3 ltimos formam o grupo de 3.

H 8! modos de colocar os objetos em la.

Entretanto, note que las como abcde | f gh e badce | ghf so las diferentes

e geram a mesma diviso de grupos. Cada diviso em grupos foi contada uma

vez para cada ordem dos objetos dentro de cada grupo. H 5!3! modos de

arrumar os objetos em cada grupo. Cada diviso em grupos foi contada 5!3!

vezes.

3
Unidade 12 Permutaes e Combinaes

8!
A resposta = 56.
5!3!

O segundo problema importante o:

Problema das combinaes simples

De quantos modos podemos selecionar p objetos distintos entre n objetos

distintos dados?

Cada seleo de p objetos chamada de uma combinao simples de classe p


dos n objetos. Assim, por exemplo, as combinaes simples de classe 3 dos obje-
tos a, b, c, d e so {a, b, c}, {a, b, d}, {a, b, e}, {a, c, d}, {a, c, e}, {a, d, e},{b, c, d},

{b, c, e}, {b, d, e} e {c, d, e}. Representamos o nmero de combinaes simples


n 5
p
 3

de classe p de n elementos por Cn ou . Assim, C5 = = 10.
p 3
Para resolver o problema das combinaes simples basta notar que selecionar

p entre os n objetos equivale a dividir osn objetos em um grupo de p objetos,


que so selecionados, e um grupo de np objetos, que so os no-selecionados.
Esse o problema de Exemplo 4 e a resposta

n!
Cnp = .
p!(n p)!

Exemplo 5 Com 5 homens e 4 mulheres, quantas comisses de 5 pessoas, com exata-

mente 3 homens, podem ser formadas?

Soluo. Para formar a comisso devemos escolher 3 dos homens e 2 das

mulheres. H C53 C42 = 10 6 = 60 comisses.

Exemplo 6 Com 5 homens e 4 mulheres, quantas comisses de 5 pessoas, com pelo

menos 3 homens, podem ser formadas?

Soluo. H comisses com: 3 homens e 2 mulheres, 4 homens e 1 mulher, 5

homens. A resposta

C52 C42 + C54 C41 + C55 = 10 6 + 5 4 + 1 = 81.

Exemplo 7 Tem-se 5 pontos sobre uma reta R e 8 pontos sobre uma reta R0 paralela

a R. Quantos tringulos e quantos quadrilteros convexos com vrtices nesses

4
Combinatria II Continuao Unidade 12

pontos existem?

Soluo. Para formar um tringulo ou voc toma um ponto em R e dois pontos


em R0 , ou toma um ponto em R0 e dois pontos em R. O nmero de tringulos

5 C82 + 8 C52 = 140 + 80 = 220.


Tambm se poderia pensar em tomar 3 dos 13 pontos e excluir dessa con-

tagem as escolhas de pontos colineares, o que daria

3
C13 C83 C53 = 286 56 10 = 220.
Para formar um quadriltero convexo, devemos tomar dois pontos em R e

dois pontos em R0 , o que pode ser feito de C53 C82 = 10 28 = 280 modos.

A seguir damos mais uma srie de exemplos para xar o mtodo de resoluo.

De quantos modos 5 crianas podem formar uma roda de ciranda? Exemplo 8

Figura 12.1:

Soluo. primeira vista parece que para formar uma roda com as cinco

crianas basta escolher uma ordem para elas, o que poderia ser feito de 5! =
120 modos. Entretanto, as rodas ABCDE e EABCD so iguais, pois na roda

o que importa a posio relativa das crianas entre si e a roda ABCDE

pode ser virada na roda EABCD. Como cada roda pode ser virada de cinco

modos, a nossa contagem de 120 rodas contou cada roda 5 vezes e a resposta

120/5 = 24.
De modo geral, o nmero de modos de colocar n objetos em crculo, de modo
que disposies que possam coincidir por rotao sejam consideradas iguais, isto
n!
, o nmero de permutaes circulares de n objetos (P C)n = = (n 1)!.
n

5
Unidade 12 Permutaes e Combinaes

O exemplo a seguir mostra um tipo de racocinio que, apesar de inesperado,

pode ser muito eciente.

Exemplo 9 Quantos so os anagramas da palavra BULGARO que no possuem duas

vogais adjacentes?

Soluo. Vamos primeiramente arrumar as consoantes e, depois, vamos entre-

mear as vogais. O nmero de modos de arrumar em la as consoantes B, L,

G, R P4 = 4! = 24. Arrumadas as consoantes, por exemplo na ordem BLGR,

devemos colocar as vogais U, A, O nos 5 espaos da gura. Como no podemos

colocar duas vogais no mesmo espao, trs dos espaos sero ocupados, cada

um com uma vogal e dois dos espaos caro vazios. Temos C53 = 10 modos

de escolher os trs espaos que sero ocupados e P3 = 3! = 6 modos de colocar


as vogais nos espaos escolhidos.

B L G R

A resposta 24 10 6 = 1440.

Exemplo 10 Quantas so as solues inteiras e no-negativas da equao x1 + x2 +


+ xn = p ?
Soluo. A resposta deste problema representada por CRnp .
Para determinar o valor de CRnp , vamos representar cada soluo da equao
por uma la de sinais + e | . Por exemplo, para a equao x + y + z = 5, as

solues (2,2,1) e (5,0,0) seriam representadas por + + | + +|+ e + + + + +||,

respectivamente. Nossa representao, as barras so usadas para separar as

incgnitas e a quantidade de sinais+ indica o valor de cada incgnita.


Para a equao x1 + x2 + + xn = p, cada soluo seria representada

por uma la com n 1 barras (as barras so para separar as incgnitas; para

separar n incgnitas, usamos n 1 barras) e p sinais +. Ora, para formar uma

la com n 1 barras e p sinais +, basta escolher dos n + p 1 lugares da la


p
os p lugares onde sero colocados os sinais +, o que pode ser feito de Cn+p1
p p
modos. Portanto, CRn = Cn+p1 .

Exemplo 11 De quantos modos podemos comprar 3 sorvetes em um bar que os oferece

em 6 sabores distintos?

6
Combinatria II Continuao Unidade 12

Soluo. A resposta no C63 = 20. C63 seria o nmero de modos de comprar

trs sorvetes diferentes.

Chamando de xk o nmero de sorvetes do k -simo sabor que vamos comprar,


devemos determinar valores inteiros e no-negativos para xk , k = 1, 2, 3, 4, 5, 6,
3 3
tais que x1 + x2 + + x6 = 3. Isso pode ser feito de CR6 = C8 = 56 modos.

7
Unidade 12 Permutaes e Combinaes

Exerccios Recomendados

1. Quantos so os anagramas da palavra CAPITULO.

a) possveis?

b) que comeam e terminam por vogal?

c) que tm as vogais e as consoantes intercaladas?

d) que tm as letras c, a, p juntas nessa ordem?

e) que tm as letras c, a, p juntas em qualquer ordem?

f ) que tm a letra p em primeiro lugar e a letra a em segundo?


Fazer
g) que tm a letra p em primeiro lugar ou a letra a em segundo?

h) que tm p em primeiro lugar ou a em segundo ou c em terceiro?

i) nos quais a letra a uma das letras esquerda de p e a letra c

uma das letras direita de p?

2. Se A um conjunto de n elementos, quantas so as funes f :AA


bijetoras?

3. De quantos modos possvel colocar 8 pessoas em la de modo que duas

dessas pessoas, Vera e Paulo, no quem juntas?

4. De quantos modos possvel colocar 8 pessoas em la de modo que duas

dessas pessoas, Vera e Paulo, no quem juntas e duas outras, Helena e

Pedro, permaneam juntas?

5. Quantas so as permutaes simples dos nmeros

1, 2, 3, . . . , 10,

nas quais o elemento que ocupa o lugar de ordem k, da esquerda para a

direita, sempre maior que k 3?

6. De quantos modos possvel dividir 15 atletas em trs times de 5 atletas,

denominados Esporte, Tupi e Minas?

7. De quantos modos possvel dividir 15 atletas em trs times de 5 atletas?

8
Combinatria II Continuao Unidade 12

8. De quantos modos possvel dividir 20 objetos em 4 grupos de 3 ou 2

grupos de 4?

9. Um campeonato disputados por 12 clubes em rodadas de 6 jogos cada.

De quantos modos possvel selecionar os jogos da primeira rodada?

10. Permutam-se de todas as formas possveis os algarismos 1, 2, 4, 6, 7 e

escrevem-se os nmeros assim formados em ordem crescente. Determine:

a) que lugar ocupa 62 417.

o
b) que nmero que ocupa o 66 lugar.

o
c) qual o 166 algarismo escrito.

d) a soma dos nmeros assim formados.

9
MA12 - Unidade 12
Permutacoes e Combinacoes

Paulo Cezar Pinto Carvalho

PROFMAT - SBM

4 de Abril de 2014
Permutacoes Simples

De quantos modos podemos ordenar em fila n objetos


distintos?
A escolha do objeto que ocupara o primeiro lugar pode ser
feita de n modos;
A escolha do objeto que ocupara o segundo lugar pode ser
feita de n 1 modos;
...
A escolha do objeto que ocupara o ultimo lugar pode ser feita
de 1 modo.
O numero total de possibilidades (cada uma das quais e
chamada de uma permutacao simples dos n objetos) e
Pn = n(n 1)(n 2) 1 = n!.

PROFMAT - SBM MA12 - Unidade 12, Permutacoes e Combinacoes slide 2/1


Exemplo

Quantos sao os anagramas da palavra CALOR? Quantos


comecam com consoantes?
Cada anagrama corresponde a uma permutacao dessas 5
letras. O numero de anagramas e P5 = 5! = 120.

Para formar um anagrama comecado por consoante:


A consoante pode ser escolhida de 3 modos;
As demais letras podem ser colocadas em qualquer ordem;
logo, ha P4 = 4! = 24 possibilidades de ordenacao.
Logo, ha 3 24 = 72 anagramas comecados por consoante.

PROFMAT - SBM MA12 - Unidade 12, Permutacoes e Combinacoes slide 3/1


Exemplo

De quantos modos podemos arrumar em fila 5 livros


diferentes de Matematica, 3 livros diferentes de Estatstica e 2
livros diferentes de Fsica, de modo que livros de uma mesma
materia permanecam juntos?
Podemos escolher a ordem das materias de 3! modos.
Feito isso, ha 5! modos de colocar os livros de Matematica
nos lugares que lhe foram destinados, 3! modos para os de
Estatstica e 2! modos para os de Fsica.
O numero total de possibilidades e
3!5!3!2! = 6 120 6 2 = 8 640

PROFMAT - SBM MA12 - Unidade 12, Permutacoes e Combinacoes slide 4/1


Permutacoes de elementos nem todos distintos

Quantos sao os anagramas da palavra BOTAFOGO?


Se as letras fossem diferentes a resposta seria 8!.
Como as tres letras O sao iguais, quando as trocamos entre si
obtemos o mesmo anagrama e nao um anagrama distinto.
Isso faz com que na nossa contagem de 8! tenhamos contado
o mesmo anagrama 3! vezes.
8!
O numero de anagramas e = 6 720.
3!

PROFMAT - SBM MA12 - Unidade 12, Permutacoes e Combinacoes slide 5/1


Generalizando

O numero de permutacoes de n objetos, dos quais sao


iguais a A, sao iguais a B, sao iguais a C , etc, e
n!
Pn,,,... =
!!! . . .
.

PROFMAT - SBM MA12 - Unidade 12, Permutacoes e Combinacoes slide 6/1


Combinacoes Simples

De quantos modos podemos selecionar p objetos distintos


entre n objetos distintos dados? Equivalentemente, quantos
sao os subconjuntos com p elementos de um conjunto com n
elementos?
Observacao: cada selecao de p objetos (ou seja, cada
subconjunto de p elementos) e chamada de uma combinacao
simples de classe p dos n objetos.

PROFMAT - SBM MA12 - Unidade 12, Permutacoes e Combinacoes slide 7/1


Combinacoes Simples
De quantos modos podemos selecionar p objetos distintos
entre n objetos distintos dados? Equivalentemente, quantos
sao os subconjuntos com p elementos de um conjunto com n
elementos?
Comecamos escolhendo, em ordem, p elementos, o que pode
ser feito de n.(n 1) . . . (n p + 1) modos.
Deste modo, cada subconjunto com p elementos e contado p!
vezes, ja que ele aparece em cada ordem possvel.
O numero de combinacoes simples de classe p de n objetos e:

n.(n 1) . . . (n p + 1) n!
Cnp = = .
p! p!(n p)!

PROFMAT - SBM MA12 - Unidade 12, Permutacoes e Combinacoes slide 8/1


Exemplo

Com 5 homens e 4 mulheres, quantas comissoes de 5 pessoas,


com exatamente 3 homens, podem ser formadas?
Para formar a comissao devemos escolher 3 dos homens e 2
das mulheres.
Logo, ha C53 C42 = 10 6 = 60 comissoes.

PROFMAT - SBM MA12 - Unidade 12, Permutacoes e Combinacoes slide 9/1


Exemplo

Com 5 homens e 4 mulheres, quantas comissoes de 5 pessoas,


com pelo menos 3 homens, podem ser formadas?
Ha comissoes com: 3 homens e 2 mulheres, 4 homens e 1
mulher, 5 homens.
A resposta e

C53 C42 + C54 C41 + C55 = 10 6 + 5 4 + 1 = 81.

Um erro comum:
Comeco escolhendo 3 homens, para garantir pelo menos tres
homens (C53 modos).
A seguir, escolho mais 2 pessoas (C62 modos).
Logo, ha C53 .C62 = 10.15 = 150 comissoes (!).

PROFMAT - SBM MA12 - Unidade 12, Permutacoes e Combinacoes slide 10/1


Lista de Exerccios
Unidade 12

1. Quantos sao os anagramas da palavra CAPITULO.

a) possveis?
b) que comecam e terminam por vogal?
c) que tem as vogais e as consoantes intercaladas?
d) que tem as letras c, a, p juntas nessa ordem?
e) que tem as letras c, a, p juntas em qualquer ordem?
f) que tem a letra p em primeiro lugar e a letra a em segundo?
g) que tem a letra p em primeiro lugar ou a letra a em segundo?
h) que tem p em primeiro lugar ou a em segundo ou c em terceiro?
i) nos quais a letra a e uma das letras a esquerda de p e a letra c e
uma das letras a direita de p?

2. Se A e um conjunto de n elementos, quantas sao as funcoes f : A A


bijetoras?

3. De quantos modos e possvel colocar 8 pessoas em fila de modo que


duas dessas pessoas, Vera e Paulo, nao fiquem juntas?

4. De quantos modos e possvel colocar 8 pessoas em fila de modo que


duas dessas pessoas, Vera e Paulo, nao fiquem juntas e duas outras,
Helena e Pedro, permanecam juntas?

5. Quantas sao as permutacoes simples dos numeros

1, 2, 3, . . . , 10,

nas quais o elemento que ocupa o lugar de ordem k, da esquerda para


a direita, e sempre maior que k 3?

6. De quantos modos e possvel dividir 15 atletas em tres times de 5


atletas, denominados Esporte, Tupi e Minas?

1
7. De quantos modos e possvel dividir 15 atletas em tres times de 5
atletas?

8. De quantos modos e possvel dividir 20 objetos em 4 grupos de 3 ou 2


grupos de 4?

9. Um campeonato e disputados por 12 clubes em rodadas de 6 jogos cada.


De quantos modos e possvel selecionar os jogos da primeira rodada?

10. Permutam-se de todas as formas possveis os algarismos 1, 2, 4, 6, 7


e escrevem-se os numeros assim formados em ordem crescente. Deter-
mine:

a) que lugar ocupa 62 417.


b) que numero que ocupa o 66o lugar.
c) qual o 166o algarismo escrito.
d) a soma dos numeros assim formados.

2
Solucoes da Lista de Exerccios
Unidade 12

1. (a) O numero total de anagramas e 8! = 40.320.


(b) Ha 4 modos de escolher a vogal que sera a primeira letra do ana-
grama e 3 modos de selecionar a vogal que sera a ultima letra do
anagrama. Depois disso, ha 6! modos de arrumar as demais letras
entre a primeira e a ultima.
A resposta e 4 3 6! = 4 3 720 = 8.640.
(c) As vogais e consoantes podem aparecer na ordem CV CV CV CV
ou na ordem VC VC VC VC. No primeiro caso, devemos colocar as
4 vogais nos 4 lugares de ordem par (4! modos) e as 4 consoantes
nos 4 lugares de ordem mpar (4! modos).
Ha 4! 4! = 24 24 = 576 anagramas do primeiro tipo. Analo-
gamente, ha 576 anagramas do segundo tipo.
A resposta e 576 + 576 = 1.152.
(d) Tudo se passa como se CAP fosse uma unica letra. Devemos,
portanto, arrumar em fila 6 objetos: CAP,I,T,U,L,O.
A resposta e 6! = 720.
(e) Primeiramente, devemos escolher a ordem em que as letras C, A,
P aparecerao. Ha 3! modos. Depois, devemos arrimar em a fila 6
objetos: o bloco de letras C, A, P e as 5 letras I, T, U, L, O. Ha
6! modos.
A resposta e 3! 6! = 3 720 = 4320.
(f) Basta arrumar em fila, depois de PA, as restantes 6 letras.
A resposta e 6! = 720.
(g) Ha 7! anagramas com a letra P em primeiro lugar e ha 7! anagra-
mas com a letra A em segundo lugar. Ha tambem 6! anagramas
com a letra P em primeiro lugar e A em segundo lugar. Ao somar-
mos 7! com 7!, encontramos o numero de anagramas com P em
primeiro lugar ou A em segundo lugar, mas contamos duas vezes
os anagramas que tem P em primeiro lugar e A em segundo lugar.
A resposta e 7! + 7! 6! = 5040 + 5040 720 = 9.360.

1
(h) Ha 7! anagramas com a letra P em primeiro lugar, 7! anagramas
com a letra A em segundo e 7! anagramas com a letra C em ter-
ceiro. Ha tambem 6! anagramas com P em primeiro lugar e A em
segundo lugar, 6! anagramas com P em primeiro e C em terceiro
e 6! anagramas com A em segundo e C em terceiro. Finalmente,
ha 5! anagramas com P em primeiro lugar, A em segundo e C em
terceiro.
Ao somarmos 7! com 7! com 7!, encontramos o numero de ana-
gramas que tem P em primeiro lugar ou A em segundo ou C em
terceiro, mas contamos alguns anagramas varias vezes.
Contamos duas vezes os anagramas que tem P em primeiro lugar
e A em segundo; o mesmo se deu com os que tem P em primeiro
e C em terceiro e com os que tem A em segundo e C em terceiro.
Descontando essas contagens indevidas, chegamos a 7! + 7! + 7!
6! 6! 6! = 3.7! 3.6!.
Entretanto, anagramas com P em primeiro lugar e A em segundo e
C em terceiro foram, inicialmente, contados tres vezes e, posterior-
mente, descontados tres vezes, o que significa que nao estao sendo
contados. Incluindo-os na contagem, obtemos a resposta correta,
que e 3.(7!) 3.(6!) + 5! = 3.(5.040) 3.(720) + 120 = 13.080.
(i) Como ha 6 ordens possveis para as letras C, A e P, os anagramas
pedidos sao exatamente 1/6 do total, ou seja, 8!/6 = 6.720.
2. O valor de f (a1 ) pode ser escolhido de n modos; o valor de f (a2 ), de
n 1 modos; . . . ; o de f (an ), de 1 modo.
A resposta e n(n 1) 1 = n!.
3. O numero total de modos de sentar 8 pessoas em 8 cadeiras e o numero
de modos de arrumar 8 pessoas em fila, 8!. O numero de modos de
arrumar 8 pessoas em fila de modo que duas dessas pessoas, Vera e
Paulo, fiquem juntas e 2.7!, pois, para formar uma tal fila, devemos
inicialmente decidir em que ordem se colocarao Vera e Paulo e, em
seguida formar uma fila de 7 objetos: o bloco formado por Vera e
Paulo; as demais 6 pessoas.
A resposta e 8! 2.7! = 40.320 10.080 = 30.240.
4. Como visto no problema anterior, o numero de filas nas quais duas
pessoas (neste caso Helena e Pedro) ficam juntas e 2.7! = 10.080. O

2
numero de filas onde Helena e Pedro e tambem Vera e Paulo ficam
juntos e obtido de modo analogo: agora sao dois blocos de duas pes-
soas, cada um podendo ser arrumado de dois modos distintos e mais
4 pessoas. Portanto, o numero de tais filas e 2.2.6! = 2.880. Logo, o
numero de filas em que Helena e Pedro ficam juntos, mas Vera e Paulo
nao, e 10, 080 2.880 = 7.200.

5. O elemento da permutacao que ocupa o 10o lugar deve ser maior que 7.
Pode ser escolhido de 3 modos. O elemento da 9a posicao deve ser maior
que 6; haveria 4 possibilidades, mas uma delas ja foi usada na escolha
do elemento que ocupa a 10a posicao. Pode ser escolhido de 3 modos.
Prosseguindo com esse raciocnio, vemos que a cada nova casa abranda-
se a restricao, criando uma possibilidade a mais, mas ao mesmo tempo
diminui-se uma possibilidade, pois uma delas foi usada na etapa. Ou
seja, ha 3 possibilidades para cada casa ate a 3a casa. O elemento
da 3a posicao deve ser maior que 3 3 = 0; havera 10 possibilidades,
mas 7 delas ja foram usadas nas etapas anteriores. Pode ser escolhido
de 10 7 = 3 modos. O elemento da 2a posicao deve ser maior que
2 3 = 1; haveria 10 possibilidades mas 8 ja foram usadas nas etapas
anteriores. Pode ser escolhido de 10 8 = 2 modos. Finalmente, o
elemento de posicao 1 deve ser maior que 1 3 = 2; haveria 10
possibilidades, mas 9 delas ja foram usadas nas etapas anteriores. Pode
ser escolhido de 10 9 = 1 modo.
A resposta e 38 .2.1 = 13.122.
5 5
6. Ha C15 modos de formar o Esporte; depois disso, C10 modos de formar
o Tupi; finalmente, 1 unico modo de formar o Minas.
5 5
A resposta e C15 C10 1 = 756.756.

7. O numero de possibilidades e igual ao numero obtido no problema


anterior dividido por 3! = 6, ja que permutando os nomes dos times a
subdivisao continua a mesma. A resposta e 756.745/6 = 126.126.

8. Escolha, sucessivamente, 3 pessoas para formar os 4 grupos de 3; isto


3 3 3 3
pode ser feito, sucessivamente, de C20 , C17 , C14 e C11 modos. A seguir,
com as 8 pessoas restantes for os 2 grupos restantes, o que pode ser
feito de C84 e C44 modos, respectivamente. Fazendo isso, contamos cada
divisao 4!.2! vezes, porque, quando formamos os mesmos grupos de 3 e

3
os mesmos grupos de 4 em outra ordem, contamos como se fosse outra
divisao em grupos.
C 3 .C 3 .C 3 .C 3 .C 4 .C 4 20!
A resposta e 20 17 14 1 8 4 = = 67.897.830.000.
4!.3! (3!) (4!)2 4!2!
4

9. Os adversarios em cada jogo podem ser escolhidos, sucessivamente, de


2 2
C12 , C10 , C82 , C62 , C42 e C22 modos. No entanto, assim contamos cada
possvel rodada 6! vezes, ja que contamos diferentes ordens dos jogos
C 2 .C 2 .C 2 .C 2 .C 2 .C 2
como se fossem rodadas diferentes. A resposta e 12 10 8 6 4 2 =
6!
12!
= 10.395
26 .6!
10. (a) Para determinar o lugar ocupado pelo numero 62.417, devemos
contar quantos numeros estao antes dele. Antes dele estao os
numeros comecados por:
i. 1 (4!=24 numeros)
ii. 2(4!=24 numeros)
iii. 4 (4!=24 numeros)
iv. 61 (3!=6 numeros)
v. 621 (4!=2 numeros)
Ha 24 + 24 + 24 + 6 + 2 = 80 numeros antes do 62.417.
A resposta e 81o
(b) Como ha 4! = 24 numeros comecados por 1, e 4! = 24 numeros
comecados por 2 e 3! = 6 numeros comecados por 41, e 3! = 6
numeros comecados por 42, e 3! = 6 numeros comecados por 46,
o 66o numero escrito e o ultimo dos numeros comecados por 46,
ou seja, 46.721.
A resposta e 46721.
(c) Como ha 5 algarismos em cada numero, o 166o algarismo escrito
e o primeiro algarismo do 36o numero.
Como ha 4! = 24 numeros comecados por 1, e 3! = 6 numeros
comecados por 21, e 3! = 6 numeros comecados por 24, o 36o
numero escrito e o ultimo dos numeros comecados por 26. Logo,
seu primeiro algarismo e 2.

4
(d) Nas casas das unidades desses numeros, aparecem apenas os alga-
rismos 1, 2, 4, 6, 7, cada um deles 4! = 24 vezes. A soma das unida-
des desses numeros e, portanto, (1+2+4+6+7).24 = 480 unidades,
ou seja, 480. A soma das dezenas e, analogamente, igual a 480 de-
zenas, ou seja, 4.800. A soma das centenas e igual a 480 centenas,
ou seja, 48.000. A soma das unidades de milhar e igual a 480 uni-
dades de milhar, ou seja, 480.000. Finalmente, a soma das dezenas
de milhar e igual a 480 dezenas de milhar, ou seja, 4.800.000. A
resposta e 480 + 4.800 + 48.000 + 480.000 + 4.800.000 = 5.333.280.

5
13
Combinatria III
Continuao
Sumrio
13.1 Introduo . . . . . . . . . . . . . . . . . . . . . . . 2

13.2 O Tringulo Aritmtico . . . . . . . . . . . . . . . . 4

13.3 O Binmio de Newton . . . . . . . . . . . . . . . . . 5

1
Unidade 13 Introduo

13.1 Introduo

A unidade se inicia com o tringulo de Tartaglia-Pascal, que uma tabela de

formato triangular (no limitada) de nmeros naturais, fcil de construir e que

permite obter de modo imediato os coecientes do desenvolvimento de (a+b)n .

1
1 1
1 2 1
1 3 3 1
1 4 6 4 1
1 5 10 10 5 1
1 6 15 20 15 6 1

Esse tringulo foi descoberto pelo matemtico chins Yang Hui (1238-1298) e

suas propriedades aritmticas foram estudadas pelo matemtico francs Blaise

Pascal (1623-1662). Este escreveu o livro Trait du Triangle Arithmtique ,

publicado em 1654, razo pela qual o tringulo leva o seu nome. Pascal, junto

com Fermat, foi o criador da Anlise Combinatria (assunto das Unidades 11-

16) e da Teoria de Probabilidades, que estudaremos nas Unidades 17-20.

Dentre as propriedades notveis do triangulo de Pascal, destacam-se a simetria

axial com relao ao eixo vertical central e a relao de Stifel


     
n1 n1 n
+ = ,
m1 m m
onde  
n n!
= ,
m m!(n m)!
nmero esse tambm denotado por Cm
n.

Essa relao a base da construo do tringulo, pois permite determinar os

elementos de uma linha conhecendo os elementos da linha anterior. Destacam-

se tambm o Teorema das Linhas o Teorema das Colunas, dentre muitas outras

propriedades.

A seguir, apresentado o Binmio de Newton, ou seja, a frmula que fornece o

desenvolvimento de (a+b)n de um modo diferente do que foi feito na Unidade 4.

2
Combinatria III Continuao Unidade 13

Aqui se utilizam argumentos combinatrios, ao invs dos argumentos algbricos

que foram utilizados l.

O Binmio de Newton era conhecido muito antes de Newton, mas leva o seu

nome porque ele teve a formidvel idia de usar esse desenvolvimento com ex-

poentes racionais para fazer uma generalizao inesperada do clssico Teorema

da Funo Implcita para equaes polinomiais f (X, Y ) = 0, onde f (0, 0) = 0,


em condies onde no se aplica o teorema clssico, ou seja, quando

f f
(0, 0) = 0 e (0, 0) = 0.
x y

Como so denidos tais desenvolvimentos? Bem, formalmente, podemos

denir, para n racional e m natural os coecientes binomiais como de costume


 
n n(n 1) (n m + 1)
= .
m m!
n

Note que, se n for inteiro (xado), ento
m
se anula para m n + 1, o

que no o caso se n for um nmero racional que no natural. Nessa

situao, o coecientes binomiais nunca se anulam. Portanto, podemos escre-

ver formalmente, como Newton fez, o desenvolvimento em srie innita

( 1) 2 ( 1) ( m + 1) m
(1+X) = 1+X+ X + + X + . (13.1)
2 m!
Essa srie foi responsvel pelo famoso paradoxo do binmio, que intrigou os

matemticos at ser denitivamente esclarecido por Gauss. Esse paradoxo se

obtm, por exemplo, fazendo em (1) a substituio X = 2 e = 1, obtendo


1 = 1 + 2 + 22 +
A razo do surgimento desse paradoxo, como explicado por Gauss, consiste em

tratar somas innitas como se fossem nitas. A igualdade s vale se a srie da

direita for convergente, o que s ocorre quando |X| < 1, e isso no o caso

quando X = 2. Por a pode-se ter mais uma comprovao da genialidade de

Gauss, que introduziu a noo de convergncia para sries, iniciando o ramo

da Anlise Matemtica. Gauss fez o estudo completo da srie hipergeomtrica,

que contm, como casos particulares, vrias sries conhecidas. Infelizmente, a

matemtica de Gauss muito pouco abordada no Ensino Mdio.

Para nalizar, resolva a lista de problemas propostos e leia a seo Sobre o

Ensino de Combinatria.

3
Unidade 13 O Tringulo Aritmtico

13.2 O Tringulo Aritmtico

1 2
Chamamos de tringulo aritmtico de Tartaglia -Pascal ao quadro abaixo,
p
formado com os diversos valores de Cn .

C00 1
C10 C11 1 1
C20 C21 C22 1 2 1
C30 C31 C32 C33 1 3 3 1
C40 C41 C42 C43 C44 1 4 6 4 1
C50 C51 C52 C53 C54 C55 1 5 10 10 5 1

Observe que, enumerando as linhas e colunas a partir de zero, Cnp aparece na

linha n e coluna p.
A propriedade que permite construir rapidamente o tringulo a relao de
3
Stifel , que diz que somando dois elementos lado a lado no tringulo obtm-se

o elemento situado embaixo do da direita. Assim, a prxima linha do tringulo

seria

1, 1 + 5 = 6, 5 + 10 = 15, 10 + 10 = 20, 10 + 5 = 15, 5 + 1 = 6, 1.

Proposio 1 p+1
Cnp + Cnp+1 = Cn+1 .
Relao de Stifel

Demonstrao Considere um conjunto A de n + 1 elementos, um dos quais x. O nmero


p+1
de subconjuntos de A com p + 1 elementos Cn+1 . Esse nmero igual soma
p+1
do nmero de subconjuntos nos quais x no gura, Cn , com o nmero de
p
subconjuntos nos quais x gura, Cn .

Outra relao importante dada pelo:

1 Tartaglia, Nicolo Fontana (1500-1557), matemtico italiano.


2 Pascal, Blaise (1623-1662), matemtico, lsofo e fsico francs.
3 Stifel, Michael (1487?-1567), algebrista alemo.

4
Combinatria III Continuao Unidade 13

Cn0 + Cn1 + Cn2 + + Cnn = 2n . Teorema 2


Teorema das Linhas

Basta observar que os dois membros so iguais ao nmero de subconjuntos


Demonstrao
de um conjunto com n elementos.

Um palcio tem 7 portas. De quantos modos pode ser aberto o palcio? Exemplo 1
Soluo. H C71 modos de abrir o palcio abrindo uma s porta, C72 modos de

abrir o palcio abrindo duas portas, etc. A resposta

C11 + C72 + + C77 = 27 C70 = 128 1 = 127.

Finalmente, a relao que declara que, em cada linha, elementos equidis-

tantes dos extremos so iguais.

Cnp = Cnnp . Proposio 3


Combinaes
Complementares
Basta observar que o nmero de modos de escolher, entre n objetos, p Demonstrao
objetos para usar igual ao de escolher np objetos para no usar.

13.3 O Binmio de Newton

4
A frmula do binmio de Newton a frmula que d o desenvolvimento
n
de (x + a) . Para obt-la basta multiplicar

(x + a) (x + a) (x + a).

O termo genrico do produto obtido tomando em p dos fatores, p = 0, 1, 2, . . . , n,


a segunda parcela e tomando nos restantes n p fatores a primeira parcela.
4 Newton, Isaac (1642-1727), matemtico e fsico ingls.

5
Unidade 13 O Binmio de Newton

Como isso pode ser feito de Cnp modos, o termo genrico do produto Cnp ap xnp
e

n
X
n
(x + a) = Cnp ap xnp
p=0

= Cn0 a0 xn + Cn1 a1 xn1 + Cn2 a2 xn2 + + Cnn an x0 .

Exemplo 2 Determine o coeciente de x3 no desenvolvimento de


 7
4 1
x .
x

Soluo. O termo genrico do desenvolvimento

 p
1
C7p (x4 )7p = C7p (1)p x285p .
x

O termo em x3 obtido se 28 5p = 3, ou seja, se p = 5.


5 5 3
O termo procurado C7 (1) x = 21x3 . O coeciente 21.

Exemplo 3 Determine o termo mximo do desenvolvimento de


 50
1
1+ .
3

Soluo. O termo genrico do desenvolvimento

 p
p 1
tp = Cnp ap xnp = C50 .
3

Vamos descobrir para que valores de p os termos crescem. Para isso, calcu-

lamos
 p  p1
p 1 p1 1
tp tp1 = C50 C50
3 3
50! 50!
=
p!(50 p)!3p (p 1)!(51 p)!3p1
 
50! 1 1
=
(p 1)!(50 p)!3p1 3p 51 p
 
50! 51 4p
= .
(p 1)!(50 p)!3p1 3p(51 p)

6
Combinatria III Continuao Unidade 13

Temos tp tp1 positivo, isto , tp > tp1 quando 514p > 0 e temos tp < tp1
quando 51 4p < 0.
Portanto, tp > tp1 quando p 6 12 e tp < tp1 quando p > 13. Logo,

t0 < t1 < < t11 < t12 > t13 > t14 > > t50 . O termo mximo
12
C50
t12 = .
312

+ Na Sala de Aula - Sobre o Ensino de Combinatria - Clique para ler

7
Unidade 13 O Binmio de Newton

Exerccios Recomendados

1. Com 7 vitaminas diferentes, quantos coquetis de duas ou mais vitaminas

podemos formar?

2. Determine p para que seja mximo:

p p
a) C10 b) C21

3. Determine o termo independente de x no desenvolvimento de

 10
3 1
x 2 .
x

4. Determine o coeciente de xn no desenvolvimento de (1 x)2 (x + 2)n .

5. Determine o valor da soma Cn0 + 3Cn1 + 32 Cn2 + + 3n Cnn .

6. Se (1 + x + x2 )n = A0 + A1 x + A2 x2 + + A2n x2n , determine o valor

de:

a) A0 + A1 + A2 + + A2n
b) A0 + A2 + A4 + + A2n .

7. Determine o termo mximo do desenvolvimento de

 100
1
1+ .
2

8. Prove que 10150 > 9950 + 10050 .

8
Combinatria III Continuao Unidade 13

Sobre o Ensino de Combinatria Na Sala de Aula

1. No faa frmulas demais ou casos particulares demais. Isso obscurece

as ideias gerais e torna as coisas mais complicadas. Quem troca o princpio

bsico da contagem por frmulas de arranjos, permutaes e combinaes tem

diculdade de resolver at mesmo o nosso segundo exemplo (o das bandeiras).

2. Aprenda e faa com que os alunos aprendam com os erros. importante,

diante de uma soluo errada, analisar porque ela est errada.

3. Voc quer mostrar que o bom ou quer que seus alunos aprendam? Se voc

prefere a segunda alternativa, resista tentao de em cada problema buscar

soluo mais elegante. O que deve ser procurado um mtodo que permita

resolver muitos problemas e no um truque que resolva maravilhosamente um

problema. Sendo mais especco: no exemplo 6, da seo de princpios bsicos,

foram apresentados dois mtodos e um truque. No se deve mostrar o truque

antes de mostrar os mtodos. A beleza de alguns truques s pode ser apreciada

por quem tem domnio dos mtodos. Combinatria no difcil; impossvel

aprender alguma coisa apenas com truques em vez de mtodos.

4. No d preferncia a raciocnios destrutivos, raciocnios do tipo contar a

mais e depois descontar o que no servia e foi contado indevidamente. Os

raciocnios que resolvem a maior parte dos problemas de Combinatria so

essencialmente construtivos. Embora em certos casos seja melhor usar um

raciocnio destrutivo, seus alunos s se sentiro seguros quando dominarem os

raciocnios construtivos. Por exemplo, no exemplo 7 da parte de combinaes, a

primeira soluo apresentada melhor do que a segunda para educar o raciocnio

do aluno.

5. Um processo seguro de tornar as coisas complicadas comear assim: esse

um problema de arranjos ou de combinaes? Como se resolveriam, por

exemplo, os problemas dos exemplos 2, 3 e 5 da Unidade 11 e os problemas

propostos nmeros 1, 5, 8 e 10 da prxima unidade? Alis, para que servem

arranjos?

9
MA12 - Unidade 13
Outras tecnicas de contagem

Paulo Cezar Pinto Carvalho

PROFMAT - SBM

4 de Abril de 2014
Permutacoes Circulares
De quantos modos 5 criancas podem formar uma roda?

A resposta nao e 5! = 120 rodas, porque as rodas ABCDE e


EACBD, por exemplo, sao na verdade a mesma roda.
Como cada roda pode aparecer em 5 posicoes, a resposta e
120/5 = 24.
De modo geral, o numero de permutacoes circulares de n
n!
elementos e = (n 1)!.
n

PROFMAT - SBM MA12 - Unidade 13, Outras tecnicas de contagem slide 2/1
Combinacoes completas (ou com repeticao)
Quantas sao as solucoes inteiras e nao-negativas da equacao
x1 + x2 + + xn = p?
Cada solucao da equacao coresponde a uma forma de escolher
p elementos dentre 1, 2, . . . n, mas permitindo repeticoes. O
numero de solucoes e representado por CRnp .
Cada solucao da equacao pode ser representada por uma fila
de p sinais + e n 1 sinais |, que separam as incognitas.
Reciprocamente, cada fila desta forma corresponde a uma
solucao.
Por exemplo: + + +| + ||+ corresponde a solucao (3,1,0,1) da
equacao x1 + x2 + x3 + x4 = 5.
Para contar a quantidade de tais filas, basta escolher p dentre
os n + p 1 lugares para colocar os sinais +.
Logo, CRnp = Cn+p1
p
.

PROFMAT - SBM MA12 - Unidade 13, Outras tecnicas de contagem slide 3/1
Exemplo

De quantos modos podemos comprar 3 sorvetes em uma


sorveteria que os oferece em 6 sabores distintos?
Chamando de xk o numero de sorvetes do k-esimo sabor que
vamos comprar, devemos determinar valores inteiros e
nao-negativos para xk , k = 1, 2, 3, 4, 5, 6, tais que
x1 + x2 + + x6 = 3.
O numero de possibilidades e CR63 = C83 = 56 modos.

PROFMAT - SBM MA12 - Unidade 13, Outras tecnicas de contagem slide 4/1
Lista de Exerccios
Unidade 13

1. De quantos modos podemos formar uma mesa de buraco com 4 joga-


dores?

2. De quantos modos podemos formar uma roda de ciranda com 5 meninos


e 5 meninas de modo que pessoas de mesmo sexo nao fiquem juntas?

3. De quantos modos podemos formar uma roda de ciranda com 6 criancas,


de modo que duas delas, Vera e Isadora, nao fiquem juntas?

4. Quantas sao as solucoes inteiras e positivas de x + y + z = 7?

5. Quantas sao as solucoes inteiras e nao-negativas de x + y + z 6 6?

6. Uma industria fabrica 5 tipos de balas que sao vendidas em caixas de


20 balas, de um so tipo ou sortidas. Quantos tipos de caixas podem
ser montados?

1
Solucoes da Lista de Exerccios
Unidade 13

1. A resposta e o numero de permutacoes circulares de 4 elementos, ou


seja, 3! = 6.

2. Ha (P C)5 = 4! modos de formar uma roda com as meninas. Depois


disso, os 5 meninos devem ser postos nos 5 lugares entre as meninas, o
que pode ser feito de 5! modos. A resposta e 4! 5! = 24 120 = 2.880.

3. E mas simples calcular o numero total de rodas e excluir aquelas em


que Vera e Isadora ficam juntas. O numero total de rodas e P C6 =
5! = 120. Para formar as rodas em que Vera e Isadora se colocarao na
roda. Ha 2 possibilidades: Vera-Isadora e Isadora-Vera. Agora tudo
se passa como se Vera e Isadora fossem uma unica crianca. Assim,
ha 2(P C)5 = 2.4! = 48 rodas em que Vera e Isadora ficam juntas. A
resposta e 120 48 = 72 rodas.

4. Chamando x de 1 + a, y de 1 + b e z de 1 + c, o problema se transforma


em encontrar todas as solucoes inteiras e nao-negativas de (a + 1) + (b +
1) + (c + 1) = 7, ou seja, de a + c + c = 4. A resposta e CR34 = C64 = 15.

5. Cada solucao inteira e nao negativa da inequacao x + y + z 6 corres-


ponde a uma solucao inteira e nao negativa da equacao x+y+z +f 6.
Logo, ha CR46 = C96 = 84 solucoes.

6. Para formar uma caixa, devemos selecionar 20 dentre os 5 tipos, valendo


repeticao na escolha. Ou seja, devemos formar solucoes inteiras e nao
negativas de x1 +x2 +x3 +x4 +x5 = 20, onde xi e o numero de bombons
20
do tipo i. A resposta e CR520 = C24 = 10.626.

1
14
Problemas de
Combinatria
Lista I

1
Unidade 14

Problemas
Lista 1
1. Em um corredor h 900 armrios, numerados de 1 a 900, inicialmente

todos fechados. 900 pessoas, numeradas de 1 a 900, atravessam o corre-

dor. A pessoa de nmero k reverte o estado de todos os armrios cujos


nmeros so mltiplos de k . Por exemplo, a pessoa de nmero 4 mexe

nos armrios de nmeros 4, 8, 12, . . . , abrindo os que encontra fechados

e fechando os que encontra abertos. Ao nal, quais armrios caro

abertos?

2. Dispomos de 5 cores distintas. De quantos modos podemos colorir os

quatro quadrantes de um crculo, cada quadrante com uma s cor, se

quadrantes cuja fronteira uma linha no podem receber a mesma cor?

3. De quantos modos podemos formar uma palavra de 5 letras de um alfa-

beto de 26 letras, se a letra A deve gurar na palavra mas no pode ser

a primeira letra da palavra? E se a palavra devesse ter letras distintas?

4. As placas dos veculos so formadas por trs letras (de um alfabeto de

26) seguidas por 4 algarismos. Quantas placas podero ser formadas?

5. Um vago do metr tem 10 bancos individuais, sendo 5 de frente e 5

de costas. De 10 passageiros, 4 preferem sentar de frente, 3 preferem

sentar de costas e os demais no tm preferncia. De quantos modos eles

podem se sentar, respeitadas as preferncias?

6. Escrevem-se os inteiros de 1 at 2 222. Quantas vezes o algarismo 0

escrito?

7. Quantos so os inteiros positivos de 4 dgitos nos quais o algarismo 5

gura?

8. Em uma banca h 5 exemplares iguais da Veja, 6 exemplares iguais da

poca e 4 exemplares iguais da Isto . Quantas colees no-vazias

de revistas dessa banca podem ser formadas?

2
Problemas de Combinatria Lista I Unidade 14

9. Uma turma tem aulas as segundas, quartas e sextas, de 13h s 14h e de

14h s 15h. As matrias so Matemtica, Fsica e Qumica, cada uma

com duas aulas semanais, em dias diferentes. De quantos modos pode

ser feito o horrio dessa turma?

10. O problema do Exemplo 1 da Unidade 11 Com 5 homens e 5 mulheres,


de quantos modos se pode formar um casal? foi resolvido por um aluno

do modo a seguir: A primeira pessoa do casal pode ser escolhida de 10

modos, pois ela pode ser homem ou mulher. Escolhida a primeira pessoa,

a segunda pessoa s poder ser escolhida de 5 modos, pois deve ser de

sexo diferente da primeira pessoa. H portanto 10 5 = 50 modos de

formar um casal. Onde est o erro?

11. Escrevem-se nmeros de 5 dgitos, inclusive os comeados em 0, em car-

tes. Como 0, 1 e 8 no se alteram de cabea para baixo e como 6,

de cabea para baixo, se transforma em 9 e vice-versa, um mesmo car-

to pode representar dois nmeros (por exemplo, 06198 e 86190). Qual

o nmero mnimo de cartes para representar todos os nmeros de 5

dgitos?

12. Qual a soma dos divisores positivos de 360?

3
MA12 - Unidade 14
Triangulo de Pascal e Binomio de Newton

Paulo Cezar Pinto Carvalho

PROFMAT - SBM

4 de Abril de 2014
Triangulo de Tartaglia-Pascal

C00
C10 C11
C20 C21 C22
C30 C31 C32 C33
C40 C41 C42 C43 C44
C50 C51 C52 C53 C54 C55
... ... ... ... ... ... ...
1
1 1
1 2 1
1 3 3 1
1 4 6 4 1
1 5 10 10 5 1
... ... ... ... ... ... ...

PROFMAT - SBM MA12 - Unidade 14, Triangulo de Pascal e Binomio de Newton slide 2/11
Triangulo de Tartaglia-Pascal
Alternativamente:
C00
C01 C11
C02 C12 C22
C03 C13 C23 C23
C04 C14 C24 C34 C44
C05 C15 C25 C35 C45 C55
... ... ... ... ... ... ...

1
1 1
1 2 1
1 3 3 1
1 4 6 4 1
1 5 10 10 5 1
... ... ... ... ... ... ...
PROFMAT - SBM MA12 - Unidade 14, Triangulo de Pascal e Binomio de Newton slide 3/11
Relacao de Stifel
Cnp + Cnp+1 = Cn+1
p+1
.

Prova: Contar de dois modos diferentes os subconjuntos com


p + 1 elementos de {1, 2, . . . , n, n + 1}:
diretamente;
contando separadamente os subconjuntos incluindo ou
excluindo n + 1.

PROFMAT - SBM MA12 - Unidade 14, Triangulo de Pascal e Binomio de Newton slide 4/11
Teorema das linhas
Cn0 + Cn1 + Cn2 + . . . + Cnn = 2n

Prova: Contar de dois modos diferentes os subconjuntos de


{1, 2, . . . , n}:
diretamente;
contando separadamente os subconjuntos de cada
cardinalidade.

PROFMAT - SBM MA12 - Unidade 14, Triangulo de Pascal e Binomio de Newton slide 5/11
Combinacoes complementares
Cnp = Cnnp (termos de uma mesma linha equidistantes dos
extremos sao iguais).

Prova: escolher um subconjunto com p elementos equivale a


escolher seu complementar.
PROFMAT - SBM MA12 - Unidade 14, Triangulo de Pascal e Binomio de Newton slide 6/11
Binomio de Newton

n
X
(x + a)n = Cnp ap x np
p=0

= Cn0 a0 x n + Cn1 a1 x n1 + Cn2 a2 x n2 + + Cnn an x 0 .

Prova:
Considere o produto (x + a)n = (x + a)(x + a) . . . (x + a)
Para formar um termo igual a ap x np , e preciso tomar p
fatores iguais a a e n p fatores iguais a x, o que pode ser
feito de Cnp modos.

PROFMAT - SBM MA12 - Unidade 14, Triangulo de Pascal e Binomio de Newton slide 7/11
Exemplo

Determine o coeficiente de x 3 no desenvolvimento de

1 7
 
4
x .
x

O termo generico do desenvolvimento e


 p
1
C7p (x 4 )7p = C7p (1)p x 285p .
x

O termo em x 3 e obtido quando 28 5p = 3, ou seja, se


p = 5.
O termo procurado e C75 (1)5 x 3 = 21x 3 .
O coeficiente e 21.

PROFMAT - SBM MA12 - Unidade 14, Triangulo de Pascal e Binomio de Newton slide 8/11
Exemplo

Mostre que Cn0 Cn1 + Cn2 + . . . + (1)n Cnn = 0 (ou seja, a


soma das combinacoes de taxa par de n elementos e igual a
soma das combinacoes de taxa mpar).
Basta formar
Pon desenvolvimento de
n p p np
(1 1) = p=0 Cn (1) 1 .

Note que, ao formar o desenvolvimento de (1 + 1)n ,


redescobrimos o Teorema das Linhas:
Cn0 + Cn1 + Cn2 + . . . + Cnn = 2n

PROFMAT - SBM MA12 - Unidade 14, Triangulo de Pascal e Binomio de Newton slide 9/11
Exemplo

Determine
 30o termo maximo do desenvolvimento de
3 1
4 + 4 .

Se uma pessoa responde ao acaso um teste com 30 questoes


com 4 alternativas cada, qual e o numero mais provavel de
questoes certas?
A probabilidade de acertar p questoes e igual ao termo de
ordem p do desenvolvimento:
 p  (30p) 30p
p 1 3 p 3
tp = C30 = C30 .
4 4 430

Vamos descobrir para que valores de p os termos crescem.


Para isso, devemos estudar o comportamento de tp tp1 .

PROFMAT - SBM MA12 - Unidade 14, Triangulo de Pascal e Binomio de Newton slide 10/11
Continuacao

p 330p p1 3
31p
tp tp1 = C30 C 30
430 430
30!330p 30!331p
= 30

p!(30 p)!4 (p 1)!(31 p)!430
30!330p
= ((31 p) 3p)
p!(31 p)!430
30!330p
= (31 4p)
p!(31 p)!430
31
tp > tp1 31 4p > 0 p < 4 = 7, 75 p 7
7 323
O maior termo e t7 = C30 (ou seja, o numero mais
430
provavel de respostas corretas e 7).

PROFMAT - SBM MA12 - Unidade 14, Triangulo de Pascal e Binomio de Newton slide 11/11
Lista de Exerccios
Unidade 14

1. Com 7 vitaminas diferentes, quantos coqueteis de duas ou mais vita-


minas podemos formar?

2. Determine p para que seja maximo:


p p
a) C10 b) C21

3. Utilize a Relacao de Stifel para demonstrar, por inducao finita, o Teo-


rema das Linhas.
p p+1
1
4. Prove o Teorema das Diagonais: Cn0 +Cn+1 2
+Cn+2 +. . .+Cn+p = Cn+p+1 .

5. Determine o termo independente de x no desenvolvimento de


 10
3 1
x 2 .
x

6. Determine o coeficiente de xn no desenvolvimento de (1 x)2 (x + 2)n .

7. Determine o valor da soma Cn0 + 3Cn1 + 32 Cn2 + + 3n Cnn .

8. Se (1 + x + x2 )n = A0 + A1 x + A2 x2 + + A2n x2n , determine o valor


de:
a) A0 + A1 + A2 + + A2n
b) A0 + A2 + A4 + + A2n .

9. Determine o termo maximo do desenvolvimento de


 100
1
1+ .
2

10. Prove que 10150 > 9950 + 10050 .

1
Solucoes da Lista de Exerccios
Unidade 14

1. C72 + C73 + + C77 = 27 C70 C71 = 128 1 7 = 120.


p
2. (a) C10 e elemento da linha 10. Em qualquer linha, o elemento maximo
e o do meio. A resposta e p = 5.
p
(b) C21 e elemento da linha 21. Em qualquer linha, o elemento maximo
e o do meio, A resposta e p = 10 ou p = 11.
3. Para n = 0, temos Cn0 = 1 = 20 , o que mostra que o teorema vale para
a linha 0. Suponhamos que ele valha para a linha n, isto e, Cn0 + Cn1 +
. . . + Cnn = 2n . Temos Cn+1
i
= Cni1 + Cni , para i = 1, 2, . . . , n. Logo,
0 1 2 n1 n n+1
Cn+1 + Cn+1 + Cn+1 + . . . + Cn+1 + Cn+1 + Cn+1 =
0 n+1
Cn+1 +(Cn0 +Cn1 )+(Cn1 +Cn2 )+. . .+(Cnn2 +Cnn1 )+(Cnn1 +Cnn )+Cn+1 =
0 n+1
(Cn+1 + Cn0 ) + 2Cn1 + . . . + 2Cnn1 + (Cnn + Cn+1 )=
2Cn0 + 2Cn1 + . . . + 2Cnn1 + 2Cnn = 2.2n = 2n+1 .
Logo o teorema tambem vale para a linha n+1. Portanto, por inducao,
vale para qualquer linha.
4. O enunciado correto e: Prove o Teorema das Diagonais: Cn0 + Cn+1
1
+
2 p p
Cn+2 + . . . + Cn+p = Cn+p+1 .
Como Cn0 = Cn+1 0
, a propriedade vale para p = 0. Suponhamos que ela
seja valida para p. Entao
p p+1)=
Cn0 + Cn+1
1 2
+ Cn+2 + . . . + Cn+p + Cn+p+1
p+1
p p+1)=Cn+p+2
Cn+p+1 + Cn+p+1 (pela relacao de Stifel),
o que mostra que a propriedade vale para p + 1. Portanto, por inducao,
vale para todo p.
 p
p (1) p
5. Tp+1 = C10 x2 (x3 )10p = C10 (1)p x305p independera de x para
30 5p = 0, ou seja, p = 6.
6
A resposta e T7 = C10 (1)6 x0 = 210.

1
n(n1) n2
6. (1 x2 ) = 1 2x + x2 e (1 + x)n = xn + nxn1 + 2
x + . Os
termos em xn no produto sao:
n(n1) n2 n(n1) n
1.xn = xn ; 2x.xn1 = 2nxn ; x2 2
x = 2
x .
n(n1) n2 5n+2)
A resposta e 1 2n + 2
= 2
.
n
X
7. A soma pedida e Cnk 3k = (1 + 3)n = 4n .
k=0

8. (a) Seja P (x) = (1 + x + x2 )n . Entao, A0 + A1 + A2 + + An =


P (1) = 3n .
(b) Sejam S0 = A0 +A2 + +A2n e S1 = A1 +A3 + +A2n1 . Entao
P (1) + P (1)
P (1) = S0 + S1 e P (1) = S0 S1 . Lodo, S0 = =
2
3n + 1
.
2

p 1 p
 120! 1
9. Tp+1 = C120 2
= pe
p!(120 p)! 2
p1 1 p1
 120! 1
Tp = C120 2
= p1 ;
(p 1)!(121 p)! 2
Tp+1 121 p
= ; Tp+1 > Tp se p 40 e Tp+1 < Tp se p 41
Tp 2p
Da T1 < T2 < < T40 < T41 > T42 > > T121 .
40
C120
O termo maximo e T41 = 40 .
2
10. Sejam a = 10150 e b = 10050 + 9950 . Temos:

a =(100 + 1)50 = C50 0


10050 + C50
1
10049 + C50
2
10048
49 50
+ + C50 100 + C50
b =10050 + (100 1)50 = C50 0
10050 C50
1
10049 + C50
2
10048
49 50
C50 100 + C50 + 10050

Logo,

2
1
a b = 2C50 10049 + 2C50 3
10047 + + 2C50
49
100 10050
1
= 2C50 10049 + + 2C50
49
100 > 0.

Portanto, a > b.

3
15
Problemas de
Combinatria
Lista II

1
Unidade 15

Problemas
Lista 2
1. De quantos modos possvel colocar r rapazes e m moas em la de

modo que as moas permaneam juntas?

2. Quantos dados diferentes possvel formar gravando nmeros de 1 a 6

sobre as faces de um cubo?

a) Suponha uma face de cada cor.

b) Suponha faces iguais.

c) Suponha que as faces so iguais e que a soma dos pontos de faces

opostas deva ser igual a 7.

3. Resolva o problema anterior, no caso b), para os outros 4 poliedros regu-

lares.
n
X
4. Determine n para que k! seja um quadrado perfeito.
k=1

5. Quantos so os anagramas da palavra ESTRELADA?

6. O conjunto A possui n elementos. Quantos so os seus subconjuntos

com p elementos?

7. Uma faculdade realiza seu vestibular em dois dias de provas, com 4 mat-

rias em cada dia. Este ano a diviso foi: Matemtica, Portugus, Biologia

e Ingls no primeiro dia e Geograa, Histria, Fsica e Qumica no segundo

dia. De quantos modos pode ser feito o calendrio de provas?

8. Qual o erro na soluo do problema abaixo?

Com 5 homens e 4 mulheres, quantas comisses de 5 pessoas, com pelo

menos 3 homens, podem ser formadas?

Soluo: Primeiramente vamos escolher 3 homens para a comisso, o

que pode ser feito de C53 = 10 modos. Agora devemos escolher mais

duas pessoas para a comisso, homens ou mulheres, entre as 6 pessoas

restantes, o que pode ser feito de C62 = 15. A resposta 10 15 = 150.

2
Problemas de Combinatria Lista II Unidade 15

9. Quantas diagonais possui:

a) um octaedro regular?

b) um icosaedro regular?

c) um dodecaedro regular?

d) um cubo?

e) um prisma hexagonal regular?

10. Sejam Im = {1, 2, . . . , m} e In = {1, 2, . . . , n}, com m 6 n. Quantas

so as funes f : Im In estritamente crescentes?

11. Quantos so os nmeros naturais de 7 dgitos nos quais o dgito 4 gura

exatamente 3 vezes e o dgito 8 exatamente 2 vezes?

12. Quantos so os subconjuntos de {a1 , a2 , . . . , an }, com p elementos, nos

quais:

a) a1 gura;

b) a1 no gura;

c) a1 e a2 guram;

d) pelo menos um dos elementos a1 , a2 gura;

e) exatamente um dos elementos a1 e a2 gura.

13. De um baralho de pquer (7, 8, 9, 10, valete, dama, rei e s, cada um

desses grupos aparecendo em 4 naipes: copas, ouros, paus, espadas),

sacam-se simultaneamente 5 cartas.

a) Quantas so as extraes possveis?

Quantas so as extraes nas quais se forma:

b) um par (duas cartas em um mesmo grupo e as outras trs em trs

outros grupos diferentes)?

c) dois pares (duas cartas em um grupo, duas em outro grupo e uma

em um terceiro grupo)?

d) uma trinca (trs cartas em um grupo e as outras duas em dois outros

grupos diferentes)?

3
Unidade 15

e) um four (quatro cartas em um grupo e uma em outro grupo)?

f ) um full hand (trs cartas em um grupo e duas em outro grupo)?

g) uma sequncia (5 cartas de grupos consecutivos, no sendo todas

do mesmo naipe)?

h) um ush (5 cartas do mesmo naipe, no sendo elas de 5 grupos

consecutivos)?

i) um straight ush (5 cartas de grupos consecutivos, todas do

mesmo naipe)?

j ) um royal straight ush (10, valete, dama, rei e s de um mesmo

naipe)?

14. O conjunto A possui p elementos e o conjunto B possui n elementos.

Determine o nmero de funes f :AB sobrejetoras para:

a) p = n; b) p = n + 1; c) p = n + 2.

15. Considere um conjunto C de 20 pontos do espao que tem um subcon-

junto C1 formado por 8 pontos coplanares. Sabe-se que toda vez que 4

pontos de C so coplanares, ento eles so pontos de C1 . Quantos so

os planos que contm pelo menos trs pontos de C?

4
MA12 - Unidade 15
Revisao de Combinatoria

Paulo Cezar Pinto Carvalho

PROFMAT - SBM

27 de Abril de 2014
Enunciados parecidos, solucoes diferentes

Considere os conjuntos A = {1, 2, . . . , m} e B = {1, 2, . . . , n}.


Quantas sao:
a) as funcoes de A em B?
b) as funcoes injetivas de A em B?
c) as funcoes bijetivas de A em B?
d) as funcoes sobrejetivas de A em B?
e) as funcoes crescentes de A em B?
f) as funcoes nao decrescentes de A em B?

PROFMAT - SBM MA12 - Unidade 15, Revisao de Combinatoria slide 2/8


Quantas sao as funcoes de A em B?

Para formar uma funcao de A em B, e preciso escolher o


elemento de B associado a cada elemento de A.
Como nao ha restricoes, ha n possibilidades para cada uma
das m escolhas.
O numero total de possibilidades e n n . . . n = nm .

PROFMAT - SBM MA12 - Unidade 15, Revisao de Combinatoria slide 3/8


Quantas sao as funcoes injetivas de A em B?

Para que exista funcao injetiva de A em B, deve-se ter n m.


Para escolher o valor de f (1), ha n possibilidades
Para o valor de f (2), ha n 1 possibilidades (deve ser 6= f (1))
Para o valor de f (3), ha n 3 possibilidades
...
Para f (m) restam n (m 1) = n m + 1 possibilidades.
O numero total de possibilidades e n (n 1) . . . (n m + 1)

PROFMAT - SBM MA12 - Unidade 15, Revisao de Combinatoria slide 4/8


Quantas sao as funcoes bijetivas de A em B?

Para que exista funcao bijetiva de A em B, deve-se ter m = n.


Neste caso, uma funcao de A em B e bijetiva se e somente se
e injetiva (caso anterior).
O numero de funcoes bijetivas e n (n 1) . . . 1 = n! (ou
seja, e igual ao numero de permutacoes dos elementos de A).

PROFMAT - SBM MA12 - Unidade 15, Revisao de Combinatoria slide 5/8


Quantas sao as funcoes sobrejetivas de A em B?

Para que exista funcao sobrejetiva de A em B, deve-se ter


n m.
Surpreendentemente, o problema geral de contar o numero de
funcoes sobrejetivas de A em B e muito mais complicado que
os anteriores.
Caso particular: m = n + 1
O numero de funcoes sobrejetivas e nCn2 (n 1)!

PROFMAT - SBM MA12 - Unidade 15, Revisao de Combinatoria slide 6/8


Quantas sao as funcoes estritamente crescentes de
A em B?

Para que exista uma funcao estritamente crescente de A em


B, deve-se ter n m.
Basta escolher os m elementos da imagem, ja que uma vez
escolhidos estes elementos, f (1) deve ser o menor deles, f (2)
o segundo menor, e assim por diante.
O numero de funcoes crescentes e igual a Cnm .

PROFMAT - SBM MA12 - Unidade 15, Revisao de Combinatoria slide 7/8


Quantas sao as funcoes nao decrescentes de A em
B?

Para definir uma funcao nao decrescente de A em B, basta


determinar quantas vezes cada elemento de B e usado como
imagem; no total, este numero de usos deve ser igual a m, o
numero de elementos de A.
O numero de funcoes nao decrescentes e igual ao numero de
solucoes inteiras nao negativas de x1 + x2 + . . . + xn = m, onde
xi representa o numero de vezes que i e usado como imagem.
Portanto, e igual a CRnm = Cm+n1
m .

PROFMAT - SBM MA12 - Unidade 15, Revisao de Combinatoria slide 8/8


Lista de Exerccios
Unidade 15

1. Em um corredor ha 900 armarios, numerados de 1 a 900, inicialmente


todos fechados. 900 pessoas, numeradas de 1 a 900, atravessam o cor-
redor. A pessoa de numero k reverte o estado de todos os armarios
cujos numeros sao multiplos de k. Por exemplo, a pessoa de numero
4 mexe nos armarios de numeros 4, 8, 12, . . . , abrindo os que encontra
fechados e fechando os que encontra abertos. Ao final, quais armarios
ficarao abertos?

2. Dispomos de 5 cores distintas. De quantos modos podemos colorir os


quatro quadrantes de um crculo, cada quadrante com uma so cor, se
quadrantes cuja fronteira e uma linha nao podem receber a mesma cor?

3. De quantos modos podemos formar uma palavra de 5 letras de um


alfabeto de 26 letras, se a letra A deve figurar na palavra mas nao
pode ser a primeira letra da palavra? E se a palavra devesse ter letras
distintas?

4. As placas dos veculos sao formadas por tres letras (de um alfabeto de
26) seguidas por 4 algarismos. Quantas placas poderao ser formadas?

5. Um vagao do metro tem 10 bancos individuais, sendo 5 de frente e 5


de costas. De 10 passageiros, 4 preferem sentar de frente, 3 preferem
sentar de costas e os demais nao tem preferencia. De quantos modos
eles podem se sentar, respeitadas as preferencias?

6. De quantos modos e possvel colocar r rapazes e m mocas em fila de


modo que as mocas permanecam juntas?

7. Quantos dados diferentes e possvel formar gravando numeros de 1 a 6


sobre as faces de um cubo?

a) Suponha uma face de cada cor.


b) Suponha faces iguais.

1
c) Suponha que as faces sao iguais e que a soma dos pontos de faces
opostas deva ser igual a 7.

8. Resolva o problema anterior, no caso b), para os outros 4 poliedros


regulares.
n
X
9. Determine n para que k! seja um quadrado perfeito.
k=1

10. Quantos sao os anagramas da palavra ESTRELADA?

11. O conjunto A possui n elementos. Quantos sao os seus subconjuntos


com p elementos?

12. Uma faculdade realiza seu vestibular em dois dias de provas, com 4
materias em cada dia. Este ano a divisao foi: Matematica, Portugues,
Biologia e Ingles no primeiro dia e Geografia, Historia, Fsica e Qumica
no segundo dia. De quantos modos pode ser feito o calendario de
provas?

13. Qual e o erro na solucao do problema abaixo?


Com 5 homens e 4 mulheres, quantas comissoes de 5 pessoas, com pelo
menos 3 homens, podem ser formadas?

Solucao: Primeiramente vamos escolher 3 homens para a comissao, o


que pode ser feito de C53 = 10 modos. Agora devemos escolher mais
duas pessoas para a comissao, homens ou mulheres, entre as 6 pessoas
restantes, o que pode ser feito de C62 = 15. A resposta e 1015 = 150.

14. Quantas diagonais possui:

a) um octaedro regular?
b) um icosaedro regular?
c) um dodecaedro regular?
d) um cubo?
e) um prisma hexagonal regular?

15. Sejam Im = {1, 2, . . . , m} e In = {1, 2, . . . , n}, com m 6 n. Quantas


sao as funcoes f : Im In estritamente crescentes?

2
16. Quantos sao os numeros naturais de 7 dgitos nos quais o dgito 4 figura
exatamente 3 vezes e o dgito 8 exatamente 2 vezes?

17. Quantos sao os subconjuntos de {a1 , a2 , . . . , an }, com p elementos, nos


quais:

a) a1 figura;
b) a1 nao figura;
c) a1 e a2 figuram;
d) pelo menos um dos elementos a1 , a2 figura;
e) exatamente um dos elementos a1 e a2 figura.

18. De um baralho de poquer (7, 8, 9, 10, valete, dama, rei e as, cada um
desses grupos aparecendo em 4 naipes: copas, ouros, paus, espadas),
sacam-se simultaneamente 5 cartas.

a) Quantas sao as extracoes possveis?


Quantas sao as extracoes nas quais se forma:
b) um par (duas cartas em um mesmo grupo e as outras tres em tres
outros grupos diferentes)?
c) dois pares (duas cartas em um grupo, duas em outro grupo e uma
em um terceiro grupo)?
d) uma trinca (tres cartas em um grupo e as outras duas em dois
outros grupos diferentes)?
e) um four (quatro cartas em um grupo e uma em outro grupo)?
f) um full hand (tres cartas em um grupo e duas em outro grupo)?
g) uma sequencia (5 cartas de grupos consecutivos, nao sendo todas
do mesmo naipe)?
h) um flush (5 cartas do mesmo naipe, nao sendo elas de 5 grupos
consecutivos)?
i) um straight flush (5 cartas de grupos consecutivos, todas do
mesmo naipe)?
j ) um royal straight flush (10, valete, dama, rei e as de um mesmo
naipe)?

3
19. O conjunto A possui p elementos e o conjunto B possui n elementos.
Determine o numero de funcoes f : A B sobrejetoras para:
a) p = n; b) p = n + 1; c) p = n + 2.

20. Considere um conjunto C de 20 pontos do espaco que tem um subcon-


junto C1 formado por 8 pontos coplanares. Sabe-se que toda vez que 4
pontos de C sao coplanares, entao eles sao pontos de C1 . Quantos sao
os planos que contem pelo menos tres pontos de C?

4
Solucoes da Lista de Exerccios
Unidade 15

1. Um armario ficara aberto se ele for mexido um numero mpar de vezes.


Por outro lado, o armario de ordem k e mexido pelas pessoas cujos
numeros sao divisores de k. Logo, estarao abertos os armarios cujos
numeros possuem um numero mpar de divisores. Isto ocorre com os
numero cujos expoentes sao todos pares na decomposicao em fatores
primos, ou seja, sao quadrados perfeitos. Assim, permaneceram abertos
os armarios cujos numeros sao quadrados perfeitos, ou seja, os numeros
12 , 22 , . . . , 302 .

2. Separemos o caso em que o primeiro e o terceiro quadrantes tem cores


iguais do caso em que eles tem cores diferentes.
No caso de cores iguais, ha 5 modos de escolher a cor unica para o pri-
meiro e o terceiro quadrante, 4 modos de escolher a cor para o segundo
quadrante e 4 modos de escolher a cor para o quarto quadrante. Ha,
portanto, 5 4 4 = 80 modos de colorir o mapa usando cores iguais
no primeiro e no terceiro quadrantes.
No caso de cores diferentes, ha 5 modos de escolher a cor para o primeiro
quadrante, 4 modos de escolher a cor para o terceiro quadrante, 3 modos
de escolher a cor para o segundo quadrante e 3 modos de escolher a cor
para o quarto quadrante. Ha 5 4 3 3 = 180 modos de colorir o
mapa usando cores iguais no primeiro e no terceiro quadrantes.
No total, temos, portanto, 80 + 180 = 260 modos de colorir a figura.

3. (a) Ha 265 = 11.881.376 palavras de 5 letras. Delas, devemos subtrair


as palavras que comecam por A, 1 264 = 456.976, e aquelas nas
quais a letra A nao figura, 255 = 9.765.625.
A resposta e 11.881.376 456.976 9.765.625 = 1.658.775.
(b) O numero total de palavras de 5 letras distintas e 26252423
22 = 7.893.600. Delas devemos subtrair as palavras que comecam
por A, 1 25 24 23 22 = 303.600 e aquelas nas quais a letra
A nao figura, 25 24 23 22 21 = 6.375.600.

1
A resposta e 7.893.600 303.600 6.375.600 = 1.214.400.

4. Cada letra pode ser escolhida de 26 modos, enquanto cada algarismo


pode ser escolhido de 10 modos. Logo, o numero total de placas e
263 104 = 175.760.000.

5. O numero de modos de acomodar os passageiros que pretendem sentar


de frente e 5 4 3 2 = 120; o numero de modos de acomodar os
passageiros que pretendem sentar de costas e 5 4 3 = 60; o numero
de modos de acomodar os demais passageiros e 3 2 1 = 6.
A resposta e 120 60 6 = 43.200.

6. Devemos inicialmente escolher a ordem em que as mocas ficarao juntas,


o que pode ser feito de m! maneiras. Em seguida, devemos arrumar
em fila r + 1 objetos, os r rapazes e o bloco das mocas, o que pode ser
feito de (r + 1)! modos.
A resposta e m!(r + 1)!.

7. (a) A face a receber o numero 1 pode ser escolhida de 6 modos, a do


numero 2 de 5 modos, e assim por diante. O numero de possibili-
dades e 6.5.4.3.2.1 = 720.
(b) Inicialmente, fazemos a conta que as faces tem cores diferentes.
Contamos, pelo problema anterior, 720 dados. Como as faces
sao indistinguveis, o mesmo dado foi contado varias vezes. Por
exemplo, pense em um dado que tenha o 6 na face de baixo (face
preta) e o 1 na face de cima (face branca). Ele e, certamente,
diferente de um dado que tenha o 1 na face de baixo (face preta) e o
6 na face de cima (face branca). Mas sendo as faces indistinguveis,
o dado que tem 6 na face de baixo e o 1 na face de cima e igual
ao dado que tem o 1 na face de baixo e o 6 na face de cima; este
e, simplesmente, aquele de cabeca para baixo. Esse mesmo dado
aparece outra vez com o 1 na face da frente e o 6 na face de tras,
com o 1 na face da esquerda e o 6 na face da direita, etc. Em suma,
o mesmo dado foi contado tantas vezes quantas sao as posicoes de
coloca-lo.
O numero de posicoes de colocar um cubo e 6 4 = 24, pois ha
6 modos de escolher a face de baixo e 4 de escolher, nessa face, o
lado que fica de frente.

2
720
A resposta e = 30.
24
(c) Um dado com faces de cores diferentes pode, agora, ser numerado
de apenas 6.4.2 = 48 modos, ja que temos 6 faces a escolher para o
numero 1 (isto determina a face do numero 6), a para o numero 2
(o que determina a face do 5) e 2 para o numero 3 (que determina
a do 4). Mas como as faces sao iguais, cada dado e contado, como
no tem anterior, 24 vezes. Logo ha apenas 48/24 = 2 dados
distintos.

8. (a) O numero de posicoes para um tetraedro e 4 3 = 12, pois ha 4


modos de escolher a face de apoio e 4 de escolher, nessa face, o
lado que fica de frente.
4!
A resposta e = 2.
12
(b) O numero de posicoes para um octaedro regular e 6 4 = 24, pois
ha e modos de escolher o vertice de apoio e 4 de escolher, dentre
as arestas que incidem nesse vertice, a que fica de frente.
8!
A resposta e = 1.680.
24
(c) O numero de posicoes para um dodecaedro regular e 12 5 = 60,
pois ha 12 modos de escolher a face de apoio e 5 de escolher, nessa
face, o lado que fica de frente.
12!
A resposta e = 7.938.360.
60
(d) O numero de posicoes para um icosaedro regular e 20 3 = 60,
pois ha 20 modos de escolher a face de apoio e 3 de escolher, nessa
face, o lado que fica de frente.
20!
A resposta e = 40.548.366.802.944.000 = 4.1016 .
60
9. Temos 1! = 1, que e um quadrado perfeito, 1! + 2! = 1 + 2 = 3, que
nao e quadrado perfeito, 1! + 2! + 3! = 1 + 2 + 6 = 9, que novamente e
quadrado perfeito, 1! + 2! + 3! + 4! = 1 + 2 + 6 + 24 = 33, que nao e
quadrado perfeito. Todos os fatorais seguintes, a partir de 5! terminam
com
Pn zero, ja que sao multiplos de 5 e 2. Logo, todas as somas da forma
1 k! para n 5 terminam com o algarismo 3 e nao sao, portanto,
quadrados perfeitos. As unicas solucoes sao n = 1 e n = 3.

3
10. Em ESTRELADA as letras A e E aparecem 2 vezes cada e as letras S,
T, R, L e D aparecem uma vez cada uma, havendo, portanto, 9 letras
na palavra.
Para formar um anagrama, devemos escolher 2 das 9 posicoes para
colocar as letras A, o que pode ser feito de C92 modos, 2 das 7 posicoes
restantes para colocar as letras E, o que pode ser feito de C72 modos, e
arrumar as letras S, T, R, L e D nas 5 posicoes restantes, o que pode
ser feito de 5! modos. A resposta e C92 .C72 .5! = 90.720.

11. Formar uma solucao com p elementos significa escolher p dos n elemen-
tos. A resposta e Cnp .

12. Basta escolher as provas do primeiro dia, o que pode ser feito de C84 = 70
modos.

13. O processo de contagem apresentado conta determinadas comissoes


mais de uma vez. Isto ocorre porque um homem que participe da
comissao pode ser inserido de dois modos diferentes: como um dos 3
homens escolhidos dos inicialmente, ou como uma das duas pessoas
escolhidas posteriormente. O pior e que nao e possvel corrigir a con-
tagem dividendo pelo numero de vezes que cada comissao e contada:
as comissoes com 3 homens sao contadas apenas uma vez, as que tem
4 homens sao contadas 4 vezes, enquanto a que contem 5 homens e
contada 10 vezes.
A solucao correta e dada no Exemplo 6, contando separadamente as
comissoes com 3, 4 e 5 homens: C53 .C42 + C54 .C41 + C55 = 81.

14. Os segmentos que unem dos vertices de um poliedro ou sao arestas ou


sao diagonais de faces ou diagonais do poliedro.

(a) O octaedro regular e um poliedro formado por 8 faces triangulares


e que tem 6 vertices e 12 arestas. Ha C62 = 15 segmentos que unem
dois vertices do poliedro, pois 12 dos quais sao arestas e 0 dos quais
e diagonal de face.
A resposta e 15 12 0 = 3.
(b) O icisaedro regular e um poliedro formado por 20 faces triangu-
2
lares e que tem 12 vertices e 30 arestas. Ha C12 = 66 segmentos

4
que unem dois vertices do poliedro, 30 dos quais sao arestas e 0
dos quais e diagonal de face.
A resposta e 66 30 0 = 36.
(c) O dodecaedro regular e um poliedro formado por 12 faces pentago-
2
nais e que tem 20 vertices e 30 arestas. Ha C20 = 190 segmentos
que unem dois vertices do poliedro, 30 dos quais sao arestas e
5(5 3)
12 = 60 dos quais sao diagonais de faces.
2
A resposta e 190 30 60 = 100.
(d) O cubo e um poliedro formado por 6 faces quadradas e que tem 8
vertices e 12 arestas. Ha C82 = 28 segmentos que unem os vertices
4(4 3)
do poliedro, 12 dos quais sao arestas e 6 12 dos quais sao
2
diagonais de faces.
A resposta e 28 12 12 = 4.
(e) O prisma hexagonal e um poliedro formado por 6 faces quadran-
gulares e 2 faces hexagonais e que tem 12 vertices e 18 arestas. Ha
2
C12 = 66 segmentos que unem os vertices do poliedro, 18 dos quais
4(4 3) 6(6 3)
sao arestas e 6 +2 = 30 dois quais sao diagonais
2 2
de faces.
A resposta e 66 18 30 = 18.
15. Uma funcao estritamente crescente e necessariamente injetiva (se f (a) =
f (b), nao pode ser a < b, pois, neste caso, f (a) < f (b), o que e absurdo;
do mesmo modo, nao pode ser a > b, pois, neste caso, f (a) > f (b),
o que e absurdo; logo, a = b), Logo, seu conjunto de valores tera
exatamente m elementos. Para construir uma tal funcao, devemos, ini-
cialmente selecionar o conjunto de valores, o que pode ser feito de Cnm
modos.
Selecionando o conjunto de valores, a funcao esta determinada porque
f (1) deve ser igual ao menor elemento do conjunto de valores, f (2)
deve ser igual ao segundo menor elemento do conjunto de valores, etc.
A resposta e, portanto, Cnm .
16. Vamos esquecer que a primeira casa do numero nao pode ser igual a
zero. Isso fara com que contemos a mais e, depois, descontaremos o
que foi contado indevidamente.

5
Ha C73 modos de escolher as casas que serao ocupadas pelo digito 4;
depois disso, ha C42 modos de selecionar as casas que serao ocupadas
pelo digito 8; finalmente, as duas casas restantes podem ser preenchidas
de 8 8 modos (nao podemos usar nessas casas os dgitos 4 e 8).
A resposta seria C73 C42 8 8 = 35 6 64 = 13.440.
Devemos subtrair os numeros comecados por zero. Se o numero comeca
por 0, ha C63 modos de escolher as casas que serao ocupadas pelo digito
4; depois disso, ha C32 modos de selecionar as casas que serao ocupadas
pelo digito 8; finalmente, a casa restante pode ser preenchida de 8
modos (nao podemos usar nessa casa os dgitos 4 e 8). Ha C63 C32 8 =
20 3 8 = 480 numeros comecados por 0.
A resposta e 13.440 480 = 12.960

17. (a) Para formar o subconjunto devemos escolher os p 1 outros ele-


mentos do subconjunto dentre os n 1 outros elemento do con-
junto.
p1
A resposta e Cn1 .
(b) Para formar o subconjunto devemos escolher os p elementos do
subconjunto dentre os n 1 outros elementos do conjunto.
p
A resposta e Cn1 .
Outra solucao:
p1
Ha Cnp p-subconjuntos e o elemento a1 figura em Cn1 deles. Logo,
p p1
ha Cn Cn1 subconjuntos nos quais o elemento a1 nao figura.
p1
A resposta e Cnp Cn1 .
Observacao: As duas solucoes apresentadas mostram que Cnp
p1 p
Cn1 = Cn1 . Essa e a famosa Relacao de Stifel.
(c) Para formar o subconjunto devemos escolher os p 2 outros ele-
mentos do subconjunto dentre os n 2 outros elementos do con-
junto.
p2
A resposta e Cn2 .
(d) O total de p-subconjuntos e Cnp . Para formar um subconjunto em
que nem a1 nem a2 figurem devemos escolher os p elementos do
subconjuntos dentre os n 2 outros elementos do conjunto. Ha,
p
portanto, Cn2 subconjuntos nos quais nem a1 nem a2 figuram.

6
Logo, o numero de subconjuntos nos quais pelo menos um desses
p
dois elementos figura e Cnp Cn2 .
p1
(e) Como visto na solucao anterior, a resposta e 2Cn2 .
2
18. (a) Como ha 32 cartas, a resposta e C32 = 201.376.
(b) Ha 8 modos de escolher o grupo do par propriamente dito (por
exemplo, valete), C42 modos de escolher os naipes das duas cartas
do par (por exemplo, copas e paus), C73 modos de escolher os
grupos das outras tres cartas (por exemplo, 10, 8 e rei) e 444 =
43 modos de escolher os naipes dessas tres cartas.
A resposta e 8 C42 C73 43 = 107.520.
(c) Ha C82 modos de escolher os grupos das cartas que formarao os dois
pares, (C42 )2 modos de escolher seus naipes, 6 modos de escolher
o grupo da outra carta e 4 modos de escolher seu naipe.
A resposta e C82 (C42 )2 6 4 = 24.192.
Observacao: Um erro muito comum e o exposto a seguir.
Ha 8 modos de escolher o grupo do primeiro par, C42 modos de
escolher os naipes do primeiro par, 7 modos de escolher o grupo
do segundo par, C42 modos de escolher os naipes do segundo par,
6 modos de escolher o grupo da outra carta e 4 modos de escolher
seu naipe. A resposta ERRADA seria 8 C42 7 C42 6
4 = 48.384. A explicacao do ERRO e simples: Ao fazermos a
inexistente distincao entre primeiro par e segundo par, contamos
pares de valetes e reis como diferentes de pares de reis e valetes.
A resposta ERRADA pode ser corrigida dividindo-a por 2.
(d) Ha 8 modos de escolher o grupo da trinca, C43 modos de escolher
os naipes das cartas da trinca, C72 modos de escolher os grupos
das outras duas cartas e 4 4 = 42 modos de escolher os naipes
dessas duas cartas.
A resposta e 8 C43 C72 42 = 10.752
(e) Ha 8 modos de escolher o grupo do four, 1 modo de escolher os
naipes das quatro cartas do four, 7 modos de escolher o grupo
da outra carta e 4 modos de escolher o naipe dessa carta.
A resposta e 8 1 7 4 = 224.

7
(f) Ha 8 modos de escolher o grupo da trinca, C43 modos de escolher
os naipes das cartas da trinca, 7 modos de escolher o grupo do
par e C42 modos de escolher os naipes das cartas do par.
A resposta e 8 C43 7 C42 = 1.344
(g) Ha apenas 4 tipos de sequencias: 7. 8. 9. 10, valete, dama, rei;
10, valete, dama, rei, as. Escolhido o tipo da sequencia, haveria
4 4 4 4 4 4 = 45 modos de escolher os naipes das cartas
das sequencias, mas 4 desses modos nao sao permitidos: todas de
ouros, todas de paus, todas de copas e todas de espadas.
A resposta e 4 [45 4] = 4.080.
(h) Os grupos das cartas podem ser escolhidos de C85 4 modos e o
naipe unico, de 4 modos.
A resposta e (C85 4) 4 = 208.
(i) Ha 4 modos de escolher os grupos de cartas e 4 modos de escolher
o naipe unico.
A resposta e 4 4 = 16.
(j) Ha 4 modos de escolher o naipe unico. A resposta e 4.
19. (a) Neste caso f e bijetiva e, se #A = #B = n, o numero de funcoes
f : A B bijetivas e n!, como foi mostrado no exerccio 4 da
secao 2.2.
(b) Neste caso dois elementos de A terao uma mesma imagem em B
e a correspondencia entre os demais n 1 elementos de A e os
demais n 1 elementos de B sera bijetiva.
Ha n+1

2
modos de escolher os dois elementos de A, n modos de
escolher a imagem deles em B e (n 1)! modos de construir uma
correspondencia bijetiva entre os elementos restantes.
n(n + 1)!
A resposta e n+1

2
n (n 1)! = .
2
(c) Neste caso temos as alternativas
i. Tres elementos de A tem a mesma imagem em B e a corres-
pondencia entre os demais n 1 elementos de A e os demais
n 1 de B e bijetiva.
Ha n+23
modos de escolher os tres elementos de A, n modos
de escolher a imagem deles em B e (n1)! modos de construir
uma correspondencia bijetiva entre os elementos restantes.

8
n+2
 n(n + 2)!
Ha 3
n (n 1)! = funcoes desse tipo.
6
ii. Ha dois pares de elementos de A com imagens identicas em
B e a correspondencia entre os demais n 2 elementos de A
e os demais n 2 elementos de B e bijetiva.
Ha 2 modos de escolher os dois elementos de B, n+2
n
n2
  
2
modos de escolher suas imagens inversas em A e (n2)! modos
de estabelecer a correspondencia entre os elementos restantes.
n(n 1)(n + 2)!
Ha n2 n+2 n2 (n 2)! =
  
2
funcoes
8
desse tipo.
A resposta e:

n(n + 2)! n(n 1)(n + 2)! n(3n + 1)(n + 2)!


+ = .
6 8 24
20. Chamemos de D o conjunto C C1 .
Ha quatro tipos de planos:

(a) determinados por tres pontos de D;


(b) determinados por dois pontos de D e um de C1 ;
(c) determinados por um ponto de D e dois de C1 ;
(d) determinados por tres pontos de C1 .
3 2
A resposta e C12 + C12 .8 + 12.C82 + 1 = 1.085

9
SOLUES
UNIDADE 15 MA12
1) Em um corredor h 900 armrios, numerados de 1 a 900, inicialmente todos
fechados. 900 pessoas, numeradas de 1 a 900, atravessam o corredor.A pessoa de
nmero k reverte o estado de todos os armrios cujos nmeros so mltiplos de k.
Por exemplo, a pessoa de nmero 4 mexe nos armrios 4, 12 , ..., 900 abrindo os que
encontra fechados e fechando os que encontra abertos. Ao final quais armrios
ficaro abertos?

Se fizermos com nove armrios vamos ter :


1 pessoa abre todos.
2 pessoa fecha somente os pares
3 pessoa fecha os armrios mltiplos de trs mpares e abre os pares
4 pessoa abre o quarto e o oitavo
5 fecha o quinto
6 fecha o seis
7 fecha o stimo
8 fecha o oitavo
9 abre o nono

Percebe-se que os armrios 1, 4 e 9 ficam abertos, (so nmeros quadrados perfeitos


que ficam abertos) Por que o nmero de pessoas que passa por eles mpar.

Agora verificamos quantos quadrados perfeitos temos entre 1 e 900.


1, 4, 16, 25, 36 , 49 ,64 , 81 , ...., 900
So 30 . Ento temos 30 armrios abertos

2)Dispomos de 5 cores distintas. De quantos modos podemos colorir os quatro


quadrantes de um crculo, cada quadrante com uma s cor, se quadrantes cuja
fronteira uma linha no puderem receber a mesma cor?

Resoluo:

[Aqui, consideremos o primeiro, segundo, terceiro e quarto quadrantes como no plano


cartesiano, ou seja, contando o primeiro como o dos pontos de coordenadas positivas
e girando no sentido anti-horrio.]

A princpio, para o primeiro quadrante temos 5 possibilidades, para o segundo


quadrante temos 4 possibilidades [no pode ser igual ao primeiro], para o terceiro
quadrante 4 possibilidades [no pode ser igual ao segundo]. E para o quarto
quadrante? Isso depende. Se o primeiro quadrante tiver a mesma cor que o terceiro,
temos que excluir uma possibilidade. J, se o terceiro quadrante tiver uma cor
diferente do primeiro, temos que excluir duas possibilidades. Temos que dividir ento
em dois casos:
O primeiro quadrante com a cor igual do terceiro:

Para o primeiro quadrante: 5 possibilidades.


Para o segundo quadrante: 4 possibilidades [diferente do primeiro].
Para o terceiro quadrante: 1 possibilidade [igual do primeiro].
Para o quarto quadrante: 4 possibilidades [pois como o primeiro igual ao terceiro,
temos que excluir apenas uma possibilidade].

Total neste caso: 5x4x1x4 = 80

O primeiro quadrante com a cor diferente do terceiro:

Para o primeiro quadrante: 5 possibilidades.


Para o segundo quadrante: 4 possibilidades [diferente do primeiro].
Para o terceiro quadrante: 3 possibilidades [diferente do segundo E DO PRIMEIRO].
Para o quarto quadrante: 3 possibilidades [diferente do primeiro e do terceiro].

Total neste caso; 5x4x3x3 = 180

Como ocorre um caso OU o outro, temos; 180 + 80 = 260.

R: 260

03) De quantos modos podemos formar uma palavra de 5 letras de um alfabeto de


26, se a letra A deve figurar na palavra mas no pode ser a primeira letra? E se a
palavra devesse ter letras distintas?

a)H 26x26x26x26x26 = 11881376 palavras de 5 letras. Delas devemos subtrair as


palavras que comeam por A, 1x26x26x26x26 = 456976, e aquelas nas quais a letra A
no figura, 25x25x25x25x25 = 9765625.

A resposta 11881376 - 456976 - 9765625 = 1658775.

b)H 4 posies para colocar a letra A; depois disso, as quatro casas vazias podem ser
preenchidas de 25,24,23 e 22 modos.

A resposta 4x25x24x23x22 = 1214400

R: 1658775 e 1214400

04) As placas de veculos so formadas por trs letras (de um alfabeto de 26)
seguidas por 4 algarismos. Quantas placas diferentes podem ser formadas?

Resoluo:

Temos que escolher 3 letras, cada uma pode ser escolhida de 26 maneiras. Temos que
escolher 4 nmeros, cada um pode ser escolhido de 10 maneiras. Logo:
26x26x26x10x10x10x10 = 175760000

R: 175760000

05) Um vago de metr em 10 bancos individuais, sendo 5 de frente e 5 de cosas. De


10 passageiros, 4 preferem sentar de frente, 3 preferem sentar de costas e os demais
no tm referncia. De quantos modos eles podem se sentar, respeitadas as
preferncias?

Resoluo:

O nmero de modos de acomodar os passageiros que pretendem sentar de frente


5x4x3x2 = 120; o nmero de modos de acomodar os passageiros que pretendem
sentar de costas 5x4x3 = 60; o nmero de modos de acomodar os demais
passageiros 3x2x1 = 6.

A resposta 120x60x6 = 43200

R: 43200

06 )De quantos modos possvel colocar r rapazes e m moas em fila de modo que as
moas permaneam juntas?

Resoluo: Devemos considerar as m moas como se fosse um dos elementos que


devemos permutar. Da, ao todo, temos r rapazes e 1 elemento a mais para permutar,
ou seja, devemos permutar (r+1) elementos. Como as moas entre si podem se
permutar de m! formas, temos que multiplicar o resultado anterior por m!. Da temos,
pelo princpio multiplicativo:

m!x(r+1)!

R: m!(r+1)!

07) Quantos dados diferentes possvel formar gravando nmeros de 1 a 6 sobre as


faces de um cubo?

a)Suponha uma face de cada cor. [720]

Resoluo:

Devemos colocar 6 nmeros em 6 lugares. A resposta P6 = 6! = 720

b)Suponha as faces iguais. [30]

Resoluo:
Faamos de conta que as faces so diferentes. Contamos pelo problema anterior 720
cubos.
Como as faces so indistinguveis, o mesmo cubo foi contado vrias vezes. Por
exemplo, pense em um cubo que tenha o 6 na face de baixo (face preta) e o 1 na face
de cima (face branca). Ele , certamente, diferente de um cubo que tenha o 1 na face
de baixo (face preta) e o 6 na face de cima (face branca).
Mas sendo as faces indistinguveis, o cubo que tem o 6 na face de baixo e o 1 na face
de cima, igual ao cubo que tem o 1 na face de baixo e o 6 na face de cima. Este ,
simplesmente, aquele de cabea para baixo. Esse mesmo dado aparece outra fez com
o 1 na face da frente e o 6 na face de trs, com o 1 na face da esquerda e o 6 na face
da direita, etc. Em suma, o mesmo dado foi contado tantas vezes quantas so as
posies de coloc-lo.
O nmero de posies de colocar um cubo 6x4 = 24, pois h 6 modos de escolher a
face de baixo e 4 de escolher, nessa face, o lado que fica de frente.

A resposta : 720/24 = 30

c)Suponha que as faces so iguais e que a soma dos pontos de faces opostas deva ser
igual a 7. [2]

Resoluo:

Precisamos notar que escolhendo o nmero de uma face, automaticamente estamos


escolhendo o nmero da face oposta, ento na prtica, temos que escolher trs
nmeros para colocar em trs faces adjacentes do cubo [os nmeros podem ser 6,5,4
ou 1,2,3]. Isto pode ser feito de 2 maneiras, pois para o primeiro lugar temos 3
possibilidades, para o segundo temos 2 e para o terceiro, temos 1, mas girando o dado
em torno do vrtice comum s trs faces [como se fosse uma permutao circular],
temos que dividir este resultado por 3. Da 6/3 = 2, que a resposta que procuramos.

10) Quantos so os anagramas da palavra ESTRELADA?

Resoluo: Considerando as letras distintas entre si, teramos 8!, porm, como as
letras E e as letras A se repetem duas vezes cada uma, devemos dividir por 2! para
cada uma dessas letras, ficamos ento com:

9!/(2!x2!) = 9!/4 = 90720

R: 90720

11) O conjunto A possui n elementos. Quantos so os seus subconjuntos com p


elementos?

Resoluo:
Para formar o subconjunto, temos que escolher p elementos dentre n elementos [
claro que estamos supondo p menor ou igual n]. Como dentro de um conjunto a
ordem dos elementos no importa, temos ento uma combinao simples de n
elementos tomados p a p.

R: C(n,p)

12) Uma faculdade realiza seu vestibular em 2 dias de provas. Este ano a diviso foi:
Matemtica, Portugus, Biologia e Ingls no primeiro dia e Geografia, Histria, Fsica
e Qumica no segundo dia. De quantos modos pode ser feito calendrio de provas?

Resoluo:

Para o primeiro dia temos: 8x7x6x5 = 1680


Porm, a ordem das matrias no importa, da temos que dividir este resultado por 4!.
Ficamos ento com 1680/4! = 1680/24 = 70

Para o segundo dia, sobram apenas as matrias que no foram escolhidas no dia
anterior. Temos ento apenas uma possibilidade. Pelo princpio multiplicativo a
resposta 70x1 = 70.

R: 70

14) Quantas diagonais possui:

a)Um octaedro regular? [3]

Resoluo:

Os segmentos que unem dois vrtices de um poliedro ou so arestas ou so diagonais


das faces ou so diagonais do poliedro.

O octaedro regular um poliedro formado por 8 faces triangulares e que tem 6


vrtices e 12 arestas. h C(6,2) = 15 segmentos que unem dois vrtices do poliedro, 12
dos quais so arestas e 0 dos quais so diagonal da face. A resposta 15-12-0 = 3

b)um icosaedro regular? [36]

Resoluo:

O icosaedro regular um poliedro formado por 20 faces triangulares e que tem 12


vrtices e 30 arestas. H C(12,2) = 66 segmentos que unem dois vrtices do poliedro,
30 dos quais so arestas e 0 dos quais diagonal da face. A resposta 66-30-0 = 36.

c)um dodecaedro regular? [100]


Resoluo:

O dodecaedro regular um poliedro formado por 12 faces pentagonais e que tem 20


vrtices e 30 arestas. H C(20,2) = 190 segmentos que unem dois vrtices do poliedro,
30 dos quais so arestas e 60 dos quais so diagonais de faces. A resposta 190-30-60
= 100.

d)um cubo? [4]

Resoluo:

O cubo um poliedro formado por 6 faces quadradas e que tem 8 vrtices e 12


arestas. H C(8,2) = 28 segmentos que unem dois vrtices do poliedro, 12 dos quais
so arestas e 12 dos quais so diagonais das faces. A resposta 28-12-12 = 4.

e)um prisma hexagonal regular? [18]

O prisma hexagonal um poliedro formado por 6 faces quadrangulares e 2 faces


hexagonais e que tem 12 vrtices e 18 arestas. H c(12,2) = 66 segmentos que unem
dois vrtices do poliedro, 18 dos quais so arestas e 30 dos quais so diagonais de
faces. A resposta 66-18-30 = 18.

15) Sejam Im = {1,2,...,m} e In = {1,2,...,n}, com m menor ou igual n. Quantas so as


funes f: Im --> In estritamente crescentes?

Resoluo:

Uma funo estritamente crescente necessariamente injetiva. Logo, seu conjunto de


valores ter exatamente m elementos. Para construir tal funo, devemos
inicialmente, selecionar o conjunto de valores, o que pode ser feito de C(n,m) modos.
Selecionado o conjunto de valores, a funo est determinada, porque f(1) deve ser
igual ao menor elemento do conjunto de valores, f(2) deve ser igual ao segundo menor
elemento do conjunto de valores, etc. A resposta C(n,m).

16)Quantos so os nmeros naturais de 7 dgitos nos quais o dgito 4 figura


exatamente 3 vezes e o dgito 8 exatamente 2 vezes?

Resoluo:

Caso 1) Nmeros que comeam por 4

H um modo de preencher a primeira casa; depois disso, h C(6,2) modos de escolher


as outras duas casas do nmero que tambm sero preenchidas com o algarismo 4;
deposi disso, h C(4,2) modos de escolher as duas casas que sero ocupadas pelo
algarismo 8; finalmente, as duas casas restantes podem ser preenchidas de 8x8 modos
(no podemos usar nessas casas os dgitos 4 e 8). H 1xC(6,2)xC(4,2)x8x8 = 5760
nmeros.

Caso 2) Nmeros que comeam por 8

H um modo de preencher a primeira casa; depois disso, h 6 modos de escolher a


outra casa do nmero que tambm ser preenchida com o algarismo 8; depois disso,
h C(5,3) modos de escolher as trs casas que sero ocupadas pelo algarismo 4;
finalmente, as duas casas restantes podem ser preenchidas de 8x8 modos (no
podemos usar nessas casas os dgitos 4 e 8) H 1x6xC(5,3)x8x8 = 3840 nmeros.

Caso 3) Nmeros que no comeam por 4 e 8

H 7 modos de preencher a primeira casa (no podemos usar nem 4, nem 8, nem 0);
depois disso, h C(6,3) modos de escolher as trs casas do nmero que sero
preenchidas com o algarismo 4; depois disso h C(3,2) modos de escolher as duas
casas que sero ocupadas pelo algarismo 8; finalmente, a casa restante ode ser
preenchida de 8 modos (no podemos usar nessa casa os dgitos 4 e 8). H
7xC(6,3)xC(3,2)x8 = 3360 nmeros.

A resposta 5760+3840+3360 = 12960

R: 12960

17) Quantos so os subconjuntos de {a1,a2,...,an} com p elementos, nos quais:

a)a1 figura; [C(n-1,p-1)]

Resoluo: Se j sabemos que no subconjunto est o elemento a1, ento agora temos
que escolher p-1 elementos [pois um dos p elementos j foi escolhido, no caso o a1]
dentre n-1 elementos [pois no podemos escolher novamente o a1]. Como a ordem da
escolha dos elementos no importa, temos C(n-1,p-1).

b)a1 no figura; [C(n-1,p)]

Resoluo: Se o elemento a1 no figura, temos que escolher p elementos dentre n-1


elementos, pois dos n elementos no podemos escolher um, no caso o a1. Como nessa
escolha a ordem no importa, temos ento C(n-1,p).

c)a1 e a2 figuram; [C(n-2,p-2)]

Resoluo: Se o a1 e o a2 j figuram no subconjunto, nos resta n-2 elementos para


escolher [pois no podemos escolher a1 ou a2 novamente]. E, como o subconjunto j
tem dois elementos definidos, temos que escolher p-2 elementos, no importando a
ordem. Temos ento um problema de combinao simples expresso por: C(n-2,n-2).
d)pelo menos um dos elementos a1,a2 figura; [2xC(n-1,p-1) + C(n-2,p-2) = C(n,p) -
C(n-2,p)]

Resoluo:
Primeira maneira: O total de p subconjuntos C(n,p). Para formar um subconjunto em
que nem a1 nem a2 figurem devemos escolher os p elementos do subconjunto dentre
os n-2 elementos do conjunto. H, portanto, C(n-2,p) subconjuntos nos quais nem a1
nem a2 figuram. Logo, o nmero de subconjuntos nos quais pelo menos um desses
dois elementos figura C(n,p) C(n-2,p).

Segunda maneira: H C(n-1,p-1) p-subconjuntos nos quais o elemento a1 figura e h


C(n-1,p-1) subconjuntos nos quais o elemento a2 figura. H, tambm, C(n-2,p-2)
subconjuntos nos quais os elementos a1 e a2 figuram ambos. Ao somarmos C(n-1,p-
1)+C(n-1,p-1) = 2C(n-1,p-1) obtemos o nmero de subconjuntos nos quais pelo menos
um dos elementos a1 e a2 figuram, mas contamos duas vezes aqueles em que a1 e a2
figuram ambos. A resposta portanto:
2C(n-1,p-1) C(n-2,p-2).

e)exatamente um dos elemenos a1 e a2 figura. [2x(C(n-2,p-1))]

Resoluo: Temos que calcular a quantidade de subconjuntos com p elementos nos


quais a1 figura e a2 NO figura, e tambm a quantidade de subconjuntos com p
elementos nos quais a2 figura e a1 NO figura. Por fim podemos som-las.
Se a1 figura no subconjunto, ento 1 dos p elementos j foi escolhido, temos ento
que escolher p-1 elementos dentre n-2 elementos, j que a1 no pode ser escolhido
novamente e a2 no pode ser escolhido. Fica ento C(n-2,p-1).
Analogamente, com a2 no subconjunto e a1 fora, temos C(n-2,p-1).
A resposta C(n-2,p-1) + C(n-2,p-1) = 2C(n-2,p-1).

18) De um baralho de pquer (7, 8, 9, 10, valete, dama, rei e s, cada um desses
grupos aparecendo em 4 naipes: copas, ouros, paus, espadas), sacam-se
simultaneamente 5 cartas.

a)Quantas so as extraes possveis? [201376]

Resoluo: Como h 32, a resposta C(32,5) = 201 376

Quantas so as extraes nas quais se forma:


b)um par (duas cartas em um mesmo grupo e as outras trs em trs outros grupos
diferentes)? [107520]

Resoluo: H 8 modos de escolher o grupo do par propriamente dito (por exemplo,


valete), C(4,2) modos de escolher os naipes das duas cartas do par (por exemplo, copas
e paus), C(7,3) modos de escolher os grupas das outras trs cartas (por exemplo, 10,8 e
rei) e 4x4x4 = 4 modos de escolher os naipes dessas trs cartas. A resposta
8xC(4,2)xC(7,3)x4 = 107520

c)dois pares (duas cartas em um grupo, duas em outro grupo e uma em um terceiro
grupo)? [24192]

Resoluo: H C(8,2) modos de escolher os grupos das cartas que formaro os dois
pares, (C(4,2)) modos de escolher seus naipes, 6 modos de escolher o grupo da outra
carta e 4 modos de escolher seu naipe. A resposta C(8,2)x(C(4,2))x6x4 = 24192

d)uma trinca (trs cartas em um grupo e as outras duas em dois outros grupos
diferentes)? [10752]

Resoluo: H 8 modos de escolher o grupo da trinca, C(4,3) modos de escolher os


naipes das cartas da trinca, C(7,2) modos de escolher os grupos das outras duas cartas
e 4x4 = 4 modos de escolher os naipes dessas duas cartas. A resposta
8xC(4,3)xC(7,2)x4 = 10752

e)um "four" (quatro cartas em um grupo e uma em outro grupo)? [224]

H 8 modos de escolher o grupo do four, 1 modo de escolher os naipes das quatro


cartas do four, 7 modos de escolher o grupo da outra carta e 4 modos de escolher o
naipe dessa carta. A resposta 8x1x7x4 = 244

f)um "full hand" (trs cartas em um grupo e duas em outro grupo)? [1344]

Resoluo: H oito modos de escolher o grupo da trinca, C(4,3) modos de escolher os


naipes das cartas da trinca, 7 modos de escolher o grupo do par e C(4,2) modos de
escolher os naipes das cartas do par. A resposta 8xC(4,3)x7xC(4,2) = 1344.

g)uma sequncia (5 cartas de grupos consecutivos, no sendo todas do mesmo


naipe)? [4080]

Resoluo: H apenas 4 tipos de sequncias: 7, 8, 9, 10, valete; 8, 9, 10, valete, dama;


9, 10, valete, dama, rei; 10, valete, dama, rei, s. Escolhido o tipo da sequncia, haveria
4x4x4x4x4 modos de escolher os naipes das cartas da sequncia, mas 4 desses modos
no so permitidos: todas de ouros, todas de paus, todas de copas e todas de espadas.
A resposta 4x[4x4x4x4x4 4] = 4080

h)um "flush" (5 cartas do mesmo naipe, no sendo elas de 5 grupos consecutivos)?


[208]

Resoluo: Os grupos das cartas podem ser escolhidos de C(8,5) 4 modos e o naipe
nico, de 4 modos. A resposta (C(8,4) 4)x4 = 208
i)um "straight flush" (5 cartas de grupos consecutivos, todos do mesmo naipe)? [16]

Resoluo: H 4 modos de escolher os grupos das cartas e 4 modos de escolher o


naipe nico. A resposta 4x4 = 16

j)um "royal straight flush" (10,valete, dama, rei e s de um mesmo naipe)? [4]

Resoluo: H 4 modos de escolher o naipe nico. A resposta 4.

19) O conjunto A possui p elementos e o conjunto B possui n elementos. Determine o


nmero de funes f: A --> B sobrejetoras para:

a)p = n; [n!]

Resoluo: Neste caso a funo f bijetiva. Sendo assim, como calculamos em


exerccio anterior, o nmero de funes bijetivas onde ambos os conjuntos tem n
elementos n!.

b)p = n+1; [((n+1)!n)/2]

Resoluo: Neste caso dois elementos de A tero uma mesma imagem em B e a


correspondncia entre os demais n-1 elementos de A e os demais n-1 elementos de B
ser bijetiva. H C(n+1,2) modos de se escolher os dois elementos de A, n modos de se
escolher a imagem deles em B e (n-1)! Modos de construir uma correspondncia
bijetiva entre os elementos restantes. A resposta C(n+1,2) x n x (n-1)! = [n(n+1)!]/2

20) Considere um conjunto C de 20 pontos do espao que tem um subconjunto C1


formado por 8 pontos coplanares. Sabe-se que toda vez que 4 pontos de C so
coplanares, ento eles so pontos de C1. Quantos so os planos que contm pelo
menos trs pontos de C?

Resoluo:
Primeira maneira:

Chamaremos de D o conjunto C C1.


H quatro tipos de planos:
i)determinados por trs pontos de D;
ii)determinados por dois pontos de D e um de C1;
iii)determinados por um ponto de D e dois de C1;
iv)determinados por trs pontos de C1;
A resposta C(12,3)+C(12,2)x8+12xC(8,2)+1 = 1085

Segunda maneira:
Para determinar um plano, devemos selecionar 3 dos 20 pontos, o que pode ser feito
de C(20,3) = 1140 modos. Nessa contagem, o plano que contm os 8 pontos de C1, foi
contado C(8,3) = 56 vezes. A resposta 1.140 56 + 1 = 1085. R: 1085
16
Problemas de
Combinatria
Lista III

1
Unidade 16

Problemas
Lista 3
1. Uma la de cadeiras no cinema tem 10 poltronas. De quantos modos 3

casais podem se sentar nessas poltronas de modo que nenhum marido se

sente separado de sua mulher?

2. Quantos so os anagramas da palavra PARAGUAIO que no possuem

consoantes adjacentes?

3. De quantos modos podemos selecionar p elementos, sem selecionar dois

nmeros consecutivos, no conjunto {1, 2, . . . , n}?

4. Onze cientistas trabalham num projeto sigiloso. Por questes de segu-

rana, os planos so guardados em um cofre protegido por muitos cade-

ados de modo que s possvel abri-los todos se houver pelo menos 5

cientistas presentes.

a) Qual o nmero mnimo possvel de cadeados?

b) Na situao do item a), quantas chaves cada cientista deve ter?

5. Depois de ter dado um curso, um professor resolve se despedir de seus 7

alunos oferecendo, durante 7 dias consecutivos, 7 jantares para 3 alunos

cada. De quantos modos ele pode fazer os convites se ele no deseja que

um mesmo par de alunos comparea a mais de um jantar?

6. Formam-se as combinaes simples de classe 5 dos elementos a1 , a2 , . . . ,


a12 , as quais so escritas com os elementos em ordem crescente de ndices.
o
Quantas so as combinaes nas quais o elemento a8 ocupa o 3 lugar?

7. De quantos modos possvel colocar em la h homens e m mulheres,

todos de alturas diferentes, de modo que os homens entre si e as mulheres

entre si quem em ordem crescente de alturas?

8. Em uma escola, x professores se distribuem em 8 bancas examinadoras

de modo que cada professor participa de exatamente duas bancas e cada

duas bancas tm exatamente um professor em comum.

2
Problemas de Combinatria Lista III Unidade 16

a) Calcule x.

b) Determine quantos professores h em cada banca.

9. A partir de um conjunto de a atletas formam-se t times de k atletas

cada. Todos os atletas participam de um mesmo nmero de times e cada

par de atletas ca junto no mesmo time um mesmo nmero de vezes.

Determine:

a) de quantos times cada atleta participa;

b) em quantos times cada par de atletas ca junto.

10. De quantos modos podemos formar uma mesa de buraco com 4 jogado-

res?

11. De quantos modos podemos formar uma roda de ciranda com 5 meninos

e 5 meninas de modo que pessoas de mesmo sexo no quem juntas?

12. De quantos modos podemos formar uma roda de ciranda com 6 crianas,

de modo que duas delas, Vera e Isadora, no quem juntas?

13. Quantas so as solues inteiras e positivas de x + y + z = 7?

14. Quantas so as solues inteiras e no-negativas de x + y + z 6 6?

15. Uma indstria fabrica 5 tipos de balas que so vendidas em caixas de

20 balas, de um s tipo ou sortidas. Quantos tipos de caixas podem ser

montados?

3
Lista de Exerccios
Unidade 16

1. Escrevem-se os inteiros de 1 ate 2 222. Quantas vezes o algarismo 0 e


escrito?
2. Quantos sao os inteiros positivos de 4 dgitos nos quais o algarismo 5
figura?
3. Em uma banca ha 5 exemplares iguais da Veja, 6 exemplares iguais
da Epoca e 4 exemplares iguais da Isto e. Quantas colecoes nao-
vazias de revistas dessa banca podem ser formadas?
4. Uma turma tem aulas as segundas, quartas e sextas, de 13h as 14h e
de 14h as 15h. As materias sao Matematica, Fsica e Qumica, cada
uma com duas aulas semanais, em dias diferentes. De quantos modos
pode ser feito o horario dessa turma?
5. O problema do Exemplo 1 da Unidade 11 Com 5 homens e 5 mulhe-
res, de quantos modos se pode formar um casal? foi resolvido por um
aluno do modo a seguir: A primeira pessoa do casal pode ser escolhida
de 10 modos, pois ela pode ser homem ou mulher. Escolhida a primeira
pessoa, a segunda pessoa so podera ser escolhida de 5 modos, pois deve
ser de sexo diferente da primeira pessoa. Ha portanto 10 5 = 50
modos de formar um casal. Onde esta o erro?
6. Escrevem-se numeros de 5 dgitos, inclusive os comecados em 0, em
cartoes. Como 0, 1 e 8 nao se alteram de cabeca para baixo e como
6, de cabeca para baixo, se transforma em 9 e vice-versa, um mesmo
cartao pode representar dois numeros (por exemplo, 06198 e 86190).
Qual e o numero mnimo de cartoes para representar todos os numeros
de 5 dgitos?
7. Qual a soma dos divisores positivos de 360?
8. Uma fila de cadeiras no cinema tem 10 poltronas. De quantos modos 3
casais podem se sentar nessas poltronas de modo que nenhum marido
se sente separado de sua mulher?

1
9. Quantos sao os anagramas da palavra PARAGUAIO que nao pos-
suem consoantes adjacentes?
10. De quantos modos podemos selecionar p elementos, sem selecionar dois
numeros consecutivos, no conjunto {1, 2, . . . , n}?
11. Onze cientistas trabalham num projeto sigiloso. Por questoes de se-
guranca, os planos sao guardados em um cofre protegido por muitos
cadeados de modo que so e possvel abri-los todos se houver pelo me-
nos 5 cientistas presentes.
a) Qual e o numero mnimo possvel de cadeados?
b) Na situacao do item a), quantas chaves cada cientista deve ter?
12. Depois de ter dado um curso, um professor resolve se despedir de seus 7
alunos oferecendo, durante 7 dias consecutivos, 7 jantares para 3 alunos
cada. De quantos modos ele pode fazer os convites se ele nao deseja
que um mesmo par de alunos compareca a mais de um jantar?
13. Formam-se as combinacoes simples de classe 5 dos elementos a1 , a2 , . . . ,
a12 , as quais sao escritas com os elementos em ordem crescente de
ndices. Quantas sao as combinacoes nas quais o elemento a8 ocupa o
3o lugar?
14. De quantos modos e possvel colocar em fila h homens e m mulheres, to-
dos de alturas diferentes, de modo que os homens entre si e as mulheres
entre si fiquem em ordem crescente de alturas?
15. Em uma escola, x professores se distribuem em 8 bancas examinadoras
de modo que cada professor participa de exatamente duas bancas e
cada duas bancas tem exatamente um professor em comum.
a) Calcule x.
b) Determine quantos professores ha em cada banca.
16. A partir de um conjunto de a atletas formam-se t times de k atletas
cada. Todos os atletas participam de um mesmo numero de times e
cada par de atletas fica junto no mesmo time um mesmo numero de
vezes. Determine:
a) de quantos times cada atleta participa;
b) em quantos times cada par de atletas fica junto.

2
17. (Profmat - MA12 2013) Penelope quer distribuir 6 presentes entre seus
sobrinhos Alfredo, Bruno, Carlos e Daniel, de modo que cada um receba
pelo menos um presente. Todos os presentes devem ser distribudos.

a) Supondo que todos os presentes sejam iguais, de quantos modos


ela pode distribuir os presentes?
b) Resolva novamente o item a), supondo agora que todos os presen-
tes sejam diferentes.

18. (Profmat - Exame de Qualificacao 2012) Maria tem 10 aneis e quer


distribu-los pelos 10 dedos de suas maos. Suponha que seja possvel
colocar todos os aneis em qualquer um dos dedos.

a) Suponha que os aneis sejam identicos. De quantas maneiras dife-


rentes ela pode distribui-los em seus dedos?
b) Suponha agora que os 10 aneis sejam todos distintos. De quantas
maneiras Maria pode distribui-los em seus dedos? Admita que a
ordem dos aneis nos dedos e relevante.

3
Solucoes da Lista de Exerccios
Unidade 16

1. Vamos primeiramente determinar quantos zeros sao escritos na casa


das unidades, depois na das dezenas, etc.
Ha 222 numeros que tem o como algarismo das unidades, pois antes do
zero podem ser colocados os inteiros de 1 (inclusive) a 222 (inclusive).
Ha 22 10 = 220 numeros que tem 0 como algarismo nas dezenas,
pois antes do zero podem ser colocados os inteiros de 1 (inclusive) a 22
(inclusive) e depois do zero, os inteiros 0 (inclusive) a 9 (inclusive).
Ha 2 100 = 200 numeros que tem 0 como algarismo das centenas,
pois antes do zero podem ser colocados os inteiros de 1 (inclusive) a 2
(inclusive) e depois do zero os inteiros de 0 (inclusive) a 99 (inclusive).
A resposta e 222 + 220 + 200 = 642.

2. E mais simples contar, primeiramente, os numeros onde o algarismo


5 nao aparece. O primeiro digito pode ser escolhido de 8 modos (nao
pode ser igual a 0 nem igual a 5) e cada um dos demais tres dgitos
pode ser selecionado de 9 modos (deve ser diferente de 5). Logo, ha
893 = 5.832 numeros de a algarismos em que nao aparece o algarismo
5.
A quantidade de numero de 4 dgitos, com ou sem o digito 5, e 9103 =
9.000 (pois ha 9 modos de selecionar o primeiro digito, que deve ser
diferente de 0, e 10 modos de selecionar cada um dos demais 4 dgitos).
Logo, ha 9.000 5.832 = 3.168 numeros de 4 algarismos em que o 5
nao aparece.

3. Devemos decidir quantos exemplares de cada revista devem ser postos


na colecao. Ha 6 possibilidades para a Veja (0, 1, 2, 3, 4, ou 5
exemplares), 7 para a Manchete e 5 para a Isto e. O numero de
colecoes e 6 7 5 = 210, e o numero de colecoes nao-vazias e 209.

4. Em cada dia, duas das materia sao ensinadas e uma folga. Ha 3 possi-
bilidades para escolher a materia que folga na segunda, 2 para escolher

1
a que folga na quarta e 1 para escolher a que folga na sexta. Portanto,
ha 6 modos para escolher as materias de cada dia. Para escolher os
horarios, ha 2 possibilidades em cada dia. Logo, o numero total de
horarios e 6 8 = 48.

5. Foi feita uma distincao artificial ao se considerar cada casal ordenado


de dois modos diferentes: comecando pela mulher ou pelo homem. Por
esta razao, o resultado encontrado foi igual ao dobro do correto.

6. Ha tres tipos de cartoes: os que virados de cabeca para baixo nao


representam numeros, como, por exemplo, 41.809; os que virados de
cabeca para baixo representam o mesmo numero, como, por exemplo,
86.198; os que virados de cabeca para baixo representam numeros di-
ferentes, como, por exemplo, 66.810. Os cartoes do ultimo tipo sao
os que permitem economia porque um mesmo cartao serve para re-
presentar dois numeros. Ha 5 5 5 5 5 = 3.125 cartoes que
virados de cabeca para baixo representam numero, iguais ou diferen-
tes, pois tais cartoes devem ter como algarismos apenas 0, 1, 8, 6 ou
9. Destes, 5 5 3 = 75 sao do segundo tipo, pois um tal cartao
deve ter as casas das extremidades preenchidas por 00, 11, 88, 69 ou
96, a segunda e a quarta casa preenchidas por 00, 11, 88, 69 ou 96, e a
casa central preenchida por 0, 1 ou 8. Portanto, os cartoes do terceiro
tipo sao em numero de 3.125 75 = 3.050. Podem ser economizados
3.050/2 = 1.525 cartoes. O numero mnimo de cartoes que se necessita
e 100.000 1.525 = 98.475.

7. A decomposicao de 360 em fatores primos e 720 = 23 .32 .5. Os divisores


inteiros e positivos de 720 sao numeros da forma 2 .3 .5 , com
{0,
P 1, 2,3}, {0, 1, 2} e {0, 1}. A soma dos divisores e S =
2 .3 .5 , com {0, 1, 2, 3}, {0, 1, 2} e {0, 1}. Para
calcular essa soma, dividimos
P as 0parcelas
P em dois grupos,
P conforme seja
1
= 0 ou = 1. S = (2 .3 .5 ) + (2 .3 .5 ) = 6 (2 .3 ) porque
a segunda soma e igual ao quntuplo da primeira. Agora, dividimos
asP parcelas emPgrupos, conforme
P 2seja =P0, = P 1 ou =P 2. S =
0 1
6[ (2 .3 ) + (2 .3 ) + (2 .3 )] = 6[ 2 + 3 2 + 9 2 ] =
6[13 2 ] = 78 2 = 78[20 + 21 + 22 + 23 ] = 78 15 = 1.170.
P P

8. Escolhida a ordem de cada casal, o que pode ser feito de 23 modos,


temos que arrumar em fila 4 espacos vazios e 3 casais, o que pode ser

2
feito de C74 modos (escolha dos espacos vazios) vezes 3! (colocacao dos
3 casais nos 3 lugares restantes).
A resposta e 23 C74 3! = 1.680.

9. Primeiro, colocamos as vogais. Como a letra A aparece 3 vezes e as


letras U, I e O aparecem 1 vez cada, o numero de modos de dispo-las
e P63,1,1,1 = 6!
3!
= 720
6
= 120. A seguir, colocamos as consoantes em
tres dos 7 espacos antes, entre e depois das vogais. O lugar do P pode
ser qualquer um destes 7 espacos, o do R qualquer dos 6 restantes e
o do G qualquer dos 5 restantes. O numero total de possibilidades e
120.7.6.5 = 25.200.

10. Vamos a formar uma fila com os numeros 1, 2, . . . , n e assinalar com E


os p numeros escolhidas e com N os n p nao escolhidos. A condicao
para que nao sejam escolhidos numeros consecutivos e que entre dois
E haja pelo menos um N. Comecamos escrevendo os n p E. A seguir,
devemos escolher, para colocar os E, p dentre os n p + 1 espacos
p
situados antes, entre e depois dos N. Isto pode ser feito de Cnp+1
modos.

11. Chegam 4 cientistas A, B, C, D. Com as chaves que possuem, abrem


alguns cadeados, mas nao todos. Existe pelo menos um cadeado que
eles nao conseguem abrir. Na situacao do numero mnimo de cadeados,
existe exatamente um cadeado que eles nao conseguem abrir. Batize
tal cadeado de ABCD. Portanto, ABCD e p cadeado cuja chave nao
esta em poder de A, nem de B, nem de C e nem de D. Qualquer outro
cientista tem a chave desse cadeado, pois esse cientista e A, B, C e
D formam um grupo de 5 cientistas e, portanto, nesse grupo alguem
possui a chave. Como o alguem nao e nem A, nem B, nem C e nem D,
deve ser o outro. Analogamente batize os demais cadeados. Verifique
agora que a correspondencia entre cadeados e seus nomes e biunvoca,
isto e, cadeados diferentes tem nomes diferentes (isso porque estamos
na situacao do numero mnimo de cadeados) e cadeados de nomes di-
ferentes sao diferentes (se X esta no nome de um cadeado e nao esta
no nome de outro, X tem a chave deste e nao tem a chave daquele).

(a) O numero mnimo de cadeados e igual ao numero de nomes de


4
cadeados, C11 = 330.

3
(b) Cada cientista X possui as chaves dos cadeados que nao possuem
4
X no nome, C10 = 210.
12. Nenhum aluno pode comparecer a mais de tres jantares. Com efeito, se
A1 vai a um jantar com A2 e A3 , ele so pode ir a outro jantar com outros
dois estudantes, digamos A4 e A5 e so poder ir a um terceiro jantar em
companhia de outros dois, digamos A6 e A7 e nao tera companhia para
ir a um quarto jantar. Como ha 21 convites e sao 7 estudantes, cada
estudante tera que compadecer a exatamente 3 jantares.
Se A1 comparece a tres jantares, podemos escolher os seus companhei-
ros dividindo os outros 6 estudantes em 3 grupos de 2, o que pode ser
C 2 C42 1
feito de 6 = 15 modos. Entao, os 3 jantares sao, digamos,
3!
A1 A2 A3 , A1 A4 A5 , A1 A6 A7 .
A2 devera comparecer a mais dos jantares, nenhum deles em companhia
de A3 , e A3 tambem devera comparecer a mais dois jantares. Portanto,
os 4 jantares que faltam sao:
A2 , A2 , A3 , A3
Como A4 deve compareces a mais dois jantares (A4 nao pode compare-
ces a ambos em companhia de A2 nem a ambos em companhia de A3 ),
esses quatro jantares sao:
A2 A4 , A2 , A3 A4 , A3 ;
A5 tem que comparecer ainda a dois jantares, nenhum deles em com-
panhia de A4 .
A2 A4 , A2 A5 , A3 A4 , A3 A5 .
Agora ha duas possibilidades:
A2 A4 A6 , A2 A5 A7 , A3 A4 A7 , A3 A5 A6 e
A2 A4 A7 , A2 A5 A6 , A3 A4 A6 , A3 A5 A7 .
Ha portanto 15 2 = 30 maneiras de escolher os grupos de convidado.
Para distribuir os 7 grupos nos 7 dias, ha 7! alternativas.
A resposta e 7! 30 = 151.200.
13. Os dois primeiros lugares so podem ser ocupados por elementos de
{a1 , a2 , . . . , a7 } e os dois ultimos por exemplo de {a9 , a10 , a11 , a12 .
A resposta e C72 C42 = 126.

4
m
14. Ha Cm+h 1 modos de escolher os lugares para os homens. Feito isso,
so ha 1 modo de formar a fila.
m m
A resposta e Cm+h 1 = Cm+h .
15. (a) Cada professor fica caracterizado pelas duas bancas a que per-
tence. O numero de professores e igual ao numero de modos de
escolher duas das oito bancas.
A resposta e C82 = 28.
(b) O numero de professores pertencentes a uma banda e igual ao
numero de modos de escolher a outra banca a que ele pertence.
A resposta e 7.
16. (a) Imagine um quadro em que cada linha e relacao dos atletas de um
time. O numero de elementos do quadro e o numero de times, t,
multiplicado pelo tamanho de cada time, k, e e tambem igual ao
numero de atletas, a, multiplicado pelo numero de times de que a
cada atleta participa, x.
Logo, ax = tk e x = tka .
(b) No mesmo quadro, o numero de pares de atletas na mesma linha
e igual ao numero de linhas, t, multiplicado pelo numero de pares
de atletas em uma linha, Ck2 , e e tambem igual ao numero de pares
de atletas, Ca2 , multiplicado pelo numero de times em que cada
par de atletas fica junto, y.
tCk2 tk(k1)
Logo, yCa2 = tCk2 e y = Ca2
= a(a1)
.

17. (a) Uma vez tendo distribudo um presente para cada um dos so-
brinhos, sobram 2 presentes, para distribuir para 4 criancas. O
numero de modos de faze-lo e igual ao numero de solucoes nao
negativas de x1 + x2 + x3 + x4 = 2, que e igual a C42 = C52 = 10.
(b) Primeiro, devemos decidir qual e a quantidade de presentes que
cada sobrinho vai receber. As possibilidades sao as seguintes:
um dos sobrinhos recebe 3 presentes e os demais 1.
O sobrinho a receber os 3 presentes pode ser escolhido de 4
modos. Os presentes dos demais podem ser escolhidos de 6
5 4 = 120 modos (os que restarem ?cam com o sobrinho que
recebe 3 presentes). O numero total de possibilidades e 4 120
= 480.

5
exatamente dois sobrinhos recebem dois presentes.
Esses sobrinhos podem ser escolhidos de C4;2 = 6 modos. Os
presentes dos outros sobrinhos podem ser escolhidos de 65
= 30 modos. Os presentes de um dos sobrinhos a receber 2
presentes podem ser escolhidos de C4;2 = 6 modos. O numero
total de possibilidades e 6 30 6 = 1080.
O numero total de modos de distribuir os presentes e 480 + 1080
= 1560.

18. (a) O numero de possibilidades e o numero de solucoes nao negativas


10 10 19!
de x1 + x2 + . . . + x10 = 10, que e CR10 = C19 = .
10!9!
(b) Para cada possibilidade do item anterior, ha 10! posicoes dis-
10 19!
posicoes dos aneis. Logo, o numero de modos e 10!C19 = .
9!

6
17
Probabilidade

Sumrio
17.1 Introduo . . . . . . . . . . . . . . . . . . . . . . . 2

17.2 Conceitos Bsicos . . . . . . . . . . . . . . . . . . . 2

1
Unidade 17 Introduo

17.1 Introduo

Iniciamos, nesta unidade, o estudo de Probabilidade, cuja parte mais ele-

mentar uma das aplicaes da Combinatria. A Teoria de Probabilidade, como

diz o nome, o estudo de fenmenos que envolvem a incerteza e se originou

como instrumento para modelar jogos de azar, como cartas e dados.

Probabilidade a base para a Estatstica, cincia utilizada nas mais diversas ati-

vidades humanas, sendo fundamental em vrias reas, como Cincias Humanas,

Cincias da Sade, Economia e Finanas, Ecologia e Teoria dos Jogos, entre

muitos outros. Do ponto de vista terico, atualmente, a Teoria de Probabili-

dade utilizada como ferramenta em algumas reas da Fsica e, cada vez mais,

em reas da prpria Matemtica. Por esse motivo, o ensino de Probabilidade

no Ensino Mdio importante e atual.

Esse assunto muito vasto, mas aqui s trataremos de alguns conceitos bsicos

e suas aplicaes. Denem-se o conjunto espao amostral e a noo de proba-

bilidade como sendo uma funo numrica com domnio no conjunto das partes

desse espao. Os subconjuntos do espao amostral so os chamados eventos.

As propriedades bsicas da funo probabilidade so dadas no Teorema 3, que

bastaro para resolver os problemas dessa unidade.

17.2 Conceitos Bsicos

Experincias que repetidas sob as mesmas condies produzem geralmente

resultados diferentes so chamadas de aleatrias. Por exemplo, retira-se uma

carta de um baralho e verica-se se ela ou no um curinga; compra-se uma

lmpada e verica-se se ela queima ou no antes de 100h de uso; joga-se um

dado at se obter um seis e conta-se o nmero de lanamentos.

Definio 1 Chamaremos de espao amostral o conjunto de todos os resultados possveis


Espao Amostral
de uma experincia aleatria. Representaremos o espao amostral por S e

s vamos considerar aqui o caso de S ser nito ou innito enumervel. Os

subconjuntos de S sero chamados de eventos. Diremos que um evento ocorre

quando o resultado da experincia pertence ao evento.

2
Probabilidade Unidade 17

Lana-se uma moeda e observa-se a face que cai voltada para cima. O Exemplo 1
espao amostral S = {cara, coroa} e h 4 eventos:
, A = {cara}, B = {coroa} e S .
um evento que no ocorre nunca e chamado de evento impossvel.

O evento A ocorre se e somente se o lanamento resulta em cara. S ocorre

sempre e chamado de evento certo.

Lana-se um dado e observa-se a face que cai voltada para cima. O espao Exemplo 2
amostral S = {1, 2, 3, 4, 5, 6} e h 64 eventos. Alguns desses eventos so:
, que no ocorre nunca; S , que ocorre sempre; A = {2, 4, 6}, que ocorre se e

somente se o resultado do lanamento for par. Se o resultado do lanamento

for seis, ocorrem os eventos {6}, {5, 6}, {2, 4, 6}, etc.

Se AeB so eventos em um mesmo espao amostral S, A B o evento Exemplo 3


que ocorre se e somente se ocorre o evento A ou ocorre o evento B , isto ,
ocorre pelo menos um dos eventos A e B ; A B o evento que ocorre se e

somente se ocorrem ambos os eventos A e B ; A \ B o evento que ocorre

se e somente se ocorre o evento A mas no ocorre o evento B ; A, chamado

de evento oposto a A, o evento que ocorre se e somente se o evento A no

ocorre.

Associaremos a cada evento um nmero, que chamaremos de probabilidade

do evento e que traduzir nossa conana na capacidade do evento ocorrer.

Uma probabilidade uma funo que associa a cada evento A um nmero Definio 2
Probabilidade
P (A) de forma que:

i) Para todo evento A, 0 6 P (A) 6 1.

ii) P (S) = 1

iii) Se A e B so eventos mutuamente excludentes , isto , eventos que no

podem ocorrer simultaneamente (isto , A B = ) ento

P (A B) = P (A) + P (B).

3
Unidade 17 Conceitos Bsicos

Exemplo 4 Lana-se uma moeda e observa-se a face que cai voltada para cima. O

espao amostral S = {cara, coroa} e h 4 eventos: , A = {cara}, B =


{coroa}, S . Uma probabilidade que pode ser denida

P1 () = 0, P1 (A) = P1 {cara} = 0, 5, P1 (B) = P1 {coroa} = 0, 5 P1 (S) = 1.

Verique que as trs condies da denio de probabilidade so satisfeitas.

Outra probabilidade que pode ser denida

P2 () = 0, P2 (A) = P2 {cara} = 0, 3, P2 (B) = P2 {coroa} = 0, 7 P2 (S) = 1.

Verique que as trs condies da denio de probabilidade so satisfeitas.

claro que se desejamos que a probabilidade traduza nossa conana na

capacidade do evento ocorrer, P1 constitui um modelo adequado quando acre-

ditamos ser o resultado cara to provvel quanto o resultado coroa. P2 , por sua
vez seria mais adequado se tivssemos lanado a moeda um nmero grande de

vezes e obtido o resultado cara em 30% dos lanamentos.

Encerrando o exemplo, um breve comentrio a respeito de notao. Deve-

ramos ter escrito P ({cara}) e no P {cara}. Entretanto, quando no houver

risco de confuso daremos preferncia notao mais simples.

Os modelos probabilsticos que usamos mais frequentemente so exatamente

os apresentados no exemplo anterior.

Um o modelo equiprobabilstico. Se temos n elementos no espao amostral


e queremos que todos os eventos unitrios tenham a mesma probabilidade,
1
devemos atribuir a cada evento unitrio a probabilidade . No poderia ser de
n
outra forma pois se S = {x1 , x2 , . . . , xn } e P (x1 ) = P (x2 ) = = P (xn ) = k ,

temos, por iii),

1 = P (S) = P {x1 , x2 , . . . , xn } = P ({x1 } {x2 } {xn })


= P ({x1 }) + P ({x2 }) + + P ({xn })
1
= k + k + + k = nk e k = .
n
Analogamente, fcil ver que, nesse modelo, se um evento X formado
j
por j elementos ento P (X) = . Ou seja, a probabilidade de um evento a
n
razo entre o nmero de casos favorveis ao evento e o nmero total de casos

4
Probabilidade Unidade 17

1
possveis. Foi esse o modelo adotado por vrios matemticos como Cardano ,
2
Pascal e Laplace entre outros, no estudo dos jogos de azar.

Outro o modelo frequencial. Se repetimos a experincia n vezes e o evento


A ocorreu em j P (A) a frequncia relativa
dessas experincias, adotamos para

do evento A, isto , o nmero de vezes que o evento A ocorreu dividido pelo


j
nmero total de repeties da experincia, ou seja, P (A) = .
n
O teorema a seguir contm as propriedades das probabilidades.

Se A e B so eventos, ento: Teorema 3


Propriedades da
i) P (A) = 1 P (A). Evento oposto
probabilidade
ii) P () = 0.
iii) P (A \ B) = P (A) P (A B).
iv) P (A B) = P (A) + P (B) P (A B).
v) Se AB ento P (A) > P (B).

Demonstrao
i) 1 = P (S) = P (A A) = P (A) + P (A). Da, P (A) = 1 P (A).
P (S) = P (S ) = P (S)+P (), pois S e so mutuamente excludentes.
ii)

Da, P () = 0.

P (A) = P [(A \ B) (A B)] = P (A \ B) + P (A B) pois A \ B


iii) e AB
so mutuamente excludentes. Da, P (A \ B) = P (A) P (A B).

iv) P (A B) = P [(A \ B) B] = P (A \ B) + P (B) pois A \ B e B


so mutuamente excludentes. Como P (A \ B) = P (A) P (A B), resulta

P (A B) = P (A) + P (B) P (A B).


v) Como P (A \ B) = P (A) P (A B), se A B resulta P (A \ B) =
P (A) P (B). Como P (A \ B) > 0, temos P (A) > P (B).

Em um grupo de r pessoas, qual a probabilidade de haver pelo menos Exemplo 5


duas pessoas que faam aniversrio no mesmo dia?

Soluo. Vamos determinar a probabilidade disso no acontecer. O nmero de

1 Cardano, Jernimo (1501-1576), matemtico italiano.


2 Laplace, Pierre Simon (1749-1827), matemtico francs.

5
Unidade 17 Conceitos Bsicos

casos possveis para os aniversrios das r pessoas 365r . O nmero de casos

favorveis a que todas aniversariem em dias diferentes 365 364


(366 r), havendo r fatores nesse produto. Portanto, a probabilidade de no

haver pelo menos duas pessoas que faam aniversrio no mesmo dia de

365 364 (366 r)


365r
e a de haver pelo menos duas pessoas que tenham o mesmo dia de aniversrio

de
365 364 (366 r)
1 .
365r
A tabela abaixo d, para alguns valores de r , a probabilidade de haver

coincidncia de aniversrios.

r Probabilidade

5 0, 03
10 0, 12
15 0, 25
20 0, 41
23 0, 51
25 0, 57
30 0, 71
40 0, 89
45 0, 94
50 0, 97
O resultado surpreendente. Em um grupo de 23 pessoas, mais provvel

haver duas pessoas com o mesmo aniversrio do que todas aniversariarem em

dias diferentes.

Exemplo 6 Em uma loteria de N nmeros h um s prmio. Salvador compra n


(1 < n < N ) bilhetes para uma s extrao e Slvio compra n bilhetes, um para
cada uma de n extraes. Qual dos dois jogadores tem mais chance de ganhar

algum prmio?

Soluo. A probabilidade de Salvador ganhar algum prmio

n
.
N

6
Probabilidade Unidade 17

A probabilidade de Slvio no ganhar nenhum prmio

(N 1)n
.
Nn
Logo, a probabilidade de Slvio ganhar algum prmio

(N 1)n
1 .
Nn
Armamos que Salvador tem mais chance de ser premiado, isto , armamos

que
n (N 1)n
>1 ,
N Nn
ou, equivalentemente, armamos que

(N 1)n n
> 1 .
Nn N

A prova dessa armao faz-se por induo.

Para n=2 temos

(N 1)n (N 1)2 2 1 2 n
n
= 2
=1 + 2 >1 =1 .
N N N N N N
Se
(N 1)n n
n
>1
N N
multiplicando por
N 1
N
obtemos
(N 1)n+1 n 1 n n+1
n+1
>1 + 2 >1 .
N N N N N

7
Unidade 17 Conceitos Bsicos

Exerccios Recomendados

1. Lanam-se dois dados no-tendenciosos. Qual a probabilidade da soma

dos pontos ser igual a 7 ?

2. 24 times so divididos em dois grupos de 12 times cada. Qual a proba-

bilidade de dois desses times carem no mesmo grupo?

3. Mostre que

P (A B C) = P (A) + P (B) + P (C)


P (A B) P (A C) P (B C) + P (A B C).

2 4
4. Se P (A) = e P (B) = , mostre que:
3 9
2
a) P (A B) > ;
3
2 5
b) 6 P (A B) 6 ;
9 9
1 4
c) 6 P (A B) 6 .
9 9
5. Cinco dados so jogados simultaneamente. Determine a probabilidade de

se obter:

a) um par;

b) dois pares;

c) uma trinca;

d) uma quadra;

e) uma quina;

f ) uma sequncia;

g) um full hand, isto , uma trinca e um par.

6. Um polgono regular de 2n+1 lados est inscrito em um crculo. Escolhem-

se trs dos seus vrtices, formando um tringulo. Determine a probabili-

dade do centro do crculo ser interior ao tringulo.

8
Probabilidade Unidade 17

7. Doze pessoas so divididas em trs grupos de 4. Qual a probabilidade

de duas determinadas dessas pessoas carem no mesmo grupo?

8. Em um grupo de 4 pessoas, qual a probabilidade de haver alguma

coincidncia de signos zodiacais?

9. Em um armrio h 5 pares de sapatos. Escolhem-se 4 ps de sapatos.

Qual a probabilidade de se formar exatamente um par de sapatos?

9
MA12 - Unidade 17
Probabilidade

Paulo Cezar Pinto Carvalho

PROFMAT - SBM

17 de Maio de 2013
Teoria da Probabilidade

Teoria da Probabilidade: modelo matematico para incerteza.


Objeto de estudo: experimentos aleatorios.
Componentes de um modelo probabilstico:
Espaco amostral (S): conjunto de todos os resultados
possveis de um experimento aleatorio.
Eventos : os subconjuntos do espaco amostral S.
Funcao de Probabilidade: uma funcao que associa a cada
evento A sua probabilidade P(A), satisfazendo:
P(A) 0, para todo evento A.
P(S) = 1 (o evento certo).
P(A B) = P(A) + P(B), se A e B sao disjuntos (ou
mutuamente excludentes).

Para que um modelo probabilstico seja util, P(A) deve


representar o grau de confianca em que o evento A ocorra.

PROFMAT - SBM MA12 - Unidade 17, Probabilidade slide 2/10


Propriedades da Funcao de Probabilidade

i) P(A) = 1 P(A).

ii) P() = 0.

iii) 0 P(A) 1.

iv) P(A B) = P(A) P(A B).

v) P(A B) = P(A) + P(B) P(A B).

vi) Se A B entao P(A) 6 P(B).

PROFMAT - SBM MA12 - Unidade 17, Probabilidade slide 3/10


Exemplo
Qual e o modelo probabilstico adequado para o lancamento
de um dado? Qual e a probabilidade de que saia um resultado
par?
O espaco amostral e S = {1, 2, 3, 4, 5, 6}.
Se o dado e um cubo geometricamente perfeito e feito com
material homogeneo (um dado equilibrado ou honesto), nao
ha razao para acreditar que uma face saia mais
frequentemente que qualquer outra.
O modelo razoavel e o equiprovavel, que atribui
probabilidades iguais aos eventos correspondentes a ocorrencia
de cada face (neste caso, iguais a 16 ).
O evento sair resultado parcorresponde ao subconjunto
A = {2, 4, 6} de S.
Sua probabilidade e
1 1 1
P(A) = P({2}) + P({4}) + P({6}) = 6 + 6 + 6 = 12 .
PROFMAT - SBM MA12 - Unidade 17, Probabilidade slide 4/10
Modelos Equiprovaveis
Modelo equiprovavel: atribui probabilidade n1 a cada evento
unitario de um espaco amostral S = {a1 , a2 , . . . , an }.
Em consequencia, a probabilidade de um evento
A = {a1 , a2 , . . . , ap } e
P(A) = P({a1 }) + . . . + P({ap }) = p n1 = pn .
Portanto:
n(A) Numero de casos favoraveis
P(A) = =
n(S) Numero de casos possveis

(nos problemas mais interessantes, tecnicas de Analise


Combinatoria sao usadas para calcular o numerador e o
denominador).
Atencao: o uso de modelos equiprovaveis so se justifica
quando algum tipo de simetria permite concluir que os
resultados possveis tem a mesma chance de ocorrer.
dados, moedas, cartas, bolas iguais em urnas, etc.
PROFMAT - SBM MA12 - Unidade 17, Probabilidade slide 5/10
Exemplo

Duas bolas sao retiradas seguidamente e sem reposicao de


uma urna com 3 bolas brancas e 5 pretas, todas identicas, a
menos da cor. Qual e a probabilidade de que a primeira seja
branca e a segunda preta?
Ha quatro possibilidades para a cor das bolas: BB, BP, PB e
PP. Dos quatro casos possveis, apenas um e favoravel. Logo,
a probabilidade e 14 .
A solucao acima esta ERRADA!
Nao ha qualquer razao para admitirmos, por exemplo, que a
probabilidade de tirar duas bolas brancas seja a mesma que a
de tirar duas bolas pretas, ja que ha mais bolas pretas do que
brancas na urna.

PROFMAT - SBM MA12 - Unidade 17, Probabilidade slide 6/10


Exemplo
Duas bolas sao retiradas seguidamente e sem reposicao de
uma urna com 3 bolas brancas e 5 pretas, todas identicas, a
menos da cor. Qual e a probabilidade de que a primeira seja
branca e a segunda preta?
O espaco amostral adequado e o que considera que ha oito
bolas b1 , . . . , b8 na urna e considera todos os possveis pares
(bi , bj ) de bolas distintas retiradas.
Como todas as bolas sao identicas, todos os pares possveis
tem a mesma chance de ocorrer (um modelo equiprovavel!).
Numero de casos possveis:
A primeira bola pode ser qualquer uma das 8.
A segunda pode ser qualquer uma das outras 7.
O numero de casos possveis e 8 7 = 56.
Numero de casos favoraveis:
A primeira bola pode ser qualquer uma das 3 bolas brancas.
A segunda pode ser qualquer uma das 5 bolas pretas.
O numero de casos favoraveis e 3 5 = 15.
15
A probabilidade e 56 .
PROFMAT - SBM MA12 - Unidade 17, Probabilidade slide 7/10
Exemplo
Em um grupo de r pessoas, qual e a probabilidade de haver
pelo menos duas pessoas que facam aniversario no mesmo
dia?
O numero de casos possveis para os aniversarios das r
pessoas e 365r .
O numero de casos favoraveis a que todas aniversariem em
dias diferentes e 365 364 (366 r ).
A probabilidade de nao haver pelo menos duas pessoas que
facam aniversario no mesmo dia e
365 364 (366 r )
365r
A probabilidade de haver pelo menos duas pessoas que
tenham o mesmo dia de aniversario e
365 364 (366 r )
1 .
365r
PROFMAT - SBM MA12 - Unidade 17, Probabilidade slide 8/10
Continuacao
Alguns valores da probabilidade de haver coincidencia de
aniversarios:
r Probabilidade
5 0, 03
10 0, 12
15 0, 25
20 0, 41
23 0, 51
25 0, 57
30 0, 71
40 0, 89
45 0, 94
50 0, 97

A partir de 23 pessoas, e mais provavel que haja coincidencia


de aniversarios do que nao haja!

PROFMAT - SBM MA12 - Unidade 17, Probabilidade slide 9/10


E se um modelo equiprovavel nao for adequado?
Um dado tem a forma de um bloco retangular com as
dimensoes da figura. Qual e a probabilidade de sair a face 1?

Como nao ha simetria perfeita, nao e correto usar um modelo


equiprovavel.
A simetria restante na figura permite escrever:
P({1}) = P({6}) = p
P({2}) = P({3}) = P({4}) = P({5}) = 12p 4
O valor de p pode ser estimado a partir das frequencias de
ocorrencia das diversas faces (e o papel da Estatstica).
PROFMAT - SBM MA12 - Unidade 17, Probabilidade slide 10/10
Lista de Exerccios
Unidade 17

1. Lancam-se dois dados nao-tendenciosos. Qual a probabilidade da soma


dos pontos ser igual a 7 ?

2. 24 times sao divididos em dois grupos de 12 times cada. Qual e a


probabilidade de dois desses times ficarem no mesmo grupo?

3. Mostre que

P (A B C) = P (A) + P (B) + P (C)


P (A B) P (A C) P (B C) + P (A B C).

2 4
4. Se P (A) = e P (B) = , mostre que:
3 9
2
a) P (A B) > ;
3
2 5
b) 6 P (A B) 6 ;
9 9
1 4
c) 6 P (A B) 6 .
9 9
5. Cinco dados sao jogados simultaneamente. Determine a probabilidade
de se obter:

a) um par;
b) dois pares;
c) uma trinca;
d) uma quadra;
e) uma quina;
f) uma sequencia;
g) um full hand, isto e, uma trinca e um par.

1
6. Um polgono regular de 2n + 1 lados esta inscrito em um crculo.
Escolhem-se tres dos seus vertices, formando um triangulo. Determine
a probabilidade do centro do crculo ser interior ao triangulo.

7. Doze pessoas sao divididas em tres grupos de 4. Qual e a probabilidade


de duas determinadas dessas pessoas ficarem no mesmo grupo?

8. Em um grupo de 4 pessoas, qual e a probabilidade de haver alguma


coincidencia de signos zodiacais?

9. Em um armario ha 5 pares de sapatos. Escolhem-se 4 pes de sapatos.


Qual e a probabilidade de se formar exatamente um par de sapatos?

2
Solucoes da Lista de Exerccios
Unidade 17

1. Ha 66 = 36 resultados possveis igualmente provaveis, em 6 dos quais


6
a soma vale 7. A resposta e 36 = 61 .
12
2. Basta escolher os times do primeiro grupo, o que pode ser feito de C24
modos. Os dois times em questao ficam juntos quando ficam ambos no
primeiro grupo ou ambos no segundo grupo. Em ambos os casos, isto
10
10
2C22 2.22!.12! 11
pode ser feito de C22 modos. Logo, a resposta e 12 = = .
C24 24!10! 23
Outra solucao:
Supondo ja escolhido o grupo do primeiro time, seus 11 companheiros
11
de grupo podem ser escolhidos de C23 modos. Dentre os grupos assim
10
formados os que tambem incluem o segundo time sao C22 , ja que sao
formados escolhendo 10 times entre os 22 restantes. Logo, a resposta e
10
C22 11
11
= .
C23 23
3. Usaremos o fato, ja provado, de que P (A B) = P (A) + P (B) P (A).
Temos:

P (A B C) =P ((A B) C) = P (A B) + P (C) P ((A B))


=P (A) + P (B) P (A) + P (C) P ((A B)).

Agora, (A B) = (A) (B) e da

P ((A B)) =P (A) + P (B) P ((A)B))


=P (A) + P (B) P (A).

Substituindo na expressao anterior, obtemos, finalmente:

P (A B C) =P (A) + P (B) + P (C) P (A) P (A)


P (B) + P (A).

1
4. (a) Como A A B, temos P (A B) P (A) = 23 .
(b) Como A e B sao disjuntos e AB B = A B, temos P (AB) +
P (B) = P (A B) e, portanto, P (AB) = P (A B) P (B) =
P (A B) 49 .
Mas, do tem anterior, 23 P (A B) 1. Da, 32 49 P (AB)
1 94 , ou seja, 29 P (AB) 59 .
(c) Observe que P (AB) + P (A) = P (A), ja que o conjunto da direita
e a uniao disjunta dos da esquerda. Da, P (A) = P (A)P (AB) =
2
3
P (AB). Como 29 P (AB) 59 , temos 32 59 P (A) 23 92 ,
ou seja, 19 P (A) 94 .

5. (a) O numero de casos possveis e 65 , pois ha 6 resultados para cada


um dos 5 dados. O numero de casos favoraveis e 6 C52 5 4 3 =
3600, pois ha 6 modos de escolher o tipo de par (pode ser de 1, de
2, . . . , de 6) e ha C52 modos de escolher os dois dados que formarao
o par; quatro outros dados, o resultado do primeiro deles pode ser
escolhido de 5 modos distintos (deve ser diferente do resultado dos
dois primeiros dados), o do segundo pode ser escolhido de 4 modos
distintos (deve ser diferente dos anteriores) e, o do terceiro, de 4
modos diferentes.
3.600 25
Logo, P (A2 ) = = 0, 463.
65 54
(b) O numero de casos possveis e 65 , pois ha 6 resultados para cada
um dos 5 dados. O numero de casos favoraveis e C62 C52 C32 4 =
1800, pois ha C62 modos de escolher os tipos de pares (podem ser
de 1 e 2, de 1 e 3, . . . , de 5 e 6), ha C52 modos de escolher os
dois dados que formarao o par menor 3 C32 modos de escolher
os dados que formarao o par maior. Para o dado restante, que
deve ter resultado diferente do dos demais dados, ha 4 resultados
possveis.
180 25
Logo, P (A3 ) = 5 = 0, 231.
6 108
(c) O numero de casos possveis e 65 , pois ha 6 resultados para cada
um dos 5 dados. O numero de casos favoraveis e 6C53 54 = 1200,
pois ha 6 modos de escolher o tipo de trinca (pode ser de 1, de
2, . . . , de 6) e ha C53 modos de escolher os tres dados que terao
resultados iguais; quatro aos outros dados, o resultado do primeiro

2
deles pode ser escolhido de 5 modos distintos (deve ser diferente
do resultado dos tres primeiros dados) e, o do segundo, de 4 modos
distintos (deve ser diferente dos anteriores).
1200 25
Logo, P (A4 ) = 5 = 0, 154.
6 162
(d) O numero de casos possveis e 65 , pois ha 6 resultados para cada
um dos 5 dados. O numero de casos favoraveis e 6 C54 5 = 150,
pois ha 6 modos de escolher o tipo de quadra (pode ser de 1, de
2, . . . , de 6) e ha C54 modos de escolher os quatro dados que terao
resultados iguais; quatro ao dado restante, seu resultado pode ser
de 5 modos distintos (deve ser diferente do resultado dos quatro
primeiros dados).
150 25
Logo, P (A6 ) = 5 = 0, 019.
6 1296
(e) O numero de casos possveis e 65 e o numero de casos favoraveis
e 6.
6 1
Logo, P (A7 ) = 5 = 0, 0008.
6 1296
(f) O numero de casos possveis e 65 , pois ha 6 resultados para cada
um dos 5 dados. Ha dois tipos possveis de sequencias: a mnima
(12345) e a maxima (23456). A mnima pode ser formada de
5! = 120 modos distintos, pois ha 5 modos de escolher o dado cujo
resultado e 1, 4 modos de escolher o dado cujo resultado e 2, etc.
Ha analogamente, 120 modos de formar a sequencia maxima.
240 5
Portanto, P (A8 ) = 5 = 0, 031.
6 162
(g) O numero de casos possveis e 65 , pois ha 6 resultados para cada
um dos 5 dados. O numero de casos favoraveis e 6 C53 5 = 300,
pois ha 6 modos de escolher o tipo de trinca (pode ser de 1, de 2,
. . . , de 6) e ha C53 modos de escolher os tres dados que formarao a
trinca; quatro aos outros dados, ha 5 modos distintos de escolher
o resultado comum deles.
300 25
Logo, P (A5 ) = 5 = 0, 039.
6 684
6. Numeramos os vertices do polgono de 0 a 2n. Imagine 0 como o vertice
mais alto, os vertices de 1 a n do lado direito e os vertices de n + 1 a

3
2n do lado esquerdo. Podemos pensar que todos os triangulos tem 0
2
2n(2n 1)
como um dos vertices. Ha C2n = = n(2n 1) modos de
2
selecionar os outros dois vertices do triangulo.
Para construir o numero de triangulos que contem o centro da circun-
ferencia em seu interior, observe inicialmente que a reta que contem o
vertice i(1 i n) e o centro da circunferencia corta novamente o
polgono no ponto medio do segmento determinado pelos vertices i + n
e i + n + 1 (vertice 2n + 1= vertice 0).
Um triangulo que contenha em seu interior o centro da circunferencia
sera necessariamente formado por um vertice do lado direito e um
vertice do lado esquerdo. Se o vertice do lado direito for o vertice
1, o do lado esquerdo so podera ser o vertice n + 1 (1 possibilidade);
se for o vertice 2, podera ser qualquer dos vertices de n + 1 a n + 2 (2
possibilidades); . . . ; se for o vertice n, podera ser qualquer dos vertices
de n + 1 a 2n (n possibilidades). O numero de casos favoraveis e
n(n + 1)
1 + 2 + + n = .
2
n+1
A resposta e .
2(2n 1)
7. Imagina o resultado do sorteio como uma fila de 12 lugares: o primeiro
lugar corresponde a primeira pessoa sorteada area o primeiro grupo;
o segundo a segunda pessoa sorteada para o segundo grupo; . . . ; o
ultimo, a quarta pessoa sorteada para o terceiro grupo. Colocada a
primeira pessoa, ha 11 posicoes para a segunda, em 3 das quais ela fica
no mesmo grupo da primeira.
3
A resposta e .
11
8. Ha 12 possibilidades para o signo de cada pessoa, para um total de 124
possibilidades. Para que nao haja coincidencia de signos, o signo da
primeira pessoa pode ser escolhido de 12 modos, o da segunda de 11,
o da terceira de 10 e o da quarta de 9, para um total de 12 11 10 9
modos. Assim, a probabilidade de que nao haja coincidencia de signos
12 11 10 9 55
e = e a probabilidade de que nao haja coincidencias
124 96

4
55 41
e 1 = .
96 96
4
9. Ha C10 modos de retirar 4 pes de sapatos. Para retirar 4 pes, havendo
nesses 4 pes exatamente 1 par de sapatos, devemos inicialmente seleci-
onar 1 par (o que pode ser feito de 5 modos) e depois selecionar 2 pes
vindo de pares diferentes dentre os 4 pares que ainda estao no armario.
Para isso devemos escolher os pares de onde virao esses sapatos (C24
modos) e, em cada par escolhido, decidir se retiramos o pe direito ou o
pe esquerdo (22 = 4 modos).
5.C42 .4 4
A resposta e 4
= .
C10 7

5
18
Probabilidade
Condicional
Sumrio
18.1 Introduo . . . . . . . . . . . . . . . . . . . . . . . 2

18.2 Probabilidade Condicional . . . . . . . . . . . . . . . 2

1
Unidade 18 Introduo

18.1 Introduo

Nessa unidade, apresentada mais uma tcnica bsica importante em pro-

babilidades, a chamada Probabilidade Condicional. Usa-se essa tcnica quando

se quer calcular a probabilidade de um evento, na presena de uma informao

privilegiada. Mais precisamente, uma maneira de calcular a probabilidade

de ocorrer um evento B, sabendo que ocorreu o evento A, ambos do mesmo

espao amostral.

Por exemplo, numa turma de 60 alunos, 30 s estudam ingls, 20 s estudam

espanhol e 10 estudam ambas as lnguas. Suponhamos que um sorteio re-

alizado, com apenas um vencedor. A probabilidade de um aluno que estuda

ambas as lnguas ser sorteado igual a

nmero de alunos que estudam ambas as lnguas 10 1


= = .
nmero total de alunos 60 6
Agora, suponhamos que o sorteio realizado, e algum nos sopra que o sor-

teado estuda ingls. Isto certamente vai inuir no nosso modo de calcular a

probabilidade do vencedor ser bilngue, pois agora o espao amostral se reduz

aos 40 alunos que estudam ingls, dos quais 10 tambm estudam espanhol;
10 1
logo, a probabilidade passa a ser = .
40 4
O resultado to simples quanto mostrado no exemplo acima, mas, se bem

aplicado, resolve problemas incrveis!

18.2 Probabilidade Condicional

Exemplo 1 Consideremos a experincia que consiste em jogar um dado no-viciado e

observar a face de cima. Consideremos o evento B = {o resultado par }.


3
Temos P (B) = = 0, 5. Essa a probabilidade de B a priori, isto ,
6
antes que a experincia se realize. Suponhamos que, realizada a experin-

cia, algum nos informe que o resultado no foi o nmero 1, isto , que

A = {o resultado diferente de 1 } ocorreu.

Nossa opinio sobre a ocorrncia de B se modica com essa informao pois

passamos a ter apenas 5 casos possveis, dos quais 3 so favorveis ocorrncia

2
Probabilidade Condicional Unidade 18

de B. Essa opinio quanticada com a introduo de uma probabilidade a


posteriori, ou probabilidade de B na certeza de A,
3
P (B|A) = = 0, 6.
5
Note que os casos possveis no so mais todos os elementos do espao

amostral S e sim os elementos de A e que os casos favorveis ocorrncia de

B no so mais todos os elementos de B e sim os elementos de AB pois s

os elementos que pertencem a A podem ocorrer.

A tabela abaixo d a distribuio dos alunos de uma turma, por sexo e por Exemplo 2
carreira pretendida.

masculino feminino total

cientca 15 5 20
humanstica 3 7 10
total 18 12 30
Escolhe-se ao acaso um aluno. Sejam M, F, C e H os eventos, o aluno

selecionado do sexo masculino, do sexo feminino, pretende uma carreira

cientca e pretende uma carreira humanstica, respectivamente. Temos

10 1
P (H) = = ;
30 3
3 1
P (H|M ) = = ;
18 6
7
P (H|F ) = ;
12
7
P (F |H) = .
10

Dados dois eventos A e B, com P (A) 6= 0, a probabilidade condicional de Definio 1


Probabilidade
B na certeza de A o nmero
Condicional
P (A B)
P (B|A) = .
P (A)

3
Unidade 18 Probabilidade Condicional

Na realidade, poucas vezes usaremos a frmula acima para calcular uma

probabilidade condicional. Us-la-emos, isto sim, para o clculo de P (A B);


P (A B) = P (A) P (B|A).

Exemplo 3 Uma urna contm 4 bolas brancas e 6 bolas pretas. Sacam-se, sucessiva-

mente e sem reposio, duas bolas dessa urna. Determine a probabilidade de

ambas serem brancas.

Soluo. Sejam B1 = {a primeira bola branca} e B2 = {a segunda bola

branca}. Temos

4 3 2
P (B1 B2 ) = P (B1 ) P (B2 |B1 ) = = .
10 9 15

Note que foi bastante simples o clculo de P (B2 |B1 ). Realmente, na certeza de

que a primeira bola foi branca, fcil calcular a probabilidade da segunda bola

ser branca, pois, para a segunda extrao, a urna est com 3 bolas brancas e 6

pretas. De modo mais geral, fcil calcular probabilidades condicionais quando

as coisas esto na ordem certa, isto , fcil calcular probabilidades de coisas

futuras na certeza de coisas passadas.

Exemplo 4 Uma urna contm 4 bolas brancas e 6 bolas pretas. Sacam-se, sucessiva-

mente e sem reposio, duas bolas dessa urna. Determine a probabilidade da

primeira bola ser branca, sabendo que a segunda bola branca.

Soluo. Sejam B1 = {a primeira bola branca} e B2 = {a segunda bola


branca}. Queremos P (B1 |B2 ). Note que essa uma probabilidade do passado

na certeza do futuro. Aqui usamos a frmula da denio de probabilidade

condicional.
P (B1 B2 )
P (B1 |B2 ) =
P (B2 )
2
P (B1 B2 ) foi calculada no exemplo anterior e vale .
15
O clculo de P (B2 ) no imediato pois no sabemos como est a urna no

momento da segunda extrao. Para calcular P (B2 ), consideramos todas as

possibilidades quanto primeira bola. Para a segunda bola ser branca, ou a

segunda branca e a primeira foi branca, ou a segunda branca e a primeira

4
Probabilidade Condicional Unidade 18

foi preta. Isto ,

P (B2 ) = P [(B1 B2 ) (P1 B2 )]


= P (B1 B2 ) + (P1 B2 )
2
= + P (P1 ) P (B2 |P1 )
15
2 6 4
= +
15 10 9
2
=
5
Logo,
P (B1 B2 ) 2 2 1
P (B1 |B2 ) = = = .
P (B2 ) 15 5 3
Uma maneira eciente de lidar com experincias que possuem vrios est-

gios o uso das rvores de probabilidade.

Figura 18.1: rvore de probabilidade

Nesses diagramas colocamos as probabilidades condicionais da extremidade

de cada galho na certeza da origem do galho. Para determinar uma proba-

bilidade usando esse diagrama, basta percorrer todos os caminhos que levam

ao evento cuja probabilidade procurada, multiplicando as probabilidades em

cada caminho e somando os produtos ao longo dos vrios caminhos. Assim,

por exemplo,
4 3 2
P (B1 B2 ) = = ;
10 9 15

5
Unidade 18 Probabilidade Condicional

4 3 6 4 2
P (B2 ) = + =
10 9 10 9 5

Exemplo 5 Escolhe-se uma entre trs moedas. Duas dessas moedas so no-viciadas

e a outra tem duas caras. A moeda selecionada lanada e obtida uma cara.

Qual a probabilidade de ter sido selecionada a moeda de duas caras?

Figura 18.2: Moeda de duas caras

P (V C)
P (V |C) =
P (C)
1 1
P (V C) = 1 =
3 3
1 2 1 2
P (C) = 1 + =
3 3 2 3
1 2 1
P (V |C) = =
3 3 2

O exemplo a seguir mostra um dos mais poderosos mtodos de estimao

em Estatstica, o mtodo da mxima verossimilhana.

Exemplo 6 Em certa cidade, os txis so numerados de 1 a N. Para estimar o nmero

N de txis da cidade, um turista anotou os nmeros de todos os txis que pegou:

47, 12, 33 e 25. Determine a probabilidade do turista ter tomado os txis que

tm esses nmeros e determine o valor de N para o qual essa probabilidade

6
Probabilidade Condicional Unidade 18

mxima.

Soluo. Sejam A = {o primeiro txi tem nmero 47 }, B ={o segundo txi

tem nmero 12}, etc. A probabilidade pedida

P (A B C D)
= P (A) P (B|A) P [C|(A B)] P [D|(A B C)]
1 1 1 1 1
= = 4
N N N N N
Essa probabilidade de ocorrer o que efetivamente ocorreu chamada de veros-

similhana. No caso, ela mxima quando N mnimo. Ora, como N > 47,
o valor de N que torna mxima a verossimilhana 47.

A estimativa de mxima verossimilhana de N 47.

Algumas pesquisas estatsticas podem causar constrangimentos aos entre- Exemplo 7


vistados com perguntas do tipo voc usa drogas? e correm o risco de no

obter respostas sinceras ou no obter respostas de espcie alguma. Para es-

timar a proporo p de usurios de drogas em certa comunidade, pede-se ao

entrevistado que, longe das vistas do entrevistador, jogue uma moeda: se o

resultado for cara, responda a voc usa drogas? e, se o resultado for coroa,

responda a sua idade um nmero par?. Assim, caso o entrevistado diga

sim, o entrevistador no saber se ele um usurio de drogas ou se apenas tem

idade par.

Se s a probabilidade de um entrevistado responder sim, s facilmente

estimado pela proporo de respostas sim obtidas nas entrevistas. A relao

entre s e p pode ser determinada pela rvore abaixo.

Figura 18.3: Mtodo indireto de entrevista

7
Unidade 18 Probabilidade Condicional

s = P (sim) = 0, 5p + 0, 5 0, 5.

Da, p = 2s 0, 5.
Por exemplo, se 30% dos entrevistados respondem sim, voc pode estimar

em 10% a proporo de usurios de drogas.

O exemplo a seguir um interessante exemplo de probabilidade geomtrica.

Quando selecionamos um ponto ao acaso em uma parte do plano extrema-

mente razovel supor que a probabilidade do ponto selecionado pertencer a uma

certa regio seja proporcional rea dessa regio.

Exemplo 8 Selecionam-se ao acaso dois pontos em um segmento de tamanho 1,

dividindo-o em trs partes. Determine a probabilidade de que se possa for-

mar um tringulo com essas trs partes.

Soluo. Sejam x [0, 1] e y [0, 1] os pontos escolhidos, x 6 y.

Figura 18.4:

Escolher x e y pertencentes a [0, 1], com x 6 y, equivale a escolher um

ponto (x, y) no tringulo T da gura abaixo.

Figura 18.5: Como escolher os pontos x e y

8
Probabilidade Condicional Unidade 18

Para que exista um tringulo de lados x, y x e 1 y devemos ter x <


yx+1y e yx<x+1y e 1 y < x + y x, o que d x < 0, 5 e
y < x + 0, 5 e y > 0, 5. Em suma, o tringulo existir se e somente se o ponto
(x, y) for selecionado na parte sombreada do tringulo T .
Sendo A o evento as trs partes formam um tringulo e sendo S o evento

certo, temos que P (A) proporcional rea da parte sombreada e P (S) = 1

proporcional rea de T . Logo,

P (A) rea sombreada 1


P (A) = = =
P (S) rea de T 4

A e B lanam sucessivamente um par de dados at que um deles obtenha Exemplo 9


soma de pontos 7, caso em que a disputa termina e o vencedor o jogador que

obteve soma 7. Se A o primeiro a jogar, qual a probabilidade de A ser o

vencedor?

Soluo. A probabilidade de obter soma 7

6 1
=
36 6
e a de no ser soma 7
1 5
1 =
6 6
Para A ganhar, ou A ganha na primeira mo, ou na segunda, ou na terceira,
1
etc. A probabilidade de A ganhar na primeira mo . Para A ganhar na
6
segunda mo, A no pode obter soma 7 na primeira mo e B no pode obter

soma 7 na primeira mo e A deve obter soma 7 na segunda mo, o que ocorre

com probabilidade
 2
5 1

6 6
Para A ganhar na terceira mo, A no pode obter soma 7 nas duas primeiras

mos e B no pode obter soma 7 nas duas primeiras mos e A deve obter soma
7 na terceira mo, o que ocorre com probabilidade

 2
5 1
,
6 6

9
Unidade 18 Probabilidade Condicional

etc.

A probabilidade de A ganhar

 2  4 1
1 5 1 5 1 6 6
+ + + = 2 = .
6 6 6 6 6 11

5
1 6

Uma soluo mais elegante pode ser obtida ignorando as mos sem vence-
1
dores. A probabilidade de A ganhar uma mo de ; de B ganhar uma mo
6
de
5 1 5
= ,
6 6 36
pois, para B ganhar, A no pode obter soma 7 e B deve obter soma 7; a de

ningum ganhar de
5 5 25
= ,
6 6 36
pois, para que ningum ganhe, A no pode obter soma 7 e B no pode obter

soma 7.

A probabilidade A ganhar a probabilidade A ganhar em uma mo em que

houve vencedor, isto ,

1
P [A (A B)] P (A) 6 6
P (A|A B) = = = 25 = .
P (A B) P (A B) 1 36
11

Como, analogamente,

P (B)
P (A|A B) = ,
P (A B)

observe que a razo entre P (A|AB) e P (B|AB) igual razo entre P (A)
e P (B), pois P (A B) simplicado. Esse o princpio de preservao das

chances relativas. Em um jogo em que pode haver empates, e repetido at

que algum vena, a razo entre as probabilidades de vitria dos dois jogadores

igual razo de suas probabilidades de vitria em uma nica partida.

Conhecendo o princpio, poderamos ter resolvido o problema do modo se-

guinte:

Em uma mo, as probabilidades de vitria de A e de B so respectivamente

de
1 5
e de .
6 36

10
Probabilidade Condicional Unidade 18

6
A razo dessas probabilidades de . A razo das probabilidades de vitria de
5
6
A e de B no jogo tambm de e, como um dos dois ganha o jogo, a soma
5
6 5
dessas probabilidades 1. Ento, essas probabilidades so iguais a e ,
11 11
respectivamente.

11
Unidade 18 Probabilidade Condicional

Exerccios Recomendados

1. Joga-se um dado no-viciado duas vezes. Determine a probabilidade con-

dicional de obter 3 na primeira jogada, sabendo que a soma dos resultados

foi 7.

2. Um estudante resolve um teste de mltipla escolha de 10 questes, com

5 alternativas por questo. Ele sabe 60% da matria do teste. Quando

ele sabe uma questo, ele acerta, e, quando no sabe, escolhe a resposta

ao acaso. Se ele acerta uma questo, qual a probabilidade de que tenha

sido por acaso?

3. Por denio, dois eventos A e B so independentes, quando ocorre

P (A B) = P (A) P (B). Trs eventos A, B e C so independentes,


por denio, quando P (AB) = P (A)P (B), P (BC) = P (B)P (C),

P (A C) = P (A) P (C) e P (A B C) = P (A) P (B) P (C). Jogue


um dado duas vezes. Considere os eventos A = {o resultado do primeiro

lanamento par}, B = {o resultado do segundo lanamento par} e C

= {a soma dos resultados par}.

a) A e B so independentes?

b) A e C so independentes?

c) B e C so independentes?

d) A, B e C so independentes?

4. Determine a probabilidade de obter ao menos

a) um seis em 4 lanamentos de um dado;

b) um duplo seis em 24 lanamentos de um par de dados.

5. Um exame de laboratrio tem ecincia de 95% para detectar uma doena


quando ela de fato existe. Entretanto o teste aponta um resultado falso-

positivo para 1% das pessoas sadias testadas. Se 0, 5% da populao tem


a doena, qual a probabilidade de uma pessoa ter a doena, dado que

o seu exame foi positivo?

6. Quantas vezes, no mnimo, se deve lanar um dado para que a probabili-

dade de obter algum seis seja superior a 0,9?

12
Probabilidade Condicional Unidade 18

7. Em uma cidade com n+1 habitantes, uma pessoa conta um boato para

outra pessoa, a qual, por sua vez, conta o boato para uma terceira pessoa,

e assim por diante. Evidentemente ningum distrado a ponto de contar

o boato para quem lhe havia contado o boato. Determine a probabilidade

do boato ser contado k vezes:

a) sem retornar ao inventor do boato.

b) sem repetir nenhuma pessoa.

1
8. Em uma cidade, as pessoas falam a verdade com probabilidade . Supo-
3
nha que A faz uma armao e que D diz que C diz que B diz que A
falou a verdade. Qual a probabilidade de A ter falado a verdade?

9. Um prisioneiro possui 50 bolas brancas, 50 bolas pretas e duas urnas

iguais. O prisioneiro deve colocar do modo que preferir as bolas nas urnas,

desde que nenhuma urna que vazia. As urnas sero embaralhadas e o

prisioneiro dever, de olhos fechados, escolher uma urna e, nesta urna,

escolher uma bola. Se a bola for branca ele ser libertado e, se for

preta, ser condenado. Como deve agir o prisioneiro para maximizar a

probabilidade de ser libertado?

13
MA12 - Unidade 18
Probabilidade Condicional

Paulo Cezar Pinto Carvalho

PROFMAT - SBM

4 de Abril de 2014
Exemplo
Um dado honesto e lancado duas vezes.
a) Qual e a probabilidade de sair 1 no 1o lancamento?
b) Qual e a probabilidade que que tenha sado 1 no 1o
lancamento, sabendo que a soma dos resultados foi 5?
a) O espaco amostral e S = {(1, 1), . . . , (6, 6)}; ha 36 casos possveis.
O evento 1 no 1o lancamento e A = {(1, 1), . . . , (1, 6)}; ha 6
casos favoraveis.
6
A probabilidade de sair 1 no 1o lancamento e 36 = 16 .
b) Como a soma foi 5, podemos reduzir o espaco amostral a B = a
soma e 5 = {(1, 4), (2, 3), (3, 2), (4, 1)}; ha 4 casos possveis,
todos igualmente provaveis.
Sao favoraveis os casos que tambem estao em A, ou seja, os
elementos de A B = {(1, 4)}; portanto, ha apenas 1 caso
favoravel.
Logo, a probabilidade desejada e P(A|B) = n(AB) 1
n(B) = 4 .
n(AB)/n(S) P(AB)
Note que P(A|B) = n(B)/n(S) = P(B)

PROFMAT - SBM MA12 - Unidade 18, Probabilidade Condicional slide 2/11


Probabilidade Condicional

Dados dois eventos A e B, com P(B) 6= 0, a probabilidade


condicional de A na certeza de B (ou simplesmente, a
probabilidade de A dado B) e o numero

P(A B)
P(A|B) = .
P(B)

A ideia: redistribuir proporcionalmente os valores de


probabilidade, de modo a atribuir a B probabilidade igual a 1.

PROFMAT - SBM MA12 - Unidade 18, Probabilidade Condicional slide 3/11


Exemplo

A tabela abaixo da a distribuicao dos alunos de uma escola, por


sexo e por carreira pretendida.
Feminino Masculino Total
Cientfica 12% 33% 45%
Humanstica 40% 15% 55%
Total 52% 48% 100%
Um aluno da escola e escolhido ao acaso. Sejam F , M, C e H os
eventos em que o aluno selecionado e do sexo feminino, do sexo
masculino, pretende carreira cientfica e pretende carreira
humanstica, respectivamente. Calcule P(F |C ) e P(H|M).

P(F C ) 0,12 4
P(F |C ) = P(C ) = 0,45 = 15 0, 27.

P(HM) 0,15 5
P(H|M) = P(M) = 0,48 = 16 0, 31.

PROFMAT - SBM MA12 - Unidade 18, Probabilidade Condicional slide 4/11


Independencia

Da definicao de probabilidade condicional:

P(A B) = P(B)P(A|B) = P(A)P(B|A).

Esta expressao e util para calcular a probabilidade da


intersecao de dois eventos, principalmente em experimentos
realizados em etapas.
Quando P(A B) = P(A)P(B), os eventos A e B sao
independentes.
Equivale a dizer que:

P(A|B) = P(A) e P(B|A) = P(B),

(desde que P(A) 6= 0 e P(B) 6= 0).

PROFMAT - SBM MA12 - Unidade 18, Probabilidade Condicional slide 5/11


Exemplo
Uma moeda tem probabilidade 0,4 de dar cara. Se ela for
lancada 5 vezes, qual e a probabilidade de que saiam
exatamente duas caras?
Os eventos sair cara no i-esimo lancamento (i = 1, . . . , 5)
sao independentes.
A probabilidade de que saia uma dada sequencia com 2
caras(c) e 3 coroas(k) (por exemplo, cckkk) e igual ao
produto das probabilidades dos eventos correspondentes a
cada lancamento; logo, e igual a 0, 42 0, 63 .
Como ha C52 sequencias deste tipo, a probabilidade de
observarmos exatamente duas caras e
C52 0, 42 0, 63 = 0, 3456.
A probabilidade de se observarem k sucessos em n realizacoes
independentes de um experimento com probabilidade p de
sucesso e Cnk p k (1 p)nk (modelo binomial).

PROFMAT - SBM MA12 - Unidade 18, Probabilidade Condicional slide 6/11


Exemplo

Uma urna contem 4 bolas brancas e 6 bolas pretas. Sacam-se,


sucessivamente e sem reposicao, duas bolas dessa urna.
a) Qual e a probabilidade de ambas serem brancas?
b) Qual e a probabilidade de que a segunda bola seja branca?
c) Qual e a probabilidade de que a primeira bola seja branca,
sabendo que a segunda e branca?
a) Sejam B1 = {a primeira bola e branca} e B2 = {a segunda
bola e branca}.
A probabilidade pedida e:
4
P(B1 B2 ) = P(B1 ) P(B2 |B1 ) = 10 93 = 15
2
.

PROFMAT - SBM MA12 - Unidade 18, Probabilidade Condicional slide 7/11


Exemplo
Uma urna contem 4 bolas brancas e 6 bolas pretas. Sacam-se,
sucessivamente e sem reposicao, duas bolas dessa urna.
a) Qual e a probabilidade de ambas serem brancas?
b) Qual e a probabilidade de que a segunda bola seja branca?
c) Qual e a probabilidade de que a primeira bola seja branca,
sabendo que a segunda e branca?
b) Ha duas possiblidades:
A primeira e branca (B1 ) e a segunda e branca (B2 )
ou
A primeira e preta (P1 ) e a segunda e branca (B2 )
Logo: P(B2 ) = P((B1 B2 ) (P1 B2 )) =
P(B1 B2 ) + P(P1 B2 ) =
P(B1 ).P(B2 |B1 ) + P(P1 ).P(B2 |P1 ) =
4 3 6 4 36
10 . 9 + 10 . 9 = 90 = 52 .

PROFMAT - SBM MA12 - Unidade 18, Probabilidade Condicional slide 8/11


Exemplo
Uma urna contem 4 bolas brancas e 6 bolas pretas. Sacam-se,
sucessivamente e sem reposicao, duas bolas dessa urna.
a) Qual e a probabilidade de ambas serem brancas?
b) Qual e a probabilidade de que a segunda bola seja branca?
c) Qual e a probabilidade de que a primeira bola seja branca,
sabendo que a segunda e branca?
c) A probabilidade condicional pedida (P(B1 |B2 )) esta na ordem
inversa da realizacao do experimento.
P(B1 B2 ) P(B1 ).P(B2 |B1 )
P(B1 |B2 ) = P(B2 ) = P(B1 ).P(B2 |B1 )+P(P1 ).P(B2 |P1 ) =

4 3
10 . 9
4 3 6 4
=
10 . 9 + 10 . 9

12 1
12+24 = 3

PROFMAT - SBM MA12 - Unidade 18, Probabilidade Condicional slide 9/11


Arvore de Probabilidades
Sao uma forma conveniente de representar experimentos
realizados em diversos estagios.

Os valores nos ramos do primeiro estagio representam


probabilidades e os demais, probabilidades condicionais.
A probabilidade de uma sequencia de ocorrencias e obtida
multiplicando-se os valores nos ramos.
PROFMAT - SBM MA12 - Unidade 18, Probabilidade Condicional slide 10/11
Exemplo
Para estimar a proporcao p de usuarios de drogas em certa
comunidade, pede-se ao entrevistado que jogue uma moeda: se o
resultado for cara, responda a voce usa drogas? e, se o resultado
for coroa, responda a sua idade e um numero par?. Assim, caso o
entrevistado diga sim, o entrevistador nao sabera se ele e um
usuario de drogas ou se apenas tem idade par. Se 30% dos
entrevistados respoderem sim, qual e o valor estimado de p?

s = P(sim) = 0, 5p + 0, 5 0, 5.
Da, p = 2s 0, 5 = 2 0, 3 0, 5 = 0, 1.
PROFMAT - SBM MA12 - Unidade 18, Probabilidade Condicional slide 11/11
Lista de Exerccios
Unidade 18

1. Joga-se um dado nao-viciado duas vezes. Determine a probabilidade


condicional de obter 3 na primeira jogada, sabendo que a soma dos
resultados foi 7.

2. Um estudante resolve um teste de multipla escolha de 10 questoes,


com 5 alternativas por questao. Ele sabe 60% da materia do teste.
Quando ele sabe uma questao, ele acerta, e, quando nao sabe, escolhe
a resposta ao acaso. Se ele acerta uma questao, qual e a probabilidade
de que tenha sido por acaso?

3. Por definicao, dois eventos A e B sao independentes, quando ocorre


P (A B) = P (A) P (B). Tres eventos A, B e C sao independentes,
por definicao, quando P (AB) = P (A)P (B), P (BC) = P (B)P (C),
P (AC) = P (A)P (C) e P (AB C) = P (A)P (B)P (C). Jogue um
dado duas vezes. Considere os eventos A = {o resultado do primeiro
lancamento e par}, B = {o resultado do segundo lancamento e par} e
C = {a soma dos resultados e par}.
a) A e B sao independentes?
b) A e C sao independentes?
c) B e C sao independentes?
d) A, B e C sao independentes?

4. Determine a probabilidade de obter ao menos


a) um seis em 4 lancamentos de um dado;
b) um duplo seis em 24 lancamentos de um par de dados.

5. Um exame de laboratorio tem eficiencia de 95% para detectar uma


doenca quando ela de fato existe. Entretanto o teste aponta um resul-
tado falso-positivo para 1% das pessoas sadias testadas. Se 0, 5% da
populacao tem a doenca, qual e a probabilidade de uma pessoa ter a
doenca, dado que o seu exame foi positivo?

1
6. Quantas vezes, no mnimo, se deve lancar um dado para que a proba-
bilidade de obter algum seis seja superior a 0,9?

7. Em uma cidade com n + 1 habitantes, uma pessoa conta um boato


para outra pessoa, a qual, por sua vez, conta o boato para uma terceira
pessoa, e assim por diante. Evidentemente ninguem e distrado a ponto
de contar o boato para quem lhe havia contado o boato. Determine a
probabilidade do boato ser contado k vezes:
a) sem retornar ao inventor do boato.
b) sem repetir nenhuma pessoa.
1
8. Em uma cidade, as pessoas falam a verdade com probabilidade . Su-
3
ponha que A faz uma afirmacao e que D diz que C diz que B diz que
A falou a verdade. Qual a probabilidade de A ter falado a verdade?

9. Um prisioneiro possui 50 bolas brancas, 50 bolas pretas e duas urnas


iguais. O prisioneiro deve colocar do modo que preferir as bolas nas ur-
nas, desde que nenhuma urna fique vazia. As urnas serao embaralhadas
e o prisioneiro devera, de olhos fechados, escolher uma urna e, nesta
urna, escolher uma bola. Se a bola for branca ele sera libertado e, se for
preta, sera condenado. Como deve agir o prisioneiro para maximizar a
probabilidade de ser libertado?

2
Solucoes da Lista de Exerccios
Unidade 18

1. Sejam X e Y os resultados do primeiro e segundo lancamentos, respecti-


P (X = 3, X + Y = 7) 1/6 1/6
vamente. P (X = 3|X +Y = 7) = = =
P (X + Y = 7) 6/36
1
.
6
Outra solucao:
Se a soma e 2, ha 6 casos possveis igualmente provaveis: X = 1, Y = 6;
X = 2, Y = 5; X = 3, Y = 4; X = 4, Y = 3; X = 5, Y = 2;X = 6,
Y = 1. Dos seis casos, somente X = 3, Y = 3 e favoravel. A resposta
1
e .
6
2.
P (nao sabe e acerta)
P (nao sabe | acerta) =
P ( acerta)
P (nao sabe) P (acerta | nao sabe)
=
P (sabe) P (acerta | sabe) + P (nao sabe) P (acerta | nao sabe)
0, 4 0, 2 2
= =
0, 6 1 + 0, 4 0, 2 17

3. P (A B) = P (A) P (B|A) = 1/2 1/2 = P (A) P (B); logo, A e B sao


independentes.
Observe que A C = A B e que P (C) = 21 P (A C) = P (A B) =
1/2 1/2 = P (A) P (C); logo, A e C sao independentes.
Observe que B C = A B e que P (C) = 21 P (B C) = P (A B) =
1/2 1/2 = P (B) P (C); logo, B e C sao independentes.
Como A B C = A B, P (A B C) = P (A B) = 1/2 1/2, que
e diferente de P (A) P (B) P (C) = 1/2 1/2 1/2; logo, A, B e C nao
sao independentes.

1
5 4

4. (a) A probabilidade de nenhum seis em quatro lancamentos e 6

5 4
0, 4823. A probabilidade de pelo menos um seis e 1 6

1 0, 4823 = 0, 5177.
(b) A probabilidade de nenhum duplo seis em 24 lancamentos de um
35 24

par de dados e 36 0, 5086. A probabilidade de pelo menos
35 24

um duplo seis e 1 36 1 0, 5086 = 0, 4914.

5.
P (doente e positivo)
P (doente | positivo) =
P (positivo)
P (doente positivo | doente)
=
P (doente) P (positivo | doente) + P (sadio) P (positivo | sadio)
0, 005 0, 95 95
= = = 0, 3231
0, 005 0, 95 + 0, 995 0, 01 294
n
6. A probabilidade de nao obter nenhum seis em n lancamentose 56 e
5 n 5 n
de obter pelo menos um de seis e 1 6
. Devemos ter 1 6
> 0, 9,
5 n

ou seja, 6 < 0, 1.
Da,
 n
5
ln < ln 0, 1
6
5
n ln < ln 0, 1
6
ln 0, 1
n> = 12, 6
ln 56

A resposta e 13.

7. (a) Cada pessoa tem n modos de escolher a quem conta o boato. Logo,
o numero de modos de o boato ser contado m vezes e nm .
O numero de modos de o boato ser contado m vezes, sem retornar
a primeira pessoa e n(n 1)m1 , pois o primeiro ouvinte pode ser
selecionado de n modos e os demais, de n 1 modos. A resposta
m1
n1 m1
e n(n1)

nm = n
.

2
(b) Cada pessoa tem n modos de escolher a quem conta o boato. Logo,
o numero de modos de boato ser contado m vezes e nm .
Para o boato ser contado m vezes, sem repetir nenhuma pessoa,
o primeiro ouvinte pode ser selecionado de n modos; o segundo,
de n 1 modos; o terceiro, de n 2 modos; . . . ; o m-esimo,
de n (m 1) = n m + 1 modos. O numero de modos de
o boato ser contado m vezes, sem retornar a primeira pessoa, e
n!
n (n 1) (n m + 1) = .
(n m)!
n!
A resposta e .
(n m)!nm
8. Considere os eventos:
A ={A falou a verdade};
B ={B disse que A falou a verdade};
C ={C disse que B disse que A falou a verdade};
D ={D disse que C disse que B disse que A falou a verdade}.
Vamos aliviar a notacao escrevendo XY para representar X Y .
P (AD)
Queremos calcular P (A|D) = .
P (D)

P (AD) = P (ABCD) + P (ABCD) + P (ABCD) + P (ABCD)


1 1 1 1 1 2 2 1 1 1 2 2 1 2 1 2 13
= + + + =
3 3 3 3 3 3 3 3 3 3 3 3 3 3 3 3 81

P (AD) = P (ABCD) + P (ABCD) + P (ABCD) + P (ABCD)


2 2 1 1 2 1 2 1 2 2 2 2 2 1 1 2 28
= + + + = .
3 3 3 3 3 3 3 3 3 3 3 3 3 3 3 3 81

13 28 41
P (D) = P (AD) + P (AD) = + = .
81 81 81
P (AD) 13/81 13
A resposta e P (A|D) = = = .
P (D) 41/81 41

3
9. Uma urna recebe uma bola branca e a outra urna recebe as demais 99
bolas. Com efeito, se a 1a urna recebe k bolas das quais a sao brancas,
a probabilidade de libertacao e
 
1 a 50 a 1 50k + a(100 2k)
f (a, k) = + = .
2 k 100 k 2 k(100 k)

Observe que para k = 50 a expressao vale 12 , independentemente do


valor de a.
Observe tambem que basta estudar agora o caso k < 50 (isto e, po-
demos considerar a primeira urna como sendo a que recebeu menos
bolas). Nesse caso, e claro que, fixando o valor de k, quando maior for
a, maior sera f (a, k). Logo, para f (a, k) ser maximo, devemos ter a = k
1 150 2k 75 k 25
e a probabilidade sera g(k) = = = 1 , que
2 100 k 100 k 100 k
e maxima para k mnimo.
Devemos, pois, ter k = 1, o que da uma probabilidade de libertacao de
74
99 =
0, 75.

4
19
Problemas de

Probabilidade

Lista I

1
Unidade 19

Problemas
Lista 1
1. Distribuindo ao acaso 5 sorvetes de creme e 5 de chocolate a 10 pessoas,
das quais 3 preferem creme, 2 preferem chocolate e as demais no tm
preferncia, qual a probabilidade de todas sarem satisfeitas?

2. Escolhem-se ao acaso duas peas de um domin comum. Qual a pro-


babilidade delas possurem um nmero comum?

3. No jogo da quina concorrem 80 dezenas e so sorteadas 5 dezenas. Clara


apostou em 8 dezenas. Qual a probabilidade de Clara acertar:
a) 3 dezenas?
b) 4 dezenas?
c) 5 dezenas?

4. Em uma roda so colocadas n pessoas. Qual a probabilidade de duas


dessas pessoas carem juntas?

5. Uma pessoa tem um molho de n chaves, das quais apenas uma abre
a porta. Se ela vai experimentando as chaves at acertar, determine a
probabilidade dela s acertar na tentativa de ordem k , supondo:
a) que a cada tentativa frustrada ela toma a sbia providncia de descartar
a chave que no serviu.
b) supondo que ela no age como no item a).

6. H 8 carros estacionados em 12 vagas em la. Determine a probabilidade:


a) das vagas vazias serem consecutivas.
b) de no haver duas vagas vazias adjacentes.

7. Se P (A) = 0, 4, P (B) = 0, 5, P (C) = P (A B) = 0, 3, P (A C) = 0


e P (B C) = 0, 1, determine:
a) P (A B C).
b) P [A (B C)].

2
Problemas de Probabilidade Lista I Unidade 19

c) P [A (B C)].
d) P [(A B) C].

8. Em certa escola a probabilidade de um aluno ser torcedor do Flamengo


0,60, de assistir novela 0,70 e de gostar de praia 0,80. Entre
que valores est compreendida a probabilidade de um aluno dessa escola,
simultaneamente: a) assistir novela e gostar de praia. b) torcer pelo
Flamengo.

9. Laura e Telma retiram cada uma um bilhete numerado de uma urna que
contm bilhetes numerados de 1 a 100. Determine a probabilidade do
nmero de Laura ser maior que o de Telma, supondo a extrao:
a) sem reposio.
b) com reposio.

10. Em uma gaveta h 10 pilhas, das quais duas esto descarregadas. Testando-
se as pilhas uma a uma at serem identicadas as duas descarregadas,
determine a probabilidade de serem feitos:
a) cinco testes.
b) mais de cinco testes.
c) menos de cinco testes.

3
MA12 - Unidade 19
Espaco Amostral Infinito

Paulo Cezar Pinto Carvalho

PROFMAT - SBM

4 de Abril de 2014
O espaco amostral pode ser infinito
A e B lancam sucessivamente um par de dados ate que um deles
obtenha soma de pontos 7 e venca o jogo. Se A e o primeiro a
jogar, qual e a probabilidade de ser o vencedor?
6 1
A probabilidade de obter soma 7 e 36 = 6 e a de nao ser soma 7 e
1 16 = 56 .
Para A ganhar, ou A ganha na primeira mao, ou na segunda, ou na
terceira, etc.
A probabilidade de A ganhar na primeira mao e 16 .
A probabilidade de A ganhar na segunda mao e ( 65 )2 16 .
(nem A nem B podem obter soma 7 na primeira mao e A deve
obter soma 7 na segunda mao).
A probabilidade de A ganhar na terceira mao e ( 56 )4 61 , e
assim por diante.
A probabilidade de A ganhar e
1
1 5 2 1 5 4 1 6
6 + (6) 6 + (6) 6 + = = 11 .
6
1( 56 )2

PROFMAT - SBM MA12 - Unidade 19, Espaco Amostral Infinito slide 2/1
O espaco amostral pode ser infinito

Selecionam-se ao acaso dois pontos em um segmento de tamanho


1, dividindo-o em tres partes. Determine a probabilidade de que se
possa formar um triangulo com essas tres partes.

Para que exista um triangulo de lados x, y x e 1 y devemos ter


x <y x +1y
y x <x +1y
1 y < x + y x.
Isto e equivalente a
x < 0, 5
y < x + 0, 5
y > 0, 5.

PROFMAT - SBM MA12 - Unidade 19, Espaco Amostral Infinito slide 3/1
Continuacao
Escolher x e y pertencentes a [0, 1], com x 6 y , equivale a escolher
um ponto (x, y ) no triangulo T da figura abaixo.

O triangulo existira se e somente se o ponto (x, y ) satisfaz x < 0, 5,


y < x + 0, 5 e y > 0, 5, o que corresponde a parte sombreada do
triangulo T .
A probabilidade de A = as tres partes formam um triangulo e
proporcional a area da parte sombreada. Logo,
P(A) = P(A) area sombreada = 1
P(S) = area de T 4

PROFMAT - SBM MA12 - Unidade 19, Espaco Amostral Infinito slide 4/1
Lista de Exerccios
Unidade 19

1. Dois jogadores lancam sucessivamente uma moeda honesta. Ganha o


primeiro que obtiver um resultado igual ao obtido na jogada anterior.
Qual e a probabilidade de que o primeiro a lancar a moeda seja o
ganhador?

2. Arlindo tem 1 real e Bernardo tem 2. Eles combinam apostar 1 real


em sucessivas dispustas de cara e coroa, ate que um deles perca todo o
seu dinheiro. Qual e a probabilidade de que Arlindo seja o ganhador?

3. Um dado e lancado seguidamente. Qual e o numero esperado de


lancamentos ate que se observe a primeira ocorrencia de um 6?

4. Selecionam-se ao acaso dois pontos em uma circunferencia. Qual a


probabilidade da corda determinada por esses pontos ter comprimento
maior do que o lado do triangulo equilatero inscrito na circunferencia?

5. Seleciona-se ao acaso um ponto X em um diametro AB de uma circun-


ferencia. Qual a probabilidade da corda que contem X e e perpendicu-
lar a AB ter comprimento maior do que o lado do triangulo equilatero
inscrito na circunferencia?

6. Cristina e Maria, que nao sao pessoas muito pontuais, marcaram um


encontro as 16 horas. Se cada uma delas chegara ao encontro em um
instante qualquer entre 16 e 17 horas e se dispoe a esperar no maximo
10 minutos pela outra, qual e a probabilidade delas se encontrarem?

7. Dois numeros x e y sao sortados, ao acaso e independentemente, no


intervalo [0, 1] e calcula-se a sua soma s = x + y. A seguir, cada um
destes numeros e arredondado para o inteiro mais proximo, obtendo-se
os numeros inteiros x, y e s. Calcule a probabilidade de que s seja igual
a x + y.

1
Solucao da Lista de Exerccios
Unidade 19

1. A primeira chance que o primeiro jogador tem de ganhar e em seu


segundo lancamento (ou seja, no terceiro lancamento do dado). Isto
ocorre se o segundo resultado for diferente do primeiro e o terceiro
igual ao segundo, o que ocorre com probabilidade 12 21 = 14 . Para que
ele ganhe no terceiro lancamento, os resultados devem ser alternados
nos 4 primeiros lancamentos e o quinto igual ao quarto, o que ocorre
com probabilidade 12 12 21 12 = 16
1
; e assim por diante. A probabilide de
1
1
que o primeiro jogador ganhe e, portanto, 41 + 16 1 1
+ 64 +. . . = 4 1 = .
1 4 3
2. Seja p a probabilidade de que Arlindo seja o ganhador. Para tal, ele
deve ganhar a primeira disputa, o que ocorre com probabilidade p.
Se isto ocorrer, ele estara exatamente na situacao inicial de Bernardo,
tendo assim probabilidade 1 p de ganhar. Logo, temos p = 12 (1 p),
o que leva a p = 13 .
3. O primeiro 6 ocorre no primeiro lancamento com probabilidade 16 ; no
segundo, com probabilidade 56 16 ; no terceiro, com probabilidade 56 56 16 ; e
assim por diante. O valor esperado do numero de lancamentos e
1. 16 + 2. 56 16 + 3. 65 65 61 + . . . =
1 16 + 1 65 16 + 1 56 56 16 + . . . +
1 56 61 + 1 56 56 61 + . . . +
... =
1 1
6 6
65 1 5 5

6 6 6
5 + 5 + + ... =
1 6
1 6 1 56
1 + + 65 56 + . . . =
5
6
1
= 6.
1 65
4. Dois pontos A e B de um circulo determinam uma corda maior que o
lado de um triangulo equilatero inscrito se e somente se o menor arco

1
AB e maior do que 120 . Isto significa, que uma vez escolhido o ponto
A, o ponto B nao deve estar no arco de 240 com ponto medio em
A, como mostra a figura. Como B e escolhido ao acaso, admitimos
que a probabilidade de que ele esteja em um arco seja proporcional ao
120 1
comprimento do arco. Assim, a probabilidade pedida e = .
360 3

120 120

120
B

5. Uma corda perpendicular a AB tem comprimento maior do que o lado


do triangulo equilatero inscrito se e somente se corta AB entre os pontos
M e N , medios de OA e OB, onde O e centro do crculo, como mostra
MN 1
a figura. Logo, a probabilidade pedida e = .
AB 2

A B
M O N

6. Contando o tempo em minutos, a partir das 16 horas, e designando por


x e y os instantes de chagada de Cristina e Maria, a regiao possvel

2
e = {(x, y) : 0 x 60, 0 y 60} (um quadrado de lado 60) e
a regiao favoravel e A = {(x, y) : 0 x 60, 0 y 60, |x y|
10}. A desigualdade |x y| 10 e equivalente a x 10 y x + 10, o
que mostra que a regiao favoravel e uma faixa em torno da diagonal
do quadrado, como mostra a figura.

60

10

0 10 60

area de A 602 2 50 50/2 11


A probabilidade desejada e: = = .
area de 602 36
7. Dois numeros x e y sao sortados, ao acaso e independentemente, no
intervalo [0, 1] e calcula-se a sua soma s = x + y. A seguir, cada um
destes numeros e arredondado para o inteiro mais proximo, obtendo-se
os numeros inteiros x, y e s. Calcule a probabilidade de que s seja igual
a x + y.

8. Os casos em que s e igual a x + y sao:

um dos numeros (por exemplo x) e menor que 0,5 e o outro maior


que 0,5 (neste caso, x = 0, y = 1 e s = 1 ).
Ambos os numeros sao menores que 0,5 e a soma x + y e menor
que 0,5 (neste caso, x = 0, y = 0 e s = 0).
Ambos os numeros sao maiores que 0,5 e a soma x + y e maior
que 1,5 (neste caso, x = 1, y = 1 e s = 2).

Deste modo, os casos favoraveis formam a regiao sobreada da figura


abaixo e a probabilidade correspondente e igual a sua area dividida
pela area do quadrado unitario, ou seja, igual a 43 .

1
4
20
Problemas de

Probabilidade

Lista II

1
Unidade 20

Problemas
Lista 2
1. 2n jogadores de igual habilidade disputam um torneio. Eles so divididos
em grupos de 2, ao acaso, e jogadores de um mesmo grupo jogam entre
si. Os perdedores so eliminados e os vencedores so divididos novamente
em grupos de 2 e assim por diante at restar apenas um jogador que
proclamado campeo. Qual a probabilidade de dois jogadores A e B
se enfrentarem durante o torneio. Qual a probabilidade do jogador A
jogar exatamente k partidas?

2. Em um torneio como o descrito no exerccio anterior, os 16 jogadores tm


habilidades diferentes e no h surpresas nos resultados (se A melhor
que B , A vence B ).

a) Qual a probabilidade do segundo melhor jogador ser vice-campeo


do torneio?
b) Qual a probabilidade do quarto melhor jogador ser vice-campeo
do torneio?
c) Qual o nmero mximo de partidas que o dcimo melhor jogador
consegue disputar? Qual a probabilidade dele disputar esse nmero
mximo de partidas?

3. Em um programa da televiso italiana, os candidatos devem escolher uma


entre trs portas. Atrs de uma dessas portas h um prmio e atrs de
cada uma das outras duas portas h um bode. Escolhida uma porta
pelo candidato, o apresentador, que sabe onde esto os bodes, abre uma
das outras portas, atrs da qual se encontra um bode, e pergunta ao
candidato se ele quer car com a porta que escolheu ou se prefere troc-
la pela outra porta que ainda est fechada. Admitindo que, quando o
candidato escolhe a porta em que est o prmio, o apresentador escolha
ao acaso uma porta para abrir, voc acha que o candidato deve trocar,
no deve trocar ou que tanto faz?

4. Qual a probabilidade de serem obtidas exatamente 5 caras em 10 lan-


amentos de uma moeda no-tendenciosa?

2
Problemas de Probabilidade Lista II Unidade 20

5. Uma urna contm 4 bolas brancas e 6 bolas pretas. Sacam-se sucessiva-


mente bolas dessa urna de acordo com o seguinte processo: cada vez que
uma bola sacada, ela devolvida urna e so acrescentadas mais duas
bolas da mesma cor que ela. Determine a probabilidade de:
a) a segunda bola sacada ser branca.
b) a primeira bola sacada ter sido branca na certeza de que a segunda
bola sacada foi preta.

6. Um juiz de futebol meio trapalho tem no bolso um carto amarelo, um


carto vermelho e um carto com uma face amarela e uma face vermelha.
Depois de uma jogada violenta, o juiz mostra um carto, retirado do bolso
ao acaso, para um atleta. Se a face que o jogador v amarela, qual a
probabilidade da face voltada para o juiz ser vermelha?

7. A e B disputam uma srie de partidas. Ganha um prmio quem primeiro


completar 10 vitrias. A mais habilidoso do que B , sendo de 0,6 a
probabilidade de A ganhar uma partida e de 0,4 a probabilidade de B
ganhar uma partida. No momento o placar est 7 4 a favor de B . Qual
a probabilidade de A ganhar o prmio?

8. Trs jogadores, A, B e C , disputam um torneio. Os trs tm probabili-


dades iguais de ganhar o torneio; tm tambm probabilidades iguais de
tirarem o segundo lugar e tm probabilidades iguais de tirarem o ltimo
lugar. necessariamente verdadeiro que cada uma das seis ordens poss-
1
veis de classicao dos trs jogadores tem probabilidade de ocorrer?
6
Justique.

9. Selecionam-se ao acaso dois pontos em uma circunferncia. Qual a pro-


babilidade da corda determinada por esses pontos ter comprimento maior
do que o lado do tringulo equiltero inscrito na circunferncia?

10. Seleciona-se ao acaso um ponto X em um dimetro AB de uma circunfe-


rncia. Qual a probabilidade da corda que contm X e perpendicular a
AB ter comprimento maior do que o lado do tringulo equiltero inscrito
na circunferncia?

3
Unidade 20

11. Cristina e Maria, que no so pessoas muito pontuais, marcaram um


encontro s 16 horas. Se cada uma delas chegar ao encontro em um
instante qualquer entre 16 e 17 horas e se dispe a esperar no mximo
10 minutos pela outra, qual a probabilidade delas se encontrarem?

4
Lista de Exerccios
Unidade 20

1. Escolhem-se ao acaso duas pecas de um domino comum. Qual e a


probabilidade delas possurem um numero comum?

2. No jogo da quina concorrem 80 dezenas e sao sorteadas 5 dezenas.


Clara apostou em 8 dezenas. Qual a probabilidade de Clara acertar:
a) 3 dezenas?
b) 4 dezenas?
c) 5 dezenas?

3. Em uma roda sao colocadas n pessoas. Qual e a probabilidade de duas


dessas pessoas ficarem juntas?

4. Uma pessoa tem um molho de n chaves, das quais apenas uma abre
a porta. Se ela vai experimentando as chaves ate acertar, determine a
probabilidade dela so acertar na tentativa de ordem k, supondo:
a) que a cada tentativa frustrada ela toma a sabia providencia de des-
cartar a chave que nao serviu.
b) supondo que ela nao age como no item a).

5. Ha 8 carros estacionados em 12 vagas em fila. Determine a probabili-


dade:
a) das vagas vazias serem consecutivas.
b) de nao haver duas vagas vazias adjacentes.

6. Se P (A) = 0, 4, P (B) = 0, 5, P (C) = P (A B) = 0, 3, P (A C) = 0


e P (B C) = 0, 1, determine:
a) P (A B C).
b) P [A (B C)].
c) P [A (B C)].
d) P [(A B) C].

1
7. Em certa escola a probabilidade de um aluno ser torcedor do Flamengo
e 0,60, de assistir novela e 0,70 e de gostar de praia e 0,80. Entre que
valores esta compreendida a probabilidade de um aluno dessa escola,
simultaneamente: a) assistir novela e gostar de praia. b) torcer pelo
Flamengo.
8. Laura e Telma retiram cada uma um bilhete numerado de uma urna
que contem bilhetes numerados de 1 a 100. Determine a probabilidade
do numero de Laura ser maior que o de Telma, supondo a extracao:
a) sem reposicao.
b) com reposicao.
9. Em uma gaveta ha 10 pilhas, das quais duas estao descarregadas.
Testando-se as pilhas uma a uma ate serem identificadas as duas des-
carregadas, determine a probabilidade de serem feitos:
a) cinco testes.
b) mais de cinco testes.
c) menos de cinco testes.
10. 2n jogadores de igual habilidade disputam um torneio. Eles sao dividi-
dos em grupos de 2, ao acaso, e jogadores de um mesmo grupo jogam
entre si. Os perdedores sao eliminados e os vencedores sao divididos
novamente em grupos de 2 e assim por diante ate restar apenas um
jogador que e proclamado campeao. Qual e a probabilidade de dois jo-
gadores A e B se enfrentarem durante o torneio. Qual e a probabilidade
do jogador A jogar exatamente k partidas?
11. Em um torneio como o descrito no exerccio anterior, os 16 jogadores
tem habilidades diferentes e nao ha surpresas nos resultados (se A e
melhor que B, A vence B).
a) Qual e a probabilidade do segundo melhor jogador ser vice-campeao
do torneio?
b) Qual e a probabilidade do quarto melhor jogador ser vice-campeao
do torneio?
c) Qual e o numero maximo de partidas que o decimo melhor joga-
dor consegue disputar? Qual e a probabilidade dele disputar esse
numero maximo de partidas?

2
12. Em um programa da televisao italiana, os candidatos devem escolher
uma entre tres portas. Atras de uma dessas portas ha um premio e atras
de cada uma das outras duas portas ha um bode. Escolhida uma porta
pelo candidato, o apresentador, que sabe onde estao os bodes, abre uma
das outras portas, atras da qual se encontra um bode, e pergunta ao
candidato se ele quer ficar com a porta que escolheu ou se prefere troca-
la pela outra porta que ainda esta fechada. Admitindo que, quando o
candidato escolhe a porta em que esta o premio, o apresentador escolha
ao acaso uma porta para abrir, voce acha que o candidato deve trocar,
nao deve trocar ou que tanto faz?

13. Qual e a probabilidade de serem obtidas exatamente 5 caras em 10


lancamentos de uma moeda nao-tendenciosa?

14. Uma urna contem 4 bolas brancas e 6 bolas pretas. Sacam-se sucessi-
vamente bolas dessa urna de acordo com o seguinte processo: cada vez
que uma bola e sacada, ela e devolvida a urna e sao acrescentadas mais
duas bolas da mesma cor que ela. Determine a probabilidade de:
a) a segunda bola sacada ser branca.
b) a primeira bola sacada ter sido branca na certeza de que a segunda
bola sacada foi preta.

15. Um juiz de futebol meio trapalhao tem no bolso um cartao amarelo,


um cartao vermelho e um cartao com uma face amarela e uma face
vermelha. Depois de uma jogada violenta, o juiz mostra um cartao,
retirado do bolso ao acaso, para um atleta. Se a face que o jogador
ve e amarela, qual e a probabilidade da face voltada para o juiz ser
vermelha?

16. A e B disputam uma serie de partidas. Ganha um premio quem pri-


meiro completar 10 vitorias. A e mais habilidoso do que B, sendo de
0,6 a probabilidade de A ganhar uma partida e de 0,4 a probabilidade
de B ganhar uma partida. No momento o placar esta 7 4 a favor de
B. Qual e a probabilidade de A ganhar o premio?

17. Tres jogadores, A, B e C, disputam um torneio. Os tres tem probabi-


lidades iguais de ganhar o torneio; tem tambem probabilidades iguais
de tirarem o segundo lugar e tem probabilidades iguais de tirarem o

3
ultimo lugar. E necessariamente verdadeiro que cada uma das seis or-
1
dens possveis de classificacao dos tres jogadores tem probabilidade
6
de ocorrer? Justifique.

18. Distribuindo ao acaso 5 sorvetes de creme e 5 de chocolate a 10 pessoas,


das quais 3 preferem creme, 2 preferem chocolate e as demais nao tem
preferencia, qual e a probabilidade de todas sarem satisfeitas?

4
Solucoes da Lista de Exerccios
Unidade 20

1. As pecas do domino sao formadas por dois, nao necessariamente dis-


tintos, dos numeros 0, 1, 2, 3, 4, 5 e 6. Ha CR72 = C82 = 28 pecas e
2
ha C28 modos de selecionar duas pecas de um domino. Para selecionar
duas pecas com um numero comum , deve-se primeiramente selecionar
o numero comum (7 possibilidades) e, depois, selecionar 2 das 7 pecas
que contem esse numero comum (C72 possibilidades).
7C72 7
A resposta e 2 = .
C28 18
5
2. O numero de sorteios possveis e C80 .

(a) O apostados acerta 3 dezenas quando sao sorteadas das 8 dezenas


em que apostou e 2 das 72 em que nao apostou. Tais sorteios
podem ser efetuados de C83 C722
modos.
3 2
C8 C72 1
A resposta e 5
(que e aproximadamente igual a ).
C80 168
(b) O apostados acerta 4 dezenas quando sao sorteadas 4 das 8 dezenas
em que apostou e 1 das 72 em que nao apostou. Tais sorteios
podem ser efetuados de C84 C721
modos.
4 1
C8 C72 1
A resposta e 5
(que e aproximadamente igual a ).
C80 4770
(c) O apostados acerta 5 dezenas quando sao sorteadas 5 das 8 dezenas
em que apostou. Tais sorteios podem ser efetuados de C85 modos.
C85 1
A resposta e 5 = .
C80 429.286
3. Colocada a primeira pessoa na roda, ha n 1 posicoes pra a segunda
pessoa, das quais 2 sao favoraveis a que ela fique junto da primeira
pessoa.
2
A resposta e .
n1

1
4. (a) Ha n posicoes igualmente provaveis que e chave certa poderia
ocupar: ser a primeira a ser testada, a segunda, . . . , a ultima. A
probabilidade de elas ocupar a k-esima posicao e 1/n.
Outra solucao:
Ha n! maneiras de ordenar as chaves a serem tentadas. Para
formar as ordenacoes que tem a chave na k-esima posicao, devemos
colocar as n 1 chaves restantes nas n 1 posicoes restantes, o
que pode ser feito de (n 1)! modos. Logo, a probabilidade de
(n 1)! 1
que a chave certa esteja na posicao k e = .
n! n
(b) As primeiras k tentativas podem ser feitas de nk modos (cada
chave pode ser escolhida de n modos, ja que chaves correspon-
dentes a tentativas frustadas nao sao descartadas). Para que se
acerte na k-esima tentativa, as primeiras k 1 chaves devem ser
incorretas (portanto, podem ser escolhidas de (n 1)k1 modos) e
a de ordem k deve ser a correta (1 modo). Logo, a probabilidade
(n 1)k1
de se acertar na k-esima tentativa e .
nk
4
5. (a) Ha C12 = 495 modos de selecionar as 4 vagas que nao serao ocu-
padas e 9 modos de escolher 4 vagas consecutivas (1 2 3 4, 2 3 4
5, . . . , 9 10 11 12).
9 1
A resposta e = .
495 55
4
(b) Ha C12 = 495 modos de selecionar as 4 vagas que nao serao ocupa-
das. Para contar o numero de possibilidades em que nao ha vagas
vazias adjacentes, devemos escolher 4 dos 9 espacos existentes an-
tes, entre e depois dos carros para ficarem vazios, Isto pode ser
feito de C94 modos. Logo a probabilidade de que nao haja vagas
126 14
consecutivas e = .
495 55
6. P (A B C) = 0, pois A B C A C e P (A C) = 0.
(a)
P (A B C) =P (A) + P (B) + P (C) P (A B)
P (A C) P (B C) + P (A B C)
=0, 4 + 0, 5 + 0, 3 0, 3 0 0, 1 + 0 = 0, 8

2
(b)

P [A (B C)] =P (A) P [A (B C)]


=P (A) P [(A B) (A C)]
=P (A) P (A B) P (A C)
+ P [(A B) (A C)]
=P (A) P (A B) P (A C)
+ P [(A B C)]
=0, 4 0, 3 0 + 0 = 0, 1

(c)

P [(A B) C] =P (A B) + P (C) P (A B C)
=0, 3 + 0, 3 0 = 0, 6

7. (a) A probabilidade de que um aluno assista novela (N ) e goste de


praia(P ) e no maximo 0,70 (ja que P (N P ) P (N ) = 0, 7). A
probabilidade mnima ocorre quando P (N P ) = 1; neste caso,
P (N P ) = P (N ) + P (P ) 1 = 0, 3.
(b) A probabilidade de que um aluno assista novela (N ), goste de
praia(P ) e torca pelo Flamengo e no maximo 0,6 (ja que P (N
P N ) P (F ) = 0, 6). A probabilidade mnima ocorre quando
P (N P F ) = 1 e, alem disso, P (N (P F )) = P (P
(N F )) = P (F (N P )) = 0. Neste caso, P (N P F ) =
P (N ) + P (P ) + P (F ) 2 = 0, 1.

8. (a) A resposta, naturalmente, e 1/2, ja que, de todos os pares de


numeros distintos de 1 a 100, em exatamente a metade o primeiro
numero e maior do que o segundo.
(b) O numero total de possveis extracoes e 100100 = 10.000, ja que
o bilhete de cada uma das mocas pode ser escolhido de 100 modos.
Em 100 destas possveis extracoes os dois numeros sao iguais e em
metade das restantes, ou seja, em 9900/2 = 4950 delas, o primeiro
numero e maior do que o segundo. Logo, a probabilidade de o
4950
numero de Laura ser maior do que o de Telma e = 0, 495.
10000

3
9. (a) Sao feitos 5 testes quando uma das quatro primeiras pilhas tes-
tadas esta descarregada, o mesmo ocorrendo com a quinta a ser
testada. A primeira pilha a ser testada pode ser escolhida de 10
modos, a segunda de 9, e assim por diante, para um total de
10 9 8 7 6 modos possveis para escolher as 5 primeiras pi-
lhas a serem testadas. Para formar uma sequencia de teste em
que a segunda defeituosa e detectada na 5a tentativa, devemos
escolher a pilha defeituosa que aparece na 5a posicao (2 modos),
a posicao da outra defeituosa (4 modos) e, finalmente, as pilhas
nao defeituosas para as demais posicoes (8 7 6 modos). Logo, a
24876 4
probabilidade pedida e = .
10 9 8 7 6 45
(b) Sao efetuados ate 5 testes quando as pilhas defeituosas aparecem
nas 5 primeiras tentativas. Como visto no tem anterior, ha 10 9
8 7 6 modos de se fazer esta tentativa. Para formar aquelas em
que as duas defeituosas estao entre as testadas devemos escolher
a posicao da primeira pilha defeituosa (5 modos), a da segunda
(4 modos) e, finalmente, as pilhas nao defeituosa para as outras
tentativas (876 modos). A probabilidade de que sejam feitos ate
54876 2
5 testes e = e, portanto, a probabilidade pedida
10 9 8 7 6 9
2 7
e igual a 1 = .
9 9
(c) Para que sejam feitos menos de 4 testes, as duas pilhas defeituosas
devem aparecer nos primeiros 4 testes. O numero total de escolhas
para os 4 primeiros testes e 10 9 8 7. Para formar uma sequencia
de teste em que as duas defeituosas aparecem nestas 4 tentativas,
devemos escolher a posicao da primeira pilha defeituosa (4 modos)
a da segunda (3 modos) e, finalmente, as pilhas nao defeituosas
para as duas outras posicoes (87 modos). A probabilidade pedida
4387 2
e =
10 9 8 7 15
1
10. (a) A probabilidade de eles se enfrentarem na primeira ronda e
2n 1
porque, posto A na tabela, ha 2n 1 posicoes possveis para B e
em 1 delas ele enfrenta B A probabilidade deles se enfrentarem

4
!2
2 1 1 1
na segunda rodada e n = n , porque posto
2 1 2 2 1 2
n
A na tabela, ha 2 1 posicoes possveis para B e em 2 delas
ele pode vir a enfrentar B na segunda rodada, o que ocorre com
probabilidade 21 12 . A probabilidade de eles se enfrentarem na
!4
22 1 1 1
terceira rodada e n = n 2 , etc.
2 1 4 2 1 2
A resposta e
1 1 1 1 1 1 1
+ n + n 2 + + n n1
2n
1 2 1 2 2 1 2 2 1 2
1 n
1 1 2 1
= n 1 = n1 .
2 1 1 2 2

(b) Se k < n, o jogador disputa exatamente k partidas se e somente


se perde a k-esima partida e ganha as k 1 partidas anteriores.
k1 1
A probabilidade de isso acontecer e 12 2 = 21k . O jogador
disputa n partidas - ou seja, chega a final - se e somente se ganha
as n 1 partidas anteriores. A probabilidade de isso acontecer e
1 n1 1
2
= 2n1 .
1
A resposta e 21k , se k < n; 2n1 , se k = n.
11. (a) O segundo jogador de melhor resultado sera vice-campeao se e
somente se nao enfrentar o melhor jogador antes da final. Posto o
melhor jogador na tabela, ha 15 posicoes possveis para o segundo
melhor e em 8 delas ele enfrenta o melhor jogador apenas na final.
8
A resposta e .
15
o
(b) Posto o 4 colocado na tabela, os demais times podem ser coloca-
dos de 15! modos. Para que o 4o melhor time seja vice-campeao,
os 3 melhores times nao podem entrar em sua chave. As posicoes
destes times podem portanto, ser escolhidas de 8, 7 e 6 modos,
respectivamente. Para distribuir os 12 times restantes, ha 12! pos-
8 7 6 12! 8
sibilidades. Logo, a probabilidade desejada e = .
15! 65
o
(c) Na primeira rodada, ha 6 adversarios que o 10 time consegue
derrotar. Na melhor das hipoteses, ele derrota um destes e, dos

5
outros 5, dois conseguem sobreviver para a proxima fase. De novo,
na melhor das hipoteses o 10o enfrenta (e vence) um deles, mas o
outro sera fatalmente eliminado. Assim, na 3a rodada, o 10o time
joga e perde. Logo, ele disputa no maximo tres partidas. Isto
ocorre quando os tres times da sua chave para os dois primeiros
jogos sao todos de habilidade inferior. O numero de modos de
escolher 3 adversarios e 151413. O numero de modos de escolher
tres adversarios entre os 6 de nvel inferior e 654. Logo, a resposta
654 4
e = .
15 14 13 91
12. Se o candidato nao troca de porta, ele ganha o premio se e so se escolhe,
originalmente, a porta certa. Logo, se ele nao troca, sua probabilidade
de ganhar o primeiro premio e igual a 13 . Em contraste, ao trocar de
porta ele ganha o premio sempre que escolheu originalmente a porta
errada, o que ocorre com probabilidade 32 . Portanto, ele deve trocar de
porta.

13. Cada um dos 10 resultado pode ser escolhido de 2 modos. Portanto,


ha 210 resultados possveis. Para formar um resultado com 5 caras, e
necessario escolher 5 dos 10 lancamentos para estas caras ocorrerem, o
5
5
C10 63
que pode ser feito de C10 modos. A probabilidade pedida e 10 = .
2 256
14. (a)

P (2a B) = P (1a B,a B) + P (1a P, 2a B)


= P (1a B) P (2a B|1a B) + P (1a P ) P (2a B|1a P )
4 6 6 4 2
= + =
10 12 10 12 5
(b)

P (1a B, 2a P )
P (1a B|2a P ) =
P (2a P )
P (1a B) P (2a P |1a B)
=
P (1a B) P (2a P |1a B) + P (1a P ) P (2a P |1a P )
4
6 1
= 4 610 126 8 = .
+ 10 12
10 12
3

6
P (ve vermelha e mostra amarela)
15. P (ve vermelha | mostra amarela) = P (mostra amarela)

1/6 1
= = .
1/2 3
16. O jogador A ganha o primeiro se e somente se B ganhar no maximo 2
das proximos 8 partidas (caso contrario, B tera sua 10a vitoria antes
de A completar sua serie de 10 vitorias).
P (B ganhar 0 partidas) = 0, 68
= 0, 0168
P (B ganhar 1 partidas) = 8 0, 67 0, 4
= 0, 0896
P (B ganhar 2 partidas) = C 2 0, 66 0, 42
8 = 0, 2090
A probabilidade de que A ganhe o premio e aproximadamente igual a
0, 0168 + 0, 0896 + 0, 2090 = 0, 3154.

17. Sejam p1 , p2 , p3 , p4 , p5 , p6 as probabilidades das possveis ordenacoes


ABC, BCA, CAB, ACB, CBA, BAC. As condicoes dadas no pro-
blema permitem escrever um sistema de equacoes lineares envolvendo
aquelas probabilidade:

1
P1 + p 4 = P2 + p 6 = P3 + p 5 =
3
1
P 3 + p 6 = P1 + p 5 = P2 + p 4 =
3
1
P 2 + p 5 = P3 + p 4 = P1 + p 6 =
3

Resolvendo o sistema, verifica-se que ele tem uma infinidade de solucoes


da forma p1 = p2 = p3 = 16 p4 = 16 p5 = 15 p6 .
Em termos mais intuitivos, basta que as ordenacoes correspondente a
mesma ordem circular tenham probabilidades iguais, ou seja, devemos
ter:
P (ABC) = P (BCA) = P (CAB)
e
P (ACB) = P (CBA) = P (BAC).

7
Por exemplo, se P (ABC) = P (BCA) = P (CAB) = 14 e P (ACB) =
1
P (CBA) = P (BAC) = 12 , os tres jogadores tem a mesma chance
de ficar em primeiro, segundo ou terceiro lugar, embora as diferen-
cias ordenacoes possveis nao tenham todas a mesma probabilidade de
ocorrer.

18. Para distribuir os sorvetes, devemos escolher as pessoas que receberao


5
sorvetes de creme (C10 modos) e dar sorvetes de chocolate as demais
(1 modo). Para distinguir os sorvetes, respeitando as preferencias,
comecamos dando sorvete de creme aos que gostam de creme e de
chocolate aos que gostam de chocolate (1 modo). Em seguida, deve-
mos distribuir 2 sorvetes de creme e 3 sorvetes de chocolate a 5 pessoas
que nao tem preferencias; para isso, devemos escolher as 2 pessoas que
receberao sorvetes de creme (C52 modos) e dar sorvetes de chocolate as
restantes (1 modo).
C52 5
A resposta e 5
= .
C10 126

8
21
Mdias e
Gavetas IPrincpio das
Sumrio
21.1 Introduo . . . . . . . . . . . . . . . . . . . . . . . 2

21.2 Mdias . . . . . . . . . . . . . . . . . . . . . . . . . 2

1
Unidade 21 Introduo

21.1 Introduo

Nessa unidade, so apresentadas algumas noes de mdias, como as mdias

aritmticas, geomtricas, harmnicas, quadrticas e ponderadas.

Em seguida, apresentado, como aplicao de clculo de mdias, o importante

Princpio das Gavetas de Dirichlet que possui inmeras aplicaes e que se

enuncia como se segue:

Se n + 1 ou mais objetos so colocados em n ou menos gavetas, ento pelo


menos uma gaveta recebe mais de um objeto.

Esse princpio, tambm chamado de Princpio da Casa dos Pombos (consegue

imaginar por qu?), tem inmeras aplicaes, algumas das quais surpreendentes.

Voc achar algumas delas na lista de exerccios propostos no nal da unidade.

21.2 Mdias

Uma ideia bastante importante a ideia de mdia. Uma mdia de uma lista de

nmeros um valor que pode substituir todos os elementos da lista sem alterar

uma certa caracterstica da lista. Se essa caracterstica a soma dos elementos

da lista, obtemos a mais simples de todas as mdias, a mdia aritmtica. A

mdia aritmtica (simples) da lista de n nmeros x1 , x2 , . . . , xn um valor x


tal que x1 + x2 + + xn = x + x + + x = nx. Portanto, temos a seguinte
denio:

Definio 1 A mdia aritmtica (simples) da lista de n nmeros x1 , x2 , . . . , xn denida


Mdia Aritmtica
por
x1 + x2 + + xn
x = .
n

Por exemplo, a mdia aritmtica dos nmeros 3, 36 e 54

3 + 36 + 54
= 31.
3
Se a caracterstica a ser considerada for o produto dos elementos da lista,

obteremos a mdia geomtrica. A mdia geomtrica (simples) dos n nmeros

2
Mdias e Princpio das Gavetas I Unidade 21

positivos x1 , x2 , . . . , xn um valor positivo g tal que x1 x2 xn = g g g =


n
g . Portanto, temos a seguinte denio:

A mdia geomtrica (simples) dos n nmeros positivos x1 , x2 , . . . , xn Definio 2


Mdia Geomtrica
denida por

g = G(x1 , x2 , . . . , xn ) = n
x1 x2 . . . xn .

Observe que s denimos a mdia geomtrica para nmeros positivos. Assim

evitamos a possibilidade da mdia no existir (por exemplo, qual seria a mdia

geomtrica entre 2 e 2?).



3
Por exemplo, a mdia geomtrica dos nmeros 3, 36 e 54 3 36 54 =
18.
Se a caracterstica for a soma dos inversos dos elementos da lista, obteremos

a mdia harmnica. A mdia harmnica (simples) dos n nmeros positivos

x1 , x2 , . . . , xn um valor h tal que

1 1 1 1 1 1 n
+ + + = + + + = .
x 1 x2 xn h h h h

Portanto, temos a seguinte denio:

A mdia harmnica (simples) dos n nmeros positivos x1 , x2 , . . . , xn Definio 3


Mdia Harmnica
denida por
n
h= 1 1 1 .
x1
+ x2
+ + xn

A mdia harmnica , pois, o inverso da mdia aritmtica dos inversos dos

nmeros.

Por exemplo, a mdia harmnica dos nmeros 3, 36 e 54

3 3 3 108 324
1 1 1 = 36+3+2 = = = 7, 9.
3
+ 36
+ 54 108
41 41

Observe que s denimos a mdia harmnica para nmeros positivos. Assim

evitamos a possibilidade da mdia no existir (por exemplo, qual seria a mdia

harmnica entre 2 e 2?).

3
Unidade 21 Mdias

Exemplo 1 Uma empresa produziu, durante o primeiro trimestre do ano passado, 500,

200 e 200 unidades, em janeiro, fevereiro e maro, respectivamente. Qual foi a

produo mdia mensal nesse trimestre?

Comentrio. Resista tentao de tirar rapidamente a mdia aritmtica e

ponto nal. Voc sempre corre o risco de um aluno perguntar porque no

podia ter tirado a mdia geomtrica.

Soluo. Que mdia essa que queremos? Queremos uma mdia M tal que,

se a produo mensal fosse sempre igual a M , a produo trimestral seria a


mesma. A produo trimestral foi de 500 + 200 + 200. Se em todos os meses

a produo fosse igual a M , a produo trimestral seria igual a 3M . Logo,

3M = 500 + 200 + 200 e


500 + 200 + 200
M= = 300.
3
A mdia desejada era a mdia aritmtica.

Resposta: 300.

Exemplo 2 Uma empresa aumentou sua produo durante o primeiro bimestre do ano

passado. Em janeiro e em fevereiro, as taxas de aumento foram de 21% e 8%,

respectivamente. Qual foi a taxa mdia de aumento mensal nesse bimestre?

Comentrio. A resposta no (21% + 8%) 2 = 14, 5%.


Soluo. Que mdia queremos? Queremos uma taxa mdia i tal que, se em

todos os meses a taxa de aumento fosse igual a i, o aumento bimestral seria o

mesmo. O aumento bimestral foi de 30,68%, conforme mostra o esquema

100 7 100 1, 21 7 100 1, 21 1, 08 = 130, 68.


Se em todos os meses tivssemos um aumento de taxa i, teramos

100 7 100(1 + i) 7 100(1 + i)2 .


Ento,

100(1 + i)2 = 100 1, 21 1, 08


(1 + i)2 = 1, 21 1, 08
1, 21 1, 08
p
1+i = = 1, 1432
i
= 0, 1432 = 14, 32%.

4
Mdias e Princpio das Gavetas I Unidade 21

A mdia procurada era uma mdia geomtrica. Mais precisamente: a taxa

mdia, aumentada de uma unidade, a mdia geomtrica das taxas mensais

aumentadas de uma unidade.

Um concurso anual distribui igualmente entre os vencedores um prmio Exemplo 3


total de R$ 1 800,00. Nos ltimos trs anos houve 2, 1 e 3 premiados, respec-

tivamente. Qual foi o prmio mdio desses ganhadores?

Comentrio. Embora o nmero mdio de ganhadores tenha sido igual a 2, o

prmio mdio no foi de R$ 1800, 00 2 = R$ 900, 00.


Soluo. Queremos uma mdia tal que, se todos os prmios fossem iguais

a essa mdia, o total distribudo seria o mesmo. Essa precisamente a mdia

aritmtica. Os prmios foram de 18002 = 900, 18001 = 1800 e 18003 =


600. O prmio mdio foi de (900 + 1800 + 600) 3 = 1100 reais.
Observe que a mdia aritmtica dos rateios igual a

1800 21 + 1800 11 + 1800 1


3
3
1
2
+ 11 + 13
= 1800
3
3
= 1800 1 1 1
2
+1+3
e que
3
1
2
+ 11 + 1
3
a mdia harmnica dos nmeros de ganhadores.

O rateio mdio o rateio que corresponderia a uma quantidade de ganha-

dores igual mdia harmnica dos nmeros de ganhadores.

Outra mdia importante a mdia quadrtica.

A mdia quadrtica dos nmeros x1 , x2 , . . . , xn denida por Definio 4


r Mdia quadrtica
x21 + x22 + + x2n
q= ,
n
isto , a mdia quadrtica a raiz quadrada da mdia aritmtica dos quadrados

dos nmeros.

5
Unidade 21 Mdias

Por exemplo, a mdia quadrtica dos nmeros 1 e 7

r
12 + 72
= 5.
2

Exemplo 4 A qualidade de uma aproximao medida pelo seu erro, que a diferena

entre o valor da aproximao e o valor real da grandeza. Por exemplo, 4 uma

aproximao de 3,8 com erro de 0,2 (tambm se diz uma aproximao de 3,8

por excesso, com erro de 0,2) e 5,5 uma aproximao de 5,7 com erro de

0, 2 (ou uma aproximao de 5,7 por falta, com erro de 0,2). Evidentemente,

quanto mais prximo de zero estiver o erro, tanto melhor ser a aproximao.

Assim, por exemplo, 39 uma aproximao de 40 (erro igual a 1) que melhor


do que a aproximao 42 (erro igual a 2).

Mede-se a qualidade de uma lista de aproximaes pela mdia quadrtica

dos seus erros. Tambm se usa o erro mdio quadrtico , que o quadrado

dessa mdia quadrtica, ou seja, a mdia aritmtica dos quadrados dos erros.

Abaixo temos duas listas de aproximaes do nmero 4:

S1 : 3; 4, 5; 3, 6 S2 : 3, 2; 4, 8
Os erros mdios quadrticos so respectivamente iguais a

12 + 0, 52 + 0, 42 0, 82 + 0, 82
= 0, 47 e = 0, 64.
3 2
S1 uma lista de aproximaes de 4 que melhor do que S2 .

Uma importante propriedade da mdia aritmtica :

Se a mdia aritmtica dos nmeros x1 , x2 , . . . , xn igual a x, pelo menos


um dos nmeros x1 , x2 , . . . , xn maior que ou igual a x.
Com efeito, se fosse x1 < x, x2 < x, , xn < x, teramos

x 1 + x2 + + xn
x1 + x2 + + xn < nx, < x, x < x,
n
o que absurdo.

Exemplo 5 Mostre que, em um grupo de 50 pessoas, h sempre pelo menos 5 que

nasceram no mesmo ms.

Soluo. O nmero mdio de pessoas por ms 50 12 = 4, 1 . . .. Logo, em

6
Mdias e Princpio das Gavetas I Unidade 21

algum ms o nmero de nascidos nesse ms (que um inteiro) maior que ou

igual a 4,1..., ou seja, maior que ou igual a 5.

Uma consequncia imediata do exemplo 5 o Princpio das Gavetas de


1
Dirichlet :

Se n + 1 ou mais objetos so colocados em n ou menos gavetas, ento pelo Proposio 5


Princpio das Gavetas
menos uma gaveta recebe mais de um objeto.

Prova. O nmero mdio de objetos por gaveta maior que ou igual a


Demonstrao
n+1
, que maior que 1. Logo, em alguma gaveta haver um nmero de
n
objetos maior que 1.

Mostre que todo inteiro positivo n tem um mltiplo que se escreve apenas Exemplo 6
com os algarismos 0 e 1.

Soluo. Considere os n+1 primeiros nmeros da sequncia 1, 11, 111,... .

Divida-os por n e considere os restos dessas divises. Esses restos s podem

ser iguais 0, 1, 2, . . . , n 1.
Pensando nos nmeros como objetos e nos restos como gavetas, temos mais

objetos do que gavetas. O Princpio das Gavetas assegura que alguma gaveta

receber mais de um objeto, isto , h dois nmeros na sequncia que do o

mesmo resto quando divididos por n, digamos 11 ... 1 ( p algarismos) e 11 ...

1 (q algarismos), p < q . A diferena desses nmeros um mltiplo de n e se

escreve 11 . . . 10 . . . 0, com p algarismos 0 e q p algarismos 1.

Cinco pontos so tomados sobre a superfcie de um quadrado de lado 2. Exemplo 7


Mostre que h dois desses pontos tais que a distncia entre eles menor que

ou igual a 2.
Soluo. Divida o quadrado de lado 2 em quatro quadrados de lado 1, ligando

os pontos mdios dos lados opostos. Pensando nos pontos como objetos e nos

quadrados como gavetas, temos mais objetos do que gavetas. O Princpio das

1 Peter Gustav Lejeune Dirichlet (1805-1859), matemtico alemo.

7
Unidade 21 Mdias

Gavetas assegura que alguma gaveta receber mais de um objeto, isto , haver

dois pontos no mesmo quadrado de lado 1. A distncia entre esses pontos no



mximo igual ao comprimento da diagonal do quadrado, que 2.

Exemplo 8 Um enxadrista, durante 11 semanas, joga pelos menos uma partida por dia

mas no joga mais de 12 partidas por semana. Mostre que possvel achar

um conjunto de dias consecutivos durante os quais ele jogou exatamente 20

partidas.

Soluo. Em 11 semanas temos 77 dias. Chamemos de Sk , k = 1, 2, . . . , 77, o


nmero de partidas jogadas desde o primeiro at o k -simo dia, inclusive. Como
ele joga pelo menos uma partida por dia, temos 1 6 S1 < S2 < < S77 .
Alm disso, S77 6 132 pois ele no joga mais de 12 partidas por semana.
Denindo S0 = 0, a quantidade de partidas jogadas do dia p ao dia q ,

inclusive, igual a Sq Sp1 . Queremos mostrar que possvel determinar p e

q de modo que Sq Sp1 = 20.


Considere os 154 nmeros

S1 , S2 , . . . , S77 , S1 + 20, S2 + 20, . . . , S77 + 20.

Eles pertencem a {1, 2, . . . , 152}. O Princpio das Gavetas assegura que

dois desses nmeros so iguais. Como S1 < S2 < < S77 , os nmeros iguais

devem estar em metades diferentes dessa lista de 154 nmeros. Ento existem

m e n tais que Sm = Sn + 20. O enxadrista joga 20 partidas entre os dias n+1


e m, inclusive.

Finalmente, denimos mdias ponderadas.

Definio 6 A mdia aritmtica ponderada dos nmeros x1 , x2 , . . . , xn com pesos res-


Mdia Ponderada
pectivamente iguais a p1 , p2 , . . . , pn denida por

p 1 x1 + p 2 x2 + + p n xn
.
p1 + p 2 + + pn

Embora a ideia primitiva seja que a mdia aritmtica ponderada uma mdia

aritmtica simples de uma lista de nmeros dos quais p1 so iguais a x1 , p 2 so

8
Mdias e Princpio das Gavetas I Unidade 21

iguais a x2 , . . . , p n so iguais a xn , no h problema em considerar pesos no

inteiros.

Alis, bastante til trabalhar com pesos relativos e considerar a mdia arit-

mtica ponderada dos nmeros x1 , x2 , . . . , xn , com pesos iguais a p1 , p2 , . . . , pn ,


respectivamente, como sendo

p1
x1
p1 + p2 + + pn
p2
+ x2
p1 + p2 + + pn
pn
+ + xn .
p1 + p2 + + pn
Assim, uma mdia aritmtica ponderada dos nmeros x1 , x2 , . . . , xn uma

expresso da forma 1 x1 + 2 x2 + + n xn , onde

1 + 2 + + n = 1.

Em um grupo de pessoas, 70% das pessoas so adultos e 30% so crianas. Exemplo 9


O peso mdio dos adultos 70kg e o peso mdio das crianas de 40kg. Qual

o peso mdio do grupo?

Soluo. a mdia aritmtica ponderada dos dois subgrupos, com pesos

relativos de 0,7 e 0,3. A resposta 0, 7 70 + 0, 3 40 = 61kg.

9
Unidade 21 Mdias

Exerccios recomendados

1. Um carro percorre metade de certa distncia d com velocidade v1 e per-

corre a outra metade com velocidade v2 . Qual a sua velocidade mdia?

2. Um carro tem velocidade v1 durante metade do tempo t de percurso e tem


velocidade v2 durante a outra metade do tempo. Qual a sua velocidade

mdia?

3. A populao de um pas cresceu 44% em uma dcada e cresceu 21%

na dcada seguinte. Qual , aproximadamente, a taxa mdia decenal de

crescimento nesses 20 anos?

4. No problema anterior, qual a taxa mdia anual de crescimento nesses 20

anos?

5. A valorizao mensal das aes de certa empresa nos quatro primeiros

meses do ano foi de +25%, +25%, 25% e 25%. Qual a valorizao

total e qual a valorizao mdia mensal nesse quadrimestre?

6. Em uma cela h trs tneis. Um conduz liberdade em 3 horas; outro,

em 5 horas, e o ltimo conduz ao ponto de partida depois de 9 horas.

Qual o tempo mdio que os prisioneiros que descobrem os tneis gastam

para escapar?

7. Suponha que, no problema anterior, os prisioneiros que entram pelo ter-

ceiro tnel, quando voltam ao ponto de partida, no se lembram de qual

foi o tnel em que entraram e, portanto, escolhem para a prxima tenta-

tiva um entre os trs tneis.

8. Prove que a mdia aritmtica x de uma lista de nmeros satisfaz m 6 x 6


M, onde m e M so respectivamente o menor e o maior dos nmeros.

9. Prove que a mdia geomtrica g de uma lista de n nmeros positivos

satisfaz m 6 g 6 M, onde m e M. so respectivamente o menor e o

maior dos nmeros.

10
Mdias e Princpio das Gavetas I Unidade 21

10. Prove que a mdia harmnica h de uma lista de n nmeros positivos

satisfaz m 6 h 6 M, onde m e M. so respectivamente o menor e o

maior dos nmeros.

11. Em um concurso, havia apenas provas de Portugus e Matemtica. O

resultado do concurso est no quadro abaixo.

Candidato Port. Mat. Classicao

Joo 5 7 2
Pedro 6 4 1
Jos 2 5 4
Paulo 4 1 3

Joo achou que havia erro na classicao porque zera mais pontos

que Pedro e classicara-se atrs dele. Houve necessariamente erro na

classicao?

12. Pneus novos duram 40 000 km, quando usados nas rodas dianteiras, e

duram 60 000 km, quando usados nas rodas traseiras.

a) Com 4 pneus novos e fazendo um rodzio adequado entre eles, quan-

tos quilmetros um carro pode rodar? Como?

b) E com 5 pneus novos? Como?

c) A resposta do item a) uma mdia entre 40 000 km e 60 000 km.

Qual?

13. A mdia aritmtica de 50 nmeros 40. Se dois desses nmeros, 125 e 75,

forem suprimidos, qual ser a mdia aritmtica dos nmeros restantes?

14. Qual a caracterstica conservada pela mdia quadrtica?

15. Prove que a mdia quadrtica q de uma lista de n nmeros positivos

satisfaz m 6 q 6 M, onde m e M so respectivamente o menor e o

maior dos nmeros.

11
Unidade 21 Mdias

Exerccios Suplementares

1. Prove que, para dois nmeros positivos x1 x2 , suas mdias aritmtica


e

A, geomtrica G, harmnica H e quadrtica Q, satisfazem H 6 G 6

A 6 Q. Prove tambm que duas quaisquer dessas mdias so iguais se e

somente se x1 = x2 .

2. Qual seria o problema de se medir a qualidade de uma lista de aproxima-

es pela mdia aritmtica dos erros?

3. Para determinar uma grandeza desconhecida x, foram feitas vrias medi-


es. Os resultados obtidos foram x1 , x2 , . . . , xn . Determine a estimativa
de x para a qual o erro mdio quadrtico mnimo.

4. Para determinar uma grandeza desconhecida x, foram feitas vrias me-


dies. Os resultados obtidos foram x1 , x2 , . . . , xn tais que x1 6 x2 6

6 xn . Determine a estimativa de x para a qual a mdia dos valores


absolutos dos erros mnima.

5. Eduardo observou que o consumo de energia eltrica em sua casa estava

aumentando muito. Fez ento um grco do consumo anual, em kWh, nos

ltimos 5 anos, tomando 1991 como ano 0. Os valores obtidos encontram-

se no quadro abaixo e Eduardo achou que o grco parecia-se com uma

reta.

ANO (x) 0 1 2 3 4
CONSUMO (y) 820 1000 1200 1350 1550

fcil ver que os pontos encontrados no so colineares, mas pode-se

notar no grco que possvel traar retas que passem bem perto dos

cinco pontos. Mostrando o grco a seus amigos Augusto e Srgio, eles

sugeriram as retas y = 170x + 850 e y = 180x + 800, respectivamente,

como as retas que mais se aproximariam dos pontos.

a) Mostre que os pontos realmente no so colineares.

b) Calcule os erros mdios quadrticos e determine qual das duas retas

mais se aproxima dos pontos.

12
Mdias e Princpio das Gavetas I Unidade 21

c) Entre todas as retas do plano, qual a que mais se aproxima dos

pontos?

6. Mostre que em qualquer conjunto de 8 inteiros h sempre dois deles cuja

diferena um mltiplo de 7.

7. Em uma festa h 20 crianas sentadas em torno de uma mesa circular.

Um garom coloca diante de cada criana, sem perguntar qual a sua

preferncia, uma taa de sorvete. Alguns desses sorvetes so de creme e

os outros so de ocos. 10 das crianas preferem creme e 10 preferem

ocos. Mostre que, sem mexer nas crianas e fazendo apenas uma rotao

da mesa, possvel fazer com que pelo menos 10 crianas tenham suas

preferncias respeitadas.

8. Mostre que em toda reunio de n pessoas h sempre duas pessoas com

o mesmo nmero de conhecidos.

9. Mostre que existe um mltiplo de 1997 cujos dgitos so todos iguais a

1.

10. Qual o nmero mnimo de pessoas que deve haver em um grupo para

que possamos garantir que nele h pelo menos 7 pessoas nascidas no

mesmo ms?

11. So dados, no plano, cinco pontos de coordenadas inteiras. Mostre que,

entre os dez segmentos determinados por esses pontos, pelo menos um

tem como ponto mdio um ponto de coordenadas inteiras.

12. Prove que se Nk + 1 objetos so colocados em N gavetas, pelo menos

uma gaveta recebe mais de k objetos.

13. 40100 candidatos esto fazendo uma prova de 20 questes de mltipla

escolha, com 5 alternativas por questo. Suponha que nenhum candidato

deixe de responder a nenhuma questo. Considere a armao: Pelo

menos k candidatos respondero de modo idntico s 4 primeiras questes


da prova. Determine o maior valor de k para o qual a armao

certamente verdadeira.

13
Unidade 21 Mdias

14. 40100 candidatos esto fazendo uma prova de 20 questes de mltipla

escolha, com 5 alternativas por questo. Suponha que nenhum candidato

deixe de responder a nenhuma questo. Considere a armao: Pelo

menos 4 candidatos respondero de modo idntico s k primeiras questes


da prova. Determine o maior valor de k para o qual a armao

certamente verdadeira.

15. Os pontos de uma reta so coloridos com 11 cores. Mostre que possvel

achar dois pontos com a mesma cor tal que a distncia entre eles um

nmero inteiro.

16. Em um campeonato cada dois times jogam entre si uma nica vez. Mostre

que em qualquer momento h sempre dois times que disputaram o mesmo

nmero de partidas.

17. Sete pontos so selecionados dentro de um retngulo 3 4. Prove que

h dois desses pontos tais que a distncia entre eles no mximo igual a

5.

18. Selecionam-se oito nmeros distintos no conjunto {1, 2, . . . , 15}. Mostre

que h pelo menos trs pares de nmeros selecionados com a mesma

diferena entre o maior e o menor nmero do par.

19. Sejam x1 e x2 nmeros reais, x1 < x2 .

a) Mostre que os nmeros reais x tais que x1 < x < x2 podem ser
escritos na forma x = 1 x1 + 2 x2 com 1 + 2 = 1, 1 e 2 positi-
vos, isto , so mdias aritmticas ponderadas, com pesos positivos,

de x1 e x2 . Essa representao nica?

b) Mostre que os nmeros reais x da forma x = 1 x1 + 2 x2 com

1 + 2 = 1, 1 e 2 positivos, pertencem a (x1 , x2 ).

c) Onde esto os pontos x = 1 x1 + 2 x2 , com 1 + 2 = 1 e 1 > 1?


d) E com 1 + 2 = 1 e 1 < 0?

20. Sejam x1 , x2 , . . . , xn nmeros reais, x1 < x2 < < xn , n > 2.

14
Mdias e Princpio das Gavetas I Unidade 21

a) Mostre que os nmeros reais x tais que x1 < x < xn podem ser

escritos na forma x = 1 x1 + 2 x2 + + n xn com

1 + 2 + + n = 1, 1 , 2 , . . . , n > 0.

Essa representao nica?

b) Mostre que os nmeros reais x da forma

x = 1 x1 + 2 x2 + + n xn , n>2

com 1 + 2 + + n = 1, 1 , 2 , . . . , n positivos, pertencem

a (x1 , xn ).

21. Em um grupo de pessoas h 30 homens e 10 mulheres. Os homens tm

altura mdia de 1,75m e, as mulheres, de 1,67m. Qual a altura mdia do

grupo?

15
MA12 - Unidade 21
Medias e Princpio das Gavetas

Paulo Cezar Pinto Carvalho

PROFMAT - SBM

6 de Junho de 2013
Medias

Ideia geral de media: substituir uma lista de valores por um


unico valor, que preserve uma certa caracterstica dessa lista.

Principais medias:
Media aritmetica: preserva a soma dos valores.
Media geometrica: preserva o produto dos valores.
Media harmonica: preserva a soma dos inversos dos valores.

PROFMAT - SBM MA12 - Unidade 21, Medias e Princpio das Gavetas slide 2/12
Definicoes

A media aritmetica (simples) da lista de n numeros


x1 , x2 , . . . , xn e definida por
x1 + x2 + + xn
x = .
n
A media geometrica (simples) dos n numeros positivos
x1 , x2 , . . . , xn e definida por

g= n
x1 x2 . . . xn .

A media harmonica (simples) dos n numeros positivos


x1 , x2 , . . . , xn e definida por
n
h= 1 1 1
.
x1 + x2 + + xn

PROFMAT - SBM MA12 - Unidade 21, Medias e Princpio das Gavetas slide 3/12
Medias Ponderadas

A media aritmetica ponderada dos numeros x1 , x2 , . . . , xn com


pesos respectivamente iguais a p1 , p2 , . . . , pn e definida por
p1 x1 + p2 x2 + + pn xn
.
p1 + p2 + + pn
Quando os pesos sao inteiros nao negativos, corresponde a
media aritmetica simples da lista em que p1 elementos sao
iguais a x1 , p2 sao iguais a x2 , etc.
Mas a definicao tambem faz sentido quando p1 , p2 , . . . , pn sao
numeros nao negativos arbitrarios.
Caso particular: quando p1 + p2 + . . . + pn = 1, a media
aritmetica ponderada reduz-se a p1 x1 + p2 x2 + . . . + pn xn .

PROFMAT - SBM MA12 - Unidade 21, Medias e Princpio das Gavetas slide 4/12
Exemplo

Uma empresa produziu, durante o primeiro trimestre do ano


passado, 500, 200 e 200 unidades, em janeiro, fevereiro e
marco, respectivamente. Qual foi a producao media mensal
nesse trimestre?
Queremos uma media M tal que, se a producao mensal fosse
sempre igual a M, a producao trimestral seria a mesma.
3M = 500 + 200 + 200
500+200+200
M= 3 = 300.
A media apropriada e a aritmetica.

PROFMAT - SBM MA12 - Unidade 21, Medias e Princpio das Gavetas slide 5/12
Exemplo

Uma empresa aumentou sua producao durante o primeiro


bimestre do ano passado. Em janeiro e em fevereiro, as taxas
de aumento foram de 21% e 8%, respectivamente. Qual foi a
taxa media de aumento mensal nesse bimestre?
Queremos uma taxa media i tal que, se em todos os meses a
taxa de aumento fosse igual a i, o aumento bimestral seria o
mesmo.
(1 + i)2 = 1, 21 1, 08

1 + i = 1, 21 1, 08 = 1, 1432
i= 0, 1432 = 14, 32% (note que a media aritmetica dos
aumentos e 14,5%).
O fator de aumento medio e a media geometrica dos fatores
de aumento mensais.

PROFMAT - SBM MA12 - Unidade 21, Medias e Princpio das Gavetas slide 6/12
Exemplo
Uma viagem de automovel de ida e volta foi feita com uma
velocidade de 48 km/h na ida e de 72 km/h na volta. Qual foi
a velocidade media em todo o percurso?
Queremos uma velocidade media v tal que, se todo o percurso
(de extensao igual a 2d) fosse feito nesta velocidade, o tempo
total de viagem seria o mesmo.
2d d d
= +
v 48 72
1 1
1 48 + 72 5
= = 288
v 2
v = 288
5 = 57, 6 km/h (note que a media aritmetica das
velocidades e 60 km/h).
A velocidade media no percurso e a media harmonica das
velocidades.

PROFMAT - SBM MA12 - Unidade 21, Medias e Princpio das Gavetas slide 7/12
Propriedade

Se a media aritmetica dos numeros x1 , x2 , . . . , xn e igual a x,


pelo menos um dos numeros x1 , x2 , . . . , xn e maior que ou
igual a x.
Se x1 < x, x2 < x, , xn < x, teramos
x1 + x2 + + xn < nx
x1 +x2 ++xn
n < x
x < x
o que e absurdo.

PROFMAT - SBM MA12 - Unidade 21, Medias e Princpio das Gavetas slide 8/12
Exemplo

Mostre que, em um grupo de 50 pessoas, ha sempre pelo


menos 5 que nasceram no mesmo mes.
O numero medio de pessoas por mes e 50 12 = 4, 1 . . ..
Logo, em algum mes o numero de nascidos nesse mes (que e
um inteiro) e maior que ou igual a 4,1..., ou seja, e maior que
ou igual a 5.

PROFMAT - SBM MA12 - Unidade 21, Medias e Princpio das Gavetas slide 9/12
O Princpio das Gavetas

Se n + 1 ou mais objetos sao colocados em n ou menos


gavetas, entao pelo menos uma gaveta recebe mais de um
objeto.
O numero medio de objetos por gaveta e maior que ou igual a
n+1
n , que e maior que 1.
Logo, em alguma gaveta havera um numero de objetos maior
que 1 (ou seja, maior que ou igual a 2).

PROFMAT - SBM MA12 - Unidade 21, Medias e Princpio das Gavetas slide 10/12
Exemplo

Mostre que todo inteiro positivo n tem um multiplo que se


escreve apenas com os algarismos 0 e 1.
Considere os n + 1 primeiros numeros da sequencia 1, 11, 111,... e
os restos das divisoes desses numeros por n. Esses restos so podem
ser iguais a 0, 1, 2, . . . , n 1.
Pensando nos numeros como objetos e nos restos como gavetas,
temos mais objetos do que gavetas.
Pelo Princpio das Gavetas, ha dois numeros (objetos) na sequencia
que dao o mesmo resto (estao na mesma gaveta) quando divididos
por n.
Sejam estes numeros 11 ... 1 (p algarismos) e 11 ... 1 (q
algarismos), p < q.
A diferenca desses numeros e um multiplo de n e se escreve
11 . . . 10 . . . 0, com p algarismos 0 e q p algarismos 1.

PROFMAT - SBM MA12 - Unidade 21, Medias e Princpio das Gavetas slide 11/12
Exemplo
Cinco pontos sao tomados sobre a superfcie de um quadrado
de lado 2. Mostre que ha dois desses pontostais que a
distancia entre eles e menor que ou igual a 2.
Divida o quadrado de lado 2 em quatro quadrados de lado 1,
ligando os pontos medios dos lados opostos.

Pensando nos pontos como objetos e nos quadrados como gavetas,


temos mais objetos do que gavetas.
Pelo Princpio das Gavetas, alguma gaveta recebera mais de um
objeto, isto e, havera dois pontos no mesmo quadrado de lado 1.
A distancia entre esses pontos e no maximo igual ao comprimento

da diagonal do quadrado, que e 2.
PROFMAT - SBM MA12 - Unidade 21, Medias e Princpio das Gavetas slide 12/12
Lista de Exerccios
Unidade 21

1. Um carro percorre metade de certa distancia d com velocidade v1 e


percorre a outra metade com velocidade v2 . Qual a sua velocidade
media?
2. Um carro tem velocidade v1 durante metade do tempo t de percurso
e tem velocidade v2 durante a outra metade do tempo. Qual a sua
velocidade media?
3. A populacao de um pas cresceu 44% em uma decada e cresceu 21% na
decada seguinte. Qual e, aproximadamente, a taxa media decenal de
crescimento nesses 20 anos?
4. A valorizacao mensal das acoes de certa empresa nos quatro primeiros
meses do ano foi de +25%, +25%, 25% e 25%. Qual a valorizacao
total e qual a valorizacao media mensal nesse quadrimestre?
5. Em um grupo de pessoas ha 30 homens e 10 mulheres. Os homens tem
altura media de 1,75m e, as mulheres, de 1,67m. Qual a altura media
do grupo?
6. Calcule as medias aritmetica, geometrica e harmonica ponderadas dos
numeros 8, 18 e 48, com pesos iguais a 1, 1 e 0,5, respectivamente.
7. Pneus novos duram 40 000 km, quando usados nas rodas dianteiras, e
duram 60 000 km, quando usados nas rodas traseiras.
a) Com 4 pneus novos e fazendo um rodzio adequado entre eles,
quantos quilometros um carro pode rodar? Como?
b) E com 5 pneus novos? Como?
c) A resposta do item a) e uma media entre 40 000 km e 60 000 km.
Qual?
8. A media aritmetica de 50 numeros e 40. Se dois desses numeros, 125
e 75, forem suprimidos, qual sera a media aritmetica dos numeros res-
tantes?

1
9. Mostre que em qualquer conjunto de 8 inteiros ha sempre dois deles
cuja diferenca e um multiplo de 7.

10. Em uma festa ha 20 criancas sentadas em torno de uma mesa circu-


lar. Um garcom coloca diante de cada crianca, sem perguntar qual a
sua preferencia, uma taca de sorvete. Alguns desses sorvetes sao de
creme e os outros sao de flocos. 10 das criancas preferem creme e 10
preferem flocos. Mostre que, sem mexer nas criancas e fazendo apenas
uma rotacao da mesa, e possvel fazer com que pelo menos 10 criancas
tenham suas preferencias respeitadas.

11. Mostre que em toda reuniao de n pessoas ha sempre duas pessoas com
o mesmo numero de conhecidos.

12. Mostre que existe um multiplo de 2013 cujos dgitos sao todos iguais a
1.

13. Qual e o numero mnimo de pessoas que deve haver em um grupo para
que possamos garantir que nele ha pelo menos 7 pessoas nascidas no
mesmo mes?

14. Sao dados, no plano, cinco pontos de coordenadas inteiras. Mostre que,
entre os dez segmentos determinados por esses pontos, pelo menos um
tem como ponto medio um ponto de coordenadas inteiras.

15. 40100 candidatos estao fazendo uma prova de 20 questoes de multipla


escolha, com 5 alternativas por questao. Suponha que nenhum candi-
dato deixe de responder a nenhuma questao. Considere a afirmacao:
Pelo menos k candidatos responderao de modo identico as 4 primei-
ras questoes da prova. Determine o maior valor de k para o qual a
afirmacao e certamente verdadeira.

16. 40100 candidatos estao fazendo uma prova de 20 questoes de multipla


escolha, com 5 alternativas por questao. Suponha que nenhum candi-
dato deixe de responder a nenhuma questao. Considere a afirmacao:
Pelo menos 4 candidatos responderao de modo identico as k primei-
ras questoes da prova. Determine o maior valor de k para o qual a
afirmacao e certamente verdadeira.

2
17. Em um campeonato cada dois times jogam entre si uma unica vez.
Mostre que em qualquer momento ha sempre dois times que disputaram
o mesmo numero de partidas.

18. Sete pontos sao selecionados dentro de um retangulo 3 4. Prove que


hadois desses pontos tais que a distancia entre eles e no maximo igual
a 5.

3
Solucoes da Lista de Exerccios
Unidade 21

d d
1. Os tempos de percurso sao t1 = e t2 = . Logo, a velocidade
2v1 2v2
d d 2
media e vm = = d = 1 , que e a media harmonica
t1 + t2 2v1
+ 2vd2 v1
+ v12
das velocidades v1 e v2 na ida e na volta.

2. A distancia total percorrida e d = tv1 + tv2 . Logo, a velocidade media


tv1 + tv2 v1 + v2
e vm = = , que e a media aritmetica das velocidades
2t 2
v1 e v2 nas duas partes do percurso.

3. Apos as duas decadas, a populacao e multiplicada por 1, 44 1, 21. A


taxa media i de crescimento decenal e a taxa que, mantendo-se cons-
tante para as duas decadas, produza o mesmo crescimento total. Logo,
2
devemos ter (1 + i) = 1, 44 1, 21, Logo, 1 + i = 1, 44 1, 21 =
1, 2 1, 1 = 1, 31. Portanto, a taxa media de crescimento decenal e
0, 32 = 32%.

4. No perodo, o valor das acoes e multiplicado por 1, 251, 250, 750, 75 =


0, 87890625. Logo, as acoesse desvalorizaram 12,11 no perodo. O fator
medio de variacao anual e 4 0, 87890625 = 0, 9682, que representa uma
desvalorizacao media anual igual a 3,18%.
301,75+101,67
5. A altura media e 40
= 1, 73.
18+118+0,548
6. A media aritmetica ponderada e 1+1+0,5
= 20. A media geometrica
1
ponderada e (81 181 480,5 ) 1+1+0,5
= 13, 78. A media harmomica pon-
1
derada e 1 1 +1 1 +1 1 = 13, 9.
8 18 48
1+1+0,5

7. (a) Se um pneu roda x mil quilometros em uma roda dianteira e y


mil quilometros em uma roda traseira, a fracao do pneu que gasta
x y
e 40 + 60 . Para conseguir a rodagem maxima sem trocar pneus,

1
todos os pneus devem gastar totalmente ao mesmo tempo (ou
seja, esta fracao deve ser igual a 1 ao final do processo para cada
pneu). Portanto, cada um deles devera rodar o mesmo numero
de quilometros em uma roda dianteira e uma roda traseira, ou
x y
seja x = y. Logo, deve-se ter 40 + 60 = 1, de onde resulta x =
24. Logo, cada pneu deve rodar 24 mil quilometros em uma roda
dianteira e 24 mil quilometros em uma roda traseira. Portanto,
o carro pode rodar 48 mil quilometros com um jogo de 4 pneus,
bastando para isto trocar os pneus traseiros pelos dianteiros aos
24 mil quilometros.
(b) Do mesmo modo, cada pneu deve rodar 24 mil quilometros em
uma roda traseira e 24 mil quilometros em uma roda dianteira.
Mas como agora temos 5 pneus, o carro pode rodar por 5/4 de
48 mil quilometros, ou seja, por 60 mil km. Para tal, basta fazer
um rodzio dos pneus a cada 12 mil km. Por exemplo, comecando
com ABCDE (com A e B na dianteira, C e D na traseira e E no
estepe), passar para BCDEA, CDEAB, DEABC e EABCD).
2
(c) O resultado em a) e igual a 1 1
+ 60.000
. Logo, e a media harmonica
40.000
de 40.000 e 60.000.
504012575
8. A nova media e 48
= 37, 5.

9. Existem 7 restos possveis quando se divide um numero por 7. Assim,


tomando os restos como gavetas e os numeros como objetos, pode-se
garantir que alguma gavetas contera dois (ou mais) objetos. Isto e, ha
pelo menos dois numeros que deixam o mesmo resto quando dividimos
por 7, o que e equivalente a dizer que a diferencia entre eles e um
multiplo de 7.

10. A mesa pode ser colocada em 20 posicoes diferentes. Seja ai (i =


1, . . . , 20) o numero de criancas cuja preferencia e atendida com a mesa
na posicao i. Entao a1 + a2 + + a20 e o numero total de preferencias
atendidas. Mas cada sorvete e colocado, sucessivamente, em frente
a cada crianca. Como ha exatamente 10 criancas que preferem cada
sabor, o numero total de preferencias atendidas por cada sorvete e 20,
para um total de 200 preferencias atendidas. Assim, temos a1 + a2 +
+ an . Como a media de a1 , . . . , a20 e 10, conclui-se que pelo menos

2
um dos numeros e maior ou igual a 10 (ou seja, ha alguma posicao em
que pelo menos 10 criancas sao atendidas).

11. Em uma reuniao com n pessoas, cada pessoa pode ter de 0 a n


1 conhecidos. No entanto, e impossvel que, ao mesmo tempo, haja
uma pessoa que nao tenha conhecidos e outra que conheca todos (a
final, estas duas pessoas se conhecem ou nao?). Portanto, em qualquer
situacao ha apenas n1 valores possveis para o numero de conhecidos,
o que implica que pelo menos duas das n pessoas tem o mesmo numero
de conhecidos.

12. Considere os restos da divisao por 2013, dos numeros 1, 11, 111, . . . .
Como ha apenas 2013 restos possveis, necessariamente ha dois restos
coincidentes. Tomando a diferenca dos dois numeros da forma 11. . . 1
resulta a existencia de um numero da forma 11 . . . 10 . . . 0 = 11 . . . 1
10k que e multiplo de 2013. Como 2013 e 10 sao primos entre si, o
numero da forma 11 . . . 1 acima e necessariamente multiplo de 2013.

13. Deve haver 73 pessoas. Podemos distribuir ate 72 pessoas de modo que
haja exatamente 6 nascidas em cada mes. Com 73 pessoas, necessari-
amente um dos meses (gavetas) contera 7 ou mais pessoas (objetos).

14. As paridades das coordenadas dos pontos de coordenadas inteiras do


plano determinam 4 gavetas: par-par, par-mpar, mpar-par e mpar-
mpar. Dados 5 pontos, pelo menos uma das gavetas contem dois pon-
tos. Ou seja, ha um par de pontos em que ambas as coordenadas tem a
mesma paridade, o que faz com que o ponto medio tenha coordenadas
inteiras.

15. As 4 primeiras questoes da prova podem ser respondidas de 5 5


5 5 = 625 modos (gavetas) diferentes. Como sao 40100 candidatos, o
numero medio de candidatos para cada possvel padrao de resposta e
40100
625
= 64, 16. Pode-se garantir, em consequencia, que uma das gavetas
contem 65 objetos; ou seja, pelo menos 65 candidatos respondem de
modo identico as primeiras 4 questoes. Este e o maior valor possvel
para k, ja que e possvel distribuir os candidatos de modo que haja no
maximo 65 por cada padrao de resposta.

16. As primeiras k questoes podem ser respondidas de 5k modos (gavetas).


Para garantir que pelo menos 4 candidatos respondam a estas questoes

3
do mesmo modo, deve-se ter pelo menos 35k +1 candidatos (objetos).
Portanto, deve-se ter 35k < 40100, o que ocorre para k 5. Portanto,
o valor maximo possvel para k e 5.

17. O numero de jogos de cada um dos n times e um numero inteiro de 0 a


n 1. Mas, como no exerccio 23, nao pode existir, simultaneamente,
um time com 0 jogos e outro com n 1 (eles ja jogaram entre si ou
nao?). Logo, ha apenas n 1 gavetas para n objetos, o que garante
que dois times enfrentaram o mesmo numero de adversarios.

18. Dividamos o retangulo em 6 retangulos 1 2. Como ha 7 pontos, ha


dois que estao no mesmo retangulo. A distancia
entre eles e no maximo
igual a diagonal do retangulo, que mede 5.

4
22
Mdias e Princpio
Gavetas II das
Sumrio
22.1 A Desigualdade das Mdias . . . . . . . . . . . . . . 2

1
Unidade 22 A Desigualdade das Mdias

Nessa unidade, faz-se a comparao entre as vrias mdias, resultando numa

desigualdade fundamental entre a mdia aritmtica, a mdia geomtrica e a

mdia harmnica.

22.1 A Desigualdade das Mdias

A desigualdade das mdias arma que a mdia aritmtica de n nmeros positivos


maior que ou igual sua mdia geomtrica e s igual se os nmeros forem

todos iguais. Isto , se x1 , x2 , . . . , xn so nmeros positivos


x1 + x2 + + xn
> n x1 x2 xn .
n
Alm disso,
x 1 + x2 + + xn
= n x1 x2 xn
n
se e somente se x1 = x2 = = xn .
Vrias e interessantes demonstraes dessa desigualdade so encontradas

em Meu Professor de Matemtica de Elon Lages Lima. Aqui faremos apenas


1
um esboo da demonstrao que foi feita por Cauchy .

Provaremos primeiramente a desigualdade no caso n = 2. Sendo A(x1 , x2 )


a mdia aritmtica dos nmeros positivos x1 e x2 e sendo G(x1 , x2 ) sua mdia

geomtrica, temos

x1 + x2
A(x1 , x2 ) G(x1 , x2 ) = x1 x2
2
x1 + x2 2 x1 x 2
=
2

( x1 x2 )2
= >0
2
e A(x1 , x2 ) G(x1 , x2 ) s igual a 0 quando x1 = x2 , o que prova a desigual-

dade no caso n = 2.

Para prov-la no caso n = 4, aplicamos o resultado anterior aos nmeros

x1 + x2 x3 + x4
e ,
2 2
obtendo s
x1 +x2 x3 +x4  
2
+ 2 x1 + x2 x3 + x4
> ,
2 2 2
1 Cauchy, Louis (1789-1857), matemtico francs.

2
Mdias e Princpio das Gavetas II Unidade 22

ou seja,
s  
x1 + x2 + x3 + x4 x1 + x2 x3 + x4
> ,
4 2 2
a igualdade s sendo obtida quando

x1 + x2 x3 + x4
e
2 2
forem iguais. Aplicando agora duas vezes a desigualdade no caso n = 2, pri-

meiramente para x1 e x2 , e posteriormente para x3 e x4 , obtemos

r
x 1 + x2 x3 + x4
q
> x1 x2 x3 x 4 = 4 x1 x2 x3 x4 ,
2 2
a igualdade sendo obtida apenas quando x1 = x2 e x3 = x4 .
Portanto,
x1 + x2 + x3 + x4
> 4 x1 x2 x3 x4 ,
4
a igualdade s sendo obtida quando x1 = x2 e x3 = x4 e

x1 + x2 x3 + x4
= ,
2 2
isto , quando x1 = x2 = x3 = x4 .
claro que, repetindo esse argumento, provaramos a desigualdade das

mdias para 8, 16, 32,... nmeros positivos.

Esse argumento permite provar, por induo, a desigualdade para n = 2k


nmeros positivos.

Provaremos agora a desigualdade para trs nmeros positivos.

Sejam x1 , x2 e x3 nmeros positivos e sejam A sua mdia aritmtica e G


sua mdia geomtrica. claro que

x1 + x 2 + x3 + A 3A + A
= = A.
4 4
Aplicando a desigualdade das mdias no caso n=4 aos nmeros x1 , x2 , x3 e

A, obtemos
x1 + x2 + x3 + A p
A= > 4 x1 x2 x3 A.
4

A4 > x1 x2 x3 A, A3 > x1 x2 x3 , A > 3 x1 x2 x3 = G a igualdade s se vericando
quando x1 = x2 = x3 = A, isto , quando x1 = x2 = x3 . Se desejssemos

3
Unidade 22 A Desigualdade das Mdias

provar a desigualdade para cinco nmeros positivos x1 , x2 , x3 , x4 e x5 , aplica-


ramos a desigualdade aos 8 nmeros x1 , x2 , x3 , x4 e x5 , A, A e A, onde A

a mdia aritmtica dos nmeros x1 , x2 , x3 , x4 e x5 .

O mesmo raciocnio pode mostrar que, se a desigualdade verdadeira para

n = k, ento ela tambm verdadeira para todo n < k.

Exemplo 1 Mostre que, entre todos os retngulos de permetro 2p, o quadrado o de

maior rea.

Soluo. Se os lados do retngulo so x e y , temos x + y = p, isto , a mdia


p
aritmtica x e y igual a . A rea do retngulo A = xy . Temos
2
x+y p
A = xy 6 = .
2 2
Portanto,
p2
A6
4
e a igualdade s obtida quando x = y. Portanto, o retngulo de maior rea
2
o quadrado de rea p /4.

Exemplo 2 Mostre que, entre todos os retngulos de rea A, o quadrado o de menor

permetro.

Soluo. Se os lados do retngulo so x e y,xy = A, isto , a mdia


temos

geomtrica de xey igual a A. O permetro do retngulo 2(x + y). Temos
x+y
2(x + y) = 4 > 4 xy = 4 A.
2

Portanto, 2(x + y) > 4 A e a igualdade s obtida quando x = y. Portanto,

o retngulo de menor permetro o quadrado de permetro 4 A.

A desigualdade das mdias pode ser generalizada como segue:

Se x1 , x2 , . . . , xn so nmeros positivos e Q, A, G e H so suas mdias

quadrtica, aritmtica, geomtrica e harmnica, respectivamente, ento Q>


A > G > H . Alm disso, duas quaisquer dessas mdias so iguais se e somente
se x1 = x2 = = xn .

4
Mdias e Princpio das Gavetas II Unidade 22

Exerccios Recomendados

1. Prove que o produto de dois nmeros de soma constante mximo quando

esses nmeros so iguais.

2. Prove que a soma de dois nmeros positivos de produto constante

mnima quando esses nmeros so iguais.

3. Prove que a mdia harmnica de n nmeros positivos

x1 , x2 , . . . , xn

sempre menor que ou igual a sua mdia geomtrica e s igual quando

todos os nmeros so iguais.

4. Prove que a mdia quadrtica de n nmeros positivos

x1 , x2 , . . . , xn

sempre maior que ou igual a sua mdia aritmtica e s igual quando

todos os nmeros so iguais.

5. Prove que se a1 , a2 , . . . , an so nmeros positivos e

b1 , b2 , . . . , bn
b1 b2 bn
uma reordenao de a1 , a2 , . . . , an ento + + + > n.
a1 a2 an
6. Prove que x2 + y 2 + z 2 > xy + yz + zx, para quaisquer x, y e z reais.

7. Prove que se a1 , a2 , a3 so positivos, ento


r
a1 + a2 + a3 a1 a2 + a1 a3 + a2 a3
> > 3 a1 a2 a3 .
3 3

8. Mostre que se a equao x3 ax2 + bx c = 0, na qual a, b e c so


6 3 2
nmeros positivos possuir trs razes reais ento a > 27b > 729c .

9. Um mgico se apresenta usando um palet cintilante e uma cala colorida

e no repete em suas apresentaes o mesmo conjunto de cala e palet.

Para poder se apresentar em 500 espetculos, qual o menor nmero de

peas de roupa que pode ter seu guarda-roupa?

5
Unidade 22 A Desigualdade das Mdias

10. Prove que entre todos os tringulos de permetro constante, o equiltero

o de maior rea.

1
11. a) Prove que, se x positivo, ento x+ > 2.
x
4
b) Qual o valor mnimo de x + , x positivo?
x
 n
1
12. Prove que a sequncia de termo geral an = 1+ estritamente
n  n
1
crescente, isto , prove que, para todo n inteiro e positivo 1+ <
n
 n+1
1
1+ .
n+1

Exerccios Suplementares

1. Prove que, se x, y e z so positivos, ento

1 1 1 9
+ + > .
x y z x+y+z

2. Prove que, se x, y e z so positivos, ento


( x + y + z)2 > 9 3 xyz.

3. Se x, y e z so nmeros positivos tais que 1 6 xy + yz + zx 6 3, qual

o conjunto de valores de xyz ? E de x + y + z?

4. Se x, y e z so nmeros positivos tais que xy + yz + zx 6 3, qual o

conjunto de valores de xyz ? E de x + y + z?

5. Se x, y e z so nmeros positivos tais que xy + yz + zx 6 1, qual o

conjunto de valores de xyz ? E de x + y + z?

6. Se x, y e z so nmeros positivos tais que 1 6 x + y + z 6 3, qual o

conjunto de valores de xyz ? E de xy + yz + zx?

7. Se x, y e z so nmeros positivos tais que 1 6 xyz 6 3, qual o conjunto


de valores de xy + yz + zx? E de x + y + z ?

6
Mdias e Princpio das Gavetas II Unidade 22

8. Se x, y e z so nmeros positivos tais que xyz = 8, qual o conjunto de

valores de xy + yz + zx? E de x + y + z ?

9. Prove que, se a desigualdade das mdias vlida para m nmeros posi-

tivos, m > 2, ento ela vlida tambm para m1 nmeros positivos.

7
MA12 - Unidade 22
A Desigualdade das Medias

Paulo Cezar Pinto Carvalho

PROFMAT - SBM

6 de Junho de 2013
A desigualdade das medias

Teorema: Sejam A, G e H, respectivamente, as medias


aritmetica, geometrica e harmonica de n valores positivos
x1 , x2 , . . . , xn .
a) A G H
b) a igualdade so ocorre quando todos os valores sao iguais

Observacao: G H decorre de A G

AG
1
+ x1 +... x1 q
1 1
x1 2
n
n
n
x1 x2 x1n
n
1
+ x1 +... x1
n x1 x2 . . . xn
x1 2 n

HG

PROFMAT - SBM MA12 - Unidade 22, A Desigualdade das Medias slide 2/7
O caso n = 2

Sejam x1 e x2 numeros positivos


x1 +x2
AG = 2 x1 x2

x1 +x2 2 x1 x2
= 2

( x1 x2 )2
= 2 0

Alem disso, a igualdade so ocorre quando x1 = x2 .

PROFMAT - SBM MA12 - Unidade 22, A Desigualdade das Medias slide 3/7
O caso n = 2k
A prova e feita por inducao.
Vale para k = 1 (corresponde a n = 2).
Suponhamos valida para n = 2k ; vamos mostrar que vale para
2n = 2k+1 .
x1 + . . . + xn + xn+1 + . . . + x2n
A=
2n
x1 +...+xn xn+1 +...+x2n
n + n
=
2

n
x1 xn + n xn+1 x2n

2

2n x1 xn xn+1 x2n = G
Alem disso, a igualdade so vale quando
x1 = . . . = xn , xn+1 = . . . = x2n e x1 xn = xn+1 x2n ,
ou seja, quando x1 = . . . = xn = xn+1 = . . . = x2n .
Logo, a propriedade e tambem valida para 2n = 2k+1 .
Por inducao, a desigualdade das medias vale para todo n da
forma 2k .
PROFMAT - SBM MA12 - Unidade 22, A Desigualdade das Medias slide 4/7
O caso geral

A prova consiste em completar a lista com valores iguais a


media aritmetica dos n numeros, ate o numero de valores
tornar-se uma potencia de 2.
Ilustramos para n = 3, quando completamos a lista x1 , x2 , x3
com um quarto valor igual a A = x1 +x32 +x3 .

A = x1 +x2 +x
4
3 +A
> 4 x1 x2 x3 A
A4 > x1 x2 x3 A
A3 > x1 x2 x3

A > 3 x1 x2 x3 = G
Alem disso, a igualdade so vale quando x1 = x2 = x3 .

PROFMAT - SBM MA12 - Unidade 22, A Desigualdade das Medias slide 5/7
Exemplo

Mostre que, entre todos os retangulos de permetro 2p, o


quadrado e o de maior area.
Se os lados do retangulo sao x e y , temos x + y = p, isto e, a
p
media aritmetica de x e y e igual a .
2
A area do retangulo e S = xy .
Pela desigualdade dasmedias:

xy 6 x+y
2 , ou seja, S 6 p2 .
p2
Logo, S 6 4 e a igualdade so e obtida quando x = y .
p
Portanto, o retangulo de maior area e o quadrado de lado 2 e
area p 2 /4.

PROFMAT - SBM MA12 - Unidade 22, A Desigualdade das Medias slide 6/7
Exemplo
Qual e o valor mnimo da area total de um bloco retangular de
volume 64 cm3 ? Quais sao as dimensoes do bloco neste caso?
Se as dimensoes do bloco sao a, b e c, temos V = abc = 64 e
S = 2(ab + ac + bc).
Aplicando a desigualdade das medias aos numeros ab, ac e
bc, temos:
ab+bc+ac

3 3 ab bc ac
3

S
V 2 = 3 642 = 16
6

Logo, S 6 16 = 96 cm2 .

A igualdade ocorre quando ab = ac = bc, ou seja, quando


a = b = c.

Portanto, o valor mnimo da area e igual a 96cm2 e ocorre


quando o bloco e um cubo de aresta igual a 3 64 = 4 cm.
PROFMAT - SBM MA12 - Unidade 22, A Desigualdade das Medias slide 7/7
Lista de Exerccios
Unidade 22

1. Prove que o produto de dois numeros de soma constante e maximo


quando esses numeros sao iguais.
2. Prove que a soma de dois numeros positivos de produto constante e
mnima quando esses numeros sao iguais.
3. Mostre que a q media quadratica de n numeros positivos x1 , x2 , . . . , xn
x21 +x22 +...+x2n
(definida por n
) e sempre maior que ou igual a sua media
aritmetica e so e igual quando todos os numeros sao iguais.
4. Prove que se a1 , a2 , . . . , an sao numeros positivos e b1 , b2 , . . . , bn e uma
b1 b2 bn
reordenacao de a1 , a2 , . . . , an entao + + + > n.
a1 a2 an
5. Prove que x2 + y 2 + z 2 > xy + yz + zx, para quaisquer x, y e z reais.
6. Prove que se a1 , a2 , a3 sao positivos, entao
r
a1 + a2 + a3 a1 a2 + a1 a3 + a2 a3
> > 3 a1 a2 a3 .
3 3

7. Mostre que se a equacao x3 ax2 + bx c = 0, na qual a, b e c sao


numeros positivos possuir tres razes reais entao a6 > 27b3 > 729c2 .
8. Um magico se apresenta usando um paleto cintilante e uma calca co-
lorida e nao repete em suas apresentacoes o mesmo conjunto de calca
e paleto. Para poder se apresentar em 500 espetaculos, qual o menor
numero de pecas de roupa que pode ter seu guarda-roupa?
9. Prove que entre todos os triangulos de permetro constante, o equilatero
e o de maior area.
1
10. a) Prove que, se x e positivo, entao x + > 2.
x
4
b) Qual o valor mnimo de x + , x positivo?
x
1
 n
1
11. Prove que a sequencia de termo geral an = 1 + e estritamente
n  n
1
crescente, isto e, prove que, para todo n inteiro e positivo 1 + <
n
 n+1
1
1+ .
n+1
12. Prove que, se x, y e z sao positivos, entao
1 1 1 9
+ + > .
x y z x+y+z

13. Prove que, se x, y e z sao positivos, entao



( x + y + z)2 > 9 3 xyz.

14. Se x, y e z sao numeros positivos tais que 1 6 xy + yz + zx 6 3, qual


e o conjunto de valores de xyz? E de x + y + z?

15. Se x, y e z sao numeros positivos tais que 1 6 xyz 6 3, qual e o


conjunto de valores de xy + yz + zx? E de x + y + z?

16. Se x, y e z sao numeros positivos tais que xyz = 8, qual e o conjunto


de valores de xy + yz + zx? E de x + y + z?

2
Solucoes da Lista de Exerccios
Unidade 22

1. Se os numeros tem soma constante, sua media aritmetica A e tambem


constante. Pela desigualdade das medias, o maior valor possvel para
a media geometrica G e igual a A, o que ocorre quando os numeros
sao iguais. Logo, o produto dos numeros (que e o quadrado de G) e
maximo quando os numeros sao iguais.

2. Se os numeros tem produto constante, sua media geometrica G e tambem


constante. Pela desigualdade das medias, o menor valor possvel para
a media aritmetica A e igual a G, o que ocorre quando os numeros sao
iguais. Logo, a soma dos numeros (que e dobro de A) e mnima quando
os numeros sao iguais.

3. Temos
n
X n
X n
X
2 2
(xk A) = (xk ) 2A xk + nA2
k=1 k=1 k=1
= nQ 2A nA + nA2 = n(Q2 A2 )
2

Como uma soma de quadrados e necessariamente nao negativa, resulta


Q2 A2 ou, equivalentemente, Q A. Alem disso, so se tem igualdade
quando cada termo da soma inicial e nulo, ou seja, quando xk = A, para
todo k, o que significa que todos os numeros xk sao iguais.
q
4. A media geometrica de ab12 , ab22 , . . . , abnn e n ab11ab22b
an
n
= 1 (note que os
produtos no numerador e denominador sao iguais, ja que b1 , b2 , . . . , bn
sao uma reordenacao de a1 , a2 , . . . , an ). Logo, sua media aritmetica e
b1 b
a1
+ a2 ++ abn
maior que ou igual a 1. Assim 2
n
n
1 ou, equivalentemente,
b1
a1
+ ab22 + + abnn n.

5. Aplicando a desigualdade entre as medias aritmetica e geometrica aos

1
pares (x2 , y 2 ), (y 2 , z 2 ) e (z 2 .y 2 ), vem

x2 + y 2
xy
2
y2 + z2
yz
2
z 2 + x2
zx
2

Somando membro a membro vem a desigualdade pedida.

6. Aplicando a desigualdade das medias a a1 a2 , a2 a3 e a3 a1 , resulta a1 a2 +a23a3 +a3 a1


p q
3
a21 a22 a23 , que equivale a a1 a2 +a23a3 +a3 a1 3 a1 a2 a3 .
2
A outra desigualdade e equivalente a a1 +a32 +a3 a1 a2 +a23a3 +a3 a1 , que,
por sua vez, e equivalente a a21 + a22 + a23 a1 a2 + a2 a3 + a3 a1 , que e
verdadeira, pelo exerccio anterior.

7. Se x 0, entao x3 ax2 +bxc e uma soma de numeros nao positivos na


qual pelo menos c e negativo. Logo, se a equacao x3 ax2 +bxc = 0
possui tres razes reais x1 , x2 , x3 , elas sao necessariamente positivas.
Usando as relacoes entre coeficientes e razes, temos a = x1 + x2 + x3 ,
b = xq1 x2 + x2 x3 + x3 x1 e c = x1 x2 x3 . Pelo exerccio anterior, temos
a b
3
3
3 c. Elevando a sexta potencia e multiplicando por 729,
vem a6 27b3 729c2 .

8. Com x paletos e y calcas, podem ser formadas xy roupas diferentes. O


valor mnimo da soma de dois numeros com produto constante ocorre
quando os numeros sao iguais. Se x pudesse assumir valores nao in-
teiros, o mnimo ocorreria para x = 500 = 22, 36. Claramente, 44
roupas nao bastam, porque o produto maximo de dois numeros com
soma 44 e igual a 22 22 < 500. Mas 45 bastam, ja que 22 23 = 506.
Ou seja, o magico pode usar trajes diferentes em todas as apresentacoes
com 23 (ou 22) calcas e 22 (ou 23) paletos.

9. Pela
p formula de Heron, a area de um triangulo de lados a, b e c e S =
1
2
p(p a)(p b)(p c). Para triangulos de permetro constante, a
area maxima quando o produto (p a)(p b)(p c) e maximo. Mas a
soma (p a) + (p b) + (p c) e igual a p, portanto constante. Logo,

2
para que (p a)(p b)(p c) seja maximo deve-se p a = p b = p c,
ou seja, a = b = c. Logo, dentre os triangulos de permetro constante,
o equilatero e o de maior area.

10. (a) A media geometrica de x e x1 e constante e igual a 1. Logo, sua


media aritmetica e sempre maior que ou igual a 1, isto e, x+ x1 2,
com igualdade quando os numeros sao iguais (ou seja, quando
x = 1).
q
(b) A media geometrica de x e 4/x e constante e igual a x x4 = 2.
Em consequencia, o valor mnimo da media aritmetica e 2 e o valor
mnimo de x + x4 e 4, que ocorre quando x = x4 , ou seja, quando
x = 2.

11. A media aritmetica dos numeros x1 = x2 = = xn = 1+ n1 , xn+1 = 1 e


 n
A = n+2
n+1
= 1+ 1
n+1
, enquanto a sua media geometrica e G = 1 + 1 n+1
n
.
1
 n
1 n+1
A desigualdade das medias fornece, portanto, 1 + n+1 < 1 + n ,
1
n+1 1 n

que e equivalente a 1 + n+1 < 1 + n (note que a desigualdade
e estrita, ja que os numeros x1 , . . . , xn nao sao todos iguais).

12. Aplicando a desigualdade entre a media aritmetica e a media harmonica


1
+ y1 + z1
aos numeros x1 , y1 e 1
z
obtemos x
3
3
x+y+z
ou, equivalentemente,
1
x
+ y1 + z1 x+y+z
9
.

13. Aplicando a desigualdade das medias aos numeros x, ye z, vem
q
x+ y+ z p
3
3 x y z= 3 xyz. Da,


( x + y + z)2 9 3 xyz.

14. Aplicando a desigualdade das medias aos numeros xy, yz e xz, re-
xy+yz+zx
p
sulta 3 2
(xyz) . Como xy + yz + zx 3, temos xyz
q 3
xy+yz+zx
3
3
1, com igualdade no caso x = y = z = 1. Por outro
lado, com x = y = n1 e z = n, temos xy + yz + zx = 2 + n12 (portanto,
1 xy + yz + zx 3) e xyz = 1/n, o que mostra que xyz pode ficar
arbitrariamente proximo a zero, bastando para isso tomar n suficien-
temente grande. Portanto, o conjunto de valores de xyz e o intervalo
(0, 1].

3
q
Pelo problema 7, x + y + z 3 xy+yz+zx
3
3, com igualdade no

caso x = y = z = 33 (a ultima desigualdade decorre de xy + yz + zx
1). Por outro lado, novamente tomando x = y = n1 e z = n, temos
xy + yz + zx = 2 + n12 (portanto, de novo, 1 xy + yz + zx 3) e
x + y + z = n + n2 , o que mostra que x + y + z pode assumir valores
arbitrariamente
grandes. Logo, o conjunto de valores de x + y + z e o
intervalo [ 3, +).

15. Como visto no problema 15, temos xy+yz+zx


p
3
3
(xyz)2 . Portanto, de
xyz 1 decorre xy+yz+zx 3, com igualdade quando x = y = z = 1.
Por outro lado, tomando-se x = y = n1 e z = n2 , tem-se xyz = 1 e
xy + yz + zx = 2n + n12 , que assume valores arbitrariamente grandes
para n natural. Logo, o conjunto de valores de xy +yz +zx e o intervalo
[3, +).

Pela desigualdade das medias, x+y+z 3 xyz. Logo, de xyz 1,
3
resulta x + y + z 3 3 xyz 3, com igualdade quando x = y = z = 1.
Por outro lado, tomando-se x = y = n1 e z = n2 , tem-se xyz = 1 e
x + y + z = n2 + n2 , que assume valores arbitrariamente grandes para n
natural. Logo, o conjunto de valores de x + y + z e o intervalo [3, +).

16. Como visto no problema 15, xy+yz+zx


p
3
3 (xyz)2 . Portanto, de xyz =
8 decorre xy + yz + zx 12, com igualdade quando x = y = z = 2.
Por outro lado, tomando x = y = n2 e z = 2n2 , vem xyz = 8 e
xy + yz + zx = 8n + 4n2 , que pode assumir valores arbitrariamente
grandes para n natural. Logo, o conjunto de valores de xy + yz + zx e
o intervalo [12, +).

Pela desigualdade das medias, x+y+z 3 xyz. Logo, de xyz = 8,
3
resulta x + y + z 3 3 8 = 6, com igualdade quando x = y = z = 2.
Por outro lado, tomando-se x = y = n2 e z = 2n2 , tem-se xyz = 8 e
xy + yz + zx = 2n2 + n4 , que assume valores arbitrariamente grandes
para n natural. Logo, o conjunto de valores de x + y + z e o intervalo
[6, +).

4
SOLUES DO LIVRO
Coleo A Matemtica Do Ensino Mdio Vl. 04
Exerccios de MA 12
Sugestoes e Respostas

Unidades 5 e 6

Unidade 5 - Exerccios Recomendados


1. O aumento de um triangulo causa o aumento de 2 palitos. Logo, o
numero de palitos constitui uma progressao aritmetica de razao 2. an =
a1 + (n 1)r = 3 + (n 1)2 = 2n + 1.
(n2 + 1)n2
4. A soma de todos os elementos da matriz e 1 + 2 + + n2 = .
2
Como a soma de todos os elementos e igual a n vezes a constante magica,
esta vale
1 (n2 + 1)n2 n(n2 + 1)
C= = .
n 2 2
5. Considerando a menor e a maior das medias que podem ser obtidas,

1 + 2 + + (n 1) 2 + 3 + + n
6 16, 1 6 .
n1 n1
n n+2
16, 1
2 2
30, 2 n 32, 2
n so pode valer 31 ou 32.
7. A soma pedida e a soma de uma progressao aritmetica de razao 1, com

1
primeiro termo igual a 10n1 e ultimo termo igual a 10n 1.
(10n1 + 10n 1)(10n 10n1 ) 102n 102n2 10n + 10n+1
S = =
2 2
1
= 2
[102n + 10n1 ] 21 [102n2 + 10n ]

n1 n n4 n+1
z }| { z }| { z }| { z }| {
1
= 2
1000 . . . 00 1000 . . . 00 12 1000 . . . 001 1000 . . . 00

n3 n n3 n1
z }| { z }| { z }| { z }| {
1
= 2
98 999 . . . 99 1000 . . . 00 = 494 999 . . . 99 55 000 . . . 00 .
8. Quem disser 55 ganha o jogo, pois nao permite ao adversario alcancar
63 e, escolhendo o complemento para 8 do numero escolhido pelo adversario,
alcancara 63.
Analogamente, as posicoes ganhadoras sao 63, 55, 47, 39, 31, 23, 15,
7. O primeiro jogador tem a estrategia ganhadora: comecar dizendo 7 e, a
partir da, escolher sempre o complemento para 8 do numero escolhido pelo
adversario.
9. O Botafogo joga 23 partidas, o Santos joga (sem contar a partida contra
o Botafogo, ja contada) 22 vezes etc. A resposta e
(23 + 0) 24
23 + 22 + 21 + + 1 + 0 = = 276.
12
10. Veja a pizza de Steiner na Unidade 4.
11a. Um ano nao-bissexto tem 52 semanas e 1 dia; um ano bissexto tem 52
semanas e 2 dias. Logo, o ano x + 1 comeca um dia da semana adiantado em
relacao ao ano x, se x nao e bissexto, e dois dias adiantado, se x e bissexto.
De 1997 a 2500 sao multiplos de 4 os anos 2000, 2004, 2008, . . ., 2496,
num total de 125 anos. Mas 2100, 2200, 2300 nao sao bissextos por serem
multiplos de 100, mas nao de 400. Ha, portanto, 122 anos bissextos.
11b. Se o ano de 1997 comecou numa quarta-feira, o ano de 2500 comecara
(2500 -1997)+122=625 dias de semana depois. Como 625 = 7 89 + 2, o
ano de 2500 comecara numa sexta-feira.
11c. Em cada bloco de 400 anos ha 100 anos que sao multiplos de 4 e, destes,
3 nao sao bissextos por serem multiplos de 100, mas nao de 400. A resposta
97
e 400 = 0, 2425.

2
1
Pn 1
Pn 3n+1 3
3k = k k
P
14a. k=1
2
3 (3 1) = 2 k=1 3 = .
4
14b. nk=1 kk! = nk=1 [(k + 1)k! k!] =
P P
Pn Pn
k=1 [(k + 1)! k!] = k=1 k! = (n + 1)! 1.
n n   n
X 1 X 1 1 X 1
14c. = = =
k=1
k(k + 1) k=1 k k + 1 k=1
k
 
1 n
1 = .
n+1 n+1

Unidade 5 - Exerccios Suplementares


1. A soma dos angulos internos de um pentagono convexo e 540o .

2. x (3 x) = 9 x (x).
3. Do inteiro a (inclusive) ao inteiro b (inclusive), ha b a + 1 inteiros.
4. Faca um diagrama para os conjuntos X = {x Z : 100 6 x 6 500},
A = {x X : x e divisvel por 2}, B = {x X : x e divisvel por 3}
e A = {x X : x e divisvel por 5}. Queremos determinar o numero de
elementos do complementar de A B C em relacao ao universo X.
5. Se para passar do 32 para o 227 e para o 942 avancamos respectivamente
p 195
p e q termos, temos 227 32 + pr e 942 = 32 + qr. Da, = . Como p
q 910
e q sao inteiros positivos, e facil descobrir todos os valores possveis para p e
195
q; basta descobrir todas as fracoes que sao iguais a .
910
(2a + n + 1)n
6. Se 100 = (a + 1) + (a + 2) + + (a + n), com n > 1, 100 = .
2
Da se conclui que (2a + n + 1)n = 200 e tanto nquanto 2a + n + 1 devem ser
divisores de 200. Para evitar muitas contas, note tambem que sempre um
dos numeros n e 2a + n + 1 e mpar.
7. Uma solucao muito bonita pode ser obtida pensando nos pontos riscados
como vertices de um polgono. Uma solucao normal pode ser obtida ob-
servando que o ultimo numero riscado na primeira volta e 991, o primeiro
riscado na segunda volta e 6, etc...

3
10. Faca a soma de todas as fracoes e subtraia a soma das redutveis, que
sao as que tem numeradores multiplos de 2 ou 3. Um diagrama de conjuntos
ajuda.
11. Voce pode substituir 1000! = 1 2 3 4 5 1000 por 7 14
21 994 = 7142 (1 2 3 142).
12a. Parta de (k + 1)4 = k 4 + 4k 3 + 6k 2 + 4k + 1 e proceda como no Exemplo
13.
Xn Xn
12b. (2k 1)(3k + 1) = (6k 2 k 1).
k=1 k=1

13a. bxc = k, k > 0, se e somente se k 6 x < k + 1. b xc = k, k > 0 se,
e somente se, k 2 6 x < k 2 + 2k + 1. Ha portanto 2k + 1 inteiros positivos x
n1
X
para os quais b xc = k. A soma pedida e (2k + 1)k.
k=1
1
13d. se x e um inteiro positivo, { x} = k, k 6 0, se e somente se k <
2
1 2 1 2 1 2
x < k + , ou seja, k k + < x < k + k + , ou ainda k k + 1 6
2 4 4
x 6 k 2 + k. Ha 2k inteiros positivos x tais que { x} = k.
15. O primeiro elemento da 31a linha e precedido por 1 + 2 + + 30 termos.
18. Em algum momento o segundo jogador recebera a soma maior do que
ou igual a 49.
19. Considere a bobina formada por crculos cujos raios formam uma pro-
gressao aritmetica cuja razao e a espessura do papel.
20a. Se nao considerassemos as linhas suprimidas, teramos a progressao
aritmetica dos numeros naturais e o primeiro elemento da linha k seria pre-
k(k 1)
cedido por 1 + 2 + + (k 1) = naturais, sendo, portanto, igual
2
2
k(k 1) k k+2
a 1+ = .
2 2
Com a supressao, o que passa a ser a linha k e a antiga linha 2k 1. Logo,
(2k 1)2 (2k 1) + 2
o primeiro elemento da linha k e = 2k 2 3k + 2.
2
20b. Como na linha k ha 2k 1 elementos, o elemento central e 2k 2 3k +
2 + (k 1) = 2k 2 2k + 1.

4
20c. Como os elementos da linha k formam uma progressao aritmetica, basta
multiplicar o termo medio pela quantidade de termos.
A resposta e (2k 2 2k + 1)(2k 1) = 4k 3 6k 2 + 4k 1.
20d. A soma pedida e
k
X k
X
3 2
(4n 6n + 4n 1) = [n4 (n 1)4 ] = k 4 04 = k 4 .
n=1 n=1

21. Basta mostrar que an e an+1 sao polinomios de grau p cujos termos de
maior grau sao identicos e cujos termos de grau p 1 sao diferentes.
22. Se F (k) = ap k p + ap1 k p1 + + a1 k + a0 , entao
n
X n
X n
X n
X n
X
F (k) = ap k p + ap1 k p1 + + a1 k+ a0 .
k=1 k=1 k=1 k=1 k=1

23. Os termos da primeira progressao sao da forma 2 + 3t, 0 6 t 6 110 e os


da segunda sao da forma 7 + 5s, 0 6 s 6 30. Devemos ter 2 + 3t = 7 + 5s.
Da, 3t = 5(1 + s) e t deve ser multiplo de 5. Se t = 5k, s = 3k 1. As
limitacoes 0 6 t 6 110 e 0 6 s 6 30 dao origem a uma limitacao para k.
24. Procure primeiramente entender porque os calendarios de 1985 e 1991
sao iguais. Em segundo lugar, note que, como ha mais anos nao-bissextos
do que bissextos, provavelmente a colecao ficara completa quando Benjamim
tiver todos os calendarios de anos bissextos.

Unidade 6 - Exerccios Recomendados


5. O valor, em mil reais, do carro com n anos de uso forma a progressao
geometrica na qual a0 = 18 e a4 = 12. Determine a1 .
6. Sejam a, aq, aq 2 os numeros.
a + aq + aq 2 = 19
a2 + a2 q 2 + a2 q 4 = 133.
Da, a(1 + q + q 2 ) = 19
a2 (1 + q 2 + q 4 ) = 133
133
Dividindo, a(1 q + q 2 ) = =7
19

5
1 + q + q2 19
Da, =
1 q + q2 7
3 2
q = ou q = .
2 3
7. Se 2 1 e primo,os divisores de 2p1 (2p 1) sao da forma 2 (2p 1) ,
p

com {0, 1, . . . , p 1} e {0, 1}. Para calcular a soma dos divisores,


some separadamente os divisores que tem = 0 e os que tem = 1.
8. A k-esima parcela da soma e 1 + 10 + 102 + + 10k1 . A soma e igual a
n n
X 10k 1 1X k n 10 10n 1 n 10n+1 10 9n
= 10 = = .
k=1
9 9 k=1 9 9 9 9 81

1
9. Em cada operacao, a quantidade de vinho diminui de .
p
 2
4 4 2 94 5
10a. 5 + 2 5 + 2 5 + = 5 + = 13 metros.
9 9 1 49
10b. O tempo que uma bola gasta, partindo do repouso, para cair de uma
p 4
altura h e 2h/g. Como as alturas (em metros) das quedas sao 5, 5,
 2 9
4
5, . . ., supondo g = 10m/s2 , os tempos de queda (em segundos) serao
9
 2
2 2
1, , , . . ..
3 3
 2
2 2 1
O tempo total de queda e 1 + + + = = 3 segundos.
3 3 1 32
A este tempo devemos adicionar o tempo gasto pela bola nas subidas,
que e o mesmo, a excecao do 1s da queda inicial.
A resposta e 5s, aproximadamente.
11. lim an = 300 + 0, 3 200 + 0, 32 300 + 0, 33 200 + . . ..
14. A2 = 5A.

Unidade 6 - Exerccios Suplementares


8. Comece pela progressao aritmetica xr, x, x+r. A progressao geometrica
x x+r
sera xr+1, x, x+r. Temos (xr+1)+x+(x+r) = 19 e = .
xr+1 x

6
x+6 x6
9. Os numeros sao x 6, x, x + 6, x 6 e = .
x x+6
10. O numero e 9 + 8 10 + 8 102 + + 8 10n + 4 10n + + 4 102n1 .
11. Cada operacao dobra o numero de folhas. Use 210 = 1024 = 103 .
14b. Sao duas progressoes geometricas.
S
14c. Sendo S a soma pedida, calcule e subtraia.
2
14c. Sendo S a soma pedida, calcule xS e subtraia.
14e. Sao tres progressoes geometricas.
15. Os triangulos sao semelhantes e a razao de semelhanca de cada um para
o anterior e sempre a mesma.
1 1
16. A abscissa do ponto assintotico e 2 1 + + . . .
2 4
1 1 1
19a. A expressao e igual a x 2 . x 4 . x 8 . . .
1 1 1
20c. O que acontece com os pontos de abscissas
, , etc?
3 32 33
21. Tomando 1600 anos como unidade de tempo, a massa existente no
instante t e M (t) = M (0)0, 5t .
22. a = 1 + 10 + 102 + + 10n1 e b = 10n + 5.
 n
4 3 4
23. Pn+1 = e An+1 + .
3 12 9
 k
2 1
26a. Somacao por partes com ak+1 = k e bk =
2
26b. Somacao por partes com ak+1 = k e bk = 2k .

7
Exerccios Resolvidos de MA 12

Unidades 7 e 8

Unidade 7

Secao 4
Exerccios Recomendados
1.

x2 = 2x1 + x0 = 2.1 + 1 = 3
x3 = 2x2 + x1 = 2.3 + 1 = 7
x4 = 2x3 + x2 = 2.7 + 3 = 17
x5 = 2x4 + x3 = 2.17 + 7 = 41

2. O numero maximo de regioes e determinado quando, para cada n, a


reta n+1 intersecta as n ja existentes. Neste caso, a nova reta subdivide
n+1 regioes, criando assim n+1 novas regioes. Logo, o numero maximo
de regioes xn determinado por n retas satisfaz xn+1 = xn +(n+1), para
n = 0, 1, 2, . . . , com x0 = 1.

3. Por inducao sobre n. O valor de x1 esta bem definido, ja que x1 = a.


Suponhamos agora que xn esteja bem definido. Entao, como xn+1 =
f (xn ), o valor de xn+1 tambem esta bem definido para todo natural n.

4. Por inducao sobre n. Os valores de x1 e x2 estao bem definidos, ja


que x1 = a e x2 = b. Suponhamos agora que xn e xn+1 estejam bem
definidos. Entao, como xn+2 = f (xn , xn+1 ), o valor de xn+2 tambem
esta bem definido. Logo, pelo Principio da Inducao Finita, o valor de
xn esta bem definido para todos natural n.

5. (a) A razao xn+1 /xn entre dois termos consecutivos e constante e igual
a 2. Logo, a sequencia e uma progressao geometrica de razao 2. Como
o primeiro termo e x1 = 3, o termo geral e dado por xn = 3.2n1 . (b) A

1
diferenca xn+1 xn entre dois termos consecutivos e constante e igual
a 3. Logo, a sequencia e uma progressao aritmetica de razao 3. Como
o primeiro termo e x1 = 2, o termo geral e dado por xn = 2 + 3n 1 =
3n + 1.

Secao 4
Exerccios Suplementares
1. O circulo n + 1 e subdividido em no maximo 2n arcos pelos n ja exis-
tentes. Cada um destes arcos subdivide uma regiao existente, deter-
minando assim 2n regioes. Logo, o numero maximo xn de regioes de-
terminadas por n crculos satisfaz a recursao xn+1 = xn + 2n, com
x1 = 2.

2. Pelo Exemplo 2 em Para Saber Mais, o numero Dn de permutacoes


caoticas satisfaz a recursao Dn+2 = (n+1)(Dn+1 +Dn ), para n 1. Por
outro lado, D1 = 0 (ja que 1 necessariamente ocupa o seu proprio lugar)
e D2 = 1 (corresponde a permutacao (2,1)). Da, D3 = 2(D2 +D1 ) = 2,
D4 = 3(D3 + D2 ) = 9 e D5 = D4 + D3 = 44.

3. Podemos proceder por Inducao Finita. E imediato verificar que D1 =


0 = 1!( 0!1 1!1 ) e D2 = 1 = 2!(( 0!1 1!1 ) + 2!1 ). Suponhamos agora que a
expressao do numero de permutacoes caoticas esteja correta para n e

2
n + 1. Temos:
Dn+1 =(n + 1)(Dn+1 Dn )
n n+1
!
X (1)k X (1)k
=(n + 1) n! + (n + 1)!
k=0
k! k=0
k!
n
!
X (1)k (1)n+1
= (n + 1)((n! + (n + 1)!) + (n + 1)!
k!
k=0
(n + 1)!
n
!
X (1)k
= (n + 1)(1 + n + 1)n! + (n + 1)(1)n+1
k=0
k!
n
!
X (1)k
= (n + 1)(n + 2)n! + ((n + 2) 1)(1)n+1
k=0
k!
n
!
k
X (1) (n + 2)! (n + 2)!
=(n + 2)! + (1)n+1 + (1)n+2
k=0
k! (n + 1)! (n + 2)!
n+2
X (1)k
=(n + 2)!
k=0
k!

Secao 6
Exerccios Recomendados
1. Do problema 2 da secao anterior, o numero maximo xn de regioes em
que n retas podem dividir o plano satisfaz a recorrencia xn+1 = xn +
n + 1, com x0 = 1. Da:
x0 = 1
x 1 = x0 + 1
x 2 = x1 + 2

xn = xn1 + n

Somando, resulta:
n(n + 1)
xn = 1 + 1 + + n = 1 +
2

3
2. Seja xn o numero de sequencias de n termos 0 ou 1 com quantidade
mpar de termos iguais a 0. O numero de sequencias de n + 1 termos
0 ou 1 com numero mpar de termos iguais a 0 e igual ao numero de
sequencias comecadas com 1, seguindo de uma sequencia de n ter-
mos com numero mpar de zeros somado ao numero de sequencias
comecadas com 0, seguido de uma sequencia de n termos com um
numero par de zeros. Portanto, xn+1 = xn + (2n xn ) = 2n (para
a segunda parcela, note que 2n e o numero total de sequencias forma-
das por 0 ou 1). Logo, xn = 2n1 , para todo n.

3. Sequencias de n + 1 termos 0, 1 ou 2 com um numero mpar de termos


iguais a 0 podem ser de dois tipos: as que comecam com 1 ou 2, seguido
por uma sequencia de n termos com numero mpar de zeros e as que
comecam com 0, seguido por uma sequencia de n termos com numero
par de zeros. Da, temos a recorrencia xn+1 = 2xn + (3n xn ), ou seja,
xn+1 = xn + 3n , com x1 = 1. Termos:

x1 = 1
x2 = x1 + 3 1

xn = xn1 + 3n1

Somando, resulta
3n 1
xn = 1 + 3 + + 3n1 = .
2

4. Para Sheila ganhar a (n + 1)-esima partida, ou ela ganha a n-esima


partida (com probabilidade xn ) e ganha a seguinte (com probabilidade
condicional 0,6) ou perde a n-esima (com probabilidade 1xn ) e ganha
a seguinte (com probabilidade condicional 0,4). Logo, a probabilidade
xn+1 de vitoria na (n + 1)-esima partida e dada por xn+1 = 0, 6xn +
0, 4(1 xn ), ou seja, xn+1 = 0, 2xn + 0, 4, com x1 = 0, 4. Para resolver
a recorrencia comecamos com uma solucao nao nula de xn+1 = 0, 2xn ;
por exemplo, an = (0, 2)n1 . Fazendo a substituicao xn = (0, 2)n1 yn ,
0,4
temos (0, 2)n yn+1 = (0, 2)n yn + 0, 4, ou seja, yn+1 = yn + (0,2) n , com

4
y1 = x1 /a1 = 0, 4. Temos:

y1 = 0, 4
0, 4
y2 = y1 +
0, 2
0, 4
y3 = y2 +
(0, 2)2

0, 4
yn = yn1 +
(0, 2)n1

Somando, vem

0, 4 0, 4 1 (0, 2)n
yn = 0, 4 + + + = 0, 4
0, 2 (0, 2)n1 0, 8(0, 2)n1

Finalmente
1 (0, 2)n
xn = (0, 2)n1 yn = .
2
5. a) ma solucao da equacao homogenea xn+1 = (n + 1)xn e an = n!.
Fazendo a substituicao xn = an yn , temos

(n + 1)!yn+1 = (n + 1)n!yn + n,

ou seja,
n
yn+1 = yn + ,
(n + 1)!
x1 !
com y1 = 1!
= 1.
Da:

y1 = 1
1
y2 = y1 +
2!

n1
yn = yn1 +
n!

5
Somando:
1 n1
yn = 1 + + + .
2! n!
n1 n1 1
Mas n!
= n!
= (n1)!
n!1 . Logo
   
1 1 1 1 1 1
yn = 1 + ( ) + + +
1! 2! (n 2)! (n 1)! (n 1)! n!
1 1
=1+1 =2
n! n!

Finalmente
xn = n!yn = 2n! 1.
b) Uma solucao da equacao homogenea (n + 1)xn+1 = nxn e an =
(1)n
n
. Fazendo a substituicao xn = an yn , temos

(1)n+1 yn+1 = (1)n+1 yn + 2n 3,

ou seja,
yn+1 = yn + (1)n+1 (2n 3),
x1
com y1 = a1
= 1.
Assim, temos:

y1 = 1
y2 = y1 + (2 3)

yn = yn1 + (1)n (2(n 1) 3)

Somando:

yn = 1 1 1 + 3 5 + 7 9 + + (1)n (2(n 1) 3)

Quando n e par,

yn = 2 + (1 + 3) + (5 + 7) + ((2(n 2) 3)) + (2(n 1) 3)


= 2 + (n 2) = n 4
n
e x n = an y n = n
= 1 n4 .

6
Quando n e mpar, yn = (n 5) (2(n 1) 3) = n e xn = an yn = 1.
As duas expressoes podem sern colocadas em uma unica, escrevendo,
por exemplo: xn = 1 2+2(1)
n
.
c) Uma solucao da equacao homogenea xn+1 = nxn e an = (n 1)!.
Fazendo a substituicao xn = an yn , temos n!yn+1 = n!yn + (n + 1)!, ou
seja, yn+1 = yn + (n + 1), com y1 = x1 /a1 = 1.
Assim

y1 = 1
y2 = y1 + 2

yn = yn1 + n

Somando,
n(n + 1)
yn = 1 + 2 + + n =
2
(n+1)!
e x n = an y n = 2
.

Secao 6
Exerccios Suplementares
1. Seja xn o numero de coloracoes para n setores e consideremos o pro-
blema de colorir n + 1 setores. O primeiro setor pode ser colorido de k
modos e cada setor subsequente pode ser colorido de k 1 modos, ja
que nao pode receber a mesma cor do anterior, resultando em k(k 1)n
coloracoes. Mas este resultado inclui os casos em que o ultimo setor
recebe a mesma cor do primeiro, o que e proibido. Os casos contados
indevidamente correspondem as coloracoes que sao validas, exceto pelo
fato de dois setores adjacentes terem a mesma cor. Considerando es-
tes dois setores como um unico, estas coloracoes a serem descontadas
correspondem a coloracoes validas em um disco dividido em n setores.
Portanto, xn+1 = k(k 1)n xn , com x2 = k(k 1). Uma solucao

7
da equacao homogenea xn+1 = xn e an = (1)n1 . Fazendo a subs-
tituicao xn = an yn , vem (1)n yn+1 = (1)n yn + k(k 1)n , ou seja
yn+1 = yn + (1)n k(k 1)n , com y2 = x2 /a2 = k(k 1).
Assim

y2 = k(k 1)
y3 = y2 + k(k 1)2

yn = yn1 + (1)n1 k(k 1)n1

Da,

yn = k(k 1) k(k 1)2 + + (1)n1 k(k 1)n1 ,

que e a soma dos termos de uma P.G. de razao k(k 1). Logo,

1 (1)n1 (k 1)n1
yn = k(k 1) = (k 1) + (1)n (k 1)n .
1 + (k 1)

Finalmente,

(k 1) + (1)n (k 1)n
xn = an yn = n
= (1)n (k 1) + (k 1)n .
(1)

2. O numero de torcedores no ano n + 1 e pn+1 = pn (1 + i j) R.


Uma solucao da recorrencia homogenea pn+1 = pn (1 + i j) e an =
(1 + i j)n1 . Substituindo pn = an yn e fazendo r = 1 + i = j, vem
rn yn+1 = rn yn R, ou seja, yn+1 = yn rRn , com y0 = p0 /a0 = p0 r.
Assim

y0 = p0 r
y1 = y0 R
R
y2 = y1
r

R
yn = yn1
rn1

8
Somando
rn 1
 
1 1
yn = p0 r R 1 + + + n1 = p0 R n1 .
r r r (r 1)

Finalmente,

rn 1
 
n R R
p n = an y n = p 0 r R = p0 rn
r1 r1 r1
 
R R
= p0 (1 + i j)n + .
ij ij
 
R
A torcida se extingue quando o coeficiente p0 ij
de rn e negativo,
ou seja quando R > p0 (i j).

Unidade 8

Exerccios Recomendados
1. (a) As razes da equacao caracterstica r2 + 5r + 6 = 0 sao r1 = 2 e
r2 = 3. Logo, a solucao geral e xn = C1 (2)n + C2 (3)n , onde
C1 e C2 sao constantes arbitrarias.
(b) A equacao caracterstica r2 + 6r + 9 = 0 tem duas razes iguais a
-3. Logo, a solucao geral e xn = C1 (3)n + C2 n(3)n , onde C1 e
C2 sao constantes arbitrarias.
(c) As razes da equacao caracterstica r2 + 4r + 4 = 0 sao os numeros
complexos r1 = 1 + i e r2 = 1 i. Logo, a solucao geral
e xn = C1 (1 + i)n + C2 (1 i)n , onde C1 e C2 sao constantes
arbitrarias. Usando o fato de que as razes sao numeros complexos
de modulo 2 e argumento 3 4
, esta solucao pode ser escrita na
forma xn = C1 cos 3n 4
+ rsen 3n
4
.
(d) Como as razes da equacao caracterstica r2 5r + 6 = 0 sao
r1 = 2 e r2 = 3, a solucao geral da equacao homogenea e xn =
C1 2n + C2 3n . Por outro lado, tentando uma solucao particular

9
da forma xn = An + B, obtemos 2An + 2B 3A = n, que se
verifica quando A = 12 e B = 43 . Portanto, xn = 12 n + 34 e uma
solucao particular e, em consequencia, a solucao geral da equacao
nao-homogenea e xn = 12 n + 34 + C1 2n + C2 3n .
(e) A solucao geral da homogenea e a mesma do exerccio anterior.
Tentando uma solucao particular da forma xn = An + B + C4n ,
obtemos 2An + 2B 3A + 2C4n = 1 + 3.4n , que se verifica para
A = 0, B = 1/2 e C = 3/2, mostrando que 21 + 23 4n e uma solucao
particular. Logo, a solucao geral da equacao nao-homogenea e
xn = 12 + 32 4n + C1 2n + C2 3n .
(f) A solucao geral da homogenea e a mesma dos exerccios anteriores.
Como xn = 2n e solucao da homogenea, na busca por uma solucao
particular temos xn = An2n , o que leva a A(n + 2)2n+2 5A(n +
1)2n+1 + 6An2n . Da, obtemos A = 21 , que fornece a solucao
particular xn = 12 n2n = n2n1 . A solucao geral da equacao
nao homogenea e xn = 12 n2n = n2n1 + C1 2n + C2 3n .
(g) A solucao geral da homogenea e a mesma dos exerccios anteriores.
Como xn = 3n e solucao da homogenea e o termo nao homogeneo
e n + 3n , vamos a buscar uma solucao particular da forma xn =
An + B + Cn3n . Substituindo na equacao, obtemos

A(n + 2) + B + C(n + 2)3n + 2 5A(n + 1) 5B


5C(n + 1)3n+1 + 6An + 6B + Cn3n = n + 3n .

Simplificando, vem 2An3A+2B +5C3n = n+3n , que e satisfeita


para A = 1/2, B = 3/4 e C = 1/5. Logo, uma solucao particular
e xn = 21 n + 34 + 15 3n e a solucao geral da equacao nao homogenea
e xn = 12 n + 34 + 15 3n + C1 2n + C2 3n .
(h) A equacao caracterstica r2 6r +9 = 0 tem duas razes iguais a 3.
Logo, a solucao geral da equacao homogenea e xn = C1 3n +C2 n3n .
Como o termo nao homogeneo e n 3n , mas tanto 3n quanto n3n
sao solucoes da homogenea, tentamos uma solucao particular da
forma An + B + Cn2 3n . Substituindo na equacao, obtemos

A(n + 2) + B + C(n + 2)2 3n+2 6A(n + 1)


6B 6C(n + 1)2 3n+1 + 9An + 9B + 9Cn2 3n = n 3n .

10
Simplificando, vem 4An4A+4B+18C3n = n3n , que e satisfeita
por A = 1/4, B = 1/4 e C = 1/18, que leva a solucao particular
3 n 3 n
xn = n4 + 41 n 43 e a solucao geral xn = n4 + 14 n 43 +C1 3n +C2 n3n .
(i) As solucoes da equacao caracterstica r2 + 1 = 0 sao r1 = i e
r2 = i. A solucao geral da equacao homogenea e xn = C1 in +
C2 (i)n = C1 cos n2
+C2 sen n
2
. Tentando uma solucao particular
da forma xn = A, obtemos 2A = 1, ou seja, xn = A = 1/2.
A solucao geral da equacao nao homogenea e, portanto, xn =
1
2
+ C1 cos n2
+ C2 sen n
2
.
(j) Como no tem h), a solucao da homogenea e xn = C1 3n + C2 n3n .
Tentando, como em h), uma solucao particular da forma An +
B + Cn2 3n , chegamos a 4An 4A + 4B + 18C3n = 1 + n3n .
Observando que e necessario elevar o grau do termo que multiplica
3n ; por outro lado, nao e preciso incluir na solucao particular
um termo em n. Devemos tentar, assim, uma solucao particular
na forma xn = A + (Bn2 + Cn3 )3n . Substituindo na equacao e
simplificando obtemos 4A + 54Bn3n + (54B + 18C)3n = 1 + 3n ,
que e satisfeita para A = 1/4, B = 1/54 e C = 1/18, levando
2 n 3 n
a solucao particular xn = 41 + n543 n183 e a solucao geral xn =
1 2 n 3 n
4
+ n543 n183 + C1 3n + C2 3n .

2. (a) No Exerccio 1(a), encontramos a solucao geral xn = C1 (2)n +


C2 (3)n . Substituindo n = 0 e n = 1 na solucao, encontramos:

C1 + C2 =3
2C1 3C2 = 6

Resolvendo, encontramos C1 = 0 e C2 = 3, levando a solucao


xn = 3(2)n .
(b) As solucoes da equacao caracterstica r2 + r 6 = 0 sao r1 = 2
e r2 = 3, conduzindo a solucao geral xn = C1 2n + C2 (3)n
para a solucao homogenea. Tentando uma solucao particular da
forma xn = An + B, encontramos que a equacao e satisfeita para
A = 2 e B = 0. Assim, xn = 2n e uma solucao particular e
xn = 2n + C1 2n + C2 (3)n e a solucao geral da equacao nao
homogenea.

11
Substituindo n = 0 e n = 1 nesta solucao, obtemos:
C1 + C2 =1
2C1 3C2 =2

Resolvendo, encontramos C1 = 0 e C2 = 1. Portanto, a solucao e


xn = 2n + 2n
(c) A equacao caracterstica r2 4r + 4 = 0 tem duas razes iguais a
2, conduzindo a solucao geral xn = C1 2n + C2 n2n para a parte ho-
mogenea. Tentando uma solucao particular da forma xn = An2 2n ,
verificamos que xn = n2 2n e uma solucao particular e que, assim,
xn = n2 2n + C1 2n + C2 n2n e a solucao geral da equacao nao ho-
mogenea.
Substituindo n = 0 e n = 1 encontramos:
C1 =3
2C1 + 2C2 =4

Resolvendo, encontramos C1 = 3 e C2 = 1. Logo, a solucao da


equacao e xn = 32n n2n + n2 2n
3. Seja xn o numero de sequencias formadas por n termos iguais a 0, 1
ou 2 sem dois zeros repetidos. As sequencias de n + 2 termos que nao
tem dois termos consecutivos podem comecar por 0, 1 ou 2 . As que
comecam por 0 tem o proximo elemento igual a 1 ou 2 e, a seguir,
uma sequencia de n termos sem zeros repetidos. Logo, ha 2xn tais
sequencias. As que comecam por 1 ou 2 tem, a seguir uma sequencia
de n + 1 termos sem zeros repetidos. Logo, ha 2xn+1 sequencias deste
tipo. Assim, xn satisfaz a recorrencia xn+2 = 2xn + 2xn+1 , ou seja,
xn+2 2xn+1 2xn = 0, com x1 = 3 e x2 = 8 (todas as 3 sequencias de
comprimento 1 cumprem o requisito e todas as 32 = 9 de comprimento
2, exceto a 00, tambem cumprem a condicao).

As razesda equacao caracterstica r2 2r 2 =
0 sao r1 = 1 + 3 e
r2 = 1 3, levando a solucao geral xn = C1 (1 + 3)n + C2 (1 + 3)n
para a recorrencia. Substituindo n = 1 e n = 2, obtemos

(1 + 3)C1 + (1 3)C2 = 3

(3 + 2 3)C1 + (3 2 3)C2 = 8

12

Resolvendo o sistema, encontramos C1 = 3+26 3 e C2 = 326 3 . Logo o
numero de sequencias por ntermos iguais
a 0,
1 ou 2 sem dois zeros
repetidos e xn = 326 3 (1 + 3)n + 3+26 3 (1 3)n .
4. Considere um tabuleiro com 2 linhas e n + 2 colunas. Para preencher o
canto esquerdo do tabuleiro, ha duas alternativas: colocar um domino
em pe, restando tabuleiro com 2 linhas e n+1 colunas a preencher, ou
colocar dois dominos deitado restando um tabuleiro com 2 linhas e n
colunas. Logo, o numero xn de modos de preencher um tabuleiro 2 n
com dominos 2 1 satisfaz a recorrencia xn+2 = xn + xn+1 , com x1 = 1
e x 2 = 2. Esta e exatamente a sequencia de Fibonacci
 estudada no
 n+1  n+1
1 1+ 5 1 1 5
Exemplo 4. Logo, temos xn = Fn = 5 2
5 2
.

5. No ano n + 2 sao geradas 21 sementes para cada semente gerada no


ano n + 1 e 44 sementes para cada semente gerada nos anos anteriores.
Logo, se xn denota o numero de sementes geradas no ano n, temos
xn+2 = 21xn+1 + 44(xn + xn1 + + x1 + x0 ),
com x1 = 1 e x2 = 44 + 21.21 = 485. Para transformar esta recorrencia
em uma recorrencia linear de segunda ordem, escrevemos a expressao
para xn+1 :
xn+1 = 21xn + 44(xn1 + xn2 + + x1 + x0 ),

Subtraindo as duas expressoes, obtemos:


xn+2 = 22xn+1 + 23xn ,
ou seja,
xn+2 22xn+1 23xn = 0

A equacao caracterstica r2 22r23 = 0 tem razes r1 = 23 e r2 = 1,


levando a solucao geral xn = C1 23n + C2 (1)n para a recorrencia.
Usando as condicoes iniciais, obtemos
23C1 C2 =21
529C1 + C2 =485

Resolvendo, encontramos C1 = 11/12 e C2 = 1/12. A solucao da


recorrencia e, assim, xn = 11
12
1
23n + 12 (1)n .

13
6. A renda xn no mes n e igual ao salario Sn mais o rendimento sobre
montante yn1 das aplicacoes financeiras no mes anterior. Ou seja, xn =
Sn + iyn1 . Por outro lado, o montante das aplicacoes financeiras no
mes n e igual ao do mes anterior, acrescido do valor poupado no mes n.
Ou seja, yn = yn1 + p1 xn . Da primeira equacao, tiramos yn1 = xn S
i
n

e yn = xn+1 Si
n+1
. Substituindo estas expressoes na segunda
 equacao,
xn+1 Sn+1 xn Sn
vem i
= i
+ p1 xn , ou seja, xn+1 1 + i
p
. Fazendo a
n1 n1
substituicao xn = an zn , encontramos k zn = k zn1 + b, ou seja,
zn = zn1 + kn1b
. Assim, como x0 = a, temos z0 = a/k 1 = ak e:

z0 = ak
z1 = z0 + b

b
zn = zn1 +
k n1

Somando, vem:
1 kn
zn = ak + b
k n1 (1 k)
e
1 kn
xn = an zn = ak n + b
1k
 
Substituindo k = 1 + pi , temos, finalmente
  n
pb i pb
xn = a+ 1+ .
i p i

7. Seja pn a probabilidade de que a taca nao seja ganha nos primeiros


n torneios. Qualquer time pode ganhar o primeiro torneio. Vamos
exprimir pn+2 em funcao de pn e pn+1 usando probabilidade condicional.
Se o segundo torneio for ganho por um time diferente do que ganhou
o primeiro (o que ocorre com probabilidade 54 ), tudo se passa como se
a serie de torneios estivesse comecando no segundo torneio. Ou seja, a
probabilidade condicional de que a taca nao seja ganha ate o torneio
n + 2 e igual a pn+1 . Se o segundo torneio for ganho pelo mesmo time
do primeiro, mas no terceiro nao (ocorre com probabilidade 15 . 54 ), tudo

14
se passa como se a serie de torneios comecasse no terceiro jogo e a
probabilidade condicional de que a taca nao seja ganha ate o torneio
n + 2 torneio e igual a pn . Finalmente, se os tres primeiros torneios
forem ganhos pelo mesmo time, a taca e ganha na terceira realizacao e,
portanto, a probabilidade condicional de que ela nao tenha sido ganha
ate o torneio n + 2 e igual a zero, para todo n 1. Assim, temos
pn+2 = 45 pn+1 + 15 . 54 pn , com p1 = p2 = 1 (ja que a taca certamente
nao e ganha nas duas primeiras realizacoes).

A equacao

caracterstica
2 4 1 4 2+2 2 22 2
r 5 r 5 . 5 = 0 tem razes r1= 5 e r2 = 5 . Logo, a solucao
n n
2+2 2 22 2
geral da recorrencia e pn = C1 5
+ C2 5
.
Substituindo n = 1 e n = 2, temos:
! !
2+2 2 22 2
C1 + C2 = 1
5 5
! !
12 + 8 2 12 8 2
C1 + C2 = 1
25 25


10+5 2 105 2
Resolvendo, encontramos C1 =  eC =
16  2 16
, levando a solucao
n n
10+5 2 2+2 2 105 2 22 2
pn = 16 5
+ 16 5

8. Seja xn o numero de modos de obter 1 ou 2 pontos no primeiro jogo.


No primeiro caso, ele tem que obter n+1 pontos nos jogos seguintes; no
segundo caso, ele tem que obter n pontos a seguir. Logo, xn+2 = xn +
xn+1 , com x1 = 1 e x2 = 2. Esta e a recorrencia que define a sequencia
 n+1  n+1
de Fibonacci Fn . Logo, xn = Fn = 15 1+2 5 15 12 5 , para
todo n.

9. Os numeros r1 = (1 5) e r2 = (1+ 5) sao as razes daequacao r2
n 2 51
2r4 = 0. Portanto, a sequencia xn = 2 25+15
(1 5) + (1+ 5)n
2 5
satisfaz a recorrencia xn+2 = 2xn+1 + 4xn . Alem disso, substituindo os
valores n = 0 e n = 1, obtemos x0 = 2 e x1 = 1. Assim, x0 e x1
sao naturais e a recorrencia nos garante que, se xn e xn+1 sao naturais,
entao xn+2 tambem e. Portanto, pelo Principio de Inducao Finita, xn
e natural para todo n.

15

10. Para cada n natural, o numero xn = (1 + 3)2n+1 + (1 3)2n+1 e
um inteiro par, pois os termos que contem 3 se cancelam e os demais
(que sao inteiros) aparecem duas vezes. Alternativamente,
2 poderamos
2
argumentar como no exerccio 10: (1 + 3) e (1 3) sao razes da
equacao r2 8r 4 = 0. Logo, xn satisfaz a equacao de recorrencia
xn+2 = 8xn+1 + 4xn . Como x0 = 2 e x1 = 20, decorre por inducao que
xn e inteiro par para todo n.

Por outro lado, 1 < 1 3 < 0, ja que 3 1, 73. Logo, 1 <
(1 3)2n+1 < 0, pata todo n. Somando (1+ 3)2n+1 a
2n+1 todos os termos
desta
2n+1 desigualdade, obtemos (1 +
2n+1 3) 1 < (1 + 3)2n+1 + (1
2n+1
3) < (1+ 3) , ou seja, (1+ 3) 1 < xn < (1+ 3)2n+1 , o
que mostra que o numero inteiro par xn e a parte inteira de (1+ 3)2n+1 .

16
Exerccios Resolvidos de MA 12

Unidades 9 e 10

Unidade 9

Exerccios Recomendados
1. 600 = 450(1 + i)3
1/3
i = 600
450
1 = 10, 06%

2. (a) 1 + I = (1 + i)12
1 + 1 = (1 + i)12
i = 21/12 1 = 5, 95%
(b) 1 + I = (1 + i)3
1 + 0, 39 = (1 + i)3
i = 1, 391/3 1 = 11, 60%

3. (a) 1 + I = (1 + i)12
I = 1, 0612 1 = 101, 22%
(b) 1 + I = (1 + i)4
I = 1, 124 1 = 57, 37%

4. (a) A taxa e de 30%/12 = 2, 5% ao mes.


1 + I = (1 + i)12
I = 1, 02512 1 = 34, 49%
(b) A taxa e de 30%/4 = 7, 5% ao trimestre.
1 + I = (1 + i)4
I = 1, 0754 1 = 33, 55%.
(c) A taxa relativa ao perodo de capitalizacao e i/k.
1 + I = (1 + ki )k
I = (1 + ki )k 1

1
5. lim(1 + ki )k 1 = ei 1

6. O numero e e o valor de montante gerado em um ano por um principal


igual a 1, a juros de 100% ao ano, capitalizados continuamente.

7. (a) e 1 = e0,12 1 = 12, 75%


(b) ln(1 + i) = ln 1, 6 = 47, 00%
(c) Aproveitando o tem anterior, 47, 00%/2 = 23, 50%.

8. Seja 0 a data de compra. Seja 3 o preco do artigo. Usemos a data 1


como data focal.
Na alternativa (a), paga-se A = 3.
1
Na alternativa (b), paga-se B = 1+i
+ 1 + (1 + i)
i2
BA= 1+i
>0
Logo, como B > A, a alternativa (a) e prefervel.

9. (a) Supondo uma prestacao vincenda de 100 e tomando a data atual


como focal:
aceitando: pago, na data 0, 70.
nao aceitando: pago, na data 1, 100, o que equivale a pagar, na
data 0,
100
1+0,27
= 78, 74
A proposta e vantajosa.
(b) Supondo duas prestacoes vincendas de 100 cada uma e tomando
a data atual como focal:
aceitando: pago, na data 0, 120.
nao aceitando: pago, na data 1, 100, e na data 2, 100, o que
equivale a pagar na data 0,
100 100
1+0,27
+ (1+0,27)2 = 140, 74

A proposta e vantajosa.
(c) Supondo tres prestacoes vincendas de 100 cada uma e tomando a
data atual como focal:
aceitando: pago, na data 0, 150.
nao aceitando: pago, na data 1, 100, na data 2, 100, e, na data 3,
100, o que equivale a pagar, na data 0,

2
100 100 100
+ (1+0.27)
1+0.27 2 + (1+0.27)3 = 189, 56

A proposta e vantajosa
180 200
10. 1+0.25
+ (1+0.25)2
= 272
O preco a vista e R$ 272,00.

11. (a) Tomando a data focal um mes antes da compra,


3
P
= 1000 11,06
1,06 0,06
P = 352, 93
(b) Tomando a data focal no ato da compra,
3
P = 1000 11,06
0,06
P = 374, 11
(c) Tomando a data focal um mes depois da compra,
3
P.1, 06 = 1000 11,06
0,06
P = 396, 56

12. O montante pago foi 400.1, 034 .1, 055 .1, 09 = 626, 30.
A taxa media de juros e calculada por 626, 30 = 400.(1+i)10 , i = 4, 59%
ao mes.

Unidade 10

Exerccios Recomendados
10
1. (a) 400
= P 11,06
1,06 0,06
P = 51, 27
10
(b) 400 = P 11,06
0,06
P = 54, 35
10
(c) 400.1, 06 = P 11,06
0,06
P = 57, 61

2. (a) P = Ai = 50000.0, 006 = 300, 00

3
A P
(b) 1+i
= c
50000
P = 1,006
.0, 006 = 298, 21
11,005360
3. O montante que voce deve acumular e 100 = 0,005
= 16.679, 16
360
Para isso, P 11,005
0,005
.1, 005360 = 16.779, 16 e P = 16, 60
100
4. O montante que voce deve acumular e 0,005
= 20.000
420
Para isso, P 11,005
0,005
.005420 = 20.000 e P = 14, 04
0,1
5. A prestacao pela tabela Price e P = 3.000 11,1 8 = 562, 33

A amortizacao pelo SAC e 3.000/8 = 375

TABELA PRICE
EPOCA PRESTACAO JUROS AMORTIZACAO ESTADO DA DIVIDA
0 - - - 3 000,00
1 562,33 300,00 262,33 2737,67
2 562,33 273,77 288,56 2.449,11
3 562,33 244,91 317,42 2.131,69
4 562,33 213,17 349,16 1.782,53
5 562,33 178,25 384,08 1.398,45
6 562,33 139,84 422,49 975,96
7 562,33 97,60 464,73 511,23
8 562,33 51,12 511,23 -

SAC
EPOCA PRESTACAO JUROS AMORTIZACAO ESTADO DA DIVIDA
0 - - - 3 000,00
1 675,00 300,00 375,00 2.625,00
2 637,50 262,50 375,00 2.250,00
3 600,00 225,00 375,00 1.875,00
4 562,50 187,50 375,00 1.500,00
5 525,00 150.00 375,00 1,125,00
6 487,50 112,50 375,00 750,00
7 450,00 75,00 375,00 375,00
8 412,00 37,50 375,00 -

6. O montante e 0, 31.1, 1t + 0, 7.1, 18t


A taxa media de juros e calculada por 0, 3.1, 1t + 0, 1.1, 18t = 1.(1 +
i)t i = (0, 3.1, 1t + 0, 1.1, 18t )1/t 1, onde t e o numero de meses do

4
investimento. Se t = 1, a taxa e 15, 60%; se t = 2, e 15,66%; se t ,
a taxa e 18%.

7. (a) Usando a data da compra como data focal e considerando um


preco igual a 30,
10 10
21 = 10 + 1+i + (1+i)2

Resolvendo, i = 51, 08%.


(b) Tomando para data focal a data da compra e considerado um
preco igual a 30,
10 10 10
21 = 1+i + (1+i)2 + (1+i)3

Resolvendo, i = 20, 20%


(c) Tomando para data focal a data da compra e considerado um
preco igual a 30,
10 10 10
21 = (1+i) 2 + (1+i)3 + (1+i)4

Resolvendo, i = 12, 81%.

8. Arbitrando o preco em 100 e usando a data da compra como data focal,


50 50
100 x > 1,05
+ 1,052

x < 7, 03%

9. (a) 100 = 94(1 + i)


i = 6, 38%
(b) 100 = 88(1 + i)2
i = 6, 60%
(c) 100 = 72(1 + i)3
i = 6, 84%

10. Sem reciprocidade, receberia 88 para pagar 100 em dois meses. Com
reciprocidade, recebe 0, 8.88 = 70, 4 para pagar 100 17, 6.1, 022 =
81, 69.
81, 69 = 70, 4.(1 + i)2
i = 7, 72%

5
Exerccios Suplementares
1. (a) i/365 < i/360
(b) 1000.0, 12.16/360 = 5, 33
1000.0, 12.16/365 = 5, 26
(c) O montante e 1000.(1 + 0, 12)16/360 = 1005, 05 e os juros sao de
R$ 5,05; O montante e 1000.(1 + 0, 12)16/365 = 1004, 98 e os juros
sao de R$ 4,98.

2. 700.0, 12.11/30 = 30, 80.

3. (a) 48% ao trimestre.


(b) I = 1, 153 1 = 52, 09% ao trimestre.
(c) 100 = 64.(1 + I); I = 56, 25% ao trimestre.
A melhor e (a) e a pior e (c).

4. 1, 124 1 = 57, 35% sao os juros quadrimestrais. Deve cobrar 57, 35%/4 =
14, 33% ao mes.

5. (a) 300.1, 153 = 456, 26


8
(b) 300.1, 153+ 30 = 437, 59
(c) 456, 26 + 456, 26.0, 15.8/30 = 474, 51
0,01
6. (a) A prestacao e 35.000 11,01180 = 420, 06

180
(b) 420, 06 11,01
0,01
= 23.056, 28

7. (a) A amortizacao e 35.000/180 = 194, 44


A dvida na epoca da 99a prestacao e 81.194, 44 = 15750.
Os juros da centesima prestacao sao 157,50 e a centesima prestacao
e igual a 194,44+157,50=351,94
(b) O estado da dvida e 80.194, 44 = 15.555, 56

8. (a) Supondo a dvida igual a 100, a prestacao para 150 meses e


0,01
P150 = 100 11,01150 = 1, 29 e a prestacao para 300 meses e

0,01
P300 = 100 11,01300 = 1, 05

A reducao e de 0, 24/1, 29 = 18%, aproximadamente

6
9. (a) A dvida igual a 300, a prestacao para 150 meses e
P150 = 300
150
+3=5
A prestacao para 300 meses e P300 = 300
300
+3=4
A reducao e de 20%.
(b) 50%.
0,12
10. A original custa, por ano, 280 1,12(11,125 ) = 69, 35.

Como a alternativa implica em um custo anual de 70,00, e melhor


comprar a original.
300
11. O custo de compara e 2.000 1,0130
= 1.777, 42
0,01
Isso equivale a um custo mensal 1.777, 42 11,0130 = 68, 87 mais a ma-

nutencao, dando um custo mensal total de 73,87.


E melhor comprar.

12. .

dinheiro 100 1516


preco 1 12,09
Poder de compra 100 1516/12, 09
= 125

A rentabilidade real foi de 25%.

7
Exerccios Resolvidos de MA 12

Unidades 11 e 12

Unidade 11

Exerccios Recomendados
1. A primeira pergunta pode ser respondida de 5 modos; a segunda, de 5
modos, etc.
A resposta e 5 5 5 = 51 0 = 9.765.625.

2. Para formar um subconjunto, deve-se decidir, para cada elemento do


conjunto, se ele pertencera ou nao ao subconjunto. Ha 2 modos de
decidir o que fazer com o primeiro elemento do conjunto, 2 modos com
o segundo, etc.
A resposta e 2 2 2 = 2n
Outra Solucao:
Quando se acrescenta um elemento a um conjunto, os subconjuntos do
novo conjunto sao os subconjuntos dobra. Entao, se An e o numero de
subconjuntos de um conjunto com n elementos, (An ) e uma progressao
geometrica de razao 2. Logo, An = A0 .2n = 2n pois o conjunto vazio
possui um unico subconjunto.

3. A primeira pessoa tem 5 escolhas; a segunda, 4; a terceira, 3. A resposta


e 5 4 3 = 60.

4. Os bancos em que os homens se sentam podem ser escolhidos de 54


321 = 120 modos, o mesmo ocorrendo com os bancos das mulheres.
Em cada banco, os casais podem se sentar de 2 modos diferentes. A
resposta e 1202 25 = 460.800.

5. As 64 casas do tabuleiro dividem-se, naturalmente, em tres grupos:

i) as 4 casas dos vertices;

1
ii) as 24 casas da borda do tabuleiro, mas que nao sao vertices;
iii) as restantes 36 casas, que sao interiores ao tabuleiro.
Vamos separar a nossa contagem conforma o tipo de casa ocupada pelo
rei negro:
i) ha 4 possveis para o rei negro e 60 para o rei branco;
ii) ha 24 possveis para o rei negro e 58 para o rei branco;
iii) ha 36 possveis para o rei negro e 55 para o rei branco.
A resposta e 4 60 24 58 + 36 55 = 3612.
Se os reis sao iguais, a resposta passa a ser a metade da resposta ante-
rior, pois, trocando a posicao dos reis, agora obtem-se a mesma confi-
guracao.
6. Havera uma torre em cada linha e em cada coluna. A posicao da torre
da primeira linha pode ser escolhida de 8 modos; a da segunda linha,
de 7, etc.
A resposta e 8 7 6 5 4 3 2 1 = 40.320.
Se as torres fossem diferentes, para cada uma das escolhas de posicao,
teramos que escolher uma das torres. A resposta seria, portanto, 8
877665544332211 = (8!)2 = 1.625.702.400.
7. Se a primeira carta e o rei de copas, a segunda pode ser escolhida de 48
modos (pode ser qualquer carta, exceto os 4 reis). Se a primeira carta e
de copas mas nao e o rei, ela pode ser escolhida de 12 modos. Neste caso,
a segunda carta pode ser escolhida de 47 modos (nao pode ser a primeira
escolhida, nem nenhum dos 4 reis). A resposta e 48 + 12 47 = 612.
8. (a) Para construir uma funcao, devemos, para cada elemento de A,
escolher sua imagem em B. Ha 47 modos de escolher a imagem
do primeiro elemento de A, 7 modos de escolher a imagem do
segundo elemento, etc.
A resposta e 7 7 7 7 = 2.401.
(b) Para a funcao ser injetora, elementos diferentes devem ter imagens
diferentes. Ha 7 modos de escolher a imagem do primeiro elemento
de A, 6 modos de escolher a imagem do segundo elemento, etc.
A resposta e 7 6 5 4 = 840.

2
9. (a) Como 720 = 24 32 51 , 720 possui 5 3 2 = 30 divisores. Aos
pares, estes divisores formam produtos iguais a 720. Logo, ha 15
modos de escrever 720 como produtos de divisores.
(b) Como 144 = 24 32 , 144 possui 3 3 = 9 divisores. Com eles,
podem ser formados 4 pares de divisores cujo produto e 144 e,
alem disso, pode ser formado 12 12. Assim, ha 5 modos de
escrever 144 como um produto de divisores.

Unidade 12

Exerccios Recomendados
1. (a) O numero total de anagramas e 8! = 40.320.
(b) Ha 4 modos de escolher a vogal que sera a primeira letra do ana-
grama e 3 modos de selecionar a vogal que sera a ultima letra do
anagrama. Depois disso, ha 6! modos de arrumar as demais letras
entre a primeira e a ultima.
A resposta e 4 3 6! = 4 3 720 = 8.640.
(c) As vogais e consoantes podem aparecer na ordem CV CV CV CV
ou na ordem VC VC VC VC. No primeiro caso, devemos colocar as
4 vogais nos 4 lugares de ordem par (4! modos) e as 4 consoantes
nos 4 lugares de ordem mpar (4! modos).
Ha 4! 4! = 24 24 = 576 anagramas do primeiro tipo. Analo-
gamente, ha 576 anagramas do segundo tipo.
A resposta e 576 + 576 = 1.152.
(d) Tudo se passa como se CAP fosse uma unica letra. Devemos,
portanto, arrumar em fila 6 objetos: CAP,I,T,U,L,O.
A resposta e 6! = 720.
(e) Primeiramente, devemos escolher a ordem em que as letras C, A,
P aparecerao. Ha 3! modos. Depois, devemos arrimar em a fila 6
objetos: o bloco de letras C, A, P e as 5 letras I, T, U, L, O. Ha
6! modos.
A resposta e 3! 6! = 3 720 = 4320.

3
(f) Basta arrumar em fila, depois de PA, as restantes 6 letras.
A resposta e 6! = 720.
(g) Ha 7! anagramas com a letra P em primeiro lugar e ha 7! anagra-
mas com a letra A em segundo lugar. Ha tambem 6! anagramas
com a letra P em primeiro lugar e A em segundo lugar. Ao somar-
mos 7! com 7!, encontramos o numero de anagramas com P em
primeiro lugar ou A em segundo lugar, mas contamos duas vezes
os anagramas que tem P em primeiro lugar e A em segundo lugar.
A resposta e 7! + 7! 6! = 5040 + 5040 720 = 9.360.
(h) Ha 7! anagramas com a letra P em primeiro lugar, 7! anagramas
com a letra A em segundo e 7! anagramas com a letra C em ter-
ceiro. Ha tambem 6! anagramas com P em primeiro lugar e A em
segundo lugar, 6! anagramas com P em primeiro e C em terceiro
e 6! anagramas com A em segundo e C em terceiro. Finalmente,
ha 5! anagramas com P em primeiro lugar, A em segundo e C em
terceiro.
Ao somarmos 7! com 7! com 7!, encontramos o numero de ana-
gramas que tem P em primeiro lugar ou A em segundo ou C em
terceiro, mas contamos alguns anagramas varias vezes.
Contamos duas vezes os anagramas que tem P em primeiro lugar
e A em segundo; o mesmo se deu com os que tem P em primeiro
e C em terceiro e com os que tem A em segundo e C em terceiro.
Descontando essas contagens indevidas, chegamos a 7! + 7! + 7!
6! 6! 6! = 3.7! 3.6!.
Entretanto, anagramas com P em primeiro lugar e A em segundo e
C em terceiro foram, inicialmente, contados tres vezes e, posterior-
mente, descontados tres vezes, o que significa que nao estao sendo
contados. Incluindo-os na contagem, obtemos a resposta correta,
que e 3.(7!) 3.(6!) + 5! = 3.(5.040) 3.(720) + 120 = 13.080.
(i) Como ha 6 ordens possveis para as letras C, A e P, os anagramas
pedidos sao exatamente 1/6 do total, ou seja, 8!/6 = 6.720.
Outra solucao:
Basta escolher as 3 posicoes a serem ocupadas pelas letras P, A,
C, o que pode ser feito de C83 = 56 modos e distribuir 5 letras
restantes nas demais posicoes, o que pode ser feito de 5! = 120
modos. O total de anagramas e 56 120 = 6.720.

4
2. O valor de f (a1 ) pode ser escolhido de n modos; o valor de f (a2 ), de
n 1 modos; . . . ; o de f (an ), de 1 modo.
A resposta e n(n 1) 1 = n!.

3. O numero total de modos de sentar 8 pessoas em 8 cadeiras e o numero


de modos de arrumar 8 pessoas em fila, 8!. O numero de modos de
arrumar 8 pessoas em fila de modo que duas dessas pessoas, Vera e
Paulo, fiquem juntas e 2.7!, pois, para formar uma tal fila, devemos
inicialmente decidir em que ordem se colocarao Vera e Paulo e, em
seguida formar uma fila de 7 objetos: o bloco formado por Vera e
Paulo; as demais 6 pessoas.
A resposta e 8! 2.7! = 40.320 10.080 = 30.240.

4. Como visto no problema anterior, o numero de filas nas quais duas


pessoas (neste caso Helena e Pedro) ficam juntas e 2.7! = 10.080. O
numero de filas onde Helena e Pedro e tambem Vera e Paulo ficam
juntos e obtido de modo analogo: agora sao dois blocos de duas pes-
soas, cada um podendo ser arrumado de dois modos distintos e mais
4 pessoas. Portanto, o numero de tais filas e 2.2.6! = 2.880. Logo, o
numero de filas em que Helena e Pedro ficam juntos, mas Vera e Paulo
nao, e 10, 080 2.880 = 7.200.

5. O elemento da permutacao que ocupa o 10o lugar deve ser maior que 7.
Pode ser escolhido de 3 modos. O elemento da 9a posicao deve ser maior
que 6; haveria 4 possibilidades, mas uma delas ja foi usada na escolha
do elemento que ocupa a 10a posicao. Pode ser escolhido de 3 modos.
Prosseguindo com esse raciocnio, vemos que a cada nova casa abranda-
se a restricao, criando uma possibilidade a mais, mas ao mesmo tempo
diminui-se uma possibilidade, pois uma delas foi usada na etapa. Ou
seja, ha 3 possibilidades para cada casa ate a 3a casa. O elemento
da 3a posicao deve ser maior que 3 3 = 0; havera 10 possibilidades,
mas 7 delas ja foram usadas nas etapas anteriores. Pode ser escolhido
de 10 7 = 3 modos. O elemento da 2a posicao deve ser maior que
2 3 = 1; haveria 10 possibilidades mas 8 ja foram usadas nas etapas
anteriores. Pode ser escolhido de 10 8 = 2 modos. Finalmente, o
elemento de posicao 1 deve ser maior que 1 3 = 2; haveria 10
possibilidades, mas 9 delas ja foram usadas nas etapas anteriores. Pode
ser escolhido de 10 9 = 1 modo.

5
A resposta e 38 .2.1 = 13.122.
5 5
6. Ha C15 modos de formar o Esporte; depois disso, C10 modos de formar
o Tupi; finalmente, 1 unico modo de formar o Minas.
5 5
A resposta e C15 C10 1 = 756.756.

7. O numero de possibilidades e igual ao numero obtido no problema


anterior dividido por 3! = 6, ja que permutando os nomes dos times a
subdivisao continua a mesma. A resposta e 756.745/6 = 126.126.

8. Escolha, sucessivamente, 3 pessoas para formar os 4 grupos de 3; isto


3 3 3 3
pode ser feito, sucessivamente, de C20 , C17 , C14 e C11 modos. A seguir,
com as 8 pessoas restantes for os 2 grupos restantes, o que pode ser
feito de C84 e C44 modos, respectivamente. Fazendo isso, contamos cada
divisao 4!.2! vezes, porque, quando formamos os mesmos grupos de 3 e
os mesmos grupos de 4 em outra ordem, contamos como se fosse outra
divisao em grupos.
C 3 .C 3 .C 3 .C 3 .C 4 .C 4 20!
A resposta e 20 17 14 1 8 4 = = 67.897.830.000.
4!.3! (3!) (4!)2 4!2!
4

Outra solucao:
Forme uma fila com 20 pessoas. Isso automaticamente as divide em
4 grupos de 3 e 2 grupo de 4: as 3 primeiras formam um grupo, as 3
seguintes formam outro etc.. Ha 20! modos de formar a fila. Entretanto,
uma mesma divisao em grupos correspondentes a varias filas diferentes,
o que faz com que, no resultado 20!, cada divisao tenha sido contada
varias vezes. Devemos corrigir nossa contagem dividendo o resultado
pelo numero de vezes que cada divisao foi contada. Trocando a ordem
dos elementos em cada grupo, o que pode ser feito de 3!.3!.3!.3!.4!.4!
modos, ou a ordem dos grupos, o que pode ser feito de 4!.2! modos,
a divisao em grupos nao se altera, mas a fila sim. Cada divisao foi,
20!
assim, contada (6!)3 .2!.3! vezes e a resposta e
(3!) (4!)2 4!2!
4

9. Os adversarios em cada jogo podem ser escolhidos, sucessivamente, de


2 2
C12 , C10 , C82 , C62 , C42 e C22 modos. No entanto, assim contamos cada
possvel rodada 6! vezes, ja que contamos diferentes ordens dos jogos

6
2 2
C12 .C10 .C82 .C62 .C42 .C22
como se fossem rodadas diferentes. A resposta e =
6!
12!
= 10.395
26 .6!
Outra solucao:
Colocados os 12 times em fila automaticamente formamos os 6 jogos
da rodada. No entanto, a mesma rodada e contada varias vezes; os
adversarios em cada jogo podem ser ordenados de 2 modos, enquanto
os jogos podem ser ordenados de 6! modos. A resposta e, portanto,
12!
.
26 .6!
10. (a) Para determinar o lugar ocupado pelo numero 62.417, devemos
contar quantos numeros estao antes dele. Antes dele estao os
numeros comecados por:
i) 1 (4!=24 numeros)
ii) 2(4!=24 numeros)
iii) 4 (4!=24 numeros)
iv) 61 (3!=6 numeros)
v) 621 (4!=2 numeros)
Ha 24 + 24 + 24 + 6 + 2 = 80 numeros antes do 62.417.
A resposta e 81o
(b) Como ha 4! = 24 numeros comecados por 1, e 4! = 24 numeros
comecados por 2 e 3! = 6 numeros comecados por 41, e 3! = 6
numeros comecados por 42, e 3! = 6 numeros comecados por 46,
o 66o numero escrito e o ultimo dos numeros comecados por 46,
ou seja, 46.721.
A resposta e 46721.
(c) Como ha 5 algarismos em cada numero, o 166o algarismo escrito
e o primeiro algarismo do 36o numero.
Como ha 4! = 24 numeros comecados por 1, e 3! = 6 numeros
comecados por 21, e 3! = 6 numeros comecados por 24, o 36o
numero escrito e o ultimo dos numeros comecados por 26. Logo,
seu primeiro algarismo e 2.
(d) Nas casas das unidades desses numeros, aparecem apenas os alga-
rismos 1, 2, 4, 6, 7, cada um deles 4! = 24 vezes. A soma das unida-
des desses numeros e, portanto, (1+2+4+6+7).24 = 480 unidades,

7
ou seja, 480. A soma das dezenas e, analogamente, igual a 480 de-
zenas, ou seja, 4.800. A soma das centenas e igual a 480 centenas,
ou seja, 48.000. A soma das unidades de milhar e igual a 480 uni-
dades de milhar, ou seja, 480.000. Finalmente, a soma das dezenas
de milhar e igual a 480 dezenas de milhar, ou seja, 4.800.000. A
resposta e 480 + 4.800 + 48.000 + 480.000 + 4.800.000 = 5.333.280.
Outra Solucao:
Ha 5! = 120 parcelas na soma. Podemos agrupa-las em 60 pares,
juntando a cada numero o que dele se obtem trocando o 1 com o
7, trocando o 2 com o 6, e conservando a posicao do 4. Em cada
par, a soma vale 88.888.
A resposta e 88.888 60 = 5.333.280

8
Exerccios Resolvidos de MA 12

Unidades 13 e 14

Unidade 13

Exerccios Recomendados
1. C72 + C73 + + C77 = 27 C70 C71 = 128 1 7 = 120.
p
2. (a) C10 e elemento da linha 10. Em qualquer linha, o elemento maximo
e o do meio. A resposta e p = 5.
p
(b) C21 e elemento da linha 21. Em qualquer linha, o elemento maximo
e o do meio, A resposta e p = 10 ou p = 11.
 p
p (1) p
3. Tp+1 = C10 x 2 (x3 )10p = C10 (1)p x305p independera de x para
30 5p = 0, ou seja, p = 6.
6
A resposta e T7 = C10 (1)6 x0 = 210.
n(n1) n2
4. (1 x2 ) = 1 2x + x2 e (1 + x)n = xn + nxn1 + 2
x + . Os
termos em xn no produto sao:
n(n1) n2 n(n1) n
1.xn = xn ; 2x.xn1 = 2nxn ; x2 2
x = 2
x .
n(n1) n2 5n+2)
A resposta e 1 2n + 2
= 2
.
n
X
5. A soma pedida e Cnk 3k = (1 + 3)n = 4n .
k=0

6. (a) Seja P (x) = (1 + x + x2 )n . Entao, A0 + A1 + A2 + + An =


P (1) = 3n .
(b) Sejam S0 = A0 +A2 + +A2n e S1 = A1 +A3 + +A2n1 . Entao
P (1) + P (1)
P (1) = S0 + S1 e P (1) = S0 S1 . Lodo, S0 = =
n
2
3 +1
.
2

1
p 1 p
 120! 1
7. Tp+1 = C120 2
= pe
p!(120 p)! 2
p1 1 p1
 120! 1
Tp = C120 2
= p1 ;
(p 1)!(121 p)! 2
Tp+1 121 p
= ; Tp+1 > Tp se p 40 e Tp+1 < Tp se p 41
Tp 2p
Da T1 < T2 < < T40 < T41 > T42 > > T121 .
40
C120
O termo maximo e T41 = 40 .
2
8. Sejam a = 10150 e b = 10050 + 9950 . Temos:

a =(100 + 1)50 = C50 0


10050 + C50
1
10049 + C50
2
10048
49 50
+ + C50 100 + C50
b =10050 + (100 1)50 = C50 0
10050 C50
1
10049 + C50
2
10048
49 50
C50 100 + C50 + 10050

Logo,

1
a b = 2C50 10049 + 2C50 3
10047 + + 2C50
49
100 10050
1
= 2C50 10049 + + 2C50
49
100 > 0.

Portanto, a > b.

Unidade 14

Exerccios Recomendados
1. Um armario ficara aberto se ele for mexido um numero mpar de vezes.
Por outro lado, o armario de ordem k e mexido pelas pessoas cujos
numeros sao divisores de k. Logo, estarao abertos os armarios cujos
numeros possuem um numero mpar de divisores. Isto ocorre com os

2
numero cujos expoentes sao todos pares na decomposicao em fatores
primos, ou seja, sao quadrados perfeitos. Assim, permaneceram abertos
os armarios cujos numeros sao quadrados perfeitos, ou seja, os numeros
12 , 22 , . . . , 302 .

2. Separemos o caso em que o primeiro e o terceiro quadrantes tem cores


iguais do caso em que eles tem cores diferentes.
No caso de cores iguais, ha 5 modos de escolher a cor unica para o pri-
meiro e o terceiro quadrante, 4 modos de escolher a cor para o segundo
quadrante e 4 modos de escolher a cor para o quarto quadrante. Ha,
portanto, 5 4 4 = 80 modos de colorir o mapa usando cores iguais
no primeiro e no terceiro quadrantes.
No caso de cores diferentes, ha 5 modos de escolher a cor para o primeiro
quadrante, 4 modos de escolher a cor para o terceiro quadrante, 3 modos
de escolher a cor para o segundo quadrante e 3 modos de escolher a cor
para o quarto quadrante. Ha 5 4 3 3 = 180 modos de colorir o
mapa usando cores iguais no primeiro e no terceiro quadrantes.
No total, temos, portanto, 80 + 180 = 260 modos de colorir a figura.

3. (a) Ha 265 = 11.881.376 palavras de 5 letras. Delas, devemos subtrair


as palavras que comecam por A, 1 264 = 456.976, e aquelas nas
quais a letra A nao figura, 255 = 9.765.625.
A resposta e 11.881.376 456.976 9.765.625 = 1.658.775.
(b) O numero total de palavras de 5 letras distintas e 26252423
22 = 7.893.600. Delas devemos subtrair as palavras que comecam
por A, 1 25 24 23 22 = 303.600 e aquelas nas quais a letra
A nao figura, 25 24 23 22 21 = 6.375.600.
A resposta e 7.893.600 303.600 6.375.600 = 1.214.400.
Outra Solucao:
Ha 4 posicoes para colocar a letra A; depois disso, as quatro casas
vazias podem ser preenchidas de 25, 24, 23 e 22 modos.
A resposta e 4 25 24 23 22 = 1.214.400.

4. Cada letra pode ser escolhida de 26 modos, enquanto cada algarismo


pode ser escolhido de 10 modos. Logo, o numero total de placas e
263 104 = 175.760.000.

3
5. O numero de modos de acomodar os passageiros que pretendem sentar
de frente e 5 4 3 2 = 120; o numero de modos de acomodar os
passageiros que pretendem sentar de costas e 5 4 3 = 60; o numero
de modos de acomodar os demais passageiros e 3 2 1 = 6.
A resposta e 120 60 6 = 43.200.

6. Vamos primeiramente determinar quantos zeros sao escritos na casa


das unidades, depois na das dezenas, etc.
Ha 222 numeros que tem o como algarismo das unidades, pois antes do
zero podem ser colocados os inteiros de 1 (inclusive) a 222 (inclusive).
Ha 22 10 = 220 numeros que tem 0 como algarismo nas dezenas,
pois antes do zero podem ser colocados os inteiros de 1 (inclusive) a 22
(inclusive) e depois do zero, os inteiros 0 (inclusive) a 9 (inclusive).
Ha 2 100 = 200 numeros que tem 0 como algarismo das centenas,
pois antes do zero podem ser colocados os inteiros de 1 (inclusive) a 2
(inclusive) e depois do zero os inteiros de 0 (inclusive) a 99 (inclusive).
A resposta e 222 + 220 + 200 = 642.

7. E mais simples contar, primeiramente, os numeros onde o algarismo


5 nao aparece. O primeiro digito pode ser escolhido de 8 modos (nao
pode ser igual a 0 nem igual a 5) e cada um dos demais tres dgitos
pode ser selecionado de 9 modos (deve ser diferente de 5). Logo, ha
893 = 5.832 numeros de a algarismos em que nao aparece o algarismo
5.
A quantidade de numero de 4 dgitos, com ou sem o digito 5, e 9103 =
9.000 (pois ha 9 modos de selecionar o primeiro digito, que deve ser
diferente de 0, e 10 modos de selecionar cada um dos demais 4 dgitos).
Logo, ha 9.000 5.832 = 3.168 numeros de 4 algarismos em que o 5
nao aparece.

8. Devemos decidir quantos exemplares de cada revista devem ser postos


na colecao. Ha 6 possibilidades para a Veja (0, 1, 2, 3, 4, ou 5
exemplares), 7 para a Manchete e 5 para a Isto e. O numero de
colecoes e 6 7 5 = 210, e o numero de colecoes nao-vazias e 209.

9. Em cada dia, duas das materia sao ensinadas e uma folga. Ha 3 possi-
bilidades para escolher a materia que folga na segunda, 2 para escolher

4
a que folga na quarta e 1 para escolher a que folga na sexta. Portanto,
ha 6 modos para escolher as materias de cada dia. Para escolher os
horarios, ha 2 possibilidades em cada dia. Logo, o numero total de
horarios e 6 8 = 48.

10. Foi feita uma distincao artificial ao se considerar cada casal ordenado
de dois modos diferentes: comecando pela mulher ou pelo homem. Por
esta razao, o resultado encontrado foi igual ao dobro do correto.

11. Ha tres tipos de cartoes: os que virados de cabeca para baixo nao
representam numeros, como, por exemplo, 41.809; os que virados de
cabeca para baixo representam o mesmo numero, como, por exemplo,
86.198; os que virados de cabeca para baixo representam numeros di-
ferentes, como, por exemplo, 66.810. Os cartoes do ultimo tipo sao
os que permitem economia porque um mesmo cartao serve para re-
presentar dois numeros. Ha 5 5 5 5 5 = 3.125 cartoes que
virados de cabeca para baixo representam numero, iguais ou diferen-
tes, pois tais cartoes devem ter como algarismos apenas 0, 1, 8, 6 ou
9. Destes, 5 5 3 = 75 sao do segundo tipo, pois um tal cartao
deve ter as casas das extremidades preenchidas por 00, 11, 88, 69 ou
96, a segunda e a quarta casa preenchidas por 00, 11, 88, 69 ou 96, e a
casa central preenchida por 0, 1 ou 8. Portanto, os cartoes do terceiro
tipo sao em numero de 3.125 75 = 3.050. Podem ser economizados
3.050/2 = 1.525 cartoes. O numero mnimo de cartoes que se necessita
e 100.000 1.525 = 98.475.

12. A decomposicao de 360 em fatores primos e 720 = 23 .32 .5. Os divisores


inteiros e positivos de 720 sao numeros da forma 2 .3 .5 , com
{0, 1, 2, 3}, {0, 1, 2} e {0, 1}. A soma dos divisores e S =
2 .3 .5 , com {0, 1, 2, 3}, {0, 1, 2} e {0, 1}. Para
P
calcular essa soma, dividimos
P as 0parcelas
P em dois grupos,
P conforme seja
1
= 0 ou = 1. S = (2 .3 .5 ) + (2 .3 .5 ) = 6 (2 .3 ) porque
a segunda soma e igual ao quntuplo da primeira. Agora, dividimos
asPparcelas emPgrupos, conforme
P 2seja =P0, = P 1 ou =P 2. S =
0 1
6[ (2 .3 ) + (2 .3 ) + (2 .3 )] = 6[ 2 + 3 2 + 9 2 ] =
6[13 2 ] = 78 2 = 78[20 + 21 + 22 + 23 ] = 78 15 = 1.170.
P P

5
Exerccios Resolvidos de MA 12

Unidades 15 e 16

Unidade 15

Exerccios Recomendados
1. Devemos inicialmente escolher a ordem em que as moca ficarao juntas,
o que pode ser feito de m! maneiras. Em seguida, devemos arrumar
em fila r + 1 objetos, os r rapazes e o bloco das mocas, o que pode ser
feito de (r + 1)! modos.
A resposta e m!(r + 1)!.

2. (a) A face a receber o numero 1 pode ser escolhida de 6 modos, a do


numero 2 de 5 modos, e assim por diante. O numero de possibili-
dades e 6.5.4.3.2.1 = 720.
(b) Inicialmente, fazemos a conta que as faces tem cores diferentes.
Contamos, pelo problema anterior, 720 dados. Como as faces
sao indistinguveis, o mesmo dado foi contado varias vezes. Por
exemplo, pense em um dado que tenha o 6 na face de baixo (face
preta) e o 1 na face de cima (face branca). Ele e, certamente,
diferente de um dado que tenha o 1 na face de baixo (face preta) e o
6 na face de cima (face branca). Mas sendo as faces indistinguveis,
o dado que tem 6 na face de baixo e o 1 na face de cima e igual
ao dado que tem o 1 na face de baixo e o 6 na face de cima; este
e, simplesmente, aquele de cabeca para baixo. Esse mesmo dado
aparece outra vez com o 1 na face da frente e o 6 na face de tras,
com o 1 na face da esquerda e o 6 na face da direita, etc. Em suma,
o mesmo dado foi contado tantas vezes quantas sao as posicoes de
coloca-lo.
O numero de posicoes de colocar um cubo e 6 4 = 24, pois ha
6 modos de escolher a face de baixo e 4 de escolher, nessa face, o
lado que fica de frente.

1
720
A resposta e = 30.
24
Outra solucao:
Todo dado pode ser imaginado com a face 1 embaixo. Realmente,
se o 1 nao estiver embaixo, e possvel rodar o dado de modo que
o 1 va para baixo. Fixando o 1 embaixo, devemos escolher quem
ocupara a face oposta a face do 1. Isso pode ser feito de 5 modos.
Digamos que tenha sido escolhido o 6. Com o 1 fixo embaixo e
o 6 fixo em cima, devemos colocar os numeros 2, 3, 4 e 5 nas
faces laterais. O 2 sempre pode ser imaginado na face da frente.
Com efeito, se o 2 nao estiver na face da frente, uma conveniente
rotacao coloca-lo-a na face da frente, sem tirar o 1 da face de baixo
nem o 6 da face de cima. Fixamos o 2 na frente, o 1 embaixo e o
6 em cima, devemos escolher quem ocupara a face oposta a face
do 2. Isso pode ser feito de 3 modos. Digamos que tenha sido
escolhido o 4. Agora, devemos colocar o 3 e o 5 nas faces da
direita e da esquerda. Note que qualquer movimento com o dado
ou retirara o 1 de baixo, ou o 6 de cima, ou o 2 da frente, ou o
4 de tras, Portanto, ha 2 modos de preencher as faces direita e
esquerda com os numeros 3 e 5.
A resposta e 5 3 2 = 30.
(c) Um dado com faces de cores diferentes pode, agora, ser numerado
de apenas 6.4.2 = 48 modos, ja que temos 6 faces a escolher para o
numero 1 (isto determina a face do numero 6), a para o numero 2
(o que determina a face do 5) e 2 para o numero 3 (que determina
a do 4). Mas como as faces sao iguais, cada dado e contado, como
no tem anterior, 24 vezes. Logo ha apenas 48/24 = 2 dados
distintos.
Outra Solucao:
Como antes, podemos fixar o dado com o numero 1 embaixo.
Agora, no entanto, isto tambem fixa o numero 6 na frente de cima.
Agora, o numero 2 pode ser fixado na face da frente (e, portanto,
o numero 5 na de tras). Assim, tudo que temos a escolher e se a
face lateral da direita e 3 ou o 4. Temos, portanto, apenas duas
possibilidades.
3. (a) O numero de posicoes para um tetraedro e 4 3 = 12, pois ha 4
modos de escolher a face de apoio e 4 de escolher, nessa face, o

2
lado que fica de frente.
4!
A resposta e = 2.
12
(b) O numero de posicoes para um octaedro regular e 6 4 = 24, pois
ha e modos de escolher o vertice de apoio e 4 de escolher, dentre
as arestas que incidem nesse vertice, a que fica de frente.
8!
A resposta e = 1.680.
24
(c) O numero de posicoes para um dodecaedro regular e 12 5 = 60,
pois ha 12 modos de escolher a face de apoio e 5 de escolher, nessa
face, o lado que fica de frente.
12!
A resposta e = 7.938.360.
60
(d) O numero de posicoes para um icosaedro regular e 20 3 = 60,
pois ha 20 modos de escolher a face de apoio e 3 de escolher, nessa
face, o lado que fica de frente.
20!
A resposta e = 40.548.366.802.944.000 = 4.1016 .
60
4. Temos 1! = 1, que e um quadrado perfeito, 1! + 2! = 1 + 2 = 3, que
nao e quadrado perfeito, 1! + 2! + 3! = 1 + 2 + 6 = 9, que novamente e
quadrado perfeito, 1! + 2! + 3! + 4! = 1 + 2 + 6 + 24 = 33, que nao e
quadrado perfeito. Todos os fatorais seguintes, a partir de 5! terminam
com
Pn zero, ja que sao multiplos de 5 e 2. Logo, todas as somas da forma
1 k! para n 5 terminam com o algarismo 3 e nao sao, portanto,
quadrados perfeitos. As unicas solucoes sao n = 1 e n = 3.

5. Em ESTRELADA as letras A e E aparecem 2 vezes cada e as letras S,


T, R, L e D aparecem uma vez cada uma, havendo, portanto, 9 letras
na palavra.
Para formar um anagrama, devemos escolher 2 das 9 posicoes para
colocar as letras A, o que pode ser feito de C92 modos, 2 das 7 posicoes
restantes para colocar as letras E, o que pode ser feito de C72 modos, e
arrumar as letras S, T, R, L e D nas 5 posicoes restantes, o que pode
ser feito de 5! modos. A resposta e C92 .C72 .5! = 90.720.
Outra solucao:
9!
O numero de anagramas e P92,2,1,1,1,1,1 = = 90.720.
2!2!1!1!1!1!1!
3
6. Formar uma solucao com p elementos significa escolher p dos n elemen-
tos. A resposta e Cnp .

7. Basta escolher as provas do primeiro dia, o que pode ser feito de C84 = 70
modos.

8. O processo de contagem apresentado conta determinadas comissoes


mais de uma vez. Isto ocorre porque um homem que participe da
comissao pode ser inserido de dois modos diferentes: como um dos 3
homens escolhidos dos inicialmente, ou como uma das duas pessoas
escolhidas posteriormente. O pior e que nao e possvel corrigir a con-
tagem dividendo pelo numero de vezes que cada comissao e contada:
as comissoes com 3 homens sao contadas apenas uma vez, as que tem
4 homens sao contadas 4 vezes, enquanto a que contem 5 homens e
contada 10 vezes.
A solucao correta e dada no Exemplo 6, contando separadamente as
comissoes com 3, 4 e 5 homens: C53 .C42 + C54 .C41 + C55 = 81.

9. Os segmentos que unem dos vertices de um poliedro ou sao arestas ou


sao diagonais de faces ou diagonais do poliedro.

(a) O octaedro regular e um poliedro formado por 8 faces triangulares


e que tem 6 vertices e 12 arestas. Ha C62 = 15 segmentos que unem
dois vertices do poliedro, pois 12 dos quais sao arestas e 0 dos quais
e diagonal de face.
A resposta e 15 12 0 = 3.
(b) O icisaedro regular e um poliedro formado por 20 faces triangu-
2
lares e que tem 12 vertices e 30 arestas. Ha C12 = 66 segmentos
que unem dois vertices do poliedro, 30 dos quais sao arestas e 0
dos quais e diagonal de face.
A resposta e 66 30 0 = 36.
(c) O dodecaedro regular e um poliedro formado por 12 faces pentago-
2
nais e que tem 20 vertices e 30 arestas. Ha C20 = 190 segmentos
que unem dois vertices do poliedro, 30 dos quais sao arestas e
5(5 3)
12 = 60 dos quais sao diagonais de faces.
2
A resposta e 190 30 60 = 100.

4
(d) O cubo e um poliedro formado por 6 faces quadradas e que tem 8
vertices e 12 arestas. Ha C82 = 28 segmentos que unem os vertices
4(4 3)
do poliedro, 12 dos quais sao arestas e 6 12 dos quais sao
2
diagonais de faces.
A resposta e 28 12 12 = 4.
Outra solucao:
Cada diagonal de um prisma n-agonal une um vertice da base de
cima a um vertice da base de baixo. O vertice da base de
cima pode ser selecionado de n modos; depois disso, o da base
de baixo pode ser selecionado de n 3 modos, pois um dos
vertices da base de baixo, se selecionado daria origem a uma
aresta e os dois vertices adjacentes nesta base dariam origem a
diagonais das faces laterais. O numero de diagonais de um prisma
n-agonal e portanto, n(n 3).
Como o cubo e um prisma quadrangular, a resposta e 4(43) = 4.
(e) O prisma hexagonal e um poliedro formado por 6 faces quadran-
gulares e 2 faces hexagonais e que tem 12 vertices e 18 arestas. Ha
2
C12 = 66 segmentos que unem os vertices do poliedro, 18 dos quais
4(4 3) 6(6 3)
sao arestas e 6 +2 = 30 dois quais sao diagonais
2 2
de faces.
A resposta e 66 18 30 = 18.
Outra solucao:
O numero de diagonais de um prisma n-agonal e, como visto em
d), n(n 3). Portanto, o numero de diagonais de um prisma
hexagonal e 6(6 3) = 18.

10. Uma funcao estritamente crescente e necessariamente injetiva (se f (a) =


f (b), nao pode ser a < b, pois, neste caso, f (a) < f (b), o que e absurdo;
do mesmo modo, nao pode ser a > b, pois, neste caso, f (a) > f (b),
o que e absurdo; logo, a = b), Logo, seu conjunto de valores tera
exatamente m elementos. Para construir uma tal funcao, devemos, ini-
cialmente selecionar o conjunto de valores, o que pode ser feito de Cnm
modos.
Selecionando o conjunto de valores, a funcao esta determinada porque
f (1) deve ser igual ao menor elemento do conjunto de valores, f (2)

5
deve ser igual ao segundo menor elemento do conjunto de valores, etc.
A resposta e, portanto, Cnm .

11. Vamos esquecer que a primeira casa do numero nao pode ser igual a
zero. Isso fara com que contemos a mais e, depois, descontaremos o
que foi contado indevidamente.
Ha C73 modos de escolher as casas que serao ocupadas pelo digito 4;
depois disso, ha C42 modos de selecionar as casas que serao ocupadas
pelo digito 8; finalmente, as duas casas restantes podem ser preenchidas
de 8 8 modos (nao podemos usar nessas casas os dgitos 4 e 8).
A resposta seria C73 C42 8 8 = 35 6 64 = 13.440.
Devemos subtrair os numeros comecados por zero. Se o numero comeca
por 0, ha C63 modos de escolher as casas que serao ocupadas pelo digito
4; depois disso, ha C32 modos de selecionar as casas que serao ocupadas
pelo digito 8; finalmente, a casa restante pode ser preenchida de 8
modos (nao podemos usar nessa casa os dgitos 4 e 8). Ha C63 C32 8 =
20 3 8 = 480 numeros comecados por 0.
A resposta e 13.440 480 = 12.960
Outra solucao:
Vamos a contar separadamente:
i) numeros que comecam com 4; ii) numeros que comecam com 8; iii)
numeros que nao comecam nem com 4 nem com 8.

i) Ha 1 modo de preencher a primeira casa; depois disso, ha C62


modos de escolher as outras duas casas do numero que tambem
serao preenchidas com o algarismo 4; depois disso, ha C42 modos
de escolher as duas casas que serao ocupadas pelo algarismo 8;
finalmente, as duas casas restantes podem ser preenchidas de 88
modos (nao podemos usar nessas casas os dgitos 4 e 8).
Ha 1 C62 C42 8 8 = 1 15 6 64 = 5.760 numeros do
tipo i).
ii) Ha 1 modo de preencher a primeira casa; depois disso, ha 6 modos
de escolher a outra casa do numero que tambem sera preenchida
com o algarismo 8; depois disso, ha C53 modos de escolher as tres

6
casa que serao ocupadas pelo algarismo 4; finalmente, as duas ca-
sas restantes podem ser preenchidas de 8 8 modos (nao podemos
usar nessas casas os dgitos 4 e 8).
Ha 1 6 C53 8 8 = 6 10 64 = 3840 numeros do tipo ii).
iii) Ha 7 modos de preencher a primeira casa (nao podemos usar nem
4, nem 8, nem 0); depois disso, ha C63 modos de escolher as tres
casas do numero que serao preenchidas com o algarismo 4; depois
disso. ha C32 modos de escolher as duas casas que serao ocupadas
pelo algarismo 8; finalmente, a casa restante pode ser preenchida
de 8 modos (nao podemos usar nessas casa os dgitos 4 e 8).
Ha 7 C63 C32 8 = 7 20 3 8 = 3.360 numeros de tipo iii).
A resposta e 5.760 + 3.840 + 3.360 = 12.960.

12. (a) Para formar o subconjunto devemos escolher os p 1 outros ele-


mentos do subconjunto dentre os n 1 outros elemento do con-
junto.
p1
A resposta e Cn1 .
(b) Para formar o subconjunto devemos escolher os p elementos do
subconjunto dentre os n 1 outros elementos do conjunto.
p
A resposta e Cn1 .
Outra solucao:
p1
Ha Cnp p-subconjuntos e o elemento a1 figura em Cn1 deles. Logo,
p1
ha Cnp Cn1 subconjuntos nos quais o elemento a1 nao figura.
p1
A resposta e Cnp Cn1 .
Observacao: As duas solucoes apresentadas mostram que Cnp
p1 p
Cn1 = Cn1 . Essa e a famosa Relacao de Stifel.
(c) Para formar o subconjunto devemos escolher os p 2 outros ele-
mentos do subconjunto dentre os n 2 outros elementos do con-
junto.
p2
A resposta e Cn2 .
(d) O total de p-subconjuntos e Cnp . Para formar um subconjunto em
que nem a1 nem a2 figurem devemos escolher os p elementos do
subconjuntos dentre os n 2 outros elementos do conjunto. Ha,
p
portanto, Cn2 subconjuntos nos quais nem a1 nem a2 figuram.

7
Logo, o numero de subconjuntos nos quais pelo menos um desses
p
dois elementos figura e Cnp Cn2 .
Outra solucao:
p1 p1
Ha Cn1 p-subconjuntos nos quais o elemento a1 figura e ha Cn1
p2
subconjuntos nos quais o elemento a2 figura. Ha, tambem, Cn2
p-subconjuntos nos quais os elementos a1 e a2 figuram ambos. Ao
p1 p1 p1
somarmos Cn1 + Cn1 = 2Cn1 obtemos o numero de subcon-
juntos nos quais pelo menos um dos elementos a1 e a2 figura, mas
contamos duas vezes aqueles que a1 e a2 figuram ambos.
p1 p2
A resposta e, portanto, 2Cn1 Cn2 .
Outra solucao:
p2
Ha, como mostrado em c), Cn2 p-subconjuntos em que os ele-
mento a1 e a2 figuram ambos.
p1
Ha Cn2 p-subconjuntos em que o elemento a1 figura e o elemento
a2 nao figura, pois, para formar um tal subconjunto, basta escolher
os outros p1 elementos do subconjunto dentre os n2 elementos
do conjunto que sao diferentes de a1 e de a2 .
p1
Ha analogamente, Cn2 p-subconjuntos em que o elemento a2
figura e o elemento a1 nao figura. Portanto, o numero de p-
subconjuntos em que figura pelo menos um desses dois elementos
p1 p2
e 2Cn2 + Cn2 .
p1
(e) Como visto na solucao anterior, a resposta e 2Cn2 .
Outra solucao:
p1 p2
Ha, como visto em d). 2Cn1 Cn2 p-subconjuntos nos quais
pelo menos um dos elementos a1 e a2 figura. Ha, como visto
p2
em c), Cn2 p-subconjuntos em que os elementos a1 e a2 figuram
ambos.
p1 p2 p2 p1 p2
A resposta e, portanto, 2Cn1 Cn2 Cn2 = 2Cn1 2Cn2 .
Outra solucao:
p1 p2
Ha como visto em d), 2Cn2 + Cn2 p-subconjuntos nos quais pelo
menos um dos elementos a1 e a2 figura. Ha, como visto em c),
p2
Cn2 p-subconjuntos em que os elementos a1 e a2 figuram ambos.
p1 p2 p2 p1
A resposta e, portanto, 2Cn2 + Cn2 Cn2 = 2Cn2 .
2
13. (a) Como ha 32 cartas, a resposta e C32 = 201.376.

8
(b) Ha 8 modos de escolher o grupo do par propriamente dito (por
exemplo, valete), C42 modos de escolher os naipes das duas cartas
do par (por exemplo, copas e paus), C73 modos de escolher os
grupos das outras tres cartas (por exemplo, 10, 8 e rei) e 444 =
43 modos de escolher os naipes dessas tres cartas.
A resposta e 8 C42 C73 43 = 107.520.
(c) Ha C82 modos de escolher os grupos das cartas que formarao os dois
pares, (C42 )2 modos de escolher seus naipes, 6 modos de escolher
o grupo da outra carta e 4 modos de escolher seu naipe.
A resposta e C82 (C42 )2 6 4 = 24.192.
Observacao: Um erro muito comum e o exposto a seguir.
Ha 8 modos de escolher o grupo do primeiro par, C42 modos de
escolher os naipes do primeiro par, 7 modos de escolher o grupo
do segundo par, C42 modos de escolher os naipes do segundo par,
6 modos de escolher o grupo da outra carta e 4 modos de escolher
seu naipe. A resposta ERRADA seria 8 C42 7 C42 6
4 = 48.384. A explicacao do ERRO e simples: Ao fazermos a
inexistente distincao entre primeiro par e segundo par, contamos
pares de valetes e reis como diferentes de pares de reis e valetes.
A resposta ERRADA pode ser corrigida dividindo-a por 2.
(d) Ha 8 modos de escolher o grupo da trinca, C43 modos de escolher
os naipes das cartas da trinca, C72 modos de escolher os grupos
das outras duas cartas e 4 4 = 42 modos de escolher os naipes
dessas duas cartas.
A resposta e 8 C43 C72 42 = 10.752
(e) Ha 8 modos de escolher o grupo do four, 1 modo de escolher os
naipes das quatro cartas do four, 7 modos de escolher o grupo
da outra carta e 4 modos de escolher o naipe dessa carta.
A resposta e 8 1 7 4 = 224.
(f) Ha 8 modos de escolher o grupo da trinca, C43 modos de escolher
os naipes das cartas da trinca, 7 modos de escolher o grupo do
par e C42 modos de escolher os naipes das cartas do par.
A resposta e 8 C43 7 C42 = 1.344
(g) Ha apenas 4 tipos de sequencias: 7. 8. 9. 10, valete, dama, rei;
10, valete, dama, rei, as. Escolhido o tipo da sequencia, haveria

9
4 4 4 4 4 4 = 45 modos de escolher os naipes das cartas
das sequencias, mas 4 desses modos nao sao permitidos: todas de
ouros, todas de paus, todas de copas e todas de espadas.
A resposta e 4 [45 4] = 4.080.
(h) Os grupos das cartas podem ser escolhidos de C85 4 modos e o
naipe unico, de 4 modos.
A resposta e (C85 4) 4 = 208.
(i) Ha 4 modos de escolher os grupos de cartas e 4 modos de escolher
o naipe unico.
A resposta e 4 4 = 16.
(j) Ha 4 modos de escolher o naipe unico. A resposta e 4.

14. (a) Neste caso f e bijetiva e, se #A = #B = n, o numero de funcoes


f : A B bijetivas e n!, como foi mostrado no exerccio 4 da
secao 2.2.
(b) Neste caso dois elementos de A terao uma mesma imagem em B
e a correspondencia entre os demais n 1 elementos de A e os
demais n 1 elementos de B sera bijetiva.
Ha n+1

2
modos de escolher os dois elementos de A, n modos de
escolher a imagem deles em B e (n 1)! modos de construir uma
correspondencia bijetiva entre os elementos restantes.
n(n + 1)!
A resposta e n+1

2
n (n 1)! = .
2
(c) Neste caso temos as alternativas
i) Tres elementos de A tem a mesma imagem em B e a corres-
pondencia entre os demais n 1 elementos de A e os demais
n 1 de B e bijetiva.
Ha n+23
modos de escolher os tres elementos de A, n modos
de escolher a imagem deles em B e (n1)! modos de construir
uma correspondencia bijetiva entre os elementos restantes.
n(n + 2)!
Ha n+2

3
n (n 1)! = funcoes desse tipo.
6
ii) Ha dois pares de elementos de A com imagens identicas em
B e a correspondencia entre os demais n 2 elementos de A
e os demais n 2 elementos de B e bijetiva.

10
Ha n2 modos de escolher os dois elementos de B, n+2 n2
  
2
modos de escolher suas imagens inversas em A e (n2)! modos
de estabelecer a correspondencia entre os elementos restantes.
n(n 1)(n + 2)!
Ha n2 n+2 n2 (n 2)! =
  
2
funcoes
8
desse tipo.
A resposta e:

n(n + 2)! n(n 1)(n + 2)! n(3n + 1)(n + 2)!


+ = .
6 8 24
15. Chamemos de D o conjunto C C1 .
Ha quatro tipos de planos:

i) determinados por tres pontos de D;


ii) determinados por dois pontos de D e um de C1 ;
iii) determinados por um ponto de D e dois de C1 ;
iv) determinados por tres pontos de C1 .
3 2
A resposta e C12 + C12 .8 + 12.C82 + 1 = 1.085
Outra solucao:
Para determinar um plano, devemos selecionar 3 dos 20 pontos, o que
3
pode ser feito de C20 = 1140 modos. Nessa contagem, o plano que
contem os 8 pontos de C1 foi contado C83 = 56 vezes.
A resposta e 1.140 56 + 1 = 1.085.

Unidade 16

Exerccios Recomendados
1. Escolhida a ordem de cada casal, o que pode ser feito de 23 modos,
temos que arrumar em fila 4 espacos vazios e 3 casais, o que pode ser
feito de C74 modos (escolha dos espacos vazios) vezes 3! (colocacao dos
3 casais nos 3 lugares restantes).
A resposta e 23 C74 3! = 1.680.

11
2. Primeiro, colocamos as vogais. Como a letra A aparece 3 vezes e as
letras U, I e O aparecem 1 vez cada, o numero de modos de dispo-las
e P63,1,1,1 = 3!
6!
= 720
6
= 120. A seguir, colocamos as consoantes em
tres dos 7 espacos antes, entre e depois das vogais. O lugar do P pode
ser qualquer um destes 7 espacos, o do R qualquer dos 6 restantes e
o do G qualquer dos 5 restantes. O numero total de possibilidades e
120.7.6.5 = 25.200.

3. Vamos a formar uma fila com os numeros 1, 2, . . . , n e assinalar com E


os p numeros escolhidas e com N os n p nao escolhidos. A condicao
para que nao sejam escolhidos numeros consecutivos e que entre dois
E haja pelo menos um N. Comecamos escrevendo os n p E. A seguir,
devemos escolher, para colocar os E, p dentre os n p + 1 espacos
p
situados antes, entre e depois dos N. Isto pode ser feito de Cnp+1
modos.

4. Chegam 4 cientistas A, B, C, D. Com as chaves que possuem, abrem


alguns cadeados, mas nao todos. Existe pelo menos um cadeado que
eles nao conseguem abrir. Na situacao do numero mnimo de cadeados,
existe exatamente um cadeado que eles nao conseguem abrir. Batize
tal cadeado de ABCD. Portanto, ABCD e p cadeado cuja chave nao
esta em poder de A, nem de B, nem de C e nem de D. Qualquer outro
cientista tem a chave desse cadeado, pois esse cientista e A, B, C e
D formam um grupo de 5 cientistas e, portanto, nesse grupo alguem
possui a chave. Como o alguem nao e nem A, nem B, nem C e nem D,
deve ser o outro. Analogamente batize os demais cadeados. Verifique
agora que a correspondencia entre cadeados e seus nomes e biunvoca,
isto e, cadeados diferentes tem nomes diferentes (isso porque estamos
na situacao do numero mnimo de cadeados) e cadeados de nomes di-
ferentes sao diferentes (se X esta no nome de um cadeado e nao esta
no nome de outro, X tem a chave deste e nao tem a chave daquele).

(a) O numero mnimo de cadeados e igual ao numero de nomes de


4
cadeados, C11 = 330.
(b) Cada cientista X possui as chaves dos cadeados que nao possuem
4
X no nome, C10 = 210.

5. Nenhum aluno pode comparecer a mais de tres jantares. Com efeito, se


A1 vai a um jantar com A2 e A3 , ele so pode ir a outro jantar com outros

12
dois estudantes, digamos A4 e A5 e so poder ir a um terceiro jantar em
companhia de outros dois, digamos A6 e A7 e nao tera companhia para
ir a um quarto jantar. Como ha 21 convites e sao 7 estudantes, cada
estudante tera que compadecer a exatamente 3 jantares.
Se A1 comparece a tres jantares, podemos escolher os seus companhei-
ros dividindo os outros 6 estudantes em 3 grupos de 2, o que pode ser
C 2 C42 1
feito de 6 = 15 modos.
3!
Entao, os 3 jantares sao, digamos, A1 A2 A3 , A1 A4 A5 , A1 A6 A7 .
A2 devera comparecer a mais dos jantares, nenhum deles em companhia
de A3 , e A3 tambem devera comparecer a mais dois jantares. Portanto,
os 4 jantares que faltam sao:
A2 , A2 , A3 , A3
Como A4 deve compareces a mais dois jantares (A4 nao pode compare-
ces a ambos em companhia de A2 nem a ambos em companhia de A3 ),
esses quatro jantares sao:
A2 A4 , A2 , A3 A4 , A3 ;
A5 tem que comparecer ainda a dois jantares, nenhum deles em com-
panhia de A4 .
A2 A4 , A2 A5 , A3 A4 , A3 A5 .
Agora ha duas possibilidades:
A2 A4 A6 , A2 A5 A7 , A3 A4 A7 , A3 A5 A6 e
A2 A4 A7 , A2 A5 A6 , A3 A4 A6 , A3 A5 A7 .
Ha portanto 15 2 = 30 maneiras de escolher os grupos de convidado.
Para distribuir os 7 grupos nos 7 dias, ha 7! alternativas.
A resposta e 7! 30 = 151.200.

6. Os dois primeiros lugares so podem ser ocupados por elementos de


{a1 , a2 , . . . , a7 } e os dois ultimos por exemplo de {a9 , a10 , a11 , a12 .
A resposta e C72 C42 = 126.
m
7. Ha Cm+h 1 modos de escolher os lugares para os homens. Feito isso,
so ha 1 modo de formar a fila.
m m
A resposta e Cm+h 1 = Cm+h .

13
8. (a) Cada professor fica caracterizado pelas duas bancas a que per-
tence. O numero de professores e igual ao numero de modos de
escolher duas das oito bancas.
A resposta e C82 = 28.
(b) O numero de professores pertencentes a uma banda e igual ao
numero de modos de escolher a outra banca a que ele pertence.
A resposta e 7.
9. (a) Imagine um quadro em que cada linha e relacao dos atletas de um
time. O numero de elementos do quadro e o numero de times, t,
multiplicado pelo tamanho de cada time, k, e e tambem igual ao
numero de atletas, a, multiplicado pelo numero de times de que a
cada atleta participa, x.
Logo, ax = tk e x = tka .
(b) No mesmo quadro, o numero de pares de atletas na mesma linha
e igual ao numero de linhas, t, multiplicado pelo numero de pares
de atletas em uma linha, Ck2 , e e tambem igual ao numero de pares
de atletas, Ca2 , multiplicado pelo numero de times em que cada
par de atletas fica junto, y.
tCk2 tk(k1)
Logo, yCa2 = tCk2 e y = Ca2
= a(a1)
.
10. A resposta e o numero de permutacoes circulares de 4 elementos, ou
seja, 3! = 6.
11. Ha (P C)5 = 4! modos de formar uma roda com as meninas. Depois
disso, os 5 meninos devem ser postos nos 5 lugares entre as meninas, o
que pode ser feito de 5! modos. A resposta e 4! 5! = 24 120 = 2.880.
12. E mas simples calcular o numero total de rodas e excluir aquelas em
que Vera e Isadora ficam juntas. O numero total de rodas e P C6 =
5! = 120. Para formar as rodas em que Vera e Isadora se colocarao na
roda. Ha 2 possibilidades: Vera-Isadora e Isadora-Vera. Agora tudo
se passa como se Vera e Isadora fossem uma unica crianca. Assim,
ha 2(P C)5 = 2.4! = 48 rodas em que Vera e Isadora ficam juntas. A
resposta e 120 48 = 72 rodas.
13. Chamando x de 1 + a, y de 1 + b e z de 1 + c, o problema se transforma
em encontrar todas as solucoes inteiras e nao-negativas de (a + 1) + (b +
1) + (c + 1) = 7, ou seja, de a + c + c = 4. A resposta e CR34 = C64 = 15.

14
14. Cada solucao inteira e nao negativa da inequacao x + y + z 6 corres-
ponde a uma solucao inteira e nao negativa da equacao x+y+z +f 6.
Logo, ha CR46 = C96 = 84 solucoes.

15. Para formar uma caixa, devemos selecionar 20 dentre os 5 tipos, valendo
repeticao na escolha. Ou seja, devemos formar solucoes inteiras e nao
negativas de x1 +x2 +x3 +x4 +x5 = 20, onde xi e o numero de bombons
do tipo i. A resposta e CR520 = C24
20
= 10.626.

15
Exerccios Resolvidos de MA 12

Unidades 17 e 18

Unidade 17

Exerccios Recomendados
1. Ha 66 = 36 resultados possveis igualmente provaveis, em 6 dos quais
6
a soma vale 7. A resposta e 36 = 61 .
12
2. Basta escolher os times do primeiro grupo, o que pode ser feito de C24
modos. Os dois times em questao ficam juntos quando ficam ambos no
primeiro grupo ou ambos no segundo grupo. Em ambos os casos, isto
10
10
2C22 2.22!.12! 11
pode ser feito de C22 modos. Logo, a resposta e 12 = = .
C24 24!10! 23
Outra solucao:
Supondo ja escolhido o grupo do primeiro time, seus 11 companheiros
11
de grupo podem ser escolhidos de C23 modos. Dentre os grupos assim
10
formados os que tambem incluem o segundo time sao C22 , ja que sao
formados escolhendo 10 times entre os 22 restantes. Logo, a resposta e
10
C22 11
11
= .
C23 23
3. Usaremos o fato, ja provado, de que
P (A B) = P (A) + P (B) P (A B).
Temos
P (A B C) =P ((A B) C) = P (A B) + P (C) P ((A B) C)
=P (A) + P (B) P (A B) + P (C) P ((A B) C).

Agora, (A B) C = (A C) (B C) e da
P ((A B) C) =P (A C) + P (B C) P ((A C) (B C))
=P (A C) + P (B C) P (A B C).

1
Substituindo na expressao anterior, obtemos, finalmente:

P (A B C) =P (A) + P (B) + P (C) P (A B) P (A C)


P (B C) + P (A B C).

4. (a) Como A A B, temos P (A B) P (A) = 32 .


(b) Como A B e B sao disjuntos e A B B = A B, temos
P (A B) + P (B) = P (A B) e, portanto, P (A B) = P (A
B) P (B) = P (A B) 49 .
Mas, do tem anterior, 32 P (AB) 1. Da, 23 49 P (AB)
1 94 , ou seja, 29 P (A B) 59 .
(c) Observe que P (A B) + P (A B) = P (A), ja que o conjunto
da direita e a uniao disjunta dos da esquerda. Da, P (A B) =
P (A) P (A B) = 23 P (A B). Como 29 P (A B) 59 ,
temos 23 59 P (A B) 32 29 , ou seja, 19 P (A B) 49 .

5. (a) O numero de casos possveis e 65 , pois ha 6 resultados para cada


um dos 5 dados. O numero de casos favoraveis e 6 C52 5 4 3 =
3600, pois ha 6 modos de escolher o tipo de par (pode ser de 1, de
2, . . . , de 6) e ha C52 modos de escolher os dois dados que formarao
o par; quatro outros dados, o resultado do primeiro deles pode ser
escolhido de 5 modos distintos (deve ser diferente do resultado dos
dois primeiros dados), o do segundo pode ser escolhido de 4 modos
distintos (deve ser diferente dos anteriores) e, o do terceiro, de 4
modos diferentes.
3.600 25
Logo, P (A2 ) = = 0, 463.
65 54
(b) O numero de casos possveis e 65 , pois ha 6 resultados para cada
um dos 5 dados. O numero de casos favoraveis e C62 C52 C32 4 =
1800, pois ha C62 modos de escolher os tipos de pares (podem ser
de 1 e 2, de 1 e 3, . . . , de 5 e 6), ha C52 modos de escolher os
dois dados que formarao o par menor 3 C32 modos de escolher
os dados que formarao o par maior. Para o dado restante, que
deve ter resultado diferente do dos demais dados, ha 4 resultados
possveis.
180 25
Logo, P (A3 ) = 5 = 0, 231.
6 108
2
Observacao:
Um erro comum e contar os casos favoraveis em dobro, racionando
do modo seguinte: Ha 6 modos de escolher o tipo do primeiro par,
5 modos de escolher o tipo do segundo par, C52 modos de escolher
os dois dados que formarao o primeiro par, C32 modos de escolher
os dados que formarao o segundo par e ha 4 modos de escolher o
resultado do dado restante. Logo, o numero de casos favoraveis e
6 5 C52 C32 4 = 3.600.
E claro que o erro provem da distincao artificial entre o primeiro
e o segundo par, que faz com que um par de 2 e um par de 5 seja
considerado diferente de um par de 5 e um par de 2.
(c) O numero de casos possveis e 65 , pois ha 6 resultados para cada
um dos 5 dados. O numero de casos favoraveis e 6C53 54 = 1200,
pois ha 6 modos de escolher o tipo de trinca (pode ser de 1, de
2, . . . , de 6) e ha C53 modos de escolher os tres dados que terao
resultados iguais; quatro aos outros dados, o resultado do primeiro
deles pode ser escolhido de 5 modos distintos (deve ser diferente
do resultado dos tres primeiros dados) e, o do segundo, de 4 modos
distintos (deve ser diferente dos anteriores).
1200 25
Logo, P (A4 ) = 5 = 0, 154.
6 162
(d) O numero de casos possveis e 65 , pois ha 6 resultados para cada
um dos 5 dados. O numero de casos favoraveis e 6 C54 5 = 150,
pois ha 6 modos de escolher o tipo de quadra (pode ser de 1, de
2, . . . , de 6) e ha C54 modos de escolher os quatro dados que terao
resultados iguais; quatro ao dado restante, seu resultado pode ser
de 5 modos distintos (deve ser diferente do resultado dos quatro
primeiros dados).
150 25
Logo, P (A6 ) = 5 = 0, 019.
6 1296
(e) O numero de casos possveis e 65 e o numero de casos favoraveis
e 6.
6 1
Logo, P (A7 ) = 5 = 0, 0008.
6 1296
(f) O numero de casos possveis e 65 , pois ha 6 resultados para cada
um dos 5 dados. Ha dois tipos possveis de sequencias: a mnima

3
(12345) e a maxima (23456). A mnima pode ser formada de
5! = 120 modos distintos, pois ha 5 modos de escolher o dado cujo
resultado e 1, 4 modos de escolher o dado cujo resultado e 2, etc.
Ha analogamente, 120 modos de formar a sequencia maxima.
240 5
Portanto, P (A8 ) = 5 = 0, 031.
6 162
(g) O numero de casos possveis e 65 , pois ha 6 resultados para cada
um dos 5 dados. O numero de casos favoraveis e 6 C53 5 = 300,
pois ha 6 modos de escolher o tipo de trinca (pode ser de 1, de 2,
. . . , de 6) e ha C53 modos de escolher os tres dados que formarao a
trinca; quatro aos outros dados, ha 5 modos distintos de escolher
o resultado comum deles.
300 25
Logo, P (A5 ) = 5 = 0, 039.
6 684
6. Numeramos os vertices do polgono de 0 a 2n. Imagine 0 como o vertice
mais alto, os vertices de 1 a n do lado direito e os vertices de n + 1 a
2n do lado esquerdo. Podemos pensar que todos os triangulos tem 0
2
2n(2n 1)
como um dos vertices. Ha C2n = = n(2n 1) modos de
2
selecionar os outros dois vertices do triangulo.
Para construir o numero de triangulos que contem o centro da circun-
ferencia em seu interior, observe inicialmente que a reta que contem o
vertice i(1 i n) e o centro da circunferencia corta novamente o
polgono no ponto medio do segmento determinado pelos vertices i + n
e i + n + 1 (vertice 2n + 1= vertice 0).
Um triangulo que contenha em seu interior o centro da circunferencia
sera necessariamente formado por um vertice do lado direito e um
vertice do lado esquerdo. Se o vertice do lado direito for o vertice
1, o do lado esquerdo so podera ser o vertice n + 1 (1 possibilidade);
se for o vertice 2, podera ser qualquer dos vertices de n + 1 a n + 2 (2
possibilidades); . . . ; se for o vertice n, podera ser qualquer dos vertices
de n + 1 a 2n (n possibilidades). O numero de casos favoraveis e
n(n + 1)
1 + 2 + + n = .
2
n+1
A resposta e .
2(2n 1)

4
7. Imagina o resultado do sorteio como uma fila de 12 lugares: o primeiro
lugar corresponde a primeira pessoa sorteada area o primeiro grupo;
o segundo a segunda pessoa sorteada para o segundo grupo; . . . ; o
ultimo, a quarta pessoa sorteada para o terceiro grupo. Colocada a
primeira pessoa, ha 11 posicoes para a segunda, em 3 das quais ela fica
no mesmo grupo da primeira.
3
A resposta e .
11
8. Ha 12 possibilidades para o signo de cada pessoa, para um total de 124
possibilidades. Para que nao haja coincidencia de signos, o signo da
primeira pessoa pode ser escolhido de 12 modos, o da segunda de 11,
o da terceira de 10 e o da quarta de 9, para um total de 12 11 10 9
modos. Assim, a probabilidade de que nao haja coincidencia de signos
12 11 10 9 55
e = e a probabilidade de que nao haja coincidencias
124 96
55 41
e 1 = .
96 96
4
9. Ha C10 modos de retirar 4 pes de sapatos. Para retirar 4 pes, havendo
nesses 4 pes exatamente 1 par de sapatos, devemos inicialmente seleci-
onar 1 par (o que pode ser feito de 5 modos) e depois selecionar 2 pes
vindo de pares diferentes dentre os 4 pares que ainda estao no armario.
Para isso devemos escolher os pares de onde virao esses sapatos (C24
modos) e, em cada par escolhido, decidir se retiramos o pe direito ou o
pe esquerdo (22 = 4 modos).
5 C42 4 4
A resposta e = .
C1 04 7

Unidade 18

Exerccios Recomendados
1. Sejam X e Y os resultados do primeiro e segundo lancamentos, respec-
tivamente.
P (X = 3, X + Y = 7) 1/6 1/6 1
P (X = 3|X + Y = 7) = = = .
P (X + Y = 7) 6/36 6

5
Outra solucao:
Se a soma e 2, ha 6 casos possveis igualmente provaveis: X = 1, Y = 6;
X = 2, Y = 5; X = 3, Y = 4; X = 4, Y = 3; X = 5, Y = 2;X = 6,
Y = 1. Dos seis casos, somente X = 3, Y = 3 e favoravel. A resposta
1
e .
6
P (nao sabe e acerta)
2. P (nao sabe | acerta) =
P ( acerta)
P (nao sabe) P (acerta | nao sabe)
=
P (sabe) P (acerta | sabe) + P (nao sabe) P (acerta | nao sabe)
0, 4 0, 2 2
= = .
0, 6 1 + 0, 4 0, 2 17
3. P (A B) = P (A) P (B|A) = 1/2 1/2 = P (A) P (B); logo, A e B sao
independentes.
Observe que A C = A B e que P (C) = 21 P (A C) = P (A B) =
1/2 1/2 = P (A) P (C); logo, A e C sao independentes.
Observe que B C = A B e que P (C) = 21 P (B C) = P (A B) =
1/2 1/2 = P (B) P (C); logo, B e C sao independentes.
Como A B C = A B, P (A B C) = P (A B) = 1/2 1/2, que
e diferente de P (A) P (B) P (C) = 1/2 1/2 1/2; logo, A, B e C nao
sao independentes.
4
4. (a) A probabilidade de nenhum seis em quatro lancamentos e 56
4
0, 4823. A probabilidade de pelo menos um seis e 1 65
1 0, 4823 = 0, 5177.
(b) A probabilidade de nenhum duplo seis em 24 lancamentos de um
35 24

par de dados e 36 0, 5086. A probabilidade de pelo menos
35 24

um duplo seis e 1 36 1 0, 5086 = 0, 4914.

6
5.
P (doente e positivo)
P (doente | positivo) =
P (positivo)
P (doente positivo | doente)
=
P (doente) P (positivo | doente) + P (sadio) P (positivo | sadio)
0, 005 0, 95 95
= = = 0, 3231
0, 005 0, 95 + 0, 995 0, 01 294
n
6. A probabilidade de nao obter nenhum seis em n lancamentose 56 e
5 n 5 n
de obter pelo menos um de seis e 1 6
. Devemos ter 1 6
> 0, 9,
5 n

ou seja, 6 < 0, 1.
Da,
 n
5
ln < ln 0, 1,
6
5
n ln < ln 0, 1,
6
ln 0, 1
n> = 12, 6.
ln 56

A resposta e 13.
7. (a) Cada pessoa tem n modos de escolher a quem conta o boato. Logo,
o numero de modos de o boato ser contado m vezes e nm .
O numero de modos de o boato ser contado m vezes, sem retornar
a primeira pessoa e n(n 1)m1 , pois o primeiro ouvinte pode ser
selecionado de n modos e os demais, de n 1 modos. A resposta
m1
n1 m1
e n(n1)

nm = n
.
(b) Cada pessoa tem n modos de escolher a quem conta o boato. Logo,
o numero de modos de boato ser contado m vezes e nm .
Para o boato ser contado m vezes, sem repetir nenhuma pessoa,
o primeiro ouvinte pode ser selecionado de n modos; o segundo,
de n 1 modos; o terceiro, de n 2 modos; . . . ; o m-esimo,
de n (m 1) = n m + 1 modos. O numero de modos de
o boato ser contado m vezes, sem retornar a primeira pessoa, e
n!
n (n 1) (n m + 1) = .
(n m)!

7
n!
A resposta e .
(n m)!nm
8. Considere os eventos:
A ={A falou a verdade};
B ={B disse que A falou a verdade};
C ={C disse que B disse que A falou a verdade};
D ={D disse que C disse que B disse que A falou a verdade}.
Vamos aliviar a notacao escrevendo XY para representar X Y .
P (AD)
Queremos calcular P (A|D) = .
P (D)

P (AD) = P (ABCD) + P (ABCD) + P (ABCD) + P (ABCD)


1 1 1 1 1 2 2 1 1 1 2 2 1 2 1 2 13
= + + + =
3 3 3 3 3 3 3 3 3 3 3 3 3 3 3 3 81

P (AD) = P (ABCD) + P (ABCD) + P (ABCD) + P (ABCD)


2 2 1 1 2 1 2 1 2 2 2 2 2 1 1 2 28
= + + + = .
3 3 3 3 3 3 3 3 3 3 3 3 3 3 3 3 81

13 28 41
P (D) = P (AD) + P (AD) = + = .
81 81 81
P (AD) 13/81 13
A resposta e P (A|D) = = = .
P (D) 41/81 41
9. Uma urna recebe uma bola branca e a outra urna recebe as demais 99
bolas. Com efeito, se a 1a urna recebe k bolas das quais a sao brancas,
a probabilidade de libertacao e
 
1 a 50 a 1 50k + a(100 2k)
f (a, k) = + = .
2 k 100 k 2 k(100 k)

Observe que para k = 50 a expressao vale 12 , independentemente do


valor de a.

8
Observe tambem que basta estudar agora o caso k < 50 (isto e, po-
demos considerar a primeira urna como sendo a que recebeu menos
bolas). Nesse caso, e claro que, fixando o valor de k, quando maior for
a, maior sera f (a, k). Logo, para f (a, k) ser maximo, devemos ter a = k
1 150 2k 75 k 25
e a probabilidade sera g(k) = = = 1 , que
2 100 k 100 k 100 k
e maxima para k mnimo.
Devemos, pois, ter k = 1, o que da uma probabilidade de libertacao de
74
99 =
0, 75.

9
Exerccios Resolvidos de MA 12

Unidades 19 e 20

Unidade 19

Exerccios Recomendados
1. Para distribuir os sorvetes, devemos escolher as pessoas que receberao
5
sorvetes de creme (C10 modos) e dar sorvetes de chocolate as demais
(1 modo). Para distinguir os sorvetes, respeitando as preferencias,
comecamos dando sorvete de creme aos que gostam de creme e de
chocolate aos que gostam de chocolate (1 modo). Em seguida, deve-
mos distribuir 2 sorvetes de creme e 3 sorvetes de chocolate a 5 pessoas
que nao tem preferencias; para isso, devemos escolher as 2 pessoas que
receberao sorvetes de creme (C52 modos) e dar sorvetes de chocolate as
restantes (1 modo).
C52 5
A resposta e 5
= .
C10 126
2. As pecas do domino sao formadas por dois, nao necessariamente dis-
tintos, dos numeros 0, 1, 2, 3, 4, 5 e 6. Ha CR72 = C82 = 28 pecas e
2
ha C28 modos de selecionar duas pecas de um domino. Para selecionar
duas pecas com um numero comum , deve-se primeiramente selecionar
o numero comum (7 possibilidades) e, depois, selecionar 2 das 7 pecas
que contem esse numero comum (C72 possibilidades).
7C72 7
A resposta e 2 = .
C28 18
5
3. O numero de sorteios possveis e C80 .
(a) O apostador acerta 3 dezenas quando sao sorteadas das 8 dezenas
em que apostou e 2 das 72 em que nao apostou. Tais sorteios
podem ser efetuados de C83 C722
modos.
3 2
C8 C72 1
A resposta e 5
(que e aproximadamente igual a ).
C80 168

1
(b) O apostados acerta 4 dezenas quando sao sorteadas 4 das 8 dezenas
em que apostou e 1 das 72 em que nao apostou. Tais sorteios
podem ser efetuados de C84 C721
modos.
4 1
C8 C72 1
A resposta e 5
(que e aproximadamente igual a ).
C80 4770
(c) O apostados acerta 5 dezenas quando sao sorteadas 5 das 8 dezenas
em que apostou. Tais sorteios podem ser efetuados de C85 modos.
C85 1
A resposta e 5 = .
C80 429.286
4. Colocada a primeira pessoa na roda, ha n 1 posicoes para a segunda
pessoa, das quais 2 sao favoraveis a que ela fique junto da primeira
pessoa.
2
A resposta e .
n1
5. (a) Ha n posicoes igualmente provaveis que a chave certa poderia
ocupar: ser a primeira a ser testada, a segunda, . . . , a ultima. A
probabilidade de ela ocupar a k-esima posicao e 1/n.
Outra solucao:
Ha n! maneiras de ordenar as chaves a serem tentadas. Para
formar as ordenacoes que tem a chave na k-esima posicao, devemos
colocar as n 1 chaves restantes nas n 1 posicoes restantes, o
que pode ser feito de (n 1)! modos. Logo, a probabilidade de
(n 1)! 1
que a chave certa esteja na posicao k e = .
n! n
k
(b) As primeiras k tentativas podem ser feitas de n modos (cada
chave pode ser escolhida de n modos, ja que chaves correspon-
dentes a tentativas frustadas nao sao descartadas). Para que se
acerte na k-esima tentativa, as primeiras k 1 chaves devem ser
incorretas (portanto, podem ser escolhidas de (n 1)k1 modos) e
a de ordem k deve ser a correta (1 modo). Logo, a probabilidade
(n 1)k1
de se acertar na k-esima tentativa e .
nk
4
6. (a) Ha C12 = 495 modos de selecionar as 4 vagas que nao serao ocu-
padas e 9 modos de escolher 4 vagas consecutivas (1 2 3 4, 2 3 4
5, . . . , 9 10 11 12).

2
9 1
A resposta e = .
495 55
4
(b) Ha C12 = 495 modos de selecionar as 4 vagas que nao serao ocupa-
das. Para contar o numero de possibilidades em que nao ha vagas
vazias adjacentes, devemos escolher 4 dos 9 espacos existentes an-
tes, entre e depois dos carros para ficarem vazios, Isto pode ser
feito de C94 modos. Logo a probabilidade de que nao haja vagas
126 14
consecutivas e = .
495 55
7. P (A B C) = 0, pois A B C A C e P (A C) = 0.

(a)

P (A B C) =P (A) + P (B) + P (C) P (A B)


P (A C) P (B C) + P (A B C)
=0, 4 + 0, 5 + 0, 3 0, 3 0 0, 1 + 0 = 0, 8

(b)

P [A (B C)] =P (A) P [A (B C)]


=P (A) P [(A B) (A C)]
=P (A) P (A B) P (A C)
+ P [(A B) (A C)]
=P (A) P (A B) P (A C)
+ P [(A B C)]
=0, 4 0, 3 0 + 0 = 0, 1

(c)

P [(A B) C] =P (A B) + P (C) P (A B C)
=0, 3 + 0, 3 0 = 0, 6

8. (a) A resposta, naturalmente, e 1/2, ja que, de todos os pares de


numeros distintos de 1 a 100, em exatamente a metade o primeiro
numero e maior do que o segundo.

3
(b) O numero total de possveis extracoes e 100100 = 10.000, ja que
o bilhete de cada uma das mocas pode ser escolhido de 100 modos.
Em 100 destas possveis extracoes os dois numeros sao iguais e em
metade das restantes, ou seja, em 9900/2 = 4950 delas, o primeiro
numero e maior do que o segundo. Logo, a probabilidade de o
4950
numero de Laura ser maior do que o de Telma e = 0, 495.
10000
9. (a) Sao feitos 5 testes quando uma das quatro primeiras pilhas tes-
tadas esta descarregada, o mesmo ocorrendo com a quinta a ser
testada. A primeira pilha a ser testada pode ser escolhida de 10
modos, a segunda de 9, e assim por diante, para um total de
10 9 8 7 6 modos possveis para escolher as 5 primeiras pilhas a
serem testadas. Para formar uma sequencia de teste em que a se-
gunda defeituosa e detectada na 5a tentativa, devemos escolher a
pilha defeituosa que aparece na 5a posicao (2 modos), a posicao da
outra defeituosa (4 modos) e, finalmente, as pilhas nao defeituosas
para as demais posicoes (8 7 6 modos). Logo, a probabilidade
24876 4
pedida e = .
10 9 8 7 6 45
(b) Sao efetuados ate 5 testes quando as pilhas defeituosas aparecem
nas 5 primeiras tentativas. Como visto no tem anterior, ha 10 9
8 7 6 modos de se fazer esta tentativa. Para formar aquelas em
que as duas defeituosas estao entre as testadas devemos escolher
a posicao da primeira pilha defeituosa (5 modos), a da segunda
(4 modos) e, finalmente, as pilhas nao defeituosa para as outras
tentativas (876 modos). A probabilidade de que sejam feitos ate
54876 2
5 testes e = e, portanto, a probabilidade pedida
10 9 8 7 6 9
2 7
e igual a 1 = .
9 9
(c) Para que sejam feitos menos de 4 testes, as duas pilhas defeituosas
devem aparecer nos primeiros 4 testes. O numero total de escolhas
para os 4 primeiros testes e 10 9 8 7. Para formar uma sequencia
de teste em que as duas defeituosas aparecem nestas 4 tentativas,
devemos escolher a posicao da primeira pilha defeituosa (4 modos)
a da segunda (3 modos) e, finalmente, as pilhas nao defeituosas
para as duas outras posicoes (87 modos). A probabilidade pedida

4
4387 2
e =
10 9 8 7 15

Unidade 20

Exerccios Recomendados
1
1. (a) A probabilidade de eles se enfrentarem na primeira ronda e
2n 1
porque, posto A na tabela, ha 2n 1 posicoes possveis para B e
em 1 delas ele enfrenta B A probabilidade
!2 deles se enfrentarem
2 1 1 1
na segunda rodada e n = n , porque posto
2 1 2 2 1 2
n
A na tabela, ha 2 1 posicoes possveis para B e em 2 delas
ele pode vir a enfrentar B na segunda rodada, o que ocorre com
probabilidade 21 12 . A probabilidade de eles se enfrentarem na
!4
22 1 1 1
terceira rodada e n = n 2 , etc.
2 1 4 2 1 2
A resposta e
1 1 1 1 1 1 1
+ n + n 2 + + n n1
2n
1 2 1 2 2 1 2 2 1 2
1 n
1 1 2 1
= n 1 = n1 .
2 1 1 2 2

(b) Se k < n, o jogador disputa exatamente k partidas se e somente


se perde a k-esima partida e ganha as k 1 partidas anteriores.
k1 1
A probabilidade de isso acontecer e 12 2 = 21k . O jogador
disputa n partidas - ou seja, chega a final - se e somente se ganha
as n 1 partidas anteriores. A probabilidade de isso acontecer e
1 n1 1
2
= 2n1 .
A resposta e 21k , se k < n; 2n1
1
, se k = n.

2. (a) O segundo jogador de melhor resultado sera vice-campeao se e


somente se nao enfrentar o melhor jogador antes da final. Posto o
melhor jogador na tabela, ha 15 posicoes possveis para o segundo

5
melhor e em 8 delas ele enfrenta o melhor jogador apenas na final.
8
A resposta e .
15
(b) Posto o 4o colocado na tabela, os demais times podem ser coloca-
dos de 15! modos. Para que o 4o melhor time seja vice-campeao,
os 3 melhores times nao podem entrar em sua chave. As posicoes
destes times podem portanto, ser escolhidas de 8, 7 e 6 modos,
respectivamente. Para distribuir os 12 times restantes, ha 12! pos-
8 7 6 12! 8
sibilidades. Logo, a probabilidade desejada e = .
15! 65
o
(c) Na primeira rodada, ha 6 adversarios que o 10 time consegue
derrotar. Na melhor das hipoteses, ele derrota um destes e, dos
outros 5, dois conseguem sobreviver para a proxima fase. De novo,
na melhor das hipoteses o 10o enfrenta (e vence) um deles, mas o
outro sera fatalmente eliminado. Assim, na 3a rodada, o 10o time
joga e perde. Logo, ele disputa no maximo tres partidas. Isto
ocorre quando os tres times da sua chave para os dois primeiros
jogos sao todos de habilidade inferior. O numero de modos de
escolher 3 adversarios e 151413. O numero de modos de escolher
tres adversarios entre os 6 de nvel inferior e 654. Logo, a resposta
654 4
e = .
15 14 13 91
3. Se o candidato nao troca de porta, ele ganha o premio se e so se escolhe,
originalmente, a porta certa. Logo, se ele nao troca, sua probabilidade
de ganhar o primeiro premio e igual a 13 . Em contraste, ao trocar de
porta ele ganha o premio sempre que escolheu originalmente a porta
errada, o que ocorre com probabilidade 32 . Portanto, ele deve trocar de
porta.

4. Cada um dos 10 resultado pode ser escolhido de 2 modos. Portanto,


ha 210 resultados possveis. Para formar um resultado com 5 caras, e
necessario escolher 5 dos 10 lancamentos para estas caras ocorrerem, o
5
5
C10 63
que pode ser feito de C10 modos. A probabilidade pedida e 10 = .
2 256

6
5. (a)

P (2a B) = P (1a B, 2a B) + P (1a P, 2a B)


= P (1a B) P (2a B|1a B) + P (1a P ) P (2a B|1a P )
4 6 6 4 2
= + =
10 12 10 12 5

(b)

P (1a B, 2a P )
P (1a B|2a P ) =
P (2a P )
P (1a B) P (2a P |1a B)
=
P (1a B) P (2a P |1a B) + P (1a P ) P (2a P |1a P )
4
6
10 12 1
= 4 6 6 8 = .
+ 10 12
10 12
3

P (ve vermelha e mostra amarela)


6. P (ve vermelha | mostra amarela) = P (mostra amarela)

1/6 1
= = .
1/2 3
7. O jogador A ganha o primeiro se e somente se B ganhar no maximo 2
das proximos 8 partidas (caso contrario, B tera sua 10a vitoria antes
de A completar sua serie de 10 vitorias).
P (B ganhar 0 partidas) = 0, 68
= 0, 0168
P (B ganhar 1 partidas) = 8 0, 67 0, 4
= 0, 0896
P (B ganhar 2 partidas) = C 2 0, 66 0, 42
8 = 0, 2090
A probabilidade de que A ganhe o premio e aproximadamente igual a
0, 0168 + 0, 0896 + 0, 2090 = 0, 3154.

8. Sejam p1 , p2 , p3 , p4 , p5 , p6 as probabilidades das possveis ordenacoes


ABC, BCA, CAB, ACB, CBA, BAC. As condicoes dadas no pro-
blema permitem escrever um sistema de equacoes lineares envolvendo
aquelas probabilidade:

7
1
P 1 + p 4 = P2 + p 6 = P3 + p 5 =
3
1
P 3 + p 6 = P1 + p 5 = P2 + p 4 =
3
1
P 2 + p 5 = P3 + p 4 = P1 + p 6 =
3

Resolvendo o sistema, verifica-se que ele tem uma infinidade de solucoes


da forma p1 = p2 = p3 = 16 p4 = 16 p5 = 15 p6 .
Em termos mais intuitivos, basta que as ordenacoes correspondente a
mesma ordem circular tenham probabilidades iguais, ou seja, devemos
ter:
P (ABC) = P (BCA) = P (CAB)
e
P (ACB) = P (CBA) = P (BAC).

Por exemplo, se P (ABC) = P (BCA) = P (CAB) = 41 e P (ACB) =


1
P (CBA) = P (BAC) = 12 , os tres jogadores tem a mesma chance
de ficar em primeiro, segundo ou terceiro lugar, embora as diferen-
cias ordenacoes possveis nao tenham todas a mesma probabilidade de
ocorrer.

9. Dois pontos A e B de um circulo determinam uma corda maior que o


lado de um triangulo equilatero inscrito se e somente se o menor arco
AB e maior do que 120 . Isto significa, que uma vez escolhido o ponto
A, o ponto B nao deve estar no arco de 240 com ponto medio em
A, como mostra a figura. Como B e escolhido ao acaso, admitimos
que a probabilidade de que ele esteja em um arco seja proporcional ao
120 1
comprimento do arco. Assim, a probabilidade pedida e = .
360 3

8
A

120 120

120
B

10. Uma corda perpendicular a AB tem comprimento maior do que o lado


do triangulo equilatero inscrito se e somente se corta AB entre os pontos
M e N , medios de OA e OB e centro do crculo, como mostra a figura.
MN 1
Logo, a probabilidade pedida e = .
AB 2

A B
M O N

11. Contando o tempo em minutos, a partir das 16 horas, e designando por


x e y os instantes de chagada de Cristina e Maria, a regiao possvel
e = {(x, y) : 0 x 60, 0 y 60} (um quadrado de lado 60) e
a regiao favoravel e A = {(x, y) : 0 x 60, 0 y 60, |x y|
10}. A desigualdade |x y| 10 e equivalente a x 10 y x + 10, o
que mostra que a regiao favoravel e uma faixa em torno da diagonal
do quadrado, como mostra a figura.

1
9
60

10

0 10 60

area de A 602 2 50 50/2 11


A probabilidade desejada e: = = .
area de 602 36

1
10
Exerccios Resolvidos de MA 12

Unidades 21 e 22

Unidade 21

Exerccios Recomendados
d d
1. Os tempos de percurso sao t1 = e t2 = . Logo, a velocidade
2v1 2v2
d d 2
media e vm = = d = 1 , que e a media harmonica
t1 + t2 2v1
d
+ 2v2 v1
+ v12
das velocidades v1 e v2 na ida e na volta.

2. A distancia total percorrida e d = tv1 + tv2 . Logo, a velocidade media


tv1 + tv2 v1 + v2
e vm = = , que e a media aritmetica das velocidades
2t 2
v1 e v2 nas duas partes do percurso.

3. Apos as duas decadas, a populacao e multiplicada por 1, 44 1, 21. A


taxa media i de crescimento decenal e a taxa que, mantendo-se cons-
tante para as duas decadas, produza o mesmo crescimento total. Logo,
devemos ter (1 + i)2 = 1, 44 1, 21, Logo, 1 + i = 1, 44 1, 21 =
1, 2 1, 1 = 1, 31. Portanto, a taxa media de crescimento decenal e
0, 32 = 32%.

4. Do mesmo modo, a taxa media anual de crescimento i e tal que, apli-


cada a cada ano, produza o mesmo crescimento total. Logo, devemos
20
ter (1 + i) = 1, 44 1, 21 = 1, 7424. Da, 1 + i = 1, 7424 = 1, 0282
20

e a taxa media anual de crescimento e 1, 0282 = 2, 82%.

6. Em 1/3 dos casos, os prisioneiros levem 3 horas para escapar; em outro


1/3, levam 5 horas; no 1/3 restante gastam 9 horas para retornar ao
ponto de partida. Esses ultimos, agora, escolhem um dos dois outros
tuneis. Assim, 1/6 dos prisioneiros levam 9+3 = 12 horas para escapar
e o restante 1/6 levam 9 + 5 = 14 horas. O tempo medio para escapar
e, portanto, 31 3 + 13 5 + 16 12 + 61 14 = 7 horas.

1
7. Seja M o tempo medio para escapar. Em 1/3 dos casos, os prisionei-
ros levam 3 horas para escapar; em outro 1/3, levem 5 horas; no 1/3
restante gastam 9 horas para retornar ao ponto de partida, do qual
precisam, em media, de um tempo adicional M para escapar. Logo,
temos: M = 31 3+ 31 5+ 13 (9+M ). Resolvendo a equacao, encontramos
M = 8, 5 horas.
Cada um dos n numeros ai satisfaz m ai M e, portanto, nm
8. P
n
i=1 ai nM . Dividindo por n, vem m x M .

Cada um dos n numeros ai satisfaz m a1 M e, portanto, mn


9. Q
n n
i=1 ai M (ja que todos os numeros envolvidos sao positivos). Ex-
traindo a raiz n-esima, vem m g M .
10. Cada um dos n numeros ai satisfaz m ai M . P Como cada um
n
deles e positivo, tem-se m1 a1i M1 e, da, m
n
1
i=1 ai nM .
Portanto, mn
Pn 1 1 nM . Finalmente, multiplicando por n, vem
i=1 ai
m h M.
11. Nao. Depende dos pesos atribudos as duas provas. Se, por exemplo,
Portugues tiver peso 4 e Matematica peso 1, Joao tera 27 pontos e
Pedro 28.
12. (a) Se um pneu roda x mil quilometros em uma roda dianteira e y
mil quilometros em uma roda traseira, a fracao do pneu que gasta
x y
e 40 + 60 . Para conseguir a rodagem maxima sem trocar pneus,
todos os pneus devem gastar totalmente ao mesmo tempo (ou
seja, esta fracao deve ser igual a 1 ao final do processo para cada
pneu). Portanto, cada um deles devera rodar o mesmo numero
de quilometros em uma roda dianteira e uma roda traseira, ou
x y
seja x = y. Logo, deve-se ter 40 + 60 = 1, de onde resulta x =
24. Logo, cada pneu deve rodar 24 mil quilometros em uma roda
dianteira e 24 mil quilometros em uma roda traseira. Portanto,
o carro pode rodar 48 mil quilometros com um jogo de 4 pneus,
bastando para isto trocar os pneus traseiros pelos dianteiros aos
24 mil quilometros.
(b) Do mesmo modo, cada pneu deve rodar 24 mil quilometros em
uma roda traseira e 24 mil quilometros em uma roda dianteira.
Mas como agora temos 5 pneus, o carro pode rodar por 5/4 de

2
48 mil quilometros, ou seja, por 60 mil km. Para tal, basta fazer
um rodzio dos pneus a cada 12 mil km. Por exemplo, comecando
com ABCDE (com A e B na dianteira, C e D na traseira e E no
estepe), passar para BCDEA, CDEAB, DEABC e EABCD).
2
(c) O resultado em a) e igual a 1 1
+ 60.000
. Logo, e a media harmonica
40.000
de 40.000 e 60.000.
504012575
13. A nova media e 48
= 37, 5.

14. A media quadratica conserva a soma dos quadrados dos numeros.

15. Cada um dos n numeros ai satisfaz m ai M , ou


Pn
seja, m2 a2i
a2i
M 2 . Logo, nm2 ni=1 a2i nM 2 , ou seja, m2 i=1 M 2 . Da,
P
n
finalmente, m q M .

Exerccios Suplementares

( x1 x2 )2
1. A G = x1 +x2
2
x 1 x 2 = 2
. Portanto, A G 0, com
igualdade se e somente se x1 = xq 2 . Aplicando este resultado aos inver-
1 1
+
sos de x1 e x2 , temos x1 2 x2 x11 x12 . Ou seja, temos H1 G1 , que e
1 1
equivalente a H G; a igualdade ocorre se e somente se x1
= x2
, ou
seja, quando x1 = x2 .

2. A media aritmetica dos erros e necessariamente igual a 0, ja que


n
X Xn
(ai m) = ( ai ) nm = nm nm = 0.
i=1 i=1

3. Deseja-se encontrar x de modo que seja mnimo


n
X Xn n
X
2 2
f (x) = (x xi ) = nx 2( xi )x + x2i ,
i=1 i=1 i=1

que e uma funcao quadratica de x. Esta funcao atinge seu valor mnimo
2 n
P
b i=1 xi
para x = 2a = 2n
= x

3
4. Deseja-se encontrar x de modo que seja mnimo
n
X
f (x) = |x xi | = |x x1 | + |x x2 | + + |x xn |.
i=1

Ha dois casos a considerar. Quando n e mpar, f (x) e decrescente


para n menor que o termo central x(n+1)/2 e decrescente para x maior
que este termo. Logo, atinge o seu valor mnimo neste ponto, que e a
mediana dos numeros x1 , x2 , . . . , xn . Quando n e par, f (x) e crescente
para x < xn/2 , constante para xn/2 < x < xn/2+1 e crescente para
x > xn/2+1 . Logo, o valor mnimo de f ocorre para todo valor de x
entre as observacoes centrais.

5. (a) Os pontos seriam colineares se e somente se os incrementos em y


fossem os mesmos para cada incremento de 1 unidade em x, o que
nao ocorre.
(b) Para a reta 1, o erro medio quadratico e:
2 2 2 2 +(13501550)2
e1 = (580820) +(9801000) +(11901200)
5
+(13601350)
= 380
Para a reta 2, o erro medio quadratico e:
2 2 2 2 2
e1 = (800820) +(9801000) +(11601200)
5
+(13401350) +(15201550)
= 680
Logo, a primeira reta produz o menor erro quadratico.
(c) Consideremos uma reta de equacao y = ax + b. A soma dos erros
quadraticos e

S =(b 820)2 + (a + b 1000)2 + (2a + b 1200)2


+ (3a + b 1350)2 + (4a + b 1550)2
=30a2 + 20ab + 5b2 27300a 11840b + 7337400.

Podemos escrever esta soma como:


 2
b 5
S = 30 a + 455 + (b 822)2 + 510
3 3
Logo, S e mnimo quando a + 3b 455 = 0 e b 822 = 0, ou seja,
quando b = 822 e a = 181. Portanto, a reta pedida tem equacao
y = 181x + 822.

4
6. Existem 7 restos possveis quando se divide um numero por 7. Assim,
tomando os restos como gavetas e os numeros como objetos, pode-se
garantir que alguma gavetas contera dois (ou mais) objetos. Isto e, ha
pelo menos dois numeros que deixam o mesmo resto quando dividimos
por 7, o que e equivalente a dizer que a diferencia entre eles e um
multiplo de 7.
7. A mesma pode ser colocada em 20 posicoes diferentes. Seja ai (i =
1, . . . , 20) o numero de criancas cuja preferencia e atendida com a mesa
na posicao i. Entao a1 + a2 + + a20 e o numero total de preferencias
atendidas. Mas cada sorvete e colocado, sucessivamente, em frente
a cada crianca. Como ha exatamente 10 criancas que preferem cada
sabor, o numero total de preferencias atendidas por cada sorvete e 20,
para um total de 200 preferencias atendidas. Assim, temos a1 + a2 +
+ an . Como a media de a1 , . . . , a20 e 10, conclui-se que pelo menos
um dos numeros e maior ou igual a 10 (ou seja, ha alguma posicao em
que pelo menos 10 criancas sao atendidas).
8. Em uma reuniao com n pessoas, cada pessoa pode ter de 0 a n
1 conhecidos. No entanto, e impossvel que, ao mesmo tempo, haja
uma pessoa que nao tenha conhecidos e outra que conheca todos (a
final, estas duas pessoas se conhecem ou nao?). Portanto, em qualquer
situacao ha apenas n1 valores possveis para o numero de conhecidos,
o que implica que pelo menos duas das n pessoas tem o mesmo numero
de conhecidos.
9. Como no exemplo 7, considere os restos da divisao por 1997, dos
numeros 1, 11, 111, . . . . Como ha apenas 1997 restos possveis, ne-
cessariamente ha dois restos coincidentes. Tomando a diferenca dos
dois numeros da forma 11. . . 1 resulta a existencia de um numero da
forma 11 . . . 10 . . . 0 = 11 . . . 1 10k que e multiplo de 1997. Como 1997
e 10 sao primos entre si, o numero da forma 11 . . . 1 acima e necessari-
amente multiplo de 1997.
10. Deve haver 73 pessoas. Podemos distribuir ate 72 pessoas de modo que
haja exatamente 6 nascidas em cada mes. Com 73 pessoas, necessari-
amente um dos meses (gavetas) contera 7 ou mais pessoas (objetos).
11. As paridades das coordenadas dos pontos de coordenadas inteiras do
plano determinam 4 gavetas: par-par, par-mpar, mpar-par e mpar-

5
mpar. Dados 5 pontos, pelo menos uma das gavetas contem dois pon-
tos. Ou seja, ha um par de pontos em que ambas as coordenadas tem a
mesma paridade, o que faz com que o ponto medio tenha coordenadas
inteiras.

12. Seja ai , i = 1, . . . , N , o numero de objetos em cada gaveta, Entao


a1 + + aN = N k + 1, ou seja, a media aritmetica de a1 , . . . , aN e
k + N1 . Como a media dos numeros inteiros a1 , . . . , aN e maior do que
k, resulta que pelo menos um deles e maior do que k.

13. As 4 primeiras questoes da prova podem ser respondidas de 5 5


5 5 = 625 modos (gavetas) diferentes. Como sao 40100 candidatos, o
numero medio de candidatos para cada possvel padrao de resposta e
40100
625
= 64, 16. Pode-se garantir, em consequencia, que uma das gavetas
contem 65 objetos; ou seja, pelo menos 65 candidatos respondem de
modo identico as primeiras 4 questoes. Este e o maior valor possvel
para k, ja que e possvel distribuir os candidatos de modo que haja no
maximo 65 por cada padrao de resposta.

14. As primeiras k questoes podem ser respondidas de 5k modos (gavetas).


Para garantir que pelo menos 4 candidatos respondam a estas questoes
do mesmo modo, deve-se ter pelo menos 35k +1 candidatos (objetos).
Portanto, deve-se ter 35k < 40100, o que ocorre para k 5. Portanto,
o valor maximo possvel para k e 5.

15. Considere os pontos da reta com coordenadas inteiras. Como ha so-


mente 11 cores disponveis, dois deles tem a mesma cor.

16. O numero de jogos de cada um dos n times e um numero inteiro de 0 a


n 1. Mas, como no exerccio 23, nao pode existir, simultaneamente,
um time com 0 jogos e outro com n 1 (eles ja jogaram entre si ou
nao?). Logo, ha apenas n 1 gavetas para n objetos, o que garante
que dois times enfrentaram o mesmo numero de adversarios.

17. Dividamos o retangulo em 6 retangulos 1 2. Como ha 7 pontos, ha


dois que estao no mesmo retangulo. A distancia
entre eles e no maximo
igual a diagonal do retangulo, que mede 5.

18. Como os numeros sao escolhidos no conjunto 1, 2, . . . , 15, os valores


possveis para a diferenca de dois numeros sao 1, 2, . . . , 14 (ou seja,

6
ha 14 valores possveis). Por outro lado, os oito numeros formam C82 =
28 pares. Destes, no maximo 1 resulta em uma diferenca igual a 14
(quando formado por 1 e 15). Em consequencia, ha pelo menos 27 pares
cujas diferencas pertencem ao conjunto 1, 2, . . . , 13. Como 27/13 >
2, ha uma gaveta (diferenca) contendo mais de dois objetos (pares).
Portanto, ha pelo menos tres pares de numeros em que a diferenca
entre o maior e o menor numero do par e a mesma.
x
19. (a) Seja 1 = xxx 1
2 x1
e 2 = xx22x 1
. Entao 1 e 2 sao numeros positivos,
ja que x x1 , x2 x e x2 x1 o sao. Alem disso, 1 + 2 =
xx1 +x2 x 2 2
x2 x1
= 1 e 1 x1 + 2 x2 = x x1xx+x2 x1
2 xx
= x. Por outro lado,
se 1 x1 + 2 x2 = x, com 1 + 2 = 1, entao 1 x1 + (1 1 )x2 = x,
x
de onde resulta 1 = xxx 1
2 x1
e 2 = xx22x 1
, mostrando que a solucao
e unica.
(b) Se x = 1 x1 + 2 x2 , com 1 + 2 = 1, 1 > 0 e 2 > 0, entao x
e uma media ponderada, com pesos positivos, de x1 e x2 . Logo,
x1 < x < x 2 .
(c) Temos x = x1 + (1 1)x1 + 2 x2 = x1 + 2 (x2 x1 ). Mas 1 > 1
implica em 2 < 0. Logo, x < x1 , ou seja, x (, x1 ).
(d) Temos x = x2 + (1 2 )x2 + 1 x1 = x2 1 (x2 x1 ). Como
1 < 0, temos x > x2 , ou seja, x (x2 , +).

20. (a) Utilizando o exerccio anterior, x pode ser escrito na forma x =


1 x1 + n xn com 1 e n positivos e 1 + n = 1. Ou seja, x
pode ser escrito na forma pedida com 2 = = n1 = 0. A
representacao nao e unica quando n > 2. Por exemplo, ainda
usando o exerccio anterior, todo x tambem pode ser escrito na
forma i xi + j xj , onde i e j sao quaisquer ndices tais que xi
x xj . Se n > 2, certamente ha mais de um modo de escolher i
e j.
(b) Basta observar que x e uma media ponderada de x1 , x2 , . . . , xn .
Como x1 e o menor dos numeros e xn e o maior e os pesos sao
todos positivos, resulta que x1 < x < xn .
301,75+101,67
21. A altura media e 40
= 1, 73.

7
Unidade 22

Exerccios Recomendados
1. Se os numeros tem soma constante, sua media aritmetica A e tambem
constante. Pela desigualdade das medias, o maior valor possvel para
a media geometrica G e igual a A, o que ocorre quando os numeros
sao iguais. Logo, o produto dos numeros (que e o quadrado de G) e
maximo quando os numeros sao iguais.

2. Se os numeros tem produto constante, sua media geometrica G e tambem


constante. Pela desigualdade das medias, o menor valor possvel para
a media aritmetica A e igual a G, o que ocorre quando os numeros sao
iguais. Logo, a soma dos numeros (que e dobro de A) e mnima quando
os numeros sao iguais.
1
+ x1 ++ x1
q
3. Da desigualdade das medias, temos x1 2
n
n
n x11 x12 x1n ,
ou seja, H1 G1 . Da, decorre, H G, com igualdade somente quando
1
x1
= x12 = = x1n , ou seja, quando todos os numeros sao iguais.

4. Temos
n
X n
X n
X
(xk A)2 = (xk )2 2A xk + nA2
k=1 k=1 k=1
= nQ 2A nA + nA2 = n(Q2 A2 )
2

Como uma soma de quadrados e necessariamente nao negativa, resulta


Q2 A2 ou, equivalentemente, Q A. Alem disso, so se tem igualdade
quando cada termo da soma inicial e nulo, ou seja, quando xk = A, para
todo k, o que significa que todos os numeros xk sao iguais.
q
5. A media geometrica de ab12 , ab22 , . . . , abnn e n ab11ab22b
an
n
= 1 (note que os
produtos no numerador e denominador sao iguais, ja que b1 , b2 , . . . , bn
sao uma reordenacao de a1 , a2 , . . . , an ). Logo, sua media aritmetica e
b1 b
a1
+ a2 ++ abn
maior que ou igual a 1. Assim 2
n
n
1 ou, equivalentemente,
b1
a1
+ ab22 + + abnn n.

8
6. Aplicando a desigualdade entre as medias aritmetica e geometrica aos
pares (x2 , y 2 ), (y 2 , z 2 ) e (z 2 .y 2 ), vem
x2 + y 2
xy
2
y2 + z2
yz
2
z 2 + x2
zx
2
Somando membro a membro vem a desigualdade pedida.
7. Aplicando a desigualdade das medias a a1 a2 , a2 a3 e a3 a1 , resulta
a1 a2 + a2 a3 + a3 a1
q
3 a21 a22 a23 ,
3
que equivale a
r
a1 a2 + a2 a3 + a3 a1
3 a1 a2 a3 .
3
2
A outra desigualdade e equivalente a a1 +a32 +a3 a1 a2 +a23a3 +a3 a1 , que,
por sua vez, e equivalente a a21 + a22 + a23 a1 a2 + a2 a3 + a3 a1 , que e
verdadeira, pelo exerccio anterior.
8. Se x 0, entao x3 ax2 +bxc e uma soma de numeros nao positivos na
qual pelo menos c e negativo. Logo, se a equacao x3 ax2 +bxc = 0
possui tres razes reais x1 , x2 , x3 , elas sao necessariamente positivas.
Usando as relacoes entre coeficientes e razes, temos a = x1 + x2 + x3 ,
b = xq1 x2 + x2 x3 + x3 x1 e c = x1 x2 x3 . Pelo exerccio anterior, temos
a b
3
3
3 c. Elevando a sexta potencia e multiplicando por 729,
vem a6 27b3 729c2 .
9. Com x paletos e y calcas, podem ser formadas xy roupas diferentes. O
valor mnimo da soma de dois numeros com produto constante ocorre
quando os numeros sao iguais. Se x pudesse assumir valores nao in-
teiros, o mnimo ocorreria para x = 500 = 22, 36. Claramente, 44
roupas nao bastam, porque o produto maximo de dois numeros com
soma 44 e igual a 22 22 < 500. Mas 45 bastam, ja que 22 23 = 506.
Ou seja, o magico pode usar trajes diferentes em todas as apresentacoes
com 23 (ou 22) calcas e 22 (ou 23) paletos.

9
10. Pela
p formula de Heron, a area de um triangulo de lados a, b e c e S =
1
2
p(p a)(p b)(p c). Para triangulos de permetro constante, a
area maxima quando o produto (p a)(p b)(p c) e maximo. Mas a
soma (p a) + (p b) + (p c) e igual a p, portanto constante. Logo,
para que (p a)(p b)(p c) seja maximo deve-se p a = p b = p c,
ou seja, a = b = c. Logo, dentre os triangulos de permetro constante,
o equilatero e o de maior area.

11. (a) A media geometrica de x e x1 e constante e igual a 1. Logo, sua


media aritmetica e sempre maior que ou igual a 1, isto e, x+ x1 2,
com igualdade quando os numeros sao iguais (ou seja, quando
x = 1).
q
(b) A media geometrica de x e 4/x e constante e igual a x x4 = 2.
Em consequencia, o valor mnimo da media aritmetica e 2 e o valor
mnimo de x + x4 e 4, que ocorre quando x = x4 , ou seja, quando
x = 2.

12. A media aritmetica dos numeros x1 = x2 = = xn = 1+ n1 , xn+1 = 1 e


 n
A = n+2
n+1
= 1+ n+11
, enquanto a sua media geometrica e G = 1 + n1 n+1 .
1
 n
A desigualdade das medias fornece, portanto, 1 + n+1 < 1 + n1 n+1 ,
1
n+1 n
que e equivalente a 1 + n+1 < 1 + n1 (note que a desigualdade
e estrita, ja que os numeros x1 , . . . , xn nao sao todos iguais).

Exerccios Suplementares
1. Aplicando a desigualdade entre a media aritmetica e a media harmonica
1
+ y1 + z1
aos numeros x1 , y1 e 1
z
obtemos x
3
3
x+y+z
ou, equivalentemente,
1
x
+ y1 + z1 x+y+z
9
.

2. Aplicando a desigualdade das medias aos numeros x, ye z, vem
q
x+ y+ z p
3
3
x y z= 3 xyz.

3. Aplicando a desigualdade das medias aos numeros xy, yz e xz, re-


xy+yz+zx
p
sulta 3 2
(xyz) . Como xy + yz + zx 3, temos xyz
3

10
q
xy+yz+zx 3

3
1, com igualdade no caso x = y = z = 1. Por outro
lado, com x = y = n1 e z = n, temos xy + yz + zx = 2 + n12 (portanto,
1 xy + yz + zx 3) e xyz = 1/n, o que mostra que xyz pode ficar
arbitrariamente proximo a zero, bastando para isso tomar n suficien-
temente grande. Portanto, o conjunto de valores de xyz e o intervalo
(0, 1].
q
Pelo Problema Recomendado 7, x + y + z 3 xy+yz+zx 3
3, com

igualdade no caso x = y = z = 33 (a ultima desigualdade decorre de
xy +yz +zx 1). Por outro lado, novamente tomando x = y = n1 e z =
n, temos xy +yz +zx = 2+ n12 (portanto, de novo, 1 xy +yz +zx 3)
e x + y + z = n + n2 , o que mostra que x + y + z pode assumir valores
arbitrariamente
grandes. Logo, o conjunto de valores de x + y + z e o
intervalo [ 3, +).

4. A desigualdade xyz 1 continua valida e xyz continua a poder assumir


valores arbitrariamente proximos a zero (basta tomar, como antes, x =
y = n1 e z = n). Logo, o conjunto de valores de xyz continua a ser o
intervalo (0, 1].
Como antes, os valores de x + y + z podem se tomar arbitrariamente
grandes. Por outro lado, como foi removida a condicao xy+yz+zx 1,
eles tambem podem se tomar arbitrariamente proximos de zero (por
exemplo, tome x = y = z = n1 . Logo, o conjunto de valores de x + y + z
e o intervalo (0, +).

5. Como no Problema Suplementar 3, xyz pode tomar valores arbitraria-


mente proximos de zero. Por outro lado, uma vez removida a restricao
xy + yz + zx 3, xyz pode tambem se tomar arbitrariamente grande
(por exemplo, tome x = y = z = n, com n natural). Logo, o conjunto
de valores de xyz e o intervalo (0, +).

A desigualdade x + y + z 3 do Problema Suplementar 3 continua
a ser valida e, alem disso, tomando x = y = n1 e z = n consegue-se
fazer x + y + z arbitrariamente grande. Logo, o conjunto de valores de
x + y + z continua sendo o intervalo [ 3, +).

6. Pela desigualdade das medias, temos 3 xyz x+y+z
3
. Como x + y + z
x+y+z 3

3, vem xyz 3
1, com igualdade quando x = y = z = 1. Por

11
outro lado, tomando x = y = n1 e z = 1, temos x + y + z = 1 + n2 [1, 3]
e xyz = n12 . Logo xyz pode assumir valores arbitrariamente proximos
de zero. Portanto, o conjunto de valores de xyz e o intervalo (0, 1].
2
Pelo Problema Recomendado 7, temos xy + yz + zx 3 x+y+z 3
.
Justamente com x + y + z 3, isto fornece xy + yz + zx 3, com
igualdade quando x = y = z = 1. Por outro lado, tomando x = y = n1
e z = 1, temos x + y + z = 1 + n2 [1, 3] e xy + yz + zx = n2 + n12 . Logo,
xy + yz + zx pode assumir valores arbitrariamente proximos de zero.
Portanto, o conjunto de valores de xy + yz + zx e o intervalo (0, 3].

7. Como visto no Problema Suplementar 3, temos xy+yz+zx


p
3
3
(xyz)2 .
Portanto, de xyz 1 decorre xy + yz + zx 3, com igualdade quando
x = y = z = 1. Por outro lado, tomando-se x = y = n1 e z = n2 , tem-se
xyz = 1 e xy + yz + zx = 2n + n12 , que assume valores arbitrariamente
grandes para n natural. Logo, o conjunto de valores de xy + yz + zx e
o intervalo [3, +).

Pela desigualdade das medias, x+y+z 3 xyz. Logo, de xyz 1,
3
resulta x + y + z 3 3 xyz 3, com igualdade quando x = y = z = 1.
Por outro lado, tomando-se x = y = n1 e z = n2 , tem-se xyz = 1 e
x + y + z = n2 + n2 , que assume valores arbitrariamente grandes para n
natural. Logo, o conjunto de valores de x + y + z e o intervalo [3, +).

8. Como visto no Problema Suplementar 3, xy+yz+zx


p
3
3 (xyz)2 . Por-
tanto, de xyz = 8 decorre xy + yz + zx 12, com igualdade quando
x = y = z = 2. Por outro lado, tomando x = y = n2 e z = 2n2 ,
vem xyz = 8 e xy + yz + zx = 8n + 4n2 , que pode assumir valores
arbitrariamente grandes para n natural. Logo, o conjunto de valores
de xy + yz + zx e o intervalo [12, +).

Pela desigualdade das medias, x+y+z 3 xyz. Logo, de xyz = 8,
3
resulta x + y + z 3 3 8 = 6, com igualdade quando x = y = z = 2.
Por outro lado, tomando-se x = y = n2 e z = 2n2 , tem-se xyz = 8 e
xy + yz + zx = 2n2 + n4 , que assume valores arbitrariamente grandes
para n natural. Logo, o conjunto de valores de x + y + z e o intervalo
[6, +).

9. Sejam A e G as medias aritmeticas e geometrica de a1 , a2 , . . . , am1 .


Aplicando a desigualdade das medias aos m numeros a1 , a2 , . . . , am1 ,

12

A, temos a1 +a2 ++am1 +A
m a1 a2 am1 A, ou seja (m1)A+A

m
m m
m1 m m1 m1 m1
G A. Da, obtemos A G A, ou seja, A G e,
finalmente, A G. Alem disso, a igualdade ocorre somente quando
a1 = a2 = = am1 = A.

13
EXAMES NACIONAIS DE QUALIFICAO
PROMAT - Exame de Qualificao 2012-1

1.
(10pts) Um corpo est contido num ambiente de temperatura constante. Decorrido o tempo
(em minutos), seja ( ) a diferena entre a temperatura do corpo e do ambiente. Segundo a
Lei do Resfriamento de Newton, ( ) uma funo decrescente de , com a propriedade de
que um decrscimo relativo

( ) ( )
( )

no intervalo de tempo depende apenas da durao desse intervalo (mas no do


momento em que essa observao se iniciou). Isto posto, responda seguinte pergunta:

Num certo dia, a temperatura ambiente era de 30o. A gua, que fervia a 100 o numa panela,
cinco minutos depois de apagado o fogo ficou com a temperatura de 60o. Qual era a
temperatura da gua 15 minutos aps apagado o fogo?

2.
(a) (5pts) Dado um nmero , quanto medem os lados do retngulo de permetro
mnimo cuja rea ?
(b) (10pts) Justifique matematicamente por que no se pode responder o item (a) se
trocarmos mnimo por mximo.

3.
Uma moeda honesta lanada sucessivas vezes.

(a) (10pts) Se a moeda for lanada 4 vezes, qual a probabilidade de que o nmero
observado de caras seja mpar? E se a moeda for lanada 5 vezes?
(b) (5pts) Observando o resultado do item (a), formule uma conjectura sobre a probabilidade
de se observar um nmero mpar de caras em lanamentos da moeda.
(c) (10pts) Demonstre, utilizando induo finita, a conjectura do item (b).
4.
um quadrado, o ponto mdio do lado e o ponto mdio do lado .
Os segmentos e cortam-se em .

(a) (5pts) Mostre que .


(b) (5pts) Calcule a razo .
(c) (5pts) Se calcule a rea do quadriltero .

Obs: Para mostrar os itens (b) e (c) voc pode usar o resultado do item (a) mesmo que no
o tenha demonstrado.

5.
Na figura abaixo, um cubo de aresta 1. e so arestas e a
face est contida em um plano horizontal . Seja o tetraedro . Seja um
ponto da aresta (diferente de e de ) e o plano paralelo a que passa por . A
interseco de com o quadriltero , como mostrado na figura.

(a) (5pts) Mostre que um retngulo.


(b) (5pts) Mostre que o permetro de igual a , independentemente do
ponto .

H
G
E
F

P
Q
X N
D M
C
A
B

(Ateno: como a folha de questes no ser olhada na correo, se usar novos


elementos na figura conveniente explicit-los no caderno de respostas.)
6.
Um truque de adivinhao de nmeros.
(a) (5pts) Descreva e justifique mtodos prticos para obter os restos da diviso por 9, 10
e 11, respectivamente, de um nmero natural escrito no sistema decimal.
(b) (5pts) Ache as solues mnimas de cada uma das seguintes congruncias:
i.
ii.
iii.
(c) (10pts) Um mgico pede a sua audincia para escolher um nmero natural de pelo
menos dois algarismos e menor do que 1000, e de lhe revelar apenas os restos ,
e da diviso de por 9, 10 e 11, respectivamente (tarefa fcil, pelo item (a)). Sem
nenhuma outra informao ele consegue descobrir . Explique como ele consegue
fazer isto.
(d) (5pts) Supondo que a plateia tenha dado as seguintes informaes ao mgico: ,
e , qual foi o valor de que o mgico achou?
PROFMAT Exame de Qualificao 2012-1

Gabarito

1.
(10pts) Um corpo est contido num ambiente de temperatura constante. Decorrido o
tempo (em minutos), seja ( ) a diferena entre a temperatura do corpo e do ambiente.
Segundo a Lei do Resfriamento de Newton, ( ) uma funo decrescente de , com a
propriedade de que um decrscimo relativo

( ) ( )
( )

no intervalo de tempo depende apenas da durao desse intervalo (mas no


do momento em que essa observao se iniciou). Isto posto, responda seguinte
pergunta:

Num certo dia, a temperatura ambiente era de 30o. A gua, que fervia a 100 o numa
panela, cinco minutos depois de apagado o fogo ficou com a temperatura de 60o. Qual era
a temperatura da gua 15 minutos aps apagado o fogo?

SOLUO: Pela Lei do Resfriamento de Newton, a funo ( ), em que o


momento em que o fogo foi apagado, cumpre as hipteses do Teorema de Caracterizao
das funes de tipo exponencial. Logo existe uma constante , com , tal que
( ) , onde ( ). Temos Logo ( ) . O
problema nos diz que ( ) Portanto e da vem .

Segue-se que ( ) ( ) e que ( ) . Portanto, 15


minutos aps o fogo ser apagado, a temperatura da gua de aproximadamente
graus.

Alternativamente, pode-se usar a informao sobre o decrscimo relativo constante de


( ) diretamente. Temos ( ) e ( ) . Portanto

( ) ( )
( )

e, assim, ( ) ( ) Pela propriedade mencionada,

( ) ( ) ( ) ( )
( ) ( )

o que nos conduz a ( ) ( ) ( ) ( ) Em seguida usamos novamente a mesma


informao, obtendo

( ) ( ) ( ) ( )
( ) ( )
o que nos conduz a ( ) ( ) ( ), e o resultado segue.

2.
(a) (5pts) Dado um nmero , quanto medem os lados do retngulo de permetro
mnimo cuja rea ?

(b) (10pts) Justifique matematicamente por que no se pode responder o item (a) se
trocarmos mnimo por mximo.

SOLUO:
(a) Sejam e as dimenses de um retngulo de rea . Ento , ou seja, a mdia
geomtrica de e , dada por , igual a . A mdia geomtrica desses dois
nmeros positivos sempre maior do que ou igual a sua mdia geomtrica, e a igualdade
se d se, e somente se, (o que, por conseguinte, resulta em ). Ento o
permetro , que 4 vezes a mdia aritmtica, mnimo e igual a quando o
retngulo um quadrado de lados iguais a .
(b) Basta mostrar que no existe retngulo de permetro mximo com rea fixada. Para
isso, suficiente mostrar que existem retngulos com essa rea de permetro to grande
quanto se queira. Por exemplo, para cada nmero natural tomamos o retngulo

de lados e . Evidentemente a rea desse retngulo Por outro lado, seu
permetro , que maior do que . Assim, dado qualquer nmero
sempre se pode achar tal que o permetro de maior do que , bastando tomar tal
que .
3.
Uma moeda honesta lanada sucessivas vezes.

(a) (10pts) Se a moeda for lanada 4 vezes, qual a probabilidade de que o nmero
observado de caras seja mpar? E se a moeda for lanada 5 vezes?
(b) (5pts) Observando o resultado do item (a), formule uma conjectura sobre a probabilidade
de se observar um nmero mpar de caras em lanamentos da moeda.
(c) (10pts) Demonstre, utilizando induo finita, a conjectura do item (b).

SOLUO:

(a) Para quatro lanamentos, (1 cara) (3 caras) . Logo, a probabilidade


de um nmero mpar de caras .
Para cinco lanamentos, (1 cara) , (3 caras) ,
(5 caras) .
Logo, para 5 lanamentos a probabilidade de um nmero mpar de caras igual a
.
(b) A conjectura que para todo natural a probabilidade de se obter um nmero mpar de
caras em lanamentos (e, automaticamente, a probabilidade se obter um nmero
par de caras tambm igual a ).
(c) Verifiquemos se a conjectura verdadeira para . A probabilidade de um nmero
mpar de caras em 1 lanamento a probabilidade de ocorrer uma cara em 1 lanamento,
e isso exatamente igual a . Portanto a conjectura vale neste caso. Agora supomos
que a conjectura verdadeira para e vamos verific-la para . Um nmero mpar de
caras em lanamentos ou tem um nmero mpar de caras nos primeiros
lanamentos e uma coroa no ltimo lanamento ou tem um nmero par de caras nos
primeiros lanamentos e uma cara no ltimo lanamento. Ento (no mpar de caras em
lanamentos) (no mpar de caras em lanamentos) (coroa) + (no par de
caras em lanamentos) (cara) .

Obs. O que estamos buscando no item (c) a soma dos coeficientes , com mpar,
dividida por . Se olharmos para a expanso de ( ) , usando o binmio de
Newton, veremos que ela a soma dos coeficientes , com par, subtrada dos
coeficientes , com mpar. Como o resultado zero, a soma dos coeficientes pares
igual dos coeficientes mpares. Por outro lado, ( ) ( ) ,e( )
a soma de todos os coeficientes . Assim, a soma dos coeficientes mpares, dividida por
, deve ser metade desse valor, isto , . Essa soluo no usa induo finita diretamente.
4.
um quadrado, o ponto mdio do lado e o ponto mdio do lado .
Os segmentos e cortam-se em .

(a) (5pts) Mostre que .


(b) (5pts) Calcule a razo .
(c) (5pts) Se calcule a rea do quadriltero .

Obs: Para mostrar os itens (b) e (c) voc pode usar o resultado do item (a) mesmo que no
o tenha demonstrado.

SOLUO:

(a) H vrias maneiras de se calcular essa proporo. Vejamos duas:


Primeira: Bastar mostrar que . Como perpendicular a , ento os
tringulos e so semelhantes. Logo,

Isso implica

Como e , todos os termos do lado direito


podem ser colocados em funo de e a igualdade segue.
Segunda: De fato no necessrio usar a perpendicularidade, pois a afirmao
vale mesmo que seja um paralelogramo. Seja o ponto mdio de .O
segmento corta em que o ponto mdio de . Ento
e . Como os tringulos e so semelhantes, segue que

(b) Aproveitando a construo da segunda soluo de (a), a mesma semelhana de


tringulos nos d . E, sendo , segue que ( ) .

(c) Usaremos [polgono ] para denotar a rea do polgono Evidentemente


Queremos calcular . Mas por causa da
semelhana entre os tringulos e , compartilhando o ngulo oposto a
e , respectivamente,
Portanto .

5.
Na figura abaixo, um cubo de aresta 1. e so arestas e a
face est contida em um plano horizontal . Seja o tetraedro . Seja um
ponto da aresta (diferente de e de ) e o plano paralelo a que passa por . A
interseco de com o quadriltero , como mostrado na figura.

(a) (5pts) Mostre que um retngulo.


(b) (5pts) Mostre que o permetro de igual a , independentemente do
ponto .

H
G
E
F

P
Q
X N
D M
C
A
B

SOLUO:

(a) Primeiro mostremos que e so paralelos a e, portanto, so paralelos


entre si. Mostraremos para , sendo anlogo o caso de . Mas isso segue de
que , que implica semelhante a e, portanto,
paralelo a . Da mesma forma, demonstra-se que e so paralelos a
. Isto mostra que os lados opostos de so iguais. Mas, de fato, so
perpendiculares, pois ortogonal a . Logo, um retngulo.
(b) Como todas as diagonais das faces tm o mesmo tamanho, as faces do tetraedro
so tringulos equilteros. Em particular, equiltero e, portanto,
equiltero. Sendo assim, igual a . Por outro lado, tambm
equiltero, implicando que tambm o . Logo . Ento
De maneira inteiramente anloga, . Logo o
permetro de igual a .
6.
Um truque de adivinhao de nmeros.
(a) (5pts) Descreva e justifique mtodos prticos para obter os restos da diviso por 9, 10
e 11, respectivamente, de um nmero natural escrito no sistema decimal.
(b) (5pts) Ache as solues mnimas de cada uma das seguintes congruncias:
i.
ii.
iii.
(c) (10pts) Um mgico pede a sua audincia para escolher um nmero natural de pelo
menos dois algarismos e menor do que 1000, e de lhe revelar apenas os restos ,
e da diviso de por 9, 10 e 11, respectivamente (tarefa fcil, pelo item (a)). Sem
nenhuma outra informao ele consegue descobrir . Explique como ele consegue
fazer isto.
(d) (5pts) Supondo que a plateia tenha dado as seguintes informaes ao mgico: ,
e , qual foi o valor de que o mgico achou?

SOLUO:

(a) Escrevamos um nmero na sua representao decimal: .


Restos da diviso por 9: Como , temos que

Logo o resto da diviso de por 9 igual ao resto da diviso de


por 9. Podemos repetir o mesmo procedimento a etc.
Restos da diviso por 10: Como , temos que

logo o resto da diviso de por 10 .

Restos da diviso por 11: Como


{
temos que

logo o resto da diviso de por 11 igual ao resto da diviso de


por 11, ao qual podemos aplicar novamente a regra acima etc.
(b) A congruncia equivalente congruncia , cuja
soluo mnima claramente . A congruncia equivalente
congruncia , cuja soluo mnima claramente . A congruncia
equivalente congruncia , cuja soluo mnima
claramente .
(c) O mgico tem que resolver o seguinte sistema de congruncias:

{
O Teorema Chins dos Restos nos diz que o sistema tem uma nica soluo mdulo
, dada pela expresso
( ) ( ) ( )
em que , e so as solues das equaes diofantinas do item (b). Logo

e s existe um valor de satisfazendo essa equao e a restrio de que


.
(d) Temos que achar natural em tal que
.
Ento

Observao. O item (d) poderia ser resolvido de maneira menos educada como segue.
Escrevamos o nmero na sua representao decimal. As informaes
sobre os restos dadas, , e , nos conduzem s seguintes
congruncias: , ,e , que
resolvidas por tentativas nos do o resultado , e .
EXAME DE QUALIFICACAO - 2012.2

Atencao: em cada questao, e sempre permitido usar a resposta de um item anterior, mesmo que esse
item anterior nao tenha sido resolvido.

Questao 1.
(a) Prove que, para quaisquer x, y, z, a, b, c R, tem-se

(ax + by + cz)2 (a2 + b2 + c2 )(x2 + y 2 + z 2 ) .

(b) Excetuando o caso trivial em que a = b = c = 0, mostre que vale a igualdade se, e somente se, existe m R
tal que x = ma, y = mb e z = mc.

Questao 2.
(a) Usando o grafico com o qual se define geometricamente o logaritmo natural ln, mostre que ln(1 + x) < x para
todo x > 0, e da ln x < x.

(b) Tomando x em vez de x nesta ultima desigualdade, prove que para todo x suficientemente grande o quociente
ln x
x pode tornar-se tao pequeno quanto desejemos.
(c) Prove ainda que essa conclusao e valida para logaritmos em qualquer base > 1.

Questao 3.
Uma moeda, com probabilidade 0, 6 de dar cara, e lancada 3 vezes.
(a) Qual e a probabilidade de que sejam observadas duas caras e uma coroa, em qualquer ordem?
(b) Dado que foram observadas duas caras e uma coroa, qual e a probabilidade de que tenha dado coroa no primeiro
lancamento?

Questao 4.
Considere a sequencia an definida como indicado abaixo:

a1 = 1
a2 = 1 + 2
a3 = 2 + 3 + 4
a4 = 4 + 5 + 6 + 7
...

(a) O termo a10 e a soma de 10 inteiros consecutivos. Qual e o menor e qual e o maior desses inteiros? Calcule
a10 .
(b) Forneca uma expressao geral para o termo an .
EXAME DE QUALIFICACAO - 2012.2

Questao 5.
Seja ABC um triangulo equilatero de lado 6 e AD um segmento perpendicular ao plano desse triangulo de
comprimento 8.
(a) Localize o ponto P do espaco que e equidistante dos quatro pontos A, B, C e D e calcule a distancia comum
R = P A = P B = P C = P D.
(b) Calcule o cosseno do angulo entre as retas reversas AC e BD.

Questao 6.
No triangulo ABC assinale o ponto P do lado AC e o ponto Q do lado BC de forma que AP = 31 AC e BQ = 23 BC.
Seja J o ponto de intersecao de AQ e BP .
JA
(a) Mostre que JQ = 34 . Sugestao: Trace QL paralelo a BP e use semelhanca de triangulos.
JB
(b) Calcule a razao JP .
(c) Decida se a area do triangulo BP Q e maior do que, menor do que ou igual a metade da area do triangulo ABC.

Questao 7.
(a) Mostre que nenhum numero natural da forma 4n + 3 pode ser escrito como o quadrado ou a soma de dois
quadrados de numeros naturais.
(b) Mostre que nenhum numero a da forma 11 . . . 1 (n dgitos iguais a 1, n > 1) e o quadrado ou a soma de dois
quadrados de numeros naturais.

Questao 8.
Considere o sistema de congruencias: (
x c1 mod n1
x c2 mod n2
Denotamos como de costume o mdc e o mmc de n1 e n2 por (n1 , n2 ) e [n1 , n2 ], respectivamente.
(a) Mostre que a e a0 sao solucoes do sistema se, e somente se, a a0 mod [n1 , n2 ].1
(b) Mostre que o sistema admite solucao se, e somente se, c2 c1 mod (n1 , n2 ).
(c) Dadas as progressoes aritmeticas (an ) de primeiro termo 5 e razao 14 e (bn ) de primeiro termo 12 e razao 21,
mostre que elas possuem termos comuns (isto e, existem r e s tais que ar = bs ). Mostre que esses termos comuns
formam uma PA e determine seu primeiro termo e sua razao.

1O enunciado esta, infelizmente incorreto: a intencao era pedir: se a e solucao entao a0 e solucao se, e somente se, a a0 mod [n1 , n2 ].
EXAME DE QUALIFICACAO - 2012.2 - GABARITO

Questao 1.
(a) Prove que, para quaisquer x, y, z, a, b, c R, tem-se

(ax + by + cz)2 (a2 + b2 + c2 )(x2 + y 2 + z 2 ) .

(b) Excetuando o caso trivial em que a = b = c = 0, mostre que vale a igualdade se, e somente se, existe m R
tal que x = ma, y = mb e z = mc.

UMA SOLUCAO

(a) Efetuando as operacoes indicadas, vemos que

(a2 + b2 + c2 )(x2 + y 2 + z 2 ) (ax + by + cz)2 = (ay bx)2 + (az cx)2 + (bz cy)2 .

Como todo quadrado em R e 0, segue-se a desigualdade proposta.


(b) Quanto a igualdade, ela e evidente no caso em que x = ma, y = mb e z = mc, para algum m R.
x
Reciprocamente, se ela vale entao, supondo, por exemplo, a 6= 0, pomos m = a. Sabendo (pelo visto acima) que
ay bx = az cx = bz cy =, de x = ma resultam 0 = ay bx = ay mab e da (como a 6= 0) tiramos y = mb.
Analogamente, obtemos z = mc.
EXAME DE QUALIFICACAO - 2012.2 - GABARITO

Questao 2.
(a) Usando o grafico com o qual se define geometricamente o logaritmo natural ln, mostre que ln(1 + x) < x para
todo x > 0, e da ln x < x.

(b) Tomando x em vez de x nesta ultima desigualdade, prove que para todo x suficientemente grande o quociente
ln x
x pode tornar-se tao pequeno quanto desejemos.
(c) Prove ainda que essa conclusao e valida para logaritmos em qualquer base > 1.

UMA SOLUCAO

(a) ln(1 + x) e a area de uma faixa de hiperbole, contida no retangulo de altura 1 e base igual ao intervalo [1, 1 + x]
do eixo das abscissas. Da ln(1 + x) < x, pois x e a area desse retangulo. Como ln x e uma funcao crescente de x,
tem-se ln x < ln(1 + x) < x.

1
(b) Colocando-se x no lugar de x, tem-se ln x < x, ou seja, 2 ln x < x. Dividindo ambos os membros por
x, vem
ln x 2
< .
x x
ln x 2 4
Se desejarmos ter x < , basta tomar x
< , isto e, x > 2 .
(c) Finalmente, se log x significar o logaritmo de x na base a, entao tomando c = log e teremos log x = c ln x, e da

log x ln x
=c ,
x x
ou seja, os dois quocientes diferem apenas por uma constante. Entao
log x 2c
,
x x
4c2
que e menor do que  se x > 2 .
EXAME DE QUALIFICACAO - 2012.2 - GABARITO

Questao 3.
Uma moeda, com probabilidade 0, 6 de dar cara, e lancada 3 vezes.
(a) Qual e a probabilidade de que sejam observadas duas caras e uma coroa, em qualquer ordem?
(b) Dado que foram observadas duas caras e uma coroa, qual e a probabilidade de que tenha dado coroa no primeiro
lancamento?

UMA SOLUCAO

(a) Para sarem duas caras e uma coroa, so ha as 3 possibilidades: coroa-cara-cara, cara-coroa-cara, e cara-cara-
coroa. Cada uma delas tem probabilidade 0, 6 0, 6 (1 0, 6) = 0, 36 0, 4 = 0, 144. Logo a probabilidade de
sarem duas caras e uma coroa e de 3 0, 144 = 0, 432.

(b) Dado que foram observadas duas caras e uma coroa, ocorre apenas uma das 3 possibilidades acima, todas
equiprovaveis. E so uma delas comeca com coroa. Entao a probabilidade de ter dado coroa no primeiro lancamento
e de 31 .
EXAME DE QUALIFICACAO - 2012.2 - GABARITO

Questao 4.
Considere a sequencia an definida como indicado abaixo:

a1 = 1
a2 = 1 + 2
a3 = 2 + 3 + 4
a4 = 4 + 5 + 6 + 7
...

(a) O termo a10 e a soma de 10 inteiros consecutivos. Qual e o menor e qual e o maior desses inteiros? Calcule
a10 .
(b) Forneca uma expressao geral para o termo an .

UMA SOLUCAO

(a) Uma maneira de fazer e ir ate a decima linha, seguindo a regra sugerida, em que o ultimo termo de uma linha
e o primeiro termo da seguinte.

a1 = 1
a2 = 1 + 2
a3 = 2 + 3 + 4
a4 = 4 + 5 + 6 + 7
a5 = 7 + 8 + 9 + 10 + 11
a6 = 11 + 12 + 13 + 14 + 15 + 16
a7 = 16 + 17 + 18 + 19 + 20 + 21 + 22
a8 = 22 + 23 + 24 + 25 + 26 + 27 + 28 + 29
a9 = 29 + 30 + 31 + 32 + 33 + 34 + 35 + 36 + 37
a10 = 37 + 38 + 39 + 40 + 41 + 42 + 43 + 44 + 45 + 46

Entao a10 e a soma de uma P.A. com primeiro termo 37, ultimo termo 46, e razao 1. Portanto
37 + 46
a10 = 10 = 415 .
2

(b) Primeiro vejamos qual e a lei que rege o primeiro termo de an . Chamemos de bn esse primeiro termo. Temos
b1 = 1, b2 = 1, b3 = 2, b4 = 4, b5 = 7 etc. Da b2 b1 = 0, b3 b2 = 1, b4 b3 = 2, b5 b4 = 3, isto e, bn bn1 = n2.
Ou seja, bn bn1 , n 2, e uma P.A. de razao 1 e primeiro termo igual a zero. Entao bn e igual a 1 mais a soma
dessa P.A. ate o termo n 2:
(n 2)(n 1)
bn = 1 + .
2
O ultimo termo de an e igual a bn + n 1. Entao, sendo an a soma de uma P.A. de n termos com o primeiro igual a
bn e o ultimo igual a bn + n 1, resulta que

bn + (bn + n 1)
an = n .
2
Colocando essa expressao explicitamente em funcao de n, temos
 
n1
an = n bn +
2
 
1
= n 1 + (n 1)(n 2 + 1)
2
 
1
= n 1 + (n 1)2 ,
2

que e um polinomio cubico em n.


(Nessas horas, vale a pena conferir se a formula bate com o que esperamos, examinando os primeiros casos. Confira!)
EXAME DE QUALIFICACAO - 2012.2 - GABARITO

Questao 5.
Seja ABC um triangulo equilatero de lado 6 e AD um segmento perpendicular ao plano desse triangulo de
comprimento 8.
(a) Localize o ponto P do espaco que e equidistante dos quatro pontos A, B, C e D e calcule a distancia comum
R = P A = P B = P C = P D.
(b) Calcule o cosseno do angulo entre as retas reversas AC e BD.

UMA SOLUCAO

(a) O ponto P deve estar no plano paralelo a ABC a 4 unidades de distancia de A, pois esse e o plano dos pontos
equidistantes de A e D. Ao mesmo tempo, ele deve estar na reta perpendicular ao plano determinado por ABC que
passa pelo centro H de ABC, pois essa reta e o conjunto de pontos que equidistam de A, B e C.

A distancia de H a qualquer um dos vertices do triangulo e igual a 2 3 (o que pode ser obtido de varios modos).

Como AHP e triangulo-retangulo, de catetos AH = 2 3 e HP = 4, e hipotenusa AP = R, entao R2 = 12 + 16 = 28,

logo R = 2 7.

(b) Chame de Q o ponto do plano de ABC tal que AQBC e paralelogramo. O angulo procurado e o angulo
= QBD.
b Todos os lados do triangulo QBD sao conhecidos: (i) BD = 10, porque BAD e retangulo com catetos
iguais a 6 (o lado do triangulo ABC) e 8 (a altura do ponto D); (ii) BQ = AC = 6; (iii) QD = 10 (pela mesma
razao de (i)). Entao, pela Lei dos Cossenos,

102 = 102 + 62 2 10 6 cos ,

de onde sai cos = 0, 3.


EXAME DE QUALIFICACAO - 2012.2 - GABARITO

Questao 6.
No triangulo ABC assinale o ponto P do lado AC e o ponto Q do lado BC de forma que AP = 13 AC e BQ = 23 BC.
Seja J o ponto de intersecao de AQ e BP .
JA
(a) Mostre que JQ = 34 . Sugestao: Trace QL paralelo a BP e use semelhanca de triangulos.
JB
(b) Calcule a razao JP .
(c) Decida se a area do triangulo BP Q e maior do que, menor do que ou igual a metade da area do triangulo ABC.

UMA SOLUCAO

(a) Para facilitar, sejam BC = 3a e AC = 3b. Tracamos QL paralela a BP . Temos AP = b e P C = 2b.


Da semelhanca dos triangulos BP C e QLC vem
LC QC 1
= = .
PC BC 3
Logo,
2b
LC =
3
e
4b
PL = .
3
Assim,
JA PA b 3
= = = .
JQ PL 4b/3 4

(b) Seja JP = x. Da semelhanca entre AJP e AQL vem

JP AJ 3
= = .
QL AQ 7
Da,
7x
QL = .
3
Da semelhanca dos triangulos BP C e QLC temos
BP BC 3
= = .
QL QC 1
Da, BP = 7x e BJ = 6x. Assim,
JB
= 6.
JP

(c) Seja 3h a distancia de A a reta BC. A area do triangulo ABC e

3a 3h 9ah
S= = .
2 2
O triangulo BP Q tem base BQ = 2a e altura igual a distancia de P a reta BC, que e igual a 2h. A area do triangulo
BP Q e
2a 2h 4ah
S1 = = .
2 2
Assim, S1 = 49 S < 12 S. Em palavras, a area do triangulo BP Q e menor do que a metade da area do triangulo ABC.
EXAME DE QUALIFICACAO - 2012.2 - GABARITO

Questao 7.
(a) Mostre que nenhum numero natural da forma 4n + 3 pode ser escrito como o quadrado ou a soma de dois
quadrados de numeros naturais.
(b) Mostre que nenhum numero a da forma 11 . . . 1 (n dgitos iguais a 1, n > 1) e o quadrado ou a soma de dois
quadrados de numeros naturais.

UMA SOLUCAO

(a) Suponhamos por absurdo que existam x, y, z N tais que z 2 = 4n + 3 ou que x2 + y 2 = 4n + 3. Teramos entao
que z 2 3 mod 4 ou que x2 + y 2 3 mod 4.
Sendo, para todo a N, a 0 mod 4, a 1 mod 4, a 2 mod 4, ou a 3 mod 4, segue que

a2 0 mod 4 ou a2 1 mod 4.

Portanto, z 2 6 3 mod 4 e x2 + y 2 6 3 mod 4, o que e uma contradicao.

(b) Para a = 11, por inspecao, o resultado e obvio. Agora suponhamos n 2 e ponhamos a = 100b + 11, onde
b 0. Portanto, temos
a = 25 4b + 4 2 + 3 = 4(25b + 2) + 3 ,

o que nos diz que a e da forma 4n + 3, logo nao e nem um quadrado nem uma soma de dois quadrados de numeros
naturais.
EXAME DE QUALIFICACAO - 2012.2 - GABARITO

Questao 8.
Considere o sistema de congruencias: (
x c1 mod n1
x c2 mod n2
Denotamos como de costume o mdc e o mmc de n1 e n2 por (n1 , n2 ) e [n1 , n2 ], respectivamente.
(a) Mostre que a e a0 sao solucoes do sistema se, e somente se, a a0 mod [n1 , n2 ]. O enunciado, da forma como
esta, e incorreto. O certo seria: Mostre que, se a e solucao, entao a0 e solucao se, e somente se, a a0 mod [n1 , n2 ].
(b) Mostre que o sistema admite solucao se, e somente se, c2 c1 mod (n1 , n2 ).
(c) Dadas as progressoes aritmeticas (an ) de primeiro termo 5 e razao 14 e (bn ) de primeiro termo 12 e razao 21,
mostre que elas possuem termos comuns (isto e, existem r e s tais que ar = bs ). Mostre que esses termos comuns
formam uma PA e determine seu primeiro termo e sua razao.

UMA SOLUCAO

(a) Obs. Se o sistema admite uma solucao a, entao todo numero da forma a + kn1 n2 e tambem uma solucao. Em
outras palavras, se a0 a mod n1 n2 , entao a0 e uma solucao. Mas isso nao da todas as solucoes, como o proprio
enunciado deixa evidente, a menos que n1 e n2 sejam primos entre si.
Suponha que a seja uma solucao. Se a0 e outra solucao do sistema, subtraindo uma da outra obtemos que
a0 a mod n1 e a0 a mod n2 . Por outro lado, essas duas condicoes juntas implicam que a0 e solucao do sistema.
Ou seja: a0 e solucao do sistema se, e somente se, a0 a mod n1 e a0 a mod n2 . Mas dizer que a0 a mod n1 e
a0 a mod n2 equivale a afirmar que a0 a mod [n1 , n2 ].

(b) Pelo que acabamos de mostrar, o sistema admite uma solucao se, e somente se, ele admite uma solucao
a > max{c1 , c2 }. Portanto, o sistema admite solucao se, e somente se, existem x, y N tais que a c1 = xn1 e
a c2 = yn2 . Sem perda de generalidade, podemos supor c2 c1 . Assim, a existencia de solucoes do sistema e
equivalente a existencia de solucoes da equacao diofantina xn1 yn2 = c2 c1 . Por sua vez, essa equacao diofantina
possui solucao se, e somente se, (n1 , n2 ) divide c2 c1 , o que equivale a c2 c1 mod (n1 , n2 ).

(c) Os termos comuns a ambas as PAs sao solucoes do sistema


(
x 5 mod 14
x 12 mod 21,

o qual possui solucoes dado que 12 5 mod (14, 21), por (b).
Listemos os primeiros termos de ambas as PAs:
an : 5, 19, 33, . . .
bn : 12, 33, 54, . . .
Assim, 33 e o menor termo comum a ambas as PAs e pela parte (a) temos que os termos comuns a ambas PAs sao
dados por cn = 33 + n[14, 21] = 33 + 42n, os quais formam uma PA de primeiro termo 33 e razao 42.
GABARITO - QUALIFICACAO - Setembro de 2012

Questao 1. (pontuacao: 1)
No octaedro regular duas faces opostas sao paralelas. Em um octaedro regular de aresta a, calcule a distancia
entre duas faces opostas.
Obs: no seu calculo, voce pode afirmar as propriedades que esta utilizando sem precisar demonstra-las, mas deve
descreve-las detalhadamente.

D G
C
O M
A
B

Uma solucao:

A figura acima mostra o octaedro regular ABCDEF de aresta a. As diagonais AC e EF determinam o centro O
do octaedro. Seja M o ponto medio da aresta BC. Como a reta BC e perpendicular ao plano (EOM ), os planos
(EBC) e (EOM ) sao perpendiculares. No triangulo retangulo EOM a altura OG relativa a hipotenusa e a distancia
do ponto O a face (EBC). Temos:

a 2
OE = 2 , metade da diagonal do quadrado BEDF ,
a
OM = 2 , distancia do centro do quadrado ABCD ao lado BC, e

EM = a 2 3 , altura do triangulo equilatero EBC.

Assim, a relacao OG.EM = OE.OM fornece OG = a 6 6 .
Como a distancia de O a face (F DA) e igual ao comprimento de OG temos que a distancia entre duas faces opostas

a 6
do octaedro regular e o dobro do comprimento de OG, ou seja, igual a 3 .

Outra solucao:

Podemos decompor a piramide ABCDE em quatro tetraedros congruentes ao tetraedro BCEO. A piramide

a2 .OE a3 2 1 a2 3 a2 3
ABCDE tem volume igual a V = 3 = e o tetraedro BCEO tem volume igual aW = 3 4 OG = 12 OG.
6
a 6
Da igualdade V = 4W segue que OG = 6 , logo a distancia entre duas faces opostas do tetraedro regular e igual a

a 6
3 .

Questao 2. (pontuacao: 1,5)


A figura abaixo mostra uma folha de papel retangular ABCD com AB = 25 cm e BC = 20 cm. Foi feita uma
dobra no segmento AE de forma que o vertice B coincidiu com o ponto P do lado CD do retangulo.
EXAME DE QUALIFICACAO - Setembro de 2012

D P C

A B

(a) Calcule o comprimento do segmento DP .


(b) Calcule a razao entre as areas dos triangulos ADP e P CE.
(c) Calcule o comprimento do segmento AE.

Uma solucao:
a) Como os triangulos AEB e AEP sao congruentes, entao AP = AB = 25 cm . Assim, pelo teorema de Pitagoras,
p
DP = 252 202 = 15 cm

D 15 P 10 C

7,5

25 E
20
12,5

A 25 B

b) Temos P C = 25 15 = 10 cm. O angulo AP E e reto pois e igual ao angulo ABE. Assim, os angulos e
da figura sao complementares e, como consequencia, os triangulos ADP e P CE sao semelhantes, pois possuem os
AD 20
mesmos angulos e a razao de semelhanca e k = PC = 10 = 2. Assim, a razao entre as areas desses triangulos e
2
k = 4.
CE DP CE 15
c) Da semelhanca dos triangulos ADP e P CE tem-se PC = AD , ou seja, 10 = 20 , o que da CE = 7, 5 cm e,
consequentemente, BE = 12, 5 cm.
O teorema de Pitagoras pode ser usado no triangulo ABE para calcular o comprimento de AE. Isto da AE =
p
252 + (12, 5)2 cm.

Observando que, neste problema, AB e o dobro de BE, o calculo acima e imediato. Se um triangulo retangulo

possui catetos a e 2a, entao sua hipotenusa mede a 5. Assim, neste caso, obtemos facilmente que

25 5
AE = 12, 5 5 = cm
2
Questao 3. (pontuacao: 1)
Em uma caixa ha tres dados aparentemente identicos. Entretanto, apenas dois deles sao normais, enquanto o
terceiro tem tres faces 1 e tres faces 6. Um dado e retirado ao acaso da caixa e lancado duas vezes.
EXAME DE QUALIFICACAO - Setembro de 2012

Se a soma dos resultados obtidos for igual a 7, qual e a probabilidade condicional de que o dado sorteado tenha
sido um dos dados normais?

Uma solucao:

Queremos obter

P (dado normal e soma 7)


P (dado normal|soma 7) =
P (soma7)
Mas

6 1
P (soma 7|dado normal) = =
36 6

63 1
P (soma 7|dado anormal) = =
36 2
(o primeiro resultado pode ser qualquer das faces; o segundo, qualquer das tres faces diferentes da obtida no primeiro
lancamento). Logo

P (dado normal e soma 7)


P (dado normal|soma 7) = =
P (soma7)

P (dado normal) P (soma 7|dado normal)


=
P (dado normal) P (soma 7|dado normal) + P(dado anormal) P (soma 7|dado anormal)

2 1
3 6 2
2 1 1 1 =
3 6 + 3 2
5

Outra solucao:

Nomeemos os dados da seguinte forma: N1 e N2 (dados normais) e A (dado anormal).


Se o dado retirado for N1 sao 36 casos possveis para dois lancamentos deste dado e somente 6 casos favoraveis
com soma 7. Se o dado retirado for N2 tambem sao 36 casos possveis para dois lancamentos deste dado e somente 6
casos favoraveis com soma 7. Se o dado retirado for A sao 36 casos possveis para dois lancamentos deste dado, mas
agora 18 casos favoraveis com soma 7.
Logo, no total sao 30 os casos possveis para que a soma de 7 e dentre estes, somente em 12 a soma e proveniente
de dados normais. Portanto

P (dado normal e soma 7) 12 2


P (dado normal|soma 7) = = = .
P (soma 7) 30 5

Questao 4. (pontuacao: 1,5)


A linha poligonal da figura comeca na origem e passa por todos os pontos de coordenadas inteiras do plano
cartesiano.
EXAME DE QUALIFICACAO - Setembro de 2012

-2 -1 1 2 3

-1

-2

(a) Seja n um numero inteiro nao negativo. Mostre que o comprimento c(n) da linha poligonal da origem ate o
ponto (n, n) e igual a 4n2 .
(b) Qual e o comprimento da linha poligonal entre os pontos (7, 10) e (11, 20)?

Uma solucao:

a) A passagem do ponto (n, n) para o ponto (n + 1, n + 1) e ilustrada no diagrama abaixo:

(-n-1, n+1) (n+1, n+1)

(n, n)

(-n-1, -n) (n, -n)

Assim, o comprimento adicional ao passar de (n, n) para (n + 1, n + 1) e 2n + (2n + 1) + (2n + 1) + (2n + 2) =


8n + 4 = 4(2n + 1).
A partir da, pode-se calcular diretamente a soma 4(2.0+1)+4(2.1+1)+...+4(2(n1)+1) = 4(1+3+...+(2n1)) =
4(1 + 2n 1).n/2 = 4n2 .

Alternativamente, pode-se recorrer a inducao finita:


A formula dada claramente vale para n = 0. Suponhamos valida para n. Para n+1, o comprimento e 4n2 +4(2n+1)
= 4(n + 1)2 ; logo, a formula vale para n + 1. Portanto, pelo PIF, vale a formula para todo n inteiro nao negativo.
EXAME DE QUALIFICACAO - Setembro de 2012

Outra solucao:
Ate chegar ao ponto (n, n), teremos passado por todos os pontos de coordenadas inteiras do retangulo [n, n 1]
[(n 1), n], que possui 2n.2n pontos; a linha poligonal tem, assim, comprimento igual a (4n2 1) + 1 (a primeira
parcela exprime o comprimento da poligonal no retangulo acima; a segunda, corresponde ao segmento final).

b) Como ilustra o diagrama abaixo, o comprimento entre (7, 10) e (11, 20, ) e c(20) c(10) + 3 + 40 + 9 =
4.202 4.102 + 52 = 1252.

(20, 20)

3
(10, 10)
(7, 10)
40

(10, -10)
9

(11, -20) (20, -20)

Questao 5. (pontuacao: 1)
Um corpo esta impregnado de uma substancia radioativa cuja meia-vida e um ano. Quanto tempo levara para que
sua radioatividade se reduza a 10% do que e?

Uma solucao:
Se M0 e a massa da substancia radioativa no ano t = 0 e M e a massa da mesma substancia apos t anos, entao
M = M0 .at , para um certo a, com 0 < a < 1. A informacao sobre a meia-vida nos diz que M0 .a1 = 21 M0 , logo a = 12 .
M0
Queremos achar t de modo que M0 .at = 10 , ou seja ( 12 )t = 1
10 . Entao, tomando logaritmos na base 10,

1
t= .
log10 2
[Como log10 2 0, 3010 entao t 3, 3 3 anos e 4 meses]

Questao 6. (pontuacao: 1,5)


p p
Qual e o menor valor da expressao 16x/y + y/(81x) quando x e y sao numeros reais positivos quaisquer?
Justifique sua resposta.

Uma solucao:
p p
A expressao dada e o dobro da media aritmetica entre 16x/y e y/(81x), logo seu valor e maior do que ou igual
ao dobro da media geometrica desses numeros. Ou seja:

4
p p qp p
16x/y + y/(81x) 2. 16x/y. y/(81x) =
3
EXAME DE QUALIFICACAO - Setembro de 2012

Alem disso, a igualdade vale se, e somente se, 16x/y = y/(81x), isto e 16.81.x2 = y 2 . Isto acontece, por exemplo,
quando x = 1 e y = 36. Em outras palavras, com x = 1 e y = 36, a expressao dada atinge seu valor mnimo, que e
igual a 34 . Ha, entretanto, infinitos pontos para os quais este valor mnimo e atingido.
Poderamos tambem completar quadrados para obter a igualdade
p p 1 1 16 1 1 1 4
16x/y + y/(81x) = ((16x/y) 4 (y/(81x)) 4 )2 + 2.( ) 4 = ((16x/y) 4 (y/(81x)) 4 )2 +
81 3
e proceder como acima.

Questao 7. (pontuacao: 1)
Mostre que, para todo n N, e inteiro o numero 17 n7 + 51 n5 + 23
35 n.

Uma solucao:
Pelo Pequeno Teorema de Fermat temos que n7 n mod 7 e n5 n mod 5, logo 7 divide n7 n e 5 divide n5 n,
para todo n. Portanto, a igualdade

1 7 1 5 23 n7 n n5 n
n + n + n= + +n
7 5 35 7 5
nos permite concluir o desejado.

Outra solucao:
Basta usar o Princpio de Inducao Finita:
1 1 23
Se n = 0 a expressao e tambem igual a zero. Observe que quando n = 1, a expressao torna-se 7 + 5 +
35 = 1.
1 7
Suponha agora que 7n + 15 n5 + 23
35 n e um numero inteiro e mostremos que 1
7 (n + 1)7 + 15 (n + 5 23
1) + 35 (n + 1)
tambem e inteiro.
Expandindo em seus binomios de Newton

1 1 1 1 1 1 23 23
{ C(7, 0)n7 + C(7, 1)n6 + ... + C(7, 7)n0 } + { C(5, 0)n5 + C(5, 1)n4 + ... + C(5, 5)n0 } + n +
7 7 7 5 5 5 35 35
vemos que todos os termos desta ultima expressao sao numeros inteiros, exceto talvez 17 n7 , 15 n5 , 23 1 1
35 n, 7 , 5 e 23
35 . Mas,
por hipotese de inducao, a soma dos tres primeiros elementos desta ultima lista e um numero inteiro e a soma dos
tres restantes e igual a 1, logo a expressao toda e um inteiro.

Questao 8. (pontuacao: 1,5)


Um numero natural m e dito um quadrado se existe a N tal que m = a2 .

(a) Mostre que o algarismo das unidades (na base 10) de um quadrado so pode ser um dos seguintes: 0, 1, 4, 5, 6
ou 9.

(b) Mostre que todo quadrado e da forma 4n ou 4n + 1.

(c) Mostre que nenhum numero que escrito na base 10 tem a forma m = dd . . . d (todos os algarismos iguais), com
m > 10 e d {1, 2, 3, 4, 5, 6, 7, 8, 9}, e um quadrado.
EXAME DE QUALIFICACAO - Setembro de 2012

Uma solucao:
a) Escrevamos a = 10b + c, com c {0, 1, 2, 3, 4, 5, 6, 7, 8, 9}. Logo,

m = a2 = (10b + c)2 = 10(10b2 + 2bc) + c2 .

Portanto, o algarismo das unidades de m coincide o algarismo das unidades de c2 . Fazendo variar c no conjunto
{0, 1, 2, 3, 4, 5, 6, 7, 8, 9}, temos os seguintes possveis valores:

c2 = 0, c2 = 1, c2 = 4, c2 = 9, c2 = 16,

c2 = 25, c2 = 36, c2 = 49, c2 = 64, c2 = 81,


o que prova a assercao.

b) Todo numero natural a se escreve na forma 4s + r, com r = 0, 1, 2 ou 3.

Temos que

r = 0: m = a2 = (4s)2 = 4(4s2 ),

r = 1: m = a2 = (4s + 1)2 = 4(4s2 + 2s) + 1,

r = 2: m = a2 = (4s + 2)2 = 4(4s2 + 4s) + 4 = 4(4s2 + 4s + 1),

r = 3: m = a2 = (4s + 3)2 = 4(4s2 + 6s) + 9 = 4(4s2 + 6s + 2) + 1,

logo m e da forma 4n ou 4n + 1.

[Observe tambem que se dois numeros a e b deixam restos r1 e r2 , respectivamente, na divisao por um numero c,
entao o produto ab deixa o mesmo resto na divisao por c que o produto r1 r2 . Assim, apenas temos que olhar para
02 , 12 , 22 , 32 e observar que estes quatro numeros deixam resto 1 ou 0 na divisao por 4].

c) Os casos d = 2, 3, 7 e 8 sao consequencias imediatas do item (a).

Os casos d = 1, 4 e 9 sao tratados a seguir. Temos que

m = 11 . . . 1 = 100x + 11 = 4(25x + 2) + 3,
logo m e da forma 4n + 3. Portanto, m nao e um quadrado.
Os numeros m = 44 . . . 4 = 4(11 . . . 1) e m = 99 . . . 9 = 9(11 . . . 1) nao podem ser quadrados, pois, caso contrario,
11 . . . 1 seria um quadrado.

O caso d = 5 segue do fato de

m = 55 . . . 5 = 100y + 55 = 4(25y + 13) + 3,

logo da forma 4n + 3.

O caso d = 6 segue do fato de m = 66 . . . 6 = 4(25z + 16) + 2, logo da forma 4n + 2.


GABARITO - QUALIFICACAO - Marco de 2013

Questao 1. (pontuacao: 1,5)

E dado um retangulo ABCD tal que em seu interior estao duas circunferencias tangentes exteriormente no ponto T ,
como mostra a figura abaixo. Uma delas e tangente aos lados AB e AD e a outra e tangente aos lados CB e CD.

D C

A B

a) Mostre que a soma dos raios dessas circunferencias e constante (so depende das medidas dos lados do retangulo).
b) Mostre que o ponto T pertence a diagonal AC do retangulo.

Uma solucao:

a) No retangulo ABCD consideremos AB = a e BC = b. Sem perda de generalidade consideraremos b a e


a 2b, pois sem esta ultima condicao as tangencias indicadas nao ocorreriam.

Sejam O e O0 os centros das circunferencias e r e r0 os respectivos raios. Seja s = r + r0 . Como a reta que contem
os centros das circunferencias passa pelo ponto de tangencia entao OO0 = OT + T O0 = r + r0 = s.

D C
r
T O
r E b
O r
r

A a B

A paralela a AB por O e a paralela a BC por O0 cortam-se em E. Temos:

i) r + OE + r0 = a, ou seja, OE = a s

ii) r + EO0 + r0 = b, ou seja, EO0 = b s.


Aplicando o teorema de Pitagoras no triangulo OEO0 temos:

s2 = (a s)2 + (b s)2

Desenvolvendo e simplificando encontramos s2 (2a + 2b)s + a2 + b2 = 0.


Como claramente s < a + b, pois as circunferencias estao no interior do retangulo, o valor de s que procuramos e
a menor raiz da equacao acima. Assim,


p
2a + 2b 4a2 + 8ab + 4b2 4(a2 + b2 ) 2a + 2b 2 2ab
s= = = a + b 2ab,
2 2
0
o que comprova que o valor de s = r + r e constante e so depende das medidas dos lados do retangulo.

b) As retas AO e O0 C sao paralelas ou cincidentes porque fazem 45o com os lados do retangulo. Se forem
coincidentes, o resultado e obvio. Senao tracamos os segmentos AT e T C e o segmento OO0 (que passa por T ). Os
angulos T OA e T O0 C sao congruentes porque sao alternos internos nessas paralelas em relacao a transversal OO0 .

D C

T O
O

A B

Temos ainda que



OT r r 2 OA
0
= 0 = = 0 .
OT r 0
r 2 OC
Como T OA = T O0 C e OT
O0 T = OA
O0 C entao os triangulos T OA e T O0 C sao semelhantes. Assim, OT A = O0 T C
e, portanto, os pontos A, T e C sao colineares.

Questao 2. (pontuacao: 1,0)

O poliedro representado na figura abaixo e tal que:


i) ha exatamente um plano de simetria;
ii) em cada vertice, os planos das faces que se tocam sao perpendiculares dois a dois, sendo possvel decompor o
solido em tres paraleleppedos;
iii) as dimensoes nunca ultrapassam 19;
iv) os comprimentos das arestas sao inteiros maiores do que 1;
v) o volume e igual a 1995.
x

2y
z
z y z

y
3x z 4y

a) Descreva o plano de simetria do poliedro.


b) Encontre os valores de x, y e z.

Uma solucao:
a) O plano de simetria do poliedro e o plano perpendicular as arestas de comprimento 3x, que passa pelos seus
pontos medios.

b) Para calcular o volume do solido, observamos que ele pode ser decomposto como uniao de tres paraleleppedos,
e
V = 12xyz + 6xyz + xyz = 19xyz

Da, 19xyz = 1995, xyz = 105 e as possibilidades para x, y e z sao: 3, 5 e 7; como as dimensoes nao podem
ultrapassar 19, x nao podera ser 7, y nao podera ser 5 ou 7. Deveremos ter, portanto, z = 7, x = 5 e y = 3, de modo
que as dimensoes sao as indicadas na figura:

5 5
7
5
6

7 3 7

3
15 7 12

Questao 3. (pontuacao: 1,5)



O objetivo desta questao e demonstrar que a funcao f (x) = cos x, x 0, nao e periodica, ou seja, nao existe

nenhum numero real positivo T tal que cos x + T = cos x para todo x 0.
a) Encontre todos os valores de T 0 para os quais f (T ) = f (0) e, a seguir, encontre todos os valores de T 0
para os quais f (T ) = f (2T ).
b) Use o tem a) para mostrar que f (x) nao e periodica.
Uma solucao:

a) Se f (T ) = f (0), T 0, entao cos T = cos0 = 1 e

T = 2k, k = 0, 1, 2, . . . T = 4k 2 2 , k = 0, 1, 2, . . . (1)

Reciprocamente se T = 4k 2 2 , k = 0, 1, 2, . . . , entao f (T ) = f (0).



Por outro lado, se f (T ) = f (2T ), T 0, entao cos 2T = cos T e


2T = T + 2m, m Z tais que T + 2m 0 ou 2T = T + 2m, m Z tais que T + 2m 0

Logo

4m2 2
T = 2m 2T 2 2T + T = 4m2 2 T =
2T , m = 0, 1, 2, . . . (2)
32 2

(para esses valores de T e imediato verificar que T + 2m 0) ou

4m2 2
T = 2m 2T + 2 2T + T = 4m2 2 T =
2T + , m = 0, 1, 2, . . . (3)
3+2 2

(para esses valores de T e imediato verificar que T + 2m 0).
4m2
2 4m2
2
Reciprocamente, se T = 32 2
ou T = 3+2 2
, m = 0, 1, 2, . . . , e imediato constatar que f (T ) = f (2T ).

b) Para mostrar que f nao e periodica, suponhamos o contrario, isto e, admitamos a existencia de um numero
positivo T tal que

cos x + T = cos x,

para todo x 0.
Entao,

cos 2T = cos T = cos0,

e, de a) (1), obtemos as igualdades 2T = 4k12 2 e T = 4k22 2 , com k1 e k2 inteiros positivos, logo

k1
2= Q,
k2

o que e impossvel dado que 2 e um numero irracional.

Questao 4. (pontuacao: 1,0)


A derivada de um polinomio p(x) = an xn + an1 xn1 + ... + a1 x + a0 e, por definicao, o polinomio

p0 (x) = nan xn1 + (n 1)an1 xn2 + ... + 2a2 x + a1 .

Admita a regra da derivada do produto:

(p.q)0 (x) = p0 (x).q(x) + p(x).q 0 (x)

e prove que a R cumpre p(a) = p0 (a) = 0 se, e somente se, p(x) = (x a)2 s(x) para algum polinomio s(x).

Uma solucao:
() Supondo p(x) = (x a)2 .s(x) = (x2 2ax + a2 ).s(x), vem que p0 (x) = 2(x a).s(x) + (x a)2 .s0 (x), logo,
p(a) = p0 (a) = 0.
() Reciprocamente, supondo p(a) = p0 (a) = 0, temos, pelo algoritmo da divisao, que p(x) = (x a)q(x) para
algum polinomio quociente q(x). Derivando esta ultima igualdade, vem

p0 (x) = q(x) + (x a).q 0 (x)

donde q(a) = 0, logo, novamente pelo algoritmo da divisao q(x) = (x a).s(x) para algum polinomio s(x), e da
p(x) = (x a)2 .s(x)

Questao 5. (pontuacao: 1,5)

a) Maria tem 10 aneis identicos e quer distribu-los pelos 10 dedos de suas maos. De quantas maneiras diferentes
ela pode fazer isto? Suponha que e possvel colocar todos os aneis em qualquer um dos dedos.
b) Suponha agora que os 10 aneis sejam todos distintos. De quantas maneiras Maria pode distribu-los em seus
dedos? Aqui tambem, suponha que e possvel colocar todos os aneis em qualquer um dos dedos e que a ordem dos
aneis nos dedos e relevante.

Uma solucao:
a) Numeramos os dedos de Maria de 1 a 10. Para descrever como Maria colocou seus aneis, basta dizer quantos
deles ha em cada dedo; se xi e o numero de aneis no i-esimo dedo, temos entao x1 + x2 + + x10 = 10. O numero
9 19!
de solucoes inteiras nao negativas dessa equacao e C10+9 = 9!10! , que e a resposta a nosso problema.
Uma outra maneira de resolver o problema e denotar por A os aneis e por um traco, - , a separacao dos aneis nos
dedos. Assim, por exemplo A A A A A - - - - A - - - - A A A A -, indicara que 5 aneis foram colocados no dedo
de numero 1, um anel no dedo de numero 5 e quatro aneis no dedo de numero 9. Como existe uma correspondencia
biunvoca entre estes anagramas com 19 smbolos (sao 10 As e 9 tracos -) e as configuracoes dos aneis nas maos,
19!
ha, neste caso, 9!10! maneiras diferentes da Maria colocar os 10 aneis.

b) Basta multiplicar o resultado encontado no tem a) por 10!, pois quando os aneis sao identicos, a ordem em que
aparecem nao e importante e cada configuracao com 10 As identicos obtidas em a) gerara 10! configuracoes com
19! 19!
aneis distintos, ja que diferentes permutacoes dos aneis gerarao configuracoes distintas. A resposta e 10! 9!10! = 9! .

Uma outra solucao e a seguinte: Supomos os aneis numerados de 1 a 10. Para descrever como Maria coloca seus
aneis, basta dizer, em cada dedo do primeiro ao decimo, a ordem em que eles aparecem da base do dedo ate a ponta.
Indicando a passagem de um dedo para o seguinte pelo smbolo -, vemos que uma descricao consiste de uma sequencia
formada pelos numeros de 1 a 10 e por nove tracos -. Para construir uma dessas sequencias, ordenamos primeiro
os numeros, o que pode ser feito de maneiras 10! diferentes. Com isso, sao criados 11 espacos entre os numeros
(contam-se tambem os espacos a esquerda e a direita da sequencia numerica), nos quais devemos distribuir os nove
tracos -. Estamos entao buscando o numero de solucoes inteiras nao negativas de y1 + y2 + + y11 = 9 (aqui yi
19!
indica quantos tracos serao colocados no i-esimo espaco vazio), que e 9!10! . A resposta a nosso problema e entao
19! 19!
10! 9!10! = 9! .

Questao 6. (pontuacao: 1,0)


Uma sequencia (an ) e tal que a1 = 1 e

a1 + a2 + + an
an+1 = para todo n 1.
n+1
Mostre que os valores de an , para n 2, sao todos iguais.

Uma solucao:
1
Basta proceder por inducao finita para mostrar qu an = 2 para todo n 2.
a1 1
Para n = 2, temos a2 = a1+1 = 2 = 2.
1
Admitamos agora que aj = 2, para j = 2, . . . n e mostremos que an+1 = 21 .

1
a1 + a2 + + an 1+ 2 + 12 + 1
2 1 + (n 1) 21 1
an+1 = = = =
n+1 n+1 n+1 2
1
Segue, entao, pelo Princpio da Inducao Finita, que an = 2 para todo n 2.

Questao 7. (pontuacao: 1,5)


Seja n N = {1, 2, 3, . . . } e considere os conjuntos:
nn o
A = {d N; d|n} e B= ; cA .
c
Denotemos por S(n) a soma dos divisores naturais de n e por S (n) a soma dos seus inversos.
a) Mostre que A = B e com isto conclua que
S(n)
S (n) = .
n
b) Mostre que n e um numero perfeito se, e somente se,

S (n) = 2.

Uma solucao:

a) Temos que
x A n = xc para algum c A

n
x= para algum c A
c

x B.
Seja A = {d1 , . . . , dr }, di 6= dj para i 6= j, logo
 
X X n n 1 1
S(n) = x= x= + + =n + + = nS (n),
d1 dr d1 dr
xA xB

da segue-se que
S(n)
S (n) =
.
n
b) Por definicao, sabemos que n e perfeito se, e somente se, S(n) = 2n. O resultado segue imediatamente, pois, em
virtude do item (a),
S(n) = 2n S (n) = 2.

Questao 8. (pontuacao: 1,0)


Mostre que se p e primo, p > 3, entao p2 deixa resto 1 na divisao por 24.
Uma solucao:

Observe que como p > 3 e primo, entao p = 3q + r com r = 1 ou 2. Temos assim dois casos a considerar:

Se r = 1, p = 3q + 1 e como p 1 e par, q deve ser par; assim q = 2k para algum k. Logo p2 = (3.2k + 1)2 =
12k(3k + 1) + 1; mas ou k e par ou 3k + 1 e par, assim temos que p2 = 24m + 1, para algum m, como queramos
mostrar.

Se r = 2, p = 3q + 2 e, sendo p e mpar, temos que q tambem sera impar, digamos q = 2k + 1, para algum k.
Substituindo, temos que p2 = (3q + 2)2 = (6k + 5)2 = 12k(3k + 5) + 24 + 1, mas ou k e par ou 3k + 5 e par, assim
temos que p2 = 24m + 1, para algum m, como queramos mostrar.
EXAME NACIONAL DE QUALIFICAO 2013 - 2

NOME:

Questo 1. Considere um tringulo equiltero de lado 3 e seja A1 sua rea. Ao ligar os pontos mdios de cada lado,
obtemos um segundo tringulo equiltero de rea A2 inscrito no primeiro. Para este segundo tringulo equiltero,
ligamos os pontos mdios de seus lados e obtemos um terceiro tringulo equiltero de rea A3 inscrito no segundo e
assim sucessivamente, gerando uma sequncia de reas (An ), n = 1, 2, 3,
. . .
9 3
Usando o Princpio de Induo Finita, mostre que a frmula An = n verdadeira para todo n 1 natural.
4

Questo 2. A sequncia (an ), n 0, definida da seguinte maneira:


a0 = 4
a1 = 6
an
an+1 = , n1
an1
a) Encontre a7 .
b) Encontre a soma dos primeiros 2013 termos da sequncia.

Questo 3. Um cone de revoluo tem altura x e est circunscrito a uma esfera de raio 1. Calcule o volume desse
cone em funo de x.

Questo 4. Na figura, temos um tringulo equiltero ABC e um segundo tringulo P QR cujos lados RP , P Q, QR
so, respectivamente, perpendiculares aos lados AB, BC, AC do tringulo ABC.

C
R

A P B

a) Mostre que o tringulo P QR equiltero. Conclua que AP = BQ = CR.


b) Se o tringulo ABC tem rea 1, encontre a rea do tringulo P QR.
EXAME NACIONAL DE QUALIFICAO 2013 - 2

Questo 5. Sejam f : R R uma funo peridica e g : R R uma funo qualquer.


a) A funo composta g f necessariamente peridica? Em caso afirmativo, demonstre; em caso negativo,
apresente um contra-exemplo.
b) A funo composta f g necessariamente peridica? Em caso afirmativo, demonstre; em caso negativo,
apresente um contra-exemplo.

Questo 6. Considere a equao:

1 3
|x||x 3| = 2|x |
2 2
a) Quais so as razes dessa equao? Explique detalhadamente como as encontrou.
1 3
b) Esboce, em um mesmo plano cartesiano, os grficos das funes f (x) = |x||x 3| e g(x) = 2|x | e marque
2 2
as razes que voc encontrou no tem a).

Questo 7. Determine todos os inteiros X que so solues da congruncia

X 49 + X 14 + X 12 2X 0 (mod7)

Questo 8. Encontre o menor natural k, k > 2008, tal que 1 + 2 + + k seja um mltiplo de 13. Justifique sua
resposta.
EXAME NACIONAL DE QUALIFICAO 2013 - 2

GABARITO

Questo 1.
Considere um tringulo equiltero de lado 3 e seja A1 sua rea. Ao ligar os pontos mdios de cada lado, obtemos
um segundo tringulo equiltero de rea A2 inscrito no primeiro. Para este segundo tringulo equiltero, ligamos
os pontos mdios de seus lados e obtemos um terceiro tringulo equiltero de rea A3 inscrito no segundo e assim
sucessivamente, gerando uma sequncia de reas (An ), n = 1, 2, 3, . . .
9 3
Usando o Princpio de Induo Finita, mostre que a frmula An = n verdadeira para todo n 1 natural.
4
Uma soluo:

Usando do teorema de Pitgoras conseguimos obter a altura, h1 , do primeiro tringulo, a saber:


 2
3 27 3 3
h21 2
=3 = = h1 =
2 4 2
Assim, conclumos que a rea do primeiro tringulo dada por

3.h1 9 3
A1 = =
2 4

9 3
Acabamos de verificar, assim, a validade da frmula An = n para n = 1.
4
Agora, supondo que a frmula seja vlida para algum k, ou seja, que

9 3
Ak = k
4
devemos mostrar que ela vlida para k + 1. Como o tringulo inscrito tem rea igual a 1/4 da do tringulo obtido
no estgio anterior, conclumos que

1 Hip.Ind. 1 9 3 9 3
Ak+1 = Ak = k = k+1
4 4 4 4

9 3
Portanto, o Princpio de Induo Finita garante a validade da frmula An = n para todo natural n 1.
4

Uma outra maneira, mais detalhada, de provar o passo indutivo a seguinte:


Denotemos por hk e Lk a altura e a medida do lado do tringulo da etapa k, respectivamente, e notemos que
 2
2 2 Lk 3Lk
(hk ) = Lk = hk =
2 2

3 2
Logo Ak = L .
4 k
Na etapa k + 1 teremos uma tringulo equiltero cuja medida do lado metade da medida do lado do tringulo
anterior. Alm disso, a altura ser
 2  2
2 Lk Lk 3Lk
(hk+1 ) = = hk+1 =
2 4 4

Portanto, a rea do tringulo da etapa k + 1

hk+1 L2k

Ak+1 =
2
2
1 3Lk
=
4 4
1
= Ak
4

Hip.Ind. 1 9 3
=
4 4k

9 3
= ,
4k+1

k + 1.
o que prova que a frmula vale para
9 3
Por Induo, a frmula An = n vale para todo natural n 1.
4

Questo 2.
A sequncia (an ), n 0, definida da seguinte maneira:
a0 = 4
a1 = 6
an
an+1 = , n1
an1
a) Encontre a7 .
b) Encontre a soma dos primeiros 2013 termos da sequncia.

Uma soluo:

a) Basta fazer um clculo direto: a7 = 6. Na verdade a sequncia dada por 4, 6, 6/4, 1/4, 1/6, 4/6, 4, 6,... e
vemos que ela se repete em ciclos de tamanho 6; os termos de ndices n = 6, 12, ..., 6k, ... k N so todos iguais a 4;
isto ser usado no tem b).
b) Para encontrarmos a soma, primeiramente observamos que a soma do seis primeiros termos a0 + a1 + . . . a5
6 1 1 4 151 151
igual a 4 + 6 + + + + = . Assim, at 2009 (incluindo-o) temos 335 blocos iguais a . Portanto, a soma
4 4 6 6 12 12
solicitada igual a

151 6 1 50726 25363


335( )+4+6+ + = = .
12 4 4 12 6

Questo 3.
Um cone de revoluo tem altura x e est circunscrito a uma esfera de raio 1. Calcule o volume desse cone em funo
de x.

Uma soluo:

x
T
O 1
1

A M r B

Sejam:
AB um dimetro da base do cone,
M o centro da base,
C o vrtice do cone,
O o centro da esfera inscrita no cone e
T o ponto de tangncia da geratriz CB do cone com a esfera.
Temos CM = x e OM = OT = 1.

Seja r o raio da base do cone. O comprimento de uma geratriz do cone CB = x2 + r 2 .
Como CM perpendicular a AB e OT perpendicular a CB, os tringulos CT O e CM B so semelhantes. Da,

OC OT x1 1
= =
CB MB 2
x +r 2 r
Elevando ao quadrado e aplicando a propriedade das propores que permite obter nova frao equivalente s
anteriores subtraindo-se numeradores e denominadores (ou fazendo os clculos), temos:
x2 2x + 1 1 x2 2x x2 x
2 2
= 2 = = r2 =
x +r r x2 x x2
1 2 x x2
O volume do cone V = r x = x= .
3 3 x2 3(x 2)

Questo 4. Na figura, temos um tringulo equiltero ABC e um segundo tringulo P QR cujos lados RP , P Q, QR
so, respectivamente, perpendiculares aos lados AB, BC, AC do tringulo ABC.

C
R

A P B

a) Mostre que o tringulo P QR equiltero. Conclua que AP = BQ = CR.


b) Se o tringulo ABC tem rea 1, encontre a rea do tringulo P QR.

Uma soluo:

a) Basta observar que cada um dos ngulos do tringulo menor P QR mede 60o para concluir que ele equiltero.
Para mostrar que AP = BQ notamos que os tringulos P AR e QBP so congruentes, pois so semelhantes e
acabamos de mostrar que P R = BQ. Analogamente AP = CR.
Logo AP = BQ = CR.

b) Primeiramente, notemos que AP R um tringulo retngulo 30o -60o -90o , ou seja, a metade
de um tringulo
AP
1
equiltero. Logo, temos que AR = 2AP , e, consequentemente, P B = AR = 2AP , ou ainda, . =
PB
2
1
Se chamarmos L o comprimento do lado do tringulo ABC e l o lado do tringulo P QR, temos que AP = L e
3
2
AR = P B = L. Da,
3
P itagoras 4 2 1 2 1
l2 = (RP )2 = L L = L2
9 9 3
1 1
e a razo entre as reas dos tringulos P QR e ABC , logo o tringulo P QR tem rea igual a .
3 3

Questo 5.
Sejam f : R R uma funo peridica e g : R R uma funo qualquer.
a) A funo composta g f necessariamente peridica? Em caso afirmativo, demonstre; em caso negativo,
apresente um contra-exemplo.
b) A funo composta f g necessariamente peridica? Em caso afirmativo, demonstre; em caso negativo,
apresente um contra-exemplo.

Uma soluo:

a) Seja T > 0 o perodo de f , ento f (x + T ) = f (x), x R.


Como
(g f )(x + T ) = g(f (x + T )) = g(f (x)) = (g f )(x), x R,

conclumos que g f tambm peridica.

b) falso. Considere, por exemplo, as funes f (x) = sen(x) e g(x) definida por

0, se x < 0


g(x) =
, se x 0


2
Ento f g no peridica.

0, se x < 0


(f g)(x) =

1, se x 0

Se existisse T > 0 tal que (f g)(x + T ) = (f g)(x), x R, tomando x = T , (f g)(0) = 1 = (f g)(T ) = 0,


uma contadio.

Questo 6.
Considere a equao:

1 3
|x||x 3| = 2|x |
2 2
a) Quais so as razes dessa equao? Explique detalhadamente como as encontrou.
1 3
b) Esboce, em um mesmo plano cartesiano, os grficos das funes f (x) = |x||x 3| e g(x) = 2|x | e marque
2 2
as razes que voc encontrou no tem a).

Uma soluo:

a) A equao equivalente as igualdades:


x2 3x = 4x 6, ou x2 7x + 6 = 0, cujas razes so x = 1 e x = 6; e
x2 3x = 4x + 6 ou x2 + x 6 = 0, cujas razes so x = 3 e x = 2.
Logo, temos quatro razes: -3, 1, 2 e 6.

b) Os grficos das funes esto esboados na figura abaixo:


f(x)

g(x)

-3 12 6

razes

Questo 7.
Determine todos os inteiros X que so solues da congruncia

X 49 + X 14 + X 12 2X 0 (mod7)

Uma soluo:

Se X 0(mod7), claro que X soluo da congruncia dada. Podemos ento supor que 7 no divide X e
procurar outras possveis solues. Neste caso, pelo Teorema de Fermat, sabemos que

X 6 1(mod7)

e que X 7 X(mod7). Conclumos que X 49 (X 7 )7 X 7 X(mod7), X 14 (X 7 )2 X 2 (mod7) e X 12 (X 6 )2


1(mod7). Substituindo na congruncia dada, temos:

X 49 + X 14 + X 12 2X X 2 X + 1(mod7)

Analisando cada caso (exceto X 0(mod7) que j sabemos ser soluo da congruncia original), temos a tabela
abaixo, na qual todas as congruncias so mdulo 7.

X 1 X2 X + 1 1

X 2 X2 X + 1 3
X 3 X2 X + 1 0

X 4 X2 X + 1 6

X 5 X2 X + 1 0

X 6 X2 X + 1 3

As solues de X 49 + X 14 + X 12 2X 0 (mod7) so X 0, X 3 e X 5, ou seja, o conjunto soluo :

S = {X : X = 7K, K Z} {X : X = 7K + 3, K Z} {X : X = 7K + 5, K Z}

Questo 8.
Encontre o menor natural k, k > 2008, tal que 1 + 2 + + k seja um mltiplo de 13. Justifique sua resposta.

Uma soluo:
k(k + 1)
Sabemos que 1 + 2 + + k = . Assim, para que a soma seja um mltiplo de 13, temos que ter que k(k + 1)
2
um mltiplo de 13 e j que 13 um nmero primo, ento ou k ou k + 1 um mltiplo de 13. Como queremos o
menor valor de k para que isto acontea, devemos ter que k + 1 um mltiplo de 13; assim k + 1 = 2015 e portanto
k = 2014.
EXAME NACIONAL DE QUALIFICAO 2013 - 2

GABARITO

Questo 1.
Considere um tringulo equiltero de lado 3 e seja A1 sua rea. Ao ligar os pontos mdios de cada lado, obtemos
um segundo tringulo equiltero de rea A2 inscrito no primeiro. Para este segundo tringulo equiltero, ligamos
os pontos mdios de seus lados e obtemos um terceiro tringulo equiltero de rea A3 inscrito no segundo e assim
sucessivamente, gerando uma sequncia de reas (An ), n = 1, 2, 3, . . .
9 3
Usando o Princpio de Induo Finita, mostre que a frmula An = n verdadeira para todo n 1 natural.
4
Uma soluo:

Usando do teorema de Pitgoras conseguimos obter a altura, h1 , do primeiro tringulo, a saber:


 2
3 27 3 3
h21 2
=3 = = h1 =
2 4 2
Assim, conclumos que a rea do primeiro tringulo dada por

3.h1 9 3
A1 = =
2 4

9 3
Acabamos de verificar, assim, a validade da frmula An = n para n = 1.
4
Agora, supondo que a frmula seja vlida para algum k, ou seja, que

9 3
Ak = k
4
devemos mostrar que ela vlida para k + 1. Como o tringulo inscrito tem rea igual a 1/4 da do tringulo obtido
no estgio anterior, conclumos que

1 Hip.Ind. 1 9 3 9 3
Ak+1 = Ak = k = k+1
4 4 4 4

9 3
Portanto, o Princpio de Induo Finita garante a validade da frmula An = n para todo natural n 1.
4

Uma outra maneira, mais detalhada, de provar o passo indutivo a seguinte:


Denotemos por hk e Lk a altura e a medida do lado do tringulo da etapa k, respectivamente, e notemos que
 2
2 2 Lk 3Lk
(hk ) = Lk = hk =
2 2

3 2
Logo Ak = L .
4 k
Na etapa k + 1 teremos uma tringulo equiltero cuja medida do lado metade da medida do lado do tringulo
anterior. Alm disso, a altura ser
 2  2
2 Lk Lk 3Lk
(hk+1 ) = = hk+1 =
2 4 4

Portanto, a rea do tringulo da etapa k + 1

hk+1 L2k

Ak+1 =
2
2
1 3Lk
=
4 4
1
= Ak
4

Hip.Ind. 1 9 3
=
4 4k

9 3
= ,
4k+1

k + 1.
o que prova que a frmula vale para
9 3
Por Induo, a frmula An = n vale para todo natural n 1.
4

Questo 2.
A sequncia (an ), n 0, definida da seguinte maneira:
a0 = 4
a1 = 6
an
an+1 = , n1
an1
a) Encontre a7 .
b) Encontre a soma dos primeiros 2013 termos da sequncia.

Uma soluo:

a) Basta fazer um clculo direto: a7 = 6. Na verdade a sequncia dada por 4, 6, 6/4, 1/4, 1/6, 4/6, 4, 6,... e
vemos que ela se repete em ciclos de tamanho 6; os termos de ndices n = 6, 12, ..., 6k, ... k N so todos iguais a 4;
isto ser usado no tem b).
b) Para encontrarmos a soma, primeiramente observamos que a soma do seis primeiros termos a0 + a1 + . . . a5
6 1 1 4 151 151
igual a 4 + 6 + + + + = . Assim, at 2009 (incluindo-o) temos 335 blocos iguais a . Portanto, a soma
4 4 6 6 12 12
solicitada igual a

151 6 50723
335( )+4+6+ = .
12 4 12

Questo 3.
Um cone de revoluo tem altura x e est circunscrito a uma esfera de raio 1. Calcule o volume desse cone em funo
de x.

Uma soluo:

x
T
O 1
1

A M r B

Sejam:
AB um dimetro da base do cone,
M o centro da base,
C o vrtice do cone,
O o centro da esfera inscrita no cone e
T o ponto de tangncia da geratriz CB do cone com a esfera.
Temos CM = x e OM = OT = 1.

Seja r o raio da base do cone. O comprimento de uma geratriz do cone CB = x2 + r 2 .
Como CM perpendicular a AB e OT perpendicular a CB, os tringulos CT O e CM B so semelhantes. Da,

OC OT x1 1
= =
CB MB 2
x +r 2 r
Elevando ao quadrado e aplicando a propriedade das propores que permite obter nova frao equivalente s
anteriores subtraindo-se numeradores e denominadores (ou fazendo os clculos), temos:
x2 2x + 1 1 x2 2x x2 x
2 2
= 2 = = r2 =
x +r r x2 x x2
1 2 x x2
O volume do cone V = r x = x= .
3 3 x2 3(x 2)

Questo 4. Na figura, temos um tringulo equiltero ABC e um segundo tringulo P QR cujos lados RP , P Q, QR
so, respectivamente, perpendiculares aos lados AB, BC, AC do tringulo ABC.

C
R

A P B

a) Mostre que o tringulo P QR equiltero. Conclua que AP = BQ = CR.


b) Se o tringulo ABC tem rea 1, encontre a rea do tringulo P QR.

Uma soluo:

a) Basta observar que cada um dos ngulos do tringulo menor P QR mede 60o para concluir que ele equiltero.
Para mostrar que AP = BQ notamos que os tringulos P AR e QBP so congruentes, pois so semelhantes e
acabamos de mostrar que P R = BQ. Analogamente AP = CR.
Logo AP = BQ = CR.

b) Primeiramente, notemos que AP R um tringulo retngulo 30o -60o -90o , ou seja, a metade
de um tringulo
AP
1
equiltero. Logo, temos que AR = 2AP , e, consequentemente, P B = AR = 2AP , ou ainda, . =
PB
2
1
Se chamarmos L o comprimento do lado do tringulo ABC e l o lado do tringulo P QR, temos que AP = L e
3
2
AR = P B = L. Da,
3
P itagoras 4 2 1 2 1
l2 = (RP )2 = L L = L2
9 9 3
1 1
e a razo entre as reas dos tringulos P QR e ABC , logo o tringulo P QR tem rea igual a .
3 3

Questo 5.
Sejam f : R R uma funo peridica e g : R R uma funo qualquer.
a) A funo composta g f necessariamente peridica? Em caso afirmativo, demonstre; em caso negativo,
apresente um contra-exemplo.
b) A funo composta f g necessariamente peridica? Em caso afirmativo, demonstre; em caso negativo,
apresente um contra-exemplo.

Uma soluo:

a) Seja T > 0 o perodo de f , ento f (x + T ) = f (x), x R.


Como
(g f )(x + T ) = g(f (x + T )) = g(f (x)) = (g f )(x), x R,

conclumos que g f tambm peridica.

b) falso. Considere, por exemplo, as funes f (x) = sen(x) e g(x) definida por

0, se x < 0


g(x) =
, se x 0


2
Ento f g no peridica.

0, se x < 0


(f g)(x) =

1, se x 0

Se existisse T > 0 tal que (f g)(x + T ) = (f g)(x), x R, tomando x = T , (f g)(0) = 1 = (f g)(T ) = 0,


uma contadio.

Questo 6.
Considere a equao:

1 3
|x||x 3| = 2|x |
2 2
a) Quais so as razes dessa equao? Explique detalhadamente como as encontrou.
1 3
b) Esboce, em um mesmo plano cartesiano, os grficos das funes f (x) = |x||x 3| e g(x) = 2|x | e marque
2 2
as razes que voc encontrou no tem a).

Uma soluo:

a) A equao equivalente as igualdades:


x2 3x = 4x 6, ou x2 7x + 6 = 0, cujas razes so x = 1 e x = 6; e
x2 3x = 4x + 6 ou x2 + x 6 = 0, cujas razes so x = 3 e x = 2.
Logo, temos quatro razes: -3, 1, 2 e 6.

b) Os grficos das funes esto esboados na figura abaixo:


f(x)

g(x)

-3 12 6

razes

Questo 7.
Determine todos os inteiros X que so solues da congruncia

X 49 + X 14 + X 12 2X 0 (mod7)

Uma soluo:

Se X 0(mod7), claro que X soluo da congruncia dada. Podemos ento supor que 7 no divide X e
procurar outras possveis solues. Neste caso, pelo Teorema de Fermat, sabemos que

X 6 1(mod7)

e que X 7 X(mod7). Conclumos que X 49 (X 7 )7 X 7 X(mod7), X 14 (X 7 )2 X 2 (mod7) e X 12 (X 6 )2


1(mod7). Substituindo na congruncia dada, temos:

X 49 + X 14 + X 12 2X X 2 X + 1(mod7)

Analisando cada caso (exceto X 0(mod7) que j sabemos ser soluo da congruncia original), temos a tabela
abaixo, na qual todas as congruncias so mdulo 7.

X 1 X2 X + 1 1

X 2 X2 X + 1 3
X 3 X2 X + 1 0

X 4 X2 X + 1 6

X 5 X2 X + 1 0

X 6 X2 X + 1 3

As solues de X 49 + X 14 + X 12 2X 0 (mod7) so X 0, X 3 e X 5, ou seja, o conjunto soluo :

S = {X : X = 7K, K Z} {X : X = 7K + 3, K Z} {X : X = 7K + 5, K Z}

Questo 8.
Encontre o menor natural k, k > 2008, tal que 1 + 2 + + k seja um mltiplo de 13. Justifique sua resposta.

Uma soluo:
k(k + 1)
Sabemos que 1 + 2 + + k = . Assim, para que a soma seja um mltiplo de 13, temos que ter que k(k + 1)
2
um mltiplo de 13 e j que 13 um nmero primo, ento ou k ou k + 1 um mltiplo de 13. Como queremos o
menor valor de k para que isto acontea, devemos ter que k + 1 um mltiplo de 13; assim k + 1 = 2015 e portanto
k = 2014.
ENQ2014.1 - Gabarito e Pauta de Correcao

Questao 1 [ 1,0 pt ]

O maximo divisor comum de dois inteiros positivos e 20. Para se chegar a esse resultado pelo processo das divisoes
sucessivas, os quocientes encontrados foram, pela ordem, 1, 5, 3, 3, 1 e 3. Encontre os dois numeros.

Solucao
Utilizando o processo das divisoes sucessivas, para os inteiros positivos a, b, obtem-se:

a = b 1 + r; 0 < r < b

b = r 5 + r1 ; 0 < r 1 < r

r = r1 3 + r2 ; 0 < r 2 < r 1

r1 = r2 3 + r3 ; 0 < r 3 < r 2

r2 = r3 1 + r4 ; 0 < r 4 < r 3

r3 = r4 3

Portanto, r4 = (a, b) = 20 e r3 = 60. Substituindo esses valores nas equacoes anteriores encontra-se a = 6180 e b = 5200.

Pauta de correcao

Demonstrar saber o que e o processo das divisoes sucessivas [0,25]

Realizar todas as etapas do processo para este caso [0,25]

Encontrar os valores corretos dos restos [0,25]

Obter os valores corretos de a e b [0,25]

Questao 2 [ 1,0 pt ::: (a)=0,5; (b)=0,5 ]

Dado um polgono regular convexo de n lados inscrito em um crculo de raio R, seja ln o comprimento dos lados e
seja an a distancia do centro do crculo aos lados do polgono (an e o apotema do polgono).

(a) Calcule l12 e a12 em funcao de R.

(b) Use o item (a) para obter o valor de tg 75 .

Solucao
(a) Dados um crculo de raio R e um dodecagono regular nele inscrito, considere um triangulo cujos lados sejam dois raios do
crculo e um dos lados do dodecagono. Este triangulo tem dois lados de medida R e um de medida l12 . O angulo do triangulo
oposto ao lado de medida l12 e central, correspondendo a um arco de medida 360 /12 = 30 . Assim, pela lei dos cossenos,

(l12 )2 = R2 + R2 2 R R cos 30 ,
logo,

(l12 )2 = R2 (2 3),

e, com isso,

q
l12 = R 2 3.

A altura do triangulo considerado acima, relativa ao lado de medida l12 , tem medida a12 , e divide o triangulo em dois
triangulos retangulos cujos catetos medem a12 e l12 /2, e cuja hipotenusa e R. Assim,
 2
l12
R2 = + (a12 )2 ,
2

logo,
 2
l12
(a12 )2 = R2
2

2 2 3 2
= R R
4

2 2+ 3
= R .
4
Portanto,
p
2+ 3 R
a12 = .
2
(b) O primeiro triangulo considerado no item (a), e isosceles e tem o angulo do vertice de medida 30 . Logo, seus outros
dois angulos medem 75 . O triangulo retangulo utilizado em (a) para o calculo de a12 tem entao catetos adjacente e oposto
de medidas p p
R 2 3 R 2+ 3
l12 /2 = e a12 = ,
2 2
respectivamente. Assim, p  p
a12
R 2 + 3 /2 2+ 3
tg 75 = = p  = p .
l12 /2 R 2 3 /2 2 3
p
Multiplicando numerador e denominador da expressao acima por 2 + 3, obtem-se

tg 75 = 2 + 3.

Pauta de correcao
Item (a)

Encontrar o valor correto de l12 [0,25]

Encontrar o valor correto de a12 [0,25]

Item (b)

Identificar triangulo retangulo com angulo interno de 75 [0,25]

Obter o valor correto de tg 75 [0,25]

Questao 3 [ 1,0 pt ]

Um quadrilatero tem os seus vertices sobre cada um dos lados de um quadrado, cujo lado tem medida 1. Sabendo
que as medidas dos lados desse quadrilatero sao a, b, c e d, prove que

2 a2 + b2 + c2 + d2 4.
Solucao
Denote por ABCD o quadrado de lado 1 e por M N OP o quadrilatero inscrito no quadrado tal que P M = a, M N = b, N O = c
e OP = d, conforme mostra a figura.
Denote ainda por x = AM , y = BN , z = CO e t = DP . Como o quadrado ABCD tem lado 1, tem-se que M B =
1 x, CN = 1 y, OD = 1 z e P A = 1 t. Usando o Teorema de Pitagoras nos triangulos retangulos AM P, M BN, N CO
e ODP, conclui-se que

a2 = x2 + (1 t)2 ,
b2 = (1 x)2 + y 2 ,
c2 = (1 y)2 + z 2 ,
d2 = (1 z)2 + t2 .

Somando, obtem-se

a2 + b2 + c2 + d2 = [x2 + (1 x)2 ] + [y 2 + (1 y)2 ]


+ [z 2 + (1 z)2 ] + [t2 + (1 t)2 ]
= (2x2 2x + 1) + (2y 2 2y + 1)
+ (2z 2 2z + 1) + (2t2 2t + 1)
= f (x) + f (y) + f (z) + f (t),

onde f (x) = 2x2 2x+1, x [0, 1]. Agora e necessario calcular os valores de maximo e mnimo da funcao f (x) = 2x2 2x+1, x
[0, 1]. Visto que f e uma funcao quadratica de coeficiente lder positivo, o valor mnimo ocorre no vertice (desde que esse vertice
esteja dentro do intervalo) e o valor maximo ocorre em um dos extremos do intervalo. Como f (0) = f (1) = 1, a simetria da
parabola assegura que o vertice esta dentro do intervalo e ocorre em x = 1/2. Como f (1/2) = 1/2, obtem-se que
1
f (x) 1, x [0, 1].
2
Desta forma, como a2 + b2 + c2 + d2 = f (x) + f (y) + f (z) + f (t), conclui-se que
1 1 1 1
2= + + + a2 + b2 + c2 + d2 1 + 1 + 1 + 1 = 4.
2 2 2 2

Pauta de correcao

Perceber que as medidas a, b, c e d sao hipotenusas de triangulos retangulos e usar o Teorema de Pitagoras: [0,25].

Perceber que a soma a2 + b2 + c2 + d2 pode ser escrita da forma f (x) + f (y) + f (z) + f (t), onde f (u) = 2u2 2u + 1:
[0,5]

Usar maximos e mnimos de funcoes quadraticas no intervalo [0, 1] para concluir as desigualdades: [0,25].
Questao 4 [ 1,0 pt ::: (a)=0,5; (b)=0,5 ]

De uma caixa contendo 50 bolas numeradas de 1 a 50 retiram-se duas bolas, sem reposicao. Determine a probabi-
lidade de:

(a) o numero da primeira bola ser divisvel por 3 e o numero da segunda bola ser divisvel por 5.

(b) o numero da primeira bola ser divisvel por 4 ou o numero da segunda bola ser divisvel por 6.

Solucao

(a) Das 50 bolas numeradas que constam na caixa, 16 bolas correspondem a numeros divisveis por 3 e 10 bolas correspon-
dem a numeros divisveis por 5. Entretanto ha 3 bolas (15, 30 e 45) que correspondem a numeros divisveis por 15,
sendo, portanto, divisveis tanto por 3 quanto por 5.
O evento retirar da caixa duas bolas, sem reposicao, de modo que o numero da primeira seja divisvel por 3 e da segunda
seja divisvel por 5, pode ser distribudo em dois eventos:
Evento A: O numero da primeira bola e divisvel por 3, mas nao por 5, e o numero da segunda bola e divisvel por 5:

13 10 130
P (A) = =
50 49 2450
Evento B: O numero da primeira bola e divisvel por 3 e tambem por 5, e o numero da segunda bola e divisvel por 5:

3 9 27

P (B) = =
50 49 2450
157
Assim, a probabilidade de o numero da primeira bola ser divisvel por 3 e o da segunda ser divisvel por 5 e 2450
.

(b) Das 50 bolas numeradas que constam na caixa, 12 bolas correspondem a numeros divisveis por 4 e 8 bolas compreendem
a numeros divisveis por 6. Entretanto ha 4 bolas (12, 24, 36 e 48) que compreendem a numeros divisveis por 12, sendo,
portanto, divisveis tanto por 4 quanto por 6.
A probabilidade de retirar da caixa duas bolas, sem reposicao, de modo que o numero da primeira seja divisvel por 4
ou o da segunda seja divisvel por 6, pode ser calculada retirando-se da probabilidade total a probabilidade do evento
o numero da primeira bola nao ser divisvel por 4 e o da segunda nao ser divisvel por 6, que nao satisfaz a condicao
inicial apresentada . Tal evento deve ser analisado sob dois outros eventos que o compoem:
Evento C: O numero da primeira bola nao e divisvel por 4 mas e divisvel por 6, e o numero da segunda bola nao e
divisvel por 6:

4 42 168 84
P (C) = = =
50 49 2450 1225
Evento D: O numero da primeira bola nao e divisvel por 4 e nem e divisvel por 6, e o numero da segunda bola nao e
divisvel por 6:

34 41 1394 697

P (D) = = =
50 49 2450 1225
Desse modo, a probabilidade de o numero da primeira bola nao ser divisvel por 4 e o da segunda nao ser divisvel por
781
6 e 1225
. Logo, a probabilidade de retirar da caixa duas bolas, sem reposicao, de modo que o numero da primeira seja
divisvel por 4 ou o da segunda seja divisvel por 6 e:
781 444
1 =
1225 1225

Pauta de correcao
Item (a)
Calcular corretamente a probabilidade de um dos dois eventos (A ou B) [0,25]

Calcular corretamente a probabilidade do outro evento e encontrar a resposta correta [0,25]

Item (b)

Calcular corretamente a probabilidade de um dos dois eventos (C ou D) [0,25]

Calcular corretamente a probabilidade do outro evento e encontrar a resposta correta [0,25]

Questao 5 [ 1,0 pt ]

Para todo n inteiro positivo, seja


1 1 1
Hn = 1 + + + + .
2 3 n
Prove, por inducao em n, que n + H1 + + Hn1 = nHn , para todo n 2.

Solucao

Seja P (n) a proposicao: n + H1 + + Hn1 = nHn , para todo n 2.


 
3 1
Para n = 2 temos que 2 + H1 = 2 + 1 = 3 = 2 = 2 1 + = 2H2 .
2 2
Suponha agora que P (n) e verdadeira para n = k, ou seja,

k + H1 + + Hk1 = kHk .

Resta provar que P (n) continua valida para n = k + 1.

De fato, (k + 1) + H1 + + Hk1 + H(k+1)1 = (k + H1 + + Hk1 ) + Hk + 1 =


 
1
kHk + Hk + 1 = (k + 1)Hk + 1 = (k + 1) Hk + = (k + 1)Hk+1
k+1
e assim P (k + 1) e verdadeira.

Pauta de correcao

Provar para n = 2 [0,25]

Provar para n = k + 1 [0,75]

Questao 6 [ 1,0 pt ::: (a)=0,25; (b)=0,75 ]

Considere o prisma ABCDEF de bases triangulares da figura.

(a) Mostre que os tetraedros ABCE e CDEF tem o mesmo volume.

(b) Mostre tambem que os tetraedros CDEF e ACDE tem o mesmo volume e conclua que o volume de um
tetraedro e a terca parte do produto da area da base pela altura.

Informacao: Assuma o fato de que dois tetraedros com bases de mesma area e alturas congruentes tem volumes
iguais.
Solucao

(a) Considerando o tetraedro ABCE com base ABC, sua altura e igual a do prisma. Considerando CDEF com base DEF ,
sua altura tambem e igual a do prisma. Como ABC e DEF sao congruentes, pela definicao de prisma, as bases dos
tetraedros tem mesma area. Como as alturas sao congruentes, ABCE e CDEF tem mesmo volume.

(b) Como ACDF e um paralelogramo, os triangulos ACD e CDF sao congruentes, logo tem mesma area. Observe que
estes dois triangulos estao contidos em um mesmo plano . Considerando ACD como base de ACDE, a altura deste
tetraedro e a distancia de E a . Sendo CDF a base de CDEF , a altura e a distancia de B a . Mas, pela definicao de
prisma, BE e paralelo a , logo, as distancias de B e E a sao iguais, e, entao, os tetraedros tem mesma altura. Como
a area da base e igual, os volumes sao iguais.
O volume do prisma e dado por Area(ABC) h, onde h e sua altura. Os volumes dos tres tetraedros ABCE, CDEF e
ACDE, nos quais o prisma pode ser decomposto, sao iguais, logo

Area(ABC) h = Volume(ABCE) + Volume(CDEF ) + Volume(ACDE)


= 3Volume(ABCE),

logo Volume(ABCE) = 31 Area(ABC) h.

Pauta de correcao
Item (a)

Concluir a igualdade dos volumes, utilizando que as bases ABC e DEF sao congruentes e que as alturas relativas a
estas bases sao iguais [0,25]

Item (b)

Perceber um dos seguintes fatos: [0,25]


que as bases ACD e CDF tem a mesma area;
que a altura de ACDE relativa ao vertice E e congruente a altura de CDEF relativa a E.

Perceber o outro desses dois fatos e concluir a igualdade dos volumes [0,25]

Concluir que o volume do tetraedro e um terco do volume do prisma, utilizando a decomposicao do prisma nos tetraedros
ACDE, CDEF e ABCE e o fato de que tem mesmo volume. [0,25]
Questao 7 [ 1,0 pt ]

Mostre que a7 a mod 21, para todo inteiro a.

Solucao
Seja a um inteiro qualquer. Observe que 21 = 3 7, com (3,7)=1 e assim [3,7]=21. Como 3 e 7 sao primos, pelo Pequeno
Teorema de Fermat, tem-se que a7 a mod 7 e a3 a mod 3. Tomando a congruencia a3 a mod 3, elevando ao quadrado,
segue que a6 a2 mod 3. Em seguida, multiplicando por a, vemos que a7 a3 mod 3, donde a7 a mod 3. Agora, como
a7 a mod 3 e a7 a mod 7, segue que a7 a mod [3, 7], isto e, a7 a mod 21.
Alternativa 1: Pode-se tambem mostrar que a7 a mod 3 usando a outra forma do Pequeno Teorema de Fermat: Se 3 | a
tem-se que a 0 mod 3 , portanto a7 a mod 3. No caso 3 - a, (a, 3) = 1 e pelo Pequeno Teorema de Fermat a2 1 mod 3.
Elevando ao cubo e em seguida multiplicando por a tem-se que a7 a mod 3.
Alternativa 2: Pode-se usar tambem classes residuais: Seja a um inteiro qualquer. Segue que a 0 mod 3, a 1 mod 3
ou a 2 mod 3. Se a 0 mod 3 tem-se que a7 a mod 3. Se a 1 mod 3 tem-se que a7 1 mod 3, donde a7 a mod 3.
No caso a 2 mod 3, elevando ao quadrado, segue que a7 27 mod 3, onde 27 2 mod 3, portanto a7 a mod 3.

Pauta de correcao
Provar que a7 a mod 7 [0, 25]
Provar que a7 a mod 3 [0, 5]
Concluir que a7 a mod [3, 7] [0, 25]

Questao 8 [ 1,0 pt ::: (a)=0,5; (b)=0,5 ]

Sejam f : X Y e g : Y X duas funcoes. Prove que:


(a) se g f e injetiva, entao f e injetiva.

(b) se f g e sobrejetiva, entao f e sobrejetiva.

Solucao

(a) O objetivo e mostrar que, dados x1 , x2 X satisfazendo f (x1 ) = f (x2 ), entao x1 = x2 . Assuma f (x1 ) = f (x2 ). Como
g : Y X e uma funcao, tem-se que g(f (x1 )) = g(f (x2 )), isto e, (g f )(x1 ) = (g f )(x2 ). Como g f : X X e
injetiva por hipotese, conclui-se que x1 = x2 , ou seja, f : X Y e injetiva.
(b) O objetivo e mostrar que, dado qualquer y Y, existe x X tal que f (x) = y. Visto que f g : Y Y e sobrejetiva,
dado qualquer y Y, existe y1 Y tal que (f g)(y1 ) = y, isto e f (g(y1 )) = y. Denotando por x = g(y1 ) X, conclui-se
que, dado y Y, existe x = g(y1 ) X tal que f (x) = y, isto e, f e sobrejetiva.

Pauta de correcao
Item (a)
Usar corretamente as definicoes de injetividade e composicao de funcoes [0,25]
Concluir corretamente a solucao do item [0,25]
Item (b)
Usar corretamente as definicoes de sobrejetividade e composicao de funcoes [0,25]
Concluir corretamente a solucao do item [0,25]
AVALIAES
AVs
AVALIAES
AV-01
MA12 Matematica Discreta Prova 1 2011

Questao 1.
Considere a sequencia (an )n1 definida como indicado abaixo:

a1 = 1
a2 = 2+3
a3 = 4+5+6
a4 = 7 + 8 + 9 + 10
...

(0.5) (a) O termo a10 e a soma de 10 inteiros consecutivos. Qual e o menor e o qual e o maior desses inteiros?

(0.5) (b) Calcule a10 .

(1.0) (c) Forneca uma expressao geral para o termo an .

Questao 2.
Um comerciante, para quem o dinheiro vale 5% ao mes, oferece determinado produto por 3 prestacoes mensais
iguais a R$ 100,00, a primeira paga no ato da compra.

(1.0) (a) Que valor o comerciante deve cobrar por esse produto, no caso de pagamento a vista?

(1.0) (b) Se um consumidor desejar pagar o produto em tres prestacoes mensais iguais, mas sendo a primeira paga um
mes apos a compra, qual deve ser o valor das parcelas?

Utilize, se desejar, os seguintes valores para as potencias de 1, 05: 1, 052 = 1, 1025; 1, 051 = 0, 9524; 1, 052 = 0, 9070.

Questao 3.
Considere o conjunto dos numeros escritos apenas com os algarismos 1, 2 e 3, em que o algarismo 1 aparece uma
quantidade par de vezes (por exemplo, 2322 e 12123). Seja an a quantidade desses numeros contendo exatamente n
algarismos.

(0.4) (a) Liste todos esses numeros para n = 1 e n = 2, indicando os valores de a1 e a2 .

(0.8) (b) Explique por que an satisfaz a equacao de recorrencia an+1 = (3n an ) + 2an , para n 1 (note que 3n e o
numero total de numeros com n algarismos iguais a 1, 2 ou 3).

(0.8) (c) Resolva a equacao de recorrencia em (b).


MA12 Matematica Discreta Prova 1 2011

Questao 4.

(1.0) (a) Mostre, por inducao finita, que

(2n 1)3n + 1
1 30 + 2 31 + 3 32 + . . . + n 3n1 = .
4

(1.0) (b) Seja (an )n1 progressao geometrica com termo inicial a1 positivo e razao r > 1, e Sn a soma dos n primeiros
r
termos da progressao. Prove, por inducao finita, que Sn r1 an , para qualquer n 1.

Questao 5.
Seja (xn )n0 sequencia definida pela relacao de recorrencia xn+1 = 2xn + 1, com termo inicial x0 R.

(0.5) (a) Encontre x0 tal que a sequencia seja constante e igual a um numero real a.

(1.0) (b) Resolva a recorrencia com a substituicao xn = yn + a, em que a e valor encontrado em (a).

(0.5) (c) Para que valores de x0 a sequencia e crescente? Justifique.


P1 - MA 12 - 2011

Questo 1.
Considere a sequncia ( an )n1 definida como indicado abaixo:

a1 = 1
a2 = 2+3
a3 = 4+5+6
a4 = 7 + 8 + 9 + 10
...

(0.5) (a) O termo a10 a soma de 10 inteiros consecutivos. Qual o menor e o qual o maior desses inteiros?

(0.5) (b) Calcule a10 .

(1.0) (c) Fornea uma expresso geral para o termo an .

UMA RESPOSTA

(a) O primeiro inteiro da soma que define an igual ao nmero de inteiros utilizados nos termos a1 , . . . , an1 , isto
, 1 + 2 + . . . + n 1 mais um, isto , igual a 21 (n 1)n + 1. O ltimo inteiro esse nmero mais n 1. Portanto,
para n = 10, o primeiro inteiro 46 e o ltimo 55.

(b) a10 a soma de uma progresso aritmtica de 10 termos, sendo o primeiro igual a 46 e o ltimo igual a 55. Ento

(46 + 55) 10
a10 = = 101 5 = 505 .
2

(c) No caso de an , trata-se da soma de uma progresso aritmtica de n termos, sendo o primeiro igual a 12 n(n 1) + 1
e o ltimo igual a 21 n(n 1) + 1 + (n 1), ou seja, 12 n(n 1) + n, como visto em (a). Ento
h i h i
1 1
2 n ( n 1 ) + 1 + 2 n ( n 1 ) + n ( n 1) n2 + ( n + 1) n n3 + n
an = n = = .
2 2 2

1
P1 - MA 12 - 2011

Questo 2.
Um comerciante, para quem o dinheiro vale 5% ao ms, oferece determinado produto por 3 prestaes mensais
iguais a R$ 100,00, a primeira paga no ato da compra.

(1.0) (a) Que valor o comerciante deve cobrar por esse produto, no caso de pagamento vista?

(1.0) (b) Se um consumidor desejar pagar o produto em trs prestaes mensais iguais, mas sendo a primeira paga um
ms aps a compra, qual deve ser o valor das parcelas?

Utilize, se desejar, os seguintes valores para as potncias de 1, 05: 1, 052 = 1, 1025; 1, 051 = 0, 9524; 1, 052 =
0, 9070.

UMA RESPOSTA

(a) Trazendo os valores da segunda e da terceira prestaes para o ato da compra, e somando, obtm-se

100 100
100 + + = 100 + 95, 24 + 90, 70 = 285, 94 .
1, 05 1, 052
Ento o comerciante poder cobrar 285,94 reais, de forma que, se deixar seu dinheiro valorizar 5% ao ms, poder
dispor de 100 reais no ato da compra (tirando 100 reais dos 285,94), 100 reais ao final do primeiro ms (deixando
95,24 reais valorizarem 5% durante um ms) e 100 reais ao final do segundo ms (deixando 90,70 reais valorizarem
5% ao ms durante dois meses).

(b) Para o parcelamento desejado pelo consumidor, as parcelas se deslocam um ms adiante. Ento em cada uma
das trs parcelas de 100 reais devem incidir juros de 5%. Portanto, so 3 parcelas de 105 reais.

2
P1 - MA 12 - 2011

Questo 3.
Considere o conjunto dos nmeros escritos apenas com os algarismos 1, 2 e 3, em que o algarismo 1 aparece uma
quantidade par de vezes (por exemplo, 2322 e 12123). Seja an a quantidade desses nmeros contendo exatamente n
algarismos.

(0.4) (a) Liste todos esses nmeros para n = 1 e n = 2, indicando os valores de a1 e a2 .

(0.8) (b) Explique por que an satisfaz a equao de recorrncia an+1 = (3n an ) + 2an , para n 1 (note que 3n o
nmero total de nmeros com n algarismos iguais a 1, 2 ou 3).

(0.8) (c) Resolva a equao de recorrncia em (b).

UMA RESPOSTA

(a) Para n = 1 s h trs nmeros possveis: 1, 2 e 3. Somente os dois ltimos tm um nmero par de algarismos
iguais a 1 (neste caso, nenhum algarismo igual a 1). Ento a1 = 2. Os nmeros de 2 algarismos so: 11, 12, 13, 21,
22, 23, 31, 32, 33, num total de 9 = 32 . Cinco deles tm uma quantidade par de algarismos iguais a 1, ento a2 = 5.

(b) (Antes de fazer o exerccio, pode-se verificar se a frmula est correta para n = 1: 5 = a2 = (31 a1 ) + 2a1 =
3 + a1 = 3 + 2 = 5.) Observa-se primeiro que a quantidade de nmeros com n algarismos tendo uma quantidade
mpar de algarismos iguais a 1 3n an , pois o nmero total de sequncias 3n .
Para obter a relao de recorrncia, observe que todo nmero de n + 1 algarismos uma concatenao de um
nmero de n algarismos com um nmero de 1 algarismo. Para que a quantidade de algarismos iguais a 1 do
nmero de n + 1 algarismos seja par preciso que: ou o nmero de algarismos iguais a 1 de cada um dos nmeros
concatenados seja mpar ou o nmero de algarismos iguais a 1 de cada um dos nmeros concatenados seja par.
Ento, para calcular an+1 , soma-se o nmero de concatenaes do primeiro caso (mpar-mpar) com o nmero de
concatenaes do segundo caso (par-par). Isto d

an+1 = (3n an ) (31 a1 ) + an a1 ,

isto , a frmula do enunciado, j que a1 = 2.

(c) Observa-se que an+1 = an + 3n , apenas simplificando-se a expresso. Isto implica

an = a1 + 31 + 32 + . . . + 3n1 = 1 + (1 + 3 + 32 + . . . + 3n1 ) ,

em que a expresso entre parnteses a soma dos n primeiros termos da progresso geomtrica de termo inicial 1 e
razo 3, que vale
3n 1
.
31
Portanto
3n + 1
an = .
2

3
P1 - MA 12 - 2011

Questo 4.

(1.0) (a) Mostre, por induo finita, que

(2n 1)3n + 1
1 30 + 2 31 + 3 32 + . . . + n 3n1 = .
4

(1.0) (b) Seja ( an )n1 progresso geomtrica com termo inicial a1 positivo e razo r > 1, e Sn a soma dos n primeiros
r
termos da progresso. Prove, por induo finita, que Sn r 1 a n , para qualquer n 1.

UMA RESPOSTA

(a) A equao verdadeira para n = 1, pois 1 30 = 1 e

(2 1 1)31 + 1
= 1.
4
Supondo vlida para n, vamos mostrar que vale para n + 1, isto , vamos mostrar que, acrescentando o termo
(n + 1) 3n , a soma resultar em
(2 ( n + 1 ) 1 )3n +1 + 1
.
4
Usando a hiptese de induo,

(2n 1)3n + 1
1 30 + 2 31 + 3 32 + . . . + n 3n1 + (n + 1)3n = + ( n + 1 )3n .
4
Manipulando a expresso direita,

(2n 1)3n + 1 [2n 1 + 4(n + 1)]3n + 1 (2n + 1)3n+1 + 1 (2 ( n + 1 ) 1 )3n +1 + 1


+ ( n + 1 )3n = = = ,
4 4 4 4
como queramos demonstrar.

r
(b) Para n = 1 a desigualdade verdadeira: como r > 1, ento > 1; e como S1 = a1 > 0, ento S1 = a1 < rr 1 a1 .
r 1
Suponha agora que a desigualdade vale para n, isto , suponha que Sn rr 1 an verdadeira. Vamos provar que
ela vale para n + 1, isto , vamos provar que Sn+1 rr 1 an+1 . Primeiro, escrevemos Sn+1 = Sn + an+1 , pois Sn+1
a soma dos primeiros n termos adicionada do termo n + 1. Usando a hiptese de induo, Sn+1 rr 1 an + an+1 .
a
Como se trata de uma progresso geomtrica an+1 = ran , ou seja, podemos trocar an por nr+1 . Ento Sn+1
r a n +1 1 r
r 1 r + an+1 , isto , Sn+1 ( r 1 + 1) an+1 = r 1 an+1 , que o que queramos demonstrar.

4
P1 - MA 12 - 2011

Questo 5.
Seja ( xn )n0 sequncia definida pela relao de recorrncia xn+1 = 2xn + 1, com termo inicial x0 R.

(0.5) (a) Encontre x0 tal que a sequncia seja constante e igual a um nmero real a.

(1.0) (b) Resolva a recorrncia com a substituio xn = yn + a, em que a valor encontrado em (a).

(0.5) (c) Para que valores de x0 a sequncia crescente? Justifique.

UMA RESPOSTA

(a) Basta achar a tal que 2a + 1 = a. Isto d a = 1. Se x0 = a ento x1 = 2x0 + 1 = 2a + 1 = a = x0 , e, da mesma


forma, x2 = x1 , x3 = x2 , . . ., xn+1 = xn para qualquer n 0, ou seja, a sequncia constante.

(b) Com a substituio sugerida, xn = yn 1. Ento yn+1 1 = 2(yn 1) + 1, isto , yn+1 = 2yn , com y0 = x0 + 1.
Ento yn = 2n y0 = 2n ( x0 + 1) e xn = yn 1 = 1 + 2n ( x0 + 1).

(c) Se x0 + 1 > 0, isto , x0 > 1, ento 2n ( x0 + 1) crescente e xn = 1 + 2n ( x0 + 1) crescente. Se x0 + 1 < 0, isto


x0 < 1, ento xn = 1 + 2n ( x0 + 1) = 1 2n | x0 + 1| descrescente. E se x0 = 1 ento xn constante. De
onde se conclui que xn crescente se, e somente se, x0 (1, +).

5
MA12 Matematica Discreta AV1 2012

Atencao: esta prova deve ser feita sem o uso de calculadoras!

Questao 1.

Uma venda imobiliaria envolve o pagamento de 12 prestacoes mensais iguais a R$ 10.000,00, a primeira no ato
da venda, acrescidas de uma parcela final de R$ 100.000,00, 12 meses apos a venda. Suponha que o valor do
dinheiro seja de 2% ao mes.
(a) (1,0) Se o comprador preferir efetuar o pagamento da parcela final junto com a ultima prestacao, de quanto
devera ser o pagamento dessa parcela?
(b) (1,0) Se o comprador preferir efetuar o pagamento a vista, qual devera ser o valor desse pagamento unico?

Sao dados alguns valores aproximados de 1, 02n :

n 1, 02n
-12 0,788
-1 0,980
12 1,268

Questao 2.

A figura abaixo mostra uma linha poligonal que parte da origem e passa uma vez por cada ponto do plano
cujas coordenadas sao numeros inteiros e nao negativos.
(a) (1,0) O conjunto dos pares de numeros inteiros e nao negativos tem a mesma cardinalidade que os numeros
naturais? Por que?
(b) (1,0) Mostre que o comprimento da linha poligonal da origem ate o ponto (n, n) e n2 + n, para qualquer inteiro
nao negativo n.
(c) (0,5) Qual e o comprimento da linha poligonal da origem ate o ponto (10, 13)?

1
MA12 Matematica Discreta AV1 2012

Questao 3.

(1,5) Mostre, por inducao finita, que se n e um inteiro positivo entao 7n 1 e divisvel por 6.

Questao 4.

Considere a recorrencia xn+2 4xn = 9n, com as condicoes iniciais x0 = x1 = 0.


(a) (0,5) Encontre a solucao geral da recorrencia homogenea xn+2 4xn = 0.
(b) (0,5) Determine os valores de A e B para os quais xn = A + nB e uma solucao da recorrencia xn+2 4xn = 9n.
(c) (1,0) Encontre a solucao da recorrencia original.

Questao 5.

Para todo numero natural n 2, considere o numero N formado por n 1 algarismos iguais a 1, n algarismos
iguais a 2 e um algarismo igual a 5, nesta ordem.
(a) (1,0) Mostre que o numero N pode ser escrito na forma

A 102n + B 10n + C
,
9
onde A, B e C sao constantes independentes de n. Indique os valores de A, B e C.
(b) (0,5) Mostre que N e um quadrado perfeito.

(c) (0,5) Quantos algarismos tem N ? Diga quais sao esses algarismos.

2
AV1 - MA 12 - 2012

Questo 1.

Uma venda imobiliria envolve o pagamento de 12 prestaes mensais iguais a R$ 10.000,00, a primeira no
ato da venda, acrescidas de uma parcela final de R$ 100.000,00, 12 meses aps a venda. Suponha que o valor
do dinheiro seja de 2% ao ms.
(a) Se o comprador preferir efetuar o pagamento da parcela final junto com a ltima prestao, de quanto
dever ser o pagamento dessa parcela?
(b) Se o comprador preferir efetuar o pagamento vista, qual dever ser o valor desse pagamento nico?

So dados alguns valores aproximados de 1, 02n :

n 1, 02n
-12 0,788
-1 0,980
12 1,268

UMA SOLUO

(a) O valor de R$100.000,00 trazido um ms para trs igual a

1
100.000, 00 ' 0, 980 100.000, 00 = 98.000, 00 .
1, 02

(b) Trazendo os valores para a data de compra, o comprador pagar

10.000, 00 10.000, 00 10.000, 00 100.000, 00


10.000, 00 + + ++ + .
1, 02 1, 022 1, 0211 1, 0212
Isso igual a

10.000, 00 (1 + 1, 021 + 1, 022 + . . . + 1, 0211 ) + 100.000, 00 1, 0212


1 1, 0212
= 10.000, 00 + 100.000, 00 1, 0212
1 1, 021
1 0, 788
' 10.000, 00 + 100.000, 00 0, 788
1 0, 980
= 106.000, 00 + 78.800, 00 = 184.800, 00 .

Portanto, se o dinheiro vale 2% ao ms, pagar o esquema de prestaes do enunciado equivale a pagar (aproxima-
damente) R$ 184.800,00 vista.

1
AV1 - MA 12 - 2012

Questo 2.

A figura abaixo mostra uma linha poligonal que parte da origem e passa uma vez por cada ponto do plano
cujas coordenadas so nmeros inteiros e no negativos.
(a) O conjunto dos pares de nmeros inteiros e no negativos tem a mesma cardinalidade que os nmeros
naturais? Por qu?
(b) Mostre que o comprimento da linha poligonal da origem at o ponto (n, n) n2 + n, para qualquer inteiro
no negativo n.
(c) Qual o comprimento da linha poligonal da origem at o ponto (10, 13)?

UMA SOLUO

(a) Chamemos de Z0 o conjunto dos inteiros no negativos. Ento o conjunto dos pontos de R2 com coordenadas
inteiras e no negativas o produto cartesiano Z20 = Z0 Z0 .
Imaginemos que a linha percorrida com velocidade 1 a partir do instante 1 em (0, 0). A figura mostra que se
no instante k a curva est num ponto de Z20 ento no instante k + 1 ela estar em um outro ponto de Z20 . Por
induo, estabelece-se uma funo f : N Z20 em que f (k) o ponto de Z20 alcanado no instante k.
Como todos os pontos so atingidos, f sobrejetiva. Como a linha no passa mais do que uma vez em cada
ponto, f injetiva. Assim, existe uma bijeo entre N e Z20 , mostrando que N e Z20 tm a mesma cardinalidade.

(b) Por inspeo a afirmao verdadeira para n = 0, pois n2 + n = 0 e realmente so 0 passos para chegar no
ponto de partida (n, n) = (0, 0). Agora suponhamos que a afirmao vlida para (n, n), isto , que realmente so
n2 + n passos at se chegar em (n, n) (hiptese de induo). Queremos mostrar que a afirmao vlida quando
aplicada para n + 1, isto , que so (n + 1)2 + (n + 1) passos at se chegar em (n + 1, n + 1).
De (n, n) at (n + 1, n + 1) so necessrios: n passos (para encontrar um dos eixos; mais especificamente, para
encontrar a abscissa, se n par, e para encontrar a ordenada, se n mpar) mais 1 passo (para avanar nesse eixo)
mais n + 1 passos (para voltar diagonal, que o conjunto dos pontos da forma ( x, x )). Assim, so necessrios
n + 1 + (n + 1) = 2n + 2 passos para ir-se de (n, n) a (n + 1, n + 1). Pela hiptese de induo, j foram n2 + n passos
para se chegar em (n, n). Portanto so (n2 + n) + (2n + 2) passos at (n + 1, n + 1). Mas

(n2 + n) + (2n + 2) = (n2 + 2n + 1) + (n + 1) = (n + 1)2 + (n + 1) ,

2
como queramos demonstrar.

Soluo alternativa 1. Para se chegar ao ponto (n, n), preciso percorrer todos os pontos de coordenadas inteiras
do quadrado [0, n] [0, n], exceto os situados em um dos lados. Existem (n + 1)2 pontos de coordenadas inteiras
no quadrado, dos quais n no so visitados. Logo, o comprimento da poligonal (n + 1)2 1 n = n2 + n.

Soluo alternativa 2. A linha poligonal da origem at o ponto (n, n) formada por n segmentos de comprimento
1, por segmentos de comprimento 2k, para k variando de 1 a n 1 e um segmento de comprimento n. Logo, seu
comprimento
( n 1) n
n + 2(1 + 2 + ... + n 1) + n = n + 2 + n = n2 + n .
2

(c) Primeiro, investiga-se se (10, 13) ocorre a 3 passos de distncia (para mais ou para menos) de (10, 10) ou de
(13, 13), no trajeto definido pela curva. Vemos que (10, 13) est 3 unidades verticalmente acima de (10, 10) e 3
unidades horizontalmente esquerda de (13, 13). Quando (n, n) par, como o caso de n = 10, a linha poligonal
prossegue na vertical para baixo, portanto no sentido contrrio ao que esperaramos se fosse encontrar (10, 13) em
3 passos. Quando (n, n) mpar, como o caso de n = 13, a linha poligonal prossegue horizontalmente para a
esquerda. Neste caso, encontrar (10, 13) aps 3 passos.
Portanto, como so 132 + 13 = 169 + 13 = 182 passos at (13, 13) e mais 3 passos at (10, 13), ento so 185 passos
at (10, 13).

3
AV1 - MA 12 - 2012

Questo 3.

Mostre, por induo finita, que se n um inteiro positivo ento 7n 1 divisvel por 6.

UMA SOLUO

Para n = 1, 7n 1 = 7 1 = 6, que divisvel por 6. Ento a afirmao vale para n = 1. Suponhamos que a
afirmao seja vlida para n, isto , suponha que 7n 1 seja mltiplo de 6. Vamos mostrar, com essa hiptese, que
7n+1 1 tambm mltiplo de 6.
Ora, 7n+1 1 = 7n+1 7n + 7n 1 = 7n (7 1) + (7n 1) = 6 7n + (7n 1). O primeiro termo mltiplo de
6, porque tem um fator 6, e o segundo tambm , pela hiptese de induo. Ento a soma mltiplo de 6 e temos
demonstrado o que queramos.

4
AV1 - MA 12 - 2012

Questo 4.

Considere a recorrncia xn+2 4xn = 9n, com as condies iniciais x0 = x1 = 0.


(a) Encontre a soluo geral da recorrncia homognea xn+2 4xn = 0.
(b) Determine os valores de A e B para os quais xn = A + nB uma soluo da recorrncia xn+2 4xn = 9n.
(c) Encontre a soluo da recorrncia original.

UMA SOLUO

(a) Se xn+2 4xn = 0 ento xn+2 = 4xn . Ento x2m = 4m x0 , para todo m 0, e x2m+1 = 4m x1 , para todo n 0.
Escrevendo de outra maneira, a soluo

x0 , x1 , 4x0 , 4x1 , 42 x0 , 42 x1 , 43 x0 , 43 x1 , . . .

1
Tambm pode-se dizer que xn = 2n x0 , para n 0 par, e xn = 2 2n x1 , para n 0 mpar.

(b) Se xn = A + nB ento xn+2 = A + (n + 2) B. Se, alm do mais, ( xn ) soluo de xn+2 4xn = 9n, ento

9n = xn+2 4xn = A + (n + 2) B 4A 4Bn = 3A + 2B 3nB .

Para que 3A + 2B 3nB seja igual a 9n basta que 3A + 2B = 0 (primeira equao) e que 3B = 9 (segunda
equao). Da segunda equao sai imediatamente que B = 3, e, colocando esse valor na primeira, que A = 2.
Ento xn = 2 3n uma soluo da equao no homognea.

(c) Agora vamos combinar a soluo geral da homognea com a soluo particular da no homognea para obter
a soluo de xn+2 4xn = 9n com x0 = x1 = 0. Seja xen = 2 3n a soluo calculada em (b), que satisfaz
xen+2 4e
xn = 9n. Essa soluo no satisfaz as condies iniciais pedidas, pois xe0 = 2 e xe1 = 5. Ento seja
( xbn ) soluo da homognea satisfazendo xb0 = +2 e xb1 = +5. Vamos verificar que ( xn ) definida por xn = xen + xbn
satisfaz ao mesmo tempo as condies iniciais e a relao de recorrncia no homognea.
Ora, x0 = xe0 + xb0 = 2 + 2 = 0 e x1 = xe1 + xb1 = 5 + 5 = 0. Alm disso,

xn+2 4xn = ( xen+2 4e


xn ) + ( xbn+2 4b
xn ) = 9n + 0 = 9n .

Assim, a soluo do problema proposto a sequncia dada por xn = 2 2n 3n 2 = 2n+1 3n 2, para n par,
5
xn = 2 2n 3n 2, para n mpar.

5
OUTRA SOLUO

Esta a soluo que muitos esperavam, que usa equao caracterstica.


(a) A equao caracterstica r2 4 = 0, cujas razes so 2 e +2. Logo a soluo geral da recorrncia xn =
C 2n + D (2)n .

Obs. Note que, embora esta resposta seja diferente da resposta (a) da soluo anterior, ambas esto corretas, mas
esto expressas em termo de outras constantes.

(b) Idntica resposta (b) da soluo anterior.

(c) Somando as duas, obtemos a soluo geral

xn = C 2n + D (2)n 2 3n .

Usando as condies iniciais x0 = 0 e x1 = 0, temos

C+D2 = 0
2C 2D 2 3 = 0

9
Resolvendo o sistema, obtemos C = 4 e D = 14 . Logo, a soluo

9 n 1
xn = 2 (2)n 2 3n .
4 4
No difcil verificar que as duas solues apresentadas so a mesma, mas escritas de formas diferentes.

6
AV1 - MA 12 - 2012

Questo 5.

Para todo nmero natural n 2, considere o nmero N formado por n 1 algarismos iguais a 1, n algarismos
iguais a 2 e um algarismo igual a 5, nesta ordem.
(a) Mostre que o nmero N pode ser escrito na forma

A 102n + B 10n + C
,
9
onde A, B e C so constantes independentes de n. Indique os valores de A, B e C.
(b) Mostre que N um quadrado perfeito.

(c) Quantos algarismos tem N? Diga quais so esses algarismos.

UMA SOLUO

(a) Usando a expanso na base decimal, podemos escrever N como

N = 102n1 + 102n2 + . . . + 10n+1 + 2 10n + 2 10n1 + . . . + 2 101 + 5 .

Ento
N = 10n+1 (1 + 10 + . . . + 10n2 ) + 2 10 (1 + 10 + . . . + 10n1 ) + 5 .

Somando as duas PGs entre parnteses,

10n1 1 10n 1
N = 10n+1 + 20 +5
10 1 10 1
102n 10n+1 + 20 10n 20 + 45
=
9
102n + 10 10n + 25
= .
9
Portanto A = 1, B = 10 e C = 25.

Obs: Outra forma de fazer multiplicar N por 9 usando o algoritmo de multiplicao e ver que fica o nmero
10 . . . 010 . . . 025, onde o bloco de zeros mais esquerda tem n 2 elementos e o bloco de zero mais direita tem
n 1 elementos.

(b) Queremos saber se N = p2 , com p N. Como 102n + 10 10n + 25 = (10n + 5)2 , ento
2
10n + 5

N= .
3

Resta saber se 10n + 5 divisvel por 3. Mas isso verdade, porque como 10n + 5 = 10 . . . 05, com um bloco de n 1
zeros, a soma dos algarismos desse nmero igual a 6.

7
10n +5
(c) A raiz de N o nmero p = 3 . Como 10n + 5 = 10 . . . 05, com um bloco de n 1 zeros, ento tem n + 1
algarismos. Ao dividir por 3, passa a ter n algarismos. Ento p tem n algarismos.
Para saber qual o nmero, podemos escrever

10n 1 6
p= + .
3 3
O termo da esquerda 33 . . . 3 (n vezes) e o da direita igual a 2. Ento p = 3 . . . 35, onde 3 aparece repetido n 1
vezes.

8
Sociedade Brasileira de Matemtica
Mestrado Profissional em Matemtica em Rede Nacional

MA12 Matemtica Discreta


Avaliao 1 - MA 12
27 de abril de 2013

1. (valor 3,0)
Paulo economizou durante muitos anos e tem, hoje, R$ 500.000,00 aplicados em um investi-
mento que rende juros de 1% ao ms. A partir do prximo ms, ele pretende fazer uma retirada
mensal de R$ 1.000,00.
a) Seja sn o saldo que resta da aplicao, aps fazer a n-sima retirada. Exprima sn+1 em
termos de sn . D tambm a condio inicial da recorrncia obtida. (pontuao parcial 0,5)
b) Obtenha uma expresso para sn em funo de n. (pontuao parcial 1,5)
c) Qual a retirada mensal mxima que Paulo pode fazer de modo que o saldo da aplicao
nunca se torne negativo? (pontuao parcial 1,0)

2. (valor 2,5)
a) Para que valores de b existe uma progresso geomtrica para a qual a soma dos n primeiros
termos igual a 3n+1 + b, para todo n natural? (pontuao parcial 1,0)
b) Quais so o primeiro termo e a razo dessa progresso? (pontuao parcial 1,5)

3. (valor 2,0)
Prove, por induo finita, que

1 1 1 1 1 n
1+ + + + + . . . n1 > ,
2 3 4 5 2 2

para todo n natural.


4. (valor 1,5)
Na figura abaixo temos uma espiral formada por infinitos semicrculos cujos centros pertencem
ao eixo das abscissas. Se o raio do primeiro semicrculo (o maior) igual a 1 e o raio de cada
semicrculo igual metade do semicrculo anterior, determine:
a) o comprimento total da espiral. (pontuao parcial 0,75)
b) a abscissa do ponto P assinttico da espiral. (pontuao parcial 0,75)

1 2 x

5. (valor 1,0)
a) Se (an ) uma progresso geomtrica de termos positivos, prove que (bn ) definida por
bn = log an uma progresso aritmtica. (pontuao parcial 0,5)
b) Se (an ) uma progresso aritmtica, prove que (bn ) definida por bn = ean uma progresso
geomtrica. (pontuao parcial 0,5)
Sociedade Brasileira de Matemtica
Mestrado Profissional em Matemtica em Rede Nacional

MA12 Matemtica Discreta


Avaliao - GABARITO AV 1 - MA 12
27 de abril de 2013

1. (valor 3,0)
Paulo economizou durante muitos anos e tem, hoje, R$ 500.000,00 aplicados em um investi-
mento que rende juros de 1% ao ms. A partir do prximo ms, ele pretende fazer uma retirada
mensal de R$ 1.000,00.
a) Seja sn o saldo que resta da aplicao, aps fazer a n-sima retirada. Exprima sn+1 em
termos de sn . D tambm a condio inicial da recorrncia obtida. (pontuao parcial 0,5)
b) Obtenha uma expresso para sn em funo de n. (pontuao parcial 1,5)
c) Qual a retirada mensal mxima que Paulo pode fazer de modo que o saldo da aplicao
nunca se torne negativo? (pontuao parcial 1,0)

Uma soluo:
a) sn+1 = 1, 01.sn 1 000, com s0 = 500 000 (ou s1 = 504 000).
b) Uma primeira soluo pode ser feita resolvendo-se a recorrncia acima. Uma soluo da
equao homognea associada an+1 = 1, 01.an an = 1, 01n1 .
1 000
Fazendo a substituio sn = an yn , obtemos yn+1 = yn 1,01 n . Usando a recorrncia recem

encontrada e somando os termos, encontramos

1 1
yn = y0 1 000(1 + 1
+ ... + )=
1, 01 1, 01n1
1
1 1,01n 1
= y0 1 000. 1 = y0 100 000.1, 01.(1 )
1 1,01 1, 01n
.
1
Da, sn = an yn = 1, 01n1 y0 100 000.1, 01n .(1 1,01n ) = 1, 01
n1
y0 100 000(1, 01n 1).

1
Finalmente, de s0 = a0 y0 , obtemos 500 000 = y,
1,01 0
ou seja y0 = 500 000.1, 01. Logo, a
expresso de sn

sn = 500 000.1, 01n 100 000(1, 01n 1) = 400 000.1, 01n + 100 000.
Uma segunda soluo desta questo pode ser feita utilizando-se o teorema sobre o valor de
1
11,01 n
uma srie uniforme. O valor das n retiradas, no ms anterior primeira retirada 1 000. 0,01 =
1
100 000(1 1,01n ).
Para obter o valor dessas retiradas no ms da n-sima retirada, devemos multiplicar o valor
no instante inicial por 1, 01n , obtendo 100 000(1, 01n 1). O valor inicial do investimento, nesta
mesma poca, igual a 500 000 1, 01n . Portanto, o saldo restante aps a n-sima retirada

500 000 1, 01n 100 000(1, 01n 1) = 400 000 1, 01n + 100 000.

c) A maior retirada possvel o rendimento mensal, igual a 0, 01 500 000 = 5 000. Podemos
chegar a este resultado resolvendo o item b) para uma retirada genrica p, para a qual obteremos,
aps a n-sima retirada, o valor (50 0000 100p) 1, 01n + 100 000.
Para que este valor nunca se torne negativo, devemos ter 500 000 100p 0, ou seja,
p 5 000.

2. (valor 2,5)
a) Para que valores de b existe uma progresso geomtrica para a qual a soma dos n primeiros
termos igual a 3n+1 + b, para todo n natural? (pontuao parcial 1,0)
b) Quais so o primeiro termo e a razo dessa progresso? (pontuao parcial 1,5)

Uma soluo:
A soma dos n primeiros termos da progesso geomtrica de primeiro termo a1 e razo q

qn 1 a1 q n a1
Sn = a1 =
q1 q1 q1
a1
Esta expresso deve ser idntica a 3.3n + b. Devemos ter, portanto, q = 3 e q1
= 3. Da,
a1 = 6 e o valor de b a1
q1
= 3.

3. (valor 2,0)
Prove, por induo finita, que

1 1 1 1 1 n
1+ + + + + . . . n1 > ,
2 3 4 5 2 2

para todo n natural.

Uma soluo:
Seja P (n) a sentena 1 + 12 + 13 + 14 + . . . 2n1
1
> n2 .
1
P (1) verdadeira, j que 211
= 1 > 12 .
Suponhamos P (n) verdadeira para algum n > 1, ou seja, 1 + 12 + 13 + 14 + . . . 2n1
1
> n2 , n > 1.
n1
Da, 1+ 12 + 31 + 14 + + 2n1
1
+ 2n11 +1 + + 21n > n2 + 2n11 +1 +. . . 21n > n2 + 2 2n = n2 + 12 = n+1
2
.
Logo, P (n + 1) tambm verdadeira. Portanto, pelo Princpio da Induo Finita, P (n)
verdadeira para todo n natural.

4. (valor 1,5)
Na figura abaixo temos uma espiral formada por infinitos semicrculos cujos centros pertencem
ao eixo das abscissas. Se o raio do primeiro semicrculo (o maior) igual a 1 e o raio de cada
semicrculo igual metade do semicrculo anterior, determine:
a) o comprimento total da espiral. (pontuao parcial 0,75)
b) a abscissa do ponto P assinttico da espiral. (pontuao parcial 0,75)

1 2 x
Uma soluo:

a) O comprimento total da espiral .1 + . 21 + . 41 + = 11 1 = 2


2

1 1 1
b) A abscissa do ponto P 2 1 + + ... = 2
2 4 1(( 12 ))
= 43 .

5. (valor 1,0)
a) Se (an ) uma progresso geomtrica de termos positivos, prove que (bn ) definida por
bn = log an uma progresso aritmtica. (pontuao parcial 0,5)
b) Se (an ) uma progresso aritmtica, prove que (bn ) definida por bn = ean uma progresso
geomtrica. (pontuao parcial 0,5)

Uma soluo:
a) Como bn+1 bn = log an+1 log an = log an+1an
= log q = constante, sendo q a razo da
progresso geomtrica (an ), ento (bn ) uma progresso aritmtica.
an+1
b) Como bn+1
bn
= e ean = ean+1 an = er = constante, sendo r a razo da progresso aritmtica
(an ), ento (bn ) uma progresso geomtrica.
Sociedade Brasileira de Matemtica
Mestrado Profissional em Matemtica em Rede Nacional

MA12 Matemtica Discreta


Avaliao Recuperao - MA 12
23 de novembro de 2013

1. (valor 2,0)
Um comerciante, para quem o dinheiro vale 2% ao ms, oferece determinado produto por
trs prestaes mensais iguais a R$ 200,00, a primeira paga no ato da compra.

a) Que valor o comerciante deve cobrar por este produto, no caso de pagamento vista?
(0,5)

b) Se um consumidor desejar pagar o produto em 3 prestaes mensais iguais, sendo a primeira


paga um ms aps a compra, qual deve ser o valor das parcelas? (1,5)

(Utilize, se desejar, os seguintes valores para as potncias de 1, 02: 1, 022 = 1, 0404, 1, 023 =
1, 0612, 1, 021 = 0, 9804, 1, 022 = 0, 9612, 1, 023 = 0, 9423).

2. (valor 2,0)
A soma dos n primeiros termos de uma progresso aritmtica dada por Sn = 3n2 20n.

a) Qual a razo da progresso? (1,0)

b) Qual o ltimo termo negativo da progresso? (1,0)

3. (valor 2,0)
Demonstre, por induo finita, a desigualdade de Bernoulli: se h 1, ento

(1 + h)n 1 + nh,
para todo natural n.
4. (valor 2,0)
Quantos so os nmeros de 4 algarismos:

a) em que todos os algarismos so mpares? (0,5)

a) em que todos os algarismos so mpares e, alm disso, aparecem em ordem no decrescente?


(Por exemplo, 1337 e 1579 so dois destes nmeros, mas 1759 e 1227 no.)(1,5)

5. (valor 2,0)
Em um armazm, h um certo nmero de caixas de um determinado produto, todas adquiridas
do fabricante em uma mesma data. Foi recebido um comunicado do fabricante, avisando que 20%
das caixas expedidas naquela data apresentavam problemas: nelas, 40% das unidades do produto
estavam defeituosas, contra 5% nas caixas normais. Uma das caixas do depsito, escolhida ao
acaso, foi aberta para ser examinada. Uma unidade foi escolhida, tambm ao acaso, desta caixa.

a) Qual a probabilidade de que esta unidade seja defeituosa? (0,5)

b) Dado que a unidade defeituosa, qual a probabilidade de que a caixa examinada seja
uma das caixas com problemas? (0,75)

c) Dado que a unidade no defeituosa, qual a probabilidade de que a caixa examinada seja
uma das caixas com problemas? (0,75)
Sociedade Brasileira de Matemtica
Mestrado Profissional em Matemtica em Rede Nacional

MA12 Matemtica Discreta


Avaliao Recuperao - MA 12
23 de novembro de 2013

1. (valor 2,0)
Um comerciante, para quem o dinheiro vale 2% ao ms, oferece determinado produto por
trs prestaes mensais iguais a R$ 200,00, a primeira paga no ato da compra.

a) Que valor o comerciante deve cobrar por este produto, no caso de pagamento vista?
(0,5)

b) Se um consumidor desejar pagar o produto em 3 prestaes mensais iguais, sendo a primeira


paga um ms aps a compra, qual deve ser o valor das parcelas? (1,5)

(Utilize, se desejar, os seguintes valores para as potncias de 1, 02: 1, 022 = 1, 0404, 1, 023 =
1, 0612, 1, 021 = 0, 9804, 1, 022 = 0, 9612, 1, 023 = 0, 9423).

2. (valor 2,0)
A soma dos n primeiros termos de uma progresso aritmtica dada por Sn = 3n2 20n.

a) Qual a razo da progresso? (1,0)

b) Qual o ltimo termo negativo da progresso? (1,0)

3. (valor 2,0)
Demonstre, por induo finita, a desigualdade de Bernoulli: se h 1, ento

(1 + h)n 1 + nh,
para todo natural n.
4. (valor 2,0)
Quantos so os nmeros de 4 algarismos:

a) em que todos os algarismos so mpares? (0,5)

a) em que todos os algarismos so mpares e, alm disso, aparecem em ordem no decrescente?


(Por exemplo, 1337 e 1579 so dois destes nmeros, mas 1759 e 1227 no.)(1,5)

5. (valor 2,0)
Em um armazm, h um certo nmero de caixas de um determinado produto, todas adquiridas
do fabricante em uma mesma data. Foi recebido um comunicado do fabricante, avisando que 20%
das caixas expedidas naquela data apresentavam problemas: nelas, 40% das unidades do produto
estavam defeituosas, contra 5% nas caixas normais. Uma das caixas do depsito, escolhida ao
acaso, foi aberta para ser examinada. Uma unidade foi escolhida, tambm ao acaso, desta caixa.

a) Qual a probabilidade de que esta unidade seja defeituosa? (0,5)

b) Dado que a unidade defeituosa, qual a probabilidade de que a caixa examinada seja
uma das caixas com problemas? (0,75)

c) Dado que a unidade no defeituosa, qual a probabilidade de que a caixa examinada seja
uma das caixas com problemas? (0,75)
MA12 Matematica Discreta AV1 2014

Questao 1 [ 2,0 pt ]

(a) Defina progressao geometrica de primeiro termo a e razao q (q 6= 0 e q 6= 1).

(b) Conjecture uma formula para o termo geral an em funcao de a, n e q. Em seguida, prove essa formula por
inducao em n.

(c) Se Sn = a1 + a2 + + an , conjecture uma formula para Sn em funcao de a, n e q. Em seguida, prove essa


formula por inducao em n.

(d) A partir dos itens (b) e (c), obtenha uma formula para Sn em funcao a, an e q.

Questao 2 [ 2,0 pt ]

Um comerciante contraiu um emprestimo de R$ 8000,00 a juros semestrais de 10%. O pagamento foi realizado em
duas parcelas, uma de R$ 5808,00 apos um ano da contratacao do emprestimo e a outra seis meses apos a primeira.

(a) Calcule o valor da segunda parcela do emprestimo.

(b) Caso o comerciante optasse por quitar a dvida em 3 parcelas semestrais fixas, a primeira a partir do 1o semestre
apos a contratacao do emprestimo, qual seria o valor das parcelas?

Questao 3 [ 2,0 pt ]

Resolva a equacao de recorrencia Tn = 4Tn1 + 2n , T0 = 9.

Questao 4 [ 2,0 pt ]

Considere a sequencia a1 = 2, a2 = 3 e an = an1 + an2 , para n 3.


Prove, por inducao em n, que:
 n
8
(a) an > , para todo n 1.
5
 n
17
(b) an < , para todo n 4.
10

Questao 5 [ 2,0 pt ]

Quantas solucoes inteiras e positivas (x > 0 e y > 0) possui a equacao 2x + 3y = 2014?


MA12 Matematica Discreta AV1 2014

Questao 1 [ 2,0 pt ]

(a) Defina progressao geometrica de primeiro termo a e razao q (q 6= 0 e q 6= 1).

(b) Conjecture uma formula para o termo geral an em funcao de a, n e q. Em seguida, prove essa formula por
inducao em n.

(c) Se Sn = a1 + a2 + + an , conjecture uma formula para Sn em funcao de a, n e q. Em seguida, prove essa


formula por inducao em n.

(d) A partir dos itens (b) e (c), obtenha uma formula para Sn em funcao a, an e q.

Solucao

(a) Uma progressao geometrica com primeiro termo a e razao q(q 6= 0 e q 6= 1) e uma sequencia de numeros cujo primeiro
termo e a e tal que, cada elemento, a partir do segundo, e igual ao anterior multiplicado pela razao.
Em smbolos, a1 = a e an = an1 q, se n 2.

(b) Calculemos alguns termos pela definicao:


a2 = a1 q = aq, a3 = a2 q = aq q = aq 2 , a4 = a3 q = aq 2 q = aq 3 .
A partir destes calculos, conjecturamos que an = aq n1 , para todo n 1.
Vamos provar esta conjectura por inducao em n.
Para n = 1 e claramente valida, pois a1 = a = aq 0 = aq 11 .
Agora suponhamos que o resultado e valido para um certo n = k 1, ou seja, ak = aq k1 .
Para n = k + 1 segue que ak+1 = ak+11 q = ak q = aq k1 q = aq k e portanto esta provada a conjectura.

(c) Temos que Sn = a1 + a2 + a3 + + an1 + an e usando o resultado acima podemos reescrever a soma
Sn = a + aq + aq 2 + + aq n2 + aq n1 . Multiplicando ambos os lados por q segue que
qSn = aq + aq 2 + aq 3 + aq n1 + aq n .
Logo temos que Sn qSn = a aq n , ou seja, (1 q)Sn = q(1 q n ) e podemos conjecturar que
a(1 q n )
Sn = , para todo n 1.
1q
Vamos provar este resultado por inducao em n.
a(1 q)
Para n = 1 e facil ver que S1 = a1 = a = .
1q
Agora suponhamos que o resultado vale para um certo n = k.
Para n = k + 1 temos que
a(1 q k ) a aq k + aq k aq k+1
Sk+1 = a1 + + ak + ak+1 = + aq k = =
1q 1q
a(1 q k+1 )
e portanto esta provada a conjectura.
1q
a(1 q n )
(d) Pelo item (b) temos que an = aq n1 e pelo item (c) Sn = .
1q
Logo segue que
a(1 q n ) a aq n a aq n1 q a an q
Sn = = = = .
1q 1q 1q 1q
Questao 2 [ 2,0 pt ]

Um comerciante contraiu um emprestimo de R$ 8000,00 a juros semestrais de 10%. O pagamento foi realizado em
duas parcelas, uma de R$ 5808,00 apos um ano da contratacao do emprestimo e a outra seis meses apos a primeira.

(a) Calcule o valor da segunda parcela do emprestimo.

(b) Caso o comerciante optasse por quitar a dvida em 3 parcelas semestrais fixas, a primeira a partir do 1o semestre
apos a contratacao do emprestimo, qual seria o valor das parcelas?

Solucao

(a) Considerando os esquemas de pagamentos da figura, ambos sao equivalentes, ou seja, R$ 8000,00, na data zero, tem o
mesmo valor de R$ 5808,00 dois semestres apos (data 2), mais um pagamento P , na data 3.

Igualando os valores, na epoca zero, dos pagamentos em ambos os esquemas, obtemos:


5808 P 5808 P P
8000 = + 8000 = + 8000 = 4800 +
1, 12 1, 13 1, 21 1, 331 1, 331
P
3200 = P = 4259, 20
1, 331
(b) Considerando os esquemas equivalentes de pagamentos da figura, o pagamento de R$ 8000,00 na data zero tem o mesmo
valor de 3 parcelas semestrais P , a partir do primeiro semestre apos a contratacao.

Igualando os valores dos pagamentos de ambos os esquemas, na data zero, obtemos:


P P P 1, 21P + 1, 1P + P
8000 = + + 8000 =
1, 1 1, 12 1, 13 1, 331
10648
10648 = 3, 31P P = P = 3216, 92
3, 31

Questao 3 [ 2,0 pt ]

Resolva a equacao de recorrencia Tn = 4Tn1 + 2n , T0 = 9.


Solucao
Uma solucao nao-nula de Tn = 4Tn1 e, por exemplo, Tn = 4n . Agora fazemos a substituicao Tn = 4n yn e obtemos
4n yn = 4 4n1 yn1 + 2n , ou seja, yn = yn1 + 2n , para n 1.
Da temos que

y1 = y0 + 21
y2 = y1 + 22
y3 = y2 + 23
......
yn = yn1 + 2n .

Somando as parcelas da esquerda e da direita, das igualdades acima, temos

y1 + y2 + y3 + + yn = y0 + 21 + y1 + 22 + y2 + 23 + + yn1 + 2n .

Cancelando os termos obtemos


21 1 (21 )n
 
1 2 3 n
yn = y0 + 2 +2 +2 + + 2 = y0 + = y0 + 1 2n .
1 21
Como Tn = 4n yn e T0 = 9, temos y0 = 9 e yn = 10 2n . Portanto,

Tn = 4n (10 2n ) = 10 4n 2n .

Questao 4 [ 2,0 pt ]

Considere a sequencia a1 = 2, a2 = 3 e an = an1 + an2 , para n 3. Prove, por inducao em n, que:


 n
8
(a) an > , para todo n 1.
5
 n
17
(b) an < , para todo n 4.
10

Solucao
8 n

(a) Seja P (n) a proposicao: an > 5
, para todo n 1.

Para n = 1 temos que a1 = 2 > 58 .


8 n

Suponhamos agora que P (n) e verdadeira ate n = k, ou seja, an > 5
, para n = 1, . . . , k. Devemos provar que P (n)
continua valida para n = k + 1. De fato,
 k  k1  k    k  k  k+1
8 8 8 5 8 13 8 8 8
ak+1 = ak + ak1 > + = 1+ == > = .
5 5 5 8 5 8 5 5 5

e assim P (k + 1) e verdadeira.

17 n

(b) Agora seja Q(n) a proposicao: an < 10
,para todo n 4.
83521 4
= 17

Para n = 4 temos que a4 = 8 < 1000 10
.
17 n

Suponhamos agora que Q(n) e verdadeira ate n = k, ou seja, an < 10
, para n = 4, . . . , k.
Devemos provar que Q(n) continua valida para n = k + 1. De fato,
 k  k1  k    k  k  k+1
17 17 17 10 17 27 17 17 17
ak+1 = ak + ak1 < + = 1+ = < = .
10 10 10 17 10 17 10 10 10
e assim Q(k + 1) e verdadeira.
Questao 5 [ 2,0 pt ]

Quantas solucoes inteiras e positivas (x > 0 e y > 0) possui a equacao 2x + 3y = 2014?

Solucao
Inicialmente observemos que se (x0 , y0 ) e uma solucao inteira (x0 , y0 Z) da equacao 2x+3y = 2014, entao (x0 3t, y0 +2t), t
Z, sao todas as solucoes inteiras da mesma. Com isso e suficiente obter uma solucao inteira.
E facil ver que (1007, 0) e uma solucao. Entao as solucoes inteiras sao dadas por x = 1007 3t, y = 2t, t Z. Como
desejamos apenas solucoes positivas (x > 0 e y > 0), devemos ter 1007 3t > 0 e 2t > 0. Resolvendo ambas as inequacoes
1007 1007
obtemos 0 < t < . Mas como = 335, 66... e t deve ser inteiro segue que 1 t 335.
3 3
Portanto a equacao 2x + 3y = 2014 possui 335 solucoes inteiras positivas, a saber x = 1007 3t, y = 2t, 1 t 335.

Outra solucao

E facil que uma solucao da equacao e x0 = 1007 e y0 = 0, mas essa solucao nao e positiva. A partir dela podemos criar
varias solucoes x = 1007 3n, y = 2n, com n inteiro positivo, que formam progressoes aritmeticas de razao 3 e 2 em x e y,
respectivamente.
Como desejamos que x tambem seja positivo devemos ter 10073n > 0, ou seja, n 335. Portanto a equacao 2x+3y = 2014
possui 335 solucoes inteiras positivas, a saber x = 1007 3n, y = 2n, 1 n 335.
MA12 Matematica Discreta AV1 2014

Questao 1 [ 2,0 pt ]

(a) Defina progressao geometrica de primeiro termo a e razao q (q 6= 0 e q 6= 1).

(b) Conjecture uma formula para o termo geral an em funcao de a, n e q. Em seguida, prove essa formula por
inducao em n.

(c) Se Sn = a1 + a2 + + an , conjecture uma formula para Sn em funcao de a, n e q. Em seguida, prove essa


formula por inducao em n.

(d) A partir dos itens (b) e (c), obtenha uma formula para Sn em funcao a, an e q.

Solucao

(a) Uma progressao geometrica com primeiro termo a e razao q(q 6= 0 e q 6= 1) e uma sequencia de numeros cujo primeiro
termo e a e tal que, cada elemento, a partir do segundo, e igual ao anterior multiplicado pela razao.
Em smbolos, a1 = a e an = an1 q, se n 2.

(b) Calculemos alguns termos pela definicao:


a2 = a1 q = aq, a3 = a2 q = aq q = aq 2 , a4 = a3 q = aq 2 q = aq 3 .
A partir destes calculos, conjecturamos que an = aq n1 , para todo n 1.
Vamos provar esta conjectura por inducao em n.
Para n = 1 e claramente valida, pois a1 = a = aq 0 = aq 11 .
Agora suponhamos que o resultado e valido para um certo n = k 1, ou seja, ak = aq k1 .
Para n = k + 1 segue que ak+1 = ak+11 q = ak q = aq k1 q = aq k e portanto esta provada a conjectura.

(c) Temos que Sn = a1 + a2 + a3 + + an1 + an e usando o resultado acima podemos reescrever a soma
Sn = a + aq + aq 2 + + aq n2 + aq n1 . Multiplicando ambos os lados por q segue que
qSn = aq + aq 2 + aq 3 + aq n1 + aq n .
Logo temos que Sn qSn = a aq n , ou seja, (1 q)Sn = q(1 q n ) e podemos conjecturar que
a(1 q n )
Sn = , para todo n 1.
1q
Vamos provar este resultado por inducao em n.
a(1 q)
Para n = 1 e facil ver que S1 = a1 = a = .
1q
Agora suponhamos que o resultado vale para um certo n = k.
Para n = k + 1 temos que
a(1 q k ) a aq k + aq k aq k+1
Sk+1 = a1 + + ak + ak+1 = + aq k = =
1q 1q
a(1 q k+1 )
e portanto esta provada a conjectura.
1q
a(1 q n )
(d) Pelo item (b) temos que an = aq n1 e pelo item (c) Sn = .
1q
Logo segue que
a(1 q n ) a aq n a aq n1 q a an q
Sn = = = = .
1q 1q 1q 1q
Questao 2 [ 2,0 pt ]

Um comerciante contraiu um emprestimo de R$ 8000,00 a juros semestrais de 10%. O pagamento foi realizado em
duas parcelas, uma de R$ 5808,00 apos um ano da contratacao do emprestimo e a outra seis meses apos a primeira.

(a) Calcule o valor da segunda parcela do emprestimo.

(b) Caso o comerciante optasse por quitar a dvida em 3 parcelas semestrais fixas, a primeira a partir do 1o semestre
apos a contratacao do emprestimo, qual seria o valor das parcelas?

Solucao

(a) Considerando os esquemas de pagamentos da figura, ambos sao equivalentes, ou seja, R$ 8000,00, na data zero, tem o
mesmo valor de R$ 5808,00 dois semestres apos (data 2), mais um pagamento P , na data 3.

Igualando os valores, na epoca zero, dos pagamentos em ambos os esquemas, obtemos:


5808 P 5808 P P
8000 = + 8000 = + 8000 = 4800 +
1, 12 1, 13 1, 21 1, 331 1, 331
P
3200 = P = 4259, 20
1, 331
(b) Considerando os esquemas equivalentes de pagamentos da figura, o pagamento de R$ 8000,00 na data zero tem o mesmo
valor de 3 parcelas semestrais P , a partir do primeiro semestre apos a contratacao.

Igualando os valores dos pagamentos de ambos os esquemas, na data zero, obtemos:


P P P 1, 21P + 1, 1P + P
8000 = + + 8000 =
1, 1 1, 12 1, 13 1, 331
10648
10648 = 3, 31P P = P = 3216, 92
3, 31

Questao 3 [ 2,0 pt ]

Resolva a equacao de recorrencia Tn = 4Tn1 + 2n , T0 = 9.


Solucao
Uma solucao nao-nula de Tn = 4Tn1 e, por exemplo, Tn = 4n . Agora fazemos a substituicao Tn = 4n yn e obtemos
4n yn = 4 4n1 yn1 + 2n , ou seja, yn = yn1 + 2n , para n 1.
Da temos que

y1 = y0 + 21
y2 = y1 + 22
y3 = y2 + 23
......
yn = yn1 + 2n .

Somando as parcelas da esquerda e da direita, das igualdades acima, temos

y1 + y2 + y3 + + yn = y0 + 21 + y1 + 22 + y2 + 23 + + yn1 + 2n .

Cancelando os termos obtemos


21 1 (21 )n
 
1 2 3 n
yn = y0 + 2 +2 +2 + + 2 = y0 + = y0 + 1 2n .
1 21
Como Tn = 4n yn e T0 = 9, temos y0 = 9 e yn = 10 2n . Portanto,

Tn = 4n (10 2n ) = 10 4n 2n .

Questao 4 [ 2,0 pt ]

Considere a sequencia a1 = 2, a2 = 3 e an = an1 + an2 , para n 3. Prove, por inducao em n, que:


 n
8
(a) an > , para todo n 1.
5
 n
17
(b) an < , para todo n 4.
10

Solucao
8 n

(a) Seja P (n) a proposicao: an > 5
, para todo n 1.
2
Para n = 1 temos que a1 = 2 > 58 . Alem disso, para n = 2, temos que a2 = 3 = 25
75
> 64
25
= 85 .
n
Suponhamos agora que P (n) e verdadeira ate n = k 2, ou seja, an > 58 , para n = 1, . . . , k. Devemos provar que P (n)
continua valida para n = k + 1. De fato,
 k  k1  k    k  k  k+1
8 8 8 5 8 13 8 8 8
ak+1 = ak + ak1 > + = 1+ = > = .
5 5 5 8 5 8 5 5 5

e assim P (k + 1) e verdadeira.

17 n

(b) Agora seja Q(n) a proposicao: an < para todo n 4.
10
,
83521 17 4 1419857 17 5
 
Para n = 4 temos que a4 = 8 < 10000 = 10 . De modo analogo, para n = 5 temos que a5 = 13 < 100000
= 10
.
n
Suponhamos agora que Q(n) e verdadeira ate n = k 5, ou seja, an < 17
10
, para n = 4, . . . , k.
Devemos provar que Q(n) continua valida para n = k + 1. De fato,
 k  k1  k    k  k  k+1
17 17 17 10 17 27 17 17 17
ak+1 = ak + ak1 < + = 1+ = < = .
10 10 10 17 10 17 10 10 10
e assim Q(k + 1) e verdadeira.
Questao 5 [ 2,0 pt ]

Quantas solucoes inteiras e positivas (x > 0 e y > 0) possui a equacao 2x + 3y = 2014?

Solucao
Inicialmente observemos que se (x0 , y0 ) e uma solucao inteira (x0 , y0 Z) da equacao 2x+3y = 2014, entao (x0 3t, y0 +2t), t
Z, sao todas as solucoes inteiras da mesma. Com isso e suficiente obter uma solucao inteira.
E facil ver que (1007, 0) e uma solucao. Entao as solucoes inteiras sao dadas por x = 1007 3t, y = 2t, t Z. Como
desejamos apenas solucoes positivas (x > 0 e y > 0), devemos ter 1007 3t > 0 e 2t > 0. Resolvendo ambas as inequacoes
1007 1007
obtemos 0 < t < . Mas como = 335, 66... e t deve ser inteiro segue que 1 t 335.
3 3
Portanto a equacao 2x + 3y = 2014 possui 335 solucoes inteiras positivas, a saber x = 1007 3t, y = 2t, 1 t 335.

Outra solucao

E facil que uma solucao da equacao e x0 = 1007 e y0 = 0, mas essa solucao nao e positiva. A partir dela podemos criar
varias solucoes x = 1007 3n, y = 2n, com n inteiro positivo, que formam progressoes aritmeticas de razao 3 e 2 em x e y,
respectivamente.
Como desejamos que x tambem seja positivo devemos ter 10073n > 0, ou seja, n 335. Portanto a equacao 2x+3y = 2014
possui 335 solucoes inteiras positivas, a saber x = 1007 3n, y = 2n, 1 n 335.
AVALIAES
AV-02
MA12 Matematica Discreta Prova 2 2011

Questao 1.
Considere os caminhos no plano iniciados no ponto (0, 0) com deslocamentos paralelos aos eixos coordenados,
sempre de uma unidade e no sentido positivo dos eixos x e y (nao se descarta a possibilidade de dois movimentos
unitarios seguidos na mesma direcao, ver ilustracao mostrando um caminho que termina em (5, 4)).
y
4

x
5

m
(1,0) (a) Explique por que o numero de caminhos que terminam no ponto (m, n) e Cm+n .

(1,0) (b) Quantos sao os caminhos que terminam no ponto (8, 7), passam por (2, 3) mas nao passam por (5, 4)?

Questao 2.
Os professores de seis disciplinas (entre as quais Portugues e Matematica) devem escolher um dia, de segunda a
sexta, de uma unica semana para a realizacao da prova de sua disciplina. Suponha que cada professor escolha o seu
dia de prova ao acaso, sem combinar com os demais professores.

(1,0) (a) Qual e a probabilidade de que as provas de Portugues e Matematica sejam realizada no mesmo dia?

(1,0) (b) Qual e a probabilidade de que os alunos facam provas em todos os dias da semana?

Questao 3.
Em um jogo, uma moeda honesta e jogada seguidamente. Cada vez que sai cara, o jogador ganha 1 real; cada vez
que sai coroa, o jogador ganha 2 reais. O jogo termina quando o jogador tiver acumulado 4 ou mais reais.

(0,5) (a) Qual e a probabilidade de que o jogador ganhe exatamente 4 reais?

(0,5) (b) Qual e a probabilidade de que no ultimo lancamento saia cara?

(1,0) (c) Dado que o jogador ganhou exatamente 4 reais, qual e a probabilidade de que tenha sado cara no ultimo
lancamento?
MA12 Matematica Discreta Prova 2 2011

Questao 4.
Uma prova de concurso e formada por questoes de multipla escolha, com 4 alternativas por questao. Admita que
nenhum candidato deixe questoes sem responder.

(1,0) (a) Qual e o numero mnimo de candidatos para que seja possvel garantir que pelo menos 3 deles darao exatamente
as mesmas respostas nas 5 primeiras questoes?

(1,0) (b) Qual e o valor maximo de n para o qual e possvel garantir que, em um concurso com 1000 candidatos, pelo
menos 2 darao as mesmas respostas nas primeiras n questoes?

Questao 5.
Uma caixa retangular sem tampa tem arestas medindo x, y e z (veja figura, onde as linhas tracejadas indicam
segmentos de arestas obstrudos por alguma face).

(0,5) (a) Exprima a area e o volume da caixa em funcao de x, y e z.

(1,0) (b) Use a desigualdade das medias para mostrar que, se o volume da caixa e igual a 32, entao sua area e maior ou
igual a 48.

(0,5) (c) Determine as medidas das arestas da caixa de area mnima com volume igual a 32.

z
y
x
AV2 - MA 12 - 2011

Questo 1.
Considere os caminhos no plano iniciados no ponto (0, 0) com deslocamentos paralelos aos eixos coordenados,
sempre de uma unidade e no sentido positivo dos eixos x e y (no se descarta a possibilidade de dois movimentos
unitrios seguidos na mesma direo, ver ilustrao mostrando um caminho que termina em (5, 4)).
y
4

x
5

m .
(1,0) (a) Explique por que o nmero de caminhos que terminam no ponto (m, n) Cm +n

(1,0) (b) Quantos so os caminhos que terminam no ponto (8, 7), passam por (2, 3) mas no passam por (5, 4)?

UMA SOLUO

(a) Chamaremos de horizontais os movimentos paralelos ao eixo x e de verticais os paralelos ao eixo y. Como todos
os movimentos so positivos e unitrios, so necessrios m movimentos horizontais e n movimentos verticais para
se chegar em (m, n), totalizando m + n movimentos. Um caminho fica totalmente determinado se dissermos quais
desses m + n movimentos so, digamos, movimentos horizontais. Portanto, precisamos saber de quantas maneiras
m .
podemos escolher m movimentos horizontais entre os m + n movimentos do caminho. Isso d Cm +n
Evidentemente poderamos ter determinado os caminhos dizendo quais so os n movimentos horizontais dentre
n
os m + n movimentos. Esse raciocnio nos levaria a Cm n m
+n . Mas Cm+n = Cm+n .

(b) Se um caminho at (8, 7) obrigado a passar por (2, 3) ento ele a juno de um caminho que vai de (0, 0) a
(2, 3) com um caminho que vai de (2, 3) a (8, 7). No entanto, queremos que o caminho que vai de (2, 3) a (8, 7) no
passe por (5, 4), ou seja, queremos que ele v de (2, 3) a (8, 7) sem ser a juno de um caminho de (2, 3) a (5, 4) com
um caminho de (5, 4) a (8, 7). Isso nos indica que precisamos calcular quantos caminhos temos de (0, 0) a (2, 3),
quantos de (2, 3) a (5, 4) e quantos de (5, 4) a (8, 7).
Segundo o item anterior, h C22+3 = C52 maneiras de ir de (0, 0) a (2, 3). H C33+1 = C43 maneiras de se ir de (2, 3) a
(5, 4), pois so necessrios 3 movimentos horizontais e 1 vertical. H C33+3 = C63 maneiras de se ir de (5, 4) a (8, 7),
pois so necessrios 3 movimentos horizontais e 3 verticais. E h C10 6 maneiras de se ir de (2, 3) a (8, 7), pois so

necessrios 6 movimentos horizontais e 4 verticais.


H, portanto, C43 C63 maneiras de se ir de (2, 3) a (8, 7) passando por (5, 4). Ento h C10
6 C3 C3 maneiras de se
4 6
ir de (2, 3) a (8, 7) sem passar por (5, 4). E, por conseguinte, h

N = C52 (C10
6
C43 C63 )

1
maneiras de se ir de (0, 0) a (8, 7) passando por (2, 3) mas no passando por (5, 4).
5! 10! 10987
Para termos um nmero, calculamos essas combinaes: C52 = 3! 2!
6 =
= 10, C10 4!6! = 432 = 210, C43 = 4 e
6! 654
C63 = 3!3! = 32 = 20. Ento
N = 10 (210 4 20) = 1300 .

2
AV2 - MA 12 - 2011

Questo 2.
Os professores de seis disciplinas (entre as quais Portugus e Matemtica) devem escolher um dia, de segunda
a sexta, de uma nica semana para a realizao da prova de sua disciplina. Suponha que cada professor escolha o
seu dia de prova ao acaso, sem combinar com os demais professores.

(1,0) (a) Qual a probabilidade de que as provas de Portugus e Matemtica sejam realizadas no mesmo dia?

(1,0) (b) Qual a probabilidade de que os alunos faam provas em todos os dias da semana?

UMA SOLUO

(a) Nesta questo, no preciso olhar para as outras disciplinas. H 5 possibilidades para o dia de prova de Por-
tugus e 5 possibilidades para o dia de prova de Matemtica. Portanto, h 25 possibilidades para o par de provas
Portugus e Matemtica. Dessas 25, apenas 5 so ocorrncias de Portugus e Matemtica no mesmo dia (uma
ocorrncia para cada dia da semana). Ento a probabilidade de que essas duas provas ocorram no mesmo dia
5/25 = 0, 2 (ou 20%).
Outra maneira de pensar: fixado o dia da prova de Matemtica, h 5 possibilidades para o dia de Portugus, e
apenas uma delas no mesmo dia que Matemtica. Isso d os mesmos 20% de chances.

(b) Vamos contar de quantas maneiras se distribuem 6 provas nos 5 dias da semana sem deixar um dia livre. Com
essa imposio, certamente um dia ter duas provas e os demais dias tero apenas uma. Ento comeamos esco-
lhendo entre as 5 possibilidades para o dia da semana que ter duas provas. Escolhido esse dia, temos que escolher
duas das seis disciplinas para preench-lo. Temos C62 escolhas. Escolhidas essas duas disciplinas, ainda restam 4
para distribuir nos 4 dias: so 4! escolhas. Portanto h 5 C62 4! maneiras de se distribuir 6 provas em 5 dias sem
deixar um dia livre.
Agora precisamos do total de maneiras de se distribuir as 6 provas durante a semana. Cada disciplina tem 5
escolhas, ento so 56 possibilidades.
Ento a probabilidade de no ficar um dia livre o quociente

5 C62 4! 5! 15 72 16 1152
6
= 6
= 4! 354 = = = 0, 1152 ,
5 5 10000 10000
ou 11,52%.

3
AV2 - MA 12 - 2011

Questo 3.
Em um jogo, uma moeda honesta jogada seguidamente. Cada vez que sai cara, o jogador ganha 1 real; cada vez
que sai coroa, o jogador ganha 2 reais. O jogo termina quando o jogador tiver acumulado 4 ou mais reais.

(0,5) (a) Qual a probabilidade de que o jogador ganhe exatamente 4 reais?

(0,5) (b) Qual a probabilidade de que no ltimo lanamento saia cara?

(1,0) (c) Dado que o jogador ganhou exatamente 4 reais, qual a probabilidade de que tenha sado cara no ltimo
lanamento?

UMA SOLUO

Nesta questo, convm fazer primeiro a rvore das possibilidades. Indicaremos cara por A e coroa por B. Em
cada n da rvore, indicamos a sequncia obtida (linha superior), o valor acumulado pelo jogador (linha do meio)
e a probabilidade daquela sequncia (linha inferior). Os ns em cinza so aqueles em que o jogo termina. Veja que
a soma das probabilidades em cada n pintado em cinza igual a 1, e que a probabilidade indicada em cada um
a probabilidade de o jogo terminar com aquela sequncia.
A B
1 2
1/2 1/2

AA AB BA BB
2 3 3 4
1/4 1/4 1/4 1/4

AAA AAB ABA ABB BAA BAB


3 4 4 5 4 5
1/8 1/8 1/8 1/8 1/8 1/8

AAAA AAAB
4 5
1/16 1/16
(a) Para ver a probabilidade de que o jogador termine com exatamente 4 reais, basta somar as probabilidades dos
ns em cinza que tm ganho de 4 reais. So eles: AAAA (1/16), AAB (1/8), ABA (1/8), BAA (1/8) e BB (1/4). A
11
soma 16 .

(b) O jogo termina com cara em todos os ns em cinza que terminam com a letra A. Ento basta somar as probabili-
5
dades de cada caso. So eles AAAA (1/16), ABA (1/8) e BAA (1/8), o que d 16 .

(c) Das situaes em que o jogador terminou com 4 reais, listadas em (a), que tm probabilidade de 11/16 de
ocorrer, apenas AAAA, ABA e BAA terminam com A (cara), com probabilidade de 5/16. Ento a probabilidade de
5/16 5
se terminar com cara dado que o jogador terminou com 4 reais 11/16 = 11 .

4
AV2 - MA 12 - 2011

Questo 4.
Uma prova de concurso formada por questes de mltipla escolha, com 4 alternativas por questo. Admita que
nenhum candidato deixe questes sem responder.

(1,0) (a) Qual o nmero mnimo de candidatos para que seja possvel garantir que pelo menos 3 deles daro exata-
mente as mesmas respostas nas 5 primeiras questes?

(1,0) (b) Qual o valor mximo de n para o qual possvel garantir que, em um concurso com 1000 candidatos, pelo
menos 2 daro as mesmas respostas nas primeiras n questes?

UMA SOLUO

(a) O conjunto de possibilidades de respostas para as 5 primeiras questes, cada uma com 4 alternativas, 45 .
possvel distribuir as respostas de 2 45 = 2048 candidatos de forma que cada conjunto de respostas se repita
exatamente duas vezes, mas se houver 2 45 + 1 = 2049 candidatos isso no mais possvel, sempre haver ao
menos 3 provas iguais nas cinco primeiras questes.

(b) Considerando agora as n primeiras questes, h 4n possibilidades de resposta. Para garantir que em 1000 can-
didatos pelo menos 2 respondam de forma igual a essas primeiras n questes, necessrio que 1000 4n + 1, isto
, 4n 999. O valor mximo de n tal que 4n 999 4 (pois 44 = 28 = 256 e 45 = 210 = 1024). Resposta: n = 4.

5
AV2 - MA 12 - 2011

Questo 5.
Uma caixa retangular sem tampa tem arestas medindo x, y e z (veja figura, onde as linhas tracejadas indicam
segmentos de arestas obstrudos por alguma face).

(0,5) (a) Exprima a rea e o volume da caixa em funo de x, y e z.

(1,0) (b) Use a desigualdade das mdias para mostrar que, se o volume da caixa igual a 32, ento sua rea maior ou
igual a 48.

(0,5) (c) Determine as medidas das arestas da caixa de rea mnima com volume igual a 32.

z
y
x

UMA SOLUO

(a) A rea da caixa igual a xy + 2xz + 2yz e seu volume igual a xyz.
(b) A soma xy + 2xz + 2yz igual a 3 vezes a mdia aritmtica simples de seus termos. Essa mdia sempre maior
do que ou igual mdia geomtrica dos mesmos termos, isto

1 p q
( xy + 2xz + 2yz) 3 xy 2xz 2yz = 3 4x2 y2 z2 .
3
p p
3
Supondo xyz = 32 (que dado no problema), resulta que 3 4x2 y2 z2 = 3 22 (25 )2 = 212 = 16. Ento, multipli-
cando por 3 dos dois lados, xy + 2xz + 2yz 48.

(c) A igualdade entre as mdias aritmtica e geomtrica ocorre se, e somente se, os termos so iguais. Neste caso,
quando xy = 2xz = 2yz. Como o volume positivo, x, y, z tm que ser positivos, em particular no nulos. Ento,
da equao 2xz = 2yz tiramos y = x, e da equao xy = 2yz tiramos z = 2x . Como xyz = 32 ento x x x
2 = 32, isto
, x3 = 64 = 26 , ou ainda x = 4. Ento x = y = 4 e z = 2.

6
MA12 Matematica Discreta AV2 2012

Nesta prova, todos os itens tem mesmo peso.

Questao 1.

Num porta-CDs, cabem 10 CDs colocados um sobre o outro, formando uma pilha vertical. Tenho 3 CDs de
MPB, 5 de rock e 2 de musica classica.
(a) De quantos modos diferentes posso empilha-los de modo que todos os CDs de rock fiquem juntos?
(b) De quantos modos posso escolher 4 CDs para levar em uma viagem, de modo que eu leve pelo menos um
CD de cada tipo de musica?

Questao 2.

Em uma caixa ha 10 bolas identicas, numeradas de 1 a 10. O numero de cada bola corresponde a um dos
pontos da figura, os quais dividem a circunferencia em 10 partes iguais. Nos itens a seguir, considere que as
bolas sao retiradas ao acaso, uma a uma e sem reposicao.
(a) Se forem retiradas duas bolas, qual e a probabilidade de que os pontos correspondentes sejam extremidades
de um diametro?
(b) Se forem retiradas tres bolas, qual e a probabilidade de que os pontos correspondentes sejam vertices de
um triangulo isosceles?

1
MA12 Matematica Discreta AV2 2012

Questao 3.

Em uma caixa foram colocados um cartao no qual esta escrito o numero 1, dois cartoes nos quais esta escrito
o numero 2, tres cartoes com o numero 3 e assim por diante, ate dez cartoes com o numero 10.
(a) Qual e o numero mnimo de cartoes que devem ser retirados da caixa, sem olhar, de modo que se tenha
certeza de que haja, entre os cartoes retirados, 5 deles com o mesmo numero?
(b) Qual e o numero mnimo de cartoes que devem ser retirados da caixa, sem olhar, de modo que se tenha
certeza de que haja, entre os cartoes retirados, pelo menos um par de cartoes com diferenca maior do que
5?

Questao 4.

A media aritmetica de 10 numeros positivos e igual a 1. Os numeros sao agrupados aos pares e os numeros de
cada par somados, resultando da um conjunto de 5 numeros positivos.
(a) O que se pode dizer sobre a media aritmetica desses 5 numeros?
(b) Mostre que o produto desses 5 numeros e menor ou igual a 32.

Questao 5.

Uma moeda, com probabilidade 0,6 de dar cara, e lancada duas vezes.
(a) Qual e a probabilidade de que se observem resultados iguais no primeiro e segundo lancamentos?
(b) Dado que os resultados observados no primeiro e segundo lancamentos sao iguais, qual e a probabilidade
condicional de que o resultado observado neles seja cara?

2
AV2 - MA 12 - 2012

Questo 1.

Num porta-CDs, cabem 10 CDs colocados um sobre o outro, formando uma pilha vertical. Tenho 3 CDs de
MPB, 5 de rock e 2 de msica clssica.
(a) De quantos modos diferentes posso empilh-los de modo que todos os CDs de rock fiquem juntos?
(b) De quantos modos posso escolher 4 CDs para levar em uma viagem, de modo que eu leve pelo menos
um CD de cada tipo de msica?

UMA SOLUO

(a) Vamos fixar as posies dos CDs atribuindo nmeros de 1 a 10 a suas posies, contando de baixo para cima.
Se todos os 5 CDs de rock ficam juntos, o primeiro pode ficar nas posies de 1 a 6, portanto so 6 escolhas para a
posio do bloco de CDs de rock. Os 5 CDs de rock podem ser arrumados de 5! = 120 maneiras dentro do bloco. As
posies restantes so 5 e os demais CDs tambm podem ser ordenados de 120 maneiras nessas posies restantes
(no importa que o bloco de CDs de rock interrompa a sequncia). Portanto so 6 120 120, isto , 86400 maneiras.

(b) Para escolher 4 CDs com pelo menos um para cada tipo de msica, podemos escolher, primeiro, um de cada
tipo. Temos 3 possibilidades para MPB, 5 para rock e 2 para msica clssica, perfazendo 3 5 2 = 30 possibilidades.
Depois dessa escolha, podemos pegar qualquer um dos 7 CDs restantes. So, portanto, 30 7 = 210 escolhas. No
entanto, temos que dividir por 2 esse valor, j que os dois CDs de mesmo gnero, digamos A e B, podem aparecer
com A na primeira escolha e B na segunda, ou vice-versa.
Outra maneira de resolver (mais complicada, mas que evita a diviso por dois no final): dos 4 CDs, dois so do
mesmo gnero (e os outros dois dos dois outros gneros restantes). Se os dois de gnero repetido forem de msica
clssica, so todos os disponveis para esse gnero, de forma que restam 3 5 escolhas para os outros dois; so,
portanto, 15 possibilidades para se ter 2 CDs repetidos de msica clssica. Se os dois de gnero repetido forem
de MPB, h C3,2 = 3 escolhas para eles; para cada uma delas, restam 2 5 escolhas dos outros dois; portanto, so
30 maneiras para se ter dois CDs de MPB. Finalmente, se os dois de gnero repetido forem de rock, h C5,2 = 10
escolhas para os dois repetidos, e 2 3 escolhas para os outros dois, perfazendo 6 10 = 60 possibilidades com dois
CDs de rock. No total, so 60 + 30 + 15 = 105 possibilidades.

1
AV2 - MA 12 - 2012

Questo 2.

Em uma caixa h 10 bolas idnticas, numeradas de 1 a 10. O nmero de cada bola corresponde a um dos
pontos da figura, os quais dividem a circunferncia em 10 partes iguais. Nos itens a seguir, considere que as
bolas so retiradas ao acaso, uma a uma e sem reposio.
(a) Se forem retiradas duas bolas, qual a probabilidade de que os pontos correspondentes sejam extremi-
dades de um dimetro?
(b) Se forem retiradas trs bolas, qual a probabilidade de que os pontos correspondentes sejam vrtices de
um tringulo issceles?

UMA SOLUO

(a) A primeira bola pode ser qualquer uma. Das 9 restantes, apenas uma ser diametralmente oposta a essa
primeira. Portanto a probabilidade de isso ocorrer de 1/9.

(b) Escolhidos 3 pontos da figura, ficam definidos tambm 3 intervalos entre eles. Lados iguais de um tringulo
ocorrem se, e somente se, os correspondentes intervalos entre os pontos so iguais. Em particular, (i) os pontos
formam um tringulo issceles se, e somente se, pelo menos dois desses intervalos so iguais; (ii) nenhum tringulo
equiltero pode ser formado, j que 10 no divisvel por 3.
Nunca havendo 3 intervalos iguais, definimos de forma unvoca o ponto do meio de um tringulo issceles
quele ladeado pelos dois intervalos iguais. H 10 possibilidades para esse ponto do meio. Os intervalos iguais que
ladeiam esse ponto do meio podem ter os tamanhos: 1/10, 2/10, 3/10 e 4/10. Portanto so 10 4 = 40 maneiras de
tomar 3 desses 10 pontos como vrtices de um tringulo issceles.
Por outro lado, h C10,3 = 10 9 8/6 = 120 maneiras de se escolher 3 entre as 10 bolas. Portanto, dessas 120
escolhas, 40 levaro a um tringulo issceles, e a probabilidade de isso ocorrer ser de 40/120 = 1/3.

2
AV2 - MA 12 - 2012

Questo 3.

Em uma caixa foram colocados um carto no qual est escrito o nmero 1, dois cartes nos quais est escrito
o nmero 2, trs cartes com o nmero 3 e assim por diante, at dez cartes com o nmero 10.
(a) Qual o nmero mnimo de cartes que devem ser retirados da caixa, sem olhar, de modo que se tenha
certeza de que haja, entre os cartes retirados, 5 deles com o mesmo nmero?
(b) Qual o nmero mnimo de cartes que devem ser retirados da caixa, sem olhar, de modo que se tenha
certeza de que haja, entre os cartes retirados, pelo menos um par de cartes com diferena maior do que
5?

UMA SOLUO

(a) Os 5 cartes de mesmo nmero no podem ser os cartes numerados de 1 a 4. Esses cartes so 10. H 6
nmeros que so candidatos a terem 5 cartes repetidos: 5, 6, 7, 8, 9 e 10. Ento, pelo Princpio das Gavetas,
suficiente retirar 10 + 4 6 + 1 = 35 cartes.

(b) No aparecem dois nmeros com diferena maior do que 5 enquanto todos os cartes retirados tiverem todos
os nmeros dentro de uma mesma sequncia de 6 nmeros consecutivos (um bloco de 6). Os blocos de 6 possveis
so: 1-2-3-4-5-6, 2-3-4-5-6-7, 3-4-5-6-7-8, 4-5-6-7-8-9 e 5-6-7-8-9-10. O bloco com mais cartes o ltimo: ele tem
5 + 6 + 7 + 8 + 9 + 10 = 45 cartes. Ento, pelo Princpio das Gavetas, com 46 cartes retirados no possvel que
todos eles estejam num mesmo bloco de 6, ou seja, certamente existir um par com diferena maior do que 5.

3
AV2 - MA 12 - 2012

Questo 4.

A mdia aritmtica de 10 nmeros positivos igual a 1. Os nmeros so agrupados aos pares e os nmeros
de cada par somados, resultando da um conjunto de 5 nmeros positivos.
(a) O que se pode dizer sobre a mdia aritmtica desses 5 nmeros?
(b) Mostre que o produto desses 5 nmeros menor ou igual a 32.

UMA SOLUO

(a) Sejam x1 , x2 , x3 , x4 , x5 , x6 , x7 , x8 , x9 , x10 os 10 nmeros positivos. A primeira informao de que


x1 + x2 + x3 + x4 + x5 + x6 + x7 + x8 + x9 + x10
= 1.
10
Ao agruparmos aos pares esses nmeros e somarmos, obteremos a mesma soma do numerador. Mas, ao tirar a
mdia dos 5, dividiremos por 5, e no por 10. Portanto o resultado ser igual a 2.

(b) Sejam y1 , y2 , y3 , y4 , y5 os 5 nmeros positivos aos quais se refere o enunciado. A mdia geomtrica dos 5
nmeros menor ou igual a sua mdia aritmtica, isto ,
y1 + y2 + y3 + y4 + y5
5 y1 y2 y3 y4 y5 .
5
Acabamos de concluir, no item (a), que o lado direito igual a 2. Da resulta

y1 y2 y3 y4 y5 25 = 32 .

4
AV2 - MA 12 - 2012

Questo 5.

Uma moeda, com probabilidade 0,6 de dar cara, lanada duas vezes.
(a) Qual a probabilidade de que se observem resultados iguais no primeiro e segundo lanamentos?
(b) Dado que os resultados observados no primeiro e segundo lanamentos so iguais, qual a probabilidade
condicional de que o resultado observado neles seja cara?

UMA SOLUO

(a) Evidentemente est-se supondo que os lanamentos so independentes. Para aparecerem resultados iguais
nos dois primeiros lanamentos, ou ocorrem duas caras, com probabilidade 0, 6 0, 6 = 0, 36, ou duas coroas,
com probabilidade 0, 4 0, 4 = 0, 16. Sendo cara-cara e coroa-coroa dois eventos disjuntos (se um deles ocorre o
outro no ocorre), a probabilidade total de ocorrerem dois resultados iguais nos dois primeiros lanamentos de
0, 36 + 0, 16 = 0, 52.

(b) Se j se sabe que vo sair dois resultados iguais, a probabilidade de que seja cara-cara de

0, 36 36 9
= = .
0, 52 52 13

5
Sociedade Brasileira de Matemtica
Mestrado Profissional em Matemtica em Rede Nacional

MA12 Matemtica Discreta


Avaliao - AV 2 - MA 12
29 de junho de 2013

1. (2,0) Penlope quer distribuir 6 presentes entre seus sobrinhos Alfredo, Bruno, Carlos e
Daniel, de modo que cada um receba pelo menos um presente. Todos os presentes devem ser
distribudos.

a) (0,5) Supondo que todos os presentes sejam iguais, de quantos modos ela pode distribuir
os presentes?

b) (1,5) Resolva novamente o item a), supondo agora que todos os presentes sejam diferentes.

2. (2,0) Sejam R o raio da base e h a altura de um cilindro circular reto.

a) (0,5) Calcule a mdia aritmtica e a mdia geomtrica dos valores Rh, Rh e 2R2 .

b) (1,5) Use a desigualdade das mdias para calcular qual a menor rea total possvel para
um cilindro circular reto com um volume V dado. Que relao deve existir entre o raio da base
e a altura desse cilindro para que ele tenha essa menor rea possvel?

3. (2,0) Joo tem dois dados. O dado A tem trs faces vermelhas e trs azuis. O dado B
tem duas faces vermelhas e quatro azuis. Ele escolhe um dos dados ao acaso e o lana. Se a
face que sai azul, ele lana a seguir o dado A; se vermelha, ele lana o dado B.

a) (0,5) Qual a probabilidade de que o segundo dado lanado seja o dado B?

b) (0,5) Qual a probabilidade de que saia uma face vermelha no segundo lanamento?

c) (1,0) Se a face que sai no segundo lanamento vermelha, qual a probabilidade de que
o primeiro dado lanado tenha sido o A?
4. (2,0) Em uma reunio h 26 pessoas, com idades variando entre 16 e 65 anos.

a) (1,0) Mostre que h na reunio pelo menos um par de pessoas cujas datas de nascimento
estejam espaadas por menos de 2 anos.

b) (0,5) Existe um ms do ano em que pelo menos k pessoas dentre as presentes na reunio
fazem aniversrio. Qual o maior valor de k para o qual esta sentena necessariamente
verdadeira?

c) (0,5) Considere a afirmao: Existe um ms em que pelo menos quatro pessoas do mesmo
sexo dentre as presentes na reunio fazem aniversrio. Quantas pessoas a mais, no mnimo,
devem chegar reunio para que se tenha certeza de que esta afirmativa seja verdadeira?

5. (2,0) No sorteio da Mega-Sena, so sorteados, consecutivamente e sem reposio, 6 n-


meros de 1 a 60.

a) (1,0) Qual a probabilidade de que o nmero 23 seja um dos sorteados?

b) (1,0) Qual a probabilidade de que o ltimo nmero sorteado seja o maior dos 6 nmeros
que foram sorteados?
Sociedade Brasileira de Matemtica
Mestrado Profissional em Matemtica em Rede Nacional

MA12 Matemtica Discreta


Avaliao - GABARITO AV 2 - MA 12
29 de junho de 2013

1. (2,0) Penlope quer distribuir 6 presentes entre seus sobrinhos Alfredo, Bruno, Carlos e
Daniel, de modo que cada um receba pelo menos um presente. Todos os presentes devem ser
distribudos.
a) (0,5) Supondo que todos os presentes sejam iguais, de quantos modos ela pode distribuir
os presentes?
b) (1,5) Resolva novamente o item a), supondo agora que todos os presentes sejam diferentes.

Uma soluo:
a) Uma vez tendo distribudo um presente para cada um dos sobrinhos, sobram 2 presentes,
para distribuir para 4 crianas. O nmero de modos de faz-lo igual ao nmero de solues
no negativas de x1 + x2 + x3 + x4 = 2, que igual a CR4,2 = C5,2 = 10.
tambm fcil enumerar todas as possibilidades: h 4 modos de uma das crianas receber
dois presentes adicionais e C4,2 = 6 modos de duas delas receberem um presente adicional cada.
Aqui esto as possibilidades:

ABCD ABCD ABCD ABCD


3111 1311 1131 1113

ABCD ABCD ABCD ABCD ABCD ABCD


2211 2121 2112 1221 1212 1122
b) Primeiro, devemos decidir qual a quantidade de presentes que cada sobrinho vai receber.
As possibilidades so as seguintes:
um dos sobrinhos recebe 3 presentes e os demais 1.
O sobrinho a receber os 3 presentes pode ser escolhido de 4 modos. Os presentes dos demais
podem ser escolhidos de 6 5 4 = 120 modos (os que restarem ficam com o sobrinho que
recebe 3 presentes). O nmero total de possibilidades 4 120 = 480.
exatamente dois sobrinhos recebem dois presentes.
Esses sobrinhos podem ser escolhidos de C4,2 = 6 modos. Os presentes dos outros sobrinhos
podem ser escolhidos de 65 = 30 modos. Os presentes de um dos sobrinhos a receber 2 presentes
podem ser escolhidos de C4,2 = 6 modos. O nmero total de possibilidades 6 30 6 = 1080.
O nmero total de modos de distribuir os presentes 480 + 1080 = 1560.
2. (2,0) Sejam R o raio da base e h a altura de um cilindro circular reto.
a) (0,5) Calcule a mdia aritmtica e a mdia geomtrica dos valores Rh, Rh e 2R2 .
b) (1,5) Use a desigualdade das mdias para calcular qual a menor rea total possvel para
um cilindro circular reto com um volume V dado. Que relao deve existir entre o raio da base
e a altura desse cilindro para que ele tenha essa menor rea possvel?

Uma soluo:
1
a) A mdia aritmtica A = (2Rh + 2R2 )/3 e a geomtrica G = (2R4 h2 ) 3 .
b) A rea total do cilindro S = 2Rh + 2R2 , enquanto que seu volume V = R2 h.
Logo

S 2V 2
A= e G3 = 2
3
Da, pela desigualdade das mdias, A G, e portanto

S 2V 2 1 1
( 2 )3 = S 3(2V 2 ) 3
3
1
Assim, a menor rea total possvel para um volume V fixo S = 3(2V 2 ) 3 .
Para que valha a igualdade (e portanto para que o cilindro tenha rea total mnima), os
elementos que compem as mdias devem ser todos iguais, logo deve-se ter Rh = 2R2 , ou seja,
h = 2R.

3. (2,0) Joo tem dois dados. O dado A tem trs faces vermelhas e trs azuis. O dado B
tem duas faces vermelhas e quatro azuis. Ele escolhe um dos dados ao acaso e o lana. Se a
face que sai azul, ele lana a seguir o dado A; se vermelha, ele lana o dado B.
a) (0,5) Qual a probabilidade de que o segundo dado lanado seja o dado B?
b) (0,5) Qual a probabilidade de que saia uma face vermelha no segundo lanamento?
c) (1,0) Se a face que sai no segundo lanamento vermelha, qual a probabilidade de que
o primeiro dado lanado tenha sido o A?

Uma soluo:
a) P (2o dado B) = P (1o dado A).P (2o dado B|1o dado A)+
1
+P (1o dado B).P (2o dado B|1o dado B) = 2
12 + 21 1
3
= 5
12
.
7
claro que P (1o dado A) = 12
; isto ser usado no prximo tem.

b) P (2a Vermelha) = P (2o dado A).P (Vermelha|A) + P (2o dado B).P (Vermelha|B)
7
= 12 12 + 12
5
13 = 72
31
.
c) P (1o dado A|2a Vermelha) = P(1o dado A e 2a Vermelha)/P (2a Vermelha).
Mas
P (1o dado A e 2a Vermelha) = P(1o dado A e 2o dado A e 2a Vermelha)+
+P (1o dado A e 2 dado B e 2a Vermelha) = 21 12 12 + 12 12 31 = 5
24
.
Logo, P (1o dado A|2a Vermelha) =
5
= P (1o dado A e 2a Vermelha)/P (2a Vermelha) = ( 24 )/( 31
72
)= 15
31
.
Estes resultados podem ser obtidos tambm usando-se diretamente o diagrama:

Segundo
lanamento
Primeiro
lanamento

1/2 Azul

Face azul A o dado


selecionado para o segundo Vermelho
1/2 1/2
lanamento

2/3 Azul
Escolha do
Face vermelha B o
dado A
dado selecionado para o
1/2 Vermelho
1/2 segundo lanamento 1/3

1/2 Azul
2/3 Face azul A o dado
1/2 selecionado para o segundo
Escolha do Vermelho
lanamento 1/2
dado B

2/3 Azul
1/3 Face vermelha B o
dado selecionado para o
segundo lanamento Vermelho
1/3

4. (2,0) Em uma reunio h 26 pessoas, com idades variando entre 16 e 65 anos.


a) (1,0) Mostre que h na reunio pelo menos um par de pessoas cujas datas de nascimento
estejam espaadas por menos de 2 anos.
b) (0,5) Existe um ms do ano em que pelo menos k pessoas dentre as presentes na reunio
fazem aniversrio. Qual o maior valor de k para o qual esta sentena necessariamente
verdadeira?
c) (0,5) Considere a afirmao: Existe um ms em que pelo menos quatro pessoas do mesmo
sexo dentre as presentes na reunio fazem aniversrio. Quantas pessoas a mais, no mnimo,
devem chegar reunio para que se tenha certeza de que esta afirmativa seja verdadeira?

Uma soluo:

a) Sejam x1 x2 ... x26 as idades em ordem no decrescente. Por hiptese temos que
x1 16 e x26 65. Como

(x2 x1 ) + (x3 x2 ) + + (x26 x25 ) = x26 x1 65 16 = 49,


ento a mdia aritmtica

(x2 x1 ) + (x3 x2 ) + + (x26 x25 )


25
menor do que 49/25 e pelo menos uma das diferenas (xi+1 xi ) menor ou igual do que
49/25 e, portanto, menor do que 2.
Obs.: A rigor, x26 x1 no , necessariamente, igual a 49; isto ocorre quando discretizamos o
tempo em anos, mas, de fato, a pessoa com idade x1 pode ter acabado de completar 16 anos e
a pessoa com idade x26 pode estar prestes a fazer 66. Em qualquer caso certamente x26 x1
estritamente menor que 50, o que faz com que a concluso seja vlida.
b) A resposta k = 3. Como h 26 pessoas e 12 possibilidades para o ms de aniversrio,
h um ms em que nasceram no mnimo 26/12 = 2, 1... pessoas. Portanto, h um ms em que
nasceram pelo menos trs pessoas. No se pode garantir que haja 4 pessoas em um mesmo ms
(basta distribuir as pessoas colocando duas em cada um de 10 meses e trs em cada um dos
outros 2 meses).
c) H 24 combinaes (gavetas) possveis de sexo e ms de aniversrio. Logo, para que
se possa garantir que haja 4 pessoas em uma mesma gaveta, preciso que haja pelo menos
24 3 + 1 = 73 objetos (pessoas). Portanto, preciso que cheguem mais 73 26 = 47 pessoas.

5. (2,0) No sorteio da Mega-Sena, so sorteados, consecutivamente e sem reposio, 6 n-


meros de 1 a 60.
a) (1,0) Qual a probabilidade de que o nmero 23 seja um dos sorteados?
b) (1,0) Qual a probabilidade de que o ltimo nmero sorteado seja o maior dos 6 nmeros
que foram sorteados?

Uma soluo:
a) Primeira soluo: A probabilidade que 23 saia em cada um dos nmeros sorteados 1/60.
Logo, a probabilidade de que saia em um deles 6 1/60 = 1/10.
Segunda soluo: O nmero de casos possveis para o sorteio 60 59 58 57 56 55.
O nmero de casos favorveis

6 59 58 57 56 55
(6 o nmero de modos de se escolher a posio do 23 e o produto dos demais fatores d o
nmero de modos de se escolher os outros 5 nmeros)
A probabilidade de aparecer o 23 , portanto,

(6 59 58 57 56 55)/(60 59 58 57 56 55) = 1/10

b) Como os nmeros so sorteados ao acaso, todas as ordenaes dos nmeros sorteados so


igualmente provveis. Logo, igualmente provvel que o maior nmero aparea em cada posio.
Assim, a probabilidade de que ele aparea na ltima posio 1/6.
MA12 Matematica Discreta AV2 2014

Questao 1 [ 2,0 pt ]

Calcule a velocidade media em cada uma das situacoes abaixo:


(a) Um carro percorre metade de uma certa distancia a uma velocidade de 100 km/h e a outra metade
da distancia a 60 km/h.
(b) Um carro percorre uma estrada por um certo tempo t a uma velocidade de 100 km/h e depois durante
o mesmo tempo t a uma velocidade de 60 km/h.

Questao 2 [ 2,0 pt ]

Encontre o valor do numero natural n que satisfaz a igualdade abaixo:


log2 2 + log2 4 + log2 8 + + log2 2n = 110 .

Questao 3 [ 2,0 pt ]

De um grupo de 12 mulheres, sendo Paula uma delas, e de 10 homens, sendo Felipe um deles, quantas
comissoes podem ser formadas com:
(a) 4 mulheres e 3 homens?
(b) 5 pessoas, sendo pelo menos 3 mulheres?
(c) 6 pessoas, sendo 3 de cada sexo e de modo que Paula e Felipe facam parte?

Questao 4 [ 2,0 pt ]

(a) Mostre que todo numero natural n tem um multiplo que se escreve apenas com os algarismos 0 e 3.
(b) Mostre que se n e relativamente primo com 10, entao n tem um multiplo com todos os algarismos
iguais a 3.

Questao 5 [ 2,0 pt ]

Duas maquinas A e B produzem 5000 pecas por dia. A maquina A produz 3000 pecas, das quais 2% sao
defeituosas. A maquina B produz as restantes 2000, das quais 1% sao defeituosas.
(a) Se uma peca for escolhida ao acaso, qual a probabilidade de ser defeituosa?
(b) Da producao total de um dia, uma peca e escolhida ao acaso e, examinando-a, constata-se que ela e
defeituosa. Qual e a probabilidade de que ela tenha sido produzida pela maquina A?
MA12 Matematica Discreta AV2 2014

Questao 1 [ 2,0 pt ]

Calcule a velocidade media em cada uma das situacoes abaixo:


(a) Um carro percorre metade de uma certa distancia a uma velocidade de 100 km/h e a outra metade
da distancia a 60 km/h.
(b) Um carro percorre uma estrada por um certo tempo t a uma velocidade de 100 km/h e depois durante
o mesmo tempo t a uma velocidade de 60 km/h.

Solucao
Velocidade media e a razao entre a distancia percorrida e o tempo gasto para percorre-la.

(a) Consideremos d a distancia total do percurso. Na primeira metade do percurso o carro percorreu a distancia
d
2 com velocidade de 100 km/h. O tempo t1 gasto foi, em horas
d
2 d d
100 = 100t1 = t1 = .
t1 2 200
Analogamente, na segunda metade do percurso o tempo t2 gasto foi, em horas
d
2 d d
60 = 60t2 = t2 = .
t2 2 120
O tempo total t = t1 + t2 , gasto no percurso, foi, em horas
d d 8d d
t = t1 + t2 = + = = .
200 120 600 75
Desse modo, a velocidade media para o percurso total foi, em km/h
d
Vm = d
= 75
75

(b) No primeiro trecho, a distancia d1 percorrida foi, em quilometros


d1
100 = d1 = 100t.
t
No segundo trecho, a distancia d2 percorrida foi, em quilometros
d2
60 = d2 = 60t.
t
Assim, a distancia total percorrida foi, em quilometros, d = d1 + d2 = 100t + 60t = 160t, e o tempo total do
percurso, em horas, foi 2t. Desse modo, a velocidade para o percurso total foi, em km/h
160t
Vm = = 80.
2t

Questao 2 [ 2,0 pt ]

Encontre o valor do numero natural n que satisfaz a igualdade abaixo:


log2 2 + log2 4 + log2 8 + + log2 2n = 110 .
Solucao
Reescrevendo a soma proposta, temos:

log2 21 + log2 22 + log2 23 + + log2 2n = 110 .

Calculemos inicialmente, de modo generico, log2 2n


k
log2 2n = k log 1 2n = k 2 2 = 2n k = 2n.
22

A soma proposta inicialmente passa a ser representada por:

2 + 4 + 6 + + 2n = 110.

Trata-se, pois, da soma de n termos em progressao aritmetica de razao r = 2, cujo primeiro termo e a1 = 2 e o
n-esimo termo e an = 2n.
Pelo calculo da soma de n termos em progressao aritmetica, temos:
n n
Sn = (a1 + an ). 110 = (2 + 2n). 110 = (n + 1).n n2 + n 110 = 0.
2 2
Resolvendo a equacao, obtemos como resultados n = 10 e n = 11 (que nao convem pois 11 6 N). Portanto, a
soma e satisfeita para n = 10.
De fato, pela complementacao dos termos da progressao aritmetica, pode-se facilmente chegar a soma:

2 + 4 + 6 + 8 + 10 + 12 + 14 + 16 + 18 + 20 = 110.

Questao 3 [ 2,0 pt ]

De um grupo de 12 mulheres, sendo Paula uma delas, e de 10 homens, sendo Felipe um deles, quantas
comissoes podem ser formadas com:
(a) 4 mulheres e 3 homens?
(b) 5 pessoas, sendo pelo menos 3 mulheres?
(c) 6 pessoas, sendo 3 de cada sexo e de modo que Paula e Felipe facam parte?

Solucao

(a) Para formarmos a comissao devemos escolher 4 mulheres, dentre as 12, e 3 homens, dentre os 10. Ha, portanto
4 3
C12 C10 = 495 120 = 59400 comissoes.

(b) Para formarmos comissoes de 5 pessoas, sendo pelo menos 3 mulheres, devemos considerar que ha comissoes
com:
3 2
3 mulheres e 2 homens: C12 C10 = 220 45 = 9900,
4 1
4 mulheres e 1 homem: C12 C10 = 495 10 = 4950,
5
5 mulheres C12 = 792.
Desse modo, ha 15642 comissoes de 5 pessoas, sendo pelo menos 3 mulheres.

(c) Como Paula ja esta definida como membro da comissao feminina, cabera a escolha dos outros dois membros,
dentre as 11 mulheres restantes. Do mesmo modo, Felipe ja esta definido como membro da comissao masculina,
cabendo a escolha dos outros dois membros, dentre os 9 homens restantes.
2
Ha, portanto, C11 C92 = 55 36 = 1980 comissoes de 6 pessoas, sendo 3 de cada sexo e de modo que Paula
e Felipe facam parte.
Questao 4 [ 2,0 pt ]

(a) Mostre que todo numero natural n tem um multiplo que se escreve apenas com os algarismos 0 e 3.
(b) Mostre que se n e relativamente primo com 10, entao n tem um multiplo com todos os algarismos
iguais a 3.

Solucao
Seja n um numero natural.

(a) Considere os n + 1 primeiros numeros da sequencia 3, 33, 333, 3333, . . .. Divida-os por n e considere os restos
dessas divisoes. Esses restos so podem ser iguais a 0, 1, 2, . . . , n 1.
Pensando nos n + 1 como objetos e nos n possveis restos como gavetas, temos mais objetos do que gavetas.
O Princpio das Gavetas assegura que alguma gaveta recebera mais de um objeto, isto e, ha dois numeros da
lista que dao o mesmo resto quando divididos por n, digamos 33 . . . 3 (p algarismos) e 33 . . . , 3 (q algarismos),
com p < q. A diferenca desses numeros e um multiplo de n e se escreve 33 . . . 30 . . . 0, com p algarismos iguais
0 e q p algarismos 1.

(b) Agora vamos supor que n e relativamente primo com 10. Pelo item (a) sabemos que n possui um multiplo da
forma 33 . . . 30 . . . 0 = 33 . . . 3 10p . Como mdc(n, 10) = 1, segue o resultado.

Questao 5 [ 2,0 pt ]

Duas maquinas A e B produzem 5000 pecas por dia. A maquina A produz 3000 pecas, das quais 2% sao
defeituosas. A maquina B produz as restantes 2000, das quais 1% sao defeituosas.
(a) Se uma peca for escolhida ao acaso, qual a probabilidade de ser defeituosa?
(b) Da producao total de um dia, uma peca e escolhida ao acaso e, examinando-a, constata-se que ela e
defeituosa. Qual e a probabilidade de que ela tenha sido produzida pela maquina A?

Solucao

(a) A maquina A produz diariamente 3000 pecas, dentre as quais 60 sao defeituosas.
A maquina B produz diariamente 2000 pecas, dentre as quais 20 sao defeituosas.
Das 5000 pecas produzidas diariamente, 80 sao defeituosas. A probabilidade P(A) de uma peca escolhida ao
acaso ser defeituosa sera
80
P (A) = = 0, 016 = 1, 6%
5000
(b) Sabe-se que a peca escolhida e defeituosa, portanto ela e uma das 80 pecas defeituosas produzidas no dia. A
probabilidade P(B) de que essa peca defeituosa tenha sido produzida pela maquina A sera
60
P (B) = = 0, 75 = 75%
80
AVALIAES
AV-03
MA12 Matematica Discreta Prova 3 2011

Questao 1.
A sequencia 0, 3, 7, 10, 14, 17, 21, . . . e formada a partir do numero 0 somando-se alternadamente 3 ou 4 ao termo
anterior, isto e: o primeiro termo e 0, o segundo e 3 a mais que o primeiro, o terceiro e 4 a mais que o segundo, o
quarto e 3 a mais que o terceiro, o quinto e 4 a mais que o quarto e assim sucessivamente.

(0,5) (a) Qual e o centesimo termo dessa sequencia?

(0,5) (b) Qual e a soma dos 100 primeiros termos dessa sequencia?

(1,0) (c) Algum termo desta sequencia e igual a 2000? Por que?

Questao 2.
Seja Rn o numero maximo de regioes determinadas no plano por n crculos.

(0,5) (a) Quais sao os valores de R1 e R2 ?

(0,5) (b) Explique por que Rn+1 = Rn + 2n, para todo n 1.

(1,0) (c) Mostre por inducao que Rn = n2 n + 2.

Questao 3.
Suponha que o dinheiro valha 10% ao mes para um comerciante que vende determinado produto por R$ 4200,00
a vista.

(1,0) (a) Se o comerciante deseja oferecer o produto para compra em duas prestacoes iguais, a primeira no ato da compra,
qual deve ser o valor dessas prestacoes?

(1,0) (b) Suponha que ele deseja oferecer o produto em 10 prestacoes iguais, a primeira no ato da compra. Escreva uma
expressao que permita calcular o valor da prestacao.

Questao 4.
Uma senha de banco e formada por 4 digtos de 0 a 9.

(1,0) (a) Quantas sao as senhas em que aparecem exatamente tres dgitos diferentes?

(1,0) (b) Quantas sao as senhas em que nao ha dgitos consecutivos iguais?

Questao 5.
Joao, ao partir para uma viagem, ficou de enviar um cartao postal para sua mae. A probabilidade de que ele envie
o cartao e igual a 0,7. Por outro lado, a probabilidade de um cartao postal se extraviar e 0,1.

(1,0) (a) Qual e a probabilidade de que a mae de Joao receba um cartao postal dele?

(1,0) (b) Se ela nao receber um cartao de Joao, qual e a probabilidade de que ele o tenha enviado?
AV3 - MA 12 - 2011

Questo 1.
A sequncia 0, 3, 7, 10, 14, 17, 21, . . . formada a partir do nmero 0 somando-se alternadamente 3 ou 4 ao termo
anterior, isto : o primeiro termo 0, o segundo 3 a mais que o primeiro, o terceiro 4 a mais que o segundo, o
quarto 3 a mais que o terceiro, o quinto 4 a mais que o quarto e assim sucessivamente.

(0,5) (a) Qual o centsimo termo dessa sequncia?

(0,5) (b) Qual a soma dos 100 primeiros termos dessa sequncia?

(1,0) (c) Algum termo desta sequncia igual a 2000? Por qu?

UMA SOLUO

(a) Chamemos de a1 , a2 , a3 , . . . os termos dessa sequncia. A sequncia dos termos com ndices mpares a1 , a3 , a5 , . . .
uma progresso aritmtica com termo inicial 0 e passo (ou razo) 7. A sequncia dos termos com ndices pares
a2 , a4 , a6 , . . . uma progresso aritmtica com termo inicial 3 e passo 7. O centsimo termo o 50o da sequncia dos
pares. Ento a100 = 3 + (50 1) 7 = 3 + 343 = 346.

(b) H maneiras diferentes de se fazer isso. Podemos agrupar a soma assim:

( a1 + a100 ) + ( a2 + a99 ) + ( a3 + a98 ) + . . . + ( a50 + a51 ) .

Veja que de a1 para a2 h um acrscimo de 3 e de a99 para a100 tambm. Ento os dois primeiros termos so iguais.
Do segundo para o terceiro h um aumento e um decrscimo de 4, logo o terceiro termo igual ao segundo. E assim
por diante. Ento todos os termos entre parnteses so iguais ao primeiro, que vale 0 + 346 = 346. Como so 50
termos, a soma d 50 346 = 17300.
Outro jeito de fazer somar separadamente as sequncias com ndices mpares e pares. No segundo caso (pares),
so 50 termos da progresso aritmtica de razo 7 comeando em 3 e terminando em 346. A soma dessa progresso
d
3 + 346
50 = 25 349 = 8725 .
2
No primeiro caso (mpares), so 50 termos, mas todos 3 unidades menores do que os termos da srie par. Ento a
soma desses 8725 subtrado de 50 3 = 150, isto , d 8575. Juntando as duas, ficamos com 17300.
Obs. Essa segunda soma tambm sairia da mesma forma como a outra, pois a PA tem primeiro termo igual a 0,
ltimo termo igual a 343, totalizando 50 termos, logo soma
0 + 343
50 = 25 343 = 8575 .
2

(c) Observe primeiro que se n mpar ento an mltiplo de 7, e se n par ento an 3 mltiplo de 7 (de fato,
valem as recprocas, mas no precisaremos disso).
Como nem 2000 = 7 285 + 5 nem 1997 = 7 285 + 2 so mltiplos de 7, ento 2000 no pode ser um an nem para
n par nem para n mpar.

1
AV3 - MA 12 - 2011

Questo 2.
Seja Rn o nmero mximo de regies determinadas no plano por n crculos.

(0,5) (a) Quais so os valores de R1 e R2 ?

(0,5) (b) Explique por que Rn+1 = Rn + 2n, para todo n 1.

(1,0) (c) Mostre por induo que Rn = n2 n + 2.

UMA SOLUO

(a) Um nico crculo no plano determina exatamente duas regies (dentro e fora). Ento R1 = 2. Agora colocamos
um segundo crculo no plano e olhamos para vrias possibilidades: (i) se ele for idntico ao primeiro, continuamos
com duas regies; (ii) se um dos crculos est inteiramente contido numa das regies delimitadas pelo outro, ento
ficam delimitadas 3 regies (mesma coisa se apenas se tangenciam); (iii) se eles se intersectam sem se tangenciarem,
ficam delimitadas 4 regies. Esse o mximo possvel, ento R2 = 4.

(b) Primeiro verifiquemos se a frmula est compatvel com a resposta anterior. Pela frmula, deveramos ter
R2 = R1 + 2 1. De fato, R1 + 2 1 = 2 + 2 = 4.
Agora imaginemos que n crculos j esto desenhados, definindo um certo nmero de regies. Ento desenha-
mos um novo crculo (diferente dos anteriores, pois neste caso a diviso de regies permaneceria a mesma), que
intersectar os crculos anteriores em um certo nmero de pontos. Como o novo crculo s pode intersectar cada
um dos outros crculos em no mximo 2 pontos, ele ter no mximo 2n interseces. Essas interseces dividiro o
crculo em arcos de crculo, que sero no mximo 2n (e no mnimo 1, que quando o crculo no intersecta nenhum
dos crculos j desenhados). Chamemos de k o nmero de arcos de crculo obtidos.
Agora suponha que numeremos esses k arcos de crculo, e vamos desenhar o n + 1-simo crculo arco por arco,
contando qual o mximo acrscimo de regies em cada etapa. O primeiro arco est inteiramente contido em uma
das regies previamente delimitadas, e a divide em duas regies. Isso acrescenta uma unidade na contagem de
regies. Como o segundo arco s pode intersectar os crculos anteriores e o primeiro arco em seus extremos, ele
tambm est inteiramente contido em uma das regies, incluindo as novas regies formadas pela introduo do
primeiro arco. Ele dividir essa regio em duas, acrescentando mais uma unidade na contagem. Esse raciocnio
pode ser repetido de forma indutiva at chegarmos no k-simo arco. No total, sero acrescentadas k regies
contagem.
Como k 2n, ento so acrescentadas no mximo 2n regies contagem, quando se passa de n crculos para
n + 1 crculos. Portanto, se n crculos no podem dividir o plano em mais do que Rn regies, ento n + 1 crculos
no podero dividir o plano em mais do que Rn + 2n regies. Isso define o valor de Rn+1 .
Observao. A rigor, dever-se-ia mostrar que, para cada n, alguma configurao de crculos divide o plano em
Rn regies, para se dizer que Rn o mximo (e no apenas uma cota superior). Para tanto, em vista do que foi
feito acima, basta achar uma lista de crculos C1 , C2 , C3 , . . . tal que, para qualquer n 1, o crculo Cn+1 intersecta

2
cada crculo C1 , . . . , Cn em 2 pontos, produzindo ao todo 2n pontos de interseco distintos entre si. Isso pode ser
realizado por
1
Ci = {( x, y); ( x )2 + y2 = 1} , i = 1, 2, 3, . . . ,
i
isto , Ci o crculo de raio 1 e centro em ( 1i , 0). Uma conta simples mostra que Cn+1 intersecta Ci nos dois pontos
s
   2
1 1 1 1 1 1
+ , 1 .
2 n+1 i 2 i n+1

1
Como os valores de i so distintos para i = 1, 2, . . . , n, os 2n pontos de interseco so todos distintos entre si.

(c) A frmula vale para n = 1, pois 12 1 + 2 = 2 = R1 . Agora, supondo que ela vale para n, isto , supondo
Rn = n2 n + 2 verdadeira, queremos mostrar que tambm vale para n + 1, isto , queremos mostrar que Rn+1 =
(n + 1)2 (n + 1) + 2. Ora, a relao de recorrncia nos d Rn+1 = Rn + 2n; valendo a hiptese de que Rn =
n2 n + 1, ento

Rn+1 = Rn + 2n
= (n2 n + 2) + 2n
= n2 + n + 2
= [(n + 1)2 2n 1] + n + 2
= ( n + 1)2 n 1 + 2
= ( n + 1)2 ( n + 1) + 2 .

3
AV3 - MA 12 - 2011

Questo 3. Suponha que o dinheiro valha 10% ao ms para um comerciante que vende determinado produto por
R$ 4200,00 vista.

(1,0) (a) Se o comerciante deseja oferecer o produto para compra em duas prestaes iguais, a primeira no ato da
compra, qual deve ser o valor dessas prestaes?

(1,0) (b) Suponha que ele deseja oferecer o produto em 10 prestaes iguais, a primeira no ato da compra. Escreva uma
expresso que permita calcular o valor da prestao.

UMA SOLUO

x 1 114200
(a) Se x for o valor da prestao, ele quer x + 1,1 = 4200. Isso d x (1 + 1,1 ) = 4200. Ento x = 21 = 2200.

(b) Pelo mesmo raciocnio, ele quer x tal que


x x x
x+ + +...+ = 4200 .
1, 1 1, 12 1, 19
Ou seja,  
x 1 + 1, 11 + 1, 12 + . . . + 1, 19 = 4200

e
1 1, 110
 
x = 4200 .
1 1, 11
Logo
1
1 1,1 4200 1
x = 4200 1
= .
1 11 1 110
1,110 1,1

4
AV3 - MA 12 - 2011

Questo 4.
Uma senha de banco formada por 4 digtos de 0 a 9.

(1,0) (a) Quantas so as senhas em que aparecem exatamente trs dgitos diferentes?

(1,0) (b) Quantas so as senhas em que no h dgitos consecutivos iguais?

UMA SOLUO

(a) Se h exatamente 3 dgitos diferentes, ento h dois dgitos iguais e mais dois outros, diferentes dele e diferentes
entre si. H 10 possibilidades para o dgito que aparece repetido. Escolhido esse dgito, precisamos de 2 dgitos
entre os 9 restantes. Temos C92 = 36 escolhas para os dgitos restantes. Portanto, na escolha dos dgitos em que o
dgito repetido est determinado, temos 360 possibilidades.
Falta agora ver de quantas maneiras diferentes eles podem ser dispostos. Primeiro escolhemos a disposio dos
dois dgitos que no se repetem. Como h C42 possibilidades de escolha de duas entre quatro posies, temos um
total de 6 disposies possveis.
Fixada as posies dos dgitos repetidos, temos 2 maneiras de colocar os outros dois dgitos.
Ento cada uma das 360 escolhas dos 3 dgitos (com o dgito que se repete determinado) pode ser arranjada de 12
maneiras distintas, o que d um total de 360 12 = 4320 senhas com exatamente 3 dgitos diferentes.

(b) H 10 possibilidades para o primeiro dgito. Como o segundo s no pode ser igual ao primeiro, h 9 possibi-
lidades para o segundo (para cada escolha do primeiro). Mais uma vez, h 9 possibilidades para o terceiro (para
cada escolha dos dois primeiros) e 9 para o quarto (para cada escolha dos trs primeiros). Ento so 10 93 = 7290
possibilidades.

5
AV3 - MA 12 - 2011

Questo 5.
Joo, ao partir para uma viagem, ficou de enviar um carto postal para sua me. A probabilidade de que ele envie
o carto igual a 0,7. Por outro lado, a probabilidade de um carto postal se extraviar 0,1.

(1,0) (a) Qual a probabilidade de que a me de Joo receba um carto postal dele?

(1,0) (b) Se ela no receber um carto de Joo, qual a probabilidade de que ele o tenha enviado?

UMA SOLUO

(a) A probabilidade de que um carto no extravie, dado que foi enviado, de 1 0, 1 = 0, 9. Portanto a probabi-
lidade de que a me de Joo receba um carto de seu filho igual probabilidade de que seja enviado e no seja
extraviado (dado que foi enviado), isto 0, 9 0, 7 = 0, 63.

(b) A probabilidade de a me no receber o carto igual a 1 0, 63 = 0, 37. A probabilidade de a me no receber


o carto por no ter sido enviado igual a 1 0, 7 = 0, 3 e a probabilidade de a me no receber o carto por ter
se extraviado 0, 1 0, 7 = 0, 07. Portanto, se for dado que ela no recebeu o carto, a probabilidade de que ele o
tenha enviado de 0, 07/0, 37 = 0, 7/3, 7 = 7/37.

6
AV3 - MA 12 - 2012

Questo 1.
1
Uma moeda, com probabilidade 3 de dar cara, lanada 40 vezes.
(a) Explique por que a probabilidade pk de se obter k caras nos 40 lanamentos dada por
 k  40k
1 2
pk = C40,k ,
3 3
para k = 0, 1, 2, . . . , 40.
(b) Calcule para que valores de k tem-se pk+1 > pk .
(c) Utilize (b) para obter o valor de k para o qual a probabilidade de se obter k caras mxima.

UMA SOLUO

(a) A probabilidade de sarem k caras e 40 k coroas em 40 lanamentos, numa ordem especfica, a probabilidade
de sair cara elevada potncia k vezes a probabilidade de sair coroa elevada potncia 40 k. Neste caso,
 k  40k
1 2
.
3 3
Mas o nmero de maneiras (ou ordens) que podem sair as k caras o nmero de maneiras de se escolher k elementos
entre 40, ou seja, C40,k . Por isso a frmula do enunciado.

(b) Lembramos que


40!
C40,k = .
k!(40 k )!
Ento pk+1 > pk se, e somente se,
 k+1  40k1  k  40k
40! 1 2 40! 1 2
> .
(k + 1)!(40 k 1)! 3 3 k!(40 k)! 3 3
Ou seja, se, e somente se,
1 1 1 2
> ,
k+1 3 40 k 3
38
cancelando os fatores comuns nos dois lados. Portanto pk+1 > pk se, e somente se, 40 k > 2k + 2, isto , k < 3 .
Como k inteiro, isto equivalente a k 12.

(c) De (b) vale


p0 < p1 < p2 < . . . < p12 < p13 .

Vale, tambm,
p13 p14 . . . p40 ,

embora valham, de fato, as desigualdades estritas, se for aplicado raciocnio anlogo quele feito em (b). O valor
mximo, ocorre, portanto, em k = 13.

1
AV3 - MA 12 - 2012

Questo 2.

A soma dos n primeiros termos de uma progresso aritmtica dada por Sn = 2n2 15n.
(a) Determine o dcimo termo da progresso.
(b) Encontre o primeiro termo positivo da progresso.

UMA SOLUO

(a) O dcimo termo S10 S9 , isto ,

(2 102 15 10) (2 92 15 9) = 23 .

(b) Queremos saber para quais valores de n o n-simo termo, isto , a expresso Sn Sn1 , maior do que zero.
Temos
(2n2 15n) (2(n 1)2 15(n 1)) = 4n 17 ,

logo o primeiro termo positivo ocorre para o primeiro n tal que 4n 17 > 0, isto , para n = 5.

2
AV3 - MA 12 - 2012

Questo 3.

Um comerciante, para quem o dinheiro vale 3% ao ms, oferece determinado produto por 3 prestaes mensais
iguais a R$ 100,00, a primeira paga um ms aps a compra.
(a) Que valor o comerciante deve cobrar por este produto, no caso de pagamento vista?
(b) Se um consumidor desejar pagar o produto em trs prestaes mensais iguais, sendo a primeira paga no
ato da compra, qual deve ser o valor das parcelas?

Utilize, se desejar, os seguintes valores para as potncias de 1,03: 1, 032 = 1, 0609, 1, 033 = 1, 0927, 1, 031 = 0, 9709,
1, 032 = 0, 9426, 1, 033 = 0, 9151.

UMA SOLUO

(a) Trazendo os valores das prestaes a valor presente e somando, obtemos o valor para o pagamento vista:

100 100 100 100


+ 2
+ = (1 + 1, 031 + 1, 032 )
1, 03 1, 03 1, 033 1, 03
100 1 1, 033
=
1, 03 1 1, 031
1 0, 9151 8, 49
' 100 = 100 = 283 .
0, 03 3
O resultado pode ser um pouco diferente dependendo de como so usados os arredondamentos.

(b) No preciso saber o valor vista. Basta trazer em 1 ms cada uma das prestaes de 100 reais. Ou seja, cada
uma deve ser de 100 1, 031 , que aproximadamente igual a 97,09 reais.

3
AV3 - MA 12 - 2012

Questo 4.

(a) Mostre, por induo finita, que


1 1 1 7
+ +...+
n+1 n+2 2n 12
para todo nmero natural n 2.
1
(b) Use este fato para explicar por que a soma 1 + 2 + 13 + 14 + . . . cresce sem limite.

UMA SOLUO

1
+ 14 , que igual a 12
(a) Primeiro vejamos que a desigualdade vale para n = 2. O lado esquerdo, neste caso, 7
.3
Agora suponhamos que a desigualdade valha para um certo n 2. Iremos mostrar que a correspondente desi-
gualdade tambm vale para n + 1. Isto , supondo que vale

1 1 1 7
+ +...+
n+1 n+2 2n 12
(hiptese de induo), mostraremos que vale

1 1 1 7
+ +...+ .
( n + 1) + 1 ( n + 1) + 2 2( n + 1) 12

Ora, mas o lado esquerdo pode ser escrito como


 
1 1 1 1 1 1
+ +...+ + + .
n+1 n+2 2n 2n + 1 2n + 2 n + 1
7
Pela hiptese de induo, a soma entre parnteses maior do que ou igual a 12 . Ento basta mostrar que

1 1 1
+ 0.
2n + 1 2n + 2 n + 1
Mas isso verdade porque
1 1 2 1
+ > = .
2n + 1 2n + 2 2n + 2 n+1

(b) A soma mencionada pode ser agrupada assim:


     
1 1 1 1 1 1 1
1+ + + + +...+ + +...+ +...
2 3 4 5 8 9 16
1 1
Cada agrupamento entre parnteses da forma + . . . + 2n
n +1 , com n = 2, 4, 8, . . ., isto , n = 2k para k = 1, 2, 3, . . ..
7
E cada um deles maior do que ou igual a 12 , conforme demonstrado em (a). Assim, a soma parcial at o termo
k 1 7
2 2 maior do que ou igual a 1 + 2 + k 12 .
7k
Como a sequncia das somas parciais crescente e para os valores de n = 2k+1 a soma parcial maior do que 12 ,
ento para qualquer valor real M > 0 existir um n tal que a soma parcial at o n-simo termo supera o valor M.
Isso mostra que a srie cresce sem limite.

4
AV3 - MA 12 - 2012

Questo 5.

Cada bolinha nas figuras abaixo deve ser colorida com uma das cores azul, branca, vermelha ou preta, de
modo que as bolinhas ligadas por um segmento tenham cores diferentes.

(a) De quantos modos se pode colorir a figura da esquerda?


(b) De quantos modos se pode colorir a figura da direita?

UMA SOLUO

(a) Na figura da esquerda, h 4 cores possveis para a bolinha na posio mais alta. Uma vez fixada essa cor, a
bolinha na altura intermediria esquerda tem 3 possibilidades, e, fixada esta, a da direita tem duas possibilida-
des. Para a bolinha inferior sobram duas possibilidades, ou a cor da bolinha superior ou a cor que no entrou em
nenhuma das 3 bolinhas mais acima. Ento so 4 3 2 2 = 48 maneiras.

(b) A figura da direita poderia ser desenhada como a da esquerda, mas sem a ligao entre as duas bolinhas que
esto na posio intermediria. Se essas duas bolinhas tm a mesma cor, ento so 4 cores para a bolinha superior,
3 para as intermedirias iguais, e 3 para a inferior (a inferior s no pode ser igual s intermedirias iguais). Isso
d 4 3 3 = 36. Se as bolinhas intermedirias tm cores diferentes a camos no caso anterior, onde encontramos 48
maneiras. Ento o nmero total de maneiras 36 + 48 = 84.

5
AV3 - MA 12 - 2012

Questo 1.
1
Uma moeda, com probabilidade 3 de dar cara, lanada 40 vezes.
(a) Explique por que a probabilidade pk de se obter k caras nos 40 lanamentos dada por
 k  40k
1 2
pk = C40,k ,
3 3
para k = 0, 1, 2, . . . , 40.
(b) Calcule para que valores de k tem-se pk+1 > pk .
(c) Utilize (b) para obter o valor de k para o qual a probabilidade de se obter k caras mxima.

UMA SOLUO

(a) A probabilidade de sarem k caras e 40 k coroas em 40 lanamentos, numa ordem especfica, a probabilidade
de sair cara elevada potncia k vezes a probabilidade de sair coroa elevada potncia 40 k. Neste caso,
 k  40k
1 2
.
3 3
Mas o nmero de maneiras (ou ordens) que podem sair as k caras o nmero de maneiras de se escolher k elementos
entre 40, ou seja, C40,k . Por isso a frmula do enunciado.

(b) Lembramos que


40!
C40,k = .
k!(40 k )!
Ento pk+1 > pk se, e somente se,
 k+1  40k1  k  40k
40! 1 2 40! 1 2
> .
(k + 1)!(40 k 1)! 3 3 k!(40 k)! 3 3
Ou seja, se, e somente se,
1 1 1 2
> ,
k+1 3 40 k 3
38
cancelando os fatores comuns nos dois lados. Portanto pk+1 > pk se, e somente se, 40 k > 2k + 2, isto , k < 3 .
Como k inteiro, isto equivalente a k 12.

(c) De (b) vale


p0 < p1 < p2 < . . . < p12 < p13 .

Vale, tambm,
p13 p14 . . . p40 ,

embora valham, de fato, as desigualdades estritas, se for aplicado raciocnio anlogo quele feito em (b). O valor
mximo, ocorre, portanto, em k = 13.

1
AV3 - MA 12 - 2012

Questo 2.

A soma dos n primeiros termos de uma progresso aritmtica dada por Sn = 2n2 15n.
(a) Determine o dcimo termo da progresso.
(b) Encontre o primeiro termo positivo da progresso.

UMA SOLUO

(a) O dcimo termo S10 S9 , isto ,

(2 102 15 10) (2 92 15 9) = 23 .

(b) Queremos saber para quais valores de n o n-simo termo, isto , a expresso Sn Sn1 , maior do que zero.
Temos
(2n2 15n) (2(n 1)2 15(n 1)) = 4n 17 ,

logo o primeiro termo positivo ocorre para o primeiro n tal que 4n 17 > 0, isto , para n = 5.

2
AV3 - MA 12 - 2012

Questo 3.

Um comerciante, para quem o dinheiro vale 3% ao ms, oferece determinado produto por 3 prestaes mensais
iguais a R$ 100,00, a primeira paga um ms aps a compra.
(a) Que valor o comerciante deve cobrar por este produto, no caso de pagamento vista?
(b) Se um consumidor desejar pagar o produto em trs prestaes mensais iguais, sendo a primeira paga no
ato da compra, qual deve ser o valor das parcelas?

Utilize, se desejar, os seguintes valores para as potncias de 1,03: 1, 032 = 1, 0609, 1, 033 = 1, 0927, 1, 031 = 0, 9709,
1, 032 = 0, 9426, 1, 033 = 0, 9151.

UMA SOLUO

(a) Trazendo os valores das prestaes a valor presente e somando, obtemos o valor para o pagamento vista:

100 100 100 100


+ 2
+ = (1 + 1, 031 + 1, 032 )
1, 03 1, 03 1, 033 1, 03
100 1 1, 033
=
1, 03 1 1, 031
1 0, 9151 8, 49
' 100 = 100 = 283 .
0, 03 3
O resultado pode ser um pouco diferente dependendo de como so usados os arredondamentos.

(b) No preciso saber o valor vista. Basta trazer em 1 ms cada uma das prestaes de 100 reais. Ou seja, cada
uma deve ser de 100 1, 031 , que aproximadamente igual a 97,09 reais.

3
AV3 - MA 12 - 2012

Questo 4.

(a) Mostre, por induo finita, que


1 1 1 7
+ +...+
n+1 n+2 2n 12
para todo nmero natural n 2.
1
(b) Use este fato para explicar por que a soma 1 + 2 + 13 + 14 + . . . cresce sem limite.

UMA SOLUO

1
+ 14 , que igual a 12
(a) Primeiro vejamos que a desigualdade vale para n = 2. O lado esquerdo, neste caso, 7
.3
Agora suponhamos que a desigualdade valha para um certo n 2. Iremos mostrar que a correspondente desi-
gualdade tambm vale para n + 1. Isto , supondo que vale

1 1 1 7
+ +...+
n+1 n+2 2n 12
(hiptese de induo), mostraremos que vale

1 1 1 7
+ +...+ .
( n + 1) + 1 ( n + 1) + 2 2( n + 1) 12

Ora, mas o lado esquerdo pode ser escrito como


 
1 1 1 1 1 1
+ +...+ + + .
n+1 n+2 2n 2n + 1 2n + 2 n + 1
7
Pela hiptese de induo, a soma entre parnteses maior do que ou igual a 12 . Ento basta mostrar que

1 1 1
+ 0.
2n + 1 2n + 2 n + 1
Mas isso verdade porque
1 1 2 1
+ > = .
2n + 1 2n + 2 2n + 2 n+1

(b) A soma mencionada pode ser agrupada assim:


     
1 1 1 1 1 1 1
1+ + + + +...+ + +...+ +...
2 3 4 5 8 9 16
1 1
Cada agrupamento entre parnteses da forma + . . . + 2n
n +1 , com n = 2, 4, 8, . . ., isto , n = 2k para k = 1, 2, 3, . . ..
7
E cada um deles maior do que ou igual a 12 , conforme demonstrado em (a). Assim, a soma parcial at o termo
k 1 7
2 2 maior do que ou igual a 1 + 2 + k 12 .
7k
Como a sequncia das somas parciais crescente e para os valores de n = 2k+1 a soma parcial maior do que 12 ,
ento para qualquer valor real M > 0 existir um n tal que a soma parcial at o n-simo termo supera o valor M.
Isso mostra que a srie cresce sem limite.

4
AV3 - MA 12 - 2012

Questo 5.

Cada bolinha nas figuras abaixo deve ser colorida com uma das cores azul, branca, vermelha ou preta, de
modo que as bolinhas ligadas por um segmento tenham cores diferentes.

(a) De quantos modos se pode colorir a figura da esquerda?


(b) De quantos modos se pode colorir a figura da direita?

UMA SOLUO

(a) Na figura da esquerda, h 4 cores possveis para a bolinha na posio mais alta. Uma vez fixada essa cor, a
bolinha na altura intermediria esquerda tem 3 possibilidades, e, fixada esta, a da direita tem duas possibilida-
des. Para a bolinha inferior sobram duas possibilidades, ou a cor da bolinha superior ou a cor que no entrou em
nenhuma das 3 bolinhas mais acima. Ento so 4 3 2 2 = 48 maneiras.

(b) A figura da direita poderia ser desenhada como a da esquerda, mas sem a ligao entre as duas bolinhas que
esto na posio intermediria. Se essas duas bolinhas tm a mesma cor, ento so 4 cores para a bolinha superior,
3 para as intermedirias iguais, e 3 para a inferior (a inferior s no pode ser igual s intermedirias iguais). Isso
d 4 3 3 = 36. Se as bolinhas intermedirias tm cores diferentes a camos no caso anterior, onde encontramos 48
maneiras. Ento o nmero total de maneiras 36 + 48 = 84.

5
Sociedade Brasileira de Matemtica
Mestrado Profissional em Matemtica em Rede Nacional

MA12 Matemtica Discreta


Avaliao - AV 3 - MA 12
13 de julho de 2013

1. (2,0) Seja (an ) uma progresso aritmtica e seja (bn ) a sequncia definida por

bn = an + an+1 , n 1

a) (0,5) Mostre que (bn ) tambm uma progresso aritmtica.


b) (1,5) Suponha que a soma dos n primeiros termos da sequncia (an ) seja igual a 2n2 + 5n,
para todo natural n 1. Obtenha uma expresso para a soma dos n primeiros termos de (bn ).

2. (2,0) Seja (an ) uma sequncia tal que a1 = 2, a2 = 5 e an+2 = an + an+1 , para todo
natural n 1. Mostre, por induo finita, que

a2 + a4 + a6 + ... + a2n = a2n+1 2,


para todo n 1.

3. (2,0) Considere a bandeira da figura abaixo, formada por seis regies. Para colori-la, h
lpis de cor de quatro cores diferentes.
a) (0,5) De quantos modos ela pode ser colorida de modo que regies adjacentes tenham
cores diferentes?
b) (1,5) Resolva o item a), supondo agora que todas as quatro cores sejam utilizadas para
pintar cada bandeira.

4. (2,0) Joo precisa comprar uma pea para seu carro, com o qual ele espera ficar por mais 3
anos. Ele pode comprar, por R$ 1400, 00, uma pea original, que vai durar todo este perodo, ou,
por R$ 500, 00, uma pea alternativa, que dura apenas 1 ano. Suponha que o valor do dinheiro
seja de 10% ao ano.
a) (1,0) Mostre que, apesar do desembolso total com a pea alternativa ser maior, ela a
mais vantajosa para Joo.
b) (1,0) Joo acha que pode conseguir um desconto na pea original. A partir de que valor
vale a pena ele optar por ela?

5. (2,0) As faces de um dado honesto so numeradas de 1 a 3 (cada nmero aparece duas


vezes). Seja pn a probabilidade de que a soma das faces obtidas em n lanamentos seja par.
2 1
a) (1,0) Explique porque a sequncia pn satisfaz a recorrncia pn+1 = 3
3
pn . Qual o
valor de p1 ?
b) (1,0) Resolva a equao de recorrncia em a) para obter uma expresso para pn .
[Sugesto: determine uma constante k tal que pn = k seja uma soluo da recorrncia e faa a
substituio pn = yn + k, para obter uma recorrncia homognea ou voc tambm pode usar o mtodo
geral visto no curso para resolver recorrncias lineares no homogneas.]
Sociedade Brasileira de Matemtica
Mestrado Profissional em Matemtica em Rede Nacional

MA12 Matemtica Discreta


Avaliao - GABARITO AV 3 - MA 12
13 de julho de 2013

1. (2,0) Seja (an ) uma progresso aritmtica e seja (bn ) a sequncia definida por

bn = an + an+1 , n 1

a) (0,5) Mostre que (bn ) tambm uma progresso aritmtica.


b) (1,5) Suponha que a soma dos n primeiros termos da sequncia (an ) seja igual a 2n2 + 5n,
para todo natural n 1. Obtenha uma expresso para a soma dos n primeiros termos de (bn ).

Uma soluo:
a) bn+1 bn = (an+1 + an+2 ) (an + an+1 ) = an+2 an = 2r, sendo que r a razo de
(an ). Logo, (bn ) uma P.A. de razo 2r.
b) Primeira soluo:
b1 + b2 + ... + bn = (a1 + a2 ) + (a2 + a3 ) + ... + (an + an+1 ) =
= (a1 + a2 + ... + an ) + (a2 + a3 + ... + an+1 ) = Sn + Sn+1 S1 =
= 2n2 + 5n + 2(n + 1)2 + 5(n + 1) 7 = 4n2 + 14n
Segunda soluo:
Podemos determinar primeiramente o primeiro termo e a razo de (an ).
A soma dos n primeiros termos de an

n n n2 r
Sn = (a1 + an ). = (a1 + a1 + (n 1)r). = r. + (a1 ).n = 2n2 + 5n
2 2 2 2
r r
Logo, 2 = 2 e (a1 2 ) = 5. Da, r = 4 e a1 = 7. Portanto, (bn ) uma P.A. cujo primeiro
termo b1 = a1 + a2 = 7 + 11 = 18 e cuja razo 2 4 = 8.
(alternativamente, podemos observar que S1 = 7 e S2 = 18; da, a1 = 7 e a2 = 11 e, portanto,
r = 4).
Portanto, a soma dos seus n primeiros termos de (bn )

n n
(b1 + bn ). = (18 + 18 + (n 1).8). = 4n2 + 14n
2 2
.

2. (2,0) Seja (an ) uma sequncia tal que a1 = 2, a2 = 5 e an+2 = an + an+1 , para todo
natural n 1. Mostre, por induo finita, que

a2 + a4 + a6 + ... + a2n = a2n+1 2,


para todo n 1.

Uma soluo:
Seja p(n) a afirmao: a2 + a4 + a6 + ... + a2n = a2n+1 2.
i) Para n = 1, temos a2 = 5 e a3 = 2 + 5 = 7. Logo, de fato temos a2 = a3 2, o que
verifica a validade de p(1 ).
ii) Suponhamos que p(n) seja vlida, ou seja, que a2 + a4 + a6 + ... + a2n = a2n+1 2. Da:
a2 + a4 + a6 + ... + a2n + a2n+2 = a2n+1 + a2n+2 2 = a2n+3 2, o que mostra que p(n + 1 )
vlida.
Logo, pelo princpio da induo finita, p(n) vlida para todo n natural.

3. (2,0) Considere a bandeira da figura abaixo, formada por seis regies. Para colori-la, h
lpis de cor de quatro cores diferentes.

a) ((0,5) De quantos modos ela pode ser colorida de modo que regies adjacentes tenham
cores diferentes?
b) (1,5) Resolva o item a), supondo agora que todas as quatro cores sejam utilizadas para
pintar cada bandeira.

Uma soluo:
B
A
C
D
F
E

a) Colorindo as regies na ordem indicada, o nmero de possibilidades 432332 =


432
b) Nas condies exigidas, no possvel pintar a bandeira com apenas uma cor ou com
duas cores apenas, mas possvel pint-la com trs cores (sem que regies adjacentes tenham a
mesma cor).
Colorindo as regies na ordem indicada com apenas 3 cores, o nmero de possibilidades
321221 = 24. Mas as 3 cores a serem usadas podem ser escolhidas de 4 modos. Logo,
o nmero de modos de colorir a bandeira usando todas as quatro cores 432 4 24 = 336.

4. (2,0) Joo precisa comprar uma pea para seu carro, com o qual ele espera ficar por mais 3
anos. Ele pode comprar, por R$ 1400, 00, uma pea original, que vai durar todo este perodo, ou,
por R$ 500, 00, uma pea alternativa, que dura apenas 1 ano. Suponha que o valor do dinheiro
seja de 10% ao ano.
a) (1,0) Mostre que, apesar do desembolso total com a pea alternativa ser maior, ela a
mais vantajosa para Joo.
b) (1,0) Joo acha que pode conseguir um desconto na pea original. A partir de que valor
vale a pena ele optar por ela?

Uma soluo:

a) Devemos comparar os seguintes esquemas de pagamento:


R$ 1400

0 1 2 3 anos

R$ 500 R$ 500 R$ 500

1 2 3 anos
0

Levando ambos para a data 2:


Pea original: 1400 (1, 1)2 = 1400 1, 21 = 1694
Pea alternativa: 500 (1, 1)2 + 500 1, 1 + 500 = 605 + 550 + 500 = 1655.
Logo, o custo com a pea alternativa inferior.
b) Para que os esquemas sejam equivalentes, o preo p da pea original deve ser tal que
p 1, 21 = 1655, ou seja, p = 1655
1,21
1367, 77 reais.

5. (2,0) As faces de um dado honesto so numeradas de 1 a 3 (cada nmero aparece duas


vezes). Seja pn a probabilidade de que a soma das faces obtidas em n lanamentos seja par.
2 1
a) (1,0) Explique porque a sequncia pn satisfaz a recorrncia pn+1 = 3
3
pn . Qual o
valor de p1 ?
b) (1,0) Resolva a equao de recorrncia em a) para obter uma expresso para pn .
[Sugesto: determine uma constante k tal que pn = k seja uma soluo da recorrncia e faa a
substituio pn = yn + k, para obter uma recorrncia homognea ou voc tambm pode usar o mtodo
geral visto no curso para resolver recorrncias lineares no homogneas.]

Uma soluo:

a) pn+1 = P (soma par em n + 1 lanamentos) =


= P (soma par em n lanamentos).P (sai par no lanamento n + 1) +
P ( soma mpar em n lanamentos).P ( sai mpar no lanamento n + 1)
Logo
1 2 2 1
pn+1 = pn + (1 pn ) = pn
3 3 3 3
O valor de p1 1/3.

2
b) Seguindo a sugesto: k = 3 13 k, o que resulta em k = 12 . Fazendo a substituio pn = yn + 12 ,
obtemos

1 2 1 1
yn+1 + = (yn + ),
2 3 3 2
ou seja yn+1 = ( 31 )yn .
A soluo geral desta recorrncia homognea yn = C( 13 )n. Logo, pn = 21 + C( 13 )n . Vamos
determinar C. Substituindo n = 1, obtemos p1 = 31 = 12 + C( 13 ), que fornece C = 21 . Portanto, a
soluo da recorrncia

1 1 1
pn = + ( )n
2 2 3
.
Vejamos agora como solucionar a mesma questo usando o mtodo geral para resolver recorrncias
no homogneas de primeira ordem.
Uma soluo da equao homognea pn+1 = ( 13 )pn an = ( 13 )n . Fazendo a substituio
pn = an yn , temos

1 2 1 1
( )n+1 yn+1 = ( )n yn ,
3 3 3 3
ou seja, yn+1 yn = 32 (3)n+1 .
Escrevendo esta igualdade para n variando de 1 a n 1 e somando:

2   (P G) 3 
yn y1 = ( ) (3)2 + ... + (3)n ) = 1 (3)n1

3 2
.
3
Da yn = y1 + 2 [1 (3)n1 ]. Mas p1 = 31 . Logo 13 y1 = 13 , ou seja y1 = 1. Logo,

3 3 1 1
yn = 1 + (3)n1 = + (3)n
2 2 2 2
Da, finalmente:
1 1 1 1
pn = ( )n yn = ( )n +
3 2 3 2

Você também pode gostar